polpoa_sg

787
study guide for principles of accounting Helen Brubeck San Jose State University Florence McGovern Bergen Community College Upper Saddle River, New Jersey 07458

Upload: maria-vega

Post on 06-Apr-2015

5.364 views

Category:

Documents


3 download

TRANSCRIPT

Page 1: polpoa_sg

studyguide forprinciples of

accounting

Helen BrubeckSan Jose State University

Florence McGovernBergen Community College

Upper Saddle River, New Jersey 07458

1eSG_MF_0131792075.QXD 11/4/06 12:15 PM Page i

Page 2: polpoa_sg

Library of Congress Cataloging-in-Publication Data

Pollard, Meg.Principles of accounting / Meg Pollard, Sherry K. Mills, Walter T. Harrison, Jr.

p. cm.Includes index.ISBN 0-13-230479-1 (pbk. : alk. paper)1.Accounting. 2. Managerial accounting. I. Mills, Sherry K. II. Harrison, Walter T. III. Title.

HF5636.P65 2007657—dc22

2006033723

Executive Editor: Jodi McPhersonVP/Editorial Director: Jeff ShelstadDevelopmental Editors: Claire Hunter, Ralph MooreExecutive Marketing Manager: Sharon KochMarketing Assistant: Patrick BarberaAssociate Director, Production Editorial: Judy LealeProduction Editor: Michael ReynoldsPermissions Supervisor: Charles MorrisManufacturing Manager: Arnold VilaCreative Director: Maria LangeCover Design: Solid State GraphicsDirector, Image Resource Center: Melinda PatelliManager, Rights and Permissions: Zina ArabiaManager, Visual Research: Beth BrenzelManager, Cover Visual Research & Permissions: Karen SanatarImage Permission Coordinator: Nancy SeisePhoto Researcher: Vanessa MooreManager, Print Production: Christy MahonComposition/Full-Service Project Management: BookMasters, IncPrinter/Binder: RR DonnelleyTypeface: New Century Schoolbook 10/12.5

Credits and acknowledgments borrowed from other sources and reproduced, with permission, in thistextbook appear on page PC-1.

Copyright © 2007 by Pearson Education, Inc., Upper Saddle River, New Jersey, 07458.Pearson Prentice Hall. All rights reserved. Printed in the United States of America. This publicationis protected by Copyright and permission should be obtained from the publisher prior to anyprohibited reproduction, storage in a retrieval system, or transmission in any form or by any means,electronic, mechanical, photocopying, recording, or likewise. For information regardingpermission(s), write to: Rights and Permissions Department.

Pearson Prentice Hall™ is a trademark of Pearson Education, Inc.Pearson® is a registered trademark of Pearson plcPrentice Hall® is a registered trademark of Pearson Education, Inc.

Pearson Education LTD. Pearson Education Australia PTY, LimitedPearson Education Singapore, Pte. Ltd Pearson Education North Asia LtdPearson Education, Canada, Ltd Pearson Educación de Mexico, S.A. de C.V.Pearson Education–Japan Pearson Education Malaysia, Pte. Ltd

10 9 8 7 6 5 4 3 2 1ISBN 0-13-179207-5

1eSG_MF_0131792075.QXD 11/4/06 12:15 PM Page ii

Page 3: polpoa_sg

Brief Contents

CHAPTER 1 Accounting and the BusinessEnvironment 2

CHAPTER 2 Recording Business Transactions 56

CHAPTER 3 The Adjusting Process 124

CHAPTER 4 Completing the Accounting Cycle 178

CHAPTER 5 Accounting for a Retail Business 228

CHAPTER 6 Internal Control and Cash 292

CHAPTER 7 Receivables 348

CHAPTER 8 Inventory 404

CHAPTER 9 Long-Term Assets: Plant Assetsand Intangibles 468

CHAPTER 10 Current Liabilities and Payroll 520

CHAPTER 11 Corporations and Stockholders’Equity 568

CHAPTER 12 Long-Term Liabilities 624

CHAPTER 13 The Statement of Cash Flows 688

CHAPTER 14 Financial Statement Analysis 746

CHAPTER 15 Introduction to ManagementAccounting 800

CHAPTER 16 Job Order Costing 846

CHAPTER 17 Process Costing 896

CHAPTER 18 Cost-Volume-Profit Analysis 956

CHAPTER 19 The Master Budget andResponsibility Accounting 1006

CHAPTER 20 Flexible Budgets and StandardCosts 1066

CHAPTER 21 Special Decisions and CapitalBudgeting 1116

Appendix A: Target Annual Report A-1

Appendix B: Accounting Information Systems andSpecial Journals B-1

Appendix C: Partnerships C-1

Appendix D: Present Value and Future ValueTables D-1

Appendix E: Check Figures E-1

Photo Credits PC-1

Glindex G-1

Company Index I-1

1eSG_MF_0131792075.QXD 11/4/06 12:15 PM Page iii

Page 4: polpoa_sg

1eSG_MF_0131792075.QXD 11/4/06 12:15 PM Page iv

Page 5: polpoa_sg

Contents

1

2

Accounting and the Business Environment 2

Business Organizations 4Types of Business Organizations 4Forms of Business Organizations 4

Accounting and Accountability 5Organization Accountability 6Financial Accounting and Management Accounting 6

Accounting Concepts and Principles 7The Entity Concept 8The Reliability (Objectivity) Principle 8The Cost Principle 8The Going-Concern Concept 9

The Accounting Equation 9Assets, Liabilities, and Owner’s Equity 9Components of Owner’s Equity 10Accounting for Business Transactions 11Evaluating Business Transactions 16

Financial Statements 16The Income Statement 16The Statement of Owner’s Equity 19The Balance Sheet 19Statement of Cash Flows 20Relationships Among the Financial Statements 20

Ethical Decision Making 21Ethics in Accounting and Business 21Standards of Professional Conduct 23! Chapter 1 Demo Doc:Transaction Analysis Using the Accounting

Equation/Financial Statement Preparation 24" Accounting in Action 34

Review and Assignment Material 35

Recording Business Transactions 56

The Role of Accounts in Summarizing Business Transactions 58Transactions 58Accounts 58Assets 58Liabilities 59Owner’s Equity 59Chart of Accounts 60

Double-Entry Accounting 60Rules of Debits and Credits 61T-Account 61Normal Balance 3

Recording and Summarizing Business Transactions 63The Transaction Analysis 65

1eSG_MF_0131792075.QXD 11/4/06 12:15 PM Page v

Page 6: polpoa_sg

Applying Transaction Analysis 66Balancing the Accounts 76Accounts After Posting to the Ledger 77Details of Journals and Ledgers 77Posting 79Four-Column Account 79

Trial Balance 80Trial Balance Errors 80Using the Trial Balance to Prepare Financial Statements 83! Chapter 2 Demo Doc: Debit/Credit Transaction Analysis 84" Accounting in Action 97

Review and Assignment Material 98

vi Contents

3 The Adjusting Process 124

Measuring Business Income Using Accounting Principles 126

Types of Adjusting Entries 127

Adjusting the Accounts 129Deferrals 130Accruals 136Summary of the Adjusting Process 138

The Adjusted Trial Balance 140

Preparing the Financial Statements 141Relationships Between the Financial Statements 143! Chapter 3 Demo Doc: Preparation of Adjusting Entries, Adjusted Trial Balance,

and Financial Statements 144" Accounting in Action 156

Review and Assignment Material 157

4 Completing the Accounting Cycle 178

The Accounting Cycle 180

The Worksheet 182

Completing the Accounting Cycle 185Preparing the Financial Statements 186Recording the Adjusting Entries 186Closing the Accounts 186The Four Closing Entries 190Post-Closing Trial Balance 194

Classifying Assets and Liabilities 194Assets 195Liabilities 195A Classified Balance Sheet 195! Chapter 4 Demo Doc: Preparing Closing Entries 197" Accounting in Action 203

Review and Assignment Material 204

5 Accounting for a Retail Business 228

The Supply Chain 230The Supplier/Retailer Relationship 230Retail Inventory Systems 232The Retailer/Customer Relationship 233

1eSG_MF_0131792075.QXD 11/4/06 12:15 PM Page vi

Page 7: polpoa_sg

Accounting for the Supplier/Retailer Relationship 233Cash and Credit Purchases 234Purchase Discounts 234Purchase Returns and Allowances 236

Accounting for the Retailer/Customer Relationship 237Cash Sales 237Credit Sales 239Sales Discounts to Other Businesses—The Supplier’s Perspective 239Sales Returns and Allowances 240

Accounting for Delivery and Other Selling Expenses 242Costs to Receive Goods from Suppliers 243Costs to Deliver Goods to Customers 245Other Selling Costs 247

Preparing a Retailer’s Financial Statements 248Income Statement 248Statement of Owner’s Equity 250Balance Sheet 251

Two Key Ratios for Decision Making 251The Gross Profit Percentage 251The Rate of Inventory Turnover 251! Chapter 5 Demo Doc: Inventory Transaction Analysis (Perpetual System) 253" Accounting in Action 259

Review and Assignment Material 260

Contents vii

6 Internal Control and Cash 292

Fraud in Business 294The Fraud Triangle 294Fraud 295

Organization Accountability for Fraud 297Objectives of Internal Control 297Control Activities 297Accountability for Internal Control 299Limitations of Internal Control 300

Internal Control for Cash 300Internal Control over Cash Receipts 300Internal Controls over Cash Payments 301The Bank Account 303Preparing the Bank Reconciliation 304Online Banking 310

Petty Cash 311Setting Up the Petty Cash Fund 311Replenishing the Petty Cash Fund 312Changing the Petty Cash Fund 313

Reporting Cash on the Balance Sheet 313! Chapter 6 Demo Doc: Bank Reconciliations 315" Accounting in Action 324

Review and Assignment Material 326

7 Receivables 348

Sales and Receivables 350Types of Sales 350Types of Receivables 353

1eSG_MF_0131792075.QXD 11/4/06 12:15 PM Page vii

Page 8: polpoa_sg

Internal Control over Receivables 355Managing the Collection of Receivables 355

The Direct Write-Off Method 356

The Allowance Method 357Estimating Uncollectibles 358

Notes Receivable 363Overview 363Accounting for Notes Receivable 365Accruing Interest Revenue 367Dishonored Notes Receivable 368

More Ratios for Decision Making 369Quick Ratio 369Days’ Sales in Receivables 370! Chapter 7 Demo Doc: Bank Reconciliations 372" Accounting in Action 379

Review and Assignment Material 380

viii Contents

8 Inventory 404

Inventory 406Types of Inventory 406Inventory Shrinkage 407Internal Controls over Inventory 407

Inventory Costing Methods 409Inventory Cost Flows 409First-In, First-Out (FIFO) Method 412Last-In, First-Out (LIFO) Method 413Average Cost Method 415Journalizing Inventory Transactions 416

Comparing FIFO, LIFO, and Average Cost 417

Valuing Inventory Using Lower-of-Cost-or-Market (LCM) 418

Reporting Inventory on the Balance Sheet 419Financial Statement Presentation of Inventory 420Effects of Inventory Errors 421

Using the Gross Profit Method to Estimate Ending Inventory 423! Chapter 8 Demo Doc: Inventory Costing 425" Accounting in Action 441

Review and Assignment Material 442

Long-Term Assets: Plant Assets and Intangibles 468

Long-Term Assets 470

Measuring the Cost of Plant Assets 472Land and Land Improvements 473Buildings, Equipment, Machinery, Furniture, and Fixtures 473A Lump-Sum (Basket) Purchase of Assets 474Capital Expenditures 474

Measuring Plant Asset Depreciation 476Depreciation Methods 477Comparing Depreciation Methods 480Other Issues in Accounting for Plant Assets 481

Disposing of a Plant Asset 483Selling a Plant Asset 485

9

1eSG_MF_0131792075.QXD 11/4/06 12:15 PM Page viii

Page 9: polpoa_sg

Exchanging Plant Assets 486Retiring a Plant Asset 486

Accounting for Natural Resources 487

Accounting for Intangible Assets 488Patents 488Copyrights 489Trademarks and Trade Names 489Franchises and Licenses 489Goodwill 489

Presenting Long-Term Assets on the Balance Sheet 490! Chapter 9 Demo Doc: Depreciation and Disposal of Depreciable Assets 492" Accounting in Action 500

Review and Assignment Material 501

Contents ix

10 Current Liabilities and Payroll 520

Current Liabilities of Known Amount 522Accounts Payable 522Short-Term Notes Payable and Interest Payable 523Sales Tax Payable 524Current Portion of Long-Term Notes Payable 525Accrued Expenses 526Unearned Revenues 526

Estimated and Contingent Liabilities 527Estimated Warranty Payable 527Contingent Liabilities 528

More Ratios for Decision Making 529

Accounting for Payroll 531Gross Pay and Net Pay 531Employee Payroll Deductions 532Optional Deductions 533Employer Payroll Taxes 533Other Payroll Considerations 534

The Payroll Process 534Payroll Register 536Payroll Checks 536Employee Earnings Record 537Internal Control over Payroll 537

Payroll Entries 539Record Employee Compensation 539Record Employer Payroll Taxes and Contributions to Employee

Benefits 539Record Payments of Compensation, Taxes, and Benefits 540

Reporting Current Liabilities 541! Chapter 10 Demo Doc: Known and Estimated Current Liabilities 543" Accounting in Action 548

Review and Assignment Material 549

11 Corporations and Stockholders’ Equity 568

Forms of Business Organizations 570

Corporations: An Overview 571Organizing a Corporation 572Stockholders’ Equity 573

1eSG_MF_0131792075.QXD 11/4/06 12:15 PM Page ix

Page 10: polpoa_sg

Classes of Stock 574Stockholders’ Equity Transactions: An Overview 575

Issuing Stock 576Issuing Common Stock 576Issuing Preferred Stock 578

Accounting for Cash Dividends 579Dividend Dates 579Dividends on Preferred and Common Stock 579Dividends on Cumulative and Noncumulative Preferred Stock 581

Stock Dividends and Stock Splits 582Stock Dividends 583Stock Splits 584Comparison of Cash Dividends, Stock Dividends, and Stock Splits 584

Treasury Stock 585Purchase of Treasury Stock 585Sale of Treasury Stock 587

Reporting Stockholders’ Equity 588Stockholders’ Equity Section of the Balance Sheet 588Statement of Stockholders’ Equity 590! Chapter 11 Demo Doc: Equity Transactions 591" Accounting in Action 597

Review and Assignment Material 599

x Contents

12 Long-Term Liabilities 624

Long-Term Liabilities: Mortgages and Leases 626Mortgage Note Payable 626Lease Liabilities 628

Bonds Payable 630Types of Bonds 631Present Value 632Bond Interest Rates 633Bond Prices 634

Issuing Bonds Payable and Paying Interest 635Issuing Bonds Payable at Maturity Value 635Issuing Bonds Payable at a Discount 635Issuing Bonds Payable at a Premium 637Adjusting Entries for Interest Expense 640

Retirement of Bonds 641Redeeming Bonds at Maturity 641Early Retirement of Bonds Payable 642

Reporting Liabilities on the Balance Sheet 643! Chapter 12 Demo Doc: Bonds Payable 644" Accounting in Action 649

Review and Assignment Material 650

CHAPTER APPENDIX 12A: Time Value of Money: Future Value and Present Value 671

CHAPTER APPENDIX 12B: Effective-Interest Method 681

13 The Statement of Cash Flows 688

Basic Concepts: Statement of Cash Flows 690

Operating, Investing, and Financing Activities 691Two Formats for Operating Activities 692

1eSG_MF_0131792075.QXD 11/4/06 12:15 PM Page x

Page 11: polpoa_sg

Noncash Investing and Financing Activities 692

Preparing the Statement of Cash Flows by the Indirect Method 693Cash Flows from Operating Activities 696Cash Flows from Investing Activities 697Cash Flows from Financing Activities 698Noncash Investing and Financing Activities 701

Preparing the Statement of Cash Flows by the Direct Method 702Cash Flows from Operating Activities 704! Chapter 13 Demo Doc: Preparing the Statement of Cash Flows Using the Indirect

Method 708" Accounting in Action 718

Review and Assignment Material 719

Contents xi

14 Financial Statement Analysis 746

Purpose of Financial Statement Analysis 748

Horizontal Analysis 749Trend Percentages 751

Vertical Analysis 752How Do We Compare One Company with Another? 753

Using Ratios to Make Decisions 756Liquidity Ratios 756Profitability Ratios 758Asset Utilization Ratios 760Debt Utilization Ratios 761Analyzing Stock Investments 762

Economic Value Added 763

Analyzing Nonfinancial Data 764President’s Letter to the Stockholders 765Management Discussion and Analysis (MD&A) 765Auditor Report 765! Chapter 14 Demo Doc: Horizontal and Vertical Analysis 766" Accounting in Action 773

Review and Assignment Material 775

15 Introduction to Management Accounting 800

Management Accountability 802

Today’s Business Environment 805

Service Companies, Merchandising Companies, andManufacturing Companies 806

Merchandising Companies 807Manufacturing Companies 808

Ethical Standards 814! Chapter 15 Demo Doc: Introduction to Management Accounting 816" Accounting in Action 822

Review and Assignment Material 824

16 Job Order Costing 846

How Much Does It Cost to Make a Product? Two Approaches 848Job Order Costing for Manufacturing Products 849Job Order Costing: Accounting for Materials and Labor 850

1eSG_MF_0131792075.QXD 11/4/06 12:15 PM Page xi

Page 12: polpoa_sg

xii Contents

Job Order Costing: Allocating Manufacturing Overhead 854Allocating Manufacturing Overhead to Jobs 855

Accounting for Completion and Sale of Finished Goods andAdjusting Manufacturing Overhead 858

Accounting for the Completion and Sale of Finished Goods 858Adjusting Underallocated or Overallocated Manufacturing Overhead at the

End of the Period 859Overview of Job Order Costing in a Manufacturing Company 861Job Order Costing in a Service Company 862! Chapter 16 Demo Doc: Job Order Costing for Manufacturers 865" Accounting in Action 869

Review and Assignment Material 871

xii Contents

17 Process Costing 896

Process Costing: An Overview 898Two Basic Costing Systems: Job Order Costing and Process Costing 898How Does the Flow of Costs Differ Between Job and Process Costing? 899

Building Blocks of Process Costing 901Conversion Costs 902Equivalent Units of Production 902

Process Costing in the First Department with No BeginningInventory 903

Step 1: Summarize the Flow of Physical Units 905Step 2: Compute Output in Terms of Equivalent Units 906Step 3: Compute the Cost per Equivalent Unit 907Step 4: Assign Costs to Units Completed and to Units in Ending Work in

Process Inventory 908

Process Costing in a Second Department 910The Weighted-Average Process Costing Method 910Steps 1 and 2: Summarize the Flow of Physical Units and Compute Output

in Terms of Equivalent Units 912Step 3: Summarize Total Costs to Account For and Compute the Cost per

Equivalent Unit 913Step 4: Assign Total Costs to Units Completed and to Units in Ending Work

in Process Inventory 914How Managers Use a Production Cost Report 916! Chapter 17 Demo Doc:Weighted-Average Process Costing 918" Accounting in Action 923

Review and Assignment Material 925

CHAPTER APPENDIX 17A: The FIFO Process Costing Method 946

18 Cost-Volume-Profit Analysis 956

Cost Behavior 958Variable Costs 958Fixed Costs 959Mixed Costs 960High-Low Method to Separate Fixed Cost from Variable Cost 961Relevant Range 962

1eSG_MF_0131792075.QXD 11/4/06 12:15 PM Page xii

Page 13: polpoa_sg

Basic CVP Analysis: What Must We Sell to Break Even? 963Assumptions 963How Much Must Chan Sell to Break Even? Three Approaches 964

Using CVP to Plan Profits 967How Much Must Chan Sell to Earn a Profit? 967Graphing Cost-Volume-Profit Relations 967

Using CVP for Sensitivity Analysis 969Changing the Selling Price 970Changing Variable Costs 970Changing Fixed Costs 970Margin of Safety 971Information Technology and Sensitivity Analysis 972

Effect of Sales Mix on CVP Analysis 972! Chapter 18 Demo Doc: Using CVP to Plan Profits 975" Accounting in Action 982

Review and Assignment Material 984

CHAPTER APPENDIX 18A: Variable Costing and Absorption Costing 1001

Contents xiii

19 The Master Budget and Responsibility Accounting 1006

Why Managers Use Budgets 1008Using Budgets to Plan and Control 1008The Benefits of Budgeting 1009

Preparing the Master Budget 1011Components of the Master Budget 1011

Preparing the Operating Budget 1016The Sales Budget 1014The Inventory, Purchases, and Cost of Goods Sold Budget 1015The Operating Expenses Budget 1016The Budgeted Income Statement 1016

Preparing the Financial Budget 1017Preparing the Cash Budget 1017The Budgeted Balance Sheet 1021Getting Employees to Accept the Budget 1022

Using Information Technology for Sensitivity Analysis andRolling Up Unit Budgets 1023

Sensitivity Analysis 1024Rolling Up Individual Unit Budgets into the Companywide Budget 1024

Responsibility Accounting 1025Four Types of Responsibility Centers 1025Responsibility Accounting Performance Reports 1027! Chapter 19 Demo Doc: Master Budget 1030" Accounting in Action 1037

Review and Assignment Material 1038

CHAPTER APPENDIX 19A: Departmental Accounting 1059

20 Flexible Budgets and Standard Costs 1066

How Managers Use Flexible Budgets 1068What Is a Flexible Budget? 1068

1eSG_MF_0131792075.QXD 11/4/06 12:15 PM Page xiii

Page 14: polpoa_sg

Using the Flexible Budget 1070

Standard Costing 1072Price Standards 1072Application 1072Quantity Standards 1074Why Do Companies Use Standard Costs? 1074Variance Analysis 1075

How Pluto Uses Standard Costing: Analyzing the Flexible BudgetVariance 1076

Direct Material Variances 1076Direct Labor Variances 1079

Manufacturing Overhead Variances 1080Allocating Overhead in a Standard Cost System 1081Overhead Flexible Budget Variance 1082Production Volume Variance 1083Summary of Overhead Variances 1083

Standard Cost Accounting Systems 1083Journal Entries 1083Standard Cost Income Statement for Management 1085! Chapter 20 Demo Doc: Flexible Budgets and Income Statement Performance

Report 1088" Accounting in Action 1095

Review and Assignment Material 1097

xiv Contents

21 Special Decisions and Capital Budgeting 1116

Relevant Information 1118How Managers Make Decisions 1118What Information Is Relevant to a Special Business Decision? 1118

How to Make Short-Term Special Decisions 1119Special Sales Order Decision 1120Dropping a Business Segment (a Product, a Department, or a

Territory) 1122Product Mix: Which Product to Emphasize 1124Outsourcing (Make or Buy) Decisions 1125Sell As-Is or Process Further? 1127How Do Short-Term and Long-Term Special Decisions Differ? 1128

Using Payback and Accounting Rate of Return to Make CapitalBudgeting Decisions 1129

Payback Period 1129Accounting Rate of Return 1131

Using Discounted Cash-Flow Models to Make Capital BudgetingDecisions 1133

Net Present Value 1133Internal Rate of Return 1137

Comparing Capital Budgeting Methods 1138! Chapter 21 Demo Doc: Special Business Decisions 1139" Accounting in Action 1144

Review and Assignment Material 1146

Appendix A: Target Annual Report A-1

Appendix B: Accounting Information Systems and Special Journals B-1

1eSG_MF_0131792075.QXD 11/4/06 12:15 PM Page xiv

Page 15: polpoa_sg

Appendix C: Partnerships C-1

Appendix D: Present Value and Future Value Tables D-1

Appendix E: Check Figures E-1

Photo Credits PC-1

Glindex G-1

Company Index I-1

Contents xv

1eSG_MF_0131792075.QXD 11/4/06 12:15 PM Page xv

Page 16: polpoa_sg

The Principles of Accounting Demo Doc System: For professors whosegreatest joy is hearing students say “I get it!”

Help your students achieve “I get it!” momentswhen you’re with them AND when you’re NOT.

1eSG_Walkthrough.qxd 11/4/06 12:17 PM Page xvi

Page 17: polpoa_sg

When you’re there showing how to solve a problem in class, students

“get it.” When you’re not there, they get stuck—it’s only natural.

Our system is designed to help you deliver the best “I get it!” moments.

(Instructor’s Edition, Instructor Demo Docs)

But it’s the really tricky situations that no one else has zeroed in on—

the 2 A.M. outside-of-class moments, when you’re not there—that

present the greatest challenge.

That’s where we come in: at these critical “they have the book, but they

don’t have you” moments. Principles of Accounting’s Demo Doc System

will help in those critical times. The ability of the Horngren System to

help in those times is what makes this package different from all other

textbooks.

1eSG_Walkthrough.qxd 11/4/06 12:17 PM Page xvii

Page 18: polpoa_sg

1The Principles of Accounting Demo Doc System

Duplicate the classroom experience anytime, anywhere.When is your “I get it!” moment?

THE FIRST EDITION1. A system of instruction that duplicates the classroom experience anytime, anywhere.

2. Demo Docs: entire problems worked through step-by-step from start to finish with thekind of comments around it that YOU would say in class. Available in Flash and in print.

3. A “no clutter” layout, so critical content is clear.

4. Consistency of voice and visual exhibits across all mediums.

5. MyAccountingLab Online Homework and Assessment tool: Marry the “I get it!”moment with the Power of Practice.

6. Your “I get it!” moment on your time.

Details1. CHAPTERS 1–4 We know it’s critical that students nail the fundamentals and language

before they can move to practice. We’re spending extra time developing the accountingcycle chapters (Chs 1-4) to make sure they will help students succeed. We’re includingextra visuals, comprehensive problems, and a Demo Doc for every chapter to give students enough to go on.

2. THE ULTIMATE SYSTEM: FUELED BY DEMO DOCS–This is the System of Learning(Text + Study Guide with Demo Docs + MyAccountingLab).

NEW DEMO DOCS – Introductoryaccounting students consistently tell us,“When doing homework, I get stuck tryingto solve problems the way they weredemonstrated in class.” Instructors consistently tell us, “I have so much tocover in so little time; I can’t afford to gobackward and review homework in class.” Those challenges inspired us todevelop Demo Docs. Demo Docs arecomprehensive worked-through problems available for every chapter ofour introductory accounting text to helpstudents when they are trying to solveexercises and problems on their own.

The idea is—help students duplicate theclassroom experience outside of class.

!

Demo DocDebit/Credit Transaction AnalysisDemo Doc: To make sure you understand this material, work through the follow-ing demonstration "demo doc" with detailed comments to help you see the conceptwithin the framework of a worked through problem.

Learning Objectives 1–3

On September 1, 2008, Michael Moe opened Moe’s Mowing, a company that pro-vides mowing and landscaping services. During the month of September, thebusiness incurred the following transactions:

a. To begin operations, Michael deposited $10,000 of personal funds inthe business’s bank account. The business received the cash andgave Michael an ownership interest.

b. Purchased equipment for $3,500 on account.

c. Purchased office supplies for $800 cash.

d. Provided $2,600 of services to a customer on account.

e. Paid $500 cash toward the equipment previously purchased onaccount in transaction b.

f. Received $2,000 in cash for services provided to a new customer.

g. Paid $200 cash to repair equipment.

h. Paid $900 cash in salary expense.

i. Received $2,100 cash from customers on account.

j. Michael withdrew $1,500 cash from the business for personal use.

Requirements

1. Create blank T-accounts for the following accounts:

Cash, 111; Accounts Receivable, 121; Supplies, 131; Equipment, 141; AccountsPayable, 211; Michael Moe, Capital, 311; Michael Moe, Withdrawals, 312; ServiceRevenue, 411; Salary Expense, 511; and Repairs Expense, 521.

2. Journalize the transactions and show how they are posted in T-accounts.

3 Total all theT accounts to determine their balances at the end of the month

1eSG_Walkthrough.qxd 11/4/06 12:17 PM Page xviii

Page 19: polpoa_sg

Entire problems that mirror the end-of-chapter material are shown solved andannotated with explanations written in a conversational style, essentially imitatingwhat an instructor might say if standing over a student’s shoulder. All Demo Docswill be available online in Flash and in print so students can easily refer to themwhen they need them.

3. CONSISTENCY – The small, incremental stuff matters. Consistency in form, func-tion, and language. From medium to medium. So when students ask “Where dothe numbers come from?” they can go to our text OR go online and know whatto do. If it’s worded one way here, it’s worded the same way there. In the casewhere alternate terms could be used, we reference them in the definition at theend of the chapter for students so they “get it.” It looks one way here, it looksthe same way there.

4. CLUTTER-FREE – We’re getting rid of clutter. Less is more. Extraneous boxesand features, non-essential bells and whistles…gone. Too much excess crowdsout what really matters—the concepts, the problems, the learning objectives.Instructors asked for fewer ”features“ in favor of more exercises with bettercross referencing. So, that's what we’ve done. Based on feedback, importantitems such as ethics and business cases are now located as part of the end ofchapter materials so that the student can follow the sequence of learningaccounting uninterrupted.

Financial Statement Preparation

IncomeStatement

Statement ofOwner’s Equity

BalanceSheet

1 Cash $24,4002 Accounts receivable 5,2003 Supplies 1,2004 Equipment 9,0005 Accounts payable $ 6006 Notes payable 15,0007 Kay Torres, Capital 20,0008 Kay Torres, Withdrawals 1,0009 Service revenue 7,500

10 Building rent expense 90011 Salary expense 1,10012 Utilities expense 30013 Total $43,100 $43,100

Account Title Debit CreditBalance

KAY TORRES TRAVEL AGENCYTrial Balance June 30, 2008

Exhibit 2-9 Trial Balance for Kay Torres Travel Agency

Trial Balance ErrorsWhen recording and posting transactions, accounting errors can—and do—occurin both manual and computerized accounting systems. The trial balance helps usfind those errors that cause total debits and total credits to be unequal. To find anerror, you start with the trial balance and work back through the accountingrecords—through the ledger and through the journal—to the transaction. Errorscan occur in the following activities:

• Preparing the trial balance

• Calculating account balances

1eSG_Walkthrough.qxd 11/4/06 12:17 PM Page xix

Page 20: polpoa_sg

5. MyAccountingLab – This online homework and assessment tool represents whenthe “I get it!” moment meets the power of practice. The power of repetition, when you get it, means that learning happens. MyAccountingLab is about helping students at their teachable moment, whether that is 1 P.M. or 1 A.M., butwhenever you are not there. MyAccountingLab is packed with algorithmic problems because practice makes perfect. It’s also packed with the exact sameend-of-chapter material that you’re used to assigning for homework. It includes aDemo Doc for each of the end-of-chapter exercises and problems that studentscan refer to as they work through the question. It helps students when it’s 1A.M.,and they’re trying to solve a problem the way it was demonstrated in class.

6. Your “I get it!” moment on your time.

1eSG_Walkthrough.qxd 11/4/06 12:17 PM Page xx

Page 21: polpoa_sg

INSTRUCTOR SUPPLEMENTS

Instructor’s Edition featuring Instructor Demo Docs

The New Look Instructor’s EditionWe’ve asked a lot of instructors how we can help them successfully implement new course-deliverymethods (e.g. online) while maintaining their regular campus schedule of classes and academicresponsibilities. In response, we developed a system of instruction for those of you who are long on commitment and expertise—but short on time and assistance.

The primary goal of the Instructor’s Edition is ease of implementation, using any delivery method—traditional, self-paced, or online. That is, the Instructor’s Edition quickly answers for you, the professor,the question “What must the student do?” Likewise, the Instructor’s Edition quickly answers for thestudent, “What must I do?”, and offers time saving tips with “best of” categories for in class discus-sion and strong examples to illustrate difficult concepts to a wide variety of students. The Instructor’sEdition also offers a quick one-shot cross reference at the exact point of importance with key additional teaching resources, so everything is in one place. The Instructor’s Edition includes summaries and teaching tips, pitfalls for new students, and “best of” practices from instructors fromacross the world.

The Instructor’s Edition also includes Instructor Demo DocsIn Instructor Demo Docs, we walk the students through how to solve a problem, as if it were the firsttime they’ve seen it. There are no lengthy passages of text. Instead, bits of expository text are woveninto the steps needed to solve the problem, in the exact sequence—for you to provide at the teach-able “I get it!” moment. This is the point at which the student has a context within which he or shecan understand the concept. We provide conversational text around each of the steps so the student stays engaged in solving the problem. We provide notes to the instructor for key teachingpoints around the Demo Docs, and “best of” practice tid-bits before each Instructor Demo Doc.

The Instructor Demo Docs are written with all of your everyday classroom realities in mind—and tryingto save your time in prepping new examples each time your book changes. Additionally, algorithmicversions of these Demo Docs are provided to students in their student guide. We keep the terminologyconsistent with the text, so there are no surprises for students as they try and work through a problem the first time.

Solutions TransparenciesThese transparency masters are the Solutions Manual in an easy-to-use format for class lectures.

Instructor’s Resource Center CD or www.prenhall.comThe password protected site and resource CD includes the following:

" The Instructor’s Edition with Instructor Demo Docs " Problem Set C

!

!

1eSG_Walkthrough.qxd 11/4/06 12:17 PM Page xxi

Page 22: polpoa_sg

" Solutions Manual with Interactive Excel Solutions

The Solutions Manual contains solutions to all end-of-chapter questions, multiple-choice questions,short exercises, exercise sets, problems sets, and Internet exercises. The Solutions Manual is availablein Microsoft Excel, Microsoft Word, and in print. You can access the solutions in MS Excel and MSWord formats by visiting the Instructor’s Resource Center on the Prentice Hall catalog site atwww.prenhall.com or on the Instructor’s CD. You will need a Pearson Educator username andpassword to retrieve materials from the Web site.

Solutions to select end-of-chapter exercises and problems are available in interactive MS Excel format so that instructors can present material in dynamic, step-by-step sequences in class. Theinteractive solutions were prepared by Kathleen O’Donnell of the State University of New York,Onondaga Community College.

" Test BankThe test item file includes more than 2,000 questions:# Multiple Choice# Matching#True/False# Computational Problems# Essay

" Test Bank is formatted for use with WebCT, Blackboard, and Course Compass.

" PowerPoints (instructor and student) summarize and reinforce key text materials. They captureclassroom attention with original problems and solved step-by-step exercises. These walk-throughsare designed to help faciliate classroom discussion and demonstrate where the numbers comefrom and what they mean to the concept at hand. There are approximately 35 slides per chapter.PowerPoints are available on the Instructor’s CD and can be downloaded from www.prenhall.com.

MyAccountingLab Online Homework and Assessment ManagerThe “I get it” moment meets power of practice.The power of repetition when you “get it” meanslearning happens. MyAccountingLab is about helping students at their teachable moments, whetherit’s 1 P.M. or 1 A.M.

MyAccountingLab is an online homework and assessment tool, packed with algorithmic versions of every text problem since practice makes perfect. It’s also packed with the exact same end of chapterthat you’re used to assigning for homework. Additionally, MyAccountingLab includes:

1. A Demo Doc for each of the end-of-chapter exercises and problems that students can refer to asthey work through the question.

2. A Guided Solution to the exact problem they are working on. It helps students when they’re trying to solve a problem the way it was demonstrated in class.

3. A full e-book so the students can reference the book at the point of practice.

4. New topic specific videos that walk students through difficult concepts.

1eSG_Walkthrough.qxd 11/4/06 12:17 PM Page xxii

Page 23: polpoa_sg

Companion Web Site The book’s Web site at www.prenhall.com/pollard—contains the following:

" Self-study quizzes—interactive study guide for each chapter " MS Excel templates that students can use to complete homework assignments for each chapter

(e-working papers)" Samples of the Flash Demo Docs for students to work through the accounting cycle

Online Courses with WebCT/BlackBoard Prentice Hall offers a link to MyAccountingLab through the Bb and WebCT Course Management Systems.

Classroom Response Systems (CRS)CRS is an exciting new wireless polling technology that makes large and small classrooms evenmore interactive, because it enables instructors to pose questions to their students, record results,and display those results instantly. Students can easily answer questions using compact remote control–type transmitters. Prentice Hall has partnerships with leading classroom response-systemsproviders and can show you everything you need to know about setting up and using a CRS system.Prentice Hall will provide the classroom hardware, text-specific PowerPoint slides, software,and support.

Visit www.prenhall.com/crs to learn more.

STUDENT SUPPLEMENTS

Runners Corporation PT Lab ManualContaining numerous simulated real-world examples, the Runners Corporation practice set is availablecomplete with data files for Peachtree, QuickBooks, and PH General Ledger. Each practice set alsoincludes business stationery for manual entry work.

A-1 Photography-Manual PT Lab Manual Containing numerous simulated real-world examples, the A-1 Photography practice set is availablecomplete with data files for Peachtree, QuickBooks, and PH General Ledger. Each set includes business stationery for manual entry work.

Study Guide including DEMO DOCS and e-Working PapersIntroductory accounting students consistently tell us, “When doing homework, I get stuck trying tosolve problems the way they were demonstrated in class.” Instructors consistently tell us, “I have somuch to cover in so little time; I can’t afford to go backwards and review homework in class.” Thosechallenges inspired us to develop Demo Docs. Demo Docs are comprehensive worked-through prob-lems available for nearly every chapter of our introductory accounting text to help students whenthey are trying to solve exercises and problems on their own. The idea is to help students duplicate the

1eSG_Walkthrough.qxd 11/4/06 12:17 PM Page xxiii

Page 24: polpoa_sg

classroom experience outside of class. Entire problems that mirror end-of-chapter material are shownsolved and annotated with explanations written in a conversational style, essentially imitating whatan instructor might say if standing over a student's shoulder. All Demo Docs will be available in theStudy Guide—in print and on CD in Flash, so students can easily refer to them when they need them.The Study Guide also includes a summary overview of key topics and multiple-choice and short-answerquestions for students to test their knowledge. Free electronic working papers are included on theaccompanying CD.

MyAccountingLab Online Homework and Assessment Manager The “I get it!” moment meets power of practice. The power of repetition when you get it; means thatlearning happens. MyAccountingLab is about helping students at their teachable moment, whetherthat is 1 P.M. or 1 A.M.MyAccountingLab is an online homework and assessment tool, packed with algorithmic versions ofevery text problem because practice makes perfect. It’s also packed with the exact same end-of-chapterthat you’re used to assigning for homework. Additionally, MyAccountingLab includes:

1. A Demo Doc for each of the end-of-chapter exercises and problems that students can refer to as they work through the question.

2. A Guided Solution to the exact problem they are working on. It helps students when they’re trying to solve a problem the way it was demonstrated in class.

3. A full e-book so the students can reference the book at the point of practice.

4. New topic specific videos that walk students through difficult concepts.

PowerPointsFor student use as a study aide or note-taking guide, these PowerPoint slides may be downloaded at the Companion Web site at www.prenhall.com/pollard.

Companion Web Site – www.prenhall.com/pollardThe book’s Web site at www.prenhall.com/pollard—contains the following:

" Self-study quizzes—interactive study guide for each chapter " MS Excel templates that students can use to complete homework assignments for each chapter

(e-working papers) " Samples of the Flash Demo Docs for students to work through the accounting cycle.

Classroom Response Systems (CRS)CRS is an exciting new wireless polling technology that makes large and small classrooms even moreinteractive because it enables instructors to pose questions to their students, record results, and display those results instantly. Students can easily answer questions using compact remote- control–type transmitters. Prentice Hall has partnerships with leading classroom response-systemsproviders and can show you everything you need to know about setting up and using a CRS system.Prentice Hall will provide the classroom hardware, text-specific PowerPoint slides, software, and support.

Visit www.prenhall.com/crs to learn more.

1eSG_Walkthrough.qxd 11/4/06 12:17 PM Page xxiv

Page 25: polpoa_sg

• VangoNotes in MP3 FormatStudents can study on the go with VangoNotes, chapter reviews in downloadableMP3 format that offer brief audio segments for each chapter:

# Big Ideas: the vital ideas in each chapter# Practice Test: lets students know if they need to keep studying# Key Terms: audio “flashcards” that review key concepts and terms# Rapid Review: a quick drill session—helpful right before tests

Students can learn more at www.vangonotes.com

1eSG_Walkthrough.qxd 11/4/06 12:17 PM Page xxv

Page 26: polpoa_sg

We’d also like to thank the following reviewers:

Shi-Mu (Simon)Yang Adelphi University Thomas Stolberg Alfred State UniversityThomas Branton Alvin Community CollegeMaria Lehoczky American Intercontinental University Suzanne Bradford Angelina CollegeJudy Lewis Angelo State UniversityRoy Carson Anne Arundel Community CollegePaulette Ratliff-Miller Arkansas State University Joseph Foley Assumption CollegeJennifer Niece Assumption CollegeBill Whitley Athens State University Shelly Gardner Augustana College

Becky Jones Baylor UniversityBetsy Willis Baylor UniversityMichael Robinson Baylor University Kay Walker-Hauser Beaufort County Community

College, WashingtonJoe Aubert Bemidji State UniversityFlorence McGovern Bergen Community College Calvin Fink Bethune Cookman CollegeMichael Blue Bloomsburg UniversityScott Wallace Blue Mountain CollegeLloyd Carroll Borough Manhattan Community CollegeKen Duffe Brookdale Community CollegeChuck Heuser Brookdale Community CollegeShafi Ullah Broward Community College SouthLois Slutsky Broward Community College SouthKen Koerber Bucks County Community College

Julie Browning California Baptist UniversityRichard Savich California State University – San

BernardinoDavid Bland Cape Fear Community CollegeRobert Porter Cape Fear Community CollegeVickie Campbell Cape Fear Community CollegeCynthia Thompson Carl Sandburg College – CarthageLiz Ott Casper College

AcknowledgmentsWe’d like to thank the following contributors:

Florence McGovern Bergen Community CollegeHelen Brubeck San Jose State University

We’d like to extend a special thank-you to the following members of our advisory panel:

Laurel Berry Bryant and Stratton CollegeJerry Millier Chaparral CollegeKathy Hepner Central Pennsylvania CollegeTrisha King Colorado Technical UniversityKaren Wisniewski County College of MorrisMichael Stamos Devry University

Patty Holmes DMACC Georgia Buckles Manchester Community CollegeBill Smith New Mexico State University Jane Konditi Northwood University Barbara Pughsley South UniversityJay Siegel Union County College

Joseph Adamo Cazenovia CollegeJulie Dailey Central Virginia Community CollegeJeannie Folk College of DuPageLawrence Steiner College of MarinDennis Kovach Community College Allegheny County -

AlleghenyNorma Montague Central Carolina Community CollegeDebbie Schmidt Cerritos CollegeJanet Grange Chicago State University Bruce Leung City College of San FranciscoPamela Legner College of DuPageBruce McMurrey Community College of DenverMartin Sabo Community College of Denver Jeffrey Jones Community College of Southern NevadaTom Nohl Community College of Southern Nevada Christopher Kelly Community College of Southern

NevadaPatrick Rogan Cosumnes River CollegeKimberly Smith County College of Morris

Jerold Braun Daytona Beach Community CollegeGreg Carlton Davidson County Community CollegeIrene Bembenista Davenport UniversityThomas Szczurek Delaware County Community CollegeCharles Betts Delaware Technical and Community

College Patty Holmes Des Moines Area Community College –

AnkenyTim Murphy Diablo Valley College

Phillipe Sammour Eastern Michigan UniversitySaturnino (Nino) Gonzales El Paso Community CollegeLee Cannell El Paso Community CollegeJohn Eagan Erie Community College

Ron O’Brien Fayetteville Technical Community College Patrick McNabb Ferris State University John Stancil Florida Southern CollegeLynn Clements Florida Southern College

Suzanne Oliver Okaloosa Walton CollegeBill Smith New Mexico State University

1eSG_Walkthrough.qxd 11/4/06 12:17 PM Page xxvi

Page 27: polpoa_sg

Alice Sineath Forsyth Technical Community CollegeJames Makofske Fresno City CollegeMarc Haskell Fresno City CollegeJames Kelly Ft. Lauderdale City College

Christine Jonick Gainesville State CollegeBruce Lindsey Genesee Community CollegeConstance Hylton George Mason UniversityCody King Georgia Southwestern State UniversityLolita Keck Globe CollegeKay Carnes Gonzaga University, SpokaneCarol Pace Grayson County CollegeRebecca Floor Greenville Technical CollegeGeoffrey Heriot Greenville Technical CollegeJeffrey Patterson Grove City CollegeLanny Nelms Gwinnet Technical CollegeChris Cusatis Gwynedd Mercy College

Tim Griffin Hillsborough Community CollegeClair Helms Hinds Community CollegeMichelle Powell Holmes Community CollegeGreg Bischoff Houston Community CollegeDonald Bond Houston Community CollegeMarina Grau Houston Community CollegeCarolyn Fitzmorris Hutchinson Community College

Susan Koepke Illinois Valley Community CollegeWilliam Alexander Indian Hills Community College –

Ottumwa Dale Bolduc Intercoast CollegeThomas Carr International College of NaplesLecia Berven Iowa Lakes Community CollegeNancy Schendel Iowa Lakes Community CollegeMichelle Cannon Ivy TechVicki White Ivy TechChuck Smith Iowa Western Community College

Stephen Christian Jackson Community CollegeDeeDee Daughtry Johnston Community CollegeRichard Bedwell Jones County Junior College

Ken Mark Kansas City Kansas Community College Ken Snow Kaplan Education CentersCharles Evans Keiser CollegeBunney Schmidt Keiser College Amy Haas Kingsborough Community College

Jim Racic Lakeland Community CollegeDoug Clouse Lakeland Community CollegePatrick Haggerty Lansing Community CollegePatricia Walczak Lansing Community CollegeHumberto M. Herrera Laredo Community CollegeChristie Comunale Long Island University Ariel Markelevich Long Island UniversityRandy Kidd Longview Community CollegeKathy Heltzel Luzerne County Community College

Lori Major Luzerne County Community College

Fred Jex Macomb Community CollegeGlenn Owen Marymount CollegeBehnaz Quigley Marymount CollegePenny Hanes Mercyhurst College, ErieJohn Miller Metropolitan Community CollegeDenise Leggett Middle Tennessee State University William Huffman Missouri Southern State CollegeTed Crosby Montgomery County Community CollegeBeth Engle Montgomery County Community CollegeDavid Candelaria Mount San Jacinto CollegeLinda Bolduc Mount Wachusett Community College

Barbara Gregorio Nassau Community CollegeJames Hurat National College of Business and Technology Denver Riffe National College of Business and

TechnologyAsokan Anandarajan New Jersey Institute of TechnologyRobert Schoener New Mexico State University Stanley Carroll New York City Technical College of

CUNYAudrey Agnello Niagara County Community CollegeCatherine Chiang North Carolina Central UniversityKaren Russom North Harris CollegeDan Bayak Northampton Community CollegeElizabeth Lynn Locke Northern Virginia Community

CollegeDebra Prendergast Northwestern Business CollegeNat Briscoe Northwestern State UniversityTony Scott Norwalk Community College

Deborah Niemer Oakland Community CollegeSuzanne Oliver Okaloosa Walton Junior CollegeJohn Boyd Oklahoma City Community CollegeKathleen O’Donnell Onondaga Community College J.T. Ryan Onondaga Community College

Toni Clegg Palm Beach Atlantic CollegeDavid Forsyth Palomar CollegeJohn Graves PCDICarla Rich Pensacola Junior CollegeJudy Grotrian Peru State CollegeJudy Daulton Piedmont Technical CollegeJohn Stone Potomac State CollegeBetty Habershon Prince George’s Community College

Kathi Villani Queensborough Community College

William Black Raritan Valley Community CollegeVerne Ingram Red Rocks Community CollegePaul Juriga Richland Community CollegePatty Worsham Riverside Community CollegeMargaret Berezewski Robert Morris CollegePhil Harder Robert Morris CollegeShifei Chung Rowan University of New Jersey

1eSG_Walkthrough.qxd 11/4/06 12:17 PM Page xxvii

Page 28: polpoa_sg

Charles Fazzi Saint Vincent CollegeLynnette Yerbuy Salt Lake Community CollegeSusan Blizzard San Antonio CollegeHector Martinez San Antonio CollegeAudrey Voyles San Diego Miramar CollegeMargaret Black San Jacinto College Merrily Hoffman San Jacinto College Randall Whitmore San Jacinto College Carroll Buck San Jose State UniversityHelen Brubeck San Jose State UniversityCynthia Coleman Sandhills Community CollegeBarbara Crouteau Santa Rosa Junior College Pat Novak Southeast Community CollegeSusan Pallas Southeast Community CollegeAl Case Southern Oregon University Gloria Worthy Southwest Tennessee Community CollegeMelody Ashenfelter Southwestern Oklahoma State

University Douglas Ward Southwestern Community CollegeBrandi Shay Southwestern Community CollegeJohn May Southwestern Oklahoma State University Jeffrey Waybright Spokane Community CollegeRenee Goffinet Spokane Community CollegeSusan Anders St. Bonaventure University John Olsavsky SUNY at FredoniaPeter Van Brunt SUNY College of Technology at Delhi

David L. Davis Tallahassee Community CollegeKathy Crusto-Way Tarrant County Community CollegeSally Cook Texas Lutheran University Bea Chiang The College of New JerseyMatt Hightower Three Rivers Community College

Susan Pope University of Akron Joe Woods University of ArkansasAllen Blay University of California, RiversideBarry Mishra University of California, RiversideLaura Young University of Central Arkansas

Jane Calvert University of Central OklahomaBambi Hora University of Central OklahomaJoan Stone University of Central OklahomaKathy Terrell University of Central OklahomaHarlan Etheridge University of Louisiana Pam Meyer University of Louisiana Sandra Scheuermann University of Louisiana Tom Wilson University of Louisiana Lawrence Leaman University of Michigan Larry Huus University of Minnesota Brian Carpenter University of Scranton Ashraf Khallaf University of Southern Indiana Tony Zordan University of St. FrancisGene Elrod University of Texas, ArlingtonCheryl Prachyl University of Texas, El PasoKarl Putnam University of Texas, El PasoStephen Rockwell University of TulsaChula King University of West Florida Charles Baird University of Wisconsin – Stout

Mary Hollars Vincennes University Lisa Nash Vincennes University Elaine Dessouki Virginia Wesleyan College

Sueann Hely West Kentucky Community and Technical College

Darlene Pulliam West Texas A&M University, CanyonJudy Beebe Western Oregon UniversityMichelle Maggio Westfield State CollegeKathy Pellegrino Westfield State CollegeNora McCarthy Wharton County Junior CollegeSally Stokes Wilmington CollegeMaggie Houston Wright State University

Gerald Caton Yavapai CollegeChris Crosby York Technical CollegeHarold Gellis York College of CUNY

1eSG_Walkthrough.qxd 11/4/06 12:17 PM Page xxviii

Page 29: polpoa_sg

About the Authors

is Professor of Accounting at American River College inSacramento, California. She received her B.A. in Economics-Business from UCLAand her M.B.A. in Business Administration from California State University,Sacramento, graduating as the Outstanding Graduate Student of the Year withmemberships in Phi Kappa Phi Honor Society and Beta Gamma Sigma HonorSociety.

Professor Pollard began her career in accounting in the audit department ofthe Los Angeles office of Touche Ross & Co. and was employed as a financial ana-lyst in the corporate headquarters of the Fortune 500 firm, Lear Siegler, Inc. Herindustry experience also has included service as director of finance for a non-profit organization.

Pollard is a Certified Public Accountant with an active license in Californiaand is a member of the California Society of Certified Public Accountants,CalCPA. She is listed in both Who’s Who Among Executive and ProfessionalWomen Educators and Who’s Who Among America’s Teachers.

Professor Pollard has taught Fundamentals of College Accounting,Financial Accounting, Managerial Accounting, Intermediate Accounting, PayrollAccounting, Auditing, Computer Spreadsheet Applications for Accounting,Careers in Accounting, Concepts in Personal Finance, and Business Mathe-matics. She is co-founder and co-instructor of the Volunteer Income Tax Assis-tance (VITA) program at American River College, an Internal Revenue Serviceprogram in which student volunteers electronically prepare and file income taxreturns for low-income individuals free of charge.

is Associate Professor of Accounting at New Mexico StateUniversity. She received her M.A. and Ph.D. in Accounting from Texas TechUniversity.

Professor Mills also has taught at the University of Texas, San Antonio, andTexas Tech University. She is the recipient of numerous teaching awards includ-ing NMSU’s Westhafer Award for Teaching Excellence, the New MexicoProfessor of the Year Award, New Mexico Society of CPA’s Accounting Educatorof the Year, and the Award for Innovation in Accounting Education by theAmerican Accounting Association. As a member of the American AccountingAssociation and the American Institute of Certified Public Accountants,Professor Mills has served on numerous committees representing innovation inaccounting education.

. is Professor Emeritus of Accounting at the HankamerSchool of Business, Baylor University. He received his B.B.A. degree from BaylorUniversity, his M.S. from Oklahoma State University, and his Ph.D. fromMichigan State University.

Professor Harrison, recipient of numerous teaching awards from studentgroups as well as from university administrators, has also taught at ClevelandState Community College, Michigan State University, the University of Texas,and Stanford University.

xxix

1eSG_MABA_0131792075.qxd 11/4/06 12:19 PM Page xxix

Page 30: polpoa_sg

A member of the American Accounting Association and the AmericanInstitute of Certified Public Accountants, Professor Harrison has served asChairman of the Financial Accounting Standards Committee of the AmericanAccounting Association, on the Teaching/Curriculum Development AwardCommittee, on the Program Advisory Committee for Accounting Education andTeaching, and on the Notable Contributions to Accounting Literature Committee.

Professor Harrison has lectured in several foreign countries and publishedarticles in numerous journals, including The Accounting Review, Journal ofAccounting Research, Journal of Accountancy, Journal of Accounting and PublicPolicy, Economic Consequences of Financial Accounting Standards, AccountingHorizons, Issues in Accounting Education, and Journal of Law and Commerce.

He is co-author of Financial Accounting, Sixth Edition, 2006 (with CharlesT. Horngren), published by Prentice Hall. Professor Harrison has received schol-arships, fellowships, and research grants or awards from PriceWaterhouseCoopers, Deloitte & Touche, the Ernst & Young Foundation, and the KPMGFoundation.

xxx About the Authors

1eSG_MABA_0131792075.qxd 11/4/06 12:19 PM Page xxx

Page 31: polpoa_sg

studyguide forprinciples of

accounting

1eSG_MABA_0131792075.qxd 11/4/06 12:19 PM Page xxxi

Page 32: polpoa_sg

1eSG_MABA_0131792075.qxd 11/4/06 12:19 PM Page xxxii

Page 33: polpoa_sg

Accounting and theBusiness Environment1

1

WHAT YOU PROBABLY ALREADY KNOW

Accounting is the language of business. Although the textbookand your teacher have both made this statement, what does itmean? It means that accounting is how businesses communicate,both with each other and with investors. Accounting terms andconcepts are encountered every day in the business world. Imaginethat you decide to move to or visit a foreign country for an extendedperiod of time. If you hope to become a part of this world, you mustlearn the language. Even though you may know you will not live inthat country for the rest of your life, you will need to understandthe basic terms of the language in order to get by on a daily basis.

Likewise, even though you may not want to become anaccountant, you will need to understand the language, such asassets, revenues, cost of goods sold, payables, and so forth, in orderto effectively communicate with your boss, your co-workers, andyour clients. Suppose you go to a job interview and the interviewerasks you what you think about their depreciation policy. If youcannot answer, then you likely will not get the job. One objective ofthis textbook is to help you learn and understand these terms sothat you will be able to express yourself in “business language”when you graduate. If you think about it, this goal is not sounusual. Every field or type of job has its own language. Thinkabout the language of football, for example. Touchdown, field goal,offside, power sweep . . . even though you may not want to work ina football-related career, if you are to understand and enjoy thesport, you need to understand its language as well.

The primary way that businesses communicate with outsiders(that is, people who are not employed by or owners of the business)is through financial statements. This chapter introduces theprimary financial statements and shows you the kinds ofaccounting items (called accounts) that you are likely to find oneach one and how they flow through to these statements. It alsoexplains the importance of these statements to people who usethem.

Learning Objectives

Describe the nature and types of business organizations.

A business can be classified by what it provides to customers or by howit is organized. The types of businesses are service companies, whichprovide services to customers; merchandise companies, which sellproducts to customers; or manufacturing companies, which manufacture

1eSG_CO1_0131792075.QXD 10/19/06 12:34 PM Page 1

Page 34: polpoa_sg

2 Chapter 1 | Accounting and the Business Environment

3

4

5

6

2

goods for sale to customers. Any of these businesses can be organized asa proprietorship (the most common form of business entity), a partnership(common for attorneys, doctors, and accountants), or a corporation (whichis owned by the stockholders).

Explain the role of accounting in business organizations.

Accounting is the information system that measures business activity,processes the data into reports, and communicates the results to theusers of those reports. Business owners, managers, investors, creditors,and taxing authorities all rely on accounting to provide information theyneed for making decisions.

Define generally accepted accounting principles and describe thebasic accounting concepts.

Generally accepted accounting principles (GAAP) are the accountingguidelines that govern how accountants measure, process, andcommunicate financial information so that it is accurate and reliable.GAAP is based on an underlying set of concepts so that each accountingrule (and hence the accounting data) are logical and make sense whencompared to other accounting rules.

Use the accounting equation to analyze business transactions.

It is critical to understand each of the basic components of theaccounting equation (Assets ! Liabilities " Owner’s Equity). Abusiness transaction is an event that can be measured and affects anyof the components of the accounting equation. The accounting equationis used to record the effects of business transactions.

Prepare and explain the relationships among the financialstatements.

Business transactions are analyzed, recorded, classified, and reported inthe financial statements, which are commonly prepared on an annualbasis and include the income statement, statement of owner’s equity,balance sheet, and statement of cash flows. Financial statementscommunicate important information to help users evaluate businessperformance. Refer to Exhibit 1-8 (p. 22) for examples of the fourfinancial statements.

Describe the ethical decision-making process.

Ethics has received renewed attention since the accounting scandals ofEnron, WorldCom, and Tyco early in the twenty-first century. Althoughmost managers, owners, and auditors perform their jobs ethically andcompetently, financially devastating results happen when they fail to doso. It goes without saying that ethics and standards of professionalconduct are important principles to cultivate in any business.

1eSG_CO1_0131792075.QXD 10/19/06 12:35 PM Page 2

Page 35: polpoa_sg

Demo Doc 1 | Chapter 1 3

Demo Doc 1Basic TransactionsLearning Objectives 1–6

Rick Baldwin opened Rick’s Delivery Service on August 1, 2008. He is the soleproprietor of the business. During the month of August, Rick had the followingtransactions:

a. Rick invested $6,000 of his personal funds in the business.

b. The business paid $650 cash for supplies.

c. The business paid $1,000 cash for bicycles and purchased $2,000worth of bicycles on account.

d. The business paid $700 on the accounts payable to the bicyclestore.

e. The business performed delivery services for customers totaling$1,200. These customers paid in cash.

f. The business also performed delivery services for customers onaccount, totaling $2,400.

g. The business collected $550 of accounts receivable.

h. The business paid rent (for the month of August) of $850.

i. The business paid employees $1,800 for the month of August.

j. Rick purchased groceries for his own use, paying $100 cash fromhis personal bank account.

k. The business received a telephone bill for $175. As of August 31,2008, it had not been paid.

l. Rick withdrew $900 cash from the business for personal use.

Requirements

1. What kind of business does Rick Baldwin have? What is his relationshipwith the business?

2. Analyze the preceding transactions in terms of their effects on theaccounting equation of Rick’s Delivery Service.

3. Prepare the income statement, statement of owner’s equity, and balancesheet of the business as of August 31, 2008.

4. Was the delivery service profitable for the month of August? Given thislevel of profit or loss, do you think the withdrawal of $900 was appropriate?Is it ethical for Rick to withdraw money from the business?

1eSG_CO1_0131792075.QXD 10/19/06 12:35 PM Page 3

Page 36: polpoa_sg

4 Chapter 1 | Demo Doc 1 Solutions

Demo Doc 1 Solutions

Part 1 Part 2 Part 3 Part 4 Part 6Part 5 Demo DocComplete

Part 1 Part 2 Part 3 Part 4 Part 6Part 5 Demo DocComplete

ASSETS = LIABILITIES +OWNER’SEQUITY

TYPE OFOWNER’SEQUITY

TRANSACTION

Cash +Accounts

Receivable + Supplies + BicyclesAccountsPayable

Rick Baldwin,Capital

(a)+ $6,000 + $6,000 Owner investment

Requirement 1

What kind of business does Rick Baldwin have? What is his relationship withthe business?

Rick’s Delivery Service is a sole proprietorship because Rick is the sole owner of anunincorporated business. As the owner, Rick is considered an investor in the busi-ness. If he is paid a salary, he may also be considered an employee of the business.

Requirement 2

Analyze the preceding transactions in terms of their effects on the accountingequation of Rick’s Delivery Service.

Notice that after recording this transaction, the equation balances. In otherwords, the formula Assets = Liabilities + Owner’s Equity still holds.

b. The business paid $650 cash for supplies.

Supplies represent an asset that is increased by $650. Because they werepaid for with cash, the Cash account (an asset) is decreased by $650.

Describe the natureand types of businessorganizations.

1

Define generallyaccepted accountingprinciples and describethe basic accountingconcepts.

3

Explain the role ofaccounting in businessorganizations.

2

a. Rick invested $6,000 of his personal funds in the business.

Rick is using his own money, but he is giving it to the business. Because thebusiness is involved, it is a recordable transaction.

From the business’s perspective, this transaction increases Cash (an asset)by $6,000 and increases Rick Baldwin, Capital (owner’s equity), by $6,000.

= +

1eSG_CO1_0131792075.QXD 10/19/06 12:35 PM Page 4

Page 37: polpoa_sg

Demo Doc 1 Solutions | Chapter 1 5

ASSETS = LIABILITIES +OWNER’SEQUITY

TYPE OF OWNER’SEQUITY

TRANSACTION

Cash +Accounts

Receivable + Supplies + BicyclesAccountsPayable

Rick Baldwin,Capital

(b) – $650 + $650

c. The business paid $1,000 cash for bicycles and purchased $2,000worth of bicycles on account.

The Bicycles account (an asset) is increased, but by how much? What is thecost of the bicycles? If $1,000 is paid in cash and $2,000 is bought on account,the total cost is $1,000 + $2,000 = $3,000. Why? This total amount is whatthe business will (eventually) end up paying in order to acquire the bicycles.This accounting is consistent with the cost principle.

So Bicycles (an asset) is increased by $3,000, whereas Cash (an asset)decreases by $1,000.

The $2,000 on account relates to accounts payable (because it will have to bepaid later). Because we now owe money that is to be paid later, it increasesAccounts Payable (a liability) by $2,000.

ASSETS = LIABILITIES +OWNER’SEQUITY

TYPE OF OWNER’SEQUITY

TRANSACTION

Cash +Accounts

Receivable + Supplies + BicyclesAccountsPayable

Rick Baldwin,Capital

(c) – $1,000 + $3,000 + 2,000

ASSETS = LIABILITIES +OWNER’SEQUITY

TYPE OF OWNER’SEQUITY

TRANSACTION

Cash +Accounts

Receivable + Supplies + BicyclesAccountsPayable

Rick Baldwin,Capital

(d) – $700 – $700

Define generallyaccepted accountingprinciples and describethe basic accountingconcepts.

3

Use the accountingequation to analyzebusiness transactions.

4

d. The business paid $700 on the accounts payable to the bicycle store.

Think of accounts payable as a list of companies to which the business owesmoney. In other words, it is a list of companies to which the business will paymoney. In this particular problem, the business owes money to the companyfrom which it purchased bicycles on account (see transaction c). When thebusiness pays the money in full, it can cross this company off the list. Rightnow, the business is paying only part of the money owed to the bicycle store.This payment decreases Accounts Payable (a liability) by $700 and decreasesCash (an asset) by $700.

= +

= +

= +

1eSG_CO1_0131792075.QXD 10/19/06 12:35 PM Page 5

Page 38: polpoa_sg

6 Chapter 1 | Demo Doc 1 Solutions

ASSETS = LIABILITIES +OWNER’SEQUITY

TYPE OF OWNER’SEQUITY

TRANSACTION

Cash +Accounts

Receivable + Supplies + BicyclesAccountsPayable

Rick Baldwin,Capital

(e) + $1,200 + $1,200 Service revenue

ASSETS = LIABILITIES +OWNER’SEQUITY

TYPE OF OWNER’SEQUITY

TRANSACTION

Cash +Accounts

Receivable + Supplies + BicyclesAccountsPayable

Rick Baldwin,Capital

(f) + $2,400 + $2,400 Service revenue

e. The business performed delivery services for customers totaling$1,200. These customers paid in cash.

When the business performs services, it means that it is doing work for cus-tomers. Doing work for customers is the way that the business makes money.By performing services, the business is earning revenues.

Earning revenues means that Service Revenues is increased (whichincreases owner’s equity) by $1,200. Because the customers paid in cash,Cash (an asset) is also increased by $1,200.

f. The business also performed delivery services for customers onaccount, totaling $2,400.

Again, the delivery service performs services for customers, which meansthat it earns revenues. Earning revenues results in an increase in ServiceRevenues (owner’s equity) of $2,400.

However, this time the customers charged the services on account. The busi-ness will receive this money in the future (when the customers eventuallypay) so it is called accounts receivable. Accounts Receivable (an asset) isincreased by $2,400.

g. The business collected $550 of accounts receivable.

Think of accounts receivable as a list of customers from whom the businesswill collect money. In other words, it is a list of customers from whom thebusiness will receive money at some future time. In this particular problem,these customers received services but did not pay at that time (see transac-tion f). Later, when the business collects (receives) the cash in full from anyparticular customer, it can cross that customer off the list. This transactiondecreases Accounts Receivable (an asset) by $550.

Because the cash is received, Cash (an asset) is increased by $550.

= +

= +

1eSG_CO1_0131792075.QXD 10/19/06 12:35 PM Page 6

Page 39: polpoa_sg

Demo Doc 1 Solutions | Chapter 1 7

h. The business paid rent (for the month of August) of $850.

The rent has already been used. By the end of August, the service has beenoperating and using the space for the entire month. The benefit of the renthas already been received or used up, which makes it a rent expense. [Notethat if the rent was paid for September, it would not yet be used up andwould still be a future benefit (an asset) and not an expense. This issue willbe discussed in Chapter 3.]

So Rent Expense is increased by $850, which decreases owner’s equity. Thequestion states that the rent was paid, which means it was paid in cash.Therefore, Cash (an asset) is decreased by $850.

ASSETS = LIABILITIES +OWNER’SEQUITY

TYPE OF OWNER’SEQUITY

TRANSACTION

Cash +Accounts

Receivable + Supplies + BicyclesAccountsPayable

Rick Baldwin,Capital

(g) + $550 – $550

ASSETS = LIABILITIES +OWNER’SEQUITY

TYPE OF OWNER’SEQUITY

TRANSACTION

Cash +Accounts

Receivable + Supplies + BicyclesAccountsPayable

Rick Baldwin,Capital

(h) – $850 – $850 Rent expense

ASSETS = LIABILITIES +OWNER’SEQUITY

TYPE OF OWNER’SEQUITY

TRANSACTION

Cash +Accounts

Receivable + Supplies + BicyclesAccountsPayable

Rick Baldwin,Capital

(i) – $1,800 – $1,800 Salary expense

i. The business paid the employees $1,800 for the month of August.

Again, the transaction states that the business paid the employees, meaningthat they paid in cash. Therefore, Cash (an asset) is decreased by $1,800.

The work the employees have given to the business has already been used.By the end of August, the delivery service has had the employees workingand delivering for customers for the entire month and the benefit of the workhas already been received, making it a salary expense. So, Salary Expensewould increase by $1,800, which decreases owner’s equity.

= +

= +

= +

1eSG_CO1_0131792075.QXD 10/19/06 12:35 PM Page 7

Page 40: polpoa_sg

8 Chapter 1 | Demo Doc 1 Solutions

j. Rick purchased groceries for his own use paying $100 cash from hispersonal bank account.

These groceries were purchased with Rick’s personal money for Rick’spersonal use. Therefore, the expense does not relate to the business and isnot a recordable transaction for the delivery service.

ASSETS = LIABILITIES +OWNER’SEQUITY

TYPE OF OWNER’SEQUITY

TRANSACTION

Cash +Accounts

Receivable + Supplies + BicyclesAccountsPayable

Rick Baldwin,Capital

(k) + $175 – $175 Utilities expense

ASSETS = LIABILITIES +OWNER’SEQUITY

TYPE OF OWNER’SEQUITY

TRANSACTION

Cash +Accounts

Receivable + Supplies + BicyclesAccountsPayable

Rick Baldwin,Capital

(l) – $900 – $900 Owner withdrawal

k. The business received a telephone bill for $175. As of August 31,2008, it had not been paid.

Utilities (such as water, gas, electricity, phone, and Internet service) are gen-erally not billed until after they have been used. If these utilities are alreadyused, then they are utilities expenses. So, utility expense would increase by$175, which is a decrease to owner’s equity.

Because the bill has not yet been paid as of August 31, it is an accountpayable. This transaction increases Accounts Payable (a liability) by $175.

l. Rick withdrew $900 cash from the business for personal use.

Although Rick is taking the money, the cash is coming from the business, soit is a recordable transaction for the business. Cash (an asset) is decreasedby $900. Because Rick is the owner, this transaction also increases OwnerWithdrawals by $900, which is a decrease to owner’s equity.

ASSETS = LIABILITIES +OWNER’SEQUITY

TYPE OF OWNER’SEQUITY

TRANSACTION

Cash +Accounts

Receivable + Supplies + BicyclesAccountsPayable

Rick Baldwin,Capital

(j)Not a transaction of the business.

= +

= +

= +

1eSG_CO1_0131792075.QXD 10/19/06 12:35 PM Page 8

Page 41: polpoa_sg

Demo Doc 1 Solutions | Chapter 1 9

Following is a summary of these transactions:

ASSETS = LIABILITIES +OWNER’SEQUITY

TYPE OF OWNER’SEQUITY

TRANSACTION

Cash +Accounts

Receivable + Supplies + BicyclesAccountsPayable

Rick Baldwin,Capital

(a) + $6,000 + $6,000Owner

investment

(b) # $ 650 + $650

(c) # $1,000 + $3,000 + 2,000

(d) # $ 700 #$ 700

(e) + $1,200 + $1,200 Service revenue

(f) + $2,400 + $2,400 Service revenue

(g) + $ 550 # $ 550

(h) # $ 850 # $ 850 Rent expense

(i) # $1,800 # $1,800 Salary expense

(j) Not a transaction of the business.

(k) + $ 175 # $ 175 Utilities expense

(l) # $ 900 # $ 900Owner

withdrawal

$1,850 $1,850 $650 $3,000 $1,475 $5,875

$7,350 $7,350

Requirement 3

Prepare the income statement, statement of owner’s equity, and balance sheetof the business as of August 31, 2008.

The income statement is the first statement that should be prepared because theother financial statements rely upon the net income number calculated on theincome statement.

The income statement lists all revenues and expenses. It uses the followingformula to calculate net income:

Revenues ! Expenses " Net income

Part 1 Part 2 Part 3 Part 4 Part 6Part 5 Demo DocComplete

Define generallyaccepted accountingprinciples and describethe basic accountingconcepts.

3

Prepare financialstatements and explainthe relationshipsbetween them.

5

= +

14444444444244444444443 144444424444443

1eSG_CO1_0131792075.QXD 10/19/06 12:35 PM Page 9

Page 42: polpoa_sg

10 Chapter 1 | Demo Doc 1 Solutions

Part 1 Part 2 Part 3 Part 4 Part 6Part 5 Demo DocComplete

Revenue Service revenue $3,600* Expenses Salary expense $ 1,800 Rent expense 850 Utilities expense 175 Total expenses 2,825 Net income $ 775

RICK’S DELIVERY SERVICEIncome Statement

Month Ended August 31, 2008

*= $1,200 + $2,400

Rick Baldwin, capital, August 1, 2008 $ 0 Add: Investment by owner 6,000 Net income for month 775 6,775 Less: Withdrawals by owner (900) Rick Baldwin, capital, August 31, 2008 $ 5,875

RICK’S DELIVERY SERVICEStatement of Owner’s Equity Month Ended August 31, 2008

So, to create an income statement, we only need to list the revenue accounts andthen subtract the list of expense accounts to calculate net income.We can read these amounts from the accounting equation worksheet.

Two transactions affect service revenue: e and f. Transaction e increases ser-vice revenue by $1,200, and transaction f increases service revenue by $2,400.Total service revenue for the month is calculated as follows:

$1,200 # $2,400 " $3,600

Rent expense of $850, salary expense of $1,800, and utilities expense of $175were recorded in transactions h, i, and k, respectively.

Net income is used on the statement of owner’s equity to calculate the newbalance in the Capital account. This calculation uses the following formula:

Beginning Capital Amount# Owner Investments# Net Income (or ! Net Loss)! Owner Withdrawals" Ending Capital Amount

Again, we just have to recreate this formula on the statement:

1eSG_CO1_0131792075.QXD 10/19/06 12:35 PM Page 10

Page 43: polpoa_sg

Demo Doc 1 Solutions | Chapter 1 11

Part 1 Part 2 Part 3 Part 4 Part 6Part 5 Demo DocComplete

Part 1 Part 2 Part 3 Part 4 Part 6Part 5 Demo DocComplete

Part 1 Part 2 Part 3 Part 4 Part 6Part 5 Demo DocComplete

The ending capital amount is used on the balance sheet. The balance sheet isjust a listing of all assets, liabilities, and equity, with the accounting equation ver-ified at the bottom:

Requirement 4

Was the delivery service profitable for the month of August? Given this level ofprofit or loss, do you think the withdrawal of $900 was appropriate? Is it ethicalfor Rick to withdraw money from the business?

From the income statement, we can see that the delivery service earned $775 ofprofit during the month of August. The level of withdrawals ($900) seems highgiven that it is more than the amount of profit earned during the month. If theowner continued to withdraw at this rate, the business could end up with a nega-tive equity balance.

As the sole owner, it is Rick’s right to withdraw money from the delivery ser-vice. As long as the withdrawal is properly recorded in the accounting system, itis ethical for Rick to withdraw money from the business.

Explain the role ofethics in accountingand business.

6

Assets Liabilities Cash $ 1,850 Accounts payable $ 1,475 Accounts receivable 1,850 Owner’s Equity Supplies 650 Rick Baldwin, capital 5,875 Bicycles 3,000 Total assets $ 7,350 Total liabilities and equity $ 7,350

RICK’S DELIVERY SERVICEBalance Sheet August 31, 2008

1eSG_CO1_0131792075.QXD 10/19/06 12:35 PM Page 11

Page 44: polpoa_sg

Quick Practice QuestionsTrue/False

1. Financial accounting produces financial information and reports to be used by managers inside a business.

2. GAAP are the rules that govern financial accounting and must be followed when preparing financial statements.

3. Financing activities include customers buying goods and services.

4. Professional accounting organizations and most companies have standards of ethical behavior.

5. A corporation’s owners are called shareholders.

6. An advantage of the partnership form of business is that the life of theentity is indefinite.

7. The owner of a sole proprietorship is not personally responsible for the debts of the business.

8. The entity concept separates business transactions from personal transactions.

9. A business has a net loss when total revenues are greater than total expenses.

10. The statement of owner’s equity reports the cash coming in and going out.

12 Chapter 1 | Quick Practice Questions

1eSG_CO1_0131792075.QXD 3/5/08 2:07 PM Page 12

Page 45: polpoa_sg

Multiple Choice

1. What is the private organization that is primarily responsiblefor formulating accounting standards?a. The Internal Revenue Serviceb. The Securities and Exchange Commissionc. The American Institute of Certified Public Accountantsd. The Financial Accounting Standards Board

2. The effect on the accounting equation of providing services to acustomer for cash is which of the following?a. Increase liabilitiesb. Decrease liabilitiesc. Decrease owner’s equityd. Increase owner’s equity

3. The effect on the accounting equation of payment of utilitieseach month is which of the following?a. Increase owner’s equityb. Increase expensesc. Increase total assetsd. Decrease liabilities

4. What is the purpose of financial accounting information?a. To help managers plan and control business operationsb. To comply with the IRS rulesc. To help investors, creditors, and others make decisionsd. To provide information to employees

5. What characteristic is necessary for information to be useful?a. It is relevantb. It is reliablec. It is comparabled. All of the above

6. Sue Mason owns a bagel shop as a sole proprietorship. Sueincludes her personal home, car, and boat on the books of herbusiness. Which of the following is violated?a. Entity conceptb. Going-concern conceptc. Cost principled. Reliability principle

7. Which of the following is the accounting equation?a. Assets – Liabilities = Owner’s equityb. Assets + Liabilities = Owner’s equityc. Assets = Liabilities + Owner’s equityd. Assets + Liabilities = Net income

8. If the assets of a business are $410,000 and the liabilities total$200,000, how much is the owner’s equity?a. $150,000b. $160,000c. $210,000d. $610,000

Quick Practice Questions | Chapter 1 13

1eSG_CO1_0131792075.QXD 10/19/06 12:35 PM Page 13

Page 46: polpoa_sg

9. Which financial statement contains a listing of assets, liabilities,and owner’s equity?a. Balance sheetb. Statement of owner’s equityc. Income statementd. Statement of cash flows

10. The claim of a business owner to the assets of the business iscalled what?a. Liabilitiesb. Owner’s equityc. Revenued. Withdrawals

14 Chapter 1 | Quick Practice Questions

1eSG_CO1_0131792075.QXD 10/19/06 12:35 PM Page 14

Page 47: polpoa_sg

Quick Practice Questions | Chapter 1 15

Quick Exercises

1-1. Fill in the following statements with the correct type of busi-ness organization.

a. A _______________________ is a separate legal entity, approved bythe state.

b. A _______________________ is an entity with one owner where thebusiness and not the owner is liable for the company’s debts.

c. A _______________________ is an entity with two or more ownerswho are personally liable for the company’s debts.

1-2. Match the following terms with the best description.

a. Entity conceptb. Reliability principlec. Cost principled. Going-concern concept____________ 1. An organization or part of an organization is sepa

rate from other organizations and individuals.____________ 2. An item should be recorded at the actual amount

paid.____________ 3. An entity is expected to remain in business in the

future.____________ 4. Accounting data should be neutral, unbiased infor

mation that can be confirmed by others.

1-3. Determine the missing amounts:

a. Assets = $50,000; Liabilities = $30,000; Owner’s equity =b. Liabilities = $35,000; Owner’s equity = $75,000; Assets =c. Assets = $105,000; Owner’s equity = $50,000; Liabilities =

1-4. Write a brief explanation for the following transactions:

OWNER’S ASSETS = LIABILITIES + EQUITY

Accounts Accounts Cash + Receivable + Supplies Payable Capital

a. 20,000 20,000b. 1,000 1,000c. #2,500 #2,500d. 5,200 5,200e. 3,000 #3,000

a. ______________________________________________________________

b. ______________________________________________________________

c. ______________________________________________________________

d. ______________________________________________________________

e. ______________________________________________________________

= +

1eSG_CO1_0131792075.QXD 10/19/06 12:35 PM Page 15

Page 48: polpoa_sg

16 Chapter 1 | Quick Practice Questions

1-5. On which of the following three financial statements wouldyou expect to find the items (a) through (f)?

Income Statement (IS)Balance Sheet (BS)Statement of Cash Flows (CF)

a. ___________ Accounts payable

b. ___________ Service revenue

c. ___________ Collections from customer

d. ___________ Utilities expense

e. ___________ Office supplies

f. ___________ Payments to suppliers

1eSG_CO1_0131792075.QXD 10/19/06 12:35 PM Page 16

Page 49: polpoa_sg

Do It Yourself! Question 1 | Chapter 1 17

ASSETS = LIABILITIES +OWNER’SEQUITY

TYPE OF OWNER’SEQUITY

TRANSACTION

(a)

Define generallyaccepted accountingprinciples and describethe basic accountingconcepts.

3

b. Jennifer’s Laundromat paid $4,000 cash for washing machines andpurchased another $5,000 worth on account.

ASSETS = LIABILITIES +OWNER’SEQUITY

TYPE OF OWNER’SEQUITY

TRANSACTION

(b)

c. Jennifer purchased a washing machine for use in her home costing$2,100 on her personal account.

ASSETS = LIABILITIES +OWNER’SEQUITY

TYPE OF OWNER’SEQUITY

TRANSACTION

(c)

Do It Yourself! Question 1

Jennifer Hill opened a laundromat business on October 1, 2008. She is the soleproprietor of the business.

Requirements1. For each transaction of the business during the month of October, analyzeand describe the transaction in terms of its effect on the accounting equationof Jennifer’s Laundromat. Summarize the transactions in one table.

a. Jennifer invested $10,000 of her personal funds in the business.

d. The business paid $500 cash for supplies.

ASSETS = LIABILITIES +OWNER’SEQUITY

TYPE OF OWNER’SEQUITY

TRANSACTION

(d)

1eSG_CO1_0131792075.QXD 10/19/06 12:35 PM Page 17

Page 50: polpoa_sg

18 Chapter 1 | Do It Yourself! Question 1

ASSETS = LIABILITIES +OWNER’SEQUITY

TYPE OF OWNER’SEQUITY

TRANSACTION

(g)

ASSETS = LIABILITIES +OWNER’SEQUITY

TYPE OF OWNER’SEQUITY

TRANSACTION

(h)

ASSETS = LIABILITIES +OWNER’SEQUITY

TYPE OF OWNER’SEQUITY

TRANSACTION

(i)

i. The business received a utility bill for $750. As of October 31, 2008,it had not been paid.

ASSETS = LIABILITIES +OWNER’SEQUITY

TYPE OF OWNER’SEQUITY

TRANSACTION

(e)

ASSETS = LIABILITIES +OWNER’SEQUITY

TYPE OF OWNER’SEQUITY

TRANSACTION

(f)

h. The business paid employees $1,500 for the month of October.

f. Jennifer’s Laundromat also performed cleaning services for cus-tomers on account totaling $3,700.

g. Jennifer’s Laundromat paid rent (for the month of October) of $1,000.

e. Jennifer’s Laundromat performed cleaning services for customerstotaling $2,500. These customers paid in cash.

1eSG_CO1_0131792075.QXD 10/19/06 12:35 PM Page 18

Page 51: polpoa_sg

Do It Yourself! Question 1 | Chapter 1 19

ASSETS = LIABILITIES +OWNER’SEQUITY

TYPE OFOWNER’S EQUITY

TRANSACTION

(a)(b)(c)(d)(e)(f)(g)(h)(i)(j)(k)(l)

ASSETS = LIABILITIES +OWNER’SEQUITY

TYPE OF OWNER’SEQUITY

TRANSACTION

(k)

k. The business collected $2,250 of accounts receivable.

l. The business paid $2,400 on the accounts payable to the washingmachine store.

ASSETS = LIABILITIES +OWNER’SEQUITY

TYPE OF OWNER’SEQUITY

TRANSACTION

(l)

ASSETS = LIABILITIES +OWNER’SEQUITY

TYPE OF OWNER’SEQUITY

TRANSACTION

(j)

j. Jennifer withdrew $2,000 cash from the business for personal use.

1eSG_CO1_0131792075.QXD 10/19/06 12:35 PM Page 19

Page 52: polpoa_sg

20 Chapter 1 | Do It Yourself! Question 1

2. Prepare the income statement, statement of owner’s equity, and balancesheet of the business as of October 31, 2008.Prepare financial state-

ments and explain therelationships betweenthem.

5

JENNIFER’S LAUNDROMATIncome Statement

Month Ended October 31, 2008

JENNIFER’S LAUNDROMATStatement of Owner’s Equity Month Ended October 31, 2008

JENNIFER’S LAUNDROMATBalance Sheet October 31, 2008

1eSG_CO1_0131792075.QXD 10/19/06 12:35 PM Page 20

Page 53: polpoa_sg

Quick Practice Solutions | Chapter 1 21

Quick Practice SolutionsTrue/False

F 1. Financial accounting produces financial information and reports to be used by managers inside a business.False–Financial accounting produces information for people outsidethe company. (p. 7)

T 2. GAAP are the rules that govern financial accounting and must be followed when preparing financial statements. (p. 6)

F 3. Financing activities include customers buying goods and services.False–Operating activities include customers buying goods and services. (p. 6)

T 4. Professional accounting organizations and most companies havestandards of ethical behavior. (p. 23)

T 5. A corporation’s owners are called shareholders. (p. 5)

F 6. An advantage of the partnership form of business is that the life of the entity is indefinite.False–The life of a partnership is limited by the owners’ choices, ordeath. (p. 5)

F 7. The owner of a sole proprietorship is not personally liable for thedebts of the business.False–The owner of a sole proprietorship is personally liable for thedebts of the business. (p. 5)

T 8. The entity concept separates business transactions from personal transactions. (p. 8)

F 9. A business has a net loss when total revenues are greater than totalexpenses.False–A business has net income when total revenues are greater than total expenses. (p. 11)

F 10.The statement of owner’s equity reports the cash coming in and goingout.

False–The statement of owner’s equity shows the changes in owner’sequity during a time period. The statement of cash flows reports cashcoming in and going out. (p. 19)

1eSG_CO1_0131792075.QXD 10/19/06 12:35 PM Page 21

Page 54: polpoa_sg

22 Chapter 1 | Quick Practice Solutions

Multiple Choice

1. What is the private organization that is primarily responsiblefor formulating accounting standards? (p. 8)a. The Internal Revenue Serviceb. The Securities and Exchange Commissionc. The American Institute of Certified Public Accountantsd. The Financial Accounting Standards Board

2. The effect on the accounting equation of providing services to acustomer for cash is which of the following? (p. 10)a. Increase liabilitiesb. Decrease liabilitiesc. Decrease owner’s equityd. Increase owner’s equity

3. The effect on the accounting equation of payment of utilitieseach month is which of the following? (p. 10)a. Increase owner’s equityb. Increase expensesc. Increase total assetsd. Decrease liabilities

4. What is the purpose of financial accounting information? (p. 7)a. To help managers plan and control business operations.b. To comply with the IRS rules.c. To help investors, creditors, and others make decisions.d. To provide information to employees.

5. What characteristic is necessary for information to be useful? (p. 8)a. It is relevantb. It is reliablec. It is comparabled. All of the above

6. Sue Mason owns a bagel shop as a sole proprietorship. Sueincludes her personal home, car, and boat on the books of herbusiness. Which of the following is violated? (p. 8)a. Entity conceptb. Going concern conceptc. Cost principled. Reliability principle

7. Which of the following is the accounting equation? (p. 9)a. Assets – Liabilities = Owner’s equityb. Assets + Liabilities = Owner’s equityc. Assets = Liabilities + Owner’s equityd. Assets + Liabilities = Net income

1eSG_CO1_0131792075.QXD 10/19/06 12:35 PM Page 22

Page 55: polpoa_sg

Quick Practice Solutions | Chapter 1 23

8. If the assets of a business are $410,000 and the liabilities total$200,000, how much is the owner’s equity? (p. 9)a. $150,000b. $160,000c. $210,000d. $610,000

9. Which financial statement contains a listing of assets, liabilities,and owner’s equity? (p. 19)a. Balance sheetb. Statement of owner’s equityc. Income statementd. Statement of cash flows

10. The claim of a business owner to the assets of the business iscalled what? (p. 9)a. Liabilitiesb. Owner’s equityc. Revenued. Withdrawals

1eSG_CO1_0131792075.QXD 10/19/06 12:35 PM Page 23

Page 56: polpoa_sg

24 Chapter 1 | Quick Practice Solutions

Quick Exercises

1-1. Fill in the following statements with the correct type of busi-ness organization. (p. 4–5)

a. A corporation is a separate legal entity,approved by the state.

b. A limited liability corporation is an entity with one owner where the business and not the owner is liable for the company’sdebts.

c. A partnership is an entity with two or more owners who are personally liable for the company’s debts.

1-2. Match the following terms with the best description. (p. 8–9)

a. Entity conceptb. Reliability principlec. Cost principled. Going-concern concept

a 1. An organization or part of an organization is separate from other organizations and individuals.

c 2. An item should be recorded at the actual amount paid.

d 3. An entity is expected to remain in business in the future.

b 4. Accounting data should be neutral, unbiased information that can be confirmed by others.

1-3. Determine the missing amounts: (p. 9)

a. Assets = $50,000; Liabilities = $30,000; Owner’s equity = $20,000(50,000 – 30,000)

b. Liabilities = $35,000; Owner’s equity = $75,000; Assets = $110,000(35,000 + 75,000)

c. Assets = $105,000; Owner’s equity = $50,000; Liabilities = $55,000(105,000 – 50,000)

1-4. Write a brief explanation for the following transactions:(p. 11–16)

OWNER’S ASSETS = LIABILITIES + EQUITY

Accounts Accounts Cash + Receivable + Supplies Payable Capital

a. 20,000 20,000b. 1,000 1,000c. #2,500 #2,500d. 5,200 5,200e. 3,000 #3,000

a. The owner invested $20,000 cash in the business.

b. Purchased $1,000 of supplies on account.

= +

1eSG_CO1_0131792075.QXD 10/19/06 12:35 PM Page 24

Page 57: polpoa_sg

Quick Practice Solutions | Chapter 1 25

c. Paid $2,500 for an expense OR the owner withdrew $2,500 for per-sonal use.

d. Performed services for customer on account, $5,200.e. Received $3,000 cash from customers on account.

1-5. On which of the following three financial statements wouldyou expect to find the items (a) through (f)? (p. 16–21)

Income Statement (IS)Balance Sheet (BS)Statement of Cash Flows (CF)a. BS Accounts payableb. IS Service revenuec. CF Collections from customerd. IS Utilities expensee. BS Office suppliesf. CF Payments to suppliers

1eSG_CO1_0131792075.QXD 10/19/06 12:35 PM Page 25

Page 58: polpoa_sg

26 Chapter 1 | Do It Yourself! Question 1 Solutions

Do It Yourself! Question 1 Solutions

Requirement 1For each transaction of the business during the month of October, analyze thetransaction in terms of its effect on the accounting equation of Jennifer’sLaundromat.

a. Jennifer invested $10,000 of her personal funds in the business.

Cash (an asset) is increased by $10,000 and Jennifer Hill, Capital (owner’sequity) is increased by $10,000.

ASSETS = LIABILITIES +OWNER’SEQUITY

TYPE OF OWNER’SEQUITY

TRANSACTION

Cash +Accounts

Receivable + Supplies +Washing

MachinesAccountsPayable

Jennifer Hill,Capital

(b) #$4,000 + $9,000 + 5,000

ASSETS = LIABILITIES +OWNER’SEQUITY

TYPE OF OWNER’SEQUITY

TRANSACTION

Cash +Accounts

Receivable + Supplies +Washing

MachinesAccountsPayable

Jennifer Hill,Capital

(a)+ $10,000 + $10,000 Owner investment

b. Jennifer’s Laundromat paid $4,000 cash for washing machines andpurchased another $5,000 worth on account.

Washing Machines (an asset) is increased by $9,000, while Cash (an asset)is decreased by $4,000. Accounts Payable (a liability) is increased by $5,000.

c. Jennifer purchased a washing machine for use in her home costing$2,100 on her personal account.

This expense does not relate to the business and is not a recordabletransaction for the business.

ASSETS = LIABILITIES +OWNER’SEQUITY

TYPE OF OWNER’SEQUITY

TRANSACTION

Cash +Accounts

Receivable + Supplies +Washing

MachinesAccountsPayable

Jennifer Hill,Capital

(c) Not a transaction of the business.

= +

= +

= +

1eSG_CO1_0131792075.QXD 10/19/06 12:35 PM Page 26

Page 59: polpoa_sg

Do It Yourself! Question 1 Solutions | Chapter 1 27

d. The business paid $500 cash for supplies.

Supplies (an asset) is increased by $500 and Cash (an asset) is decreasedby $500.

ASSETS = LIABILITIES +OWNER’SEQUITY

TYPE OF OWNER’SEQUITY

TRANSACTION

Cash +Accounts

Receivable + Supplies +Washing

MachinesAccountsPayable

Jennifer Hill,Capital

(f) + $3,700 + $3,700 Service revenue

ASSETS = LIABILITIES +OWNER’SEQUITY

TYPE OF OWNER’SEQUITY

TRANSACTION

Cash +Accounts

Receivable + Supplies +Washing

MachinesAccountsPayable

Jennifer Hill,Capital

(d) # $500 + $500

ASSETS = LIABILITIES +OWNER’SEQUITY

TYPE OF OWNER’SEQUITY

TRANSACTION

Cash +Accounts

Receivable + Supplies +Washing

MachinesAccountsPayable

Jennifer Hill,Capital

(e) + $2,500 + $2,500 Service revenue

e. Jennifer’s Laundromat performed cleaning services for customerstotaling $2,500. These customers paid in cash.

Service Revenues (owner’s equity) is increased by $2,500. Cash (an asset)is increased by $2,500.

f. Jennifer’s Laundromat also performed cleaning services for cus-tomers on account totaling $3,700.

Service Revenues (owner’s equity) is increased by $3,700. AccountsReceivable (an asset) is increased by $3,700.

= +

= +

= +

1eSG_CO1_0131792075.QXD 10/19/06 12:35 PM Page 27

Page 60: polpoa_sg

h. The business paid employees $1,500 for the month of October.

Salary Expense is increased by $1,500, which is a decrease to owner’sequity. Cash (an asset) is decreased by $1,500.

28 Chapter 1 | Do It Yourself! Question 1 Solutions

ASSETS = LIABILITIES +OWNER’SEQUITY

TYPE OF OWNER’SEQUITY

TRANSACTION

Cash +Accounts

Receivable + Supplies +Washing

MachinesAccountsPayable

Jennifer Hill,Capital

(i) + $750 # $750 Utilities expense

ASSETS = LIABILITIES +OWNER’SEQUITY

TYPE OF OWNER’SEQUITY

TRANSACTION

Cash +Accounts

Receivable + Supplies +Washing

MachinesAccountsPayable

Jennifer Hill,Capital

(g)# $1,000 # $1,000 Rent expense

ASSETS = LIABILITIES +OWNER’SEQUITY

TYPE OF OWNER’SEQUITY

TRANSACTION

Cash +Accounts

Receivable + Supplies +Washing

MachinesAccountsPayable

Jennifer Hill,Capital

(h)# $1,500 # $1,500 Salary expense

i. The business received a utility bill for $750. As of October 31, 2008,it had not been paid.

Utilities Expense is increased by $750, which is a decrease to owner’sequity. Accounts Payable (a liability) is increased by $750.

= +

= +

= +

g. Jennifer’s Laundromat paid rent (for the month of October) of$1,000.

Rent Expense is increased by $1,000, which is a decrease to owner’sequity. Cash (an asset) is decreased by $1,000.

1eSG_CO1_0131792075.QXD 10/19/06 12:35 PM Page 28

Page 61: polpoa_sg

Do It Yourself! Question 1 Solutions | Chapter 1 29

k. The business collected $2,250 of accounts receivable.

This transaction decreases Accounts Receivable (an asset) by $2,250.Because the cash is received, Cash (an asset) is increased by $2,250.

ASSETS = LIABILITIES +OWNER’SEQUITY

TYPE OF OWNER’SEQUITY

TRANSACTION

Cash +Accounts

Receivable + Supplies +Washing

MachinesAccountsPayable

Jennifer Hill,Capital

(k)+ $2,250 # $2,250

l. The business paid $2,400 on the accounts payable to the washingmachine store.

This decrease to Accounts Payable (a liability) of $2,400 also decreasesCash (an asset) by $2,400.

ASSETS = LIABILITES +OWNER’SEQUITY

TYPE OF OWNER’SEQUITY

TRANSACTION

Cash +Accounts

Receivable + Supplies +Washing

MachinesAccountsPayable

Jennifer Hill,Capital

(l)#$2,400 #$2,400

ASSETS = LIABILITIES +OWNER’SEQUITY

TYPE OF OWNER’SEQUITY

TRANSACTION

Cash +Accounts

Receivable + Supplies +Washing

MachinesAccountsPayable

Jennifer Hill,Capital

(j) # $2,000 # $2,000Owner

withdrawal

= +

= +

= +

j. Jennifer withdrew $2,000 cash from the business for personal use.

This recordable transaction for the business decreases Cash (an asset) by$2,000 and increases Owner Withdrawals, by $2,000, which decreasesowner’s equity.

1eSG_CO1_0131792075.QXD 10/19/06 12:35 PM Page 29

Page 62: polpoa_sg

30 Chapter 1 | Do It Yourself! Question 1 Solutions

AS

SE

TS

=L

IAB

ILIT

IES

+O

WN

ER

’SE

QU

ITY

TY

PE

OF

OW

NE

R’S

EQ

UIT

YT

RA

NS

AC

TIO

N

Cas

h+

Acc

oun

tsR

ecei

vabl

e+

Su

pp

lies

+W

ash

ing

Mac

hin

esA

ccou

nts

Pay

able

Jen

nif

erH

ill,

Cap

ital

(a)+

$10

,000

+ $1

0,00

0O

wne

r in

vest

men

t

(b)

!4,

000

+ $9

,000

+ 5,

000

(c)

Not

a t

rans

acti

on

of t

he b

usin

ess.

(d)

!50

0+

$500

(e)

+ 2,

500

+ 2,

500

Ser

vice

re

venu

e

(f)

+ $3

,700

+ 3,

700

Ser

vice

re

venu

e

(g)

!1,

000

!1,

000

Ren

t exp

ense

(h)

!1,

500

!1,

500

Sal

ary

expe

nse

(i)

+ 75

0!

750

Uti

liti

es e

xpen

se

(j)!

2,00

0!

2,00

0O

wne

r w

ithd

raw

al

(k)

+ 2,

250

2,25

0

(l)

!2,

400

2,40

0

$3,3

50$1

,450

$500

$9,0

00$3

,350

$10,

950

$14,

300

Req

uire

men

t 2

14

44

44

44

44

44

44

42

44

44

44

44

44

44

44

31

44

44

44

42

44

44

44

3

+=

$14,

300

1eSG_CO1_0131792075.QXD 10/20/06 12:16 PM Page 30

Page 63: polpoa_sg

Do It Yourself! Question 1 Solutions | Chapter 1 31

Assets Liabilities Cash $ 3,350 Accounts payable $ 3,350 Accounts receivable 1,450 Owner’s Equity Supplies 500 Jennifer Hill, capital 10,950 Furniture 9,000 Total assets $14,300 Total liabilities and equity $14,300

JENNIFER’S LAUNDROMATBalance Sheet October 31, 2008

Jennifer Hill, capital, October 1, 2008 $ 0 Add: Investment by owner 10,000 Net income for month 2,950 12,950 Less: Withdrawals by owner (2,000)Jennifer Hill, capital, October 31, 2008 $10,950

JENNIFER’S LAUNDROMATStatement of Owner’s Equity Month Ended October 31, 2008

Revenue Service revenue $ 6,200 Expenses Salary expense $ 1,500 Rent expense 1,000 Utilities expense 750 Total expenses 3,250 Net income $ 2,950

JENNIFER’S LAUNDROMATIncome Statement

Month Ended October 31, 2008

*= $2,500 + $3,700

Requirement 2Prepare the income statement, statement of owner’s equity, and balance sheetof the business as of October 31, 2008.

1eSG_CO1_0131792075.QXD 10/19/06 12:35 PM Page 31

Page 64: polpoa_sg

The Power of PracticeFor more practice using the skills learned in this chapter, visit MyAccountingLab.There you will find algorithmically generated questions that are based on theseDemo Docs and your main textbook’s Review and Assess Your Progress sections.

To go to MyAccountingLab and follow these steps:

1. Direct your URL to www.myaccountinglab.com.2. Log in using your name and password.3. Click the MyAccountingLab link.4. Click Study Plan in the left navigation bar.5. From the table of contents, select Chapter 1, Accounting and the Business

Environment.6. Click a link to work on the tutorial exercises.

32 Chapter 1 | The Power of Practice

1eSG_CO1_0131792075.QXD 10/19/06 12:35 PM Page 32

Page 65: polpoa_sg

WHAT YOU PROBABLY ALREADY KNOW

If you have a checking account, you know that once a month youreceive a statement from the bank. The statement shows thebeginning cash balance, increases, decreases, and the ending cashbalance. The account balance is the amount that the bank owesyou, their customer. Your balance represents a liability to the bank.The deposits you make increase the bank’s liability to you. Thewithdrawals or checks you write decrease that liability. Instead ofusing the terms increase and decrease, businesses have used asystem of accounting for more than 500 years with debits andcredits. Either a debit or a credit may signify an increase to theaccount, depending upon the type of account: asset, liability, orowner’s equity. Your checking account balance is a liability toyour bank; does a debit or credit indicate an increase? When youtake money out of the bank, it decreases the bank’s liability to youbecause you’ve received back a portion of your account balance andis shown as a debit on the bank statement. When you depositmoney into your account, it increases the banks’ liability to you andis reflected as a credit on the bank statement. So, you can see thatyou probably already know that the rule for a liability account isthat increases are shown as credits and decreases as debits.

Learning Objectives

Describe the role of accounts in summarizing business transactions.

Remember that every business transaction has two sides: You receivesomething and you give something. Likewise, every transaction affectstwo or more specific accounts.

The first several chapters of this book contain many new terms that areused in the remaining chapters and are important for you to understand.These terms include the journal, account, ledger, trial balance, and chart ofaccounts. In order to describe the process of recording and summarizingbusiness transactions, it is crucial that you understand these terms nowbefore you proceed with your study of accounting. Also, carefully reviewthe detailed description of the various specific asset, liability, and owner’sequity accounts in the main text.

Explain double-entry accounting.

A business transaction affects two or more specific accounts. Eachaccount has two sides: the left (debit) side, and the right (credit) side.Remember that debit only means left and credit only means right.

Recording BusinessTransactions2

1

2

1eSG_CO2_0131792075.QXD 10/19/06 12:57 PM Page 33

Page 66: polpoa_sg

Increases are recorded on one side and decreases on the other.Depending upon the type of account, a debit may indicate an increase ora decrease. Assets are on the left side of the equation; they increase onthe left (debit) side. Liabilities and owner’s equity are on the right side ofthe equation; they increase on the right (credit) side. Understanding thisbasic concept is crucial. Review Exhibit 2-2 (p. 62) for the debit andcredit rules.

Record and summarize business transactions.

Business transactions are analyzed and then recorded in the journal. Thejournal shows the accounts that are debited and credited to recordbusiness transactions in chronological order. Debits and credits onlyreflect increases and decreases to the accounts. To obtain a specificaccount balance, it is helpful to collect the debit and credit informationrecorded in the journal in one place. Posting—that is, copying the debitsand credits from the journal into the account ledger—provides thisinformation. The difference between the total debits and total credits ineach account is the balance. The balance is shown on the larger side,normally the side to record the increase. Review how to recordtransactions in the journal under “Process of Recording and SummarizingBusiness Transactions,” and especially the five steps of “The TransactionAnalysis.” The more you practice recognizing debits and credits, the easierit will be to understand. Also, review the posting process in Exhibit 2-5(p. 65).

Set up and use a trial balance for preparing financial statements.

After transactions are recorded in the journal and posted to the ledger, atrial balance is prepared. The trial balance lists all the accounts with theirbalances. In a manual accounting system, it is useful as a check todetermine that the total debits equal the total credits. If they areunequal, an error has been made and must be investigated beforeproceeding. Review the trial balance in Exhibit 2-9 (p. 80).

34 Chapter 2 | Recording Business Transactions

3

4

1eSG_CO2_0131792075.QXD 10/19/06 12:57 PM Page 34

Page 67: polpoa_sg

Demo Doc 1Debit/Credit Transaction AnalysisLearning Objectives 1–4

Knight Airlines provides private plane transportation for businesspeople. Knighthad the following trial balance on April 1, 2008:

Cash $50,000 Accounts receivable 8,000 Accounts payable $16,000 Maureen Knight, capital 42,000 Total $58,000 $58,000

Account Title Debit Credit Balance

KNIGHT AIRLINESTrial Balance April 1, 2008

During April, the business had the following transactions:

a. Purchased a new airplane for $50,000. Knight paid $10,000 downand signed a note payable for the remainder.

b. Purchased supplies worth $1,000 on account.

c. Paid $5,000 on account.

d.Transported customers on their planes for fees totaling $25,000.Received $7,500 in cash, with the remaining fees on account.

e. Received $18,000 on account.

f. Paid the following in cash: interest, $1,200; utilities $2,300;salaries, $7,000.

g. Received a bill for airplane repair costs of $3,500 that will be paidnext month.

h. Maureen Knight withdrew $6,000 for personal use.

Requirements

1. Open the following accounts, with the balances indicated, in the ledger ofKnight Airlines. Use the T-account format. (Account numbers are not required.)

• Assets: Cash, $50,000; Accounts Receivable, $8,000; Supplies, no balance;Airplanes, no balance

• Liabilities: Accounts Payable, $16,000; Notes Payable, no balance• Owner’s Equity: Maureen Knight, Capital, $42,000; Maureen Knight,

Withdrawals, no balance• Revenues: Service Revenue, no balance• Expenses: (none have balances) Interest Expense, Utilities Expense, Salary

Expense, Repairs Expense

2. Journalize each transaction. Key journal entries by transaction letter.

3. Post to the ledger.

4. Prepare the trial balance of Knight Airlines at April 30, 2008.

1eSG_CO2_0131792075.QXD 11/22/06 10:15 AM Page 35

Page 68: polpoa_sg

36 Chapter 2 | Demo Doc 1 Solutions

Describe the role ofaccounts in summariz-ing businesstransactions.

1

Explain double-entryaccounting.

2

Record and summarizebusiness transactions.

3

Part 1 Part 2 Part 3 Part 4 Demo DocComplete

Demo Doc 1 Solutions

Remember, an account, the basic summary device, is a record showingincreases, decreases, and the balance of a particular asset, liability, or owner’sequity. A T-account is a visual diagram of the additions and subtractions made tothe accounts. A chart of accounts is a list of all the business’s account titles andaccount numbers assigned to those titles. A chart of accounts does not includeaccount balances. Review the sample chart of accounts in Exhibit 2-1 (p. 61) ofthe main text.

Opening a T-account simply means drawing a blank account (the T) andputting the account title on top. To help find the accounts later, they are usuallyorganized into assets, liabilities, owner’s equity, revenue, and expenses (in thatorder). If the account has a starting balance, it must be put in on the correct side.

Remember that debits are always on the left side of the T-account and cred-its are always on the right side. This rule is true for every account.

The correct side is the side of an increase in the account (unless you arespecifically told differently in the question), because we expect all accounts tohave a positive balance, or more increases than decreases.

For assets, an increase is a debit, so we would expect all assets to have adebit balance. For liabilities and owner’s equity, an increase is a credit, so wewould expect these accounts to have a credit balance. By the same reasoning, weexpect revenues to have a credit balance and expenses and withdrawals to have adebit balance.

The balances listed in Requirement 1 are simply the amounts from thestarting trial balance. We actually did not need to be told how much to put ineach account because we could have read the numbers directly from the April 1trial balance.

Requirement 1

Open the following accounts, with the balances indicated, in the ledger ofKnight Airlines. Use the T-account format. (Account numbers are not required.)

• Assets: Cash, $50,000; Accounts Receivable, $8,000; Supplies, no balance;Airplanes, no balance

• Liabilities: Accounts Payable, $16,000; Notes Payable, no balance

• Owner’s Equity: Maureen Knight, Capital, $42,000; Maureen Knight,Withdrawals, no balance

• Revenues: Service Revenue, no balance

• Expenses: (none have balances) Interest Expense, Utilities Expense, SalaryExpense, Repairs Expense

1eSG_CO2_0131792075.QXD 10/19/06 12:57 PM Page 36

Page 69: polpoa_sg

Demo Doc 1 Solutions | Chapter 2 37

Describe the role ofaccounts in summariz-ing businesstransactions.

1

Explain double-entryaccounting.Record and summarizebusiness transactions.

3

ASSETS LIABILITIES REVENUES

Cash Accounts Payable Service Revenue

Bal. 50,000 Bal. 16,000

Accounts Receivable Notes Payable

Bal. 8,000 EXPENSES

Interest Expense

Supplies OWNER’S EQUITY

Maureen Knight, Capital

Bal. 42,000 Utilities Expense

Airplanes

Maureen Knight,Withdrawals Salary Expense

Repairs Expense

Part 1 Part 2 Part 3 Part 4 Demo DocComplete

a. Purchased a new airplane for $50,000. Knight paid $10,000 downand signed a note payable for the remainder.

The accounts involved are Airplanes, Cash, and Notes Payable. The airplanecost $50,000 and $10,000 was paid in cash, so that means that the notepayable was for $40,000 ($50,000 – $10,000).Airplanes (an asset) is increased, which is a debit. Cash (an asset) isdecreased, which is a credit. Notes Payable (a liability) is increased, whichis a credit.

1 2 3 4

Accounts Affected Type !! "" Dr. or Cr.

Airplanes Asset !! Dr.

Cash Asset "" Cr.

Notes Payable Liability !! Cr.

Requirement 2

Journalize each transaction. Key journal entries by transaction letter.

Feel free to reference Exhibit 2-5 (p. 64) for help in completing this exercise.

2

1eSG_CO2_0131792075.QXD 10/19/06 12:57 PM Page 37

Page 70: polpoa_sg

38 Chapter 2 | Demo Doc 1 Solutions

Journal Entry:

a. Airplanes (Asset, 1; debit) 50,000 Cash (Asset, 2; credit) 10,000 Notes Payable (Liability, 1; credit) 40,000

Purchased airplane.

Accounts Dr. Cr.

Journal Entry:

b. Supplies (Asset,1; debit) 1,000 Accounts Payable (Liability,1; credit) 1,000

Purchased supplies on account.

Accounts Dr. Cr.

Journal Entry:

c. Accounts Payable (Liability, 2; debit) 5,000 Cash (Asset, 2; credit) 5,000

Payment on accounts payable.

Accounts Dr. Cr.

b. Purchased supplies worth $1,000 on account.

One account involved is Supplies. Because the purchase has not yet beenpaid for in cash, the other account involved is Accounts Payable. Supplies (anasset) is increased, which is a debit. Accounts Payable (a liability) isincreased, which is a credit.

1 2 3 4

Accounts Affected Type !! "" Dr. or Cr.

Supplies Asset !! Dr.

Accounts Payable Liability !! Cr.

c. Paid $5,000 on account.

The accounts involved are Accounts Payable and Cash. Accounts Payable (aliability) is decreased, which is a debit. Cash (an asset) is decreased, whichis a credit.

1 2 3 4

Accounts Affected Type !! "" Dr. or Cr.

Accounts Payable Liability "" Dr.

Cash Asset "" Cr.

1eSG_CO2_0131792075.QXD 10/19/06 12:57 PM Page 38

Page 71: polpoa_sg

Demo Doc 1 Solutions | Chapter 2 39

Journal Entry:

d. Accounts Receivable (Asset, 1; debit) 17,500 Cash (Asset, 1; debit) 7,500

Service Revenue (Revenue, 1; credit) 25,000 Provided services for cash and on account.

Accounts Dr. Cr.

d.Transported customers on their planes for fees totaling $25,000.Received $7,500 in cash, with the remaining fees on account.

Knight’s business is flying customers to where they want to go. Transportingcustomers is “performing a service,” and the business earned service reve-nue. The other accounts involved are Cash (because cash was received) andAccounts Receivable (because some customers charged to their account). Thetotal revenue was $25,000 and $7,500 was paid in cash, which means that$17,500 ($25,000 – $7,500) was charged to the customer’s account.

Service Revenue (revenues) is increased, which is a credit. Cash (an asset) isincreased, which is a debit. Accounts Receivable (an asset) is increased,which is a debit.

1 2 3 4

Accounts Affected Type !! "" Dr. or Cr.

Accounts Receivable Asset !! Dr.

Cash Asset !! Dr.

Service Revenue Revenue !! Cr.

e. Received $18,000 on account.

The accounts involved are Cash and Accounts Receivable. Cash (an asset) isincreased, which is a debit. Accounts Receivable (an asset) is decreased,which is a credit.

1 2 3 4

Accounts Affected Type !! "" Dr. or Cr.

Cash Asset !! Dr.

Accounts Receivable Asset "" Cr.

Journal Entry:

e. Cash (Asset, 1; debit) 18,000 Accounts Receivable (Asset, 2; credit) 18,000

Receipt of payment from customer on account.

Accounts Dr. Cr.

1eSG_CO2_0131792075.QXD 10/19/06 12:57 PM Page 39

Page 72: polpoa_sg

40 Chapter 2 | Demo Doc 1 Solutions

Journal Entry:

f. Interest Expense (Expense, 1; debit) 1,200 Utilities Expense (Expense, 1; debit) 2,300 Salary Expense (Expense, 1; debit) 7,000

Cash (Asset, 2; credit) 10,500 Payment for expenses.

Accounts Dr. Cr.

Journal Entry:

g. Repairs Expense (Expense, 1; debit) 3,500 Accounts Payable (Liability, 1; credit) 3,500

Received repair bill.

Accounts Dr. Cr.

f. Paid the following` in cash: interest, $1,200; utilities, $2,300;salaries, $7,000.

Interest, utilities, and salaries are usually not paid until after the benefithas been received/used. These past benefits are considered expenses. Theaccounts involved are Interest Expense, Utilities Expense, Salary Expense,and Cash.Interest Expense, Utilities Expense, and Salary Expense (all expenses) areincreased, which are debits. Cash (an asset) is decreased, which is a credit.

1 2 3 4

Accounts Affected Type !! "" Dr. or Cr.

Interest Expense Expense !! Dr.

Utilities Expense Expense !! Dr.

Salary Expense Expense !! Dr.

Cash Asset "" Cr.

g. Received a bill for airplane repair costs of $3,500 that will be paidnext month.

Repairs are not billed until after they have been performed. So the billreceived was for repairs made in the past. This past benefit is recorded as anexpense. So the accounts involved are Repairs Expense and Accounts Payable.Repairs Expense (an expense) is increased, which is a debit. AccountsPayable (a liability) is increased, which is a credit.

1 2 3 4

Accounts Affected Type !! "" Dr. or Cr.

Repairs Expense Expense !! Dr.

Accounts Payable Liability !! Cr.

1eSG_CO2_0131792075.QXD 10/19/06 12:57 PM Page 40

Page 73: polpoa_sg

Demo Doc 1 Solutions | Chapter 2 41

Record and summarizebusiness transactions.

3

Journal Entry:

h. Maureen Knight, Withdrawals (Equity, 2; debit) 6,000 Cash (Asset, 2; credit) 6,000

Cash withdrawal by owner.

Accounts Dr. Cr.

h. Maureen Knight withdrew $6,000 for personal use.

The accounts involved are Maureen Knight, Withdrawals, and Cash.Withdrawals is increased, which is a debit. This transaction results in anincrease to the Withdrawals account, which is the same as a decrease toowner’s equity, (both are debits). Cash (an asset) is decreased, which is acredit.

1 2 3 4

Accounts Affected Type !! "" Dr. or Cr.

Maureen Knight, Withdrawals Equity "" Dr.

Cash Asset "" Cr.

Part 1 Part 2 Part 3 Part 4 Demo DocComplete

The entire group of accounts is called the ledger. A manual system would have abook of account pages and a computerized system would be a printout of allaccounts.

Amounts in the journal entries are put into the individual ledger T-accounts.Debits go on the left side, and credits go on the right side.

To add up a T-account, total the debit/left side and total the credit/right side.Subtract the smaller number from the bigger number and put the difference onthe side of the bigger number. This procedure gives the balance in the T-account(the net total of both sides combined).

For example, with Accounts Receivable, the two numbers on the left sidetotal $25,500 ($8,000 + $17,500). The credit/right side totals $18,000. The differ-ence is $7,500 ($25,500 – $18,000). We put the $7,500 on the debit side becausethat was the side of the bigger number of $25,500.

Another way to think of totaling T-accounts is:

Beginning Balance in T-account! Increases to T-account" Decreases to T-account T-account Balance (total)

Requirement 3

Post to the ledger.

1eSG_CO2_0131792075.QXD 10/19/06 12:57 PM Page 41

Page 74: polpoa_sg

42 Chapter 2 | Demo Doc 1 Solutions

ASSETS LIABILITIES REVENUES

Cash Accounts Payable Service Revenue

Bal. 50,000 Bal. 16,000 d. 25,000a. 10,000 b. 1,000 25,000c. 5,000 c. 5,000

d. 7,500 g. 3,500e. 18,000 15,500

f. 10,500EXPENSESh. 6,000

44,000 Notes Payable Interest Expense

a. 40,000 f. 1,200

Accounts Receivable 40,000 1,200

Bal. 8,000d. 17,500 Utilities Expense

e. 18,000 OWNER’S EQUITY f. 2,300

7,500 Maureen Knight, Capital 2,300

Bal. 42,000

Supplies 42,000 Salary Expense

b. 1,000 f. 7,000

1,000 Maureen Knight, 7,000Withdrawals

Airplanes h. 6,000 Repairs Expense

a. 50,000 6,000 g. 3,500

50,000 3,500

Set up and use a trialbalance for preparingfinancial statements.

4

Part 1 Part 2 Part 3 Part 4 Demo DocComplete

All of the debits and credits are now ready for the trial balance—a listing of allthe account titles with their balances at the end of an accounting period. Again,the accounts are listed in the assets, liabilities, equity, revenues, expenses orderfor consistency.

Requirement 4

Prepare the trial balance of Knight Airlines at April 30, 2008.

1eSG_CO2_0131792075.QXD 10/19/06 12:57 PM Page 42

Page 75: polpoa_sg

Demo Doc 1 Solutions | Chapter 2 43

Cash $ 44,000 Accounts receivable 7,500 Supplies 1,000 Airplanes 50,000 Accounts payable $ 15,500 Notes payable 40,000 Maureen Knight, capital 42,000 Maureen Knight, withdrawals 6,000 Service revenue 25,000 Interest expense 1,200 Utilities expense 2,300 Salary expense 7,000 Repairs expense 3,500 Total $122,500 $122,500

Account Title Debit Credit Balance

KNIGHT AIRLINESTrial Balance April 30, 2008

Part 1 Part 2 Part 3 Part 4 Demo DocComplete

1eSG_CO2_0131792075.QXD 10/19/06 12:57 PM Page 43

Page 76: polpoa_sg

44 Chapter 2 | Quick Practice Questions

Quick Practice QuestionsTrue/False

1. A ledger is a chronological record of transactions.

2. A chart of accounts lists all the accounts and their balances.

3. An asset is an economic resource that will benefit the business in the future.

4. A note receivable is a written pledge that the customer will pay a fixed amount of money by a certain date.

5. Posting is the process of transferring information from the trial balance to the financial statements.

6. Prepaid expenses are listed as expenses on the income statement.

7. When an owner withdraws cash from the business, assets and owner’s equity decrease.

8. When a business makes a payment on account, assets decrease and liabilities increase.

9. Every transaction affects only two accounts.

10. T-accounts help to summarize transactions.

1eSG_CO2_0131792075.QXD 10/19/06 12:57 PM Page 44

Page 77: polpoa_sg

Quick Practice Questions | Chapter 2 45

Multiple Choice

1. A business transaction is first recorded in which of thefollowing?a. Chart of accountsb. Journalc. Ledgerd. Trial balance

2. A trial balance contains which types of accounts?a. Assetsb. Revenue and expense accountsc. All ledger accountsd. Assets and revenue accounts

3. An owner withdrawing cash for personal use would cause whichof the following effects?a. No effect on assetsb. Decrease in owner’s equityc. Decrease in net incomed. Both b and c

4. Which of the following is the account that would be debitedwhen supplies are purchased for cash?a. Cashb. Owner’s Capitalc. Accounts Payabled. Supplies

5. A business makes a cash payment of $12,000 to a creditor. Whichof the following occurs?a. Cash is credited for $12,000.b. Cash is debited for $12,000.c. Accounts Payable is credited for $12,000.d. Both a and c.

6. The last step in the journalizing process is which of thefollowing?a. Determine the accounts involved in the transaction.b. Post the transaction to the ledger.c. Enter the transaction in the journal.d. Identify the transaction and its data.

7. When you borrow cash from a bank, what two accounts areaffected?a. Cash and Notes Receivableb. Cash and Accounts Payablec. Cash and Notes Payabled. The only account affected is Cash

1eSG_CO2_0131792075.QXD 10/19/06 12:57 PM Page 45

Page 78: polpoa_sg

46 Chapter 2 | Quick Practice Questions

Journal Entry:

a. Service Revenue 650 Cash 650

b Accounts Receivable 650 Service Revenue 650 c. Accounts Receivable 650 Cash 650 d. Cash 650 Accounts Receivable 650

Accounts Dr. Cr.

8. Which of the following is the correct journal entry for apurchase of supplies for $100 cash?

Journal Entry:

a. Equipment 9,000 Cash 9,000

b Accounts Payable 9,000 Equipment 9,000 c. Equipment 9,000 Accounts Payable 9,000 d. Cash 9,000 Equipment 9,000

Accounts Dr. Cr.

Journal Entry:

a. Supplies 100 Cash 100

b Cash 100 Supplies 100 c. Accounts Payable 100 Supplies 100 d. Supplies 100 Owner’s Capital 100

Accounts Dr. Cr.

9. Which of the following is the correct journal entry forpurchasing $9,000 worth of equipment on account?

10. Which of the following is the correct journal entry to record thereceipt of $650 on account for services previously rendered?

1eSG_CO2_0131792075.QXD 10/19/06 12:57 PM Page 46

Page 79: polpoa_sg

Quick Practice Questions | Chapter 2 47

Journal Entry:

Accounts Receivable 2,000 Service Revenue 2,000

Accounts Dr. Cr.

Journal Entry:

Cash 10,000 Owner, Capital 10,000

Accounts Dr. Cr.

Journal Entry:

Supplies 500 Accounts Payable 500

Accounts Dr. Cr.

Journal Entry:

Cash 3,000Service Revenue 3,000

Accounts Dr. Cr.

a. __________________________________________________________

b. __________________________________________________________

c. __________________________________________________________

Quick Exercises

2-1. Indicate whether a debit or credit is required to record anincrease for each of these accounts.

_______ Cash _______ Prepaid Rent_______ Owner, Withdrawals _______ Notes Payable_______ Salary Expense _______ Land_______ Service Revenue _______ Utilities Expense

2-2. Write a brief explanation for the following transactions:

d. _________________________________________________________

1eSG_CO2_0131792075.QXD 10/19/06 12:57 PM Page 47

Page 80: polpoa_sg

48 Chapter 2 | Quick Practice Questions

Journal Entry:

Date Accounts Dr. Cr.

Journal Entry:

Date Accounts Dr. Cr.

e. __________________________________________________________

2-3. Identify the following as an asset, liability, owner’s equity,revenue, or expense account. Also indicate the normal balance.

Account Normal Balance

a. Building ________________ ________________

b. Accounts Payable ________________ ________________

c. Cash ________________ ________________

d. Accounts Receivable ________________ ________________

e. Prepaid Insurance ________________ ________________

f. Supplies ________________ ________________

g. Utilities Expense ________________ ________________

h. Owner, Capital ________________ ________________

i. Owner, Withdrawals ________________ ________________

2-4. Journalize the following transactions for the Reid Public elations Company using the following accounts: Cash;Accounts Receivable; Notes Receivable; Supplies; PrepaidInsurance; Accounts Payable; Notes Payable; Reid, Capital;Reid, Withdrawals; Service Revenue; Salary Expense; RentExpense; and Insurance Expense.

March 1 J. Reid invested $25,000 cash to begin her public relationscompany.

Journal Entry:

Accounts Payable 300 Cash 300

Accounts Dr. Cr.

March 2 Paid $ 3,000 for March rent.

1eSG_CO2_0131792075.QXD 10/19/06 12:57 PM Page 48

Page 81: polpoa_sg

Quick Practice Questions | Chapter 2 49

Journal Entry:

Date Accounts Dr. Cr.

Journal Entry:

Date Accounts Dr. Cr.

Journal Entry:

Date Accounts Dr. Cr.

Journal Entry:

Date Accounts Dr. Cr.

March 4 Purchased $825 of supplies on account.

March 5 Performed $10,000 of services for a client on account.

March 8 Paid salaries of $2,500.

March 15 Paid the semiannual insurance premium of $1,800 for theperiod March 15 through September 15.

March 20 Signed a bank note and borrowed $20,000 cash.

Journal Entry:

Date Accounts Dr. Cr.

1eSG_CO2_0131792075.QXD 10/19/06 12:57 PM Page 49

Page 82: polpoa_sg

50 Chapter 2 | Quick Practice Questions

Cash $ 30,000 Accounts receivable $ 2,000 Supplies 600 Land 50,000 Accounts payable 2,600 Notes payable 35,000 Jo Coleman, capital 42,550 Jo Coleman, withdrawals 2,400 Service revenue 9,300 Salary expense 2,500 Rent expense 1,200 Interest expense 500 Utilities expense 250 Total $ 83,200 $ 95,770

Account Title Debit Credit Balance

COLEMAN COPY CENTERTrial Balance March 31, 2008

March 25 Received $10,000 from customers on account. (See March 5.)

2-5. Find the errors in the trial balance presented here and pre-pare a corrected trial balance.

Cash Accounts receivable Supplies Land Accounts payable Notes payable Jo Coleman, capital Jo Coleman, withdrawals Service revenue Salary expense Rent expense Interest expense Utilities expense Total

Account Title Debit Credit Balance

COLEMAN COPY CENTERTrial Balance March 31, 2008

Journal Entry:

Date Accounts Dr. Cr.

1eSG_CO2_0131792075.QXD 10/19/06 12:58 PM Page 50

Page 83: polpoa_sg

Do It Yourself! Question 1 | Chapter 2 51

Cash $ 6,000 Accounts receivable 1,200 Accounts payable $ 700 Ted Johnson, capital 6,500 Total $ 7,200 $ 7,200

Account Title Debit Credit Balance

TED’S ROOFING SHOPTrial Balance

September 1, 2008

Describe the role ofaccounts in summariz-ing businesstransactions.

1

Explain double-entryaccounting.

2

Record and summarizebusiness transactions.

3

Do It Yourself! Question 1

Ted’s Roofing Shop had the following trial balance on September 1, 2008:

Journal Entry:

Date Accounts Dr. Cr.

Requirements

1. Journalize each of the following transactions. Key journal entries bytransaction letter. (Account numbers are not required.)

a. Performed roof repairs for customers and earned $800 in cash and$1,500 of revenue on account.

1 2 3 4

Accounts Affected Type !! "" Dr. or Cr.

1eSG_CO2_0131792075.QXD 10/19/06 12:58 PM Page 51

Page 84: polpoa_sg

52 Chapter 2 | Do It Yourself! Question 1

Journal Entry:

Date Accounts Dr. Cr.

c. Took out a loan of $2,000 cash from City Bank.

1 2 3 4

Accounts Affected Type !! "" Dr. or Cr.

Journal Entry:

Date Accounts Dr. Cr.

b. Paid $200 cash for supplies.

1 2 3 4

Accounts Affected Type !! "" Dr. or Cr.

1eSG_CO2_0131792075.QXD 10/19/06 12:58 PM Page 52

Page 85: polpoa_sg

Do It Yourself! Question 1 | Chapter 2 53

Journal Entry:

Date Accounts Dr. Cr.

e. Paid the following in cash: interest, $75; repairs, $825; salaries,$1,000.

1 2 3 4

Accounts Affected Type !! "" Dr. or Cr.

Journal Entry:

Date Accounts Dr. Cr.

d. Paid $3,000 cash to purchase repair tools.

1 2 3 4

Accounts Affected Type !! "" Dr. or Cr.

1eSG_CO2_0131792075.QXD 10/19/06 12:58 PM Page 53

Page 86: polpoa_sg

54 Chapter 2 | Do It Yourself! Question 1

Journal Entry:

Date Accounts Dr. Cr.

g. Paid $500 on account.

1 2 3 4

Accounts Affected Type !! "" Dr. or Cr.

Journal Entry:

Date Accounts Dr. Cr.

f.Received a telephone bill of $100 that will be paid next month.

1 2 3 4

Accounts Affected Type !! "" Dr. or Cr.

1eSG_CO2_0131792075.QXD 10/19/06 12:58 PM Page 54

Page 87: polpoa_sg

Do It Yourself! Question 1 | Chapter 2 55

2. Open the following accounts, with the balances indicated, in the ledger ofTed’s Roofing Shop. Use the T-account format.

• Assets: Cash, $6,000; Accounts Receivable, $1,200, Supplies, no balance;Repair Tools, no balance

• Liabilities: Accounts Payable, $700; Notes Payable, no balance

• Owner’s Equity: Ted Johnson, Capital, $6,500; Ted Johnson, Withdrawals, nobalance

• Revenues: Service Revenue, no balance

• Expenses: (none have balances) Interest Expense, Repairs Expense, SalaryExpense, Utilities Expense

Journal Entry:

Date Accounts Dr. Cr.

i. Ted withdrew $1,300 for personal use.

1 2 3 4

Accounts Affected Type !! "" Dr. or Cr.

Journal Entry:

Date Accounts Dr. Cr.

h. Received $1,100 on account.

1 2 3 4

Accounts Affected Type !! "" Dr. or Cr.

1eSG_CO2_0131792075.QXD 10/19/06 12:58 PM Page 55

Page 88: polpoa_sg

3. Prepare the trial balance of Ted’s Roofing Shop at September 30, 2008.

Account Title Debit Credit Balance

TED’S ROOFING SHOPTrial Balance

September 30, 2008

Set up and use a trialbalance for preparingfinancial statements.

4

Record and summarizebusiness transactions.

3

ASSETS LIABILITIES REVENUES

Cash Accounts Payable Service Revenue

EXPENSES

Notes Payable Interest Expense

Accounts Receivable

Repairs Expense

OWNER’S EQUITY

Ted Johnson, Capital

Supplies Salary Expense

Ted Johnson,Withdrawals

Repair Tools Utilities Expense

Post all transactions in Requirement 1 to the ledger.

1eSG_CO2_0131792075.QXD 10/19/06 12:58 PM Page 56

Page 89: polpoa_sg

Quick Practice Solutions | Chapter 2 57

Quick Practice SolutionsTrue/False

F 1. A ledger is a chronological record of transactions.False–A journal contains a chronological record of transactions. A ledger is a collection of the accounts and summarizes their balances. (p. 64)

F 2. A chart of accounts lists all the accounts and their balances.False–A chart of accounts lists all the accounts along with account numbers. A trial balance lists all accounts and their balances.(p. 60)

T 3. An asset is an economic resource that will benefit the business in the future. (pp. 58–59)

T 4. A note receivable is a written pledge that the customer will pay a fixed amount of money by a certain date. (p. 59)

F 5. Posting is the process of transferring information from the trial balance to the financial statements.False–Posting is the process of transferring information from the journal to the ledger. (p. 64)

F 6. Prepaid expenses are listed as expenses on the income statement.False–Prepaid expenses are assets and are listed on the balance sheet. Can you name an example of a prepaid expense? (p. 59)

T 7. When an owner withdraws cash from the business, assets and owner’s equity decrease. (p. 62)

F 8. When a business makes a payment on account, assets decrease and liabilities increase.False–When a business makes a payment on account, assets decrease and liabilities decrease. (p. 62)

F 9. Every transaction affects only two accounts.False–Every transaction affects at least two accounts. Watch the wording here! Notice that the question uses the word only instead of at least. Can you describe a transaction that affects more than two accounts? (p. 64)

T 10. T-accounts help to summarize transactions. (p. 61)

1eSG_CO2_0131792075.QXD 10/19/06 12:58 PM Page 57

Page 90: polpoa_sg

58 Chapter 2 | Quick Practice Solutions

Multiple Choice

1. A business transaction is first recorded in which of thefollowing? (p. 64)a. Chart of accountsb. Journalc. Ledgerd. Trial balance

2. A trial balance contains which types of accounts? (p. 80)a. Assetsb. Revenue and expense accountsc. All ledger accountsd. Assets and revenue accounts

3. An owner withdrawing cash for personal use would cause whichof the following effects? (p. 62)a. No effect on assetsb. Decrease owner’s equityc. Decrease net incomed. Both b and c

4. Which of the following is the account that would be debitedwhen supplies are purchased for cash? (p. 59)a. Cashb. Owner’s Capitalc. Accounts Payabled. Supplies

5. A business makes a cash payment of $12,000 to a creditor. Whichof the following occurs? (p. 89)a. Cash is credited for $12,000.b. Cash is debited for $12,000.c. Accounts Payable is credited for $12,000.d. Both a and c.

6. The last step in the journalizing process is which of thefollowing? (p. 64)a. Determine the accounts involved in the transaction.b. Post the transaction to the ledger.c. Enter the transaction in the journal.d. Identify the transaction and its data.

7. When you borrow cash from a bank, what two accounts areaffected? (p. 68)a. Cash and Notes Receivableb. Cash and Accounts Payablec. Cash and Notes Payabled. The only account affected is Cash

1eSG_CO2_0131792075.QXD 10/19/06 12:58 PM Page 58

Page 91: polpoa_sg

Quick Practice Solutions | Chapter 2 59

Journal Entry:

a. Supplies 100 Cash 100

b Cash 100 Supplies 100 c. Accounts Payable 100 Supplies 100 d. Supplies 100 Owner’s Capital 100

Accounts Dr. Cr.

Journal Entry:

a. Equipment 9,000 Cash 9,000

b Accounts Payable 9,000 Equipment 9,000 c. Equipment 9,000 Accounts Payable 9,000 d. Cash 9,000 Equipment 9,000

Accounts Dr. Cr.

9. Which of the following is the correct journal entry forpurchasing $9,000 worth of equipment on account? (p. 87)

10. Which of the following is the correct journal entry to recordthe receipt of $650 on account for services previouslyrendered? (p. 93)

Journal Entry:

a. Service Revenue 650 Cash 650

b Accounts Receivable 650 Service Revenue 650 c. Accounts Receivable 650 Cash 650 d. Cash 650 Accounts Receivable 650

Accounts Dr. Cr.

8. Which of the following is the correct journal entry for apurchase of supplies for $100 cash? (p. 88)

1eSG_CO2_0131792075.QXD 10/19/06 12:58 PM Page 59

Page 92: polpoa_sg

60 Chapter 2 | Quick Practice Solutions

Quick Exercises

2-1. Indicate whether a debit or credit is required to record anincrease for each of these accounts. (p. 62)

Dr. Cash Dr. Prepaid RentDr. Owner, Withdrawals Cr. Notes PayableDr. Salary Expense Dr. LandCr. Service Revenue Dr. Utilities Expense

2-2. Write a brief explanation for the following transactions:(pp. 69–76)

Journal Entry:

Cash 10,000 Owner, Capital 10,000

Accounts Dr. Cr.

a. Owner investment of cash.

Journal Entry:

Supplies 500 Accounts Payable 500

Accounts Dr. Cr.

b. Purchased supplies on account.

Journal Entry:

Cash 3,000 Service Revenue 3,000

Accounts Dr. Cr.

c. Received cash for services performed.

Journal Entry:

Accounts Receivable 2,000 Service Revenue 2,000

Accounts Dr. Cr.

d. Performed services on account.

1eSG_CO2_0131792075.QXD 10/19/06 12:58 PM Page 60

Page 93: polpoa_sg

Quick Practice Solutions | Chapter 2 61

Journal Entry:

Accounts Payable 300 Cash 300

Accounts Dr. Cr.

March 2 Paid $ 3,000 for March rent.

e. Paid cash on account.

2-3. Identify the following as an asset, liability, owner’s equity,revenue, or expense account. Also indicate the normal bal-ance. (pp. 58–60)

Account Normal Balance

a. Building Asset Debit

b. Accounts Payable Liability Credit

c. Cash Asset Debit

d. Accounts Receivable Asset Debit

e. Prepaid Insurance Asset Debit

f. Supplies Asset Debit

g. Utilities Expense Expense Debit

h. Owner, Capital Owner’s Equity Credit

i. Owner, Withdrawals Owner’s Equity Debit

2-4. Journalize the transactions for the Reid Public RelationsCompany using the following accounts: Cash; AccountsReceivable; Notes Receivable; Supplies; Prepaid Insurance;Accounts Payable; Notes Payable; Reid, Capital; Reid,Withdrawals; Service Revenue; Salary Expense; Rent Expense;and Insurance Expense. (pp. 62–65)

March 1 J. Reid invested $25,000 cash to begin her public relationscompany.

Journal Entry:

March 2 Rent Expense 3,000 Cash 3,000

Date Accounts Dr. Cr.

Journal Entry:

March 1 Cash 25,000 Reid, Capital 25,000

Date Accounts Dr. Cr.

1eSG_CO2_0131792075.QXD 10/19/06 12:58 PM Page 61

Page 94: polpoa_sg

Journal Entry:

March 15 Prepaid Insurance 1,800 Cash 1,800

Date Accounts Dr. Cr.

Journal Entry:

March 8 Salary Expense 2,500 Cash 2,500

Date Accounts Dr. Cr.

Journal Entry:

March 4 Supplies 825 Accounts Payable 825

Date Accounts Dr. Cr.

Journal Entry:

March 5 Accounts Receivable 10,000 Service Revenue 10,000

Date Accounts Dr. Cr.

March 4 Purchased $825 of supplies on account.

March 5 Performed $10,000 of services for a client on account.

March 8 Paid salaries of $2,500.

March 15 Paid the semiannual insurance premium of $1,800 for theperiod March 15 through September 15.

March 20 Signed a bank note and borrowed $20,000 cash.

March 25 Received $10,000 from customers on account. (See March 5.)

Journal Entry:

March 20 Cash 20,000 Notes Payable 20,000

Date Accounts Dr. Cr.

Journal Entry:

March 25 Cash 10,000 Accounts Receivable 10,000

Date Accounts Dr. Cr.

1eSG_CO2_0131792075.QXD 10/19/06 12:58 PM Page 62

Page 95: polpoa_sg

Quick Practice Solutions | Chapter 2 63

2-5. Find the errors in the trial balance and prepare a correctedtrial balance. (pp. 80–83)

Cash $30,000 Accounts receivable 2,000 Supplies 600 Land 50,000 Accounts payable $ 2,600 Notes payable 35,000 Jo Coleman, capital 42,550 Jo Coleman, withdrawals 2,400 Service revenue 9,300 Salary expense 2,500 Rent expense 1,200 Interest expense 500 Utilities expense 250 Total $ 89,450 $ 89,450

Account Title Debit Credit Balance

COLEMAN COPY CENTERTrial Balance March 31, 2008

1eSG_CO2_0131792075.QXD 10/19/06 12:58 PM Page 63

Page 96: polpoa_sg

64 Chapter 2 | Do It Yourself! Question 1 Solutions

Journal Entry:

b. Supplies (Asset, 1; debit) 200 Cash (Asset, 2; credit) 200

Purchased supplies for cash.

Date Accounts Dr. Cr.

b. Paid $200 cash for supplies

1 2 3 4

Accounts Affected Type !! "" Dr. or Cr.

Supplies Asset !! Dr.

Cash Asset "" Cr.

Journal Entry:

a. Accounts Receivable (Asset, 1; debit) 1,500 Cash (Asset, 1; debit) 800

Service Revenue (Revenue, 1; credit) 2,300 Provided services for cash and on account.

Date Accounts Dr. Cr.

Do It Yourself! Question 1 Solutions

Requirement 1

Journalize each of the following transactions. Key journal entries by transactionletter. (Account numbers are not required.)

a. Performed roof repairs for customers and earned $800 on in cashand $1,500 of revenue on account.

1 2 3 4

Accounts Affected Type !! "" Dr. or Cr.

Accounts Receivable Asset !! Dr.

Cash Asset !! Dr.

Service Revenue Revenue !! Cr.

1eSG_CO2_0131792075.QXD 11/22/06 10:16 AM Page 64

Page 97: polpoa_sg

Do It Yourself! Question 1 Solutions | Chapter 2 65

d. Paid $3,000 cash to purchase repair tools.

1 2 3 4

Accounts Affected Type !! "" Dr. or Cr.

Repair Tools Asset !! Dr.

Cash Asset "" Cr.

e. Paid the following in cash: interest, $75; repairs, $825; salaries, $1,000.

1 2 3 4

Accounts Affected Type !! "" Dr. or Cr.

Interest Expense Expense !! Dr.

Repairs Expense Expense !! Dr.

Salary Expense Expense !! Dr.

Cash Asset "" Cr.

Journal Entry:

c. Cash (Asset, 1; debit) 2,000 Notes Payable (Liability, 1; credit) 2,000

Took out loan for cash.

Date Accounts Dr. Cr.

Journal Entry:

d. Repair Tools (Asset, 1; debit) 3,000 Cash (Asset, 2; credit) 3,000

Purchased repair tools.

Date Accounts Dr. Cr.

c. Took out a loan of $2,000 cash from City Bank

1 2 3 4

Accounts Affected Type !! "" Dr. or Cr.

Cash Asset !! Dr.

Notes Payable Liability !! Cr.

Journal Entry:

e. Interest Expense (Expense, 1; debit) 75 Repairs Expense (Expense, 1; debit) 825 Salary Expense (Expense, 1; debit) 1,000

Cash (Asset, 2; credit) 1,900 Payment for expenses.

Date Accounts Dr. Cr.

1eSG_CO2_0131792075.QXD 10/19/06 12:58 PM Page 65

Page 98: polpoa_sg

66 Chapter 2 | Do It Yourself! Question 1 Solutions

g. Paid $500 on account.

1 2 3 4

Accounts Affected Type !! "" Dr. or Cr.

Accounts Payable Liability "" Dr.

Cash Asset "" Cr.

h. Received $1,100 on account.

1 2 3 4

Accounts Affected Type !! "" Dr. or Cr.

Cash Asset !! Dr.

Accounts Receivable Asset "" Cr.

Journal Entry:

h. Cash (Asset, 1; debit) 1,100 Accounts Receivable (Asset, 2; credit) 1,100

Receipt of payment from customer on account.

Date Accounts Dr. Cr.

Journal Entry:

f. Utilities Expense (Expense, 1; debit) 100 Accounts Payable (Liability, 1; credit) 100

Received utility bill.

Date Accounts Dr. Cr.

Journal Entry:

g. Accounts Payable (Liability, 2; debit) 500 Cash (Asset, 2; credit) 500

Payment on accounts payable.

Date Accounts Dr. Cr.

f. Received a telephone bill of $100 that will be paid next month.

1 2 3 4

Accounts Affected Type !! "" Dr. or Cr.

Utilities Expense Expense !! Dr.

Accounts Payable Liability !! Cr.

1eSG_CO2_0131792075.QXD 10/19/06 12:58 PM Page 66

Page 99: polpoa_sg

Do It Yourself! Question 1 Solutions | Chapter 2 67

Requirement 2

Open the following accounts, with the balances indicated, in the ledger of Ted’sRoofing Shop. Use the T-account format.

• Assets: Cash, $6,000; Accounts Receivable, $1,200, Supplies, no balance;Repair Tools, no balance

• Liabilities: Accounts Payable, $700; Notes Payable, no balance

• Owner’s Equity: Ted Johnson, Capital, $6,500; Ted Johnson, Withdrawals, nobalance

• Revenues: Service Revenue, no balance

• Expenses: (none have balances) Interest Expense, Repairs Expense, SalaryExpense, Utilities Expense

i. Ted withdrew $1,300 for personal use.

1 2 3 4

Accounts Affected Type !! "" Dr. or Cr.

Ted Johnson, Withdrawals Equity "" Dr.

Cash Asset "" Cr.

Journal Entry:

i. Ted Johnson, Withdrawals (Equity, 2; debit) 1,300 Cash (Asset, 2; credit) 1,300

Cash withdrawal by owner.

Date Accounts Dr. Cr.

1eSG_CO2_0131792075.QXD 10/19/06 12:58 PM Page 67

Page 100: polpoa_sg

ASSETS LIABILITIES REVENUES

Cash Accounts Payable Service Revenue

Bal. 6,000 Bal. 700 a. 2,300a. 800 f. 100 2,300

b. 200 g. 500c. 2,000 300

d. 3,000EXPENSESe. 1,900

g. 500 Notes Payable Interest Expenseh. 1,100 c. 2,000 e. 75

i. 1,3002,000 75

3,000

OWNER’S EQUITY

Accounts Receivable Ted Johnson, Capital Repairs Expense

Bal. 1,200 Bal. 6,500 e. 825a. 1,500 6,500 825

h. 1,100

1,600 Ted Johnson,Withdrawals Salary Expense

i. 1,300 e. 1,000

1,300 1,000

Supplies Utilities Expense

b. 200 f. 100

200 100

Repair Tools

d. 3,000

3,000

Post all transactions in Requirement 1 to the ledger.

68 Chapter 2 | Do It Yourself! Question 1 Solutions

1eSG_CO2_0131792075.QXD 10/19/06 12:58 PM Page 68

Page 101: polpoa_sg

Do It Yourself! Question 1 Solutions | Chapter 2 69

Requirement 3Prepare the trial balance of Ted’s Roofing Shop at September 30, 2008.

Cash $ 3,000 Accounts receivable 1,600 Supplies 200 Repair tools 3,000 Accounts payable $ 300 Notes payable 2,000 Ted Johnson, capital 6,500 Ted Johnson, withdrawals 1,300 Service revenue 2,300 Interest expense 75 Repairs expense 825 Salary expense 1,000 Utilities expense 100 Total $ 11,100 $ 11,100

Account Title Debit Credit Balance

TED’S ROOFING SHOPTrial Balance

September 30, 2008

1eSG_CO2_0131792075.QXD 10/19/06 12:58 PM Page 69

Page 102: polpoa_sg

The Power of PracticeFor more practice using the skills learned in this chapter, visit MyAccountingLab.There you will find algorithmically generated questions that are based on theseDemo Docs and your main textbook’s Review and Accounting Practice sections.

Go to MyAccountingLab and follow these steps:

1. Direct your URL to www.myaccountinglab.com.2. Log in using your name and password.3. Click the MyAccountingLab link.4. Click Study Plan in the left navigation bar.5. From the table of contents, select Chapter 2, Recording Business

Transactions.6. Click a link to work on the tutorial exercises.

70 The Power of Practice | Chapter 2

1eSG_CO2_0131792075.QXD 10/19/06 12:58 PM Page 70

Page 103: polpoa_sg

The AdjustingProcess3

1

WHAT YOU PROBABLY ALREADY KNOW

When you receive your car insurance bill, the period of coverage isalways in the future. The bill may indicate that your payment mustbe received no later than 12:01 AM on the day after your currentcoverage expires to maintain your policy. Your payment is actuallya prepayment, prepaid insurance. Prepaid insurance is an assetbecause the insurance coverage is a future benefit. But every daythat the car is protected by the insurance policy, part of the benefitis used up. When an asset is used up, it becomes an expense.Technically, every day you are incurring an expense of 1/365th ofyour annual premium. Assume that you paid $730 for an annualinsurance policy in December 2007 for the period covering January1–December 31, 2008. Each day beginning January 1, you areusing up $2 ($730/365 days) of the prepaid insurance and incurringan expense or benefit of $2. At the end of January 1, what is yourfuture benefit? It is $728 because you’ve benefited from theinsurance coverage service you received that day. Technically, youhave prepaid insurance with a reduced value of $728 and anexpense of $2; the total $730 payment is split between the twoaccounts. Each day, an additional $2 expense is accrued and theasset account contains $2 less in value. Although it would be toocumbersome to “adjust” these accounts on a daily basis, businessesgenerally make adjustments to their records whenever financialstatements are prepared.

Learning Objectives

Describe the accounting principles that help businesses measureincome.

• The revenue principle tells us when revenue is earned and whatamount of revenue to record. The matching principle requires that allof the expenses incurred during a period are recorded and matched(recorded in the same time period) against the revenues.

• The time period concept ensures that information is reported often.

Refer to Exhibit 3-1 (p. 127) of the main text for a visual overview of theaccounting principles for measuring income.

Describe the four types of adjusting entries.

Adjusting entries are recorded at the end of the accounting period whenfinancial statements are prepared. The objective of adjusting entries is

2

1eSG_CO3_0131792075.QXD 10/19/06 1:16 PM Page 71

Page 104: polpoa_sg

72 Chapter 3 | The Adjusting Process

3

4

to update the account balances and to accurately measure net incomeor net loss. The four types of adjusting entries are:

• Prepaid expenses: Cost (future expenses) that are paid for in advanceof being used.

• Unearned revenue: Cash received from customers before revenue isearned.

• Accrued expenses: An expense the business has incurred but not yetpaid.

• Accrued revenue: A revenue that has been earned but not collectedin cash.

Review Exhibits 3-2 and 3-3 (pp. 128–129) in the main text for a visualoverview of the types of adjusting entries.

Make adjusting entries.

The following two-step process will facilitate preparing the adjustingjournal entries:

1. Determine whether a revenue account needs to be recorded (cred-ited) or an expense account needs to be recorded (debited).

2.The other account in the entry MUST be either an asset or a liabilityaccount.

Be careful: Cash will never be included in an adjusting journal entry.

You will learn how to prepare adjusting entries in Demo Doc 1. ReviewExhibits 3-5 through 3-8 (pp. 138–141) carefully for a review of theadjusting journal entry process.

Prepare an adjusted trial balance.

After the adjusting journal entries are journalized, they are posted andan adjusted trial balance is prepared. The list of updated accountbalances is prepared to determine that debits equal credits beforepreparing the financial statements. Exhibit 3-8 (p. 141) shows the flow ofinformation from the trial balance to the adjusted trial balance on theworksheet.

Prepare financial statements from an adjusted trial balance.

Follow the flow of data from the adjusted trial balance in Exhibit 3-8 (p. 141) to the financial statements in Exhibit 3-9 through 3-11 (p. 142).

5

1eSG_CO3_0131792075.QXD 10/19/06 1:16 PM Page 72

Page 105: polpoa_sg

Demo Doc 1 | Chapter 3 73

Adjusting Entries for Accrual AccountingLearning Objectives 1–5

Wood’s Restaurant’s December 31 (year-end) trial balance (before adjustments) isas follows:

WOOD’S RESTAURANTTrial Balance

Year Ended December 31, 2008

Cash $ 10,600 Accounts receivable 14,000 Supplies 1,200 Prepaid rent 3,000 Furniture 15,000 Accumulated depreciation, furniture $ 4,500 Accounts payable 2,600 Salary payable 0 Daniel Wood, capital 40,000 Daniel Wood, withdrawals 11,500 Service revenue 24,000 Salary expense 10,000 Rent expense 5,000 Supplies expense 800 Depreciation expense 0 Total $ 71,100 $ 71,100

Account Title Debit Credit Balance

Demo Doc 1

Requirements

1. Open the T-accounts and enter their unadjusted balances.2. Journalize the following adjusting entries at December 31, 2008. Key the

entries by letter.

a. Employees are paid $200 every Friday for the previous five days ofwork. December 31, 2008, is a Wednesday.

b. Depreciation on the furniture is $1,500 for the year.

c. Supplies on hand at December 31, 2008, are $400.

d. Six months of rent ($3,000) was paid in advance on November 1, 2008.No adjustment has been made to the Prepaid Rent account since then.

e. Accrued revenue totaling $1,800 must be recorded.

3. Post the adjusting entries.

4. Prepare the trial balance on the worksheet, enter the adjusting entries, andprepare an adjusted trial balance.

5. Prepare the income statement, statement of owner’s equity, and balancesheet for Wood’s Restaurant.

1eSG_CO3_0131792075.QXD 10/19/06 1:16 PM Page 73

Page 106: polpoa_sg

74 Chapter 3 | Demo Doc 1 Solutions

Describe the four typesof adjusting entries.

Part 1 Part 2 Part 3 Part 4 Part 5 Part 6 Part 7Demo DocComplete

ASSETS LIABILITIES REVENUES

Cash Accounts Payable Service Revenue

Bal. 10,600 Bal. 2,600 Bal. 24,000

Accounts Receivable Salary Payable

Bal. 14,000 EXPENSES

Rent Expense

Supplies OWNER’S EQUITY Bal. 5,000

Bal. 1,200

Daniel Wood, Capital Salary Expense

Prepaid Rent Bal. 40,000 Bal. 10,000

Bal. 3,000Daniel Wood, Depreciation Expense

Furniture Withdrawals

Bal. 15,000 Bal. 11,500

Supplies Expense

Accumulated Bal. 800Depreciation, Furniture

Bal. 4,500

Part 1 Part 2 Part 3 Part 4 Part 5 Part 6 Part 7Demo DocComplete

Demo Doc 1 Solutions

Requirement 1

Open the T-accounts and enter their unadjusted balances.

Requirement 2

Journalize the following adjusting entries at December 31, 2008. Key theentries by letter.

a. Employees are paid $200 every Friday for the previous five days ofwork. December 31, 2008, is a Wednesday.

If employees are paid $200 for five days of work, then they are paid $200/5 =$40 per day. By the end of the day on Wednesday, December 31, the employ-ees have worked for three days and have not been paid. Therefore, Woodowes employees $40 ! 3 = $120 of salary at December 31.If the salaries have not been paid, then they are payable (or in other words,they are owed) and must be recorded as some kind of payable account. Wemight consider using Accounts Payable, but this account is usually reserved

2

Make adjustingentries.

3

1eSG_CO3_0131792075.QXD 10/19/06 1:16 PM Page 74

Page 107: polpoa_sg

Demo Doc 1 Solutions | Chapter 3 75

Journal Entry:

Dec. 31 Salary Expense (3 ! $200/5) (Expense,1; debit) 120 Salary Payable (Liability,1; credit) 120 To accrue salary expense.

PostDate Accounts and Explanation Ref. Dr. Cr.

Journal Entry:

Dec. 31 Depreciation Expense, Furniture (Expense1; debit) 1,500 Accumulated Depreciation, Furniture (Contra-Asset,1; credit) To record depreciation on building.

PostDate Accounts and Explanation Ref. Dr.

for bills received. The employees do not send Wood a bill, they simply expectto be paid and Wood knows that the salaries are owed. So we put thisamount owed into another payable account. In this case, Salary Payable ismost appropriate.Because salary is not owed until work is performed, we know that Wood’semployees have already worked. This past benefit is recorded as an expense(in this case, Salary Expense).The result is an increase to Salary Expense (a debit) and an increase to theliability Salary Payable (a credit) of $120.

Make adjustingentries.

3

Describe the four typesof adjusting entries.

2

Make adjustingentries.

3

b. Depreciation on the furniture is $1,500 for the year.

The entry to record depreciation expense is always the same. It is only thenumber (dollar amount) in the entry that changes. This adjustment alwaysincreases Depreciation Expense (a debit) and increases the contra-assetaccount of Accumulated Depreciation (a credit). Because we are given thedepreciation expense of $1,500, we simply write the entry with that amount.

c. Supplies on hand at December 31, 2008, are $400.

Before adjustments, the Supplies account holds $1,200. If only $400 ofsupplies remains, then the other $800 must have been used:

$1,200 ! $400 " $800

Supplies are an asset, a future benefit to Wood. Once the supplies are used,they are a past benefit. When they are no longer assets, the Supplies assetmust be decreased by $800 (a credit). Past benefits are expenses, so SuppliesExpense is increased (a debit).

1eSG_CO3_0131792075.QXD 10/19/06 1:16 PM Page 75

Page 108: polpoa_sg

76 Chapter 3 | Demo Doc 1 Solutions

d. Six months of rent ($3,000) was paid in advance on November 1, 2008.No adjustment has been made to the Prepaid Rent account since.

Wood prepaid $3,000 for six months of rent on November 1, which meansthat Wood pays $3,000/6 = $500 a month for rent. At December 31, twomonths have passed since the prepayment, so two months of the prepaymenthave been used. The amount of rent used is:

2 # $500 " $1,000

When something is prepaid, it is a future benefit (an asset) because the busi-ness is now entitled to receive goods or services. Once those goods or servicesare received (in this case, once Wood has occupied the building being rented),it becomes a past benefit, and therefore an expense. So Rent Expense mustbe increased (a debit) and the Prepaid Rent (an asset) must be decreased (acredit).

Journal Entry:

Dec. 31 Rent Expense (2 ! $3,000/6) (Expense,1; debit) 1,000 Prepaid Rent (Asset,2; credit) 1,000 To record rent expense.

PostDate Accounts and Explanation Ref. Dr. Cr.

Journal Entry:

Dec. 31 Supplies Expense ($1,200 – $400) (Expense,1; debit) 800 Supplies (Asset,2; credit) 800 To record supplies expense.

PostDate Accounts and Explanation Ref. Dr. Cr.

Describe the four typesof adjusting entries.

2

Make adjustingentries.

3

Describe the account-ing principles that helpbusinesses measureincome.

1

Describe the four typesof adjusting entries.

2

Make adjustingentries.

3

e. Accrued revenue totaling $1,800 must be recorded.

Accrued revenue is another way of saying “accounts receivable” (or receipt ofpayment in the future). If accrued revenue is recorded, it means thataccounts receivable is also recorded (that is, customers received goods or ser-vices from the business, but the business has not yet received the cash). Thebusiness is entitled to these receivables because the revenue has beenearned.Note that not all revenue is accrued revenue. Only the revenue that isearned but not immediately received from the customer (that is, theaccounts receivable) is referred to as accrued. Revenues that are earnedand received immediately in cash are not accrued revenues.Service Revenue must be increased by $1,800 (a credit) and the AccountsReceivable asset account must be increased by $1,800 (a debit).

1eSG_CO3_0131792075.QXD 10/19/06 1:16 PM Page 76

Page 109: polpoa_sg

Demo Doc 1 Solutions | Chapter 3 77

Journal Entry:

Dec. 31 Accounts Receivable (Asset,1; debit) 1,800 Service Revenue (Revenue,1; credit) 1,800 To accrue service revenue.

PostDate Accounts and Explanation Ref. Dr. Cr.

Part 1 Part 2 Part 3 Part 4 Part 5 Part 6 Part 7Demo DocComplete

Requirement 3

Post the adjusting entries.

ASSETS LIABILITIES REVENUES

Cash Accounts Payable Service Revenue

Bal. 10,600 Bal. 2,600 24,000e. 1,800

Bal. 25,800

Accounts Receivable Salary Payable

14,000 a. 120 EXPENSES

e. 1,800 Bal. 120

Bal. 15,800 Rent ExpenseOWNER’S EQUITY 5,000

Supplies 1,000

1,200 Bal. 6,000c. 800 Daniel Wood, Capital

Bal. 400 Bal. 40,000 Salary Expense

10,000Prepaid Rent a. 120

3,000 Daniel Wood, Bal. 10,120d. 1,000 Withdrawals

Bal. 2,000 Bal. 11,500 Depreciation Expense

b. 1,500

Furniture Bal. 1,500

Bal. 15,000

Supplies Expense

Accumulated 800Depreciation, Furniture c. 800

4,500 Bal. 1,600b. 1,500Bal. 6,000

1eSG_CO3_0131792075.QXD 10/19/06 1:16 PM Page 77

Page 110: polpoa_sg

78 Chapter 3 | Demo Doc 1 Solutions

Part 1 Part 2 Part 3 Part 4 Part 5 Part 6 Part 7Demo DocComplete

Cash $10,600 Accounts receivable 14,000 Supplies 1,200

Prepaid rent 3,000 Furniture 15,000 Accumulated depreciation, furniture $ 4,500 Accounts payable 2,600 Salary payable 0 Daniel Wood, capital 40,000 Daniel Wood, withdrawals 11,500 Service revenue 24,000 Salary expense 10,000

Rent expense 5,000 Supplies expense 800 Depreciation expense 0

Total $71,100 $71,100

Account Title Dr. Cr. Dr. Cr. Dr. Cr.

Trial Balance Adjustments

WOOD’S RESTAURANTPreparation of Adjusted Trial Balance

Year Ended December 31, 2008

AdjustedTrial Balance

$10,60015,800

400e. $1,800

a. 120

c. $8002,000

15,000

11,500

10,120d. 1,000 6,000c. 800 1,600b. 1,500 1,500

$5,220 $5,220 $74,520 $74,520

$6,0002,600

12040,000

25,800

d. 1,000

b. 1,500

a. 120

e. 1,800

Part 1 Part 2 Part 3 Part 4 Part 5 Part 6 Part 7Demo DocComplete

Requirement 4

Prepare the trial balance on the worksheet, enter the adjusting entries, andprepare an adjusted trial balance.

Prepare an adjustedtrial balance.

4

Requirement 5

Prepare the income statement, statement of owner’s equity, and balance sheetfor Wood’s Restaurant.

Prepare financial state-ments from an adjustedtrial balance.

5

1eSG_CO3_0131792075.QXD 10/19/06 1:16 PM Page 78

Page 111: polpoa_sg

Demo Doc 1 Solutions | Chapter 3 79

Revenue: Service revenue $ 25,800 Expenses: Salary expense $ 10,120 Rent expense 6,000 Supplies expense 1,600 Depreciation expense 1,500 Total expenses 19,220 Net income $ 6,580

WOOD’S RESTAURANTIncome Statement

Year Ended December 31, 2008

Remember, the one account that has not yet been updated is Daniel Wood,Capital. The amount of $40,000 in this account is the amount from the beginningof the year (January 1). To update the account, we need to prepare the statementof owner’s equity.

Daniel Wood, capital, January 1, 2008 $40,000 Add: Net income for month 6,580 46,580 Less: Withdrawals by owner (11,500) Daniel Wood, capital, December 31, 2008 $35,080

WOOD’S RESTAURANTStatement of Owner’s Equity Year Ended December 31, 2008

Part 1 Part 2 Part 3 Part 4 Part 5 Part 6 Part 7Demo DocComplete

We use this updated Daniel Wood, capital amount on the balance sheet.

Part 1 Part 2 Part 3 Part 4 Part 5 Part 6 Part 7Demo DocComplete

1eSG_CO3_0131792075.QXD 10/19/06 1:16 PM Page 79

Page 112: polpoa_sg

80 Chapter 3 | Demo Doc 1 Solutions

WOOD’S RESTAURANTBalance Sheet

December 31, 2008

Assets Liabilities Cash $10,600 Accounts payable $ 2,600 Accounts receivable 15,800 Salary payable 120 Supplies 800 Total liabilities $ 2,720 Prepaid rent 2,000 Furniture $15,000 Owners’ Equity Less: Accumulated Daniel Wood, capital 35,080 depreciation (6,000) 9,000 Total assets $37,800 Total liabilities and owner’s equity $37,800

Part 1 Part 2 Part 3 Part 4 Part 5 Part 6 Part 7Demo DocComplete

1eSG_CO3_0131792075.QXD 10/19/06 1:16 PM Page 80

Page 113: polpoa_sg

Quick Practice QuestionsTrue/False

1. Revenue is recorded when it is earned; usually when a good or service has been delivered to the customer.

2. The time period concept provides for periodic reporting at regular intervals.

3. An accounting year that ends on a date other than December 31 is called an interim year.

4. The revenue principle requires that a cash deposit for future construction be recorded as revenue.

5. Adjusting journal entries are made at the end of the period.

6. The income statement is the first financial statement that is prepared.

7. Every adjusting journal entry affects one income statement account and one balance sheet account.

8. An accrual is an expense that is recorded after it is paid.

9. Accumulated Depreciation is a liability account.

10. Unearned Service Revenue appears on the income statement.

Quick Practice Questions | Chapter 3 81

1eSG_CO3_0131792075.QXD 10/19/06 1:16 PM Page 81

Page 114: polpoa_sg

82 Chapter 3 | Quick Practice Questions

Multiple Choice

1. What items should be matched according to the matchingprinciple?a. Debits with creditsb. Assets with liabilitiesc. Expenses with revenuesd. Accruals with prepaids

2. What is it called when cash is received before revenue isrecorded?a. A deferralb. An accrualc. Revenue recognitiond. A prepaid expense

3. What do adjusting entries properly measure?a. Net income for the periodb. The assets, liabilities, and owner’s equity on the balance sheetc. Both a and bd. Neither a nor b

4. Which of the following entities would most likely have anUnearned Revenue account?a. A local pizza storeb. An accounting firmc. A department stored. A magazine publisher

5. Georgia Industries paid $48,000 for two years of insurancecoverage on July 1, 2007. The company prepares financialstatements on July 31, 2007. What is the amount of insuranceexpense on July 31?a. $48,000b. $ 2,000c. $24,000d. $46,000

6. Using the information from question 5, what is the adjustedbalance in Prepaid Insurance on December 31, 2007?a. $36,000b. $24,000c. $12,000d. $38,000

1eSG_CO3_0131792075.QXD 10/19/06 1:16 PM Page 82

Page 115: polpoa_sg

Quick Practice Questions | Chapter 3 83

7. Sports Illustrated receives $120,000 on September 1, 2008, forone year’s worth of magazine subscriptions for the yearbeginning September 1, 2008. What is the journal entry to recordthe prepaid subscriptions?

Journal Entry:

a. Accounts Receivable 120,000 Unearned Subscription Revenue 120,000

b. Cash 120,000 Subscription Revenue 120,000 c. Cash 120,000 Unearned Subscription Revenue 120,000 d. Accounts Receivable 120,000 Subscription Revenue 120,000

PostDate Accounts Ref. Dr. Cr.

8. Which of the following accounts is depreciated?a. Buildingb. Landc. Suppliesd. Prepaid Insurance

9. What is book value?a. The sum of all the depreciation recorded for the assetb. The cost of the depreciable assetc. The cost of the depreciable asset divided by the useful lifed. The cost of the depreciable asset minus accumulated depreciation

10. Mason Company has a weekly payroll of $5,000. Wages are paidevery Friday for the work performed Monday through Friday ofthat week. Assuming that the accounting period ends on aTuesday, what amount of Wages Expense should be recorded onthat date?a. $1,000b. $2,000c. $3,000d. $4,000

1eSG_CO3_0131792075.QXD 10/19/06 1:16 PM Page 83

Page 116: polpoa_sg

Quick Exercises

3-1. Using the following information, post the required adjust-ments into the worksheet and extend all the account balancesinto the adjusted trial balance.

a. Supplies of $400 were used up this period.b. Prepaid insurance expired, $500.c. Accrued service revenue, $1,200.d. Depreciation on machinery, $650.e. Service revenue collected in advance now earned, $750.

84 Chapter 3 | Quick Practice Questions

Cash $29,965 Accounts receivable 3,260 Supplies 2,475 Prepaid insurance 2,500 Equipment 13,500 Accumulated depreciation, equipment $10,250 Accounts payable 4,500 Unearned service revenue 950 Joan Reid, capital 18,200 Joan Reid, withdrawals 2,700 Service revenue 29,000 Insurance expense 3,250 Depreciation expense, equipment 4,550 Supplies expense 700 Total $62,900 $62,900

Account Dr. Cr. Dr. Cr. Dr. Cr.

Trial Balance Adjustments

AdjustedTrial Balance

1eSG_CO3_0131792075.QXD 10/19/06 1:16 PM Page 84

Page 117: polpoa_sg

3-2. For each of the following situations, indicate whether anexpense or revenue should be recorded and the amount of theadjustment on January 31, 2008.

Revenue Adjustmentor Expense Amount

a. Purchased $1,500 of supplies during January. On January 31,$800 of supplies remains. ––––––––– ___________

b. The five-day weekly payroll is $6,000. Employees worked the last two days of January and have not been paid by January 31. ––––––––– ––––––––––

c. Earned $750 of Unearned Revenue in January. ––––––––– ––––––––––

d. Depreciation on equipment is $3,600for the year. ––––––––– ––––––––––

e. Services of $2,300 were performed on January 31 and have not been recorded. ––––––––– ––––––––––

3-3. Journalize the required adjusting journal entries using theinformation in Quick Exercise 3-2.

a.

Quick Practice Questions | Chapter 3 85

Journal Entry:

PostDate Accounts Ref. Dr. Cr.

Journal Entry:

PostDate Accounts Ref. Dr. Cr.

b.

1eSG_CO3_0131792075.QXD 10/19/06 1:16 PM Page 85

Page 118: polpoa_sg

3-4. Following is the trial balance for Coleman Copy Center:

e.

COLEMAN COPY CENTERTrial BalanceMarch 31, 2008

Cash $30,000 Accounts receivable 2,000 Supplies 600

Land 50,000 Accounts payable $ 2,600

Notes payable 35,000 Jo Coleman, capital 42,550 Jo Coleman, withdrawals 2,400 Service revenue 9,300 Salary expense 2,500 Rent expense 1,200

Interest expense 500 Utilities expense 250

Total $89,450 $89,450

Account Title Debit Credit Balance

Journal Entry:

PostDate Accounts Ref. Dr. Cr.

Journal Entry:

PostDate Accounts Ref. Dr. Cr.

d.

Journal Entry:

PostDate Accounts Ref. Dr. Cr.

c.

1eSG_CO3_0131792075.QXD 10/19/06 1:16 PM Page 86

Page 119: polpoa_sg

Quick Practice Questions | Chapter 3 87

3-5. Using the trial balance for Coleman Copy Center in QuickExercise 3-4, prepare the balance sheet at March 31, 2008.

COLEMAN COPY CENTERStatement of Owner’s Equity Month Ending March 31, 2008

COLEMAN COPY CENTERBalance Sheet March 31, 2008

COLEMAN COPY CENTERIncome Statement

Month Ending March 31, 2008

Prepare (a) an income statement and (b) a statement ofowner’s equity for the month ending March 31, 2008.

1eSG_CO3_0131792075.QXD 10/19/06 1:16 PM Page 87

Page 120: polpoa_sg

Do It Yourself! Question 1

Angela’s Business Services has the following balances on its December 31 (year-end) trial balance (before adjustments):

ANGELA’S BUSINESS SERVICESTrial Balance

December 31, 2008

Cash $40,400 Prepaid insurance 4,800 Supplies 13,000 Office equipment 25,000 Accumulated depreciation, equipment $7,500

Accounts Payable 5,300 Salary Payable 0 Unearned service revenue 6,800 Angela Waring, capital 60,000 Angela Waring, withdrawals 8,000 Service revenue 80,000 Salary expense 45,000

Insurance expense 13,200 Supplies expense 10,200 Depreciation expense 0 Total $159,600 $159,600

Account Title Debit Credit Balance

Journal Entry:

PostDate Accounts Ref. Dr. Cr.

Journal Entry:

PostDate Accounts Ref. Dr. Cr.

Requirements:

1. Journalize the following adjusting entries at December 31, 2008. Key theentries by letter.

a. Only $1,500 of the unearned service revenue remains unearned.

b. Depreciation on the office equipment is $2,500 for the year.

1eSG_CO3_0131792075.QXD 10/19/06 1:16 PM Page 88

Page 121: polpoa_sg

Do It Yourself! Question 1 | Chapter 3 89

c. Employees earned salaries of $4,000 that have not been paid.

Journal Entry:

PostDate Accounts Ref. Dr. Cr.

Journal Entry:

PostDate Accounts Ref. Dr. Cr.

Journal Entry:

PostDate Accounts Ref. Dr. Cr.

d. Supplies worth $5,100 have been used.

e. Four months of insurance ($4,800) was paid in advance onDecember 1, 2008. No adjustment has been made to the prepaidinsurance account since.

1eSG_CO3_0131792075.QXD 10/19/06 1:16 PM Page 89

Page 122: polpoa_sg

ASSETS LIABILITIES REVENUES

Cash Accounts Payable Service Revenue

Prepaid Insurance Salary Payable

EXPENSESInsurance Expense

Unearned ServiceRevenue

Supplies

Salary Expense

Office Equipment OWNER’S EQUITY

Angela Warning, Capital Depreciation Expense

AcumulatedDepreciation, Equipment

Angela Warning, Supplies ExpenseWithdrawals

3. Prepare the trial balance on the worksheet, enter the adjusting entries, andprepare an adjusted trial balance.

Cash Prepaid insurance Supplies

Office equipment Accumulated depreciation, equipment Accounts payable Salary payable Unearned service revenue Angela Waring, capital Angela Waring, withdrawals Service revenue Salary expense

Insurance Supplies expense Depreciation expense

Total

Account Title Dr. Cr. Dr. Cr. Dr. Cr.

Trial Balance Adjustments

AdjustedTrial Balance

2. Open T-accounts, enter the unadjusted balances in the accounts, and postthe adjusting entries.

1eSG_CO3_0131792075.QXD 10/19/06 1:16 PM Page 90

Page 123: polpoa_sg

Do It Yourself! Question 1 | Chapter 3 91

Angela Waring, capital, January 1, 2008

Add:

Less: Angela Waring, capital, December 31, 2008

ANGELA’S BUSINESS SERVICESStatement of Owner’s Equity Year Ended December 31, 2008

Assets Liabilities Owner’s Equity Total assets Total liabilities and owner’s equity

ANGELA’S BUSINESS SERVICESBalance Sheet

December 31, 2008

Revenue: Expenses: Net income

ANGELA’S BUSINESS SERVICESIncome Statement

Year Ended December 31, 2008

4. Prepare the income statement, statement of owner’s equity, and balancesheet for Angela’s Business Services.

1eSG_CO3_0131792075.QXD 10/19/06 1:16 PM Page 91

Page 124: polpoa_sg

92 Chapter 3 | Do It Yourself! Question 2

Journal Entry:

PostDate Accounts Ref. Dr. Cr.

Journal Entry:

PostDate Accounts Ref. Dr. Cr.

Journal Entry:

PostDate Accounts Ref. Dr. Cr.

Do It Yourself! Question 2

Everly Industries is preparing its financial statements for the year endedDecember 31, 2009. Three accounting issues have been discovered.

Requirement

1. Make the necessary adjusting entry for each situation.

a. Employees work five days a week (Monday through Friday) and arepaid $7,500 for the previous week of work each Friday. December31, 2009, falls on a Thursday.

b. The Supplies account shows an unadjusted balance of $1,000.However, only $350 of supplies remains on hand at December 31,2009.

c. The company forgot to record four months of interest expense ($80per month) that has been incurred but not yet paid.

1eSG_CO3_0131792075.QXD 10/19/06 1:16 PM Page 92

Page 125: polpoa_sg

Quick Practice Solutions | Chapter 3 93

Quick Practice SolutionsTrue/False

T 1. Revenue is recorded when it is earned; usually when a good or service has been delivered to the customer. (p. 126)

T 2. The time period concept provides for periodic reporting at regular intervals. (p. 126)

F 3. An accounting year that ends on a date other than December 31 is called an interim year.False–An accounting year that ends on a date other than December31 is called a fiscal year. (p. 127)

F 4. The revenue principle requires that a cash deposit for future construction be recorded as revenue.False–The revenue principle requires that revenue be recorded when it has been earned. The cash received for future construction has not been earned yet. (p. 127)

T 5. Adjusting journal entries are made at the end of the period.(pp. 127–128)

T 6. The income statement is the first financial statement that is prepared. (p. 141)

T 7. Every adjusting journal entry affects one income statement accountand one balance sheet account. (p. 138)

F 8. An accrual is an expense that is recorded after it is paid.False–An accrual is an expense that is recorded before it is paid.(pp. 126–129)

F 9. Accumulated depreciation is a liability account.False–Accumulated depreciation is a contra-asset account. (p. 134)

F 10. Unearned Service Revenue appears on the income statement.False–Unearned Service Revenue is a liability and appears on the balance sheet. (pp. 141–143)

1eSG_CO3_0131792075.QXD 10/19/06 1:16 PM Page 93

Page 126: polpoa_sg

94 Chapter 3 | Quick Practice Solutions

Multiple-Choice

1. What items should be matched according to the matchingprinciple? (p. 126)a. Debits with creditsb. Assets with liabilitiesc. Expenses with revenuesd. Accruals with prepaids

2. What is it called when cash is received before revenue isrecorded? (p. 128)a. A deferralb. An accrualc. Revenue recognitiond. A prepaid expense

3. What do adjusting entries properly measure? (pp. 127–128)a. Net income for the periodb. The assets, liabilities, and owner’s equity on the balance sheetc. Both a and bd. Neither a nor b

4. Which of the following entities would most likely have anUnearned Revenue account? (pp. 134–135)a. A local pizza storeb. An accounting firmc. A department stored. A magazine publisher

5. Georgia Industries paid $48,000 for two years of insurancecoverage on July 1, 2007. The company prepares financialstatements on July 31, 2007. What is the amount of insuranceexpense on July 31? (pp. 130–131)a. $48,000b. $ 2,000c. $24,000d. $46,000

6. Using the information from question 5, what is the adjustedbalance in Prepaid Insurance on December 31, 2007? (pp. 130–131)a. $36,000b. $24,000c. $12,000d. $38,000

1eSG_CO3_0131792075.QXD 3/5/08 2:13 PM Page 94

Page 127: polpoa_sg

8. Which of the following accounts is depreciated? (pp. 132–134)a. Buildingb. Landc. Suppliesd. Prepaid Insurance

9. What is book value? (p. 134)a. The sum of all the depreciation recorded for the assetb. The cost of the depreciable assetc. The cost of the depreciable asset divided by the useful lifed. The cost of the depreciable asset minus accumulated depreciation

10. Mason Company has a weekly payroll of $5,000. Wages are paidevery Friday for the work performed Monday through Friday ofthat week. Assuming that the accounting period ends on aTuesday, what amount of Wages Expense should be recorded onthat date? (pp. 136–137)a. $1,000b. $2,000c. $3,000d. $4,000

Quick Practice Solutions | Chapter 3 95

Journal Entry:

a. Accounts Receivable 120,000 Unearned Subscription Revenue 120,000

b. Cash 120,000 Subscription Revenue 120,000 c. Cash 120,000 Unearned Subscription Revenue 120,000 d. Accounts Receivable 120,000 Subscription Revenue 120,000

PostDate Accounts Ref. Dr. Cr.

7. Sports Illustrated receives $120,000 on September 1, 2008, forone year’s worth of magazine subscriptions for the yearbeginning September 1, 2008. What is the journal entry to recordthe prepaid subscriptions? (pp. 130–131)

1eSG_CO3_0131792075.QXD 10/19/06 1:16 PM Page 95

Page 128: polpoa_sg

Quick Exercise

3-1. Using the following information, post the required adjust-ments into the worksheet and extend all of the account bal-ances into the adjusted trial balance. (p. 140)

a. Supplies of $400 were used up this period.b. Prepaid insurance expired, $500.c. Accrued service revenue, $1,200.d. Depreciation on machinery, $650.e. Service revenue collected in advance now earned, $750.

96 Chapter 3 | Quick Practice Solutions

Cash $29,965 Accounts receivable 3,260 Supplies 2,475

Prepaid insurance 2,500 Equipment 13,500 Accumulated depreciation, equipment 10,250 Accounts payable 4,500 Unearned service revenue 950 Joan Reid, capital 18,200 Joan Reid, withdrawals 2,700 Service revenue 29,000

Insurance expense 3,250 Depreciation expense, equipment 4,550 Supplies expense 700

Total $62,900 $62,900

Account Title Dr. Cr. Dr. Cr. Dr. Cr.

Trial Balance Adjustments

AdjustedTrial Balance

$29,9654,4602,075

c. $1,200

e. 750

a. $4002,000

13,500

2,700

b. 500 3,750d. 650 5,200a. 400 1,100

$3,500 $3,500 $64,750 $64,750

$10,9004,500

20018,200

30,950

b. 500

d. 650

c. 1,200e. 750

1eSG_CO3_0131792075.QXD 10/20/06 12:21 PM Page 96

Page 129: polpoa_sg

3-2. For each of the following situations, indicate whether anexpense or revenue should be recorded and the amount of theadjustment on January 31, 2008. (p. 138)

Revenue Adjustmentor Expense Amount

a. Purchased $1,500 of supplies during January. On January 31,$800 of supplies remains. Expense $ 700

b. The five-day weekly payroll is $6,000. Employees worked the last two days of January and have not been paid by January 31. Expense $2,400

c. Earned $750 of Unearned Revenue in January. Revenue $ 750

d. Depreciation on equipment is $3,600 for the year. Expense $ 300

e. Services of $2,300 were performed on January 31 and have not been recorded. Revenue $2,300

3-3. Journalize the required adjusting journal entries using theinformation in Quick Exercise 3-2. (p. 138)

a.

Journal Entry:

Jan. 31 Supplies Expense 700 Supplies 700

PostDate Accounts Ref. Dr. Cr.

Journal Entry:

Jan. 31 Salary Expense 2,400 Salary Payable 2,400

PostDate Accounts Ref. Dr. Cr.

Journal Entry:

Jan. 31 Unearned Revenue 750 Service Revenue 750

PostDate Accounts Ref. Dr. Cr.

b.

c.

1eSG_CO3_0131792075.QXD 10/19/06 1:16 PM Page 97

Page 130: polpoa_sg

e.

98 Chapter 3 | Quick Practice Solutions

COLEMAN COPY CENTERTrial BalanceMarch 31, 2008

Cash $30,000 Accounts receivable 2,000 Supplies 600 Land 50,000 Accounts payable $2,600

Notes payable 35,000 Jo Coleman, capital 42,550 Jo Coleman, withdrawals 2,400 Service revenue 9,300 Salary expense 2,500 Rent expense 1,200 Interest expense 500

Utilities expense 250 Total $89,450 $89,450

Account Title Debit Credit Balance

Journal Entry:

Jan. 31 Accounts Receivable 2,300 Service Revenue 2,300

PostDate Accounts Ref. Dr. Cr.

3-4. Following is the trial balance for Coleman Copy Center:

Journal Entry:

Jan. 31 Depreciation Expense, Equipment 300 Accumulated Depreciation, Equipment 300

PostDate Accounts Ref. Dr. Cr.

d.

1eSG_CO3_0131792075.QXD 10/19/06 1:16 PM Page 98

Page 131: polpoa_sg

b.

Quick Practice Solutions | Chapter 3 99

Jo Coleman, capital, May 1, 2008 $ 42,550 Add: Net income 4,850 $ 47,400 Less: Withdrawals 2,400 Jo Coleman, capital, May 1, 2008 $ 45,000

COLEMAN COPY CENTERStatement of Owner’s Equity Month Ending March 31, 2008

COLEMAN COPY CENTERBalance Sheet March 31, 2008

Assets Liabilities Cash $ 30,000 Accounts payable $ 2,600 Accounts receivable 2,000 Salary payable 35,000 Supplies 600 Total liabilities $ 37,600 Land 50,000 Owner’s Equity Jo Coleman, capital 45,000 Total liabilities and Total assets $82,600 owner’s equity $82,600

3-5. Using the trial balance for Coleman Copy Center in QuickExercise 3-4, prepare the balance sheet at March 31, 2008.(pp. 141–143)

Prepare (a) an income statement and (b) a statement of owner’sequity for the month ending March 31, 2008. (pp. 141–143)

a.

Revenue: Service revenue $9,300 Expenses: Salary expense 2,500 Rent expense 1,200 Interest expense 500 Utilities expense 250 Total expenses 4,450 Net income $4,850

COLEMAN COPY CENTERIncome Statement

Month Ending March 31, 2008

1eSG_CO3_0131792075.QXD 10/19/06 1:16 PM Page 99

Page 132: polpoa_sg

100 Chapter 3 | Do It Yourself! Question 1 Solutions

Journal Entry:

Dec. 31 Depreciation Expense, Equipment (Expense1; debit) 1,500 Accumulated Depreciation, Equipment (Contra-Asset,1; credit) 1,500 To record depreciation on equipment.

PostDate Accounts and Explanation Ref. Dr. Cr.

Journal Entry:

Dec. 31 Salary Expense (Expense,1; debit) 4,000 Salary Payable (Liability,1; credit) 4,000 To accrue salary expense.

PostDate Accounts and Explanation Ref. Dr. Cr.

Journal Entry:

Dec. 31 Supplies Expense (Expense,1; debit) 5,100 Supplies (Asset,2; credit) 5,100 To accrue supplies expense.

PostDate Accounts and Explanation Ref. Dr. Cr.

Journal Entry:

Dec. 31 Unearned Service Revenue ($6,800 – $1,500) (Liability,2; debit) 5,300 Service Revenue (Revenue,1; credit) 5,300 To record service revenue collected in advance that is now earned.

PostDate Accounts and Explanation Ref. Dr. Cr.

b. Depreciation on the office equipment is $2,500 for the year.

c. Employees earned salaries of $4,000 that have not been paid.

d. Supplies worth $5,100 have been used.

Do It Yourself! Question 1 Solutions

Requirement 1

Journalize the following adjusting entries at December 31, 2008. Key theentries by letter.

a. Only $1,500 of the unearned service revenue remains unearned.

1eSG_CO3_0131792075.QXD 10/19/06 1:16 PM Page 100

Page 133: polpoa_sg

e. Four months of insurance ($4,800) was paid in advance on Dec-ember 1, 2008. No adjustment has been made to the prepaid insur-ance account since.

Do It Yourself! Question 1 Solutions | Chapter 3 101

Journal Entry:

Dec. 31 Insurance Expense (1 3 $4,800/4) (Expense,1; debit) 1,200 Prepaid Insurance (Asset,2; credit) 1,200 To record insurance expense.

PostDate Accounts and Explanation Ref. Dr. Cr.

Requirement 2

Open T-accounts, enter the unadjusted balances in the account, and post theadjusting entries.

ASSETS LIABILITIES REVENUES

Cash Accounts Payable Service Revenue

Bal. 40,400 Bal. 5,300 80,000a. 5,300

Bal. 85,300

Prepaid Insurance Salary Payable

4,800 c. 4,000

e. 1,200 Bal. 4,000 EXPENSES

Bal. 3,600 Insurance ExpenseUnearned Service 13,200

Revenue e. 1,200Supplies 6,800 Bal. 14,400

13,000 a. 5,300d. 5,100 Bal. 1,500

Bal. 7,900 Salary Expense

45,000Office Equipment OWNER’S EQUITY c. 4,000

Bal. 25,000 Bal. 49,000Angela Waring,

Capital Depreciation Expense

Accumulated Bal. 60,000 b. 2,500Depreciation, Equipment Bal. 2,500

7,500 Angela Waring,b. 2,500 Withdrawals Supplies ExpenseBal. 10,000 Bal. 8,000 10,200

d. 5,100

Bal. 15,300

1eSG_CO3_0131792075.QXD 10/19/06 1:16 PM Page 101

Page 134: polpoa_sg

102 Chapter 3 | Do It Yourself! Question 1 Solutions

Cash $ 40,400 Prepaid insurance 4,800 Supplies 13,000

Office equipment 25,000 Accumulated depreciation, equipment $ 7,500 Accounts payable 5,300 Salary payable 0 Unearned service revenue 6,800 Angela Waring, capital 60,000 Angela Waring, withdrawals 8,000 Service revenue 80,000 Salary expense 45,000

Insurance expense 13,200 Supplies expense 10,200 Depreciation expense 0

Total $159,600 $159,600

Account Title Dr. Cr. Dr. Cr. Dr. Cr.

Trial Balance Adjustments

ANGELA’S BUSINESS SERVICESPreparation of Adjusted Trial Balance

Year Ended December 31, 2008

AdjustedTrial Balance

$ 40,400

$ 10,0005,3004,0001,500

60,000

85,300

$166,100

3,6007,900

c. 4,000

a.$ 5,300

d. 5,100e. $1,200

25,000

8,000

49,000e. 1,200 14,400d. 5,100 15,300b. 2,500 2,500$18,100 $18,100 $166,100

b. 2,500

c. 4,000

a. 5,300

Requirement 3

Prepare the trial balance on the worksheet, enter the adjusting entries, andprepare an adjusted trial balance.

Requirement 4

Prepare the income statement, statement of owner’s equity, and balance sheetfor Angela’s Business Service.

Revenue: Service revenue $85,300 Expenses: Salary expense $49,000 Supplies expense 15,300 Insurance expense 14,400 Depreciation expense 2,500 Total expenses 81,200 Net income $ 4,100

ANGELA’S BUSINESS SERVICESIncome Statement

Year Ended December 31, 2008

1eSG_CO3_0131792075.QXD 10/19/06 1:16 PM Page 102

Page 135: polpoa_sg

Do It Yourself! Question 1 Solutions | Chapter 3 103

Angela Waring, capital, January 1, 2008 $ 60,000 Add: Net income for month 4,100 $ 64,100 Less: Withdrawals by owner (8,000) Angela Waring, capital, December 31, 2008 $ 56,100

ANGELA’S BUSINESS SERVICESStatement of Owner’s Equity Year Ended December 31, 2008

ANGELA’S BUSINESS SERVICESBalance Sheet

December 31, 2008

Assets Liabilities Cash $40,400 Accounts payable $ 5,300 Prepaid insurance 3,600 Salary payable 4,000 Supplies 7,900 Unearned revenue 1,500 Office equipment $25,000 Total liabilities 10,800 Less: Accumulated Owner’s Equity depreciation (10,000) 15,000 Angela Waring, capital 56,100 Total assets $66,900 Total liabilities and owner’s equity $66,900

1eSG_CO3_0131792075.QXD 10/19/06 1:16 PM Page 103

Page 136: polpoa_sg

104 Chapter 3 | Do It Yourself! Question 2 Solutions

Journal Entry:

Dec. 31 Salary Expense (4 3 $7,500/5) (Expense,1; debit) 6,000 Salary Payable (Liability,1; credit) 6,000 To accrue salary expense.

PostDate Accounts and Explanation Ref. Dr. Cr.

Journal Entry:

Dec. 31 Supplies Expense ($1,000 – $350) (Expense,1; debit) 650 Supplies (Asset,2; credit) 650 To record supplies expense.

PostDate Accounts and Explanation Ref. Dr. Cr.

Journal Entry:

Dec. 31 Interest Expense (4 3 $80) (Expense,1; debit) 320 Interest Payable (Liability,1; credit) 320 To accrue interest expense.

PostDate Accounts and Explanation Ref. Dr. Cr.

Do It Yourself! Question 2 Solutions

Requirement 1

Make the necessary adjusting entry for each situation.

a. Employees work five days a week (Monday through Friday) and arepaid $7,500 for the previous week of work each Friday. December31, 2009, falls on a Thursday.

$7,500/5 days = $1,500 salary per day of workMonday through Thursday = 4 days of work

4 ! $1,500 = $6,000

b. The Supplies account shows an unadjusted balance of $1,000.However, only $350 of supplies remains on hand at December 31,2009.

$1,000 – $350 = $650 of supplies used

c. The company forgot to record four months of interest expense ($80per month) that has been incurred but not yet paid.

4 months ! $80 per month = $320

1eSG_CO3_0131792075.QXD 10/19/06 1:16 PM Page 104

Page 137: polpoa_sg

The Power of PracticeFor more practice using the skills learned in this chapter, visit MyAccountingLab.There you will find algorithmically generated questions that are based on theseDemo Docs and your main textbook’s Review and Accounting Practice sections.

1. Go to MyAccountingLab and follow these steps:

2. Direct your URL to www.myaccountinglab.com.3. Log in using your name and password.4. Click the MyAccountingLab link.5. Click Study Plan in the left navigation bar.6. From the table of contents, select Chapter 3, The Adjusting Process.7. Click a link to work on the tutorial exercises.

The Power of Practice | Chapter 3 105

1eSG_CO3_0131792075.QXD 10/19/06 1:16 PM Page 105

Page 138: polpoa_sg

1eSG_CO3_0131792075.QXD 10/19/06 1:16 PM Page 106

Page 139: polpoa_sg

Completing theAccounting Cycle4

1

WHAT YOU PROBABLY ALREADY KNOW

If you work, you probably notice that your pay stub includes year-to-date earnings. When a new year begins, the year-to-date totalsfrom last year are gone and the year-to-date earnings include onlythe current year. Last year’s records are not erased or unimportant.Earnings from last year are reported for each employee on a W-2form, Wage and Tax Statement. This form is sent to employees inJanuary of the new year to attach to their income tax return. Theyear-to-date earnings are zeroed out to be ready to accumulateearnings in the new year. The same thing happens in a business. Atthe end of the accounting year, the earnings and revenue accountsare zeroed out to get ready for the new year.

Learning Objectives

Explain the steps in the accounting cycle.

The accounting cycle is the process by which companies gatherfinancial information to produce their financial statements. You arealready familiar with the first four steps of the accounting cycle by whatyou have studied and practiced so far in Chapters 1–3. Namely, identifytransactions that affect the business, journalize the transactions, postthe journal entries to the ledger, and prepare the accounting worksheet.In this chapter, you learn how to use the worksheet to prepare thefinancial statements, journalize adjusting entries and post them to theledger, and journalize and post the closing entries. The final step is toprepare a post-closing trial balance. Understanding the accountingcycle is important, so be sure to review Exhibit 4-1 (p. 181) in the maintextbook, which summarizes these eight steps.

Prepare a worksheet.

A worksheet is a multicolumned document or spreadsheet that is usedto summarize accounting data at the end of the period. In Chapter 3,Exhibit 3-8 (p. 141) illustrated the first of three columns of data in theworksheet; the trial balance, adjusting journal entries, and the adjustedtrial balance. The adjusted balances are brought forward to one of thelast two sets of columns, either the income statement columns or thebalance sheet columns. The income statement debit column includesexpenses and the credit column includes revenue accounts. The balancesheet debit column includes assets and withdrawals; the credit columnincludes liabilities, owner’s equity, and accumulated depreciationaccounts. Review Exhibits 4-2 through 4-6 (p. 184) to understand theflow of steps in preparing the worksheet.

2

1eSG_CO4_0131792075.Qxd 10/19/06 1:21 PM Page 107

Page 140: polpoa_sg

108 Chapter 4 | Completing the Accounting Cycle

3

4

Prepare financial statements using the worksheet.

The worksheet is a working document. It helps to make the adjustingentries, prepare financial statements, and close out the temporaryaccounts. Review Exhibits 4-7 and 4-8 (pp. 187–188) to understand theflow of information from the worksheet to the financial statements.

Close the revenue, expense, and withdrawal accounts.

The revenue, expense, and withdrawal accounts are temporary accounts;the account balances are zeroed (closed) out at the end of the year to getready for recording transactions in the new year. Income Summary is atemporary account that is used only for this process. The closing processalso updates the ending Capital account balance for the net income or netloss and withdrawals during the year. Review Exhibits 4-10 through 4-12(pp. 191–193) to enhance your understanding of the closing process.

Classify assets and liabilities as current or long-term.

Current assets and liabilities are those that will be dealt with withineither a one-year period or the business’s normal operating cycle,whichever is longer. Long-term assets and liabilities are all assets andliabilities that are not current. It’s important to remember that a liability(such as a car loan or a mortgage payment) may need to be splitbetween current and long-term liability classifications on the balancesheet, such as when an installment on a loan is due within one year, butfuture installments are due thereafter. Refer to Exhibit 4-14 (p. 196) toreview the classified balance sheet.

5

1eSG_CO4_0131792075.Qxd 10/19/06 1:21 PM Page 108

Page 141: polpoa_sg

Demo Doc 1 | Chapter 4 109

Closing EntriesLearning Objectives 1–5

This question continues on from the Wood’s Restaurant problem given in Chapter 3(see p. 73 of your Study Guide).

Use the data from Wood’s Restaurant’s adjusted trial balance at December 31,2008:

Cash $10,600 Accounts receivable 15,800 Supplies 400

Prepaid rent 2,000 Furniture 15,000 Accumulated depreciation, furniture $6,000 Accounts payable 2,600 Salary payable 120 Daniel Wood, capital 40,000 Daniel Wood, withdrawals 11,500 Service revenue 25,800 Salary expense 10,120

Rent expense 6,000 Supplies expense 1,600 Depreciation expense 1,500

Total $74,520 $74,520

Account Title Dr. Cr.

Adjusted Trial Balance

WOOD’S RESTAURANTAdjusted Trial Balance

December 31, 2008

Demo Doc 1

Requirements

1. Prepare Wood’s accounting worksheet showing the adjusted trial balance,the income statement accounts, and the balance sheet accounts.

2. Journalize and post the closing entries.

3. Which assets are current? Which assets are long-term?

4. Which liabilities are current? Which liabilities are long-term?

1eSG_CO4_0131792075.Qxd 10/19/06 1:21 PM Page 109

Page 142: polpoa_sg

110 Chapter 4 | Demo Doc 1 Solutions

Part 1 Part 2 Part 3 Part 4 Part 5 Part 6 Part 7Demo DocComplete

Cash $10,600 Accounts receivable 15,800 Supplies 400

Prepaid rent 2,000 Furniture 15,000 Accumulated depreciation, furniture $6,000 Accounts payable 2,600 Salary payable 120 Daniel Wood, capital 40,000 Daniel Wood, withdrawals 11,500 Service revenue 25,800 Salary expense 10,120

Rent expense 6,000 Supplies expense 1,600 Depreciation expense 1,500

Total Net income

Account Title Dr. Cr. Dr. Cr. Dr. Cr.

Trial Balance Adjustments

WOOD’S RESTAURANTWorksheet

December 31, 2008

AdjustedTrial Balance

Demo Doc 1 Solutions

Requirement 1

Prepare Wood’s accounting worksheet showing the adjusted trial balance, theincome statement accounts, and the balance sheet accounts.

Explain the steps inthe accounting cycle

1

Prepare a worksheet2Prepare financialstatements using theworksheet

3

Preparing financial statements from the adjusted accounting data is step 5 of theaccounting cycle.

First, all income statement accounts must be copied over to the income state-ment columns of the worksheet. Revenues and expenses are on the income state-ment. Copy these accounts to the income statement section. Remember to copythe debit balances to the debit column and the credit balances to the credit col-umn (on top of next page).

The remaining accounts belong on the balance sheet. Copy all assets, liabili-ties, and equity accounts to the balance sheet section of the worksheet.Remember to copy all debit balances to the debit column and all credit balancesto the credit column (on bottom of next page).

1eSG_CO4_0131792075.Qxd 10/19/06 1:21 PM Page 110

Page 143: polpoa_sg

Demo Doc 1 Solutions | Chapter 4 111

Cash $10,600 Accounts receivable 15,800 Supplies 400

Prepaid rent 2,000 Furniture 15,000 Accumulated depreciation, furniture $6,000 Accounts payable 2,600 Salary payable 120 Daniel Wood, capital 40,000 Daniel Wood, withdrawals 11,500 Service revenue 25,800 Salary expense 10,120

Rent expense 6,000 Supplies expense 1,600 Depreciation expense 1,500

Total Net income

Account Title Dr. Cr. Dr. Cr. Dr. Cr.

Trial Balance Adjustments

WOOD’S RESTAURANTWorksheet

December 31, 2008

AdjustedTrial Balance

$10,1206,0001,6001,500

$25,800

Cash $10,600 Accounts receivable 15,800 Supplies 400

Prepaid rent 2,000 Furniture 15,000 Accumulated depreciation, furniture $6,000 Accounts payable 2,600 Salary payable 120 Daniel Wood, capital 40,000 Daniel Wood, withdrawals 11,500 Service revenue 25,800 Salary expense 10,120

Rent expense 6,000 Supplies expense 1,600 Depreciation expense 1,500

Total Net income

Account Title Dr. Cr. Dr. Cr. Dr. Cr.

Trial Balance Adjustments

WOOD’S RESTAURANTWorksheet

December 31, 2008

AdjustedTrial Balance

$10,60015,800

400

$10,120

2,00015,000

11,500

6,0001,6001,500

$25,800

$6,0002,600

12040,000

1eSG_CO4_0131792075.Qxd 10/19/06 1:21 PM Page 111

Page 144: polpoa_sg

112 Chapter 4 | Demo Doc 1 Solutions

Net income is calculated by subtracting the expenses from the revenues:

$25,800 ! $19,220 " $6,580

Net income is added to the credit side of the balance sheet to make total deb-its equal total credits. Net income increases the Capital account (as seen inRequirement 2 of this question, where the closing entries are journalized).

Cash $10,600 Accounts receivable 15,800 Supplies 400

Prepaid rent 2,000 Furniture 15,000 Accumulated depreciation, furniture $6,000 Accounts payable 2,600 Salary payable 120 Daniel Wood, capital 40,000 Danield Wood, withdrawals 11,500 Service revenue 25,800 Salary expense 10,120

Rent expense 6,000 Supplies expense 1,600 Depreciation expense 1,500

Total $74,520 $74,520 Net income

Account Title Dr. Cr. Dr. Cr. Dr. Cr.

Trial Balance Adjustments

WOOD’S RESTAURANTWorksheet

December 31, 2008

AdjustedTrial Balance

$10,60015,800

400

$10,120

2,00015,000

11,500

6,0001,6001,500

$19,2206,580

$25,800

$25,800

$25,800

$55,300

$55,300

$48,7206,580

$55,300

$25,800

$6,0002,600

12040,000

Part 1 Part 2 Part 3 Part 4 Part 5 Part 6 Part 7Demo DocComplete

Requirement 2

Journalize and post the closing entries.

Preparing closing entries is necessary for two reasons. First, we need to clear outthe revenue, expense, and withdrawal accounts to a zero balance because theymust begin the next year empty. Second, we need to update the Capital account,which is step 7 in the accounting cycle.

In Chapter 1, we discussed the formula to calculate the balance in theCapital account:

Beginning Capital Amount# Owner Investments

Adjusted Capital Amount

# Net Income (or ! Net Loss)! Owner Withdrawals

Ending Capital Amount

Explain the steps inthe accounting cycle

1

Close the revenue,expense, and with-drawals accounts

4

1eSG_CO4_0131792075.Qxd 10/19/06 1:21 PM Page 112

Page 145: polpoa_sg

Demo Doc 1 Solutions | Chapter 4 113

Capital

Beginning Capital AmountInvestmentsAdjusted Capital AmountNet Income

WithdrawalsEnding Capital Amount

Remember the first reason to prepare closing entries: We need to clear outthe income statement accounts so that they are empty to begin the next year.What do we need to do to bring the Service Revenue account to zero? It has acredit balance of $25,800, so to bring that to zero, we need to debit $25,800.

We know part of our first closing entry:

This formula is the key to preparing the closing entries. We will use this for-mula, but we will do it inside the Capital T-account.

What is in the Capital account right now? From the trial balance, we can see ithas a balance of $40,000. Where did that balance come from? It is the adjusted cap-ital amount. In this particular problem, because no investments were made (asseen from the problem in Chapter 3), it is also the ending balance from last period.

So we have an advantage: The first component of the formula (adjusted cap-ital amount) is already in the T-account.

The next component is net income. It is not already in the Capital account.We do not have a T-account with net income in it, but we can create one.

We create a new T-account called Income Summary. We place in it all thecomponents of net income and come out with the net income number at the bot-tom. From Chapter 1, remember the formula for net income:

Revenues – Expenses = Net income

To calculate net income, we need to get all revenues and expenses into theIncome Summary account.

Let’s look at the Service Revenue T-account:

Service Revenue

Bal. 25,800

Journal Entry:

Dec. 31 Service Revenue 25,800 ????? 25,800

PostDate Accounts Ref. Dr. Cr.

Step 1

What is the credit side of this entry? The reason we were looking at ServiceRevenue to begin with was to help calculate net income using the IncomeSummary. So the other side of the entry must go to the Income Summary:

1eSG_CO4_0131792075.Qxd 10/19/06 1:21 PM Page 113

Page 146: polpoa_sg

114 Chapter 4 | Demo Doc 1 Solutions

Closing the revenue account to zero is step 1 of journalizing closing entries.

Journal Entry:

Dec. 31 Service Revenue 25,800 Income Summary 25,800

PostDate Accounts Ref. Dr. Cr.

Step 1

Service Revenue

25,8001. 25,800

Bal. 0

Income Summary

1. 25,800

Part 1 Part 2 Part 3 Part 4 Part 5 Part 6 Part 7Demo DocComplete

Rent Expense Depreciation Expense

Bal. 6,000 Bal. 1,500

Salary Expense Supplies Expense

Bal. 10,120 Bal. 1,600

Journal Entry:

Dec. 31 Income Summary 19,220 Salary Expense 10,120 Rent Expense 6,000 Supplies Expense 1,600 Depreciation Expense 1,500

PostDate Accounts Ref. Dr. Cr.

Step 2

The next part of net income is the expenses. In this case, we have four differ-ent expenses:

Each of these expenses has a debit balance. In order to bring these accountsto zero, we must credit them. The balancing debit will go to the Income Summaryaccount:

1eSG_CO4_0131792075.Qxd 10/19/06 1:21 PM Page 114

Page 147: polpoa_sg

Demo Doc 1 Solutions | Chapter 4 115

Closing the expense accounts is step 2 of journalizing closing entries.

Rent Expense Depreciation Expense

6,000 1,5002. 6,000 2. 1,500

Bal. 0 Bal. 0

Salary Expense Supplies Expense

10,120 1,6002. 10,120 2. 1,600

Bal. 0 Bal. 0

Part 1 Part 2 Part 3 Part 4 Part 5 Part 6 Part 7Demo DocComplete

Now let’s look at the Income Summary account:

Income Summary

1. 25,8002. 19,220

Bal. 6,580

The purpose of creating this account was to get a net income number. (If youdid the Chapter 3 problem, note that this balance is the same net income numberthat appears on the income statement.)

We can now take this number and put it into the Capital account. How do weremove it from the Income Summary? The Income Summary has a credit balanceof $6,580, which is the net income, so to remove this amount, we must debit theIncome Summary for $6,580:

Journal Entry:

Dec. 31 Income Summary ($25,800 – $19,220) 6,580 ????? 6,580

PostDate Accounts Ref. Dr. Cr.

Step 3

What is the credit side of this entry? The reason we created the IncomeSummary account to begin with was to help calculate the profit or loss for theCapital account. So the credit side of the entry must go to Daniel Wood, Capital:

Journal Entry:

Dec. 31 Income Summary ($25,800 – $19,220) 6,580 Daniel Wood, Capital 6,580

PostDate Accounts Ref. Dr. Cr.

Step 3

1eSG_CO4_0131792075.Qxd 10/19/06 1:21 PM Page 115

Page 148: polpoa_sg

116 Chapter 4 | Demo Doc 1 Solutions

Closing the Income Summary account is step 3 of journalizing closing entries.

Income Summary

1. 25,8002. 19,220

Bal. 6,5803. 6,580

Bal. 0

Part 1 Part 2 Part 3 Part 4 Part 5 Part 6 Part 7Demo DocComplete

Daniel Wood,Withdrawals

Bal. 11,500

This entry adds the net income to the Capital account. Note that it alsobrings the Income Summary account to a zero balance.

The last component of the Capital account formula is withdrawals. AWithdrawals account already exists:

What do we need to do to bring the Withdrawals account to zero? It has adebit balance of $11,500, so to bring that to zero we need to credit $11,500. Thebalancing debit will go to the Daniel Wood, Capital account:

Journal Entry:

Dec. 31 Daniel Wood, Capital 11,500 Daniel Wood, Withdrawals 11,500

PostDate Accounts Ref. Dr. Cr.

Step 4

This entry subtracts the withdrawals from the Capital account.Closing out the Withdrawals account is step 4 of journalizing closing entries.The Capital account now has the following transactions:

Daniel Wood, Capital

40,000 Beginning Capital Amount3. 6,580 Net Income

Withdrawals 4. 11,500Bal. 35,080 Ending Capital Amount

The formula to update the Capital amount has been recreated inside theCapital T-account.

1eSG_CO4_0131792075.Qxd 10/19/06 1:21 PM Page 116

Page 149: polpoa_sg

Demo Doc 1 Solutions | Chapter 4 117

Daniel Wood,Withdrawals

Bal. 11,5004. 11,500

Bal. 0

Daniel Wood, Capital40,000

3. 6,5804. 11,500

Bal. 35,080

Notice that all temporary accounts (that is, the revenue, the expense, thewithdrawals, and the income summary accounts) now show a zero balance.

Requirement 3

Which assets are current? Which assets are long-term?

Part 1 Part 2 Part 3 Part 4 Part 5 Part 6 Part 7Demo DocComplete

Current assets are assets whose benefit will be realized within one year (orreporting period). In this problem, the current assets include the following:

Cash: Cash is used constantly and its benefits are immediate.

Accounts Receivable: Generally, customers pay what they owe to the com-pany in less than one year (reporting period).

Supplies: Supplies are usually purchased close to the time of use (benefit),well within one year or reporting period.

Prepaid Rent: Generally, prepayments are not made more than one year(period) in advance, which means the prepayments (benefits) will be usedwithin one year (reporting period).

Total Current Assets "

Cash # Account # Supplies # PrepaidReceivable Rent

"

$10,600 # $15,800 # $400 # $2,000 " $28,800

Long-term assets are assets whose benefit will be realized in more than oneyear (or reporting period). In this problem, the only long-term asset is:

Furniture: The furniture will be used (benefited from) for many years. Weknow it is a long-term asset because the use of the furniture is represented asdepreciation, which is being taken over many years.

Total Long-Term Assets "

Furniture ! Accumulated "Depreciation

$15,000 ! $6,000 " $9,000

Classify assets andliabilities as current or long-term

5

1eSG_CO4_0131792075.Qxd 10/19/06 1:21 PM Page 117

Page 150: polpoa_sg

118 Chapter 4 | Demo Doc 1 Solutions

Part 1 Part 2 Part 3 Part 4 Part 5 Part 6 Part 7Demo DocComplete

Part 1 Part 2 Part 3 Part 4 Part 5 Part 6 Part 7Demo DocComplete

Current liabilities are liabilities that will be paid (that is, obligations met) withinone year (or reporting period). In this problem, the current liabilities include thefollowing:

Accounts Payable: Accounts payable generally consists of bills from suppliers(such as utilities, providers of raw materials, and inventory). It is rare thatsuch suppliers would be willing to wait a year (or reporting period) for pay-ment. Most often, such bills are due within 30 days.

Salary Payable: Generally, employees are not willing to wait longer than amonth to be paid.

Total Current Libilities "Accounts

# "Payable Salary Payable

$2,600 # $120 " $2,720

This problem does not include any long-term liabilities.

Total Long-Term Liabilities " $0

Classify assets andliabilities as current or long-term

5 Requirement 4

Which liabilities are current? Which liabilities are long-term?

1eSG_CO4_0131792075.Qxd 10/19/06 1:21 PM Page 118

Page 151: polpoa_sg

Quick Practice QuestionsTrue/False

_____ 1. The last step in the accounting cycle is preparing the financialstatements.

_____ 2. The adjusted trial balance columns of a worksheet contain the accountbalances that appear on the financial statements.

_____ 3. If the sum of the worksheet income statement debit column is greaterthan the income statement credit column, it indicates net income.

_____ 4. Capital, revenue, expenses, and withdrawals are closed out at the endof the year.

_____ 5. The postclosing trial balance contains only balance sheet accounts.

_____ 6. The entry to close the Withdrawals account would include a debit tothe owner’s Capital account.

_____ 7. Long-term liabilities are debts that are not due for at least six months.

_____ 8. The capital in the balance sheet credit column of a worksheet repre-sents the beginning capital amount plus any additional investmentsduring the period.

_____ 9. Permanent accounts include revenue and expenses.

_____10. The difference between the debit and credit totals of the balance sheetcolumns of the worksheet is net income or net loss.

Quick Practice Questions | Chapter 4 119

1eSG_CO4_0131792075.Qxd 10/19/06 1:21 PM Page 119

Page 152: polpoa_sg

120 Chapter 4 | Quick Practice Questions

Multiple Choice

1. What is a worksheet?a. A formal statement issued to investorsb. A document required by the Internal Revenue Servicec. A replacement for the general journald. A multicolumn document used by accountants to aid in the prepara-

tion of the financial statements

2. Which of the following accounts would appear in the balancesheet credit column of the worksheet?a. Equipmentb. Salary Payablec. Rent Revenued. Insurance Expense

3. Which of the following accounts is not closed out?a. Accumulated Depreciationb. Service Revenuec. Depreciation Expensed. Owner’s Withdrawals

4. What is the measure of how quickly an item can be convertedinto cash?a. Contribution marginb. Liquidityc. Profitabilityd. Leverage

5. What type of asset is expected to be converted to cash, sold, orconsumed during the next 12 months or within the business’snormal operating cycle if longer than a year?a. Permanent assetb. Quick assetc. Current assetd. Fixed asset

6. What is the time span during which cash is used to acquiregoods and services that are sold to customers and collected in cash?a. Operating cycleb. Cash-to-cash cyclec. Liquidity cycled. Receivables-to-cash cycle

7. In what category would Inventory appear on a classifiedbalance sheet?a. Long-term liabilityb. Plant assetc. Current assetd. Current liability

1eSG_CO4_0131792075.Qxd 10/19/06 1:21 PM Page 120

Page 153: polpoa_sg

Quick Practice Questions | Chapter 4 121

8. Consider the following account balances for Philip’s Rentals asof December 31, 2008:

Cash $10,300 Prepaid rent $ 3,600Accounts payable 7,800 Equipment 15,000Accumulated depreciation 2,000 Supplies 1,200Philip Browning, capital 9,300 Unearned revenue 1,600Philip Browning, withdrawals 2,200 Notes payable 7,500

(due 12/31/2010)

What are the total current assets and total assets, respectively,for Philip’s Rentals?a. $15,100 and $30,100b. $13,900 and $27,100c. $15,100 and $28,100d. $13,700 and $30,100

9. Use the data from question 8. What are the total currentliabilities for Philip’s Rentals?a. $ 7,800b. $ 9,400c. $ 9,800d. $11,400

10. An error has been made on the worksheet if which of thefollowing situations is true?a. The trial balance columns and the adjusted trial balance columns are

not equal.b. The income statement columns and the balance sheet columns are

not equal.c. The adjusted trial balance debit column and credit column do not

equal.d. All of these indicate an error on the worksheet.

1eSG_CO4_0131792075.Qxd 10/19/06 1:21 PM Page 121

Page 154: polpoa_sg

122 Chapter 4 | Quick Practice Questions

Quick Exercises

4-1. For each of the following accounts, indicate whether it nor-mally has a debit or credit balance and whether it appears in the income statement or balance sheet columns of theworksheet.

Normal Debit (Dr.) Income Statement or Credit (Cr.) (IS) or Balance

Balance Sheet (BS)

a. Equipment _______________ _____________b. Salary Expense _______________ _____________c. Unearned Revenue _______________ _____________d. Accumulated Depreciation _______________ _____________e. Accounts Payable _______________ _____________f. Service Revenue _______________ _____________

4-2. Complete the worksheet information in the adjusted trial bal-ance columns.

WOOD’S RESTAURANTWorksheet

December 31, 2009

Cash $30,800 Accounts receivable 5,800 Prepaid insurance 2,400

Building 17,000 Accumulated depreciation, building $8,000 Accounts payable 1,600 Salary payable 1,000 D. Wood, capital 50,320 D. Wood, withdrawals 7,500 Service revenue 25,800 Insurance expense 2,500 Rent expense 5,500

Salary expense 11,120 Depreciation expense 2,000 Supplies expense 2,100

Total $86,720 $86,720

Account Title Dr. Cr. Dr. Cr. Dr. Cr.

Trial Balance Adjustments

AdjustedTrial Balance

a. $6,000

b. 1,200

b. $1,200

d. 2,400c. 500

$10,100 $10,100

c. 500

d. 2,400

a. 6,000

1eSG_CO4_0131792075.Qxd 10/19/06 1:21 PM Page 122

Page 155: polpoa_sg

Quick Practice Questions | Chapter 4 123

4-3. Complete the remainder of the following worksheet:

Cash $8,400 Accounts receivable 10,100 Supplies 2,400

Prepaid rent 4,000 Furniture 24,400 Accumulated depreciation, furniture $4,000 Accounts payable 3,100 Unearned revenue 2,420 D. Wood, capital 42,000 D. Wood, withdrawals 8,000 Service revenue 25,000 Rent expense 6,000 Salary expense 9,820

Depreciation expense 1,500 Supplies expense 1,900 Total $76,520 $76,520

Net income

Account Title Dr. Cr. Dr. Cr. Dr. Cr.

Trial Balance Adjustments

AdjustedTrial Balance

WOOD’S RESTAURANTWorksheet

December 31, 2010

1eSG_CO4_0131792075.Qxd 10/19/06 1:21 PM Page 123

Page 156: polpoa_sg

124 Chapter 4 | Quick Practice Questions

Journal Entry:

PostDate Account Title Ref. Dr. Cr.

Journal Entry:

PostDate Account Title Ref. Dr. Cr.

Journal Entry:

PostDate Account Title Ref. Dr. Cr.

Journal Entry:

PostDate Account Title Ref. Dr. Cr.

4-4. Given the following adjusted account balances, record theclosing entries for Sports Unlimited on December 31, 2007.

Joseph Golf, Capital $ 85,000Service Revenue 104,400Depreciation Expense, Building 2,000Salary Expense 28,000Supplies Expense 8,500Interest Revenue 15,400Rent Expense 15,000Joseph Golf, Withdrawals 3,000

1eSG_CO4_0131792075.Qxd 10/19/06 1:21 PM Page 124

Page 157: polpoa_sg

Quick Practice Questions | Chapter 4 125

4-5. Given the following adjusted account balances on December 31,2007, calculate total (a) current assets, (b) current liabilities, (c)long-term assets, and (d) long-term liabilities.

Cash $ 25,000Accounts Receivable 12,500Land 100,000Joseph Golf, Capital 75,000Accounts Payable 15,200Building 245,000Accum. Depreciation, Building 70,000Salary Payable 1,500Notes Payable (due 12/31/09) 39,000

1eSG_CO4_0131792075.Qxd 10/19/06 1:21 PM Page 125

Page 158: polpoa_sg

126 Chapter 4 | Do It Yourself! Question 1

Do It Yourself! Question 1This question continues on from the Angela’s Business Services problem givenin Chapter 3.

Use the data from Angela’s Business Services trial balance at December 31, 2008:

Cash $ 40,400 Prepaid insurance 3,600 Supplies 7,900 Office equipment 25,000 Accumulated depreciation, equipment $ 10,000 Accounts payable 5,300 Salary payable 4,000 Unearned revenue 1,500 Angela Waring, capital 60,000 Angela Waring, withdrawals 8,000 Service revenue 85,300 Salary expense 49,000 Insurance expense 14,400 Supplies expense 15,300 Depreciation expense 2,500 Total $166,100 $166,100

Account Title Dr. Cr.

Adjusted Trial Balance

ANGELA’S BUSINESS SERVICESAdjusted Trial Balance

December 31, 2008

Close the revenue,expense, and with-drawals accounts

5

Journal Entry:

PostDate Accounts Ref. Dr. Cr.

Journal Entry:

Post Date Accounts Ref. Dr. Cr.

Requirement

Journalize and post the closing entries.

1eSG_CO4_0131792075.Qxd 10/19/06 1:21 PM Page 126

Page 159: polpoa_sg

Do It Yourself! Question 1 | Chapter 4 127

Journal Entry:

Post Date Accounts Ref. Dr. Cr.

Journal Entry:

Post Date Accounts Ref. Dr. Cr.

Service Revenue Income Summary

Insurance Expense

Angela Waring,Withdrawals

Salary Expense

Angela Waring,Capital

Depreciation Expense

Supplies Expense

1eSG_CO4_0131792075.Qxd 10/19/06 1:21 PM Page 127

Page 160: polpoa_sg

Journal Entry:

Post Date Accounts Ref. Dr. Cr.

Journal Entry:

Post Date Accounts Ref. Dr. Cr.

Journal Entry:

Post Date Accounts Ref. Dr. Cr.

Journal Entry:

Post Date Accounts Ref. Dr. Cr.

Close the revenue,expense, and with-drawals accounts

5

Revenues Income Summary Capital

Expenses

Withdrawals

Do It Yourself! Question 2

Krake Theaters has the following data for 2008:

Total revenues $10,000Total expenses 13,000Owner withdrawals 1,600Owner investments 4,000

The Capital account had a balance of $8,200 at January 1, 2008.

Requirement

Journalize and post the closing entries.

1eSG_CO4_0131792075.Qxd 10/19/06 1:21 PM Page 128

Page 161: polpoa_sg

Quick Practice Solutions | Chapter 4 129

Quick Practice SolutionsTrue/False

F 1. The last step in the accounting cycle is preparing the financial statements.False–The last step in the accounting cycle is preparing the post-closing trial balance. (p. 182)

T 2. The adjusted trial balance columns of a worksheet contain the account balances that appear on the financial statements. (p. 183)

F 3. If the sum of the work sheet income statement debit column is greater than the income statement credit column, it indicates net income.False–If the sum of the worksheet income statement debit column is greater than the income statement credit column, it indicates a net loss. (p. 185)

F 4. Capital, revenue, expense, and withdrawals are closed out at the end of the year.False–The revenue, expense, and withdrawals are closed out at the end of the year. Capital is a permanent account and is notclosed out. (p. 190)

T 5. The post-closing trial balance contains only balance sheet accounts. (p. 194)

T 6. The entry to close the Withdrawals account would include a debit tothe owner’s Capital account. (p. 192)

F 7. Long-term liabilities are debts that are not due for at least six months.False–Long-term liabilities are those due beyond one year. (p. 195)

F 8. The capital in the balance sheet credit column of a worksheet represents the beginning capital amount plus any additional capital invstments during the period. (p. 185)

F 9. Permanent accounts include revenue and expenses.False–Permanent accounts include assets, liabilities, and owner’s equity. Revenue and expenses are temporary accounts. (p. 190)

T 10. The difference between the debit and credit totals of the balance sheet columns of the worksheet is net income or net loss. (p. 185)

1eSG_CO4_0131792075.Qxd 10/20/06 12:26 PM Page 129

Page 162: polpoa_sg

130 Chapter 4 | Quick Practice Solutions

Multiple Choice

1. What is a worksheet? (p. 182)a. A formal statement issued to investorsb. A document required by the Internal Revenue Service.c. A replacement for the general journald. A multicolumn document used by accountants to aid in the preparation

of the financial statements

2. Which of the following accounts would appear in the balancesheet credit column of the worksheet? (p. 184)a. Equipmentb. Salary Payablec. Rent Revenued. Insurance Expense

3. Which of the following accounts is not closed out? (p. 190)a. Accumulated Depreciationb. Service Revenuec. Depreciation Expensed. Owner’s Withdrawals

4. What is the measure of how quickly an item can be convertedinto cash? (p. 194)a. Contribution marginb. Liquidityc. Profitabilityd. Leverage

5. What type of asset is expected to be converted to cash, sold, orconsumed during the next 12 months or within the business’snormal operating cycle if longer than a year? (p. 196)a. Permanent assetb. Quick assetc. Current assetd. Fixed asset

6. What is the time span during which cash is used to acquiregoods and services that are sold to customers and collected in cash? (p. 195)a. Operating cycleb. Cash-to-cash cyclec. Liquidity cycled. Receivables-to-cash cycle

7. In what category would Inventory appear on a classifiedbalance sheet? (p. 195)a. Long-term liabilityb. Plant assetc. Current assetd. Current liability

1eSG_CO4_0131792075.Qxd 10/19/06 1:21 PM Page 130

Page 163: polpoa_sg

Quick Practice Solutions | Chapter 4 131

8. Consider the following account balances for Philip’s Rentals asof December 31, 2008:

Cash $10,300 Prepaid rent $ 3,600Accounts payable 7,800 Equipment 15,000Accumulated depreciation 2,000 Supplies 1,200Philip Browning, capital 9,300 Unearned revenue 1,600Philip Browning, 2,200 Notes payable 7,500

withdrawals (due 12/31/2010)

What are the total current assets and total assets, respectively,for Philip’s Rentals? (p. 195)a. $15,100 and $30,100b. $13,900 and $27,100c. $15,100 and $28,100d. $13,700 and $30,100

9. Use the data from question 8. What are the total currentliabilities for Philip’s Rentals? (p. 195)a. $ 7,800b. $ 9,400c. $ 9,800d. $11,400

10. An error has been made on the worksheet if which of thefollowing situations is true? (p. 183)a. The trial balance columns and the adjusted trial balance columns are

not equal.b. The income statement columns and the balance sheet columns are

not equal.c. The adjusted trial balance debit column and credit column do not equal.d. All of these indicate an error on the worksheet.

1eSG_CO4_0131792075.Qxd 10/19/06 1:21 PM Page 131

Page 164: polpoa_sg

132 Chapter 4 | Quick Practice Solutions

Quick Exercise Solutions

4-1. For each of the following accounts, indicate whether it nor-mally has a debit or credit balance and whether it appears inthe income statement or balance sheet columns of theworksheet. (p. 191)

Normal Debit (Dr.) Income Statement (IS)or Credit (Cr.) or

Balance Balance Sheet (BS)

a. Equipment Dr. BS

b. Salary Expense Dr. IS

c. Unearned Revenue Cr. BS

d. Accumulated Depreciation Cr. BS

e. Accounts Payable Cr. BS

f. Service Revenue Cr. IS

4-2. Complete the worksheet information in the adjusted trial balance columns. (p. 182)

Cash $30,800 Accounts receivable 5,800 Prepaid insurance 2,400

Building 17,000 Accumulated depreciation, building $ 8,000 Accounts payable 1,600 Salary payable 1,000 D. Wood, capital 50,320 D,. Wood, withdrawals 7,500 Service revenue 25,800 Insurance expense 2,500

Rent expense 5,500 Salary expense 11,120

Depreciation expense 2,000 Supplies expense 2,100 Total $86,720 $86,720

Account Title Dr. Cr. Dr. Cr. Dr. Cr.

Trial Balance Adjustments

AdjustedTrial Balance

$30,80011,8001,200

a. $6,000b. $1,200

17,000

7,500

b. 1,2005,5003,700

d. 2,400 13,520c. 500 2,500

2,100$10,100 $10,100 $95,620 $95,620

$ 8,5001,6003,400

50,320

31,800

c. 500

d. 2,400

a. 6,000

WOOD’S RESTAURANTWorksheet

December 31, 2009

1eSG_CO4_0131792075.Qxd 10/19/06 1:22 PM Page 132

Page 165: polpoa_sg

4-3. Complete the remainder of the following worksheet: (p. 182)

Quick Practice Solutions | Chapter 4 133

WOOD’S RESTAURANTWorksheet

December 31, 2010

Cash $8,400 Accounts receivable 10,100 Supplies 2,400

Prepaid rent 4,000 Furniture 24,400 Accumulated depreciation, furniture $4,000 Accounts payable 3,100 Unearned revenue 2,420 D. Wood, capital 42,000 D. Wood, withdrawals 8,000 Service revenue 25,000 Rent expense 6,000 Salary expense 9,820

Depreciation expense 1,500 Supplies expense 1,900 Total $76,520 $76,520

Net income

Account Title Dr. Cr. Dr. Cr. Dr. Cr.

Trial Balance Adjustments

AdjustedTrial Balance

6,0009,8201,5001,900

$19,2205,780

$25,000

25,000

$25,000

$25,000

$57,300

$57,300

$51,5205,780

$57,300

$8,40010,1002,4004,000

24,400

8,000

$4,0003,1002,420

42,000

4-4. Given the following adjusted account balances, record theclosing entries for Sports Unlimited on December 31, 2007.(pp. 190–194)

Joseph Golf, Capital $ 85,000Service Revenue 104,400

Depreciation Expense, Building 2,000

Salary Expense 28,000

Supplies Expense 8,500

Interest Revenue 15,400

Rent Expense 15,000

Joseph Golf, Withdrawals 3,000

1eSG_CO4_0131792075.Qxd 10/19/06 1:22 PM Page 133

Page 166: polpoa_sg

134 Chapter 4 | Quick Practice Solutions

Journal Entry:

12/31/07 Service Revenue 104,400 Interest Revenue 15,400 Income Summary 119,800 To close out revenue accounts into Income Summary.

PostDate Accounts Ref. Dr. Cr.

Journal Entry:

Income Summary 66,300 Joseph Golf, Capital 66,300 To close out Income Summary into Capital.

PostDate Accounts Ref. Dr. Cr.

Journal Entry:

Joseph Golf, Withdrawals 3,000 Joseph Golf, Capital 3,000 To close out Withdrawals into Capital.

PostDate Accounts Ref. Dr. Cr.

Journal Entry:

Income Summary 53,500 Depreciation Expense 2,000 Salary Expense 28,000 Supplies Expense 8,500 Rent Expense 15,000 To close out expense accounts into Income Summary.

PostDate Accounts Ref. Dr. Cr.

1eSG_CO4_0131792075.Qxd 10/19/06 1:22 PM Page 134

Page 167: polpoa_sg

Quick Practice Solutions | Chapter 4 135

4-5. Given the following adjusted account balances on December 31,2007, calculate total (a) current assets, (b) current liabilities, (c)long-term assets, and (d) long-term liabilities. (p. 196)

Cash $ 25,000Accounts Receivable 12,500Land 100,000Joseph Golf, Capital 75,000Accounts Payable 15,200Building 245,000Accum. Depreciation, Building 70,000Salaries Payable 1,500Notes Payable (due 12/31/09) 39,000

a. Cash $25,000Accounts Receivable 12,500Total Current Assets $37,500

b. Accounts Payable $15,200Salaries Payable 1,500Total Current Liabilities $16,700

c. Land $100,000Building 245,000Less: Acc. Depr., Building (70,000)Total Long-Term Assets $275,000

d. Notes Payable (due 12/31/09) $39,000Total Long-Term Liabilities $39,000

1eSG_CO4_0131792075.Qxd 10/19/06 1:22 PM Page 135

Page 168: polpoa_sg

136 Chapter 4 | Do It Yourself! Question 1 Solutions

Journal Entry:

Dec. 31 Service Revenue 85,300 Income Summary 85,300

Step 1

PostDate Accounts Ref. Dr. Cr.

Journal Entry:

Dec. 31 Income Summary 81,200 Salary Expense 49,000 Supplies Expense 15,300 Insurance Expense 14,400 Depreciation Expense 2,500

Step 2

PostDate Accounts Ref. Dr. Cr.

Journal Entry:

Dec. 31 Income Summary ($85,300 – $81,200) 4,100 Angela Waring, Capital 4,100

Step 3

PostDate Accounts Ref. Dr. Cr.

Journal Entry:

Dec. 31 Angela Waring, Capital 8,000 Angela Waring, Withdrawals 8,000

PostDate Accounts Ref. Dr. Cr.

Step 4

Do It Yourself! Question 1 Solutions

Requirement

Journalize and post the closing entries.

1eSG_CO4_0131792075.Qxd 10/19/06 1:22 PM Page 136

Page 169: polpoa_sg

Do It Yourself! Question 1 Solutions | Chapter 4 137

Service Revenue Income Summary85,300 1. 85,300

1. 85,300 2. 81,200

Bal. 0 Bal. 4,100

3. 4,100

Bal. 0

Insurance Expense14,400

2. 14,400Bal. 0 Angela Waring,

Withdrawals

8,000Salary Expense 4. 8,00049,000 Bal. 0

2. 49,000

Bal. 0Angela Waring,

CapitalDepreciation 60,000

Expense2,500

3. 4,100

2. 2,500 4. 8,000Bal. 0 Bal. 56,100

Supplies Expense15,300

2. 15,300Bal. 0

1eSG_CO4_0131792075.Qxd 10/19/06 1:22 PM Page 137

Page 170: polpoa_sg

138 Chapter 4 | Do It Yourself! Question 2 Solutions

Journal Entry:

Dec. 31 Income Summary 13,000 Expenses 13,000

Step 2

PostDate Accounts Ref. Dr. Cr.

Journal Entry:

Dec. 31 Capital 3,000 Income Summary ($13,000 – $10,000) 3,000

Step 3

PostDate Accounts Ref. Dr. Cr.

Journal Entry:

Dec. 31 Capital 16,000 Withdrawals 16,000

Step 4

PostDate Accounts Ref. Dr. Cr.

Journal Entry:

Dec. 31 Revenues 10,000 Income Summary 10,000

Step 1

PostDate Accounts Ref. Dr. Cr.

Do It Yourself! Question 2 Solutions

Requirement

Journalize and post the closing entries.

1eSG_CO4_0131792075.Qxd 10/19/06 1:22 PM Page 138

Page 171: polpoa_sg

Do It Yourself! Question 2 Solutions | Chapter 4 139

Revenues Income Summary

10,000 1. 10,0001. 10,000 2. 13,000

Bal. 0 Bal. 3,000

3. 3,000

Bal. 0

Expenses Withdrawals

13,000 1,6002. 13,000 4. 1,600

Bal. 0 Bal. 0

Capital8,200 Beginning Capital Amount4,000 Owner Investments

Bal. 12,200 Ending Capital AmountNet loss 3. 3,000Withdrawals 4. 1,600

Bal. 7,600 Ending Capital Amount

1eSG_CO4_0131792075.Qxd 10/19/06 1:22 PM Page 139

Page 172: polpoa_sg

140 Chapter 4 | The Power of Practice

The Power of PracticeFor more practice using the skills learned in this chapter, visit MyAccountingLab.There you will find algorithmically generated questions that are based on theseDemo Docs and your main textbook’s Review and Accounting Practice sections.

To go to MyAccountingLab and follow these steps:

1. Direct your URL to www.myaccountinglab.com.2. Log in using your name and password.3. Click the MyAccountingLab link.4. Click Study Plan in the left navigation bar.5. From the table of contents, select Chapter 4, Completing the Accounting Cycle.6. Click a link to work on the tutorial exercises.

1eSG_CO4_0131792075.Qxd 10/19/06 1:22 PM Page 140

Page 173: polpoa_sg

Accounting for aRetail Business5

WHAT YOU PROBABLY ALREADY KNOW

You want to order a pair of pants from a mail-order catalog. The pricelisted in the catalog is $50. You find a 10% off coupon in the catalogfor first-time customers that you plan to use. You also see that youwill be charged $6.95 for shipping and handling for an order of thissize. How much will the pair of pants cost you? The selling price listedis $50, but $50 is not the cost to you. The 10% coupon results in a $5discount ($50 ! 0.10) decreasing the cost to $45 ($50 – 5). However,the shipping and handling charge of $6.95 adds to the cost of thepants. Your cost for the pants is $51.95 ($50.00 – $5.00 + $6.95).

Businesses calculate the cost of assets purchased in the samemanner. When inventory is acquired, the cost is calculated as:

Purchase price on the invoice+ Cost of shipping or freight– Discounts taken

Total inventory cost

Learning Objectives

Describe the supply chain that links suppliers, retailers, andcustomers.

Suppliers, wholesalers, and retailers have different operations and dealwith different types of customers. These businesses choose one of twoinventory recording methods: the periodic system (where inventoryrecords are updated from time to time on a regular basis) and theperpetual system (where inventory records are updated constantly). SeeExhibit 5-1, Panel B (p. 231) for a description of the activities betweensuppliers and retailers.

Journalize transactions between the supplier and retailer.

Companies can receive discounts from their suppliers for paying theiraccounts early (known as purchase discounts). If purchased goods are notacceptable or unnecessary, they may be returned (purchase returns andallowances). See Exhibit 5-1, Panel A (p. 230) for a description of theactivities between retailers and suppliers, manufacturers, and wholesalers.

Journalize transactions between the retailer and customer.

Sellers can offer discounts as an incentive for prompt payment fromcustomers (known as sales discounts). Customers can use credit cardsto pay, requiring that commissions be paid to the credit card companyby the seller. Customers may return goods to the seller (sales returnsand allowances). See Exhibit 5-1, Panel B (p. 231) for a description of theactivities between retailers and customers.

1

2

3

1eSG_CO5_0131792075.Qxd 10/19/06 1:27 PM Page 141

Page 174: polpoa_sg

142 Chapter 5 | Accounting for a Retail Business

4

5

6

Journalize shipment transactions and identify other sellingexpenses in a retail business.

As seen in the What You Probably Already Know section, many factorscan alter an item’s cost, including shipping. Shipping costs can be paidby the seller (FOB destination) or buyer (FOB shipping). See Exhibit 5-6,(p. 243) for a description of FOB terms.

Prepare a retailer’s financial statements.

The income statement of a merchandiser may be prepared as asingle-step income statement (listing all revenues followed by allexpenses to determine net income or loss) or as a multi-step incomestatement (showing various subtotals such as gross profit andincome from operations). The other statements are similar to those fora service company. Review a merchandiser’s financial statements inExhibit 5-7 (p. 249).

Compute the gross profit percentage and inventory turnover rate.

Two ratios that provide important information for a merchandiser arethe gross profit percentage (computed by dividing gross profit by netsales revenue) and inventory turnover (the ratio of cost of goods sold toaverage inventory). See Exhibit 5-7 (p. 249) for an example of how touse these ratios.

1eSG_CO5_0131792075.Qxd 10/19/06 1:27 PM Page 142

Page 175: polpoa_sg

Demo Doc 1 | Chapter 5 143

Demo Doc 1Inventory Transaction Analysis (Perpetual System)Learning Objectives 1–6

Danner Inc. began operations on January 1, 2008. Danner had the followingtransactions during the year:

Jan. 10 Purchased inventory for $400 with credit terms of 2/15, net 30.12 Paid for the January 1 purchase in full.

Feb. 1 Sold 10 units costing $21 each to a customer for $360 onaccount. This sale had credit terms of 1/15, net 40.

9 Customer returned three units from his February 1 orderbecause he did not like the color of the goods.

18 Customer paid for the February 1 order (less returns) in full.May 5 Purchased inventory for $250 with credit terms of 2/10, net 30.

6 Paid special freight costs of $30 on the May 5 inventory pur-chase in cash.

14 Found that 15% of the goods purchased on May 5 weredefective. Danner returned these goods.

June 1 Paid for the May 5 purchase (less returns) in full.Oct. 1 Sold $160 of goods to a customer for $220 with credit terms

of 1/20, net 30.19 Received cash payment in full for the October 1 sale.

Requirements

1. Journalize these transactions using the perpetual method. Explanationsare not required.

2. Show the Inventory and COGS T-accounts for the year.

3. Use the COGS formula to calculate COGS for the year.

4. Prepare the top portion of Danner’s 2008 income statement (ending withgross profit).

5. Calculate Danner’s inventory turnover rate for 2008.

1eSG_CO5_0131792075.Qxd 10/19/06 1:27 PM Page 143

Page 176: polpoa_sg

144 Chapter 5 | Demo Doc 1 Solution

Journal Entry:

Jan. 1 Inventory 400 Accounts Payable 400

PostDate Accounts Ref. Dr. Cr.

Demo Doc 1 SolutionRequirement 1

Journalize these transactions using the perpetual method. Explanations arenot required.

Jan. 1 Purchased inventory for $400 with the credit terms of 2/15, net 30.

Part 1 Part 2 Part 3 Part 4 Part 5 Demo DocComplete

The perpetual method requires that inventory and COGS be updated immedi-ately whenever they are changed (unlike the periodic method, which delays mostinventory and COGS journal entries until the end of the year). Under the per-petual method, all transactions in this problem must be journalized when theyoccur.

The Inventory account is involved here because inventory was purchased.Inventory is increased by $400 (a debit). Because the inventory was not paid forin cash (it was purchased on account), Accounts Payable must also be increasedby $400 (a credit).

Note that the actual credit terms do not matter at this point, only that thepurchase was not made in cash.

However, note that 2/15, net 30 means that the customer will get a 2% dis-count if the full amount is paid within 15 days. Otherwise, full payment is due in30 days.

Describe the supplychain that links suppli-ers, retailers, and customers

1

Journalize transactionsbetween the supplierand retailer

2

Jan. 12 Paid for the January 1 purchase in full.Remember that 2/15 means that if full payment is made within 15 days, the cus-tomer gets a 2% discount.

We are paying the supplier, so we can decrease our Accounts Payable by $400(a debit). Cash also decreases (a credit), but by how much? January 12 is within15 days of the original purchase, so Danner is entitled to take the discount.Therefore, Danner only has to pay 100% – 2% = 98% of the purchase price to sat-isfy the debt owed. So the cash paid is:

98% ! $400 " $392

The difference is an adjustment to the Inventory account. The cost principle saysthat we should journalize assets at cost. The true cost of the inventory is now lessthan we originally thought. So Inventory is decreased (a credit) by this difference.

Journalize transactionsbetween the supplierand retailer

2

1eSG_CO5_0131792075.Qxd 10/19/06 1:27 PM Page 144

Page 177: polpoa_sg

Demo Doc 1 Solution | Chapter 5 145

The amount of the adjustment to Inventory was made to balance the entry.In all journal entries, total debits equal total credits.

The amount of the adjustment to Inventory is the amount needed to makethe total debits in the entry equal to the total credits in the entry. In this case, acredit of $400 – $392 = $8 is required. The amount to balance, $8, represents theearly payment discount:

$400 ! 2% " $8

Feb. 1 Sold 10 units costing $21 each to a customer for $360 onaccount. This sale had credit terms of 1/15, net 40.

This transaction involves two parts. First, Danner earns sales revenue of $360,which increases Sales Revenue (a credit) and (because it is not paid for in cash,but rather sold on account) increases Accounts Receivable (a debit).

Second, Danner is also selling inventory. This means that Inventory isdecreased (a credit) and COGS is increased (a debit) by:

10 ! $21 " $210

Remember that 1/15, net 40 means that the customer will get a 1% discount if thefull amount is paid within 15 days. Otherwise, full payment is due in 40 days.

Journal Entry:

Feb. 1 Accounts Receivable 360 Sales Revenue 360

COGS (10 units 3 $21) 210 Inventory 210

PostDate Accounts Ref. Dr. Cr.

Journal Entry:

Jan. 12 Accounts Payable 400 Inventory (to balance*) 8 Cash [(100% – 2%) 3 $400] 392 *400 – 392 = 8

PostDate Accounts Ref. Dr. Cr.

Journalize transactionsbetween the retailerand customer

3

Feb. 9 Customer returned three units from his February 1 orderbecause he did not like the color of the goods.

Because the customer is returning goods (and the goods are not defective) to thecompany, Danner’s Inventory increases (a debit) by 3 ! $21 = $63, which thencauses COGS to decrease (a credit) by $63.

The customer has not yet paid, so the amount of Accounts ReceivableDanner can collect from the customer decreases (a credit) by 3/10 ! $360 = $108.Instead of decreasing Sales Revenue, we increase Sales Returns and Allowances(a debit) by $108. In this way, Danner can keep track of sales returns and makebetter business decisions.

Journalize transactionsbetween the retailerand customer

3

1eSG_CO5_0131792075.Qxd 10/19/06 1:27 PM Page 145

Page 178: polpoa_sg

146 Chapter 5 | Demo Doc 1 Solutions

Journal Entry:

Feb. 18 Cash ($360 – $108) 252 Accounts Receivable 252

PostDate Accounts Ref. Dr. Cr.

Journal Entry:

Feb. 9 Inventory (3 3 $21) 63 COGS 63

Sales Returns and Allowances [(3/10) 3 $360] 108 Accounts Receivable 108

PostDate Accounts Ref. Dr. Cr.

Feb. 18 Customer paid for the February 1 order (less returns) in full.Remember that 1/15 means that if full payment is made within 15 days, the cus-tomer gets a 1% discount.

However, the customer is paying 18 days after the sale, which is longer thanthe 15 days the discount allows. Therefore, the customer must pay the fullamount.

Cash is increased (a debit) by $360 – $108 = $252 (original sale of $360 lessthe sales return of $108). Because the customer pays Danner, Accounts Receivableis decreased (a credit) by $252.

Journalize transactionsbetween the retailerand customer

3

May 5 Purchased inventory for $250 with credit terms of 2/10, net 30.Inventory is increased by $250 (a debit). Because the inventory was not paid forin cash but rather on account, Accounts Payable must also be increased by $250(a credit).

Remember that 2/10, net 30 means that the customer will get a 2% discount ifthe full amount is paid within 10 days. Otherwise, full payment is due in 30 days.

Journalize transactionsbetween the supplierand retailer

2

Journal Entry:

May 5 Inventory 250 Accounts Payable 250

PostDate Accounts Ref. Dr. Cr.

May 6 Paid special freight costs of $30 on the May 5 inventory pur-chase in cash.

The total cost of the inventory is the purchase price plus any additional purchas-ing costs (such as shipping or taxes). Therefore, we include the extra $30 offreight as part of the cost of the inventory.

Inventory increases by $30 (a debit). Because these costs are being paid incash, the Cash account decreases (a credit) by $30.

Journalize shipmenttransactions and iden-tify other sellingexpenses in a retailbusiness

4

1eSG_CO5_0131792075.Qxd 10/19/06 1:27 PM Page 146

Page 179: polpoa_sg

Demo Doc 1 Solution | Chapter 5 147

Journal Entry:

May 6 Inventory 30 Cash 30

PostDate Accounts Ref. Dr. Cr.

May 14 Found that 15% of the goods purchased on May 5 were defec-tive. Danner returned these goods.

When these goods are returned to the supplier, they are taken out of inventory,decreasing Inventory (a credit) by 15% ! $250 = $37.50. Because Danner has notyet paid for the goods, Accounts Payable also decreases by the related amount (adebit).

Journalize transactionsbetween the supplierand retailer

2

Journal Entry:

May 14 Accounts Payable (15% 3 $250) 37.50 Inventory 37.50

PostDate Accounts Ref. Dr. Cr.

June 1 Paid for the May 5 purchase (less returns) in full.Remember that 2/10 means that if full payment is made within 10 days, the cus-tomer gets a 2% discount.

Accounts Payable is decreased by the original payable less returns made:$250 – $37.50 = $212.50 (a debit). Cash is also decreased (a credit). AlthoughJune 1 is 27 days after the original purchase and within the deadline for paymentof 30 days, it is not early enough to take the discount. Therefore, the cash paid isthe full amount of $212.50. In order for Danner to be entitled to take the dis-count, the payment must be made by May 15 (May 5 plus 10 days).

Journalize transactionsbetween the supplierand retailer

2

Journal Entry:

June 1 Accounts Payable 212.50 Cash 212.50

PostDate Accounts Ref. Dr. Cr.

Oct. 1 Sold $160 of goods to a customer for $220 with credit termsof 1/20, net 40.

The company earns sales revenue of $220, increasing Sales Revenue (a credit)and (because it is not paid for in cash) increasing Accounts Receivable (adebit).

As the company sells inventory, Inventory decreases (a credit) and COGSincreases (a debit) by $160.

Note that the actual credit terms do not matter at this point, only that thesale was not made in cash.

However, 1/20, net 40 means that the customer will get a 1% discount if thefull amount is paid within 20 days. Otherwise, full payment is due in 40 days.

Journalize transactionsbetween the retailerand customer

3

1eSG_CO5_0131792075.Qxd 10/19/06 1:27 PM Page 147

Page 180: polpoa_sg

148 Chapter 5 | Demo Doc 1 Solution

Even though the sale was on account, the actual sale must be journalized atthis time.

Journal Entry:

Oct. 1 Accounts Receivable 220 Sales Revenue 220

COGS 160 Inventory 160

PostDate Accounts Ref. Dr. Cr.

Oct. 19 Received cash for payment in full for the October 1 sale.This payment is within the 20-day period, so the customer is entitled to take thediscount. The customer pays 100% – 1% = 99% of the receivable amount, or 99% !$220 = $217.80.

Accounts Receivable is decreased by the full amount of $220 (a credit)because the bill has been paid and no more can be collected from the customer.Cash is increased by $217.80 (a debit) and the difference (the amount to balanceis the amount of the discount, $220 ! 10%) goes to Sales Discounts.

Journalize transactionsbetween the retailerand customer

3

Journal Entry:

Oct. 19 Cash [(100% – 1%) 3 $220] 217.80 Sales Discounts (to balance*) 2.20 Accounts Receivable 220 *220 = 217.80 – 2.20

PostDate Accounts Ref. Dr. Cr.

Requirement 2

Show the Inventory and COGS T-accounts for the year.

Part 1 Part 2 Part 3 Part 4 Part 5 Demo DocComplete

The entries are posted into the T-accounts (just as in previous chapters). However,for this question, we only want to see the Inventory and COGS T-accounts in detail:

Journalize transactionsbetween the supplierand retailer

2

Journalize transactionsbetween the retailerand customer

3 INVENTORY COGS

Jan. 1 400 Feb. 1 210Jan. 12 8 Feb. 9 63Feb. 1 210 Oct. 1 160

Feb. 9 63 Bal. 307May 5 250May 6 30

May 14 37.50Oct. 1 160

Bal. 327.50

1eSG_CO5_0131792075.Qxd 10/19/06 1:27 PM Page 148

Page 181: polpoa_sg

Demo Doc 1 Solution | Chapter 5 149

Requirement 3

Use the COGS formula to calculate COGS for the year.

Part 1 Part 2 Part 3 Part 4 Part 5 Demo DocComplete

We can use the COGS formula to calculate COGS: Journalize transactionsbetween the supplierand retailer

2

Journalize transactionsbetween the retailerand customer

3COGS " Beginning # Inventory $ Ending

inventory purchases inventory" 0 # (400 – 8) # (250 # 30 $ 37.50) $ 327.50" 0 # 392 # 242.50 $ 327.50" 0 # 634.50 $ 327.50" 307

Requirement 4

Prepare the top portion of Danner’s 2008 income statement (ending withgross profit).

Prepare a retailer’sfinancial statements

5

Part 1 Part 2 Part 3 Part 4 Part 5 Demo DocComplete

Sales Discounts and Sales Returns and Allowances are contra-accounts to SalesRevenue. As we did with Accumulated Depreciation, these contra-accounts mustbe shown on the financial statements, then combined with their associatedaccount to create the net value (in this case, net sales revenue).

Service revenue $580.00 Less: Sales discounts $ 2.20 Sales returns and allowances 108.00 (110.20) Net sales revenue $469.80 Cost of goods sold (307.00) Gross profit $162.80

DANNER CORP.Income Statement

Year Ended December 31, 2008

(= $360 + $220)

1eSG_CO5_0131792075.Qxd 10/19/06 1:27 PM Page 149

Page 182: polpoa_sg

150 Chapter 5 | Demo Doc 1 Solution

Inventory turnover " COGS/Average inventory

Average (when used in a financial ratio) generally means the beginning balanceplus the ending balance divided by 2.

2008 Inventory Turnover" $307/[($0 + $327.50)/2]" $307/$163.75" 1.9 times

Part 1 Part 2 Part 3 Part 4 Part 5 Demo DocComplete

Requirement 5

Calculate Danner’s inventory turnover rate for 2008.

Part 1 Part 2 Part 3 Part 4 Part 5 Demo DocComplete

Compute the grossprofit percentage andinventory turnover rate

6

1eSG_CO5_0131792075.Qxd 10/19/06 1:27 PM Page 150

Page 183: polpoa_sg

Quick Practice QuestionsTrue/False

_____ 1. Cost of Goods Sold is included in a merchandiser’s income statementbut excluded from a service company’s income statement.

_____ 2. Under the periodic inventory system, the only way to determine thecost of goods sold is to take a physical count of the merchandise onhand.

_____ 3. Most businesses use the periodic inventory system because it offersmanagement more control over inventory.

_____ 4. A sales return requires two entries to be journalized if the seller uses aperpetual inventory system.

_____ 5. Sales Returns and Allowances is an expense account.

_____ 6. The single-step income statement shows gross profit and income fromoperations.

_____ 7. An inventory count is not performed if the perpetual inventory systemis used.

_____ 8. Advertising expenses would be considered general expenses on theincome statement.

_____ 9. A higher inventory turnover is preferable to a lower turnover.

_____10. A company with a gross profit percentage of 40% must have a highernet income than one with a gross profit percentage of 30%.

Quick Practice Questions | Chapter 5 151

1eSG_CO5_0131792075.Qxd 10/19/06 1:27 PM Page 151

Page 184: polpoa_sg

Multiple Choice1. What do credit terms 1/10, n/30 indicate?

a. A 10% discount is available if payment is made within 30 daysb. A 10% discount is available if payment is made within 10 daysc. A 1% discount is available if payment is made within 30 daysd. A 30% discount is available if payment is made within 10 days

2. Which of the following is necessary to journalize the purchase of merchandise under a perpetual inventory system?a. A credit to Cashb. A debit to Accounts Payablec. A credit to Inventoryd. A debit to Inventory

3. What account is credited when a discount is taken for promptpayment under a perpetual inventory system?a. Accounts Payableb. Accounts Receivablec. Purchase Discountsd. Inventory

4. What is the entry required to journalize the payment of a $200freight bill, assuming the shipping terms are FOB shippingpoint, under a perpetual inventory system?a. Debit Inventory and credit Cashb. Debit Accounts Payable and credit Inventoryc. Debit Inventory and credit Purchase Discountsd. Debit Purchase Discounts and credit Inventory

5. How does the purchaser account for transportation chargeswhen goods are shipped to them FOB of destination?a. No journal entry would be made for the transportation chargesb. Debit Delivery Expense for the amount of the transportation chargesc. Debit Freight In for the amount of the transportation chargesd. Debit Inventory for the amount of the transportation charges

6. Which of the following balances is normally a debit?a. Sales Revenueb. Sales Returns and Allowancesc. Net Sales Revenued. Gross Profit

7. When the seller is liable for the shipping costs, what account isdebited when payment is made?a. Delivery Expenseb. Freight Inc. Inventoryd. Cash

8. Which of the following is necessary to journalize an adjustmentto account for inventory shrinkage under a perpetual system?a. A credit to Miscellaneous Expenseb. A credit to Cost of Goods Soldc. A credit to Inventoryd. A debit to Miscellaneous Expense

152 Chapter 5 | Quick Practice Questions

1eSG_CO5_0131792075.Qxd 10/19/06 1:27 PM Page 152

Page 185: polpoa_sg

9. Which of the following accounts should be closed to IncomeSummary?a. Beginning Inventoryb. Sales Returns and Allowancesc. Owner Withdrawalsd. Ending Inventory

10. What does inventory turnover indicate?a. How quickly inventory is received from the supplier after the order

is placedb. How many days it takes the inventory to travel between the seller’s

warehouse and the buyer’s warehousec. How rapidly inventory is soldd. How many days it takes from the time an order is received until the

day it is shipped

Quick Practice Questions | Chapter 5 153

1eSG_CO5_0131792075.Qxd 10/19/06 1:27 PM Page 153

Page 186: polpoa_sg

154 Chapter 5 | Quick Practice Questions

Quick Exercises

5-1. Werner Company purchased $11,000 of merchandise. Thepurchase invoice is for $11,200, which includes transportationcharges of $200. The company returned $2,900 of the goodsreceived before paying the invoice. The company paid theinvoice within the discount terms 2/10, n/30.Compute thefollowing amounts:

a. The amount of the discount

b. The total amount for the merchandise journalized in the Inventoryaccount

c. The amount that the purchaser would remit if paid after the discount period

5-2. Indicate whether the following accounts are:

Closed out with a debit to the account (DR)Closed out with a credit to the account (CR)Not closed out at all (NC)

a. ___________ Sales Revenueb. ___________ Sales Returns and Allowancesc. ___________ Salary Expensed. ___________ Inventorye. ___________ Depreciation Expensef. ___________ Accumulated Depreciationg. ___________ Accounts Receivableh. ___________ Interest Revenuei. ___________ Interest Expensej. ___________ Cost of Goods Sold

5-3. The following data are for the Griswold Corporation:

Sales revenue $600,000

Freight-in 42,000

Beginning inventory 77,000

Purchase discounts 19,000

Sales returns and allowances 33,000

Ending inventory 81,000

Inventory purchases 415,000

Sales discounts 35,000

Purchase returns and allowances 39,000

1eSG_CO5_0131792075.Qxd 10/19/06 1:27 PM Page 154

Page 187: polpoa_sg

Journal Entry:

PostDate Accounts Ref. Dr. Cr.

Journal Entry:

PostDate Accounts Ref. Dr. Cr.

Journal Entry:

PostDate Accounts Ref. Dr. Cr.

Journal Entry:

PostDate Accounts Ref. Dr. Cr.

Requirements

1. Calculate Net Sales Revenue.

2. Calculate the Cost of Goods Available.

3. Calculate the Cost of Goods Sold.

5-4. Moyer Company had the following transactions duringAugust. Assuming that the perpetual inventory system isused, journalize these transactions.

Aug. 1 Purchased $2,900 of merchandise on account fromRyan Company, terms 3/15, n/60.

9 Paid transportation cost of $440 directly to thetrucking company for the August 5 purchase.

10 Returned $600 of defective merchandise pur-chased on August 5.

15 Paid for the August 5 purchase, less the return andthe discount.

1eSG_CO5_0131792075.Qxd 11/22/06 10:18 AM Page 155

Page 188: polpoa_sg

156 Chapter 5 | Quick Practice Questions

Journal Entry:

PostDate Accounts Ref. Dr. Cr.

Journal Entry:

PostDate Accounts Ref. Dr. Cr.

Journal Entry:

PostDate Accounts Ref. Dr. Cr.

Journal Entry:

PostDate Accounts Ref. Dr. Cr.

Journal Entry:

PostDate Accounts Ref. Dr. Cr.

Journal Entry:

PostDate Accounts Ref. Dr. Cr.

5-5. Journalize the transactions in Quick Exercise 5-4 for RyanCompany. Assume that the cost of goods sold is 50% of thesales price.

1eSG_CO5_0131792075.Qxd 11/22/06 10:18 AM Page 156

Page 189: polpoa_sg

Do It Yourself! Question 1 | Chapter 5 157

Do It Yourself! Question 1

Franco Bros. began operations on January 1, 2008. Franco had the followingtransactions during the year:

Jan. 1 Purchased inventory for $150 under credit terms 2/10, net 30.

8 Paid for the January 1 purchase in full.

Mar. 1 Purchased inventory for $240 under credit terms 2/20, net 45.

Apr. 1 Paid for the March 1 purchase in full.

July 1 Sold $80 worth of goods to a customer for $120 undercredit terms 5/15, net eom (end of month).

12 Received cash payment in full for the July 1 sale.

Sept. 1 Found that 10% of the goods purchased on March 1 weredefective. Franco Bros. returned these goods.

Oct. 1 Received cash refund for the goods returned on September 1.

Dec. 1 Sold $210 worth of goods to a customer for $320 undercredit terms 1/10, net eom (end of month).

6 Customer returned 20% of his December 1 order because hedid not like the color of the goods.

12 Customer paid for the December 1 order (less returns) in full.

Requirements

1. Journalize these transactions using the perpetual method. Explanations arenot required.

Jan. 1 Purchased inventory for $150 under credit terms 2/10, net 30.

Describe the supplychain that links suppli-ers, retailers, andcustomers

1

Journalize transactionsbetween the supplierand retailer

2

Journalize transactionsbetween the retailerand customer

3

Journalize shipmenttransactions and iden-tify other sellingexpenses in a retailbusiness

4

Journal Entry:

PostDate Accounts Ref. Dr. Cr.

Jan. 8 Paid for the January 1 purchase in full.

Journal Entry:

PostDate Accounts Ref. Dr. Cr.

1eSG_CO5_0131792075.Qxd 11/22/06 10:18 AM Page 157

Page 190: polpoa_sg

Mar. 1 Purchased inventory for $240 under credit terms 2/20, net 45.

158 Chapter 5 | Do It Yourself! Question 1

Journal Entry:

PostDate Accounts Ref. Dr. Cr.

Journal Entry:

PostDate Accounts Ref. Dr. Cr.

Journal Entry:

PostDate Accounts Ref. Dr. Cr.

Journal Entry:

PostDate Accounts Ref. Dr. Cr.

Apr. 1 Paid for the March 1 purchase in full.

July 1 Sold $80 worth of goods to a customer for $120 under creditterms 5/15, net eom (end of month).

July 12 Received cash payment in full for the July 1 sale.

1eSG_CO5_0131792075.Qxd 10/19/06 1:27 PM Page 158

Page 191: polpoa_sg

Sept. 1 Found that 10% of the goods purchased on March 1 weredefective. Franco Bros. returned these goods.

Do It Yourself! Question 1 | Chapter 5 159

Journal Entry:

PostDate Accounts Ref. Dr. Cr.

Journal Entry:

PostDate Accounts Ref. Dr. Cr.

Journal Entry:

PostDate Accounts Ref. Dr. Cr.

Oct. 1 Received cash refund for the goods returned on September 1.

Dec. 1 Sold $210 worth of goods to a customer for $320 undercredit terms 1/10, net eom (end of month).

Dec. 6 Customer returned 20% of his December 1 order because hedid not like the color of the goods.

Journal Entry:

PostDate Accounts Ref. Dr. Cr.

1eSG_CO5_0131792075.Qxd 10/19/06 1:27 PM Page 159

Page 192: polpoa_sg

Dec. 12 Customer paid for the December 1 order (less returns) in full.

160 Chapter 5 | Do It Yourself! Question 1

INVENTORY COGS

Journal Entry:

PostDate Accounts Ref. Dr. Cr.

2. Show the Inventory and COGS T-accounts for the year.Journalize transactionsbetween the supplierand retailer

2

Journalize transactionsbetween the retailerand customer

3

3. Prepare the top portion of Franco’s 2008 income statement (ending withgross profit).

Prepare a retailer’sfinancial statements

5

1eSG_CO5_0131792075.Qxd 10/19/06 1:27 PM Page 160

Page 193: polpoa_sg

Quick Practice SolutionsTrue/False

T 1. Cost of Goods Sold is included in a merchandiser’s income statement but excluded from a service company’s income statement. (p. 237)

T 2. Under the periodic inventory system, the only way to determine the cost of goods sold is to take a physical count of the merchandise on hand. (p. 232)

F 3. Most businesses use the periodic inventory system because it offers management more control over inventory.False–The perpetual inventory system offers management more control over inventory and is the system most businesses use. (p. 233)

T 4. A sales return requires two entries to be journalized if the seller uses a perpetual inventory system. (p. 236)

F 5. Sales Returns and Allowances is an expense account.False–Sales Returns and Allowances is a contra account to Sales Revenue. (p. 240)

F 6. The single-step income statement shows gross profit and income from operations.False–The multi-step income statement shows gross profit and income from operations. (p. 248)

F 7. An inventory count is not performed if the perpetual inventory system is used.False–Inventory counts are performed for all inventory systems.(p. 232–233)

F 8. Advertising expenses would be considered general expenses on the income statement.False–Advertising would be considered a selling expense on the income statement. (p. 247)

T 9. A higher inventory turnover is preferable to a lower turnover.(p. 251–252)

F 10. A company with a gross profit percentage of 40% must have a higher net income than one with a gross profit percentage of 30%.False–The gross profit percentage indicates the amount of gross profit per dollar of sales. It does not consider the operating expenses, which are deducted from gross profit to determine netincome. (p. 251)

Quick Practice Solutions | Chapter 5 161

1eSG_CO5_0131792075.Qxd 10/20/06 12:31 PM Page 161

Page 194: polpoa_sg

Multiple Choice

1. What do credit terms 1/10, n/30 indicate? (p. 235)a. A 10% discount is available if payment is made within 30 daysb. A 1% discount is available if payment is made within 10 daysc. A 1% discount is available if payment is made within 30 daysd. A 30% discount is available if payment is made within 10 days

2. Which of the following is necessary to journalize the purchase ofmerchandise under a perpetual inventory system? (p. 234)a. A credit to Cashb. A debit to Accounts Payablec. A credit to Inventoryd. A debit to Inventory

3. What account is credited when a discount is taken for promptpayment under a perpetual inventory system? (p. 237)a. Accounts Payableb. Accounts Receivablec. Purchase Discountsd. Inventory

4. What is the entry required to journalize the payment of a $200freight bill, assuming the shipping terms are FOB shippingpoint, under a perpetual inventory system? (p. 244)a. Debit Inventory and credit Cashb. Debit Accounts Payable and credit Inventoryc. Debit Inventory and credit Purchase Discountsd. Debit Purchase Discounts and credit Inventory

5. How does the purchaser account for transportation chargeswhen goods are shipped to them FOB of destination? (p. 245)a. No journal entry would be made for the transportation charges.b. Debit Delivery Expense for the amount of the transportation charges.c. Debit Freight In for the amount of the transportation charges.d. Debit Inventory for the amount of the transportation charges.

6. Which of the following balances is normally a debit? (p. 241)a. Sales Revenueb. Sales Returns and Allowancesc. Net Sales Revenued. Gross Profit

7. When the seller is liable for the shipping costs, what account isdebited when payment is made? (p. 243)a. Delivery Expenseb. Freight Inc. Inventoryd. Cash

162 Chapter 5 | Quick Practice Solutions

1eSG_CO5_0131792075.Qxd 10/20/06 12:31 PM Page 162

Page 195: polpoa_sg

8. Which of the following is necessary to journalize an adjustmentto account for inventory shrinkage under a perpetual system? (p. 286)a. A credit to Miscellaneous Expenseb. A credit to Cost of Goods Soldc. A credit to Inventoryd. A debit to Miscellaneous Expense

9. Which of the following accounts should be closed to IncomeSummary? (p. 287)a. Beginning Inventoryb. Sales Returns and Allowancesc. Owner Withdrawalsd. Ending Inventory

10. What does inventory turnover indicate? (p. 251–252)a. How quickly inventory is received from the supplier after the order is

placedb. How many days it takes the inventory to travel between the seller’s

warehouse and the buyer’s warehousec. How rapidly inventory is soldd. How many days it takes from the time an order is received until the

day it is shipped

Quick Practice Solutions | Chapter 5 163

1eSG_CO5_0131792075.Qxd 10/20/06 12:31 PM Page 163

Page 196: polpoa_sg

Quick Exercise

5-1. Werner Company purchased $11,000 of merchandise. The pur-chase invoice is for $11,200, which includes transportationcharges of $200. The company returned $2,900 of the goodsreceived before paying the invoice. The company paid theinvoice within the discount terms 2/10, n/30. Compute the following amounts: (p. 235)

a. The amount of the discount$11,000 – $2,900 = $8,100 net sales$8,100 ! 0.02 = $162

b. The total amount for the merchandise journalized in the Inventoryaccount$11,200 – $2,900 – $162 = $8,138

c. The amount that the purchaser would remit if paid after the dis-count period$11,200 – $2,900 = $8,300

5-2. Indicate whether the following accounts are: (p. 287)

Closed out with a debit to the account (DR)Closed out with a credit to the account (CR)Not closed out at all (NC)

164 Chapter 5 | Quick Practice Solutions

a. DR Cash Sales Revenueb. CR Sales Returns and Allowancesc. CR Salary Expensed. NC Inventorye. CR Depreciation Expensef. NC Accumulated Depreciationg. NC Accounts Receivableh. DR Interest Revenuei. CR Interest Expensej. CR Cost of Goods Sold

5-3. The following data are for the Griswold Corporation: (p. 248)

Sales revenue $600,000

Freight-in 42,000

Beginning inventory 77,000

Purchase discounts 19,000

Sales returns and allowances 33,000

Ending inventory 81,000

Inventory purchases 415,000

Sales discounts 35,000

Purchase returns and allowances 39,000

Requirement 1Calculate Net Sales Revenue.$600,000 – $33,000 – $35,000 = $532,000

1eSG_CO5_0131792075.Qxd 10/20/06 12:31 PM Page 164

Page 197: polpoa_sg

Requirement 2Calculate the Cost of Goods Available.$77,000 + $415,000 + $42,000 – $19,000 – $39,000 = $476,000

Requirement 3Calculate the Cost of Goods Sold.$476,000 – $81,000 = $395,000

5-4. Moyer Company had the following transactions duringAugust. Assuming that the perpetual inventory system isused, journalize these transactions. (p. 234–236)

Aug. 5 Purchased $2,900 of merchandise on account fromRyan Company, terms 3/15, n/60.

9 Paid transportation cost of $440 directly to thetrucking company for the August 5 purchase.

10 Returned $600 of defective merchandise purchasedon August 5.

15 Paid for the August 5 purchase, less the return andthe discount.

Quick Practice Solutions | Chapter 5 165

Journal Entry:

PostDate Accounts Ref. Dr. Cr.

Journal Entry:

Post Date Accounts Ref. Dr. Cr.

Journal Entry:

Post Date Accounts Ref. Dr. Cr.

Journal Entry:

Post Date Accounts Ref. Dr. Cr.

Aug. 5 Inventory 2,900 Accounts Payable 2,900

Aug. 9 Inventory 440 Cash 440

Aug. 10 Accounts Payable 600 Inventory 600

Aug. 15 Accounts Payable 2,300 Inventory 69 Cash 2,231

1eSG_CO5_0131792075.Qxd 10/20/06 12:31 PM Page 165

Page 198: polpoa_sg

166 Chapter 5 | Quick Practice Solutions

5-5. Journalize the transactions in Quick Exercise 5-4 for RyanCompany. Assume that the cost of goods sold is 50% of thesales price. (p. 237)

Journal Entry:

Aug. 5 Accounts Receivable 2,900 Sales 2,900

PostDate Accounts Ref. Dr. Cr.

Journal Entry:

Aug. 5 Costs of Goods Sold 1,450 Inventory

PostDate Accounts Ref. Dr. Cr.

Journal Entry:

Aug. 9 No entry required

PostDate Accounts Ref. Dr. Cr.

Journal Entry:

Aug. 10 Sales Returns and Allowances 600 Accounts Receivables 600

PostDate Accounts Ref. Dr. Cr.

Journal Entry:

Aug. 10 Inventory 300 Costs of Goods Sold 300

PostDate Accounts Ref. Dr. Cr.

Journal Entry:

Aug. 15 Cash 2,231 Sales Discounts 69 Accounts Receivable 2,300

PostDate Accounts Ref. Dr. Cr.

1eSG_CO5_0131792075.Qxd 10/19/06 1:27 PM Page 166

Page 199: polpoa_sg

Do It Yourself! Question 1 Solutions | Chapter 5 167

Journal Entry:

Jan.1 Inventory 150 Accounts Payable 150

PostDate Accounts Ref. Dr. Cr.

Do It Yourself! Question 1 Solutions

Requirement 1

Journalize these transactions. Explanations are not required.

Jan. 1 Purchased inventory for $150 under credit terms 2/10, net 30.

Jan. 8 Paid for the January 1 purchase in full.

Journal Entry:

Jan. 8 Accounts Payable 150 Inventory 3 Cash [(100% – 2%) 3 $150] 147

PostDate Accounts Ref. Dr. Cr.

Journal Entry:

Mar. 1 Inventory 240 Accounts Payable 240

PostDate Accounts Ref. Dr. Cr.

Journal Entry:

Apr. 1 Accounts Payable 240 Cash 240

PostDate Accounts Ref. Dr. Cr.

Mar. 1 Purchased inventory for $240 under credit terms 2/20, net 45.

Apr. 1 Paid for the March 1 purchase in full.

1eSG_CO5_0131792075.Qxd 11/22/06 10:19 AM Page 167

Page 200: polpoa_sg

168 Chapter 5 | Do It Yourself! Question 1 Solutions

July 1 Sold $80 worth of goods to a customer for $120 under creditterms 5/15, net eom (end of month).

Journal Entry:

July 12 Cash [(100% – 5%) 3 $120] 114 Sales Discounts (to balance*) 6 Accounts Receivable 120 *120 – 114 = 6

PostDate Accounts Ref. Dr. Cr.

Journal Entry:

Sept. 1 Accounts Receivable (10% 3 $240) 24 Inventory 24

PostDate Accounts Ref. Dr. Cr.

Journal Entry:

July 1 Accounts Receivable 120 Sales Revenue 120

COGS 80 Inventory 80

PostDate Accounts Ref. Dr. Cr.

July 12 Received cash payment in full for the July 1 sale.

Sept. 1 Found that 10% of the goods purchased on March 1 weredefective. Franco Bros. returned these goods.

Oct. 1 Received cash refund for the goods returned on September 1.

Journal Entry:

Oct. 1 Cash 24 Accounts Receivable 24

PostDate Accounts Ref. Dr. Cr.

Dec. 1 Sold $210 worth of goods to a customer for $320 under creditterms 1/10, net eom (end of month).

Journal Entry:

Dec. 1 Accounts Receivable 320 Sales Revenue 320

COGS 210 Inventory 210

PostDate Accounts Ref. Dr. Cr.

1eSG_CO5_0131792075.Qxd 10/19/06 1:27 PM Page 168

Page 201: polpoa_sg

Do It Yourself! Question 1 Solutions | Chapter 5 169

Dec. 6 Customer returned 20% of his December 1 order because hedid not like the color of the goods.

Journal Entry:

Dec. 6 Inventory (20% 3 $210) 42 COGS 42

Sales Returns and Allowances (20% 3 $320) 64 Accounts Receivable 64

PostDate Accounts Ref. Dr. Cr.

Dec. 12 Customer paid for the December 1 order (less returns) in full.

Journal Entry:

Dec. 12 Cash ($320 – $64) 256 Accounts Receivable 256

PostDate Accounts Ref. Dr. Cr.

Requirement 2

Show the inventory and COGS T-accounts for the year.

INVENTORY COGSJan. 1 150 July 1 80

Jan. 8 3 Dec. 1 210

Mar. 1 240 Dec. 6 42July 1 80 Bal. 248Sept. 1 24Dec. 1 210

Dec. 6 42Bal. 115

Requirement 3

Prepare the top portion of Franco’s 2008 income statement (ending withgross profit).

Service revenue $440 Less: Sales discounts $ 6 Sales returns and allowances 64 (70) Net sales revenue $370 Cost of goods sold (248) Gross profit $122

FRANCO BROS.Income Statement

Year Ended December 31, 2008

(= $120 + $320)

1eSG_CO5_0131792075.Qxd 11/22/06 10:20 AM Page 169

Page 202: polpoa_sg

170 Chapter 1 | The Power of Practice

The Power of PracticeFor more practice using the skills learned in this chapter, visit MyAccountingLab.There you will find algorithmically generated questions that are based on theseDemo Docs and your main textbook’s Review and Accounting Practice sections.

To go to MyAccountingLab and follow these steps:

1. Direct your URL to www.myaccountinglab.com.2. Log in using your name and password.3. Click the MyAccountingLab link.4. Click Study Plan in the left navigation bar.5. From the table of contents, select Chapter 5, Accounting for a Retail Business.6. Click a link to work on the tutorial exercises.

1eSG_CO5_0131792075.Qxd 10/19/06 1:27 PM Page 170

Page 203: polpoa_sg

Internal Control and Cash6

1

WHAT YOU PROBABLY ALREADY KNOW

When you shop in a department store, you probably noticed electronictags on some of the goods. The cashier removes the tag upon purchaseto avoid sounding an alarm when you exit the store through thesecurity gates. You may also notice that fine jewelry is likelydisplayed in a locked case that can only be opened by an employee.The employee stays with you until the item is returned to the caseand locked or purchased. Cartons of cigarettes are also usuallysecured behind locked doors or cabinets.

If you work as a cashier, it’s likely that you have your own cashdrawer. Periodically the cash is collected and deposited in a safe ortaken to the bank. At the end of the shift, the cash is counted andcompared to the sales rung up for the period to determine that theappropriate amount of cash is in the drawer. These observations arejust a few of the procedures and policies that businesses employ toachieve a good system of internal control.

Learning Objectives

Define fraud and describe the different types of fraud in business.

Fraud involves deceit or trickery that causes financial harm to a business orits stakeholders. Two broad types of fraud are employee embezzlement andmanagement fraud, and can include misappropriation (or theft) of businessassets and fraudulent financial reporting. Many potential fraudulent activitiesfall under these broad definitions. Fraud is an intentional behavior and threefactors are often included: perceived pressure, perceived opportunity, andrationalization. Be sure to review the “Fraud” and “The Fraud Triangle”sections in the main text, including Exhibit 6-1 (p. 295), which shows theconnection between these three factors.

Describe an internal control system.

An internal control system serves to safeguard assets, encourage accuratefinancial reporting and efficient and effective operations, and comply withapplicable laws and operations. Some components of a good system ofinternal control include separation of responsibilities, physical safeguards,authorization procedures that check the validity of disbursements, keepingadequate records and documentation, and independent checks and auditsto validate performance and accounting records and statements. Reviewthe overview of internal control in Exhibit 6-3 (p. 299). This topic is criticalfor business owners and managers.

2

HarrCh06v1.qxd 10/23/06 8:30 AM Page 171

Page 204: polpoa_sg

172 Chapter 6 | Internal Control and Cash

3

4

Apply internal controls for cash and prepare a bank reconciliation.

Because cash is liquid, desirable, and easily concealed, especially diligentinternal controls are required. Challenges exist whether cash is receivedover the counter or by mail. The assignment and separation of employeeresponsibilities is important for handling cash. Cashiers should each usea separate drawer. The cash should be counted and checked against thesales register information. Remittances that are mailed in are opened,and the checks and source documentation are forwarded to two separateindividuals. A third party verifies that the amount deposited agrees withthe source documentation. Review the cash receipt controls in Exhibits6-4 and 6-5 (pp. 301–302).

Often the cash balance on a depositor’s books is different from the amounton the bank statement. The bank reconciliation reconciles, or brings intoagreement, the bank account balance on the depositor’s records with thebank’s records. Review the bank statement, cash records, and bankreconciliation in Exhibits 6-8 through 6-10. (pp. 307–309).

As you review Exhibit 6-10, think about the objective of the bankreconciliation: to arrive at the correct book balance. This focus should helpyou understand the rationale for why the various items are added to orsubtracted from the balance per bank and the balance per books. Whenthe “Balance per books” is different from the “Adjusted book balance,”journal entries must be recorded for all of the items that appear betweenthose two amounts. These journal entries affect Cash and bring the bookbalance to the correct amount. Continue to review the journal entriesrelated to Exhibit 6-10.

Record journal entries for the petty cash fund.

A petty cash fund is a small amount of cash used for minor expendituressuch as cab fare, tips, and supplies. When the fund is initially set up, PettyCash (an asset) is debited and Cash is credited to establish the fund. Thisentry is only made when the fund is established or the petty cash fundamount is changed. When petty cash is disbursed, tickets are prepared bythe custodian as a record of the expenditure. When the cash in the fundbecomes low, the custodian submits the petty cash tickets for reimbursementor replenishment. A check is issued to Petty Cash for the total of the petty cashtickets. These activities bring the petty cash fund back to the original amountestablished. Review the Petty Cash Ticket in Exhibit 6-12 (p. 312) and the“Petty Cash” section of the textbook.

Explain the reporting of cash on the balance sheet.

Cash is the first item on the balance sheet because it is the most liquidasset. It is not uncommon for companies to have more than one bankaccount or items included in the Cash amount. Cash includes anythingthat a bank can accept for deposit. Only cash that is unrestricted(available) should be included as Cash. Often, the first item on the balancesheet may be referred to as “Cash and cash equivalents.” Cash equivalentsare short-term investments that can be converted into a known amount ofcash. These items mature within 90 days of the balance sheet date and mayinclude time deposits, money market funds, certificates of deposit, andU.S. Treasury bills and Treasury notes. Note that stock and bondinvestments are not cash equivalents. Review the “Reporting Cash on theBalance Sheet” section of the textbook.

5

HarrCh06v1.qxd 10/23/06 8:30 AM Page 172

Page 205: polpoa_sg

Demo Doc 1 | Chapter 6 173

Cash

July 1 Bal. 2,100July 8 400

July 14 300July 25 900

July 29 120July 30 500

July 31 Bal. 720

Bank ReconciliationsLearning Objectives 1–3

Hunter Corp. has the following information for July 2008:

Demo Doc 1

Bank Statement for July 2008

Balance, July 1, 2008 $2,100

Deposits

July 14 300

Checks

July 8 $400

July 10 230*

July 25 900 (1,530)

Other items:

NSF check from Jim Andrews (150)

Interest on account balance 25

EFT—collection of installment payments from customers 800

EFT—monthly rent expense (700)

Service charges (75)

Balance, July 31, 2008 $770

* The July 10 check was not written by Hunter. It was written by another bank customer and taken from Hunter’s account in error.

Hunter deposits all cash receipts and makes all payments by check.

HarrCh06v1.qxd 10/23/06 8:30 AM Page 173

Page 206: polpoa_sg

174 Chapter 6 | Demo Doc 1

Requirements

1. Prepare Hunter’s bank reconciliation at July 31, 2008.2. Journalize any entries required by Hunter and update Hunter’s Cash T-account.

Explanations are not required.3. The employee at Hunter who opens the mail and physically collects the cash

is the same person who updates the cash receipts journal and prepares thebank reconciliation. Is this practice part of a good internal control system?

HarrCh06v1.qxd 10/23/06 8:30 AM Page 174

Page 207: polpoa_sg

Demo Doc 1 Solutions | Chapter 6 175

Cash

July 1 Bal. 2,100July 8 400

July 14 300July 25 900

July 29 120July 30 500

July 31 Bal. 720

Part 1 Part 2 Part 3 Part 4 Demo DocComplete

Demo Doc 1 SolutionsRequirement 1

Prepare Hunter’s bank reconciliation at July 31, 2008.

To prepare the bank reconciliation, we need to make adjustments to both the bankbalance and Hunter’s Cash balance. First, we must determine what these adjust-ments are.

The bank reconciliation begins with a column for the bank and a column forthe company (Hunter). Put the starting Cash balance for each side at the top.

Bank Books (Hunter)

Balance, July 31 $770 Balance, July 31 $720

As we find them, reconciling items will be added to and subtracted from eachside.

Bank Books (Hunter)

Balance, July 31 $770 Balance, July 31 $720

Add: Add:

Less: Less:

A reconciling item arises because a valid transaction has not been recorded byboth parties. For example, if the bank records service charges and Hunter does not, areconciling item is required to bring Hunter’s Cash balance to the correct amount.

Each reconciling item will be described in the appropriate column and addedor subtracted from the Cash balance of the party that has not yet recorded thattransaction/entry.

Deposits in Transit

According to the Cash T-account, Hunter made two deposits: $300 on July 14 and$120 on July 29.

Apply internal controlsfor cash and prepare abank reconciliation

3

HarrCh06v1.qxd 10/23/06 8:30 AM Page 175

Page 208: polpoa_sg

176 Chapter 6 | Demo Doc 1 Solutions

Cash

July 1 Bal. 2,100July 8 400

July 14 300July 25 900

July 29 120July 30 500

July 31 Bal. 720

However, the bank statement only shows one (the July 14 deposit for $300).The July 29 deposit for $120 has not yet been recorded by the bank. This deposit intransit increases the bank account when the bank processes and records thedeposit.

Bank Books (Hunter)

Balance, July 31 $770 Balance, July 31 $720

Add: Add:

Deposits in transit $120

Less: Less:

Outstanding Checks and Bank Error

According to the Cash T-account, Hunter wrote three checks: July 8, $400; July 25,$900; and July 30, $500.

The bank statement shows three checks; however, only two of them were writ-ten by Hunter (the July 8 check for $400 and the July 25 check for $900).

The July 10 check for $230 shown on the bank statement is a bank error anddoes not relate to Hunter. This error needs to be corrected by the bank. (It would bea good idea for Hunter to contact the bank to confirm that it is correcting this mis-take.) This correction increases Cash on the bank’s side.

The July 30 check for $500 has not yet been recorded by the bank. Thisoutstanding check decreases the bank account when it is recorded. The bankrecords this check in the (near) future when it is cashed.

Bank Books (Hunter)

Balance, July 31 $770 Balance, July 31 $720

Add: Add:

Deposits in transit $120

Correction of error $230

Less: Less:

Outstanding checks ($500)

NSF Check

A check deposited by Hunter for $150 was returned to the bank for insufficientfunds. Hunter has not yet recorded the return of this customer check.

HarrCh06v1.qxd 10/23/06 8:30 AM Page 176

Page 209: polpoa_sg

Demo Doc 1 Solutions | Chapter 6 177

The $150 the customer owed has not been paid because Hunter was unable tocash the customer’s check. The account receivable must be reinstated and Hunter’sCash account decreases.

Bank Books (Hunter)

Balance, July 31 $770 Balance, July 31 $720

Add: Add:

Deposits in transit $120

Correction of error $230

Less: Less:

Outstanding checks ($500) NSF check ($150)

Interest Earned

Interest revenue of $25 was earned on Hunter’s bank balance but not yet recordedby Hunter. This amount increases Hunter’s Cash account.

Bank Books (Hunter)

Balance, July 31 $770 Balance, July 31 $720

Add: Add:

Deposits in transit $120 Interest earned on $25bank balance

Correction of error $230

Less: Less:

Outstanding checks ($500) NSF check ($150)

Installment Payments Received

Installment payments from customers of $800 were collected by the bank via EFTbut have not yet been recorded by Hunter. These transactions increase Hunter’sCash account.

Bank Books (Hunter)

Balance, July 31 $770 Balance, July 31 $720

Add: Add:

Deposits in transit $120 Interest earned on $25bank balance

Correction of error $230 Installment $800payments collected

Less: Less:

Outstanding checks ($500) NSF check ($150)

HarrCh06v1.qxd 10/23/06 8:30 AM Page 177

Page 210: polpoa_sg

178 Chapter 6 | Demo Doc 1 Solutions

Rent Expense

The rent payment of $700 was made by the bank (on Hunter’s behalf) but has notyet been recorded by Hunter. This transaction decreases Hunter’s Cash account.

Bank Books (Hunter)

Balance, July 31 $770 Balance, July 31 $720

Add: Add:

Deposits in transit $120 Interest earned on $25bank balance

Correction of error $230 Installment $800payments collected

Less: Less:

Outstanding checks ($500) NSF check ($150)

Rent payment ($700)

Service Charges

Service charges of $75 were incurred with the bank but not yet recorded by Hunter.This entry decreases Hunter’s Cash account.

Bank Books (Hunter)

Balance, July 31 $770 Balance, July 31 $720

Add: Add:

Deposits in transit $120 Interest earned on $25bank balance

Correction of error $230 Installment $800payments collected

Less: Less:

Outstanding checks ($500) NSF check ($150)

Rent payment ($700)

Service Charges ($75)

Total $620 Total $620

Part 1 Part 2 Part 3 Part 4 Demo DocComplete

After adding in a title and adding up both sides of the reconciliation, wehave:

HarrCh06v1.qxd 10/23/06 8:30 AM Page 178

Page 211: polpoa_sg

Demo Doc 1 Solutions | Chapter 6 179

Bank:Balance, July 31, 2008 $770 Add: July 29 deposit in transit 120 Bank error (July 10 check not belonging to Hunter) 230 1,120

Less: July 30 outstanding checks (500) Adjusted bank balance, July 31, 2008 $620

Books:Balance, July 31, 2008 $720 Add: Bank collection of installment payments 800 Interest earned on account 25 1,545

Less: Rent payment $700 NSF check 150 Service charge 75 (390) (925) Adjusted book balance, July 31, 2008 $620

HUNTER CORP.Bank Reconciliation

July 31, 2008

Part 1 Part 2 Part 3 Part 4 Demo DocComplete

Notice that both columns on the reconciliation have the same total. Thischeck helps to ensure that all calculations are correct. If these totals were not thesame, it would indicate an error and/or missing data.

Requirement 2

Journalize any entries required by Hunter and update Hunter’s Cash T-account.Explanations are not required.

Any reconciling items on Hunter’s side of the bank reconciliation should be journal-ized. Usually these entries are made in the order in which they appear on the bankreconciliation.

Installment Payments

Cash increases (a debit) and Accounts Receivable decreases (a credit) by $800.

Apply internal controlsfor cash and prepare abank reconciliation

3

Journal Entry:

July 31 Cash (Asset,1; debit) 800 Accounts Receivable (Asset,2; credit) 800

Date Accounts Dr. Cr.

HarrCh06v1.qxd 10/23/06 8:30 AM Page 179

Page 212: polpoa_sg

180 Chapter 6 | Demo Doc 1 Solutions

Journal Entry:

July 31 Cash (Asset,1; debit) 25 Interest Revenue (Revenue,1; credit) 25

Date Accounts Dr. Cr.

Journal Entry:

July 31 Rent Expense (Expense,1; debit) 700 Cash (Asset,2; credit) 700

Date Accounts Dr. Cr.

Interest Earned

Cash increases (a debit) and Interest Revenue increases (a credit) by $25.

Journal Entry:

July 31 Accounts Receivable (Asset,1; debit) 150 Cash (Asset,2; credit) 150

Date Accounts Dr. Cr.

Rent Payment

Cash decreases (a credit) and Rent Expense increases (a debit) by $700.

NSF Check

Cash decreases (a credit) and Accounts Receivable increases (a debit) by $150.

Service Charges

Cash decreases (a credit) and Miscellaneous Expense increases (a debit) by $75.

Journal Entry:

July 31 Miscellaneous Expense (Expense,1; debit) 75 Cash (Asset,2; credit) 75

Date Accounts Dr. Cr.

Post these adjustments to the Cash T-account:

Cash

July 31 Bal. 72080025

July 31 70015075

July 31 Bal. 620

HarrCh06v1.qxd 10/23/06 8:30 AM Page 180

Page 213: polpoa_sg

Demo Doc 1 Solutions | Chapter 6 181

Part 1 Part 2 Part 3 Part 4 Demo DocComplete

m

The final Cash balance is $620, which is also the total on the bank reconcilia-tion. Both totals must agree, and this check makes sure that everything was donecorrectly.

Requirement 3

The employee at Hunter who opens the mail and physically collects the cash isthe same person who updates the cash receipts journal and prepares the bankreconciliation. Is this practice part of a good internal control system?

An employee who collects the cash and records the receipt of the cash and performsthe bank reconciliation has a substantial opportunity for fraud.

The employee could steal the cash and delay recording the cash receipt, or per-haps never record the cash receipt. An employee in such a position could hide thisdishonest act for a long period of time by manipulating the bank reconciliations.

To avoid this problem, most internal control systems require separation ofduties; that is, employees who handle cash (both receipts and payments) are notthe same employees who maintain the accounting records and prepare the bankreconciliations.

Define fraud anddescribe the differenttypes of fraud in business

1

Describe an internalcontrol system 2

Part 1 Part 2 Part 3 Part 4 Demo DocComplete

HarrCh06v1.qxd 10/23/06 8:30 AM Page 181

Page 214: polpoa_sg

182 Chapter 6 | Demo Doc 2

Demo Doc 2Petty CashLearning Objectives 1, 2, 4, 5

Young Brothers established a $300 petty cash fund on July 1, 2008. On July 31,2008, the petty cash box contained $70 cash and the following receipts:

July 5 Travel expenses $80

July 12 Donuts for board meeting 50

July 23 Office supplies 60

July 29 Delivery charges 40

On August 1, 2008, the petty cash balance was replenished.

Requirements

1. Journalize the entry to establish the fund.

2. What is the total cash amount paid from petty cash in July? Journalize theentry to record the expenses incurred from petty cash during July. (Assumeall charges are recorded as office supplies, delivery expense, travel expense,or meals expense.) On what date(s) are these expenses recorded?

3. A Young employee notices that the petty cash has been short several monthsin a row. Although the amounts involved are small (immaterial), the trend isconsistent.What should the employee do?

4. Is petty cash considered to be cash or a cash equivalent?

HarrCh06v1.qxd 10/23/06 8:30 AM Page 182

Page 215: polpoa_sg

Demo Doc 2 Solutions | Chapter 6 183

Journal Entry:

July 1 Petty Cash (Asset,1; debit) 300 Cash in Bank (Asset,2; credit) 300

Date Accounts Dr. Cr.

Part 1 Part 2 Part 3 Part 4 Demo DocComplete

Journal Entry:

Aug. 1 Office Supplies (Asset,1; debit) 60 Delivery Expense (Expense,1; debit) 40 Travel Expense (Expense,1; debit) 80 Meals Expense (Expense,1; debit) 50 Cash in Bank ($300 – $70) (Asset,2; credit) 230

Date Accounts Dr. Cr.

Part 1 Part 2 Part 3 Part 4 Demo DocComplete

Demo Doc 2 SolutionsRequirement 1

Journalize the entry to establish the fund.

When the fund is established, cash is withdrawn from Young’s bank accounts andput into the petty cash box. This transaction increases Petty Cash (a debit) anddecreases Cash (a credit) by $300.

Requirement 2

What is the total cash amount paid from petty cash in July? Journalize the entryto record the expenses incurred from petty cash during July. (Assume all chargesare recorded as office supplies, delivery expense, travel expense, or mealsexpense.) On what date(s) are these expenses recorded?

The receipts in the petty cash box total $80 + $50 + $60 + $40 = $230. This receipttotal means that $300 – $230 = $70 should be left in the petty cash box.

These expenses are not recorded at the time they are incurred. The amountsinvolved are immaterial, so instead we can wait to record them until the petty cashis replenished.

Record journal entriesfor the petty cash fund4

Record journal entriesfor the petty cash fund4

HarrCh06v1.qxd 10/23/06 8:30 AM Page 183

Page 216: polpoa_sg

184 Chapter 6 | Demo Doc 2 Solutions

Part 1 Part 2 Part 3 Part 4 Demo DocComplete

Part 1 Part 2 Part 3 Part 4 Demo DocComplete

Part 1 Part 2 Part 3 Part 4 Demo DocComplete

Requirement 3

A Young employee notices that the petty cash has been short several months ina row. Although the amounts involved are small (immaterial), the trend isconsistent.What should the employee do?

Describe an internalcontrol system2

Define fraud anddescribe the differenttypes of fraud in business

1 It is easy to have cash overages and shortages from month to month. Record keep-ing for petty cash is often spotty because it is usually handled by someone who isnot familiar with accounting. However, consistent shortages every month suggestthe possibility of unethical behavior on the part of the petty cash handler.

Having someone review petty cash transactions periodically provides a goodinternal control.

The employee who notices this trend should discuss it with the person respon-sible for petty cash. If the issue cannot be resolved, then the employee shouldreport it to a supervisor.

Young could implement some internal controls to better monitor petty cash.These controls could include requiring the use of petty cash tickets with anauthorized signature (the person signing would presumably review the receiptsfor correctness).

Requirement 4

Is petty cash considered to be cash or a cash equivalent?

Petty cash is actual cash. The only difference between petty cash and a standardCash T-account is that petty cash is located on the premises of the business ratherthan in a bank account. Therefore, petty cash is considered to be cash for purposesof balance sheet reporting.

Explain the reportingof cash on the balancesheet

5

HarrCh06v1.qxd 10/23/06 8:30 AM Page 184

Page 217: polpoa_sg

Quick Practice Questions | Chapter 6 185

Quick Practice QuestionsTrue/False

_____ 1. A deposit in transit has been recorded by the company but not by thebank.

_____ 2. An NSF check would be recorded on the books by debiting AccountsReceivable.

_____ 3. Employee embezzlement typically involves fraudulent financial report-ing by management.

_____ 4. An audit is the assessment of financial statements to determine whetherthey are fairly presented.

_____ 5. Different people should perform various accounting duties to minimizeerrors and the opportunities for fraud.

_____ 6. Funds disbursed from the petty cash fund will be recorded as a credit tothe Petty Cash account.

_____ 7. The person who prepares checks for payment would be a suitableemployee to reconcile the bank account.

_____ 8. The party signing a check is referred to as the maker.

_____ 9. Outstanding checks would include only those checks written for the cur-rent month that have not cleared or been canceled by the bank.

_____10. It is a good control to have just one person open the checks and depositthem in the bank.

HarrCh06v1.qxd 10/23/06 8:30 AM Page 185

Page 218: polpoa_sg

186 Chapter 6 | Quick Practice Questions

Multiple Choice

1. Which of the following is not an objective of internal control?a. Help safeguard the assets a business uses in its operationsb. Guarantee a company will not go bankruptc. Encourage adherence to company policiesd. Promote operational efficiency

2. Which of the following items used to reconcile cash does notrequire an adjusting entry?a. Bank service chargeb. Interest earnedc. A note collected by the bankd. Deposits in transit

3. Which of the following statements about bank reconciliations is correct?a. Should not be prepared by an employee who handles cash transactionsb. Is part of a sound internal control systemc. Is a formal financial statementd. Both (a) and (b) are correct

4. Which of the following items does not cause a difference betweenthe cash balance per bank and book?a. NSF checksb. Deposits in transitc. Outstanding checksd. Canceled checks

5. The following data are available for Wonder Boutique forOctober:Book balance, October 31 $5,575Outstanding checks 584Deposits in transit 2,500Service charges 75Interest revenue 25

What is the adjusted book balance on October 31 for WonderBoutique based on these data?a. $5,500b. $5,525c. $5,550d. $7,466

6. The bank statement lists a $700 deposit as $70. On a bankreconciliation, the correction for this error will appear as whichof the following?a. Addition to the book balanceb. Deduction from the book balancec. Addition to the bank balanced. Deduction from the bank balance

HarrCh06v1.qxd 10/23/06 8:30 AM Page 186

Page 219: polpoa_sg

Quick Practice Questions | Chapter 6 187

7. Which of the following processes for payment compresses theapproval for payment into a single step?a. Electronic funds transferb. Evaluated receipts settlementc. Electronic data interchanged. Rationalization

8. For which items must journal entries be prepared?a. Any errors made on the books revealed by the bank reconciliationb. Any errors made by the bank revealed by the bank reconciliationc. All items on the bank’s sided. Only outstanding checks

9. Which of the following is not a control over petty cash?a. Keeping an unlimited amount of cash on handb. Supporting all fund disbursements with a petty cash ticketc. Replenishing the fund through normal cash disbursement proceduresd. Designating one employee to administer the fund

10. If the petty cash fund is not replenished on the balance sheetdate, which of the following will be true?a. Assets will be overstatedb. Income will be overstatedc. Neither (a) nor (b)d. Both (a) and (b)

Quick Exercises

6-1. Classify each of the following reconciling items of the Breadand Butter Company as one of the following:

a. An addition to the bank balanceb. A deduction from the bank balancec. An addition to the book balanced. A deduction from the book balancee. Not a reconciling item

__________ 1. Collection of note receivable plus interest revenue bybank

__________ 2. Bookkeeper recorded check #849 as $557 instead ofthe correct amount of $755

__________ 3. Bank service charges__________ 4. Bank credited the account for interest revenue__________ 5. Bank added deposit to Bread and Butter’s account in

error__________ 6. Deposits in transit__________ 7. Bank withdrew $1,270 from Bread and Butter’s

account for a check written for $12,700__________ 8. Bookkeeper failed to record a check that was returned

with the bank statement__________ 9. Check deposited and returned by the bank

marked NSF__________10. Outstanding checks

HarrCh06v1.qxd 10/23/06 8:30 AM Page 187

Page 220: polpoa_sg

188 Chapter 6 | Quick Practice Questions

6-2. On November 1, 2008, Heather Station established a $300 pettycash fund. At the end of November the petty cash fund contained:

Cash on hand $ 48.00Petty cash tickets for:Postage $73.50Office supplies 87.55Miscellaneous items 90.95

252.00Total $300.00

a. Prepare the journal entry to establish the petty cash fund onNovember 1, 2008.

Journal Entry:

Date Accounts Dr. Cr.

b. Prepare the journal entry on November 30, 2008, to replenish thepetty cash fund.

Journal Entry:

Date Accounts Dr. Cr.

HarrCh06v1.qxd 10/23/06 8:30 AM Page 188

Page 221: polpoa_sg

Quick Practice Questions | Chapter 6 189

Journal Entry:

Date Accounts Dr. Cr.

Journal Entry:

Date Accounts Dr. Cr.

Journal Entry:

Date Accounts Dr. Cr.

Journal Entry:

Date Accounts Dr. Cr.

Journal Entry:

Date Accounts Dr. Cr.

6-3. Using the following information, journalize the necessaryentries after preparing the bank reconciliation for LouisBrothers on May 31, 2008. (Not all items will require an entry.)

a. Outstanding checks total $1,533.25.b. The bookkeeper recorded a $1,524 check as $15,240 in payment

of the current month’s rent.c. A deposit of $300 from a customer was credited to Louis Brothers for

$3,000 by the bank.d. A customer’s check for $1,380 was returned for nonsufficient funds.e. The bank service charge based on the bank statement is $70.

HarrCh06v1.qxd 10/23/06 8:30 AM Page 189

Page 222: polpoa_sg

190 Chapter 6 | Quick Practice Questions

Bank: Balance, September 30, 2008 $ Add: Less: Adjusted bank balance, September 30, 2008 $ Books: Balance, September 30, 2008 $ Add: Less: Adjusted book balance, September 30, 2008 $

BATTER COMPANYBank Reconciliation

September 30, 2008

6-4. The following data have been gathered for Batter Company to assist you in preparing the September 30, 2008, bank reconciliation:

a. The September 30 bank balance was $5,460.b. The bank statement included $30 of service charges.c. There was an EFT deposit of $1,800 on the bank statement for the

monthly rent due from a tenant.d. Checks #541 and #543, for $205 and $420, respectively, were not

among the canceled checks returned with the statement.e. The September 30 deposit of $3,800 did not appear on the bank

statement.f. The bookkeeper had erroneously recorded a $500 check as $5,000.

The check was payment for an amount due on account.g. Included with the canceled checks was a check written by Bitter

Company for $200, which was deducted from Batter Company’saccount.

h. The bank statement included an NSF check written by TateCompany for a $360 payment on account.

i. The Cash account showed a balance of $2,925 on September 30.

Prepare the September 30, 2008, bank reconciliation for Batter Company.

HarrCh06v1.qxd 10/23/06 8:30 AM Page 190

Page 223: polpoa_sg

6-5. The following data have been gathered for RagpickerCompany.

a. The service charges for February amount to $90.b. Outstanding checks amount to $650.c. The bank erroneously credited Ragpicker Company’s account for

$300 for a deposit made by another company.d. Check #665 for $3,000 for the cash purchase of office equipment was

erroneously recorded by the bookkeeper as $2,080.e. A deposit ticket correctly prepared for $975 appeared on the bank

statement as a deposit for $795.f. A customer’s check for $560 was returned with the bank statement

and stamped NSF.g. Check #650 for $125 for utilities expense was erroneously recorded

by the bookkeeper as $1,250.

Calculate the correct cash balance on February 28, 2008, by performing the part of the bank reconciliation beginning with the balance per bank as shown. (Note: Not all of the preceding data may be needed.)

Quick Practice Questions | Chapter 6 191

Bank: Balance, February 28, 2008 $7,975 Add: Less: Adjusted bank balance, February 28, 2008 $

RAGPICKER COMPANYBank Reconciliation

February 28, 2008

HarrCh06v1.qxd 10/23/06 8:30 AM Page 191

Page 224: polpoa_sg

192 Chapter 6 | Do It Yourself! Question 1

Bank Statement for May 2008

Balance, May 1, 2008 $4,500

Deposits

May 9 $600

May 18 1,000 1,600

Checks

May 4 900

May 12 2,300

May 22 1,500 (4,700)

Other items:

EFT—payment of loan payable (1,300)

NSF check from Bennet Smith (400)

Service charges (100)

EFT—monthly rent collection 1,200

Interest on account balance 50

Balance, May 31, 2008 $850

Do It Yourself! Question 1Bank Reconciliations

Quint Inc. has the following information for May 2008:

Cash

May 1 Bal. 4,500May 4 900

May 9 600May 12 2,300

May 18 1,000May 22 1,500

May 28 700May 30 500

May 31 Bal. 1,600

The loan payment includes principal of $950 and interest of $350.

The rent collection is from tenants leasing extra space in Quint’s office building.

Quint deposits all cash receipts and makes all payments by check.

HarrCh06v1.qxd 10/23/06 8:30 AM Page 192

Page 225: polpoa_sg

Do It Yourself! Question 1 | Chapter 6 193

Journal Entry:

Date Accounts Dr. Cr.

Requirements

1. Prepare Quint’s bank reconciliation at May 31, 2008.

2. Journalize any entries required by Quint and update Quint’s Cash T-account.Explanations are not required.

Journal Entry:

Date Accounts Dr. Cr.

HarrCh06v1.qxd 10/23/06 8:30 AM Page 193

Page 226: polpoa_sg

194 Chapter 6 | Do It Yourself! Question 1

Journal Entry:

Date Accounts Dr. Cr.

Journal Entry:

Date Accounts Dr. Cr.

Journal Entry:

Date Accounts Dr. Cr.

HarrCh06v1.qxd 10/23/06 8:30 AM Page 194

Page 227: polpoa_sg

Do ItYourself! Question 2 | Chapter 6 195

Journal Entry:

Date Accounts Dr. Cr.

Do It Yourself! Question 2

Petty Cash

Xander Co. established a $400 petty cash fund on May 1, 2008. On May 31, 2008,the petty cash box contained $80 cash and the following receipts:

May 6 Office supplies $75

May 13 Delivery charges 90

May 24 Pizza for office party 70

May 30 Office supplies 85

On May 31, 2008, the petty cash balance was replenished.

Requirements

1. Journalize the entry to establish the fund.

2. Journalize the entry to record the expenses incurred from petty cash duringMay. (Assume all charges are recorded as Supplies Expense, Delivery Expense, orMeals Expense.)

Journal Entry:

Date Accounts Dr. Cr.

HarrCh06v1.qxd 10/23/06 8:30 AM Page 195

Page 228: polpoa_sg

196 Chapter 6 | Quick Practice Solutions

Quick Practice SolutionsTrue/False

T 1. A deposit in transit has been recorded by the company but not by the bank. (p. 304)

T 2. An NSF check would be recorded on the books by debiting Accounts Receivable. (p. 308)

F 3. Employee embezzlement typically involves fraudulent financial reporting by management.False–Employee embezzlement is employee fraud involving misappropriation of assets, which occurs when employees steal from their employers. Management fraud typically involves fraudulent financialreporting by management. (p. 296)

T 4. An audit is the assessment of financial statements to determine whether they are fairly presented. (p. 299)

T 5. Different people should perform various accounting duties to minimize errors and the opportunities for fraud. (p. 297)

F 6. Funds disbursed from the petty cash fund will be recorded as a creditto the Petty Cash account.False–When the petty cash fund is replenished, funds disbursed fromthe fund will be recorded as a credit to the Cash account. No entries affect Petty Cash for the disbursement of funds. (p. 312)

F 7. The person who prepares checks for payment would be a suitable employee to reconcile the bank account.False–Responsibilities for the custody, approval, and accounting tasks should be held by separate employees. (pp. 297–298)

T 8. The party signing a check is referred to as the maker. (p. 303)

F 9. Outstanding checks would include only those checks written for the current month that have not cleared or been canceled by the bank.False–Outstanding checks include all checks written that have not cleared the bank. They could be from the current month or previous periods. (p. 304)

F 10. It is a good control to have just one person open the checks and deposit them in the bank.False–Separate individuals should be assigned custody, approval,and accounting tasks. (p. 301)

HarrCh06v1.qxd 10/23/06 8:30 AM Page 196

Page 229: polpoa_sg

Multiple Choice

1. Which of the following is not an objective of internal control? (p. 297)a. Help safeguard the assets a business uses in its operationsb. Guarantee a company will not go bankruptc. Encourage adherence to company policiesd. Promote operational efficiency

2. Which of the following items used to reconcile cash does notrequire an adjusting entry? (p. 304)a. Bank service chargeb. Interest earnedc. A note collected by the bankd. Deposits in transit

3. Which of the following statements about bank reconciliations iscorrect? (p. 310)a. Should not be prepared by an employee who handles cash transactionsb. Is part of a sound internal control systemc. Is a formal financial statementd. Both (a) and (b) are correct

4. Which of the following items does not cause a difference betweenthe cash balance per bank and book? (pp. 304–305)a. NSF checksb. Deposits in transitc. Outstanding checksd. Canceled checks

5. The following data are available for Wonder Boutique forOctober:Book balance, October 31 $5,575Outstanding checks 584Deposits in transit 2,500Service charges 75Interest revenue 25

What is the adjusted book balance on October 31 for WonderBoutique based on these data? (pp. 304–306)a. $5,500b. $5,525c. $5,550d. $7,466

6. The bank statement lists a $700 deposit as $70. On a bankreconciliation, the correction for this error will appear as whichof the following? (p. 306)a. Addition to the book balanceb. Deduction from the book balancec. Addition to the bank balanced. Deduction from the bank balance

Quick Practice Solutions | Chapter 6 197

HarrCh06v1.qxd 10/23/06 8:30 AM Page 197

Page 230: polpoa_sg

7. Which of the following processes for payment compresses theapproval for payment into a single step? (p. 303)a. Electronic funds transferb. Evaluated receipts settlementc. Electronic data interchanged. Rationalization

8. For which items must journal entries be prepared? (p. 306)a. Any errors made on the books revealed by the bank reconciliationb. Any errors made by the bank revealed by the bank reconciliationc. All items on the bank’s sided. Only for outstanding checks

9. Which of the following is not a control over petty cash? (p. 311)a. Keeping an unlimited amount of cash on handb. Supporting all fund disbursements with a petty cash ticketc. Replenishing the fund through normal cash disbursement proceduresd. Designating one employee to administer the fund

10. If the petty cash fund is not replenished on the balance sheetdate, which of the following will be true? (pp. 312–313)a. Assets will be overstatedb. Income will be overstatedc. Neither (a) nor (b)d. Both (a) and (b)

Quick Exercise Solutions

6-1. Classify each of the following reconciling items of the Breadand Butter Company as one of the following:

a. An addition to the bank balanceb. A deduction from the bank balancec. An addition to the book balanced. A deduction from the book balancee. Not a reconciling item

c 1. Collection of note receivable plus interest revenue by bankd 2. Bookkeeper recorded check #849 as $557 instead of the

correct amount of $755d 3. Bank service chargesc 4. Bank credited the account for interest revenueb 5. Bank added deposit to Bread and Butter’s account in errora 6. Deposits in transitb 7. Bank withdrew $1,270 from Bread and Butter’s account for

a check written for $12,700d 8. Bookkeeper failed to record a check that was returned with

the bank statementd 9. Check deposited and returned by the bank marked NSFb 10. Outstanding checks

198 Chapter 6 | Quick Practice Solutions

HarrCh06v1.qxd 10/23/06 8:30 AM Page 198

Page 231: polpoa_sg

6-2. On November 1, 2008, Heather Station established a $300 pettycash fund. At the end of November the petty cash fund con-tained:

Cash on hand $ 48.00Petty cash tickets for:Postage $73.50Office supplies 87.55Miscellaneous items 90.95

252.00Total $300.00

a. Prepare the journal entry to establish the petty cash fund onNovember 1, 2008.

Quick Practice Questions | Chapter 6 199

Journal Entry:

Nov. 1 Petty Cash 300 Cash in Bank 300 To establish the petty cash fund.

Date Accounts Dr. Cr.

b. Prepare the journal entry on November 30, 2008, to replenish thepetty cash fund.

Journal Entry:

Nov. 30 Postage Expense 73.50 Office Supplies 87.55 Miscellaneous Expense 90.95 Cash 252.00 To replenish petty cash.

Date Accounts Dr. Cr.

HarrCh06v1.qxd 10/23/06 8:30 AM Page 199

Page 232: polpoa_sg

6-3. Using the following information, journalize the necessaryentries after preparing the bank reconciliation for LouisBrothers on May 31, 2008. (Not all items will require an entry.)

a. Outstanding checks total $1,533.25.b. The bookkeeper recorded a $1,524 check as $15,240 in payment

of the current month’s rent.c. A deposit of $300 from a customer was credited to Louis Brothers for

$3,000 by the bank.d. A customer’s check for $1,380 was returned for nonsufficient funds.e. The bank service charge based on the bank statement is $70.

200 Chapter 6 | Quick Practice Solutions

Journal Entry:

Date Accounts Ref. Dr. Cr.

Journal Entry:

Date Accounts Ref. Dr. Cr.

Journal Entry:

Date Accounts Ref. Dr. Cr.

Journal Entry:

Date Accounts Ref. Dr. Cr.

a. No entry required

b. May 31 Cash 13,716 Rent Expense 13,716

c. No entry required

d. May 31 Accounts Receivable 1,380 Cash 1,380

HarrCh06v1.qxd 10/23/06 8:30 AM Page 200

Page 233: polpoa_sg

6-4. The following data have been gathered for Batter Company to assist you in preparing the September 30, 2008, bank reconciliation:

a. The September 30 bank balance was $5,460.b. The bank statement included $30 of service charges.c. There was an EFT deposit of $1,800 on the bank statement for the

monthly rent due from a tenant.d. Checks #541 and #543, for $205 and $420, respectively, were not

among the canceled checks returned with the statement.e. The September 30 deposit of $3,800 did not appear on the bank

statement.f. The bookkeeper had erroneously recorded a $500 check as $5,000.

The check was payment for an amount due on account.g. Included with the canceled checks was a check written by Bitter

Company for $200, which was deducted from Batter Company’saccount.

h. The bank statement included an NSF check written by TateCompany for a $360 payment on account.

i. The cash account showed a balance of $2,925 on September 30.

Prepare the September 30, 2008, bank reconciliation for BatterCompany.

Quick Practice Solutions | Chapter 6 201

Bank: Balance, September 30, 2008 $5,460 Add: Deposit in transit $3,800 Bank error—Batter Co. check 200 4,000 Less: Outstanding checks Check #541 205 Check #543 420 (625) Adjusted bank balance, September 30, 2008 $8,835 Books: Balance, September 30, 2008 $2,925 Add: EFT rent deposit $1,800 Bookkeeper error ($5,000 – 500) 4,500 6,300 Less: Bank service charge 30 NSF check 360 (390) Adjusted book balance, September 30, 2008 $8,835

BATTER COMPANYBank Reconciliation

September 30, 2008

HarrCh06v1.qxd 10/23/06 8:30 AM Page 201

Page 234: polpoa_sg

6-5. The following data have been gathered for RagpickerCompany.

a. The service charges for February amount to $90.b. Outstanding checks amount to $650.c. The bank erroneously credited Ragpicker Company’s account for

$300 for a deposit made by another company.d. Check #665 for $3,000 for the cash purchase of office equipment was

erroneously recorded by the bookkeeper as $2,080.e. A deposit ticket correctly prepared for $975 appeared on the bank

statement as a deposit for $795.f. A customer’s check for $560 was returned with the bank statement

and stamped NSF.g. Check #650 for $125 for utilities expense was erroneously recorded

by the bookkeeper as $1,250.

Calculate the correct cash balance on February 28, 2008, by performingthe part of the bank reconciliation beginning with the balance per bankas shown. (Note: Not all of the preceding data may be needed.)

202 Chapter 6 | Quick Practice Solutions

Bank: Balance, February 28, 2008 $7,975 Add: Bank error—deposit of $975 recorded as $795 180 8,155 Less: Outstanding checks $650 Bank error 300 (950) Adjusted bank balance, February 28, 2008 $7,205 Note: Remember that the adjusted bank balance is the correct book balance.

RAGPICKER COMPANYBank Reconciliation

February 28, 2008

HarrCh06v1.qxd 10/23/06 8:30 AM Page 202

Page 235: polpoa_sg

Do It Yourself! Question 1 Solutions | Chapter 6 203

Bank: Balance, May 31, 2008 $850 Add: May 28 deposit in transit 700 1,550 Less: May 30 outstanding check (500) Adjusted bank balance, May 31, 2008 $1,050 Books: Balance, May 31, 2008 $1,600 Add: Bank collection of rent 1,200 Interest earned on account 50 2,850 Less: Mortgage payment $1,300 NSF—B. Smith 400 Service charge 100 (1,800) Adjusted book balance, May 31, 2008 $1,050

QUINT INC.Bank Reconciliation

May 31, 2008

Do It Yourself! Question 1 Solutions

Requirement 1

Prepare Quint’s bank reconciliation at May 31, 2008.

HarrCh06v1.qxd 10/23/06 8:30 AM Page 203

Page 236: polpoa_sg

204 Chapter 6 | Do It Yourself! Question 1 Solutions

Cash

May 31 Bal. 1,6001,200

50May 31 950

350400100

May 31 Bal. 1,050

Journal Entry:

May 31 Cash (Asset,1; debit) 1,200 Rent Revenue (Revenue,1; credit) 1,200 May 31 Cash (Asset,1; debit) 50 Interest Revenue (Revenue,1; credit) 50 May 31 Mortgage Payable (Liability,2; debit) 950 Cash (Asset,2; credit) 950

May 31 Interest Expense (Expense,1; credit) 350 Cash (Asset,2; credit) 350 May 31 Accounts Receivable (Asset,1; debit) 400 Cash (Asset,2; credit) 400 May 31 Miscellaneous Expense (Expense,1; debit) 100 Cash (Asset,2; credit) 100

Date Accounts Dr. Cr.

Requirement 2

Journalize any entries required by Quint and update Quint’s Cash T-account.Explanations are not required.

HarrCh06v1.qxd 10/23/06 8:30 AM Page 204

Page 237: polpoa_sg

Do It Yourself! Question 2 Solutions | Chapter 6 205

Journal Entry:

May 1 Petty Cash (Asset,1; debit) 400 Cash in Bank (Asset,2; credit) 400

Date Accounts Dr. Cr.

Journal Entry:

May 31 Office Supplies ($75 + $85) (Expense,1; debit) 160 Delivery Expense (Expense,1; debit) 90 Meals Expense (Expense,1; debit) 70 Cash in Bank ($400 – $80) (Asset,2; credit) 320

Date Accounts Dr. Cr.

Do It Yourself! Question 2 Solutions

Requirement 1

Journalize the entry to establish the fund.

Requirement 2

Journalize the entry to record the expenses incurred from petty cash during May.

HarrCh06v1.qxd 10/23/06 8:30 AM Page 205

Page 238: polpoa_sg

The Power of PracticeFor more practice using the skills learned in this chapter, visit MyAccountingLab.There you will find algorithmically generated questions that are based on theseDemo Docs and your main textbook’s Review and Accounting Practice sections.

To go to MyAccountingLab and follow these steps:

1. Direct your URL to www.myaccountinglab.com.2. Log in using your name and password.3. Click the MyAccountingLab link.4. Click Study Plan in the left navigation bar.5. From the table of contents, select Chapter 6, Internal Control and Cash.6. Click a link to work on the tutorial exercises.

206 Chapter 6 | The Power of Practice

HarrCh06v1.qxd 10/23/06 8:30 AM Page 206

Page 239: polpoa_sg

Receivables7

1

WHAT YOU PROBABLY ALREADY KNOW

You probably already know that if a friend borrows money fromyou, you may not be repaid. You would not loan the money if youdidn’t believe that your friend is creditworthy and will likely repaythe debt. However, until the money is received, you have noguarantee. If the friend asks to borrow more money beforerepaying the original loan, you may be more likely to refusebecause your risk of nonpayment is increased. Your friend has nohistory of successful repayment yet. If the friend never pays, youincur a loss equal to the amount of the loan.

The same concerns exist for a business. Sales on account aremade only after a customer is approved by the business’s creditdepartment. Despite the most thorough investigation, somecustomers still may not pay the amount due. The uncollectibleaccounts receivable results in a reduction to the asset and to netincome.

Learning Objectives

Describe the types of sales and receivables and discuss therelated internal controls.

By now, you should understand that whether a business sells goods orservices, customers may pay in several different ways: Cash, credit,debit, and bank card sales are all possibilities. The account results in areceivable—either an account receivable or a note receivable. Oneimportant feature of a strong system of internal control over bothcollections and credit management is to separate responsibility for thecustody of assets from the accounting and operating departments. Theindividual handling cash should not be granting credit, nor should thatperson be accounting for receivables. Understanding the main issuesassociated with extending credit and handling receivables is animportant lesson from this chapter.

Use the direct write-off method to account for uncollectiblereceivables.

The direct write-off method is simple to employ, but the method is notin accordance with GAAP. No estimate of the uncollectible accountsexpense is journalized. When it is determined which customer’sreceivable is uncollectible, the following entry is journalized:

2

1eSG_C07_0131792075.QXD 10/23/06 8:51 AM Page 207

Page 240: polpoa_sg

208 Chapter 7 | Receivables

3

4

5

Quick ratio = Cash ! Short-term investments ! Net current receivables

Total current liabilities

Check out “The Direct Write-Off Method” in the main text.

Use the allowance method to account for uncollectible receivables.

The allowance method matches the sales revenues with theuncollectible accounts expense (also known as the bad debt expense).An estimate of the uncollectible accounts expense must be made in theperiod of sale using either the aging of receivables or the percent-of-sales methods. The entry required at the end of the period is:

Journal Entry:

Uncollectible Accounts Expense X Allowance for Uncollectible Accounts X

PostDate Accounts Ref. Dr. Cr.

Journal Entry:

Uncollectible Accounts Expense X Accounts Recievable X

PostDate Accounts Ref. Dr. Cr.

The Allowance for Uncollectible Accounts is a contra-asset account. Thisaccount is credited, rather than Accounts Receivable, because it isunknown on the entry date which specific customers will eventually notpay. When it is determined which customer’s receivable is uncollectible,the Allowance account is reduced (debited) and the specific customeraccounts receivable is reduced (credited). Accounting for uncollectiblescan be a challenging concept, so be sure that you have a goodunderstanding of this topic before moving on to the next chapter.

Account for notes receivable.A note receivable is a formal written promise to pay the amountborrowed by the debtor plus interest. Interest must be journalized forthe period of indebtedness. Study the key components of a note inExhibit 7-7 (p. 364). Review “Computing Interest on a Note”, “Accountingfor Notes Receivable”, and “Accruing Interest Revenue” in the main text.

Calculate the quick ratio and days’ sales in receivables.A measure of liquidity is the quick ratio (also called the acid-test ratio).The current assets most quickly converted into cash are compared to thetotal current liabilities. A higher result is usually more favorable. Theratio is calculated as follows:

The days’ sales in receivables indicate the number of days it takes onaverage to collect from customers. The objective is to minimize thecollection period. Review the ratio computations in “More Ratios forDecision Making” in the main text.

1eSG_C07_0131792075.QXD 10/23/06 8:51 AM Page 208

Page 241: polpoa_sg

Demo Doc 1 | Chapter 7 209

Uncollectible Accounts ReceivableLearning Objectives 1–3, 5

Hart Inc.’s December 31, 2007, balance sheet reported:

Accounts receivable $800Allowance for uncollectible accounts ($40)Accounts receivable (net) $760

Requirements

1. Is Hart using the allowance method or the direct write-off method toaccount for uncollectible receivables? How much of the December 31, 2007,balance of accounts receivable did Hart expect to collect?

2. During 2008, Hart wrote off accounts receivables totaling $35 from AmandaBlake. Journalize these write-offs as one transaction. How does thistransaction affect the net accounts receivable balance? How would thistransaction have been journalized if the direct write-off method was beingused?

3. During 2008, Hart earned $2,800 of service revenues, all on account.Journalize these revenues as one transaction.

4. During 2008, Hart collected $2,745 cash from customers. Journalize thistransaction and calculate the gross accounts receivable balance atDecember 31, 2008.

5. Assume that Hart estimates uncollectible accounts expense to be 1.5% ofrevenues. Journalize the entry to adjust the allowance at December 31,2008. What is the December 31, 2008 adjusted balance in the allowanceaccount?

6. (Ignore Requirement 5.) Assume that Hart has the following information atDecember 31, 2008:

Gross Accounts Percentage Estimated Age Receivable Uncollectible

< 30 days $100 2%

30–60 days 500 4%

> 60 days 220 10%

Total $820

Journalize the entry to adjust the allowance at December 31, 2008.What isthe December 31, 2008, balance in the allowance? Show how accountsreceivable would be reported on the balance sheet at December 31, 2008.

7. Calculate Hart’s day’s sales in accounts receivable for 2008 (assume Hartuses the aging-of-accounts receivable method in Requirement 6). Whatdoes this ratio mean?

8. The employee at Hart who opens the mail and physically collects the cashis the same person who updates the cash receipts journal and accountsreceivable ledger. Is this system a good practice of internal control?

Demo Doc 1

1eSG_C07_0131792075.QXD 10/23/06 8:51 AM Page 209

Page 242: polpoa_sg

210 Chapter 7 | Demo Doc 1 Solutions

Part 1 Part 2 Part 3 Part 4 Part 5 Part 6 Part 7 Part 8Demo DocComplete

Part 1 Part 2 Part 3 Part 4 Part 5 Part 6 Part 7 Part 8Demo DocComplete

Journal Entry:

Allowance for Uncollectible Accounts (Contra-Asset,2; debit) 35 Accounts Receivable—Amanda Blake (Asset,2; credit) 35

PostDate Accounts Ref. Dr. Cr.

Demo Doc 1 SolutionsRequirement 1

Is Hart using the allowance method or the direct write-off method to accountfor uncollectible receivables? How much of the December 31, 2007, balance ofaccounts receivable did Hart expect to collect?

Use the direct write-off method to accountfor uncollectiblereceivables

2

Use the allowancemethod to account for uncollectiblereceivables

3

Hart is using the allowance method, which is indicated by an allowance for uncol-lectible accounts. If Hart were using the direct write-off method, Hart would notneed an allowance for uncollectible accounts.

Gross accounts receivable is the total amount of receivables that exist. ForHart, this amount is $800. The allowance is (by definition) the amount of receiv-ables we do not expect to collect.

The total receivables minus the amount we do not expect to collect (that is,the gross accounts receivable minus the allowance) is the amount we do expect tocollect (that is, the net accounts receivable).

Hart expects to collect $760 of the accounts receivable.

Requirement 2

During 2008, Hart wrote off accounts receivables totaling $35 from AmandaBlake. Journalize these write-offs as one transaction. How does this transactionaffect the net accounts receivable balance? How would this transaction havebeen journalized if the direct-write-off method was being used?

Writing off an account receivable means removing it from the accountingbooks/records because it has been determined that this specific amount will notbe collected. We must reduce (credit) the Accounts Receivable. Additionally, nowthat we find that one of the accounts will not be collected, we can take it out ofour estimate of uncollectible accounts (the Allowance for Uncollectible Accounts),which decreases this account (a debit).

Use the direct write-off method to accountfor uncollectiblereceivables

2

Use the allowancemethod to account for uncollectiblereceivables

3

Use this standard format to write off accounts receivable when using theallowance method.The entry structure is always the same, only the amount changes.

1eSG_C07_0131792075.QXD 10/23/06 8:51 AM Page 210

Page 243: polpoa_sg

Demo Doc 1 Solutions | Chapter 7 211

Note that this entry does not change the net accounts receivable. Grossaccounts receivable decreases, but so does the allowance so overall the changeis zero:

Gross accounts receivable change of – 35– Allowance for uncollectible accounts (change of – 35)Accounts receivable (net) no change

The impact of this transaction is:

Gross accounts receivable $800 – $35 " $765– Allowance for uncollectible accounts ($40 – $35 " $ 5)Accounts receivable (net) $760 – $0 " $760(same amount to collect)

Under the direct write-off method, no allowance is used, so the debit in the write-offentry increases Uncollectible Accounts Expense.

Journal Entry:

Uncollectible Accounts Expense (Expense,1; debit) 35 Accounts Receivable—Amanda Blake (Asset,2; credit) 35

PostDate Accounts Ref. Dr. Cr.

Requirement 3

During 2008, Hart earned $2,800 of service revenues, all on account. Journalizethese revenues as one transaction.

Part 1 Part 2 Part 3 Part 4 Part 5 Part 6 Part 7 Part 8Demo DocComplete

When revenues are earned, we increase the Revenue account (a credit). In thiscase, we are not receiving cash, so instead we increase Accounts Receivable (a debit) to show that we intend to collect this amount later from our customer(s).

Journal Entry:

Accounts Receivable (Asset, 1; debit) 2,800 Service Revenue (Revenue, 1; credit) 2,800

PostDate Accounts Ref. Dr. Cr.

1eSG_C07_0131792075.QXD 10/23/06 8:51 AM Page 211

Page 244: polpoa_sg

212 Chapter 7 | Demo Doc 1 Solutions

When cash is collected, we increase the Cash account (a debit) and decreaseAccounts Receivable (a credit).

Journal Entry:

Cash (Asset, 1; debit) 2,745 Accounts Receivable (Asset, 2; credit) 2,745

PostDate Accounts Ref. Dr. Cr.

From the initial data given in the question, we can see that gross accountsreceivable had a balance of $800 at the beginning of the year ($760 is the net bal-ance). Accounts Receivable increased in the year as revenues were earned.Accounts Receivable decreased when uncollectible accounts were written off andwhen cash was collected. Using this information, we can calculate the ending bal-ance in (gross) Accounts Receivable:

Accounts Receivable

Dec. 31, 2007 Bal. 800

2008 Revenues 2,800 2008 Write-Offs 35

2008 Cash Collections 2,745

Dec 31, 2008 Bal. 820

Requirement 4

During 2008, Hart collected $2,745 cash from customers. Journalize thistransaction and calculate the gross accounts receivable balance atDecember 31, 2008.

Part 1 Part 2 Part 3 Part 4 Part 5 Part 6 Part 7 Part 8Demo DocComplete

Requirement 5

Assume that Hart estimates uncollectible accounts expense to be 1.5% ofrevenues. Journalize the entry to adjust the allowance at December 31, 2008.What is the December 31, 2008 adjusted balance in the allowance account?

Part 1 Part 2 Part 3 Part 4 Part 5 Part 6 Part 7 Part 8Demo DocComplete

The problem states, “Hart estimates uncollectible accounts expense to be 1.5% ofrevenues.” The key phrase here is “1.5% of revenues,” which tells us that Hart isusing the percentage-of-sales method to calculate the expense and allowance.

Use the direct write-off method to accountfor uncollectiblereceivables

2

1eSG_C07_0131792075.QXD 10/23/06 8:51 AM Page 212

Page 245: polpoa_sg

Demo Doc 1 Solutions | Chapter 7 213

Use this standard journal entry format to journalize uncollectible accountsexpense and adjust the allowance. The entry structure is always the same, onlythe amount changes.

The balance in the Allowance account must be calculated. The beginning bal-ance in the allowance for 2008 is the ending balance for 2007 (the $40 shown atthe beginning of the question, as shown in the balance sheet presentation on p. 209). During the year, write-offs will decrease the allowance ($35, as inRequirement 2) and the year-end adjustment will increase it ($42, as in thisrequirement). We can fill in this information to calculate an ending balance of$47 in the Allowance account.

Allowance for Uncollectible Accounts

Dec. 31, 2007 Bal. 40

2008 Write-Offs 35

2008 Uncollectible AccountsExpense Adjustment 42

Dec. 31, 2008 Bal. 47

Journal Entry:

Uncollectible Accounts Expense (Expense,1; debit) 42 Allowance for Uncollectible Accounts (Contra-Asset,1; credit) 42

PostDate Accounts Ref. Dr. Cr.

Under the percentage-of-sales method, the percentage of sales equals theuncollectible accounts expense. Therefore, we can calculate that 1.5% of $2,800,or $42, equals the uncollectible accounts expense. We then journalize $42 ofUncollectible Accounts Expense.

Journalizing the Uncollectible Accounts Expense increases that account (adebit) and also increases the total estimate of uncollectible accounts: theAllowance (a credit).

Requirement 6

(Ignore Requirement 5.) Assume that Hart has the following information atDecember 31, 2008:

Gross Account Percentage Estimated Age Receivable Uncollectible

< 30 days $100 2%

30–60 days 500 4%

> 60 days 220 10%

Total $820

1eSG_C07_0131792075.QXD 10/23/06 8:51 AM Page 213

Page 246: polpoa_sg

We need an additional credit in the T-account to make it balance (to makethe total correct).

We can use the $44 ending balance in the T-account (along with the beginningbalance of $40, the write-offs of $35, and the $5 credit balance) to calculate theuncollectible accounts expense of $39. This amount must be used in the journalentry ($44 required balance – $5 credit balance = $39 amount for journal entry).

Gross Percentage Estimated Amount EstimatedAccounts Uncollectible Uncollectible

Age Receivable

< 30 days $100 ! 2% = $ 2

30–60 days 500 ! 4% = 20

> 60 days 220 ! 10% = 22

Total $820 $44 Ending Allowance Balance

Allowance for Uncollectible Accounts

Dec. 31, 2007 Balance 40

2008 Write-Offs 35

2008 Uncollectible AccountsExpense Adjustment X

Dec. 31, 2008 Target Bal. 44

So

40 – 35 ! X " 44X " 44 – 40 ! 35 " 39

Journalize the entry to adjust the allowance at December 31, 2008.What is theDecember 31, 2008, balance in the allowance? Show how accounts receivablewould be reported on the balance sheet at December 31, 2008.

Part 1 Part 2 Part 3 Part 4 Part 5 Part 6 Part 7 Part 8Demo DocComplete

The problem does not explicitly state which method is being used; however, thetable clearly shows estimated uncollectible percentages of accounts receivable.This information shows us that Hart is using the aging-of-accounts receivablemethod to calculate the allowance and then the expense.

Under the aging-of-accounts receivables method, the percentage of receiv-ables equals the ending balance in the allowance. Therefore, we can calculatethat (2% ! $100) + (4% ! $500) + (10% ! $220) = $44, which is the required (or tar-get) ending balance in the allowance.

Use the direct write-off method to accountfor uncollectiblereceivables

2

Journal Entry:

Uncollectible Accounts Expense (Expense,1; debit) 39 Allowance for Uncollectible Accounts (Contra-Asset,1; credit) 39

PostDate Accounts Ref. Dr. Cr.

1eSG_C07_0131792075.QXD 10/23/06 8:51 AM Page 214

Page 247: polpoa_sg

Demo Doc 1 Solutions | Chapter 7 215

Part 1 Part 2 Part 3 Part 4 Part 5 Part 6 Part 7 Part 8Demo DocComplete

The days’ sales ratio is calculated as: Calculate the quickratio and days’ sales inreceivables

5

Days’ sales in average"

Average net accounts receivable x 365 daysaccounts receivable Net sales

From Requirement 3, we know that service revenues for 2008 are $2,800.From Requirement 6, we know that net accounts receivables were $760 onDecember 31, 2007, and $776 on December 31, 2008.

So for Hart:

Days’ sales in average"

1/2 # ($760 ! $776) # 365 daysaccounts receivable $2,800

" (1/2 # $1,536) # 365 days$2,800

" $768 # 365 days$2,800

" $280,320$2,800

For Hart, the average amount of time that it takes to collect an accountreceivable is 100.1 days (more than three months).

Days’ sales in average accounts receivable = 100.1 days

Use this standard journal entry format to journalize uncollectible accountsexpense and adjust the allowance. The entry structure is always the same, onlythe amount changes.

On the balance sheet, we would see the gross accounts receivable combinedwith the Allowance contra-account:

Accounts receivable $820

– Allowance for uncollectible accounts (44)Accounts receivable (net) $776

Requirement 7

Calculate Hart’s day’s sales in accounts receivable for 2008 (assume Hart usesthe aging-of-accounts receivable method in Requirement 6). What does thisratio mean?

1eSG_C07_0131792075.QXD 10/23/06 8:51 AM Page 215

Page 248: polpoa_sg

216 Chapter 7 | Demo Doc 1 Solutions

Part 1 Part 2 Part 3 Part 4 Part 5 Part 6 Part 7 Part 8Demo DocComplete

Part 1 Part 2 Part 3 Part 4 Part 5 Part 6 Part 7 Part 8Demo DocComplete

If an employee collects the cash and journalizes the receipt of the cash andupdates the accounts receivable ledger, the opportunity for fraud increasesgreatly. The employee could steal the cash and delay journalizing the cashreceipt, or perhaps never journalize the cash receipt. The employee could alsohide this dishonest act for a long period of time by manipulating the accountsreceivable ledger.

To avoid this problem, most internal control systems require separation ofduties; that is, employees who handle cash (both receipts and payments) are notthe same employees who maintain the accounting records.

Describe the types ofsales and receivablesand discuss the relatedinternal controls

1

Requirement 8

The employee at Hart who opens the mail and physically collects the cash isthe same person who updates the cash receipts journal and accountsreceivable ledger. Is this system a good practice of internal control?

1eSG_C07_0131792075.QXD 10/23/06 8:51 AM Page 216

Page 249: polpoa_sg

Demo Doc 1 | Chapter 7 217

Demo Doc 2Notes ReceivableLearning Objective 4

On November 1, 2008, Jordan Inc. borrowed $1,800 cash from Donald Corp.Jordan signed a three-month, 10% note. Jordan paid the note plus interest in fullon the due date. Both Jordan and Donald have a December 31 year-end.

Requirements

1. When is the note due? What is the total interest that will be paid on thisnote? What is its maturity value?

2. Prepare all journal entries for this note for both companies from November1, 2008, through the due date. Explanations are not required.

1eSG_C07_0131792075.QXD 10/23/06 8:51 AM Page 217

Page 250: polpoa_sg

218 Chapter 7 | Demo Doc 2 Solutions

Demo Doc 2 SolutionsRequirement 1

When is the note due? What is the total interest that will be paid on this note?What is its maturity value?

Demo DocCompletePart 1 Part 2

The note was issued on November 1, 2008. Because it is a three-month note, it isdue three months from that date, on February 1, 2009.

The amount of interest incurred over the entire life of the note is calculated as:

Account for notesreceivable

4

Interest"

Amount # Annual interest # Time elapsedincurred of debt rate (in years)

So in this case:

Interest" $1,800 # 10% #

3 monthsincurred 12 months

" $45

The maturity value is calculated as:

Maturity value " Principal ! Interest incurred over life of the note

So in this case:

Maturity value " $1,800 ! $45 " $1,845

Requirement 2

Prepare all journal entries for this note for both companies from November 1,2008, through the due date. Explanations are not required.

Demo DocCompletePart 1 Part 2

November 1, 2008 Jordan borrowed $1,800 from Donald.Jordan borrowed cash from Donald. Donald decreases (a credit) Cash by $1,800.Because Donald can expect to get this money back (that is, collect it) in thefuture, we can also set up a Notes Receivable asset (a debit) for $1,800.

Donald:

Account for notesreceivable

4

Journal Entry:

Nov. 1 Note Receivable (Asset,1; debit) 1,800 Cash (Asset,2; credit) 1,800

PostDate Accounts Ref. Dr. Cr.

1eSG_C07_0131792075.QXD 10/23/06 8:51 AM Page 218

Page 251: polpoa_sg

Demo Doc 2 | Chapter 7 219

Jordan increases Cash (a debit) and because the money must be paid back, wecan set up a Notes Payable liability (a credit) for $1,800.

Jordan:

Journal Entry:

Nov. 1 Cash (Asset,1; debit) 1,800 Note Payable (Liability,1; credit) 1,800

PostDate Accounts Ref. Dr. Cr.

December 31, 2008 Accrued 10% interest on note.Both companies have a December 31 year-end and need to adjust their

accounting information on this date. By this time, the note has been outstandingfor two months, which means interest has been incurred on the note.

The amount of interest incurred is calculated as:

Interest " Amount # Annual interest # Time elapsedincurred of debt rate (in years)

So in this case:

Note that all interest rates given are assumed to be annual rates, unless specifi-cally stated otherwise.

Donald earned interest revenue (a credit) of $30. Because the cash has not yetbeen received, we must also set up an Interest Receivable account (a debit) of $30.

Donald:

Journal Entry:

Dec. 31 Interest Receivable (Asset,1; credit) 30 Interest Revenue ($1,800 ! 10% ! 2/12) (Revenue,1; debit) 30

PostDate Accounts Ref. Dr. Cr.

Jordan incurred interest expense (a debit) of $30. Because the cash has yet to bepaid, we must also set up an Interest Payable account (a credit) of $30.

Jordan:

Journal Entry:

Dec. 31 Interest Expense ($1,800 ! 10% ! 2/12) (Expense,1; debit) 30 Interest Payable (Liability,1; credit) 30

PostDate Accounts Ref. Dr. Cr.

Interest " $1,800 # 10% # 2 monthsincurred 12 months

" $30

1eSG_C07_0131792075.QXD 10/23/06 8:51 AM Page 219

Page 252: polpoa_sg

220 Chapter 7 | Demo Doc 2 Solutions

February 1, 2009 Paid note and interest in full.On this day, the note and interest are fully paid. For Donald, Notes Receivable isdecreased (a credit) by $1,800. Additionally, Donald receives the interest accruedon December 31, so Interest Receivable is decreased (a credit) by $30.

However, more interest is involved. Donald also earned interest betweenDecember 31 and February 1 (one month):

So Donald journalizes Interest Revenue (a credit) of $15.All of these amounts are being paid in cash, so Donald’s Cash account is

increased (a debit) by $1,800 + $30 + $15 = $1,845 (the maturity value).

Donald:

Journal Entry:

Feb. 1 Cash (maturity value) (Asset,1; debit) 1,845 Note Receivable (Asset,2; credit) 1,800 Interest Revenue ($1,800 ! 10% ! 2/12) (Revenue,1; credit) 15 Interest Receivable (Asset,2; credit) 30

PostDate Accounts Ref. Dr. Cr.

Journal Entry:

Feb. 1 Note Payable (Liability,2; debit) 1,800 Interest Payable (Liability,2; debit) 30 Interest Expense ($1,800 ! 10% ! 2/12) (Expense,1; debit) 15 Cash (to balance) (Asset,2; credit) 1,845

PostDate Accounts Ref. Dr. Cr.

With payment of the note and interest, Jordan will decrease Notes Payable by$1,800 and Interest Payable by $30 (debits). Jordan will also journalize addi-tional interest expense of $15 (a debit) and decrease Cash (a credit) by $1,845.

Jordan:

Part 1 Part 2 Demo DocComplete

Interest " $1,800 # 10% # 1 monthsincurred 12 months

" $15

1eSG_C07_0131792075.QXD 10/23/06 8:51 AM Page 220

Page 253: polpoa_sg

Quick Practice Questions | Chapter 7 221

Quick Practice QuestionsTrue/False

_____ 1. The Allowance for Uncollectible Accounts is a contra-account toAccounts Receivable.

_____ 2. Under the allowance method, the recovery of an account previouslywritten off has no effect on net income.

_____ 3. Under the allowance method, the entry to write off an account that isdetermined to be uncollectible includes a credit to Allowance forUncollectible Accounts.

_____ 4. Under the allowance method, the entry to write off an account deemeduncollectible has no effect on the total assets of the firm.

_____ 5. The direct write-off method is the preferred way to apply the accrualbasis for measuring uncollectible account expense because it matchesrevenues and expenses on the income statement.

_____ 6. Under the direct write-off method, the entry to write off an accountdeemed uncollectible has no effect on the total assets of the firm.

_____ 7. A written promise to pay a specified amount of money at a particularfuture date is referred to as a promissory note.

_____ 8. If the maker of a note does not pay at maturity, the maker is said todishonor the note.

_____ 9. The quick ratio includes cash, inventory, and net accounts receivable inthe numerator.

_____10. Retailers must pay a small fee for accepting credit cards from cus-tomers as payment for goods.

1eSG_C07_0131792075.QXD 10/23/06 8:51 AM Page 221

Page 254: polpoa_sg

222 Chapter 7 | Quick Practice Questions

Multiple Choice

1. Chuck Battle’s account of $5,000 must be written off. Which ofthe following would be journalized assuming that the allowancemethod is used?a. A debit to Battle’s Accounts Receivable and a credit to Allowance for

Uncollectible Accountsb. A debit to Allowance for Uncollectible Accounts and a credit to

Battle’s Accounts Receivablec. A debit to Cash and a credit to Uncollectible Accounts Expensed. A debit to Cash and a credit to Battle’s Accounts Receivable

2. The current credit balance in Allowance for UncollectibleAccounts before adjustment is $658. An aging schedule reveals$3,700 of uncollectible accounts. What is the ending balance inthe Allowance for Uncollectible Accounts?a. $3,042b. $3,700c. $4,029d. $4,358

3. The current debit balance in Allowance for UncollectibleAccounts before adjustment is $742. An aging schedule reveals$3,500 of uncollectible accounts. What is the amount of thejournal entry for Estimated Uncollectible Accounts?a. $ 742b. $2,758c. $3,500d. $4,242

4. What is the type of account and normal balance of Allowance forUncollectible Accounts?a. Asset, debitb. Contra-asset, creditc. Liability, creditd. Contra-liability, debit

5. If the direct write-off method is used for uncollectiblereceivables, what account is debited when writing off acustomer’s account?a. Accounts Receivableb. Allowance for Uncollectible Accountsc. Uncollectible Accounts Expensed. Sales Returns and Allowances

6. What is the effect on the financial statements of writing off anuncollectible account under the direct write-off method?a. Increases expenses and decreases liabilitiesb. Decreases net income and decreases assetsc. Decreases assets and increases owner’s equityd. Increases expenses and increases assets

7. A 90-day note dated August 26 matures on which of thefollowing dates?a. November 24b. November 23c. November 25d. November 26

1eSG_C07_0131792075.QXD 10/23/06 8:51 AM Page 222

Page 255: polpoa_sg

8. A 90-day, 12% note for $20,000, dated July 10, is received from acustomer. What is the maturity value of the note?a. $20,000b. $20,600c. $21,200d. $22,400

9. Carolina Supply accepted an 8-month, $16,000 note receivable,with 8% interest, from Reading Corporation on August 1, 2008.Carolina Supply’s year-end is December 31. What is the amountof interest to be accrued on December 31, 2008?a. $ 320b. $ 533c. $ 853d. $1,280

10. Which of the following is journalized on the payee’s books whena debtor dishonors a note receivable?a. Debit Uncollectible Accounts Expenseb. Debit Accounts Receivablec. No entry requiredd. Debit Notes Receivable

Quick Practice Questions | Chapter 7 223

1eSG_C07_0131792075.QXD 10/23/06 8:51 AM Page 223

Page 256: polpoa_sg

Quick Exercises

7-1. Prepare the adjusting journal entry on December 31, 2008, forthe following independent situations:

a. The Allowance for Uncollectible Accounts shows a $700 credit bal-ance prior to adjustment. Net credit sales during the year are$216,000 and 4% are estimated to be uncollectible.

224 Chapter 7 | Quick Practice Questions

Journal Entry:

PostDate Accounts Ref. Dr. Cr.

Journal Entry:

PostDate Accounts Ref. Dr. Cr.

b. The Allowance for Uncollectible Accounts shows a $500 credit bal-ance prior to adjustment. An aging schedule prepared on December31 reveals an estimated uncollectible accounts amount of $7,300.

c. The Allowance for Uncollectible Accounts shows a $525 debit bal-ance prior to adjustment. An aging schedule prepared on December31 reveals an estimated uncollectible accounts amount of $5,100.

Journal Entry:

PostDate Accounts Ref. Dr. Cr.

d. The Allowance for Uncollectible Accounts shows an $800 credit bal-ance prior to adjustment. Net credit sales during the year are$229,000 and 3.5% are estimated to be uncollectible.

1eSG_C07_0131792075.QXD 10/23/06 8:51 AM Page 224

Page 257: polpoa_sg

7-2. Compute the ending balance in the Allowance for UncollectibleAccounts after the adjusting entries in Exercise 7-1 have beenprepared for the four independent situations, a – d.a. ___________b. ___________c. ___________d. ___________

7-3. Journalize the following independent transactions assumingthe allowance method is used.a. August 5, 2008 Wrote off Jones Corp.’s account receivable for

$2,200 as uncollectible.b. August 17, 2008 Collected the $2,200 from Jones Corp. in full.c. August 31, 2008 Recorded Uncollectible Accounts Expense of

$16,500.

Journal Entry:

PostDate Accounts Ref. Dr. Cr.

Journal Entry:

Journal Entry:

Journal Entry:

Journal Entry:

PostDate Accounts Ref. Dr. Cr.

1eSG_C07_0131792075.QXD 10/23/06 8:51 AM Page 225

Page 258: polpoa_sg

226 Chapter 7 | Quick Practice Questions

7-4. On December 31, 2008, Rainbow Appliances has $275,000 inAccounts Receivable and an Allowance account with a creditbalance of $240. Current period net credit sales were $771,000and cash sales were $68,000.

Rainbow Appliances performs an aging schedule, and the results are summa-rized here, along with the appropriate percentages that Rainbow applies toeach category.

Gross Accounts Percentage EstimatedAge Receivable Uncollectible

Not yet due $150,000 1%

31–60 days past due 50,000 5%

61–90 days past due 40,000 10%

91–120 days past due 25,000 25%

Over 120 days past due 10,000 50%

Total $275,000

a. Assuming Rainbow uses the aging approach of accounting foruncollectible accounts, prepare the adjusting entry required at theend of the accounting period.

Journal Entry:

PostDate Accounts Ref. Dr. Cr.

b. Assume now Rainbow uses the percentage-of-sales method ofaccounting for uncollectible accounts. If historical data indicatethat approximately 3% of net credit sales are uncollectible, what isthe amount of uncollectible accounts expense that should be jour-nalized? _______________What is the balance in the Allowance for Uncollectible Accountsafter adjustment? _______________

1eSG_C07_0131792075.QXD 10/23/06 8:51 AM Page 226

Page 259: polpoa_sg

7-5. Peterson Company, which has a December 31 year-end, com-pleted the following transactions during 2008 and 2009:

2008Oct. 14 Sold merchandise to Bruce Company, receiving a 60-day,

9% note for $10,000.Nov. 16 Sold merchandise to Marine Company receiving a

72-day, 8% note for $9,100.Dec. 13 Received amount due from Bruce Company.

31 Accured interest on the Marine Company note.

2009Jan. 27 Collected in full from Marine Company.

Requirement

1. Prepare the necessary journal entries for the preceding transactions.

Journal Entry:

PostDate Accounts Ref. Dr. Cr.

Journal Entry:

PostDate Accounts Ref. Dr. Cr.

Journal Entry:

PostDate Accounts Ref. Dr. Cr.

Journal Entry:

PostDate Accounts Ref. Dr. Cr.

Journal Entry:

PostDate Accounts Ref. Dr. Cr.

1eSG_C07_0131792075.QXD 10/23/06 8:51 AM Page 227

Page 260: polpoa_sg

Do It Yourself! Question 1

Uncollectible Accounts ReceivableNow Company’s December 31, 2007, balance sheet reported:

Accounts receivable $1,000Allowance for uncollectible accounts (85)Accounts receivable (net) $ 915

Requirements

1. How much of the December 31, 2008, balance of accounts receivable didNow expect to collect?

2. During 2008, Now wrote off accounts receivable totaling $110. Journalizethese write-offs as one transaction. How does this transaction affect thenet accounts receivable balance?

Journal Entry:

PostDate Accounts Ref. Dr. Cr.

Journal Entry:

PostDate Accounts Ref. Dr. Cr.

3. During 2008, Now earned $13,000 of service revenues, all on account.Journalize these revenues as one transaction.

4. During 2008, Now collected $12,840 cash from customers. Journalize thistransaction and calculate the gross accounts receivable balance atDecember 31, 2008.

Journal Entry:

PostDate Accounts Ref. Dr. Cr.

1eSG_C07_0131792075.QXD 10/23/06 8:51 AM Page 228

Page 261: polpoa_sg

Do It Yourself! Question 1 | Chapter 7 229

5. Assume that Now estimates uncollectible accounts expense to be 0.75% ofrevenues. Journalize the entry to adjust the allowance at December 31,2008.What is the December 31, 2008, balance in the allowance?

Allowance for Uncollectible Accounts

Allowance for Uncollectible Accounts

Journal Entry:

PostDate Accounts Ref. Dr. Cr.

6. (Ignore Requirement 5.) Assume that Now has the following information atDecember 31, 2008:

Gross Accounts Percentage Estimated Age Receivable Uncollectible

< 30 days $500 1%

30–60 days 450 10%

> 60 days 100 15%

Total $1,050

Journalize the entry to adjust the allowance at December 31, 2008.What isthe December 31, 2008, balance in the allowance? Show how accountsreceivable would be reported on the balance sheet at December 31, 2008.

Accounts Receivable

Journal Entry:

PostDate Accounts Ref. Dr. Cr.

1eSG_C07_0131792075.QXD 10/23/06 8:51 AM Page 229

Page 262: polpoa_sg

230 Chapter 7 | Do It Yourself! Question 2

Do It Yourself! Question 2

Notes ReceivableOn June 1, 2008, Anderson Corp. borrowed $6,000 cash from Neo Enterprises.Anderson signed a 10-month, 5% note. Anderson paid the note plus interest infull on the due date. Both Anderson and Neo have December 31 year-ends.

Requirements

1. When is the note due? What is the total interest incurred over the life of thenote? What is the maturity value of the note?

2. Prepare all journal entries for this note for both companies from June 1,2008, through the due date. Explanations are not required.

June 1, 2008

Neo:

Journal Entry:

PostDate Accounts Ref. Dr. Cr.

Journal Entry:

PostDate Accounts Ref. Dr. Cr.

Anderson:

1eSG_C07_0131792075.QXD 10/23/06 8:51 AM Page 230

Page 263: polpoa_sg

Do It Yourself! Question 2 | Chapter 7 231

Anderson:

Journal Entry:

PostDate Accounts Ref. Dr. Cr.

Journal Entry:

PostDate Accounts Ref. Dr. Cr.

Journal Entry:

PostDate Accounts Ref. Dr. Cr.

March 31, 2009

Neo:

Anderson:

Journal Entry:

PostDate Accounts Ref. Dr. Cr.

December 31, 2008

Neo:

1eSG_C07_0131792075.QXD 10/23/06 8:51 AM Page 231

Page 264: polpoa_sg

Quick Practice SolutionsTrue/False

T 1. The Allowance for Uncollectible Accounts is a contra-account to Accounts Receivable. (p. 357)

T 2. Under the allowance method, the recovery of an account previously written off has no effect on net income. (p. 357)

F 3. Under the allowance method, the entry to write off an account thatis determined to be uncollectible includes a credit to Allowance forUncollectible Accounts.False–The entry to write off an account that is determined to be uncollectible includes a debit to Allowance for Uncollectible Accounts and a credit to Accounts Receivable. (p. 358)

T 4. Under the allowance method, the entry to write off an account deemed uncollectible has no effect on the total assets of the firm.(p. 358)

F 5. The direct write-off method is the preferred way to apply the accrual basis for measuring uncollectible accounts expense becauseit matches revenues and expenses on the income statement.False–The allowance method is the preferred way to apply the accrual basis for measuring uncollectible account expense because it matches revenues and expenses on the income statement.(pp. 356–357)

F 6. Under the direct write-off method, the entry to write off an account deemed uncollectible has no effect on total assets of the firm.False–The write off of an account under the direct write-off method results in a credit to Accounts Receivable, which reducestotal assets. (p. 356 )

T 7. A written promise to pay a specified amount of money at a particular future date is referred to as a promissory note. (p. 363)

T 8. If the maker of a note does not pay at maturity, the maker is said todishonor the note. (p. 368)

F 9. The quick ratio includes cash, inventory, and net accounts receivable in the numerator.False–The quick ratio includes cash, short-term investments, and net accounts receivable in the numerator. (p. 369)

T 10. Retailers must pay a small fee for accepting credit cards from customers as payment for goods. (p. 352)

232 Chapter 7 | Quick Practice Solutions

1eSG_C07_0131792075.QXD 10/23/06 8:51 AM Page 232

Page 265: polpoa_sg

Multiple Choice

1. Chuck Battle’s account of $5,000 must be written off. Which ofthe following would be journalized assuming that the allowancemethod is used? (pp. 361–362)a. A debit to Battle’s Accounts Receivable and a credit to Allowance for

Uncollectible Accountsb. A debit to Allowance for Uncollectible Accounts and a credit to

Battle’s Accounts Receivablec. A debit to Cash and a credit to Uncollectible Accounts Expensed. A debit to Cash and a credit to Battle’s Accounts Receivable

2. The current credit balance in Allowance for UncollectibleAccounts before adjustment is $658. An aging schedule reveals$3,700 of uncollectible accounts. What is the ending balance inthe Allowance for Uncollectible Accounts? (pp. 361–362)a. $3,042b. $3,700c. $4,029d. $4,358

3. The current debit balance in Allowance for UncollectibleAccounts before adjustment is $742. An aging schedule reveals$3,500 of uncollectible accounts. What is the amount of thejournal entry for Estimated Uncollectible Accounts? (p. 362)a. $ 742b. $2,758c. $3,500d. $4,242

4. What is the type of account and normal balance of Allowance forUncollectible Accounts? (p. 362)a. Asset, debitb. Contra-asset, creditc. Liability, creditd. Contra-liability, debit

5. If the direct write-off method is used for uncollectiblereceivables, what account is debited when writing off acustomer’s account? (pp. 356–357)a. Accounts Receivableb. Allowance for Uncollectible Accountsc. Uncollectible Accounts Expensed. Sales Returns and Allowances

6. What is the effect on the financial statements of writing off anuncollectible account under the direct write-off method? (pp. 356–357)a. Increases expenses and decreases liabilitiesb. Decreases net income and decreases assetsc. Decreases assets and increases owner’s equityd. Increases expenses and increases assets

Quick Practice Solutions | Chapter 7 233

1eSG_C07_0131792075.QXD 10/23/06 8:51 AM Page 233

Page 266: polpoa_sg

7. A 90-day note dated August 26 matures on which of thefollowing dates? (p. 364)a. November 24b. November 23c. November 25d. November 26

8. A 90-day, 12% note for $20,000, dated July 10, is received from acustomer. What is the maturity value of the note? (p. 365)a. $20,000b. $20,600c. $21,200d. $22,400

9. Carolina Supply accepted an 8-month, $16,000 note receivable,with 8% interest, from Reading Corporation on August 1, 2008.Carolina Supply’s year-end is December 31. What is the amountof interest to be accrued on December 31, 2008? (p. 365)a. $ 320b. $ 853c. $ 533d. $1,280

10. Which of the following is journalized on the payee’s books whena debtor dishonors a note receivable? (p.368)a. Uncollectible Accounts Debt Expenseb. Debit Accounts Receivablec. No entry requiredd. Debit Notes Receivable

234 Chapter 7 | Quick Practice Solutions

1eSG_C07_0131792075.QXD 10/23/06 8:51 AM Page 234

Page 267: polpoa_sg

d. The Allowance for Uncollectible Accounts shows an $800 credit bal-ance prior to adjustment. Net credit sales during the year are$229,000 and 3.5% are estimated to be uncollectible.

Quick Exercise

7-1. Prepare the adjusting journal entry on December 31, 2008 forthe following independent situations: (pp. 358–359)

a. The Allowance for Uncollectible Accounts shows a $700 credit bal-ance prior to adjustment. Net credit sales during the year are$216,000 and 4% are estimated to be uncollectible.

Journal Entry:

12/31/08 Uncollectible Accounts Expense 8,640 Allowance for Uncollectible Accounts 8,640 ($216,000 3 0.04) = $8,640

PostDate Accounts Ref. Dr. Cr.

Journal Entry:

12/31/08 Uncollectible Accounts Expense 6,800 Allowance for Uncollectible Accounts 6,800 ($73,000 – $500) = $6,800

PostDate Accounts Ref. Dr. Cr.

Journal Entry:

12/31/08 Uncollectible Accounts Expense 5,625 Allowance for Uncollectible Accounts 5,625 ($5,100 – $525) = $5,625

PostDate Accounts Ref. Dr. Cr.

b. The Allowance for Uncollectible Accounts shows a $500 credit bal-ance prior to adjustment. An aging schedule prepared on December31 reveals an estimated uncollectible accounts amount of $7,300.

c. The Allowance for Uncollectible Accounts shows a $525 debit bal-ance prior to adjustment. An aging schedule prepared on December31 reveals an estimated uncollectible accounts amount of $5,100.

Journal Entry:

12/31/08 Uncollectible Accounts Expense 8,015 Allowance for Uncollectible Accounts 8,015 ($229,000 3 0.035) = $8,015

PostDate Accounts Ref. Dr. Cr.

1eSG_C07_0131792075.QXD 10/23/06 8:51 AM Page 235

Page 268: polpoa_sg

7-2. Compute the ending balance in the Allowance for UncollectibleAccounts after the adjusting entries in Exercise 7-1 have beenprepared for the four independent situations, a – d. (p. 362)

a. $9,340 ($ 700 + $8,640)b. $7,300 ($ 500 + $6,800)c. $5,100 ($5,625 – $ 525)d. $8,815 ($ 800 + $8,015)

7-3. Journalize the following independent transactions assumingthe allowance method is used. (pp. 357–367)

a. August 5, 2008 Wrote off Jones Corp.’s account receivable for $2,200 as uncollectible.

b. August 17, 2008 Collected the $2,200 from Jones Corp. in full.

c. August 31, 2008 Recorded uncollectible accounts expense of $16,500.

236 Chapter 7 | Quick Practice Solutions

Journal Entry:

8/17/08 Accounts Receivable —Jones Corp. 2,200 Allowance for Uncollectible Accounts 2,200 To write off Jones Corp. accounts receivable.

PostDate Accounts Ref. Dr. Cr.

Journal Entry:

8/5/08 Allowance for Uncollectible Accounts 2,200 Accounts Receivable—Jones Corp. 2,200 To write off Jones Corp. account receivable.

PostDate Accounts Ref. Dr. Cr.

Journal Entry:

8/17/08 Cash 2,200 Accounts Receivable—Jones Corp. 2,200 To record cash collected from Jones Corp.

PostDate Accounts Ref. Dr. Cr.

Journal Entry:

8/31/08 Uncollectible Accounts Expense 16,500 Allowance for Uncollectible Accounts 16,500 To record estimated uncollectible accounts.

PostDate Accounts Ref. Dr. Cr.

1eSG_C07_0131792075.QXD 10/23/06 8:51 AM Page 236

Page 269: polpoa_sg

b. Assume now Rainbow uses the percentage-of-sales method ofaccounting for uncollectible accounts. If historical data indicate thatapproximately 3% of net credit sales are uncollectible, what is theamount of uncollectible accounts expense that should be journalized? $23,130

What is the balance in the allowance for uncollectible accounts after adjustment? $23,370 ($240 + 23,130)

Quick Practice Solutions | Chapter 7 237

7-4. On December 31, 2008, Rainbow Appliances has $275,000 inAccounts Receivable and an Allowance account with a creditbalance of $240. Current period net credit sales were $771,000,cash sales were $68,000. (pp. 359–361)

Rainbow Appliances performs an aging schedule, and the results aresummarized here, along with the appropriate percentages that Rainbowapplies to each category.

Gross Accounts Percentage EstimatedAge Receivable Uncollectible

Not yet due $150,000 1%

31–60 days past due 50,000 5%

61–90 days past due 40,000 10%

91–120 days past due 25,000 25%

Over 120 days past due 10,000 50%

Total $275,000

a. Assuming Rainbow uses the aging approach of accounting foruncollectible accounts, prepare the adjusting entry required at theend of the accounting period.

Journal Entry:

12/31/08 Uncollectible Accounts Expense 19,010 Allowance for Uncollectible Accounts 19,010 ($150,000 3 0.01) + ($50,000 3 0.05) + ($40,000 3 .10) + ($25,000 3 0.25) + ($10,000 3 0.5) = $19,250 – $240

PostDate Accounts Ref. Dr. Cr.

1eSG_C07_0131792075.QXD 10/23/06 8:51 AM Page 237

Page 270: polpoa_sg

Journal Entry:

10/14/08 Notes Receivable 10,000 Sales Revenue 10,000

PostDate Accounts Ref. Dr. Cr.

Journal Entry:

11/16/08 Notes Receivable 9,100 Sales Revenue 9,100

PostDate Accounts Ref. Dr. Cr.

Journal Entry:

12/31/08 Interest Receivable 91 Interest Revenue 91

PostDate Accounts Ref. Dr. Cr.

Journal Entry:

12/13/08 Cash 10,150 Notes Receivable 10,000 Interest Revenue 150

PostDate Accounts Ref. Dr. Cr.

Journal Entry:

12/13/08 Cash 9,246 Notes Receivable 9,100 Interest Revenue 55 Interest Receivable 91

PostDate Accounts Ref. Dr. Cr.

7-5. Peterson Company, which has a December 31 year-end, com-pleted the following transactions during 2008 and 2009:(pp. 365–367)

2008Oct. 14 Sold merchandise to Bruce Company, receiving a 60-day,

9% note for $10,000.Nov. 16 Sold merchandise to Marine Company receiving a 72-day,

8% note for $9,100.Dec. 13 Received amount due from Bruce Company.

31 Accrued interest on the Marine Company note.

2009Jan. 27 Collected in full from Marine Company.

Requirement 1

Prepare the necessary journal entries for the preceding transactions.

1eSG_C07_0131792075.QXD 10/23/06 8:51 AM Page 238

Page 271: polpoa_sg

Do It Yourself! Question 1 Solutions | Chapter 7 239

Do It Yourself! Question 1 Solutions

Requirement 1

How much of the December 31, 2007, balance of accounts receivable did Nowexpect to collect?

Now expects to collect $915 of the accounts receivable balance.

Requirement 2

During 2008, Now wrote off accounts receivables totaling $110. Journalizethese write-offs as one transaction. How does this transaction affect the netaccounts receivable balance?

Journal Entry:

Allowance for Uncollectible Accounts (Contra-Asset, 2; debit) 110 Accounts Receivable—Amanda Blake (Asset, 2; credit) 110

PostDate Accounts Ref. Dr. Cr.

Journal Entry:

Accounts Receivable (Asset, 1; debit) 13,000 Service Revenue (Revenue, 1; credit) 13,000

PostDate Accounts Ref. Dr. Cr.

Journal Entry:

Cash (Asset, 1; debit) 12,840 Accounts Receivable (Asset, 2; credit) 12,840

PostDate Accounts Ref. Dr. Cr.

This transaction has no impact on the net receivables balance.

Requirement 3

During 2008, Now earned $13,000 of service revenues, all on account.Journalize these revenues as one transaction.

Requirement 4

During 2008, Now collected $12,840 cash from customers. Journalize thistransaction and calculate the gross accounts receivable balance at December 31,2008.

1eSG_C07_0131792075.QXD 11/22/06 10:21 AM Page 239

Page 272: polpoa_sg

240 Chapter 7 | Do It Yourself! Question 1 Solutions

Requirement 5

Assume that Now estimates uncollectible accounts expense to be 0.75% ofrevenues. Journalize the entry to adjust the allowance at December 31, 2008.What is the December 31, 2008, balance in the allowance?

0.75% # $13,000 " $97.50

Accounts Receivable

Dec. 31, 2007 Bal. 1,0002008 Revenues 13,000 2008 Write-Offs 110

2008 Cash Collections 12,840

Dec. 31, 2008 Bal. 1,050

Journal Entry:

Uncollectible Accounts Expense (Expense, 1; debit) 97.50 Allowance for Uncollectible Accounts (Contra-Asset, 1; credit) 97.50

PostDate Accounts Ref. Dr. Cr.

Requirement 6

(Ignore Requirement 5.) Assume that Now has the following information atDecember 31, 2008:

Gross Accounts Percentage EstimatedAge Receivable Uncollectible

< 30 days $500 1%

30–60 days 450 10%

> 60 days 100 15%

Total $1,050

Allowance for Uncollectible Accounts

Dec. 31, 2007 Bal. 85.002008 Write_Offs 110

2008 Uncollectible Accounts Expense 97.50

Dec. 31, 2008 Bal. 72.50

1eSG_C07_0131792075.QXD 10/23/06 8:51 AM Page 240

Page 273: polpoa_sg

So

85 – 110 ! X " 65

X " 65 – 85 ! 110 " 90

Do It Yourself! Question 1 Solutions | Chapter 7 241

Journalize the entry to adjust the allowance at December 31, 2008.What is theDecember 31, 2008, balance in the allowance? Show how accounts receivablewould be reported on the balance sheet at December 31, 2008.

Gross Percentage Amount Amount Estimated Estimated

Age Receivable Uncollectible Uncollectible

< 30 days $500 # 1% = $5

30–60 days $450 # 10% = 45

> 60 days $100 # 15% = 15

Total $1,050 $65

Allowance for Uncollectible Accounts

Dec. 31, 2007 Bal. 852008 Write-Offs 110

2008 Uncollectible Accounts Expense X

Dec. 31, 2008 Bal. 65

Journal Entry:

Uncollectible Accounts Expense (Expense, 1; debit) 90 Allowance for Uncollectible Accounts (Contra-Asset, 1; credit) 90

PostDate Accounts Ref. Dr. Cr.

On the balance sheet:

Accounts receivable $1,050

– Allowance for uncollectible accounts ($65)

Accounts receivable (net) $985

1eSG_C07_0131792075.QXD 10/23/06 8:51 AM Page 241

Page 274: polpoa_sg

242 Chapter 7 | Do It Yourself! Question 2 Solutions

Journal Entry:

June 1 Note Receivable (Asset,1; debit) 6,000 Cash (Asset,2; credit) 6,000

PostDate Accounts Ref. Dr. Cr.

Interest " Amount #

Annual interest #

Time elapsedincurred of debt rate (in years)

" $6,000 # 5% # 10 months12 months

" $250

Maturity " Principal ! Interest incurredover life of note

" $6,000 ! $250

" $6,250

Do It Yourself! Question 2 Solutions

Requirement 1

When is the note due? What is the total interest incurred over the life of thenote? What is the maturity value of the note?

The note is due 10 months from June 1, 2008, on April 1, 2009.The amount of interest incurred over the entire life of the note is calculated as:

Journal Entry:

June 1 Cash (Asset,1; debit) 6,000 Note Payable (Liability,1; credit) 6,000

PostDate Accounts Ref. Dr. Cr.

Anderson:

Requirement 2

Prepare all journal entries for this note for both companies from June 1, 2008,through the due date. Explanations are not required.

June 1, 2008 Anderson borrowed $6,000 from Neo.

Neo:

1eSG_C07_0131792075.QXD 10/23/06 8:51 AM Page 242

Page 275: polpoa_sg

December 31, 2008 Accrued 5% interest on note.

Neo:

Do It Yourself! Question 2 Solutions | Chapter 7 243

Journal Entry:

Dec. 31 Interest Expense ($6,000 ! 5% ! 7/12) (Expense,1; debit) 175 Interest Payable (Liability,1; credit) 175

PostDate Accounts Ref. Dr. Cr.

Journal Entry:

Mar. 31 Cash (to balance) (Asset,1; debit) 6,250 Note Receivable (Asset,2; credit) 6,000 Interest Revenue ($6,000 ! 5% ! 3/12) (Revenue,1; credit) 75 Interest Receivable (Asset,2; credit) 175

PostDate Accounts Ref. Dr. Cr.

Journal Entry:

Mar. 31 Note Payable (Liability,2; debit) 6,000 Interest Payable (Liability,2; debit) 175 Interest Expense ($6,000 ! 5% ! 3//12) (Expense,1; debit) 75 Cash (to balance) (Asset,2; credit) 6,250

PostDate Accounts Ref. Dr. Cr.

Anderson:

March 31, 2009 Paid note and interest in full.

Neo:

Journal Entry:

Dec. 31 Interest Receivable (Asset,1; debit) 175 Interest Revenue ($6,000 ! 5% ! 7/12) (Revenue,1; credit) 175

PostDate Accounts Ref. Dr. Cr.

Anderson:

1eSG_C07_0131792075.QXD 10/23/06 8:51 AM Page 243

Page 276: polpoa_sg

The Power of PracticeFor more practice using the skills learned in this chapter, visit MyAccountingLab.There you will find algorithmically generated questions that are based on theseDemo Docs and your main textbook’s Review and Accounting Practice sections.

To go to MyAccountingLab and follow these steps:

1. Direct your URL to www.myaccountinglab.com.2. Log in using your name and password.3. Click the MyAccountingLab link.4. Click Study Plan in the left navigation bar.5. From the table of contents, select Chapter 7, Receivables.6. Click a link to work on the tutorial exercises.

244 Chapter 7 | The Power of Practice

1eSG_C07_0131792075.QXD 10/23/06 8:51 AM Page 244

Page 277: polpoa_sg

Inventory8

1

WHAT YOU PROBABLY ALREADY KNOW

Assume that you want to invest in the stock market. You purchase100 shares of a stock mutual fund in January at $24 per share,another 100 shares in February at $27 per share, and another 100shares in April at $30 per share. In December, you decide to sell200 shares of stock to purchase a used car. The market value of thestock at the date of sale is $35 per share. You know that you willreceive $7,000 (200 shares ! $35/share) and that the market priceof the shares is higher than what you paid, so you have a gain. Tocompute the amount of the gain you will have to report on your taxreturn, you must determine the cost of the shares. Becausepurchases were made over a period of time at several differentprices, how is the cost computed for the 200 shares sold? Can weassume that the shares sold were the first 100 shares wepurchased at $24 per share plus the next 100 shares purchased at$27 per share for a total cost of $5,100, that is, (100 shares ! $24) +(100 shares ! $27)? Can we calculate the cost using an average?Yes, either of these methods is allowed by the Internal RevenueService. The same problem exists for businesses in determining thecost of the inventory units sold when the unit cost varies. Generallyaccepted accounting principles (GAAP) also allows a choice fromseveral methods to calculate the cost of goods sold.

Learning Objectives

Describe inventory and discuss the related internal controls.

Recall from Chapter 5 that Inventory is the account that holds the cost ofgoods purchased for resale, not for use, by the business. This chapterdeals with inventory of retail businesses. It is important that inventory isthe subject of internal controls to account for any discrepanciesbetween the physical count of inventory and its value in the accountingrecords. Carefully review the inventory shrinkage and internal controlssections of the main textbook, especially Exhibit 8-2 (p. 408).

Compute inventory costs using first-in, first-out (FIFO), last-in,first-out (LIFO), and average cost methods and journalizeinventory transactions.

The inventory cost method selected for use is an assumed outflow ofgoods to determine the cost of goods sold expense and endinginventory; the actual physical outflow of goods sold may differ. FIFO is apopular method that assumes the oldest goods are sold first, leavingthe newest goods in ending inventory. LIFO is the opposite assumption;

2

HarrCh08v1.qxd 10/23/06 9:13 AM Page 245

Page 278: polpoa_sg

246 Chapter 8 | Inventory

it assumes that the newest goods are sold first, leaving the oldest goodsin ending inventory. Average cost assumes that the goods sold as wellas those in ending inventory have the same cost. Review the inventoryrecords and journal entries for these methods in Exhibits 8-5 (p. 413)and 8-7 through 8-9 (pp. 414–416).

Compare the effects of the different costing methods on thefinancial statements.

The cost of goods sold will usually be different for each of the methods.In times of inflation, FIFO will result in higher net income and higherending inventory amounts than LIFO. The average method falls betweenFIFO and LIFO results. Review the comparative results of these methodsin Exhibits 8-10 and 8-11 (p. 417).

Apply the lower-of-cost-or-market (LCM) rule to value inventory

The inventory amount on the balance sheet is reduced to the marketvalue if that amount is lower than the cost. This rule is an application ofthe conservatism concept. Review the “Lower-of Cost-or-Market Rule”section in the main text. Note the required journal entry and balancesheet presentation.

Report inventory on the balance sheet and measure the effect ofinventory errors.

When measuring the effects of inventory errors, it is helpful to rememberthat:

Cost of Goods Sold ! Cost of Ending Inventory " Cost of Goods Available

The cost of goods available is a defined amount. Therefore, if the cost ofending inventory is understated, the cost of goods sold must beoverstated by the same amount to compensate for the error. The reverseis also true; if the cost of ending inventory is overstated, the cost ofgoods sold will be understated by the same amount to compensate forthe error. Understating ending inventory results in an understatement ofnet income. The reverse is also true; overstating ending inventoryresults in an overstatement of net income.

The ending inventory for one period becomes the beginning inventoryfor the next. An error in ending inventory is carried over into thesucceeding period. Whatever effect the ending inventory error had onthe income statement in the initial period causes the opposite effect onnet income in the next period.

Review the impact of ending inventory errors in Exhibit 8-13 through 8-15(pp. 421–422).

Estimate ending inventory by the gross profit method.

Sometimes a business may need to estimate its ending inventory. Whena natural disaster occurs and inventory is destroyed, an estimate mustbe determined for insurance purposes. The gross profit method is usedin this type of case. Review the gross profit method of estimatinginventory in Exhibits 8-16 and 8-17. (pp. 423–424).

3

4

5

6

HarrCh08v1.qxd 10/23/06 9:13 AM Page 246

Page 279: polpoa_sg

Demo Doc 1 | Chapter 8 247

Inventory at January 1 400 units @ $2 each

Inventory purchases, March 200 units @ $3 each

Sales, May 160 units

Inventory purchases, July 100 units @ $4 each

Sales, September 460 units

Inventory purchases, November 250 units @ $5 each

Inventory Costing Methods and Lower-of-Cost-or-MarketRuleLearning Objectives 1–4

Collins Industries perpetual inventory records show the following data for 2008:

Demo Doc 1

Ignore any effects of inventory shrinkage for this problem.

Requirements

1. Calculate COGS for the year ended December 31, 2008, and inventory atDecember 31, 2008, under each of the following costing assumptions:

• FIFO

• LIFO

• Average cost

2. Sales revenues were $4,000 for 2008. Calculate gross profit under eachmethod.

3. Now assume that Collins has a periodic inventory system. Which methodwould maximize net income? Which method would minimize incometaxes?

4. Assume that Collins is using FIFO.The ending inventory has a market priceof $4.50 per unit. Calculate the lower-of-cost-or-market and make anynecessary adjustment.

HarrCh08v1.qxd 10/23/06 9:13 AM Page 247

Page 280: polpoa_sg

248 Chapter 8 | Demo Doc 1 Solutions

Number of Units Sold " 160 in May ! 460 in September" 620 Units for the Year

COGS " Beginning ! Inventory – EndingInventory Purchases Inventory

Units in Units Units in Units Sold " Beginning ! Purchased – Ending

Inventory Inventory

Demo Doc 1 SolutionsRequirement 1

Calculate COGS for the year ended December 31, 2008, and inventory atDecember 31, 2008, under each of the following costing assumptions:

• FIFO

• LIFO

• Average cost

Part 1 Part 2 Part 3 Part 4 Part 5 Part 6 Part 7Demo DocComplete

Before doing any costing calculations, it is important to determine the goodsavailable for sale (both in units and dollars). We must also determine the numberof units that were sold and the number of units in ending inventory.

Beginning inventory

400 units @ $2 " $ 800

Inventory purchases

200 units @ $3 " $ 600100 units @ $4 " $ 400250 units @ $5 " $1,250

Goods available for sale 950 units $3,050

or

This formula is expressed in dollars, but it also works in units:

Goods BeginnningAvailable for " Inventory ! PurchasesSale

COGS " Goods Available – Ending for Sale Inventory

HarrCh08v1.qxd 10/23/06 9:13 AM Page 248

Page 281: polpoa_sg

Demo Doc 1 Solutions | Chapter 8 249

Units in Units Sold " Units Available for Sale – Ending

Inventory

Units in 620 Units " 950 Units – Ending

Inventory

Units in Ending Inventory " 330

Part 1 Part 2 Part 3 Part 4 Part 5 Part 6 Part 7Demo DocComplete

Purchases Cost of Goods Sold Inventory on Hand

Unit Total Unit Total Unit Total Date Quantity Cost Cost Quantity Cost Cost Quantity Cost CostJan. 400 $2.00 $800

or

FIFO

FIFO means first-in, first-out. In other words, we always sell the oldest item wehave. So what is left in inventory? The newest units.

The beginning inventory was 400 units at $2 each, totaling $800 (400 ! $2).

Compute inventorycosts using first-in,first-out (FIFO), last-in,first-out (LIFO), andaverage cost methodsand journalize inven-tory transactions.

2

In March, 200 units costing $3 each were purchased for $600 (200 ! $3).After this purchase, the original 400 units and the 200 units just purchased werein inventory.

Purchases Cost of Goods Sold Inventory on Hand

Unit Total Unit Total Unit Total Date Quantity Cost Cost Quantity Cost Cost Quantity Cost CostJan. 400 $2.00 $800

Mar. 200 $3.00 $600 400 $2.00 $800

200 $3.00 $600

In May, 160 units were sold. Under FIFO, the oldest inventory units are soldfirst. The oldest inventory is the beginning inventory of 400 units at $2 per unit.The units sold must have come from this group.

HarrCh08v1.qxd 10/23/06 9:13 AM Page 249

Page 282: polpoa_sg

250 Chapter 8 | Demo Doc 1 Solutions

Purchases Cost of Goods Sold Inventory on Hand

Unit Total Unit Total Unit Total Date Quantity Cost Cost Quantity Cost Cost Quantity Cost CostJan. 400 $2.00 $800

Mar. 200 $3.00 $600 400 $2.00 $800

200 $3.00 $600

May 160 $2.00 $320 240 $2.00 $480

200 $3.00 $600

In September, 460 units were sold. At this time, the oldest units in inventorywere the remaining 240 units from the beginning inventory group that cost $2each. These units must have been sold.

In July, another 100 units costing $4 each were purchased for a total of $400(100 ! $4). These 100 units were added to the inventory already present at theend of May.

Purchases Cost of Goods Sold Inventory on Hand

Unit Total Unit Total Unit Total Date Quantity Cost Cost Quantity Cost Cost Quantity Cost CostJan. 400 $2.00 $800

Mar. 200 $3.00 $600 400 $2.00 $800

200 $3.00 $600

May 160 $2.00 $320 240 $2.00 $480

200 $3.00 $600

July 100 $4.00 $400 240 $2.00 $480

200 $3.00 $600

100 $4.00 $400

What remains in inventory are the 240 units (400 – 160) from the beginninginventory group and the entire March purchase of 200 units.

HarrCh08v1.qxd 10/23/06 9:13 AM Page 250

Page 283: polpoa_sg

Demo Doc 1 Solutions | Chapter 8 251

In addition, 220 – 200 = 20 other units were sold.These other units must have come from the July purchase of 100 units at $4

per unit (the next oldest units). After the September sale, 80 (100 – 20) units cost-ing $4 each are left in inventory.

Purchases Cost of Goods Sold Inventory on Hand

Unit Total Unit Total Unit Total Date Quantity Cost Cost Quantity Cost Cost Quantity Cost CostJan. 400 $2.00 $800

Mar. 200 $3.00 $600 400 $2.00 $800

200 $3.00 $600

May 160 $2.00 $320 240 $2.00 $480

200 $3.00 $600

July 100 $4.00 $400 240 $2.00 $480

200 $3.00 $600

100 $4.00 $400

Sept. 240 $2.00 $480

However, 240 units are not enough; another 460 – 240 = 220 units were sold.Some of these units must be from the next oldest inventory: the March purchaseof 200 units at $3 per unit.

Purchases Cost of Goods Sold Inventory on Hand

Unit Total Unit Total Unit Total Date Quantity Cost Cost Quantity Cost Cost Quantity Cost CostJan. 400 $2.00 $800

Mar. 200 $3.00 $600 400 $2.00 $800

200 $3.00 $600

May 160 $2.00 $320 240 $2.00 $480

200 $3.00 $600

July 100 $4.00 $400 240 $2.00 $480

200 $3.00 $600

100 $4.00 $400

Sept. 240 $2.00 $480

200 $3.00 $600

HarrCh08v1.qxd 10/23/06 9:13 AM Page 251

Page 284: polpoa_sg

252 Chapter 8 | Demo Doc 1 Solutions

Purchases Cost of Goods Sold Inventory on Hand

Unit Total Unit Total Unit Total Date Quantity Cost Cost Quantity Cost Cost Quantity Cost CostJan. 400 $2.00 $800

Mar. 200 $3.00 $600 400 $2.00 $800

200 $3.00 $600

May 160 $2.00 $320 240 $2.00 $480

200 $3.00 $600

July 100 $4.00 $400 240 $2.00 $480

200 $3.00 $600

100 $4.00 $400

Sept. 240 $2.00 $480

200 $3.00 $600

20 $4.00 $ 80 80 $4.00 $320

In November, 250 more units costing $5 each were purchased for a total of$1,250 (250 ! $5). These units were added to the 80 that remained in inventoryafter the September sale.

Purchases Cost of Goods Sold Inventory on Hand

Unit Total Unit Total Unit Total Date Quantity Cost Cost Quantity Cost Cost Quantity Cost CostJan. 400 $2.00 $ 800

Mar. 200 $3.00 $ 600 400 $2.00 $ 800

200 $3.00 $ 600

May 160 $2.00 $320 240 $2.00 $ 480

200 $3.00 $ 600

July 100 $4.00 $ 400 240 $2.00 $ 480

200 $3.00 $ 600

100 $4.00 $ 400

Sept. 240 $2.00 $480

200 $3.00 $600

20 $4.00 $ 80 80 $4.00 $ 320

Nov. 250 $5.00 $1,250 80 $4.00 $ 320

250 $5.00 $1,250

HarrCh08v1.qxd 10/23/06 9:13 AM Page 252

Page 285: polpoa_sg

Demo Doc 1 Solutions | Chapter 8 253

Now we must total all of the columns in the table.

Purchases Cost of Goods Sold Inventory on Hand

Unit Total Unit Total Unit Total Date Quantity Cost Cost Quantity Cost Cost Quantity Cost CostJan. 400 $2.00 $ 800

Mar. 200 $3.00 $ 600 400 $2.00 $ 800

200 $3.00 $ 600

May 160 $2.00 $ 320 240 $2.00 $ 480

200 $3.00 $ 600

July 100 $4.00 $ 400 240 $2.00 $ 480

200 $3.00 $ 600

100 $4.00 $ 400

Sept. 240 $2.00 $ 480

200 $3.00 $ 600

20 $4.00 $ 80 80 $4.00 $ 320

Nov. 250 $5.00 $1,250 80 $4.00 $ 320

250 $5.00 $1,250

Dec. 550 $2,250 620 $1,480 330 $1,570

FIFO

So under FIFO, COGS = $1,480 and inventory at December 31, 2008 = $1,570.We can use the inventory formula to check our calculation.

COGS " Beginning Inventory ! Purchases – Ending InventoryCOGS " $800 ! $2,250 – $1,570 " $1,480orCOGS " $3,050 – $1,570 " $1,480

Checking the calculation in this way is one method of determining whether anyinventory shrinkage occurred.

LIFO

Describe inventoryand discuss therelated internal controls.

1

Compute inventorycosts using first-in,first-out (FIFO), last-in,first-out (LIFO), andaverage cost methodsand journalize inven-tory transactions.

2

Part 1 Part 2 Part 3 Part 4 Part 5 Part 6 Part 7Demo DocComplete

Under LIFO, the newest units are sold first. We must track each sale individuallyas it is made under the perpetual recording system. Such a system prevents usfrom selling units that we have not yet acquired.

HarrCh08v1.qxd 10/23/06 9:13 AM Page 253

Page 286: polpoa_sg

254 Chapter 8 | Demo Doc 1 Solutions

Purchases Cost of Goods Sold Inventory on Hand

Unit Total Unit Total Unit TotalDate Quantity Cost Cost Quantity Cost Cost Quantity Cost Cost

Jan. 400 $2.00 $800

In July, 100 more units costing $4 each were purchased for a total of $400(100 ! $4). These units are added into the inventory that is already there (fromthe end of May).

Beginning inventory is 400 units at $2 each for a total of $800 (400 ! $2).

Purchases Cost of Goods Sold Inventory on Hand

Unit Total Unit Total Unit TotalDate Quantity Cost Cost Quantity Cost Cost Quantity Cost CostJan. 400 $2.00 $800

Mar. 200 $3.00 $600 400 $2.00 $800

200 $3.00 $600

The first sale is in May, when 160 units are sold. At this time, what were thenewest units in stock? The 200 units at $3 per unit purchased in March. So the160 units sold must have come from this group. The calculation is as follows:

May COGS " 160 !! $3 " $480

The original 400 units from beginning inventory are still in inventory. In addi-tion, 40 units (200 – 160) from the March purchase at $3 per unit remain ininventory.

Purchases Cost of Goods Sold Inventory on Hand

Unit Total Unit Total Unit TotalDate Quantity Cost Cost Quantity Cost Cost Quantity Cost CostJan. 400 $2.00 $800

Mar. 200 $3.00 $600 400 $2.00 $800

200 $3.00 $600

May 160 $3.00 $480 400 $2.00 $800

40 $3.00 $120

We purchase 200 units at $3 each in March for a total of $600 (200 ! $3).These units and the beginning 400 units are now in inventory.

HarrCh08v1.qxd 10/23/06 9:13 AM Page 254

Page 287: polpoa_sg

Demo Doc 1 Solutions | Chapter 8 255

Purchases Cost of Goods Sold Inventory on Hand

Unit Total Unit Total Unit TotalDate Quantity Cost Cost Quantity Cost Cost Quantity Cost CostJan. 400 $2.00 $800

Mar. 200 $3.00 $600 400 $2.00 $800

200 $3.00 $600

May 160 $3.00 $480 400 $2.00 $800

40 $3.00 $120

July 100 $4.00 $400 400 $2.00 $800

40 $3.00 $120

100 $4.00 $400

A second sale took place in September, when 460 units were sold. At thattime, what were the newest units in stock? The 100 units at $4 per unit pur-chased in July. These units must have been sold.

Purchases Cost of Goods Sold Inventory on Hand

Unit Total Unit Total Unit TotalDate Quantity Cost Cost Quantity Cost Cost Quantity Cost CostJan. 400 $2.00 $800

Mar. 200 $3.00 $600 400 $2.00 $800

200 $3.00 $600

May 160 $3.00 $480 400 $2.00 $800

40 $3.00 $120

July 100 $4.00 $400 400 $2.00 $800

40 $3.00 $120

100 $4.00 $400

Sept. 100 $4.00 $400

This quantity is not enough to cover the entire sale, because another 460 –100 = 360 units were sold.

Some of these other units must have come from the next newest group, the200 units at $3 per unit purchased in March. However, only 40 of these units areleft after the sale in May.

HarrCh08v1.qxd 10/23/06 9:13 AM Page 255

Page 288: polpoa_sg

256 Chapter 8 | Demo Doc 1 Solutions

Purchases Cost of Goods Sold Inventory on Hand

Unit Total Unit Total Unit Total Date Quantity Cost Cost Quantity Cost Cost Quantity Cost CostJan. 400 $2.00 $800

Mar. 200 $3.00 $600 400 $2.00 $800

200 $3.00 $600

May 160 $3.00 $480 400 $2.00 $800

40 $3.00 $120

July 100 $4.00 $400 400 $2.00 $800

40 $3.00 $120

100 $4.00 $400

Sept. 100 $4.00 $40040 $3.00 $120

Another 360 – 40 = 320 units sold in September still must be accounted for.These other units must come from the beginning inventory of 400 units at $2 perunit. After the sale, 80 of the beginning units (costing $2 each) are still in inventory.

Purchases Cost of Goods Sold Inventory on Hand

Unit Total Unit Total Unit TotalDate Quantity Cost Cost Quantity Cost Cost Quantity Cost CostJan. 400 $2.00 $800

Mar. 200 $3.00 $600 400 $2.00 $800

200 $3.00 $600

May 160 $3.00 $480 400 $2.00 $800

40 $3.00 $120

July 100 $4.00 $400 400 $2.00 $800

40 $3.00 $120

100 $4.00 $400

Sept. 100 $4.00 $400

40 $3.00 $120

320 $2.00 $640 80 $2.00 $160

In November, another purchase of 250 units for $5 each was made, totaling$1,250 (250 ! $5). These units joined the previous 80 units (from the end ofSeptember) in inventory.

HarrCh08v1.qxd 10/23/06 2:17 PM Page 256

Page 289: polpoa_sg

Demo Doc 1 Solutions | Chapter 8 257

Purchases Cost of Goods Sold Inventory on Hand

Unit Total Unit Total Unit Total Date Quantity Cost Cost Quantity Cost Cost Quantity Cost CostJan. 400 $2.00 $ 800

Mar. 200 $3.00 $ 600 400 $2.00 $ 800

200 $3.00 $ 600

May 160 $3.00 $480 400 $2.00 $ 800

40 $3.00 $ 120

July 100 $4.00 $ 400 400 $2.00 $ 800

40 $3.00 $ 120

100 $4.00 $ 400

Sept. 100 $4.00 $400

40 $3.00 $120

320 $2.00 $640 80 $2.00 $ 160

Nov. 250 $5.00 $1,250 80 $2.00 $ 160

250 $5.00 $1,250

Now the table must be totaled.

Purchases Cost of Goods Sold Inventory on Hand

Unit Total Unit Total Unit Total Date Quantity Cost Cost Quantity Cost Cost Quantity Cost CostJan. 400 $2.00 $ 800

Mar. 200 $3.00 $ 600 400 $2.00 $ 800

200 $3.00 $ 600

May 160 $3.00 $ 480 400 $2.00 $ 800

40 $3.00 $ 120

July 100 $4.00 $ 400 400 $2.00 $ 800

40 $3.00 $ 120

100 $4.00 $ 400

Sept. 100 $4.00 $ 400

40 $3.00 $ 120

320 $2.00 $ 640 80 $2.00 $ 160

Nov. 250 $5.00 $1,250 80 $2.00 $ 160

250 $5.00 $1,250

Dec. 550 $2,250 620 $1,640 330 $1,410

LIFO

HarrCh08v1.qxd 10/23/06 9:13 AM Page 257

Page 290: polpoa_sg

258 Chapter 8 | Demo Doc 1 Solutions

From these columns, we can see that COGS for the year is $1,640 and endinginventory at December 31 is 330 units costing $1,410.

We can use the inventory formula to check our calculations:

COGS " Beginning Inventory ! Purchases – Ending InventoryCOGS " $800 ! $2,250 – $1,410 " $1,640

orCOGS " $3,050 – $1,410 " $1,640

Average Cost

Part 1 Part 2 Part 3 Part 4 Part 5 Part 6 Part 7Demo DocComplete

Describe inventory anddiscuss the relatedinternal controls.

1

The beginning inventory of 400 units costing $2 each and March purchase of 200units at $3 each are recorded as they were for FIFO and LIFO:

Purchases Cost of Goods Sold Inventory on Hand

Unit Total Unit Total Unit Total Date Quantity Cost Cost Quantity Cost Cost Quantity Cost CostJan. 400 $2.00 $800

Mar. 200 $3.00 $600 400 $2.00 $800

200 $3.00 $600

Compute inventorycosts using first-in,first-out (FIFO), last-in,first-out (LIFO), andaverage cost methodsand journalize inventorytransactions.

2

Under the average cost method, we need to calculate the average cost perunit with every sale.

The first sale was in May. At that time in inventory, we had:

400 units # $2 per unit " $800200 units # $3 per unit " $600600 units $1,400

So at this time, the average cost per unit was:

Average Cost"

Cost of Units in Inventoryper Unit Units in Inventory

Average Cost " $1.400 " $2.33 perper Unit 600 units unit

The COGS for the May sale is:

160 units !! $2.33 per unit " $373

We have 600 units – 160 units = 440 units left in inventory at $2.33 per unit= $1,025.

HarrCh08v1.qxd 10/23/06 9:13 AM Page 258

Page 291: polpoa_sg

Demo Doc 1 Solutions | Chapter 8 259

In July, 100 more units were purchased for $4 each. These units were addedto the inventory that remained after the May sale.

Purchases Cost of Goods Sold Inventory on Hand

Unit Total Unit Total Unit TotalDate Quantity Cost Cost Quantity Cost Cost Quantity Cost Cost

400 $2.00 $ 800

Jan.

Mar. 200 $3.00 $600 400 $2.00 $ 800

200 $3.00 $ 600

May 160 $2.33 $373 440 $2.33 $1,025

Purchases Cost of Goods Sold Inventory on Hand

Unit Total Unit Total Unit TotalDate Quantity Cost Cost Quantity Cost Cost Quantity Cost Cost

Jan. 400 $2.00 $ 800

Mar. 200 $3.00 $600 400 $2.00 $ 800

200 $3.00 $ 600

May 160 $2.33 $373 440 $2.33 $1,025

July 100 $4.00 $400 440 $2.33 $1,025

100 $4.00 $ 400

The next sale was in September. At this time in inventory, we had:

440 units ! $2.33/unit " $1,025100 units ! $4/unit " $ 400540 units $1,425

So at this time, the average cost per unit was:

Averagae Cost " $1,425 " $2.64 per per Unit 540 units unit

September COGS " 460 units x $2.64/unit " $1,214

Now 540 units – 460 units = 80 units are left in inventory at the average cost of$2.64 = $211.

HarrCh08v1.qxd 10/23/06 2:18 PM Page 259

Page 292: polpoa_sg

260 Chapter 8 | Demo Doc 1 Solutions

In November, 250 units costing $5 each were purchased. These units wereadded to the 80 already in inventory (after the September sale).

Purchases Cost of Goods Sold Inventory on Hand

Unit Total Unit Total Unit TotalDate Quantity Cost Cost Quantity Cost Cost Quantity Cost Cost

Jan. 400 $2.00 $ 800

Mar. 200 $3.00 $600 400 $2.00 $ 800

200 $3.00 $ 600

May 160 $2.33 $ 373 440 $2.33 $1,025

July 100 $4.00 $400 440 $2.33 $1,025

100 $4.00 $ 400

Sept. 460 $2.64 $1,214 80 $2.64 $ 211

Purchases Cost of Goods Sold Inventory on Hand

Unit Total Unit Total Unit TotalDate Quantity Cost Cost Quantity Cost Cost Quantity Cost Cost

Jan. 400 $2.00 $ 800

Mar. 200 $3.00 $ 600 400 $2.00 $ 800

200 $3.00 $ 600

May 160 $2.33 $ 373 440 $2.33 $1,025

July 100 $4.00 $ 400 440 $2.33 $1,025

100 $4.00 $ 400

Sept. 460 $2.64 $1,214 80 $2.64 $ 211

Nov. 250 $5.00 $1,250 80 $2.64 $ 211

250 $5.00 $1,250

Now we add up all the columns in the table:

HarrCh08v1.qxd 10/23/06 9:13 AM Page 260

Page 293: polpoa_sg

Demo Doc 1 Solutions | Chapter 8 261

Purchases Cost of Goods Sold Inventory on Hand

Unit Total Unit Total Unit Total Date Quantity Cost Cost Quantity Cost Cost Quantity Cost Cost

Jan. 400 $2.00 $ 800

Mar. 200 $3.00 $ 600 400 $2.00 $ 800

200 $3.00 $ 600

May 160 $2.33 $ 373 440 $2.33 $1,025

July 100 $4.00 $ 400 440 $2.33 $1,025

100 $4.00 $ 400

Sept. 460 $2.64 $1,214 80 $2.64 $ 211

Nov. 250 $5.00 $1,250 80 $2.64 $ 211

250 $5.00 $1,250

Dec. 550 $2,250 620 $1,587 330 $1,461

Describe inventory anddiscuss the relatedinternal controls.

1

Compare the effectsof the different cost-ing methods on thefinancial statements.

3

So under average cost, COGS = $1,587 and inventory at December 31, 2008 =$1,461.

We can check our calculations using the inventory formula:*

COGS " Beginning Inventory ! Purchases – Ending InventoryCOGS " $800 ! $2,250 – $1,461 " $1,589

*Note that the $2 difference is due to rounding the per-unit cost under the average cost method.

Requirement 2

Sales revenues were $4,000 for 2008. Calculate gross profit under each method.

Part 1 Part 2 Part 3 Part 4 Part 5 Part 6 Part 7Demo DocComplete

FIFO LIFO Average Cost

Sales Revenue $4,000 $4,000 $4,000

– COGS 1,480 1,640 1,587

Gross Profit $2,520 $2,360 $2,413

Gross Profit " Sales Revenue – COGS

Requirement 3

Now assume that Collins has a periodic inventory system. Which methodwould maximize net income? Which method would minimize income taxes?

Part 1 Part 2 Part 3 Part 4 Part 5 Part 6 Part 7Demo DocComplete

Average cost

HarrCh08v1.qxd 10/23/06 9:13 AM Page 261

Page 294: polpoa_sg

262 Chapter 8 | Demo Doc 1 Solutions

We already determined that 330 units remain in ending inventory. Under FIFO,the cost of these units is $1,570. The market price of these units is:

330 units !! $4.50 per unit " $1,485

If cost is $1,570 and market is $1,485, the lower-of-cost-or-market is $1,485 (themarket value of the inventory).

The balance in the Inventory T-account is currently the cost of $1,570.Therefore, Inventory must be decreased to the market value of $1,485. SoInventory is decreased (credit) by:

$1,570 – $1,485 " $85

The other side of the journal entry is an adjustment to COGS. This entry willhave to be a debit to balance out the credit to inventory.

Compare the effectsof the different cost-ing methods on thefinancial statements.

3 For the different methods, we have the following COGS:

FIFO $1,480

LIFO $1,640

Average cost $1,587

Of these, FIFO is the lowest and LIFO is the highest. Note that FIFO andLIFO will usually be the extremes, with average cost somewhere in the middle.

FIFO gives the lowest COGS, which means that it gives the highest grossprofit. You can see this result in Requirement 2. Therefore, FIFO would maximizenet income.

LIFO gives the highest COGS, which means that it gives the lowest grossprofit. You can also see this result in Requirement 2. LIFO minimizes net income,which in turn minimizes income taxes.

Note that if prices are decreasing over time (such as for high-tech items thatquickly become obsolete), then the reverse of this analysis is true (FIFO giveshighest COGS).

Requirement 4

Presume that Collins is using FIFO.The ending inventory has a market price of$4.50 per unit. Calculate the lower-of-cost-or-market and make any necessaryadjustment.

Part 1 Part 2 Part 3 Part 4 Part 5 Part 6 Part 7Demo DocComplete

Apply the lower-of-cost-or-market (LCM)rule to value inventory.

4

Journal Entry:

Dec. 31 Cost of Goods Sold 85 Inventory ($1,570 – $1,485) 85

Date Accounts Dr. Cr.

Part 1 Part 2 Part 3 Part 4 Part 5 Part 6 Part 7Demo DocComplete

HarrCh08v1.qxd 10/23/06 9:13 AM Page 262

Page 295: polpoa_sg

Demo Doc 2 | Chapter 8 263

Demo Doc 2Gross Profit Method and Inventory ErrorsLearning Objectives 5, 6

On December 31, 2008, Talon Corp.’s warehouse and accounting records weredestroyed in a flood. For insurance purposes, Talon must estimate the value ofthe inventory lost.

Through records from their bank and suppliers, Talon compiled the followinginformation:

Sales revenue for 2008 $20,000Inventory at December 31, 2007 $ 6,000Inventory purchases for 2008 $23,000

Talon has historically had a gross profit rate of 10%.

Requirements

1. Estimate Talon’s ending inventory value for 2008 using the gross profitmethod.

2. Assume that the actual value of inventory lost was $12,000.What is Talon’strue COGS? Is COGS overstated or understated? How will the value ofCOGS affect Talon’s estimate of net income for 2008?

HarrCh08v1.qxd 10/23/06 9:13 AM Page 263

Page 296: polpoa_sg

264 Chapter 8 | Demo Doc 2 Solutions

Demo DocCompletePart 1 Part 2

COGS "Beginning ! Inventory – EndingInventory Purchases Inventory

Gross Profit " Gross Profit " Sales – COGSPercentage Sales Revenue Sales

10% " $20,000 – COGS

$20,000

Demo Doc 2 SolutionsRequirement 1

Estimate Talon’s ending inventory value for 2008 using the gross profit method.

The gross profit method uses the COGS formula:

We are given information about purchases and beginning inventory, but to calcu-late ending inventory, we will need an estimate for COGS.

The formula for the gross profit percentage is:

So we know that 10% = ($20,000 – COGS)/$20,000.

From this, we can calculate COGS = $18,000. Using this figure in the COGSformula:

$18,000 " $6,000 ! $23,000 – Ending Inventory$11,000 " Ending Inventory

Requirement 2

Assume that the actual value of inventory lost was $12,000.What is Talon’s trueCOGS? Is COGS overstated or understated? How will the value of COGS affectTalon’s estimate of net income for 2008?

Demo DocCompletePart 1 Part 2

Part 1 Part 2 Demo DocComplete

Using the actual value of $12,000 for ending inventory, we can recalculate COGS as:

COGS " $6,000 ! $23,000 – $12,000 " $17,000

Because Talon is estimating COGS of $18,000, COGS is overstated by:

$18,000 – $17,000 " $1,000

If Talon uses the wrong COGS number of $18,000 to calculate net income, thennet income will be understated by $1,000.

Report inventory onthe balance sheet andmeasure the effect ofinventory errors.

5

Estimate ending inven-tory by the gross profitmethod.

6

HarrCh08v1.qxd 10/23/06 9:13 AM Page 264

Page 297: polpoa_sg

Quick Practice Questions | Chapter 8 265

Quick Practice QuestionsTrue/False

_____ 1. Under FIFO, the ending inventory cost comes from the oldest purchases.

_____ 2. FIFO is the opposite of LIFO.

_____ 3. The LIFO method can result in misleading inventory costs on the bal-ance sheet because the oldest prices are left in ending inventory.

_____ 4. When inventory costs are rising, LIFO will result in the lowest grossprofit.

_____ 5. When using a perpetual inventory system, a business will debitInventory and credit Cost of Goods Sold each time a sale is recorded.

_____ 6. If a company had 10 units of beginning inventory with a unit cost of$10 and a subsequent purchase of 15 units at a unit cost of $12, theaverage cost of one unit sold would be $11.

_____ 7. When applying lower-of-cost-or-market rules to ending inventory valu-ation, market value generally refers to the company’s current sellingprice for its inventory.

_____ 8. Understating beginning inventory in the current year will understatecost of goods sold in the current year.

_____ 9. Overstating ending inventory in 2007 will understate cost of goods soldfor 2008.

_____10. The gross profit method is a method used to estimate inventory thatcan be used to determine losses for insurance claims due to a fire ornatural disaster.

HarrCh08v1.qxd 10/23/06 9:13 AM Page 265

Page 298: polpoa_sg

266 Chapter 8 | Quick Practice Questions

Multiple Choice

1. Anticipating no gains but providing for all probable losses canbe most closely associated with which of the following?a. Conservatismb. Disclosure principlec. Consistency principled. Materiality concept

2. Which of the following are required to record the sale ofmerchandise on credit under a perpetual inventory system?a. Debit Accounts Receivable; credit Sales Revenueb. Debit Cost of Goods Sold; credit Purchasesc. Debit Cost of Goods Sold; credit Inventoryd. Both (a) and (c) are necessary entries

3. What is the effect of using FIFO during a period of rising pricesunder a perpetual inventory system?a. Less net income than LIFOb. Less operating expenses than LIFOc. Higher gross profit than LIFOd. Higher cost of goods sold than average costing

4. Which of the following is not a reason for choosing the LIFOmethod?a. LIFO reports the most up-to-date inventory values on the balance sheet.b. LIFO uses more current costs in calculating cost of goods sold.c. LIFO allows owners and managers to manage reported income.d. LIFO generally results in lower income taxes paid.

5. Which of the following is true for ending inventory when pricesare falling and the LIFO inventory system is used?a. LIFO ending inventory is less than FIFO.b. LIFO ending inventory is greater than FIFO.c. LIFO ending inventory is equal to FIFO.d. LIFO ending inventory is equally likely to be higher or lower than FIFO.

6. The following data are for Daisy’s Florist Shop for the firstseven months of its fiscal year:Beginning inventory $53,500Purchases 75,500Net sales revenue 93,700Normal gross profit percent 30%

What is the estimated inventory on hand as determined by thegross profit method?a. $ 28,110b. $ 63,410c. $ 65,590d. $100,890

HarrCh08v1.qxd 10/23/06 9:45 AM Page 266

Page 299: polpoa_sg

Quick Practice Questions | Chapter 8 267

7. Which of the following statements is true about a companymaking an accounting change in its financial statements?a. It is generally entitled to make one accounting change per year.b. It must report the change in accounting method.c. Companies can never make accounting changes because of the consis-

tency principle.d. It must petition the Financial Accounting Standards Board for per-

mission to make the change.

8. When is an item considered material?a. When it facilitates comparison with the financial statements of

another company in the same industryb. When its inclusion in the financial statements would cause a state-

ment user to change a decisionc. When its dollar value is greater than 10% of net incomed. When it is accounted for using a treatment that is not normally

allowed by generally accepted accounting principles

9. Ending inventory for Commodity X consists of 20 units. Underthe FIFO method, the cost of the 20 units is $5 each. Currentreplacement cost is $4.50 per unit. Using the lower-of-cost-or-market rule to value inventory, the balance sheet would showending inventory at what amount?a. $ 4.75b. $ 5.00c. $ 90.00d. $100.00

10. Inventory at the end of the current year is overstated by $20,000.What effect will this error have on the following year’s netincome?a. Net income will be overstated $20,000b. Net income will be understated $20,000c. Net income will be correctly statedd. Net income will be understated $40,000

HarrCh08v1.qxd 10/23/06 9:13 AM Page 267

Page 300: polpoa_sg

268 Chapter 8 | Quick Practice Questions

Quick Exercises

8-1. Compute the missing income statement amounts for each ofthe following independent companies:

Beginning Ending Cost of Company Net Sales Inventory Purchases Inventory Goods Sold Gross Profit

A $ 93,000 $14,600 $65,000 (a) ______ $58,300 (b) ______

B (c) ______ $31,600 (d) ______ $23,600 $96,200 $52,500

C $ 89,300 $23,600 $54,000 (f) ______ (e) ______ $23,900

D $105,000 $11,200 (h) ______ $ 9,400 (g) ______ $48,200

8-2. Which inventory method would best meet each of the follow-ing specific management goals? Show your answer by insert-ing the proper letter beside each statement.

a. Specific unit cost

b. LIFO

c. FIFO

__________ 1. Management desires to properly match net sales rev-enue with the most recent cost of goods.

__________ 2. Management desires to minimize the company’s end-ing inventory balance during a period of falling prices.

__________ 3. The company sells rare antique items.__________ 4. Management desires to show the current value of

inventory on the balance sheet.__________ 5. Management desires to minimize the company’s tax

liability during a period of rising prices.

8-3. The following data are available for the month of March:

March 1 balance 20 units at $16 each

March 10 purchase 40 units at $18 each

March 17 purchase 30 units at $20 each

March 30 purchase 25 units at $21 each

On March 31, 35 units are on hand.

HarrCh08v1.qxd 10/23/06 9:13 AM Page 268

Page 301: polpoa_sg

Quick Practice | Chapter 8 269

Effect on Cost Effect on Net Item Error of Goods Sold Income

1. Beginning inventory is understated.

2. Ending inventory is understated.

3. Beginning inventory is overstated.

4. Ending inventory is overstated.

Requirement

1. Calculate cost of goods sold under the following methods:

a. FIFO

b. LIFO

c. Average cost (round the per unit cost to the nearest cent; round thefinal answer to the nearest dollar)

8-4. Plastic Products Company lost some of its inventory due to aflood and needs to determine the amount of the inventorylost. The following data are available for 2008:

Net sales revenue $400,000Estimated gross profit rate 35%January 1, beginning inventory 11,600Net purchases 275,000Inventory on hand, after flood 6,500

Requirements1. Compute what the estimated ending inventory should be using the

gross profit method.

2. Calculate the amount of the inventory loss.

8-5. Determine the effect on cost of goods sold and net income forthe current year of the following inventory errors. Indicateyour answer with either a + (overstated) or a – (understated).

HarrCh08v1.qxd 10/23/06 9:13 AM Page 269

Page 302: polpoa_sg

270 Chapter 8 | Do It Yourself! Question 1

Compute inventorycosts using first-in,first-out (FIFO), last-in,first-out (LIFO), andaverage cost methodsand journalize inven-tory transactions.

2

Do It Yourself! Question 1Sam Inc., inventory records show the following data for July 2008:

Inventory at July 1 10 units @ $1 each

Inventory purchases, July 5 80 units @ $2 each

Sales, July 10 50 units

Inventory purchases, July 15 20 units @ $3 each

Sales, July 20 40 units

Inventory purchases, July 25 30 units @ $4 each

Ignore any effects of inventory shrinkage for this problem.

Requirements

1. Calculate COGS for the month ended July 31, 2008, and inventory at July31, 2008, using the FIFO method.

FIFOPurchases Cost of Goods Sold Inventory on Hand

Unit Total Unit Total Unit TotalDate Quantity Cost Cost Quantity Cost Cost Quantity Cost Cost

July 1

July 5

July 10

July 15

July 20

July 25

July 31

HarrCh08v1.qxd 10/23/06 9:13 AM Page 270

Page 303: polpoa_sg

Do It Yourself! Question 1 | Chapter 8 271

Purchases Cost of Goods Sold Inventory on Hand

Unit Total Unit Total Unit TotalDate Quantity Cost Cost Quantity Cost Cost Quantity Cost Cost

July 1

July 5

July 10

July 15

July 20

July 25

July 31

LIFO

2. Calculate COGS for the month ended July 31, 2008 and inventory at July31, 2008, using the LIFO method.

Compute inventorycosts using first-in,first-out (FIFO), last-in,first-out (LIFO), andaverage cost methodsand journalize inventorytransactions

2

HarrCh08v1.qxd 10/23/06 9:13 AM Page 271

Page 304: polpoa_sg

4. Sales revenues were $500 for July 2008. Calculate gross profit under eachmethod.

5. The market value of ending inventory is $180. If Sam uses LIFO, give anynecessary adjustment for the lower-of-cost-or-market rule.

272 Chapter 8 | Do It Yourself! Question 1

Journal Entry:

Date Accounts Dr. Cr.

Compare the effectsof the different costingmethods on the finan-cial statements.

3

Apply the lower-of-cost-or-market (LCM)rule to value inventory.

4

Purchases Cost of Goods Sold Inventory on Hand

Unit Total Unit Total Unit TotalDate Quantity Cost Cost Quantity Cost Cost Quantity Cost Cost

July 1

July 5

July 10

July 15

July 20

July 25

July 31

Average Cost

3. Calculate COGS for the month ended July 31, 2008, and inventory at July31, 2008, using the average cost method.

Compute inventorycosts using first-in,first-out (FIFO), last-in,first-out (LIFO), andaverage cost methodsand journalize inventorytransactions.

2

HarrCh08v1.qxd 10/23/06 9:13 AM Page 272

Page 305: polpoa_sg

Do It Yourself! Question 2 | Chapter 8 273

Estimate ending inven-tory by the gross profitmethod.

6

Do It Yourself! Question 2

On December 31, 2008, Virga Brothers lost all its inventory during a hurricane.Virga was able to gather the following information.

Inventory at January 1, 2008 $ 40,000

Inventory purchases for 2008 $ 90,000

Sales revenue for 2008 $180,000

Historically, Virga experienced gross profit of 40%.

Requirements

1. Estimate the value of Virga’s lost inventory.

HarrCh08v1.qxd 10/23/06 9:13 AM Page 273

Page 306: polpoa_sg

Quick Practice SolutionsTrue/False

F 1. Under FIFO, the ending inventory cost comes from the oldestpurchases.False–FIFO leaves in ending inventory the last or newest costs.(p. 412)

T 2. FIFO is the opposite of LIFO. (p. 413)

T 3. The LIFO method can result in misleading inventory costs on the balance sheet because the oldest prices are left in ending inventory.(p. 417)

T 4. When inventory costs are rising, LIFO will result in the lowest gross profit. (p. 417)

F 5. When using a perpetual inventory system, a business will debit Inventory and credit Cost of Goods Sold each time a sale is recorded.False–Using a perpetual inventory system, a business will debitCost of Goods Sold and credit Inventory each time a sale is recorded. (pg. 421)

F 6. If a company had 10 units of beginning inventory with a unit cost of $10 and a subsequent purchase of 15 units at a unit cost of $12, the average cost of one unit sold would be $11.False–Average cost is determined by dividing the cost of goods available, (10 units ! $10) + (15 units ! $12) = $280, by the number of units available (10 + 15 = 25): $280/25 = $11.20. (p. 415)

F 7. When applying lower-of-cost-or-market rules to ending inventory valuation, market value generally refers to the company’s current selling price for its inventory.False–Market value generally means current replacement cost.(p. 418)

T 8. Understating beginning inventory in the current year will understate cost of goods sold in the current year. (p. 422)

F 9. Overstating ending inventory in 2007 will understate cost of goods

274 Chapter 8 | Quick Practice Solutions

sold for 2008.False–Overstating ending inventory in 2007 will overstate cost of goods sold for 2008. (p. 422)

T 10. The gross profit method is a method used to estimate inventory that can be used to determine losses for insurance claims due to a fire or natural disaster. (p. 423)

HarrCh08v1.qxd 10/23/06 9:13 AM Page 274

Page 307: polpoa_sg

Multiple Choice

1. Anticipating no gains but providing for all probable losses canbe most closely associated with which of the following? (p. 418)a. Conservatismb. Disclosure principlec. Consistency principled. Materiality concept

2. Which of the following are required to record the sale ofmerchandise on credit under a perpetual inventory system? (p. 409)a. Debit Accounts Receivable; credit Sales Revenueb. Debit Cost of Goods Sold; credit Purchasesc. Debit Cost of Goods Sold; credit Inventoryd. Both a and c are necessary entries

3. What is the effect of using FIFO during a period of rising pricesunder a perpetual inventory system? (p. 417)a. Less net income than LIFOb. Less operating expenses than LIFOc. Higher gross profit than LIFOd. Higher cost of goods sold than average costing

4. Which of the following is not a reason for choosing the LIFOmethod? (p. 417)a. LIFO reports the most up-to-date inventory values on the balance sheet.b. LIFO uses more current costs in calculating cost of goods sold.c. LIFO allows owners and managers to manage reported income.d. LIFO generally results in lower income taxes paid.

5. Which of the following is true for ending inventory when pricesare falling and the LIFO inventory system is used? (p. 413)a. LIFO ending inventory is less than FIFO.b. LIFO ending inventory is greater than FIFO.c. LIFO ending inventory is equal to FIFO.d. LIFO ending inventory is equally likely to be higher or lower than FIFO.

6. The following data are for Daisy’s Florist Shop for the firstseven months of its fiscal year:Beginning inventory $53,500Purchases 75,500Net sales revenue 93,700Normal gross profit percent 30%

What is the estimated inventory on hand as determined by thegross profit method? (p. 423)a. $ 28,110b. $ 63,410c. $ 65,590d. $100,890

Quick Practice Solutions | Chapter 8 275

HarrCh08v1.qxd 10/23/06 9:46 AM Page 275

Page 308: polpoa_sg

7. Which of the following statements is true about a companymaking an accounting change in its financial statements? (p. 420)a. It is generally entitled to make one accounting change per year.b. It must report the change in accounting method.c. Companies can never make accounting changes because of the consis-

tency principle.d. It must petition the Financial Accounting Standards Board for per-

mission to make the change.

8. When is an item considered material? (p. 419)a. When it facilitates comparison with the financial statements of

another company in the same industryb. When its inclusion in the financial statements would cause a state-

ment user to change a decisionc. When its dollar value is greater than 10% of net incomed. When it is accounted for using a treatment that is not normally

allowed by generally accepted accounting principles

9. Ending inventory for Commodity X consists of 20 units. Underthe FIFO method, the cost of the 20 units is $5 each. Currentreplacement cost is $4.50 per unit. Using the lower-of-cost-or-market rule to value inventory, the balance sheet would showending inventory at what amount? (p. 418)a. $ 4.75b. $ 5.00c. $ 90.00d. $100.00

10. Inventory at the end of the current year is overstated by $20,000.What effect will this error have on the following year’s netincome? (p. 422)a. Net income will be overstated $20,000b. Net income will be understated $20,000c. Net income will be correctly statedd. Net income will be understated $40,000

276 Chapter 8 | Quick Practice Solutions

HarrCh08v1.qxd 10/23/06 9:13 AM Page 276

Page 309: polpoa_sg

Quick Exercise

8-1. Compute the missing income statement amounts for each ofthe following independent companies: (p. 423)

Quick Practice Solutions | Chapter 8 277

Cost of Beginning Ending Goods

Company Net Sales Inventory Purchases Inventory Sold Gross Profit

A $ 93,000 $14,600 $65,000 (a) $58,300 (b)

B (c) $31,600 (d) $23,600 $96,200 $52,500

C $ 89,300 $23,600 $54,000 (f) (e) $23,900

D $105,000 $11,200 (h) $ 9,400 (g) $48,200

(a) $ 14,600 + $65,000 – $ 58,300 = $21,300(b) $ 93,000 – $58,300 = $ 34,700(c) $ 96,200 + $52,500 = $148,700(d) $ 23,600 + $96,200 – $ 31,600 = $88,200(e) $ 89,300 – $23,900 = $ 65,400(f) $ 23,600 + $54,000 – $ 65,400 = $12,200(g) $105,000 – $48,200 = $ 56,800(h) $ 9,400 + $56,800 – $ 11,200 = $55,000

8-2. Which inventory method would best meet each of the follow-ing specific management goals? Show your answer by insert-ing the proper letter beside each statement. (pp. 409–416)

a. Specific unit costb. LIFOc. FIFO

b 1. Management desires to properly match net sales revenuewith the most recent cost of goods.

c 2. Management desires to minimize the company’s endinginventory balance during a period of falling prices.

a 3. The company sells rare antique items.c 4. Management desires to show the current value of inven-

tory on the balance sheet.b 5. Management desires to minimize the company’s tax liabil-

ity during a period of rising prices.

8-3. The following data are available for the month of March:(pp. 412–416)

March 1 balance 20 units at $16 each

March 10 purchase 40 units at $18 each

March 17 purchase 30 units at $20 each

March 30 purchase 25 units at $21 each

On March 31, 35 units are on hand.

HarrCh08v1.qxd 10/23/06 9:13 AM Page 277

Page 310: polpoa_sg

278 Chapter 8 | Quick Practice Solutions

Requirement 1

Calculate cost of goods sold under the following methods:

a. FIFO

115 units available – 35 ending units " 80 units soldCost of goods sold:

(20 !! $16) ! (40 !! $18) ! (20 !! $20) " $320 ! $720 ! $400 " $1,440

b. LIFO

(25 !! $21) ! (30 !! $20) ! (25 !! $18) " $525 ! $600 ! $450 " $1,575

c. Average cost (round the per unit cost to the nearest cent; round thefinal answer to the nearest dollar)

(20 !! $16) ! (40 !! $18) ! (30 !! $20) ! (25 !! $21) " $320 ! $720 ! $600 ! $525 " $2,165$2,165/115 units " $18.83 ! 80 " $1,506

8-4. Plastic Products Company lost some of its inventory due to aflood and needs to determine the amount of the inventorylost. The following data are available for 2008: (p. 423)

Net sales revenue $400,000

Estimated gross profit rate 35%

January 1, beginning inventory 11,600

Net purchases 275,000

Inventory on hand, after flood 6,500

Requirement 1

Compute what the estimated ending inventory should be using the grossprofit method.

Beginning inventory $ 11,600Net purchases 275,000Cost of goods available 286,600

Estimated cost of goods sold:Net sales revenue $400,000Less: Estimated gross profit of 35% (140,000)Estimated cost of goods sold 260,000

Estimated cost of ending inventory $ 26,600

Requirement 2

Calculate the amount of the inventory loss.

Estimated cost of ending inventory $26,600Less: Inventory on hand, after flood 6,500Amount of inventory loss $20,100

HarrCh08v1.qxd 10/23/06 9:13 AM Page 278

Page 311: polpoa_sg

8-5. Determine the effect on cost of goods sold and net income forthe current year of the following inventory errors. Indicateyour answer with either a + (overstated) or a – (understated).(p. 421)

Quick Practice Solutions | Chapter 8 279

Effect on Cost Effect on Net Item Error of Goods Sold Income

1. Beginning inventory is understated. – +

2. Ending inventory is understated. + –

3. Beginning inventory is overstated. + –

4. Ending inventory is overstated. – +

HarrCh08v1.qxd 10/23/06 9:13 AM Page 279

Page 312: polpoa_sg

280 Chapter 8 | Do It Yourself! Question 1 Solutions

Do It Yourself! Question 1 Solutions

Requirement 1

Calculate COGS for the month ended July 31, 2008, and inventory at July 31,2008, using the FIFO method.

Beginning inventory

10 units @ $1 per unit = $ 10

Inventory purchases

July 5 80 units @ $2 per unit = $160

July 15 20 units @ $3 per unit = $ 60

July 25 30 units @ $4 per unit = $120

Goods available 130 units $350for sale

Number of Units Sold " 50 on July 10 ! 40 on July 20 " 90 units90 units " 10 units ! 130 units – Units in Ending Inventory

or

90 units " 140 units – Units in Ending InventoryUnits in Ending Inventory " 50 units

Check:COGS " $10 ! $340 – $180 " $170

orCOGS " $350 – $180 " $170

Purchases Cost of Goods Sold Inventory on Hand

Unit Total Unit Total Unit TotalDate Quantity Cost Cost Quantity Cost Cost Quantity Cost Cost

July 1 10 $1.00 $ 10July 5 80 $2.00 $160 10 $1.00 $ 10

80 $2.00 $160July 10 10 $1.00 $ 10

40 $2.00 $ 80 40 $2.00 $ 80July 15 20 $3.00 $ 60 40 $2.00 $ 80

20 $3.00 $ 60July 20 40 $2.00 $ 80 20 $3.00 $ 60July 25 30 $4.00 $120 20 $3.00 $ 60

30 $4.00 $120July 31 130 $340 90 $170 50 $180

FIFO

HarrCh08v1.qxd 10/23/06 9:13 AM Page 280

Page 313: polpoa_sg

Do It Yourself! Question 1 Solutions | Chapter 8 281

Purchases Cost of Goods Sold Inventory on Hand

Unit Total Unit Total Unit TotalDate Quantity Cost Cost Quantity Cost Cost Quantity Cost Cost

July 1 10 $1.00 $ 10July 5 80 $2.00 $160 10 $1.00 $ 10

80 $2.00 $160

July 10 50 $1.89 $ 94 40 $1.89 $ 76

July 15 20 $3.00 $ 60 40 $1.89 $ 7620 $3.00 $ 60

July 20 40 $2.27 $ 91 20 $2.27 $ 45

July 25 30 $4.00 $120 20 $2.27 $ 4530 $4.00 $120

July 31 130 $340 90 $185 50 $165

Average Cost

Check:COGS ! $10 " $340 – $150 ! $200

orCOGS ! $350 – $150 ! $200

Requirement 3

Calculate COGS for the month ended July 31, 2008, and inventory at July 31,2008, using the average cost method.

Purchases Cost of Goods Sold Inventory on Hand

Unit Total Unit Total Unit TotalDate Quantity Cost Cost Quantity Cost Cost Quantity Cost Cost

July 1 10 $1.00 $ 10July 5 80 $2.00 $160 10 $1.00 $ 10

80 $2.00 $160July 10 50 $1.00 $100 10 $1.00 $ 10

30 $2.00 $ 60July 15 20 $3.00 $ 60 10 $1.00 $ 10

30 $2.00 $ 60July 20 20 $3.00 $ 60 10 $1.00 $ 10

20 $2.00 $ 40 10 $2.00 $ 20July 25 30 $4.00 $ 120 10 $1.00 $ 10

10 $2.00 $ 2030 $4.00 $120

July 31 130 $340 90 $200 50 $150

LIFO

Requirement 2

Calculate COGS for the month ended July 31, 2008, and inventory at July 31,2008, using the LIFO method.

HarrCh08v1.qxd 10/23/06 2:19 PM Page 281

Page 314: polpoa_sg

282 Chapter 8 | Do It Yourself! Question 1 Solutions

Check:COGS " $10 ! $340 – $165 " $185

orCOGS " $350 – $165 " $185

Requirement 4

Sales revenues were $500 for July 2008. Calculate gross profit under each method.

FIFO LIFO Average Cost

Sales Revenue $500 $500 $500

– COGS 170 200 185

Gross Profit $330 $300 $315

Journal Entry:

July 31 Cost of Goods Sold 20 Inventory ($200 – $180) 20

Date Accounts Dr. Cr.

Requirement 5

The market value of ending inventory is $180. If Sam uses LIFO, give anynecessary adjustment for the lower-of-cost-or-market rule.

Cost (under LIFO) " $200 Market " $180Lower of cost or market " $180

HarrCh08v1.qxd 10/23/06 9:13 AM Page 282

Page 315: polpoa_sg

Do It Yourself! Question 2 Solutions | Chapter 8 283

Do It Yourself! Question 2 Solution

Requirement 1

Estimate the value of Virga’s lost inventory.

Gross Profit"

Gross Profit " Sales – COGSPercentage Sales Revenue Sales

40% " ($180,000 – COGS)/$180,000

COGS " $108,000

COGS "Beginning

!Inventory – Ending

Inventory Purchases Inventory

$108,000 " $40,000 ! $90,000 – Ending Inventory

Ending Inventory " $22,000

HarrCh08v1.qxd 10/23/06 9:13 AM Page 283

Page 316: polpoa_sg

The Power of PracticeFor more practice using the skills learned in this chapter, visit MyAccountingLab.There you will find algorithmically generated questions that are based on theseDemo Docs and your main textbook’s Review and Accounting Practice sections.

To go to MyAccountingLab and follow these steps:

1. Direct your URL to www.myaccountinglab.com.2. Log in using your name and password.3. Click the MyAccountingLab link.4. Click Study Plan in the left navigation bar.5. From the table of contents, select Chapter 8, Inventory.6. Click a link to work on the tutorial exercises.

284 Chapter 8 | The Power of Practice

HarrCh08v1.qxd 10/23/06 9:13 AM Page 284

Page 317: polpoa_sg

Long-Term Assets:Plant Assets andIntangibles

9WHAT YOU PROBABLY ALREADY KNOW

You probably already know that when you decide to get a car, youmust decide whether you want to purchase or lease it. If you leasea car, you pay a monthly amount for the use of that vehicle, whichis a benefit or expense to you. If you purchase a car for cashinstead, you still receive a monthly benefit, although you make nofuture payments. The benefit or cost incurred is called depreciationexpense. The more a car is used, the less remaining future valuecan be derived from that asset. In business, the asset is reduced forthe loss in usefulness or future benefit as the asset is used.

Learning Objectives

Define and describe the life cycle of long-term assets.

As you learned in Chapter 3 a long-term asset is defined as a long-lived,asset—one that will generally be used by the business for longer than oneyear. Long-term assets include plant assets, such as land, buildings,equipment, and natural resources. Long-term assets can also be intangibleassets, such as rights, patents, and trademarks. Because such assets havefuture service potential over the life of the asset, a portion of its cost isexpensed against the revenues earned from its use over time. Rememberthat accounting covers the stages of the life of a plant asset: acquisition(make-ready costs), ownership (expenditures and depreciation), anddisposal (sell, trade in, or junk the asset), and various factors affect thecosts associated with each stage. Review Exhibit 9-1 (p. 471) for factorsthat affect long-term assets.

Calculate and record the cost to acquire plant assets.

The cost of a plant asset should include all of the necessary costs toacquire the asset and get it ready for use. In addition to the purchaseprice of the plant asset, other items that may be necessary and wouldincrease the cost amount of the asset are shown in Exhibit 9-2 (p. 472) ofthe main textbook. If discounts are available and taken advantage of,those amounts would reduce the cost of the plant asset. Review Exhibit9-2 for examples of items that are considered in the cost of plant assets.Review the section “A Lump-Sum (Basket) Purchase of Assets” in themain text to see how the cost of individual plant assets is determinedwhen a single price is charged for the group.

Calculate and record depreciation of plant assets.

Depreciation is the allocation of cost over a plant asset’s useful life. Theexpense of depreciation is matched against the revenue generated, asshown in Exhibit 9-4 (p. 476). The three most popular methods of

1

2

3

HarrCh09v1.qxd 10/23/06 9:15 AM Page 285

Page 318: polpoa_sg

286 Chapter 9 | Long-Term Assets: Plant Assets and Intangibles

depreciation are straight-line, units-of-production, and double-declining-balance. The adjusting entry to depreciate any plant asset is to debitDepreciation Expense and credit Accumulated Depreciation.

Three elements necessary to calculate depreciation are:

a. Asset cost: known amount on the booksb. Estimated useful life: period of asset usefulnessc. Estimated residual value: expected value at the end of the useful life

Review “Depreciation Methods” in the main text for examples of thevarious depreciation methods.

Calculate and record the disposal of plant assets.

When a plant asset is sold, it should be depreciated until the date ofdisposal. To account for an asset’s disposal, you would debit the cash orother proceeds received, debit the accumulated depreciation, and creditthe plant asset cost. The difference between the asset cost andaccumulated depreciation is book value. If the book value is greater thanthe proceeds, a debit must also be recorded as a loss on disposal. If thebook value is less than the proceeds, a credit must also be recorded as again on disposal.

A plant asset may also be exchanged for a new asset. The book value ofthe old asset is removed as already described. The cash paid onexchange is credited and the market value of the new asset is debited.Any difference between the market value of the new asset and the bookvalue of the old asset plus cash paid is the gain or loss. Review therelated examples in “Disposing of a Plant Asset” in the main text.

Calculate and record depletion of natural resources.

Natural resources are long-term assets that include iron ore, natural gas,and timber. As the inventory of the iron, gas, or other natural resource isused up, it is considered to be depleted. The depletion entry is similar todepreciation (debit Depletion Expense and credit Accumulated Depletion).The depletion amount is determined using the units-of-production formula.Accumulated Depletion is a contra-asset account like AccumulatedDepreciation. Review “Accounting for Natural Resources” in the main text.

Account for intangible assets.

Intangible assets are rights that provide future value or benefit to theorganization. Patents, copyrights, franchises, and trademarks areexamples of these assets. Those intangible assets with a defined usefullife are amortized by the straight-line method. The entry to amortize theintangible asset is to debit Amortization Expense and credit theintangible asset account.

Goodwill represents the excess of the amount paid to purchase acompany over the equity of the company. Goodwill is not amortized butmay need to be written down due to a loss of value. Review “Accountingfor Intangibles,” especially the treatment of goodwill, in the main text.

Report long-term assets on the balance sheet.

Review Exhibit 9-11 (p. 491) to see how plant and intangible assets arepresented on the balance sheet.

4

5

6

7

HarrCh09v1.qxd 10/23/06 9:15 AM Page 286

Page 319: polpoa_sg

Demo Doc 1 | Chapter 9 287

Demo Doc 1DepreciationLearning Objectives 1–4, 7

Peters Corp. purchased a truck for $13,800 cash on January 1, 2008. Peters alsohad to pay taxes of $1,200 cash. The truck had a residual value of $1,000 and auseful life of 7 years or 100,000 miles driven. Peters has a December 31 year-end.The truck was driven for 15,000 miles in 2008, 12,000 miles in 2009, and 17,000miles in 2010.

Requirements

1. Calculate the total cost of the truck.

2. Calculate the depreciation expense and accumulated depreciation balanceat December 31, 2008, 2009, and 2010, using the straight-line, units-of-production, and double-declining-balance methods.

3. Using the double-declining-balance method only, show how the truckaccount would look on the December 31 balance sheet for 2008, 2009, and2010.

4. Which of the three methods maximizes income for 2008? Which methodminimizes income taxes for 2008?

5. Peters sold the truck on September 1, 2011, for $7,000 cash. Journalize thesale transaction using each method. (The truck was driven for 8,000 milesin 2011.)

HarrCh09v1.qxd 10/23/06 9:15 AM Page 287

Page 320: polpoa_sg

288 Chapter 9 | Demo Doc 1 Solutions

Part 1 Part 2 Part 3 Part 4 Part 5 Part 6 Part 7 Part 8 Part 9Demo DocComplete

Part 1 Part 2 Part 3 Part 4 Part 5 Part 6 Part 7 Part 8 Part 9Demo DocComplete

Demo Doc 1 Solutions

The total cost of the truck is the total cost to make it ready for use. Any expendi-ture that must be paid in order to use the asset is part of the asset’s total cost.In this case, the truck cannot be used until the taxes are paid on the truck.Therefore, the taxes are added to the total cost of the truck.

Purchase price $13,800Taxes 1,200Total cost of truck $15,000

Requirement 2

Calculate the depreciation expense and accumulated depreciation balance atDecember 31, 2008, 2009, and 2010, using the straight-line, units-of-production,and double-declining-balance methods.

Straight-Line Method

The straight-line method allocates an equal amount of depreciation over the use-ful life.

Straight-line depreciation is calculated as:

Remember that cost minus residual value is sometimes called depreciable costbecause it is the total depreciation that will be recorded over the asset’s life. Atthe end of the asset’s life, the book value equals the residual value.

In this particular question:

Remember that the depreciation expense will be the same for each year.Depreciation expense does not change (unless the asset is used for a partial year,as demonstrated in Requirement 5 of this question). This is why the method iscalled straight-line: Because if the annual depreciation expense were charted ona graph, it would be a straight line.

$ , $ ,$ ,

15 000 1 0007

2 000!

"years

Depreciation Exxpense per Year

Cost Residual ValueYears of Useful Life

Ann!

" uual Depreciation Expense

Calculate and recordthe cost to acquireplant assets.

2

Define and describethe life cycle of long-term assets

1

Calculate and recorddepreciation of plantassets

3

Requirement 1

Calculate the total cost of the truck.

HarrCh09v1.qxd 10/23/06 9:15 AM Page 288

Page 321: polpoa_sg

Demo Doc 1 Solutions | Chapter 9 289

Accumulated Depreciation,Truck12/31/08 2,0002008 Bal. 2,00012/31/09 2,0002009 Bal. 4,00012/31/10 2,0002010 Bal. 6,000

So in 2008, accumulated depreciation is the 2008 depreciation expense (becauseit is the only year of depreciation so far), which equals $2,000. In 2009, accumu-lated depreciation is the sum of the 2008 and 2009 depreciation expense, or$2,000 + $2,000 = $4,000. In 2010, accumulated depreciation is the sum of the2008, 2009, and 2010 depreciation expense: $2,000 + $2,000 + $2,000 = $6,000.

If you want to make things a little easier on yourself, instead of adding up allof the accumulated depreciation from scratch, you can instead add the currentyear’s depreciation expense to the prior balance. In other words:

Accumulated Depreciation This Year " Accumulated Depreciation Last Year # This Year’s Depreciation Expense

The 2009 accumulated depreciation is $4,000, plus $2,000 depreciation expensefor 2010 equals $6,000 accumulated depreciation for 2010.

The truck’s book value is its cost minus its accumulated depreciation. Thisnet value (that is, book value) is shown for the truck on the balance sheet.

Straight-Line Method

Depreciation for the Year

Asset Depreciation Depreciable Depreciation Accumulated Book Date Cost Rate Cost Expense Depreciation Value

1-1-2008 $15,000 $15,000

12-31-2008 14/7 ! $1,000 = $2,000 $2,000 13,000

12-31-2009 14/7 ! 1,000 = 2,000 4,000 11,000

12-31-2010 14/7 ! 1,000 = 2,000 6,000 9,000

Depreciation Expense in 2008 " Depreciation Expense in 2009 " Depreciation Expense in 2010 " $2,000

Accumulated depreciation is the total of all the depreciation expense that thecompany has accumulated up to a certain time. In other words, it is the sum ofthe depreciation expense in every year that has passed.

You can use a T-account to calculate accumulated depreciation each year:

Part 1 Part 2 Part 3 Part 4 Part 5 Part 6 Part 7 Part 8 Part 9Demo DocComplete

HarrCh09v1.qxd 10/23/06 9:15 AM Page 289

Page 322: polpoa_sg

290 Chapter 9 | Demo Doc 1 Solutions

$15,000 – $1,000 ! $0.14 Depreciation Expense100,000 miles per Actual Mile Driven

Annual Actual Rate Expense

2008 15,000 miles ! $0.14 per mile = $2,100

2009 12,000 miles ! $0.14 per mile = $1,680

2010 17,000 miles ! $0.14 per mile = $2,380

Units-of-Production Method

The unit method is similar to the straight-line method, but instead of calculatingdepreciation expense per year, we calculate it per unit. It allocates an equalamount of depreciation for each unit of production. Notice how the formula issimilar to the straight-line method:

Or, in this particular question:

Cost Residual ValueUnits of Production in U

"

sseful LifeDepreciation Expense per Unit!

Define and describethe life cycle of long-term assets

1

Calculate and recorddepreciation of plantassets

3

Because a different number of miles is driven every year, the annual depreciationexpense will be different from year to year; however, the depreciation rate perunit (mile) remains constant.

Units of production is another way of measuring an asset’s life or productiv-ity. For example, we could say that a machine will last for 5 years, or we mightsay that it will have 50,000 hours of operation. Both statements are reasonableways to express how long the machine will last. The straight-line method focuseson the years (for example, 5 years of life) and the unit method focuses on the prod-uction (such as the 50,000 hours). Assessing the situation will indicate whetherany ways can be used to measure an asset’s life other than by years. In this ques-tion, the miles driven were highlighted and are the only other measure of assetlife we can use for this particular situation.

Under the unit method, we calculate depreciation as:

Depreciation ExpenseThis Year Units Used Th! iis Year Depreciation Expense

Unit#

So, in this question, we can calculate depreciation expense on the truck each year as:

We calculate accumulated depreciation the same way we did for the straight-linemethod (only the depreciation expense is calculated differently from method tomethod).

We can use a T-account to calculate accumulated depreciation each year:

Accumulated Depreciation,Truck12/31/08 2,1002008 Bal. 2,10012/31/09 1,6802009 Bal. 3,78012/31/10 2,3802010 Bal. 6,160

HarrCh09v1.qxd 11/22/06 10:23 AM Page 290

Page 323: polpoa_sg

Demo Doc 1 Solutions | Chapter 9 291

We can also calculate accumulated depreciation directly:

Accumulated Depreciation This Year " Accumulated Depreciation Last Year # This Year’s Depreciation Expense

So, in this question, we can calculate accumulated depreciation each year as:

2008 $0 # $2,100 " $2,1002009 $2,100 # $1,680 " $3,7802010 $3,780 # $2,380 " $6,160

Units-of-Production Method

Depreciation for the Year

Asset Depreciation Number Depreciation Accumulated Book Date Cost per Unit of Units Expense Depreciation Value

1-1-2008 $15,000 $15,000

12-31-2008 $0.14 ! 15,000 = $2,100 $2,100 12,900

12-31-2009 0.14 ! 12,000 = 1,680 3,780 11,220

12-31-2010 0.14 ! 17,000 = 2,380 6,160 8,840

Part 1 Part 2 Part 3 Part 4 Part 5 Part 6 Part 7 Part 8 Part 9Demo DocComplete

Double-Declining-Balance Method

This method is somewhat more complicated than straight-line or units-of-productiondepreciation. It allocates more depreciation in the early years than in the later years.

Instead of a set depreciation amount, we use a depreciation rate:

Double-Declining-Balance (DDB) Depreciation Rate " 2/Years of Useful Life

Or, in this particular question:

DDB rate " 2/7

You may notice that the years of useful life is the same denominator as we usedin the straight-line method. For this reason, the method is called double-declining-balance: it is two times the amount used for straight-line (that is, 2 !1/years of useful life).

To get the depreciation expense each year, we need to use the following formula:

This Year’s Depreciation Expense " Book Value ! Depreciation Rate

where

Book Value " Cost – Last Year’s Accumulated Depreciation

Sometimes the cost – last year’s accumulated depreciation is called the net valueof the asset. You will see why in Requirement 3 of this question.

Because the accumulated depreciation is used in the depreciation expenseformula, we need to calculate both together every year; however, the methods wecan use to calculate accumulated depreciation are the same as before.

Define and describethe life cycle of long-term assets

1

Calculate and recorddepreciation of plantassets

3

HarrCh09v1.qxd 10/23/06 9:15 AM Page 291

Page 324: polpoa_sg

292 Chapter 9 | Demo Doc 1 Solutions

We can also use a T-account to calculate accumulated depreciation each year:

2008 Depreciation Expense " ($15,000 – $0) $2/7 " $4,286Accumulated Depreciation " $0 # $4,286 " $4,286

2009 Depreciation Expense " ($15,000 – $4,286) $ 2/7 " $3,061Accumulated Depreciation " $4,286 # $3,061 " $7,347

2009 Depreciation Expense " ($15,000 – $7,347) $ 2/7 " $2,187Accumulated Depreciation " $7,347 # $2,187 " $9,534

Accumulated Depreciation,Truck12/31/08 4,2862008 Bal. 4,28612/31/09 3,0612009 Bal. 7,34712/31/10 2,1872010 Bal. 9,534

It is important to keep an eye on accumulated depreciation with the double-declining-balance method. Remember, we did not use the residual value to calcu-late depreciation expense.

However, we need to ensure that the book value of the asset does not gobelow the residual value. When the book value of the asset reaches the residualvalue, we stop taking depreciation expense (even if the asset is still being used).

Double-Declining-Balance Method

Depreciation for the Year

Asset Depreciation Accumulated Book Date Cost DDB Rate Book Value Expense Depreciation Value

1-1-2008 $15,000 $15,000

12-31-2008 2/7 ! $15,000 = $4,286 $4,286 10,714

12-31-2009 2/7 ! 10,714 = 3,061 7,347 7,653

12-31-2010 2/7 ! 7,653 = 2,187 9,534 5,466

HarrCh09v1.qxd 10/23/06 9:15 AM Page 292

Page 325: polpoa_sg

Demo Doc 1 Solutions | Chapter 9 293

2008 2009 2010

Truck $15,000 $15,000 $15,000Accumulated Depreciation (4,286) (7,347) (9,534)

Truck (net) $10,714 $7,653 $5,466

Part 1 Part 2 Part 3 Part 4 Part 5 Part 6 Part 7 Part 8 Part 9Demo DocComplete

Although the question only requires an answer for the double-declining-balancemethod, keep in mind that the balance sheet presentation is the same for alldepreciation methods:

Cost – Accumulated Depreciation Net Value of Asset

This net value of the asset is the same amount that is used in the double-declining-balance calculation for depreciation expense in the following year.

So on the balance sheet for each year (under the double-declining-balancemethod), you would see:

Requirement 4

Which of the three methods maximizes income for 2008? Which methodminimizes income taxes for 2008?

Calculate and recorddepreciation of plantassets

3The depreciation expense for each method in 2008 is:

Straight-line $2,000

Units-of-production $2,100

Double-declining-balance $4,286

Revenues – Expenses " Net Income

So higher expense (holding revenue constant) gives a lower net income.In this example, the straight-line method has the lowest depreciation

expense, which means that it has the highest net income.The double-declining-balance method has the highest depreciation expense,

which means that it has the lowest net income, and therefore the lowest incometaxes.

Requirement 3

Using the double-declining-balance method only, show how the truck accountwould look on the December 31 balance sheet for 2008, 2009, and 2010.

Part 1 Part 2 Part 3 Part 4 Part 5 Part 6 Part 7 Part 8 Part 9Demo DocComplete

Report long-termassets on the balancesheet

7

HarrCh09v1.qxd 10/23/06 9:15 AM Page 293

Page 326: polpoa_sg

294 Chapter 9 | Demo Doc 1 Solutions

Straight-Line Method

When an asset is sold, we must journalize that sale. However, before we journal-ize the disposal, we must update the depreciation on the asset.

Depreciation represents the portion (the benefit) of the asset that has beenused. The truck was sold on September 1, which means that Peters got to use itfor 8 months of 2011 before it was sold. We must represent that use as deprecia-tion expense.

The depreciation expense that we record for 8 months is not the same as theamount we would record for an entire year, because it is a shorter period of time(and therefore the asset was used less). Therefore, we must calculate a partialyear’s depreciation.

Under straight-line depreciation, the annual depreciation expense is $2,000(that is, for 12 months). So for 8 months:

This depreciation would then be recorded as:

2011 DepreciationExpense

$2,0008 months

12" $

mmonths$1,333"

Journal Entry:

Sept. 1 Depreciation Expense 1,333 Accumulated Depreciation, Truck 1,333

PostDate Accounts Ref. Dr. Cr.

This amount for 2011 brings the total accumulated depreciation to:

$6,000 # $1,333 " $7,333.

Now we can record the sale of the truck.Cash was received, so it increases (a debit) by $7,000.The truck has been sold, so that account decreases to a zero balance (a

credit) by $15,000. The Accumulated Depreciation goes along with it (contra-accounts always go with their associated account), so that account decreases to azero balance (a debit) as well by $7,333.

Part 1 Part 2 Part 3 Part 4 Part 5 Part 6 Part 7 Part 8 Part 9Demo DocComplete

Requirement 5

Peters sold the truck on September 1, 2011, for $7,000 cash. Journalize the saletransaction using each method. (The truck was driven for 8,000 miles in 2011.)

Calculate and recorddepreciation of plantassets

3

Calculate and recordthe disposal of plantassets

4

HarrCh09v1.qxd 10/23/06 9:15 AM Page 294

Page 327: polpoa_sg

Demo Doc 1 Solutions | Chapter 9 295

Journal Entry:

Sept. 1 Cash 7,000 Accumulated Depreciation, Truck 7,333 Loss on Sale of Truck (to balance) 667 Truck 15,000

PostDate Accounts Ref. Dr. Cr.

Obviously, the entry is not complete because it does not balance. In order toget it to balance, we need equal debits and credits, which means that we need a$15,000 – $7,000 – $7,333 = $667 debit for the entry to work.

This $667 is the balancing amount. It is either a gain on sale or a loss onsale. Because the balancing amount is a debit, it is a loss (an increase in expensesis a debit, which is like a loss).

So the completed entry is:

Part 1 Part 2 Part 3 Part 4 Part 5 Part 6 Part 7 Part 8 Part 9Demo DocComplete

Journal Entry:

Sept. 1 Cash 7,000 Accumulated Depreciation, Truck 7,333 ????? ????? Truck 15,000

PostDate Accounts Ref. Dr. Cr.

These amounts are shown in the following journal entry:

HarrCh09v1.qxd 10/23/06 9:15 AM Page 295

Page 328: polpoa_sg

296 Chapter 9 | Demo Doc 1 Solutions

In order to get it to balance, we need a $15,000 – $7,000 – $7,280 = $720 debit.Because the balancing amount is a debit, it is a loss (an increase in expenses

is a debit, which is like a loss).

Journal Entry:

Sept. 1 Cash 7,000 Accumulated Depreciation, Truck 7,280 ????? ????? Truck 15,000

PostDate Accounts Ref. Dr. Cr.

Journal Entry:

Sept. 1 Depreciation Expense 1,120 Accumulated Depreciation, Truck 1,120

PostDate Accounts Ref. Dr. Cr.

Accumulated depreciation is now:

$6,160 # $1,120 " $7,280

Cash was received, so it increases (a debit) by $7,000.The truck has been sold, so that account decreases to a zero balance (a

credit) by $15,000. Accumulated Depreciation decreases to a zero balance (adebit) by $7,280.

These amounts are shown in the following journal entry:

Units-of-Production Method

We must record the depreciation expense for the first eight months of the year:

Notice that we did not need to multiply by 8/12 like we did in the straight-linemethod because the short period of use is already incorporated into the 8,000miles. If the truck had been used for a full year, the number of miles would havebeen bigger, and so depreciation expense would have been higher. With the units-of-production method, all that matters is the actual number of miles the truckwas driven.

Depreciation is recorded as:

Actual Rate2011 Depreciation

Expense8,000mil

"ees

$0.14 $1,120$ "

Calculate and recorddepreciation of plantassets

3

Calculate and recordthe disposal of plantassets

4

HarrCh09v1.qxd 10/23/06 9:15 AM Page 296

Page 329: polpoa_sg

Demo Doc 1 Solutions | Chapter 9 297

2011 Depreciation Expense ($15,000 $9,534)" ! $22/7 $1,561.71 12 months of depreciation

$1,

" "

5561.718 months

12 months$1,041 8 months

$ " " oofdepreciation

Calculate and recorddepreciation of plantassets

3

Calculate and recordthe disposal of plantassets

4

Journal Entry:

Sept. 1 Depreciation Expense 1,041 Accumulated Depreciation, Truck 1,041

PostDate Accounts Ref. Dr. Cr.

This amount brings the accumulated depreciation to:

$9,534 # $1,041 " $10,575

Cash was received, so it increases (a debit) by $7,000.

Journal Entry:

Sept. 1 Cash 7,000 Accumulated Depreciation, Truck 7,280 Loss on Sale of Truck (to balance) 720 Truck 15,000

PostDate Accounts Ref. Dr. Cr.

Part 1 Part 2 Part 3 Part 4 Part 5 Part 6 Part 7 Part 8 Part 9Demo DocComplete

So the completed entry is shown here:

Double-Declining-Balance Method

HarrCh09v1.qxd 10/23/06 9:15 AM Page 297

Page 330: polpoa_sg

298 Chapter 9 | Demo Doc 2

Journal Entry:

Sept. 1 Cash 7,000 Accumulated Depreciation, Truck 10,575 ????? ????? Truck 15,000

PostDate Accounts Ref. Dr. Cr.

The truck has been sold, so that account decreases to a zero balance (acredit) by $15,000. Accumulated Depreciation decreases to a zero balance (adebit) by $10,575.

These amounts are shown in the following journal entry:We need a $7,000 + $10,575 – $15,000 = $2,575 credit for the entry to work.Because the balancing amount is a credit, it is a gain (an increase in rev-

enues is a credit, which is like a gain).So the completed entry is:

Part 1 Part 2 Part 3 Part 4 Part 5 Part 6 Part 7 Part 8 Part 9Demo DocComplete

Journal Entry:

Sept. 1 Cash 7,000 Accumulated Depreciation, Truck 10,575 Gain on Sale of Truck (to balance) 2,575 Truck 15,000

PostDate Accounts Ref. Dr. Cr.

HarrCh09v1.qxd 11/22/06 10:24 AM Page 298

Page 331: polpoa_sg

Demo Doc 2 | Chapter 9 299

Demo Doc 2Natural Resource AssetsLearning Objective 5

Xander Inc. purchased a coal mine for $900 million cash on January 1, 2008.After the purchase, an independent analyst determined that the value of the landwas $200 million and that the value of the coal was $800 million (based on anestimate of 20 million tons of coal below the ground).

In 2008, Xander mined and sold 1 million tons of coal.

Requirements

1. Give Xander’s journal entry to record the purchase of the mine.

2. Give Xander’s journal entry to record depletion expense for 2008.

HarrCh09v1.qxd 11/22/06 10:24 AM Page 299

Page 332: polpoa_sg

300 Chapter 9 | Demo Doc 2 Solutions

Demo Doc 2 Solutions

Demo DocCompletePart 1 Part 2

Xander purchased two assets at one time for one price. In this lump-sum pur-chase, we need to determine how much of the purchase price to allocate to eachasset.

Making this distinction is important because it affects depletion calculationsin the future (because the cost of the asset is an important number in these cal-culations).

We use the independent valuations to determine a proportional value for theassets. According to the analyst, the total value of the assets purchased is:

$200 million # $800 million " $1 billion

Land has a proportion of 200,000,000/1,000,000,000 = 20%.The coal has a proportion of 800,000,000/1,000,000,000 = 80%.The cost of each asset is assigned as this proportion of the total cost.So the cost of the land is:

20% !! $900 million total cost " $180 million

The cost of the coal is:

80% !! $900 million total cost " $720 million

In the journal entry, Land and Coal Reserves are increased (debited) by theseamounts and Cash is decreased (credit).

Calculate and recordthe cost to acquireplant assets

2

Calculate and recorddepletion of naturalresources

5

Journal Entry:

Land (20% 3 $900,000,000) 180,000,000 Coal Reserves (80% 3 $900,000,000) 720,000,000 Cash 900,000,000

PostDate Accounts Ref. Dr. Cr.

Requirement 1

Give Xander’s journal entry to record the purchase of the mine.

HarrCh09v1.qxd 10/23/06 9:15 AM Page 300

Page 333: polpoa_sg

Demo Doc 2 Solutions | Chapter 9 301

Depletion expense is the same as depreciation expense, except that this term isonly used for natural resource assets. Depletion is always calculated using theunits-of-production method (never the straight-line or double-declining-balancemethods).

The units are the amount of natural resources purchased. In this case, theunits are tons of coal.

Under the unit method, we calculate depletion as:

When we journalize depreciation, we increase (debit) Depreciation Expense andincrease (credit) Accumulated Depreciation. The entry for depletion is the same,except that we use Depletion Expense (debit) and Accumulated Depletion(credit).

Depletion ExpenseThis Year

Actual Units Use" dd This Year Depletion Expense per Unit

Depl

$

eetion Expensefor 2008

1,000,000 tons $36 p" $ eer ton $36,000,000"

CostUnits of Prodduction

in Useful LifeDepletion Expense pe" rr Unit

$720,000,00020,000,000 tons

" $36 per tton

Journal Entry:

Depletion Expense ($36 3 1,000,000) 36,000,000 Accumulated Depletion, Coal Reserves 36,000,000

PostDate Accounts Ref. Dr. Cr.

Part 1 Part 2 Demo DocComplete

Requirement 2

Give Xander’s journal entry to record depletion expense for 2008.

Demo DocCompletePart 1 Part 2

Define and describethe life cycle of long-term assets

1

Calculate and recorddepletion of naturalresources

5

HarrCh09v1.qxd 10/23/06 9:15 AM Page 301

Page 334: polpoa_sg

302 Chapter 9 | Demo Doc 3

Demo Doc 3Intangible AssetsLearning Objective 6

On July 1, 2008, Franco Co. acquired a patent from Juarez Inc. for $5,000 cashand by signing a $10,000, 6% note payable. Franco believes that the patent willhave a life of 10 years.

On the same date, Franco purchased all outstanding shares of Germano Inc.for $50,000. The book value of Germano’s net assets at this time was $35,000 andthe market value was $40,000.

Requirements

1. Journalize Franco’s purchase of the patent. What kind of asset is thepatent? Why do you think so?

2. Journalize Franco’s amortization expense for the patent in 2008.

3. Franco did not make any interest payments on the note in 2008. JournalizeFranco’s interest expense for the year.

4. Calculate the amount of goodwill that will be recorded for Franco as aresult of the Germano purchase.

5. Give any necessary entry to adjust the value of Franco’s goodwill if it isdetermined to be worth $2,500 at the end of the year.

HarrCh09v1.qxd 10/23/06 9:15 AM Page 302

Page 335: polpoa_sg

Demo Doc 3 Solutions | Chapter 9 303

Part 1 Part 2 Part 3 Part 4 Part 5 Demo DocComplete

Demo Doc 3 Solutions

Cash decreases (a credit) by $5,000 and Notes Payable increases (a credit) by$10,000.

The Patent account is also increased (a debit) by:

$5,000 # $10,000 " $15,000

Journal Entry:

July 1 Patent 15,000 Cash 5,000 Note Payable 10,000

PostDate Accounts Ref. Dr. Cr.

Account for intangibleassets

6

Requirement 1

Journalize Franco’s purchase of the patent.What kind of asset is the patent?Why do you think so?

The patent is an intangible asset, because the patent is a right to produce acertain product or use a certain technology. A right is not a physical asset: It can-not be touched, which means it is intangible.

Requirement 2

Journalize Franco’s amortization expense for the patent in 2008.

Part 1 Part 2 Part 3 Part 4 Part 5 Demo DocComplete

Intangible assets are amortized. Amortization is essentially the same process asdepreciation for tangible assets. The only difference is that for intangible assets,we usually do not keep an Accumulated Amortization account, but insteaddirectly reduce the asset account.

Amortization expense is usually calculated using the straight-line method.

Amortization Expense (annual)Cost of Intan

"ggible Asset

Years of Useful Life

$15,00010

"yyears

$1,500 per year"

Account for intangibleassets

6

HarrCh09v1.qxd 10/23/06 9:15 AM Page 303

Page 336: polpoa_sg

304 Chapter 9 | Demo Doc 3 Solutions

Because the patent was purchased on July 1, only six months have been used.Therefore, we must calculate a partial year’s amortization:

Amortization Expense, Patents increases (a debit) by $750.Remember that for intangible assets, we do not have an accumulated

account. Instead we must decrease the Patent account (a credit) directly for $750.

$1,5006 months

12 months$750 amortization

$ "eexpense for 6

months

Journal Entry:

Dec. 31 Amoritization Expense, Patents 750 Patent 750

PostDate Accounts Ref. Dr. Cr.

Requirement 3

Franco did not make any interest payments on the note in 2008. JournalizeFranco’s interest expense for the year.

Part 1 Part 2 Part 3 Part 4 Part 5 Demo DocComplete

Interest expense for the year is:

Interest Expense is increased (a debit) and Interest Payable is increased (acredit) by $300.

$10,000 6%6 months

12 months$300$ $ "

Journal Entry:

Dec. 31 Interest Expense ($10,000 3 6% 3 6/12) 300 Interest Payable 300

PostDate Accounts Ref. Dr. Cr.

HarrCh09v1.qxd 10/23/06 9:15 AM Page 304

Page 337: polpoa_sg

Demo Doc 3 Solutions | Chapter 9 305

This figure for goodwill could also be calculated by preparing the journal entryfor Franco to purchase Germano.

Franco increases its Net Assets (debit) by the market value of $40,000. Cashdecreases (credit) by the amount paid for the purchase.

The remaining amount in the entry to make it balance is Goodwill:

Goodwill PurchasePrice

Market Value of NetA

" !sssets

$50,000 $40,000 $10,000" ! "

Account for intangibleassets

6

Journal Entry:

July 1 Net Assets 40,000 Goodwill (to balance) 10,000 Cash 50,000

PostDate Accounts Ref. Dr. Cr.

Requirement 4

Calculate the amount of goodwill that will be journalized for Franco as a resultof the Germano purchase.

Part 1 Part 2 Part 3 Part 4 Part 5 Demo DocComplete

HarrCh09v1.qxd 10/23/06 9:15 AM Page 305

Page 338: polpoa_sg

306 Chapter 9 | Demo Doc 3 Solutions

Requirement 5

Give any necessary entry to adjust the value of Franco’s goodwill if it isdetermined to be worth $2,500 at the end of the year.

Part 1 Part 2 Part 3 Part 4 Part 5 Demo DocComplete

Franco journalized the goodwill at $10,000. Because it is now worth only $2,500,it has a loss in value of:

$10,000 – $2,500 " $7,500

The loss increases the Loss on Goodwill account (a debit) and decreases theGoodwill account (a credit).

Journal Entry:

Dec. 31 Loss on Goodwill ($10,000 – $7,500) 7,500 Goodwill 7,500

PostDate Accounts Ref. Dr. Cr.

Part 1 Part 2 Part 3 Part 4 Part 5 Demo DocComplete

Account for intangibleassets

6

HarrCh09v1.qxd 10/23/06 9:15 AM Page 306

Page 339: polpoa_sg

Quick Practice Questions | Chapter 9 307

Quick Practice QuestionsTrue/False

_____ 1. The cost of land improvements includes fencing, paving, sprinkler sys-tems, and lighting.

_____ 2. Land improvements are not subject to annual depreciation.

_____ 3. Book value is equal to the cost of the asset less the expected residualvalue.

_____ 4. The modified accelerated cost recovery system of depreciation is usedfor income tax purposes and segments assets into classes by asset life.

_____ 5. A loss on sale of an asset occurs when the book value is less than thecash received.

_____ 6. The depreciable cost of a plant asset is the original cost less theexpected residual value.

_____ 7. Depletion expense is computed in the same manner as units-of-production.

_____ 8. Goodwill is recorded only by a company when it purchases anothercompany and is not subject to amortization.

_____ 9. A characteristic of a plant asset is that it is used in the production ofincome for a business.

_____10. Routine repairs and maintenance are capital expenditures.

HarrCh09v1.qxd 10/23/06 9:15 AM Page 307

Page 340: polpoa_sg

308 Chapter 9 | Quick Practice Questions

Multiple Choice

1. Which of the following is not a plant asset?a. Landb. Buildingc. Copyrightd. Equipment

2. The cost of a building would include all of the following except:a. Architectural fees b. Clearing and grading the land prior to construction of the building c. Cost of repairs made to an old building to get it ready for occupancy d. Costs of construction

3. Five hundred acres of land are purchased for $130,000. Additionalcosts include $5,000 brokerage commission, $10,000 for removalof an old building, $6,000 for paving, and an $800 survey fee.What is the cost of the land? a. $135,800b. $145,800 c. $155,000 d. $155,800

HarrCh09v1.qxd 10/23/06 9:15 AM Page 308

Page 341: polpoa_sg

Quick Practice Questions | Chapter 9 309

4. Westchester Company recently sold some used furniture for$3,800 cash. The furniture cost $19,600 and had accumulateddepreciation through the date of sale totaling $17,300. Whichentry is used to journalize the sale of the furniture?

Journal Entry:

a. Cash 3,800 Accumulated Depreciation, Furniture 15,800 Furniture 19,600

PostDate Accounts Ref. Dr. Cr.

Journal Entry:

b. Cash 3,800 Furniture 3,800

PostDate Accounts Ref. Dr. Cr.

Journal Entry:

c. Cash 3,800 Gain on Sale of Furniture 3,800

PostDate Accounts Ref. Dr. Cr.

Journal Entry:

d. Cash 3,800 Accumulated Depreciation, Furniture 17,300 Furniture 19,600 Gain on Sale of Furniture 1,500

PostDate Accounts Ref. Dr. Cr.

5. New equipment with a list price of $100,000, credit terms of 3/10,n/30, and transportation cost of $7,000 is acquired by a company.Insurance while in transit amounts to $200. Insurance on theequipment during its first year of use amounts to $800.Assuming the equipment is paid for within the discount period,what is the amount debited to Equipment? a. $97,000 b. $104,200 c. $105,000 d. $107,200

HarrCh09v1.qxd 10/23/06 9:15 AM Page 309

Page 342: polpoa_sg

6. Which of the following expenditures would be debited to anexpense account? a. Cost to replace the company car’s engine b. Addition of elevator to a building c. Replacement of tires d. All of the above

7. What is the effect of treating a revenue expenditure as a capitalexpenditure? a. Understates expenses and understates owner’s equity b. Overstates assets and overstates owner’s equity c. Overstates expenses and understates net income d. Understates expenses and understates assets

8. What type of account is Accumulated Depreciation?a. A contra-liability account b. An expense account c. A contra-asset account d. A contra-equity account

9. When the amount of use of a plant asset varies from year to year,which method of determining depreciation best matchesrevenues and expenses? a. Straight-line method b. Double-declining-balance method c. Units-of-production method d. Either the straight-line method or the double-declining-balance method

10. Which depreciation method generally results in the greatestdepreciation expense in the first full year of an asset’s life? a. Double-declining-balance methodb. Units-of-production methodc. Straight-line methodd. Either the straight-line or the double-declining-balance method

310 Chapter 9 | Quick Practice Questions

HarrCh09v1.qxd 10/23/06 9:15 AM Page 310

Page 343: polpoa_sg

Quick Exercises

9-1. Morgan Construction bought land, a building, and equipmentfor a lump sum of $740,000. Following are the appraised fairmarket values of the newly acquired assets:

Land, $450,000Building, $400,000Equipment, $150,000

Determine the cost of each asset.

a. ________ Landb. ________ Buildingc. ________ Equipment

9-2. Sue Glover purchased a tract of land and contracted with abuilder to build an office building on the property. She alsoengaged other contractors for lighting, fencing, paving, and soforth.

Based on the following transactions, determine the total costs allo-cated to the Land, Building, and Land Improvements accounts.

a. Purchased land for $135,000.b. Paid a contractor $333,000 to design and build the office building.c. Paid a demolition company $40,000 to remove an old structure on

the property.d. Paid $14,000 in delinquent taxes on the property.e. Paid $34,700 for fencing.f. Paid $39,500 for paving.g. Paid an electrical contractor $14,900 for outdoor lighting.

Cost of land __________Cost of building __________Cost of land improvements __________

9-3. Venus Company acquired equipment on January 1, 2008, for$470,000. The equipment has an estimated useful life of 5years and an estimated residual value of $30,000. Calculatedepreciation expense for 2008 and 2009 under each of the fol-lowing methods. The equipment is estimated to produce150,000 units. During 2008 and 2009, the equipment produced24,000 and 60,000 units, respectively. Round the answer to thenearest dollar where necessary.

2008 2009a. Straight-line method _________ __________b. Double-declining-balance method _________ __________c. Units-of-production method _________ __________

Quick Practice Questions | Chapter 9 311

HarrCh09v1.qxd 10/23/06 9:15 AM Page 311

Page 344: polpoa_sg

312 Chapter 9 | Quick Practice Questions

9-4. On April 1, 2008, Carter Craft & Company purchased a min-eral deposit by paying $50,000 in cash and signing a $440,000promissory note. A geological report estimated the mineraldeposit contained 140,000 tons of ore. Carter Craft & Companyexpects the asset to have a zero residual value when fullydepleted. During 2008, 40,000 tons of ore were mined.

Prepare the journal entry for December 31, 2008, to record the deple-tion of the mineral deposit.

Journal Entry:

PostDate Accounts Ref. Dr. Cr.

Journal Entry:

PostDate Accounts Ref. Dr. Cr.

Journal Entry:

PostDate Accounts Ref. Dr. Cr.

9-5. On July 31, 2007, Austin Manufacturing acquired an existingpatent for $340,000. The remaining legal life of the patent is 13years; however, management thinks the patent will hold eco-nomic benefit for the company for only 7 more years.

Prepare journal entries for July 31, 2007, to acquire the patent andfor December 31, 2007, to amortize the patent.

HarrCh09v1.qxd 10/23/06 9:15 AM Page 312

Page 345: polpoa_sg

Do It Yourself! Question 1 | Chapter 9 313

Calculate and recorddepreciation of plantassets

3

Depreciation Accumulated Book Value (Cost – Year Expense Depreciation Accumulated Depreciation)

2008

2009

2010

Calculate and recorddepreciation of plantassets

3

Calculate and recorddepreciation of plantassets

3

Depreciation Accumulated Book Value (Cost – Year Expense Depreciation Accumulated Depreciation)

2008

2009

2010

Do It Yourself! Question 1

DepreciationWinters Co. purchased equipment for $8,000 cash on January 1, 2008. Theequipment had a residual value of $500 and a useful life of 6 years or 2,000hours of operation. Winters has a December 31 year-end.

The equipment was used for 400 hours in 2008, 200 hours in 2009, and 300hours in 2010.

Requirements

1. Calculate the Depreciation Expense and Accumulated Depreciation accountbalances at December 31, 2008, 2009, and 2010, using the straight-linemethod.

2. Calculate the Depreciation Expense and Accumulated Depreciation accountbalances at December 31, 2008, 2009, and 2010, using the units-of-production method.

3. Calculate the Depreciation Expense and Accumulated Depreciation accountbalances at December 31, 2008, 2009, and 2010, using the double-declining-balance method.

Depreciation Accumulated Book Value (Cost – Year Expense Depreciation Accumulated Depreciation)

2008

2009

2010

HarrCh09v1.qxd 10/23/06 9:15 AM Page 313

Page 346: polpoa_sg

4. Using the units-of-production method only, show how the equipmentwould look on the December 31 balance sheets for 2008, 2009, and 2010.

5. Winters sold the equipment on March 1, 2011, for $4,000 cash. Showjournal entries for depreciation and for the sale transaction using eachmethod. (The equipment was used for 100 hours in 2011.)

Straight-Line Method

314 Chapter 9 | Do It Yourself! Question 1

Journal Entry:

PostDate Accounts Ref. Dr. Cr.

Journal Entry:

PostDate Accounts Ref. Dr. Cr.

Journal Entry:

PostDate Accounts Ref. Dr. Cr.

Report long-termassets on the balancesheet

7

Calculate and recorddepreciation of plantassets

3

Calculate and recordthe disposal of plantassets

4

Units-of-Production Method

HarrCh09v1.qxd 10/23/06 9:15 AM Page 314

Page 347: polpoa_sg

Do It Yourself! Question 1 | Chapter 9 315

Double-Declining-Balance Method

Journal Entry:

PostDate Accounts Ref. Dr. Cr.

Journal Entry:

PostDate Accounts Ref. Dr. Cr.

Journal Entry:

PostDate Accounts Ref. Dr. Cr.

HarrCh09v1.qxd 10/23/06 9:15 AM Page 315

Page 348: polpoa_sg

316 Chapter 9 | Do It Yourself! Question 2

Calculate and recorddepletion of naturalresources

5

Journal Entry:

PostDate Accounts Ref. Dr. Cr.

Do It Yourself! Question 2

Natural Resource Assets

Woody Inc. purchased logging rights in a county forest for $800,000 cash. Woodyestimates that 40,000 tons of lumber can be harvested from the forest. BecauseWoody only purchased the right to log, it does not own the land.

Requirement

Show the journal entry for Woody’s depletion expense for the first year,assuming that 15,000 tons of lumber were cut and sold.

HarrCh09v1.qxd 10/23/06 9:15 AM Page 316

Page 349: polpoa_sg

Do It Yourself! Question 3 | Chapter 9 317

Journal Entry:

PostDate Accounts Ref. Dr. Cr.

Account for intangibleassets

6

Account for intangibleassets

6

Journal Entry:

PostDate Accounts Ref. Dr. Cr.

2. Journalize Kevin’s amortization expense for the trademark in 2008.

Do It Yourself! Question 3

Intangible Assets

On October 1, 2008, Kevin Inc. acquired a trademark from Daniel Co. for $10,000cash. Kevin believes that the trademark will have a life of 20 years.

Requirements

1. Journalize Kevin’s purchase of the trademark.

HarrCh09v1.qxd 10/23/06 9:15 AM Page 317

Page 350: polpoa_sg

Quick Practice SolutionsTrue/False

T 1. The cost of land improvements includes fencing, paving, sprinkler systems, and lighting. (p. 473)

F 2. Land improvements are not subject to annual depreciation.False–Land improvements are subject to depreciation. (p.473)

F 3. Book value is equal to the cost of the asset less the expected residual value.False–Book value is equal to the cost of the asset less the accumulated depreciation. (p. 478)

T 4. The modified accelerated cost recovery system of depreciation is used for income tax purposes and segments assets into classes by asset life. (p. 483)

F 5. A loss on a sale of an asset occurs when the book value is less thanthe cash received.False–A gain on a sale of an asset occurs when the book value is less than the cash received. (p. 485)

T 6. The depreciable cost of a plant asset is the original cost less the expected residual value. (p. 476)

T 7. Depletion expense is computed in the same manner as units-of-production. (p. 487)

T 8. Goodwill is recorded only by a company when it purchases anothercompany and is not subject to amortization. (pp. 489–490)

T 9. A characteristic of a plant asset is that it is used in the productionof income for a business. (p. 470)

F 10. Routine repairs and maintenance are capital expenditures.False–Routine repairs and maintenance are expenses, or revenueexpenditures. (p. 475)

318 Chapter 9 | Quick Practice Solutions

HarrCh09v1.qxd 10/23/06 9:15 AM Page 318

Page 351: polpoa_sg

Multiple Choice

1. Which of the following is not a plant asset? (p. 489)a. Landb. Buildingc. Copyrightd. Equipment

2. The cost of a building would include all of the following except:(p. 473)a. Architectural fees b. Clearing and grading the land prior to construction of the buildingc. Cost of repairs made to an old building to get it ready for occupancyd. Costs of construction

3. Five hundred acres of land are purchased for $130,000.Additional costs include $5,000 brokerage commission, $10,000for removal of an old building, $6,000 for paving, and an $800survey fee. What is the cost of the land? (p. 473)a. $135,800b. $145,800 ($130,000 + $5,000 + $10,000 + $800) c. $155,000 d. $155,800

Quick Practice Solutions | Chapter 9 319

HarrCh09v1.qxd 10/23/06 9:15 AM Page 319

Page 352: polpoa_sg

320 Chapter 9 | Quick Practice Solutions

Journal Entry:

a. Cash 3,800 Accumulated Depreciation, Furniture 15,800 Furniture 19,600

PostDate Accounts Ref. Dr. Cr.

Journal Entry:

b. Cash 3,800 Furniture 3,800

PostDate Accounts Ref. Dr. Cr.

Journal Entry:

c. Cash 3,800 Gain on Sale of Furniture 3,800

PostDate Accounts Ref. Dr. Cr.

Journal Entry:

d. Cash 3,800 Accumulated Depreciation, Furniture 17,300 Furniture 19,600 Gain on Sale of Furniture 1,500

PostDate Accounts Ref. Dr. Cr.

4. Westchester Company recently sold some used furniture for$3,800 cash. The furniture cost $19,600 and had accumulateddepreciation through the date of sale totaling $17,300. Whichentry is used to journalize the sale of the furniture? (pp. 473–474)

HarrCh09v1.qxd 10/23/06 9:15 AM Page 320

Page 353: polpoa_sg

5. New equipment with a list price of $100,000, credit terms of3/10, n/30, and transportation cost of $7,000 is acquired by a com-pany. Insurance while in transit amounts to $200. Insurance onthe equipment during its first year of use amounts to $800.Assuming the equipment is paid for within the discount period,what is the amount debited to Equipment? (pp. 473–474)a. $ 97,000 b. $104,200 ($100,000 – $3,000 + $7,000 + $200)c. $105,000 d. $107,200

6. Which of the following expenditures would be debited to anexpense account? (p. 475)a. Cost to replace the company car’s engine b. Addition of elevator to a building c. Replacement of tiresd. All of the above

7. What is the effect of treating a revenue expenditure as a capitalexpenditure? (p. 475)a. Understates expenses and understates owner’s equity b. Overstates assets and overstates owner’s equity c. Overstates expenses and understates net income d. Understates expenses and understates assets

8. What type of account is Accumulated Depreciation? (p. 484)a. A contra-liability account b. An expense account c. A contra-asset accountd. A contra-equity account

9. When the amount of use of a plant asset varies from year to year,which method of determining depreciation best matchesrevenues and expenses? (p. 478)a. Straight-line method b. Double-declining-balance method c. Units-of-production methodd. Either the straight-line method or the double-declining-balance method

10. Which depreciation method generally results in the greatestdepreciation expense in the first full year of an asset’s life? (p. 483)a. Double-declining-balance methodb. Units-of-production methodc. Straight-line methodd. Either the straight-line or the double-declining-balance method

Quick Practice Solutions | Chapter 9 321

HarrCh09v1.qxd 10/23/06 9:15 AM Page 321

Page 354: polpoa_sg

Quick Exercises

9-1. Morgan Construction bought land, a building, and equipmentfor a lump sum of $740,000. Following are the appraised fairmarket values of the newly acquired assets:

Land, $450,000Building, $400,000Equipment, $150,000

Determine the cost of each asset. (p. 474)

a. $333,000 Land = ($450,000/$1,000,000) ! $740,000 = $333,000b. $296,000 Building = ($400,000/$1,000,000) ! $740,000 = $296,000c. $111,000 Equipment = ($150,000/$1,000,000) ! $740,000 = $111,000

9-2. Sue Glover purchased a tract of land and contracted with abuilder to build an office building on the property. She alsoengaged other contractors for lighting, fencing, paving, and soforth.

Based on the following transactions, determine the total costs allocatedto the Land, Building, and Land Improvements accounts. (p. 473)

a. Purchased land for $135,000.b. Paid a contractor $333,000 to design and build the office building.c. Paid a demolition company $40,000 to remove an old structure on

the property.d. Paid $14,000 in delinquent taxes on the property.e. Paid $34,700 for fencing.f. Paid $39,500 for paving.g. Paid an electrical contractor $14,900 for outdoor lighting.

Cost of land $189,000 ($135,000 + $40,000 + $14,000= $189,000; transactions a, c, and d)

Cost of building $333,000 (transaction b)Cost of land improvements $ 89,100 ($34,700 + $39,500 + $14,900

= $89,100; transactions e, f, and g)

9-3. Venus Company acquired equipment on January 1, 2008, for$470,000. The equipment has an estimated useful life of 5years and an estimated residual value of $30,000. Calculatedepreciation expense for 2008 and 2009 under each of the fol-lowing methods. The equipment is estimated to produce150,000 units. During 2008 and 2009, the equipment produced24,000 and 60,000 units, respectively. Round the answer to thenearest dollar where necessary. (pp. 477–479)

2008 2009a. Straight-line method $ 88,000 $ 88,000

b. Double-declining-balance method $188,000 $112,800DDB rate = 1/5 years ! 2 = 40% (0.40)($470,000 – 0) ! 0.40 = $188,000($470,000 – $188,000) ! 0.40 = $112,800

$470,000 – $30,000 = $88,000/year5 years

322 Chapter 9 | Quick Practice Solutions

HarrCh09v1.qxd 10/23/06 9:15 AM Page 322

Page 355: polpoa_sg

c. Units-of-production method $ 70,400 $176,000

9-4. On April 1, 2008, Carter Craft & Company purchased a min-eral deposit by paying $50,000 in cash and signing a $440,000promissory note. A geological report estimated the mineraldeposit contained 140,000 tons of ore. Carter Craft & Companyexpects the asset to have a zero residual value when fullydepleted. During 2008, 40,000 tons of ore were mined.

Prepare the journal entry for December 31, 2008, to record the deple-tion of the mineral deposit. (pp. 487–488)

$470,000 – $30,000 $ 60,000 units = $176,000

150,000

$470,000 – $30,000$ 24,000 units = $70,400

150,000

Quick Practice Solutions | Chapter 9 323

Journal Entry:

12/31/08 Depletion Expense 140,000 Accumulated Depletion, Ore 140,000 To record depletion of mineral deposits: ($50,000 + $440,000/140,000 tons = $3.50/ton 3 40,000 tons = $140,000).

PostDate Accounts Ref. Dr. Cr.

9-5. On July 31, 2007, Austin Manufacturing acquired an existingpatent for $340,000. The remaining legal life of the patent is 13years; however, management thinks the patent will hold eco-nomic benefit for the company for only 7 more years.

Prepare journal entries for July 31, 2007, to acquire the patent andfor December 31, 2007, to amortize the patent. (pp. 488–489)

Journal Entry:

7/31/07 Patents 340,000 Cash 340,000 To record purchase of patent.

PostDate Accounts Ref. Dr. Cr.

Journal Entry:

12/31/07 Amoritization Expense, Patents 20,238 Patents 20,238 To amoritize patents ($340,000 3 5/84 months = $20,238).

PostDate Accounts Ref. Dr. Cr.

HarrCh09v1.qxd 10/23/06 9:15 AM Page 323

Page 356: polpoa_sg

324 Chapter 9 | Do It Yourself! Question 1 Solutions

Depreciation Accumulated Book Value (Cost – Year Expense Depreciation Accumulated Depreciation)

2008 $1,250 $1,250 $6,750

2009 1,250 2,500 5,500

2010 1,250 3,750 4,250

Actual Rate Expense

2008 400 hours ! $3.75 per hour = $1,500

2009 200 hours ! $3.75 per hour = $ 750

2010 300 hours ! $3.75 per hour = $1,125

Prior EndingAccumulated Expense Accumulated

2008 $0 + $1,500 = $1,500

2009 $1,500 + $750 = $2,250

2010 $2,250 + $1,125 = $3,375

Accumulated depreciation each year is:

Requirement 2

Calculate the Depreciation Expense and Accumulated Depreciation accountbalances at December 31, 2008, 2009, and 2010, using the units-of-productionmethod.

Depreciation expense each year is:

$8,000 $500 $3.75 Depreciation2,000

!

hhours oflifetime operation

Expense per Hou" rr of ActualOperation

Do It Yourself! Question 1 Solutions

Requirement 1

Calculate the Depreciation Expense and Accumulated Depreciation accountbalances at December 31, 2008, 2009, and 2010, using the straight-line method.

$8,000 $5006 years

$1,250 DepreciationExpen

!"

sse per Year

2008 Accumulated Depreciation $" 11,2502009 Accumulated Depreciation $2,5002

"0010 Accumulated Depreciation $3,750"

HarrCh09v1.qxd 10/23/06 9:15 AM Page 324

Page 357: polpoa_sg

Do It Yourself! Question 1 Solutions | Chapter 9 325

Depreciation Accumulated Book Value (Cost – Year Expense Depreciation Accumulated Depreciation)

2008 $1,500 $1,500 $6,500

2009 750 2,250 5,750

2010 1,125 3,375 4,625

Depreciation Accumulated Book Value (Cost –Year Expense Depreciation Accumulated Depreciation)

A

2008 $2,667 $2,667 $5,333

2009 1,778 4,445 3,555

2010 1,185 5,630 2,370

2008 2009 2010

Equipment $8,000 $8,000 $8,000

– Accumulateddepreciation (1,500) (2,250) (3,375)

Equipment (net) $6,500 $5,750 $4,625

Requirement 3

Calculate the Depreciation Expense and Accumulated Depreciation accountbalances at December 31, 2008, 2009, and 2010, using the double-declining-balance method.

DDB rate 2/6 1/3

2008 Depreciation Expens

" "

ee ($8,000 $0) 1/3 $2,667Accumulated Deprec

" ! "!

iiation $0 $2,667 $2,667

2009 Depreciation Ex

" # "

ppense ($8,000 $2,667) 1/3 $1,778Accumulate

" ! $ "

dd Depreciation $2,667 $1,778 $4,445

2010 Dep

" # "

rreciation Expense ($8,000 $4,445) 1/3 $1,1" ! $ " 885Accumulated Depreciation $4,445 $1,185 $" # " 55,630

Requirement 4

Using the units-of-production method only, show how the equipment wouldlook on the December 31 balance sheets for 2008, 2009, and 2010.

HarrCh09v1.qxd 10/23/06 9:15 AM Page 325

Page 358: polpoa_sg

326 Chapter 9 | Do It Yourself! Question 1 Solutions

Journal Entry:

Mar. 1 Depreciation Expense 208 Accumulated Depreciation, Equipment 208

PostDate Accounts Ref. Dr. Cr.

Total accumulated depreciation is:

$3,750 ! $208 " $3,958

Journal Entry:

Mar. 1 Cash 4,000 Accumulated Depreciation, Equipment 3,958 Loss on Sale of Equipment (to balance) 42 Equipment 8,000

PostDate Accounts Ref. Dr. Cr.

Units-of-Production Method

Actual Rate2011Depreciation

Expense100 hours" ## "$3.75 $375

Journal Entry:

Mar. 1 Depreciation Expense 375 Accumulated Depreciation, Equipment 375

PostDate Accounts Ref. Dr. Cr.

Total accumulated depreciation is:

$3,375 ! $375 " $3,750

Requirement 5

Winters sold the equipment on March 1, 2011, for $4,000 cash. Show journalentries for depreciation and for the sale transaction using each method. (Theequipment was used for 100 hours in 2011.)

Straight-Line Method

2011Depreciation

Expense$1,250

2 months12 m

" #oonths

$208"

HarrCh09v1.qxd 11/22/06 10:25 AM Page 326

Page 359: polpoa_sg

Double-Declining-Balance Method

2011 Depreciation Expense ($8,000 $5,630) 1" ! $ //3 $790 for 12 months

$7902 months

12 month

"

$ss

$132 for 2months

"

Do It Yourself! Question 1 Solutions | Chapter 9 327

Journal Entry:

Mar. 1 Cash 4,000 Accumulated Depreciation, Equipment 3,750 Loss on Sale of Equipment (to balance) 250 Equipment 8,000

PostDate Accounts Ref. Dr. Cr.

Journal Entry:

Mar. 1 Depreciation Expense 132 Accumulated Depreciation, Equipment 132

PostDate Accounts Ref. Dr. Cr.

Total accumulated depreciation is:

$5,630 # $132 " $5,762

Journal Entry:

Mar. 1 Cash 4,000 Accumulated Depreciation, Equipment 5,762 Gain on Sale of Equipment (to balance) 1,762 Equipment 8,000

PostDate Accounts Ref. Dr. Cr.

HarrCh09v1.qxd 10/23/06 9:15 AM Page 327

Page 360: polpoa_sg

328 Chapter 9 | Do It Yourself! Question 2 Solution

Journal Entry:

Depletion Expense ($20 ! 15,000) 300,000 Accumulated Depletion, Lumber 300,000

PostDate Accounts Ref. Dr. Cr.

Do It Yourself! Question 2 Solution

Requirement

Show the journal entry for Woody’s depletion expense for the first year,assuming that 15,000 tons of lumber were cut and sold.

DepletionExpense per Ton

$800,00040,000 ton

"ss

$20 per ton

15,000 tons $20 per ton $300,

"

$ " 0000

HarrCh09v1.qxd 10/23/06 9:15 AM Page 328

Page 361: polpoa_sg

Do It Yourself! Question 3 Solutions | Chapter 9 329

Journal Entry:

Oct. 1 Trademark 10,000 Cash 10,000

PostDate Accounts Ref. Dr. Cr.

Journal Entry:

Dec. 31 Amortization Expense, Trademarks 125 Trademark 125

PostDate Accounts Ref. Dr. Cr.

Do It Yourself! Question 3 Solutions

Requirement 1

Journalize Kevin’s purchase of the trademark.

Requirement 2

Journalize Kevin’s amortization expense for the trademark in 2008.

AmortizationExpense

$10,00020 years

$500 pe" " rr year

$5003 months

12 months$125 for 3 mo$ " nnths

HarrCh09v1.qxd 10/23/06 9:15 AM Page 329

Page 362: polpoa_sg

The Power of PracticeFor more practice using the skills learned in this chapter, visit MyAccountingLab.There you will find algorithmically generated questions that are based on theseDemo Docs and your main textbook’s Review and Accounting Practice sections.

To go to MyAccountingLab and follow these steps:

1. Direct your URL to www.myaccountinglab.com.2. Log in using your name and password.3. Click the MyAccountingLab link.4. Click Study Plan in the left navigation bar.5. From the table of contents, select Chapter 9, Long-Term Assets: Plant Assets

and Intangibles.6. Click a link to work on the tutorial exercises.

330 Chapter 9 | The Power of Practice

HarrCh09v1.qxd 10/23/06 9:15 AM Page 330

Page 363: polpoa_sg

Current Liabilitiesand Payroll10

1

WHAT YOU PROBABLY ALREADY KNOW

You probably already know that as an employee, you do not receive inyour paycheck an amount equal to the number of hours worked timesyour hourly rate. The total that you earn is called gross pay, which isthe amount before withholdings are deducted. You probably noticedthat money is withheld for federal income tax, social security tax,Medicare tax, and state income tax, if applicable in your state. Youdid not request that these amounts be deducted; they are requiredwithholdings that your employer must make. Your employer does notkeep this money; it is remitted to the appropriate taxing agencies.When you receive your W-2 form by January 31 of the succeedingyear, you can see that you are given credit for the amount of thesetaxes your employer withheld on your behalf. In this chapter, we willsee how your employer accounts for your paycheck.

Learning Objectives

Account for current liabilities of known amount.

Some current liabilities that are recorded at known amounts includeaccounts payable, short-term notes payable, sales tax payable, currentportion of long-term notes payable, accrued expenses or liabilities, andunearned or deferred revenues. Review the “Current Liabilities ofKnown Amount” section of the textbook, and be sure to take note of thepresentation of current liabilities in the balance sheet for Bob’s SportingGoods on page 524.

Account for estimated and contingent liabilities.

Sometimes a liability has been incurred but the amount is uncertain. An example of this would be the estimated warranty payable thatmanufacturers may report as a liability. A contingent liability is only apossible obligation that may occur depending upon future events or circumstances. A lawsuit is an example of a contingent liability.Review Exhibit 10-2 (p. 529) for contingent liability classifications andtreatments.

Calculate the current ratio and working capital.

The current ratio and working capital are two key measures of liquidity.The current ratio indicates the amount of current assets available foreach dollar of current liabilities. A higher ratio is usually consideredpreferable. Working capital = current assets – current liabilities. A largeramount of working capital is usually considered preferable; certainlycurrent assets should exceed current liabilities.

2

3

HarrCh10v1.qxd 10/23/06 9:17 AM Page 331

Page 364: polpoa_sg

332 Chapter 10 | Current Liabilities and Payroll

4 Calculate payroll amounts.

The gross pay is the total amount earned by the employee and includessuch items as the salary amount, hourly pay rate multiplied by the hoursworked, commissions, bonuses, and overtime. Net pay is the amount theemployee receives, which is equal to the gross pay less withholdings.Payroll deductions may include income taxes withheld, social security(FICA) tax, insurance premiums, retirement savings, and charitablecontributions. Review “Accounting for Payroll” in the main text.

Describe the payroll process.

A number of documents are involved in a payroll system. Some of thesedocuments include time cards or time sheets, payroll tax forms, W-2Employee Wage and Tax statements, and the employee earnings record.

The employer files a payroll tax form with the state and federalgovernments. It includes information such as gross wages, incometaxes withheld, and Social Security and Medicare taxes.

An employee earnings record contains all of the payroll informationfrom gross pay to net pay, by employee. This record is used by theemployer to prepare the W-2 forms, Employee Wage and TaxStatements. Both the employee and IRS receive copies of this form forincome tax filing.

Review the payroll documents in Exhibits 10-6, 10-9 and 10-10. (pp. 535, 537, and 538).

A payroll register contains all of the payroll information for eachemployee on each pay date. Review the payroll register in Exhibit 10-7(p. 536).

Record basic payroll transactions.

To record the periodic payroll, gross pay is debited to Salary Expense.All of the amounts withheld and the net pay due to the employees arecredited to current liability accounts. Employers are also liable forpayroll taxes. FICA and state and federal unemployment payroll taxexpenses are incurred by employers and must be paid.

Most employers offer their employees some benefits such as healthinsurance or retirement benefits. Similar to the gross payroll and payrolltaxes, the benefits are additional expenses to the employer. Review the“Payroll Entries” section of the main textbook, particularly the journalentries involved.

Report current liabilities on the balance sheet.

Review the current liabilities section of the balance sheet at Exhibit 10-11(p. 542).

5

6

7

HarrCh10v1.qxd 10/23/06 9:17 AM Page 332

Page 365: polpoa_sg

Demo Doc 1 | Chapter 10 333

Demo Doc 1General Current LiabilitiesLearning Objectives 1–3, 7

Freddie Enterprises sells products with one-year warranties included in the sell-ing price. During August 2008, Freddie sold goods for $250,000 cash. These goodscost $180,000 to manufacture. Freddie is required by law to collect 7% sales taxon all sales.

Freddie estimates warranty costs to be 1.5% of the selling price. During August2008, Freddie made $3,000 of repairs under warranty (paid in cash to a repairservice).

On August 31, 2008, Freddie remitted all sales tax collected in August to thestate government.

Requirements

1. Journalize all of Freddie’s transactions in the month of August 2008.

2. Is sales tax payable a contingent liability? Why or why not?

3. Is estimated warranty payable a contingent liability? Why or why not?

4. At August 31, 2008, the only other current liability Freddie had wasaccounts payable of $1,000. Current assets were $1,900. What is Freddie’scurrent ratio?

HarrCh10v1.qxd 10/23/06 9:17 AM Page 333

Page 366: polpoa_sg

334 Chapter 10 | Demo Doc 1 Solutions

Journal Entry:

Cash ($250,000 ! 1.07) 267,500 Sales Revenue 250,000 Sales Tax Payable ($250,000 ! 7%) 17,500

PostDate Accounts Ref. Dr. Cr.

2 Account for estimatedand contingent liabilities

1 Account for current liabilities of knownamount

Part 1 Part 2 Part 3 Demo DocComplete

Demo Doc 1 SolutionsRequirement 1

Journalize all of Freddie’s transactions in the month of August 2008.

During August 2008, Freddie sold goods for $250,000 cash. Thesegoods cost $180,000 to manufacture. Freddie is required by law tocollect 7% sales tax on all sales.

Freddie sold $250,000 worth of products, which increases Sales Revenue (acredit) by $250,000.

However, the cash that Freddie collected was more than $250,000 because itincluded the sales tax.

Freddie collected $250,000 ! (1 + 7%) = $267,500 cash from the customer.

The $250,000 ! 7% = $17,500 Freddie collected in sales tax is not revenuebecause it was not earned by Freddie and does not belong to Freddie. Thesetaxes belong to the government and are owed/payable by Freddie to the gov-ernment. Therefore, we increase (a credit) the Sales Tax Payable account by$17,500.

Freddie sold these goods, and so an adjustment to Inventory is necessary aswell. COGS increases (a debit) and Inventory decreases (a credit) by$180,000, Freddie’s cost of the products.

Journal Entry:

Cost of Goods Sold 180,000 Inventory 180,000

PostDate Accounts Ref. Dr. Cr.

Freddie estimates warranty costs to be 1.5% of the selling price.

We must also account for the warranties included in the selling price of thegoods. Once the products are sold, the warranty is in effect. The warranty

HarrCh10v1.qxd 10/23/06 9:17 AM Page 334

Page 367: polpoa_sg

Demo Doc 1 Solutions | Chapter 10 335

means that Freddie has an obligation (that is, a liability) to fix the productsif they break down. We must journalize an Estimated Warranty Payable lia-bility (a credit) of:

1.5% ! $250,000 " $3,750

When the liability is journalized, so is the Estimated Warranty Expense (adebit) of $3,750. This journal entry is an example of good matching (asrequired by the matching principle under GAAP) because the expense isrecorded at the same time as the sales revenue, not when the actual cost isincurred.

Journal Entry:

Estimated Warranty Payable 3,000 Cash 3,000

PostDate Accounts Ref. Dr. Cr.

Journal Entry:

Warranty Expense ($250,000 ! 0.015) 3,750 Estimated Warranty Payable 3,750

PostDate Accounts Ref. Dr. Cr.

During August 2008, Freddie made $3,000 of repairs under warranty(paid in cash to a repair service).

When Freddie Enterprises makes warranty repairs, it is meeting its war-ranty obligation (that is, it is reducing its warranty liability). These repairsdecrease Estimated Warranty Payable (a debit) by $3,000.

Because the repairs were paid for in cash, the Cash account also decreases (acredit) by $3,000.

Notice that the Warranty Expense account is not affected by the repairs. Theexpense was already journalized at the time of sale. To debit it again nowwould be double-counting the expense.

Note that $3,750 – $3,000 = $750 is left in the Estimated Warranty Payableaccount. This amount remains to cover any future repairs that might bemade under the warranty.

On August 31, 2008, Freddie remitted all sales tax collected inAugust to the state government.

In the first transaction, Freddie collected $17,500 in sales tax (7% of the saleof $250,000 worth of products). Sales tax payable is just like any otherpayable account: As payments are made, both Cash and the payabledecrease.

HarrCh10v1.qxd 10/23/06 9:17 AM Page 335

Page 368: polpoa_sg

336 Chapter 10 | Demo Doc 1 Solutions

7 Report current liabilitieson the balance sheet

3 Calculate current ratioand working capital

Part 1 Part 2 Part 3 Demo DocComplete

Part 1 Part 2 Part 3 Demo DocComplete

Journal Entry:

Sales Tax Payable 17,500 Cash 17,500

PostDate Accounts Ref. Dr. Cr.

7 Report current liabilitieson the balance sheet

Requirement 2

Is sales tax payable a contingent liability? Why or why not?

Sales taxes must be collected and remitted to the government by law. Ethicalcompanies have no choice but to meet the sales tax payable obligation. Therefore,this amount will be paid and it is not a contingent liability; it is not dependentupon any outside event.

Requirement 3

Is estimated warranty payable a contingent liability? Why or why not?

The estimated warranty payable is an estimate. It is not known for sure whetherthe products will break down, or how much it might cost to repair them if they do.For this reason, estimated warranty payable is a contingent liability because itdepends upon the performance of the products after they leave Freddie’s control.

Because payment for warranty repairs is probable and estimable, warrantyexpense and estimated warranty payable are recorded in a journal entry, eventhough they are contingent upon outside events.

Requirement 4

At August 31, 2008, the only other current liability Freddie had was accountspayable of $1,000. Current assets were $1,900.What is Freddie’s current ratio?

We know that Freddie’s current assets are $1,900, and current liabilities are thebalance of the Estimated Warranty Payable account ($750) and the accountspayable of $1,000.

Current RatioCurrent Assets

Current Liabili"

tties

HarrCh10v1.qxd 10/23/06 9:17 AM Page 336

Page 369: polpoa_sg

Demo Doc 1 Solutions | Chapter 10 337

So Freddie’s current ratio is calculated as follows:

Current Ratio$1,9 0

($750 $1,000)

$1,900$1,

"#

"

0

7750

" 1.09

Part 1 Part 2 Part 3 Demo DocComplete

HarrCh10v1.qxd 10/23/06 9:17 AM Page 337

Page 370: polpoa_sg

338 Chapter 10 | Demo Doc 2

Demo Doc 2Current Portion of Long-Term DebtLearning Objectives 1, 7

On August 1, 2008, Squirrel Co. signed a $400,000 note payable. Squirrel agreedto pay back $25,000 per month, beginning on January 1, 2009, and ending onApril 1, 2010. Interest of 12% is paid monthly beginning January 1, 2009.

Requirement:

Show the presentation of this note on Squirrel’s December 31, 2008, balancesheet.

HarrCh10v1.qxd 10/23/06 9:17 AM Page 338

Page 371: polpoa_sg

Demo Doc 2 Solutions | Chapter 10 339

Demo DocCompletePart 1

Demo DocCompletePart 1

1 Account for current liabilities of knownamount

7 Report current liabilitieson the balance sheet

Demo Doc 2 SolutionRequirement

Show the presentation of this note on Squirrel’s December 31, 2008, balancesheet.

Of the $400,000 note, payments totaling $25,000 ! 12 = $300,000 will be repaidwithin the next year.

This $300,000 is the current portion of the note because it will be repaidwithin the next year.

The remaining $400,000 – $300,000 = $100,000 will be repaid in more thanone year, so it is the long-term portion of the note.

Interest was incurred on the entire $400,000 liability balance that was out-standing over the five months since August 1, 2008.

The $20,000 of interest incurred is journalized as Interest Payable. This interestpayable is current because it is due in less than one year.Note that this $20,000 is not the total interest due on the note. It is only theamount that has already been incurred at December 31, 2008.

Current Liabilities

Current portion of note payable $300,000Interest payable 20,000

Long-Term Liabilities

Notes payable (net of current portion) $100,000

InterestIncurred Total Debt Interest

RateFr" ! ! aaction of

Year

$400,000 12%5

12" ! !

" $20,000

HarrCh10v1.qxd 10/23/06 9:17 AM Page 339

Page 372: polpoa_sg

340 Chapter 10 | Demo Doc 3

Payroll LiabilitiesLearning Objectives 4–6

Gannon Corp. employees earn a total of $500,000 gross pay per week. All employ-ees have the following items withheld from their pay:

15% Income taxes

8% FICA taxes

4% Pension contributions

2% Union dues

Gannon pays the following payroll taxes:

8% FICA taxes

6% Unemployment taxes

Requirements

1. For a normal week, journalize the following transactions:

a. Cash payment of employee salaries

b. Accrue Gannon’s payroll taxes

c. Gannon’s payment of all payroll taxes

d. Gannon’s payment of union dues

e. Gannon’s payment of pension contributions

2. What is the employees’ net pay in a normal week?

3. What is Gannon’s total payroll expense in a normal week?

4. The employee at Gannon who hires new employees is also the person whoprocesses the payroll. Is this arrangement a practice of good internalcontrol?

Demo Doc 3

HarrCh10v1.qxd 10/23/06 9:17 AM Page 340

Page 373: polpoa_sg

Demo Doc 3 Solutions | Chapter 10 341

Journal Entry:

Salary Expense (gross pay) 500,000 Employee Income Tax Payable ($500,000 3 15%) 75,000 FICA Tax Payable ($500,000 3 8%) 40,000 Pension Contributions Payable ($500,000 3 4%) 20,000 Union Dues Payable ($500,000 3 2%) 10,000 Cash (net/take home pay) 355,000

PostDate Accounts Ref. Dr. Cr.

Part 1 Part 2 Part 3 Part 4 Demo DocComplete

5 Describe the payrollprocess

6 Record basic payrolltransactions

Demo Doc 3 SolutionsRequirement 1

For a normal week, journalize the following transactions:

a. Cash payment of employee salaries

The entire gross pay of $500,000 is all journalized as Salary Expense (a debit).However, this entire amount is not all paid to the employee in cash.

Employee Income Tax Payable increases (a credit) by:

$500,000 ! 15% " $75,000

FICA Tax Payable increases (a credit) by:

$500,000 ! 8% " $40,000

Pension Contributions Payable increases (a credit) by:

$500,000 ! 4% " $20,000

Union Dues Payable increases (a credit) by:

$500,000 ! 2% " $10,000

Cash decreases (a credit) by the pay that is not withheld:

$500,000 – $75,000 – $40,000 – $20,000 – $10,000 "$355,000

To summarize:

Employee Income Tax PayableFICA Tax PayablePPension Contributions PayableUnion Dues Payyable

500,000500,000500,000500,000

15%8%

!

!

!

!

44%2%

75,00040,00020,00010,000

"

"

"

"

Calculate payrollamounts

4

HarrCh10v1.qxd 10/23/06 9:17 AM Page 341

Page 374: polpoa_sg

342 Chapter 10 | Demo Doc 3 Solutions

The $355,000 cash amount is often referred to as “take home pay” because it isthe amount the employees actually receive (that is, take home) on their paychecks.

Note that Gannon is not paying these taxes out of its own pocket. It is usingmoney that has been held back from employee paychecks to make these payments.

The taxes withheld by Gannon are similar to the treatment of sales taxespayable. Gannon has the cash for them (because Gannon never paid this cash tothe employees, but withheld it instead) and will pass it on to the appropriateagencies in the near future just as sales taxes collected are journalized as apayable and passed on to the state government.

b. Accrue Gannon’s payroll taxes

Gannon must journalize Payroll Tax Expense (a debit) for all taxes the companymust pay.

FICA Tax Payable increases (a credit) by Gannon’s portion of the taxes:

$500,000 ! 8% " $40,000

Unemployment Tax Payable increases (a credit) by:

$500,000 ! 6% " $30,000

Journal Entry:

Payroll Tax Expense (to balance) 70,000 FICA Tax Payable ($500,000 3 8%) (matching amount) 40,000 Unemployment Tax Payable ($500,000 3 6%) 30,000

PostDate Accounts Ref. Dr. Cr.

4 Calculate payrollamounts

6 Record basic payrolltransactions

5 Describe the payrollprocess

4 Calculate payrollamounts

5 Describe the payrollprocess

6 Record basic payrolltransactions

Unlike the liabilities journalized in transaction a (that are paid by theemployees by being taken out of their paychecks), these amounts are being paiddirectly by Gannon out of its own pocket.

c. Gannon’s payment of all payroll taxes

Cash decreases (a credit) by the total of all taxes paid.

Employee Income Tax Payable decreases (a debit) by its balance of $75,000.

FICA Tax Payable decreases (a debit) by its balance of:

$40,000 # $40,000 " $80,000

Unemployment Tax Payable decreases (a debit) by its balance of $30,000.

Journal Entry:

Employee Income Tax Payable 75,000 FICA Tax Payable ($40,000 + $40,000 matching amount) 80,000 Unemployment Tax Payable 30,000 Cash 185,000

PostDate Accounts Ref. Dr. Cr.

HarrCh10v1.qxd 10/23/06 9:17 AM Page 342

Page 375: polpoa_sg

Demo Doc 3 Solutions | Chapter 10 343

d. Gannon’s payment of union dues

Union Dues Payable decreases (a debit) by its balance of $10,000. Cash alsodecreases (a credit) by $10,000.

Journal Entry:

Union Dues Payable 10,000 Cash 10,000

PostDate Accounts Ref. Dr. Cr.

Journal Entry:

Pension Contributions Payable 20,000 Cash 20,000

PostDate Accounts Ref. Dr. Cr.

4 Calculate payrollamounts

4 Calculate payrollamounts

Part 1 Part 2 Part 3 Part 4 Demo DocComplete

Part 1 Part 2 Part 3 Part 4 Demo DocComplete

6 Record basic payrolltransactions

e. Gannon’s payment of pension contributions

Pension Contributions Payable decreases (a debit) by its balance of $20,000. Cashalso decreases (a credit) by $20,000.

6 Record basic payrolltransactions

Requirement 2

What is the employees’ net pay in a normal week?

Employees’ net pay is the amount of cash they receive each week. It is the$355,000 calculated in Requirement 1a.

Requirement 3

What is Gannon’s total payroll expense in a normal week?

Payroll expense is the total cost of having an employee, which includes theemployee’s salary as well as any additional taxes that do not come out of thatsalary (that is, that are paid out of the employer’s pocket).

HarrCh10v1.qxd 10/23/06 9:17 AM Page 343

Page 376: polpoa_sg

344 Chapter 10 | Demo Doc 3 Solutions

5 Describe the payrollprocess

Part 1 Part 2 Part 3 Part 4 Demo DocComplete

Part 1 Part 2 Part 3 Part 4 Demo DocComplete

So payroll expense includes salary expense of $500,000 and payroll taxes expenseof $70,000.

Total Payroll Expense " $500,000 # $70,000 " $570,000

Requirement 4

The employee at Gannon who hires new employees is also the person whoprocesses the payroll. Is this arrangement a practice of good internal control?

This practice is an internal control weakness. It is possible for this person to cre-ate fictitious employees and then issue payroll checks to them.This situation creates an opportunity for fraud. The person who is responsible forhiring employees should not be the person who processes payroll.

HarrCh10v1.qxd 10/23/06 9:17 AM Page 344

Page 377: polpoa_sg

Quick Practice Questions | Chapter 10 345

Quick Practice QuestionsTrue/False

_____ 1. Sales tax payable is shown as a long-term liability on the balancesheet.

_____ 2. An accrued expense is an expense that has not yet been paid.

_____ 3. A contingent liability is not an actual liability.

_____ 4. Optional deductions would include employee income tax, social securitytax, union dues, and insurance premiums.

_____ 5. State and federal unemployment taxes are two required payroll deduc-tions for employees.

_____ 6. The FICA Social Security tax is withheld from employees and is alsopaid by the employer in the same amount.

_____ 7. The document that includes every employee’s gross pay, deductions,and net pay for the payroll period is called the Wage and TaxStatement.

_____ 8. Two employees who have the same gross pay may have differentamounts withheld for income taxes depending on the number ofallowances claimed on the W-4 form.

_____ 9. An example of a contingent liability would be when you cosign a notepayable for a friend.

_____10. If a company has a note payable at December 31 for $300,000, whichwill be paid in three equal installments every five months, $100,000should be classified as a current liability.

Multiple Choice

1. Which of the following is true about current liabilities?a. Are due within one year or one operating cycle, whichever is longerb. Must be of a known amountc. Must be of an estimated amountd. Are subtracted from long-term liabilities on the balance sheet

2. Which of the following best describes unearned revenue? a. Revenue that has been earned and collected b. Revenue that has been earned but not yet collected c. Revenue that has been collected but not yet earned d. Revenue that has not been collected nor earned

3. When is Warranty Expense debited?a. In the period the product under warranty is repaired or replaced b. In the period after the product is sold c. In the period after the product is repaired or replaced d. In the period the revenue from selling the product was earned

HarrCh10v1.qxd 10/23/06 9:17 AM Page 345

Page 378: polpoa_sg

346 Chapter 10 | Quick Practice Questions

4. When a product is repaired under warranty, the entry includeswhich of the following?a. A debit to Warranty Expense b. A credit to Warranty Expense c. A debit to Estimated Warranty Payable d. A credit to Estimated Warranty Payable

5. Consider the following account balances for Philip’s Rentals asof December 31, 2008:

Cash $10,300 Prepaid Rent 3,600

Accounts Payable 7,800 Equipment 15,000

Accumulated Depreciation 2,000 Supplies 1,200

Philip Browning, Capital 9,300 Unearned Revenue 1,600

Philip Browning, 2,200 Notes Payable 7,500Withdrawals (due 12/31/2010)

What is the current ratio for Philip’s Rentals? a. 1.61 b. 1.03 c. 1.29 d. 1.38

6. Which of the following taxes have a ceiling on the amount ofannual employee earnings subject to the tax? a. Only the FICA tax b. Only the FICA tax and the federal unemployment taxc. Only the state and federal unemployment taxes d. The FICA tax and the state and federal unemployment taxes

7. Sumiko Greer is paid $26 per hour with time and a half herregular hourly pay rate for all hours exceeding 40 per week.During the week ended January 12, Sumiko worked 45 hours.What is the gross payroll? a. $1,105b. $1,170c. $1,235d. $1,365

8. Travel America has 24 employees who are paid on a monthlybasis. For the most recent month, gross earnings were $78,000, ofwhich $27,000 is subject to unemployment taxes (federal at 0.8%and state at 5.4%). Federal income tax withholdings are 20% oftotal earnings. All employees have $15 per month withheld forcharitable contributions. All earnings are subject to 8% FICA tax.What is the total employer’s payroll tax expense?a. $4,216b. $7,114c. $7,914d. $9,656

9. Referring to the information in the preceding question, what isthe amount of salaries payable?a. $51,309b. $54,471c. $55,800d. $56,160

HarrCh10v1.qxd 10/23/06 9:17 AM Page 346

Page 379: polpoa_sg

Quick Practice Questions | Chapter 10 347

10. Under what condition is a contingent liability journalized as anexpense and a liability?a. Under no conditionb. When the likelihood of an actual loss is remotec. When the likelihood of an actual loss is reasonably possibled. When the likelihood of an actual loss is probable and the amount can

be estimated

Quick Exercises

10-1. Federal United purchased equipment costing $88,000 onOctober 2, 2008, by paying a 30% cash down payment and signing a 9%, 120-day note payable for the balance. FederalUnited’s year-end is December 31.Journalize the following:a. The purchase of the equipment on October 2, 2008b. The accrual of interest on December 31, 2008c. Payment of the note on January 30, 2009

Journal Entry:

PostDate Accounts Ref. Dr. Cr.

Journal Entry:

PostDate Accounts Ref. Dr. Cr.

Journal Entry:

PostDate Accounts Ref. Dr. Cr.

HarrCh10v1.qxd 10/23/06 9:17 AM Page 347

Page 380: polpoa_sg

10-2. Ideal Food Services had cash sales of $787,000 during the monthof August 2008 and collected the 7% sales tax on these salesrequired by the state in which Ideal Food Services operates.a. Journalize the cash sale and the sales tax on August 31.b. Journalize the September 15 transaction when the sales tax is

remitted to the proper agency.

10-3. Freedom Vacuums warrants all of its products for one fullyear against any defect in manufacturing. Sales for 2007 and2008 were $731,000 and $854,000, respectively. FreedomVacuums expects warranty claims to run 4.5% of annual sales.Freedom paid $30,150 and $38,290, respectively, in 2007 and2008 in warranty claims.a. Compute Freedom’s warranty expense for 2007 and 2008.

b. Compute the balance in Estimated Warranty Payable on December 31,2008, assuming the January 1, 2007, balance in the account was$2,980.

Journal Entry:

PostDate Accounts Ref. Dr. Cr.

Journal Entry:

PostDate Accounts Ref. Dr. Cr.

348 Chapter 10 | Quick Practice Questions

HarrCh10v1.qxd 10/23/06 9:17 AM Page 348

Page 381: polpoa_sg

Quick Practice Questions | Chapter 1 349

10-4. Curtis Building Services has one employee, George North, whoearns $36 per hour for a 40-hour work week. He earns time anda half for all overtime hours. George has earned $89,200 inwages prior to the current week. From George’s pay, CurtisBuilding Services deducts 20% for federal income tax, and 8%for FICA taxes (up to $90,000 per year). The company alsowithholds $100 per week for his health insurance. The federalunemployment tax rate is 0.8% up to $7,000 of employee earn-ings per year. The state unemployment tax rate is 5.4% up to$7,000 of employee earnings per year. Curtis pays $100 perweek for medical insurance premiums for his employee.a. Compute the gross pay and the net pay for George North for the

current week ending December 18, 2008. George worked 48 hours.Round all amounts to the nearest dollar.Gross Pay:Net Pay:

b. Journalize the payroll expense on December 18, 2008.

c. Journalize the payroll taxes imposed on Curtis Building Serviceson December 18, 2008.

Journal Entry:

PostDate Accounts Ref. Dr. Cr.

Journal Entry:

PostDate Accounts Ref. Dr. Cr.

HarrCh10v1.qxd 10/23/06 9:17 AM Page 349

Page 382: polpoa_sg

Journal Entry:

PostDate Accounts Ref. Dr. Cr.

Journal Entry:

PostDate Accounts Ref. Dr. Cr.

Journal Entry:

PostDate Accounts Ref. Dr. Cr.

10-5. Use the data in Quick Exercise 10-4 for the following:a. Journalize the payment of payroll to the employee on December 18,

2008.b. Journalize the payment of the income tax withheld and FICA for

the employee and employer on December 18, 2008.c. Journalize the payment of the health insurance premiums with-

held on December 31, 2008.

350 Chapter 10 | Quick Practice Questions

HarrCh10v1.qxd 10/23/06 9:17 AM Page 350

Page 383: polpoa_sg

Do It Yourself! Question 1 | Chapter 10 351

Journal Entry:

PostDate Accounts Ref. Dr. Cr.

Journal Entry:

PostDate Accounts Ref. Dr. Cr.

1 Account for currentliabilities of knownamount

2 Account for estimatedand contingent liabilities

Journal Entry:

PostDate Accounts Ref. Dr. Cr.

Do It Yourself! Question 1

General Current LiabilitiesNitro Brothers sells products with warranties included in the selling price.During October 2008, Nitro sold goods for $10,000 cash. These goods cost $8,000to manufacture. Nitro is required by law to collect 8% sales tax on all sales.

Nitro estimates warranty costs to be 1% of selling price. During October2008, Nitro made $60 of repairs under warranty (paid in cash to a repair service).

On October 31, 2008, Nitro remitted all sales tax collected in October to thestate government.

Requirement

Journalize all of Nitro’s transactions in the month of October 2008.

HarrCh10v1.qxd 10/23/06 9:17 AM Page 351

Page 384: polpoa_sg

352 Chapter 10 | Do It Yourself! Question 1

Journal Entry:

PostDate Accounts Ref. Dr. Cr.

Journal Entry:

PostDate Accounts Ref. Dr. Cr.

HarrCh10v1.qxd 10/23/06 9:17 AM Page 352

Page 385: polpoa_sg

Do It Yourself! Question 2 | Chapter 10 353

7 Report current liabilitieson the balance sheet

Do It Yourself! Question 2

Current Portion of Long-Term DebtOn October 1, 2008, Pulter Industries signed a $2,000 note payable. Pulteragreed to pay back $100 per month, beginning on November 1, 2008 and endingon June 1, 2010.

Pulter is also required to pay 10% interest on the note each month.

Requirement

Show the presentation of this note on Pulter’s December 31, 2008, balancesheet.

HarrCh10v1.qxd 10/23/06 9:17 AM Page 353

Page 386: polpoa_sg

354 Chapter 10 | Do It Yourself! Question 3

Journal Entry:

PostDate Accounts Ref. Dr. Cr.

Journal Entry:

PostDate Accounts Ref. Dr. Cr.

b. Oxygen’s payroll taxes

4 Calculate payrollamounts

6 Record basic payrolltransactions

4 Calculate payrollamounts

6 Record basic payrolltransactions

Do It Yourself! Question 3

Payroll LiabilitiesOxygen Co. employees earn $200,000 gross pay per week. All employees havethe following items withheld from their pay:

20% Income taxes

8% FICA taxes

3% 401(k) plan contributions

1% Union dues

Oxygen pays the following payroll taxes:

8% FICA taxes

6% Unemployment taxes

Requirement

For a normal week, journalize the following transactions:

a. Cash payment of employee salaries

HarrCh10v1.qxd 10/23/06 9:17 AM Page 354

Page 387: polpoa_sg

c. Oxygen’s payment of all payroll taxes

Do It Yourself! Question 3 | Chapter 10 355

6 Record basic payrolltransactions

6 Record basic payrolltransactions

Journal Entry:

PostDate Accounts Ref. Dr. Cr.

Journal Entry:

PostDate Accounts Ref. Dr. Cr.

Journal Entry:

PostDate Accounts Ref. Dr. Cr.

d. Oxygen’s payment of union dues

e. Oxygen’s payment of 401(k) plan contributions

6 Record basic payrolltransactions

HarrCh10v1.qxd 10/23/06 9:17 AM Page 355

Page 388: polpoa_sg

Quick Practice SolutionsTrue/False

F 1. Sales tax payable is shown as a long-term liability on the balance sheet.False–Sales tax payable is shown as a current liability on the balance sheet. (p. 524)

T 2. An accrued expense is an expense that has not yet been paid.(p. 526).

T 3. A contingent liability is not an actual liability. (p. 528)

F 4. Optional deductions would include employee income tax, social security tax, union dues, and insurance premiums.False–Employee income tax and Social Security tax are requireddeductions. (p. 532)

F 5. State and federal unemployment taxes are two required payroll deductions for employees.False–State and federal unemployment taxes are paid by the employer. (p. 532)

T 6. The FICA Social Security tax is withheld from employees and is also paid by the employer in the same amount. (p. 533)

F 7. The document that includes every employee’s gross pay, deductions,and net pay for the payroll period is called the Wage and Tax Statement.False–The document that includes every employee’s gross pay,deductions, and net pay for the payroll period is called the payroll register. A Wage and Tax Statement is a W-2 Form, which is sent to employees and the IRS for tax filing purposes. (p. 536)

T 8. Two employees who have the same gross pay may have different amounts withheld for income taxes depending on the number of allowances claimed on the W-4 form. (p. 532)

T 9. An example of a contingent liability would be when you cosign a note payable for a friend. (p. 528)

F 10. If a company has a note payable at December 31 for $300,000,which will be paid in three equal installments every five months,$100,000 should be classified as a current liability.False–$200,000 should be classified as a current liability because that amount will be paid within 10 months, lessthan 1 year. (pp. 541–542)

356 Chapter 10 | Quick Practice Solutions

HarrCh10v1.qxd 10/23/06 9:17 AM Page 356

Page 389: polpoa_sg

Multiple Choice

1. Which of the following is true about current liabilities? (p. 524)a. Are due within one year or one operating cycle, whichever is longerb. Must be of a known amountc. Must be of an estimated amountd. Are subtracted from long-term liabilities on the balance sheet

2. Which of the following best describes unearned revenue? (p. 526)a. Revenue that has been earned and collected b. Revenue that has been earned but not yet collected c. Revenue that has been collected but not yet earnedd. Revenue that has not been collected nor earned

3. When is Warranty Expense debited? (p. 527)a. In the period the product under warranty is repaired or replaced b. In the period after the product is sold c. In the period after the product is repaired or replaced d. In the period the revenue from selling the product was earned

4. When a product is repaired under warranty, the entry includeswhich of the following? (p. 527)a. A debit to Warranty Expense b. A credit to Warranty Expensec. A debit to Estimated Warranty Payabled. A credit to Estimated Warranty Payable

5. Consider the following account balances for Philip’s Rentals asof December 31, 2008:

Cash $10,300 Prepaid Rent 3,600

Accounts Payable 7,800 Equipment 15,000

Accumulated Depreciation 2,000 Supplies 1,200

Philip Browning, Capital 9,300 Unearned Revenue 1,600

Philip Browning, 2,200 Notes Payable 7,500Withdrawals (due 12/31/2010)

What is the current ratio for Philip’s Rentals? (p. 530)a. 1.61b. 1.03 c. 1.29 d. 1.38

6. Which of the following taxes have a ceiling on the amount ofannual employee earnings subject to the tax? (pp. 533–534)a. Only the FICA tax b. Only the FICA tax and the federal unemployment tax c. Only the state and federal unemployment taxes d. The FICA tax and the state and federal unemployment taxes

Quick Practice Solutions | Chapter 10 357

HarrCh10v1.qxd 10/23/06 9:17 AM Page 357

Page 390: polpoa_sg

7. Sumiko Greer is paid $26 per hour with time and a half herregular hourly pay rate for all hours exceeding 40 per week.During the week ended January 12, Sumiko worked 45 hours.What is the gross payroll? (p. 531)a. $1,105b. $1,170c. $1,235d. $1,365

8. Travel America has 24 employees who are paid on a monthlybasis. For the most recent month, gross earnings were $78,000, ofwhich $27,000 is subject to unemployment taxes (federal at 0.8%and state at 5.4%). Federal income tax withholdings are 20% oftotal earnings. All employees have $15 per month withheld forcharitable contributions. All earnings are subject to 8% FICA tax.

What is the total employer’s payroll tax expense? (pp. 532–533)a. $4,216b. $7,114c. $7,914d. $9,656

9. Referring to the information in the preceding question, what isthe amount of salaries payable? (pp. 532–533)a. $51,309b. $54,471c. $55,800d. $56,160

10. Under what condition is a contingent liability journalized as anexpense and a liability? (pp. 528–529)a. Under no conditionb. When the likelihood of an actual loss is remotec. When the likelihood of an actual loss is reasonably possibled. When the likelihood of an actual loss is probable and the amount can

be estimated

Quick Exercise10-1. Federal United purchased equipment costing $88,000 on

October 2, 2008, by paying a 30% cash down payment and sign-ing a 9%, 120-day note payable for the balance. FederalUnited’s year-end is December 31. (pp. 523–524)

Journalize the following:a. The purchase of the equipment on October 2, 2008b. The accrual of interest on December 31, 2008c. The payment of the note on January 30, 2009

Journal Entry:

Post Date Accounts Ref. Dr. Cr. a. Oct. 2 Equipment 88,000 Cash 26,400 Notes Payable 61,600 To record purchase of equipment.

HarrCh10v1.qxd 10/23/06 9:17 AM Page 358

Page 391: polpoa_sg

Quick Practice Solutions | Chapter 10 359

Journal Entry:

a. Aug. 31 Cash 842,090 Sales 787,000 Sales Taxes Payable 55,090 To record cash sales including 7% sales tax.

Post Date Accounts Ref. Dr. Cr.

Journal Entry:

b. Sept. 15 Sales Tax Payable 55,090 Cash 55,090 To record sales tax remittance.

Post Date Accounts Ref. Dr. Cr.

Journal Entry:

b. Sept. 15 Sales Tax Payable 55,090 Cash 55,090 To record sales tax remittance.

Post Date Accounts Ref. Dr. Cr.

Journal Entry:

Post Date Accounts Ref. Dr. Cr.

Journal Entry:

c. Jan. 30 Notes Payable 61,600 Interest Payable 1,386 Interest Expense 462 Cash 63,448 To pay off the note payable plus interest ($61,600 3 0.09 3 30/360 = $462).

b. Dec. 31 Interest Expense 1,386 Interest Payable 1,386 To accrue interest expense 10/2/08–12/31/08 ($61,600 3 0.09 3 90/360 = $1,386).

Post Date Accounts Ref. Dr. Cr.

10-2. Ideal Food Services had cash sales of $787,000 during the monthof August 2008 and collected the 7% sales tax on these salesrequired by the state in which Ideal Food Services operates.(pp. 524–525)a. Journalize the cash sale and the sales tax on August 31.b. Journalize the September 15 transaction when the sales tax is

remitted to the proper agency.

HarrCh10v1.qxd 10/23/06 9:17 AM Page 359

Page 392: polpoa_sg

10-3. Freedom Vacuums warrants all of its products for one fullyear against any defect in manufacturing. Sales for 2007 and2008 were $731,000 and $854,000, respectively. FreedomVacuums expects warranty claims to run 4.5% of annual sales.Freedom paid $30,150 and $38,290, respectively, in 2007 and2008 in warranty claims. (pp. 527–528)

a. Compute Freedom’s warranty expense for 2007 and 2008.For 2007: $731,000 ! 0.045 = $32,895For 2008: $854,000 ! 0.045 = $38,430

b. Compute the balance in Estimated Warranty Payable on December 31, 2008, assuming the January 1, 2007, balance in theaccount was $2,980.$5,865 = ($2,980 + $32,895 + $38,430 – $30,150 – $38,290)

10-4. Curtis Building Services has one employee, George North,who earns $36 per hour for a 40-hour work week. He earnstime and a half for all overtime hours. George has earned$89,200 in wages prior to the current week. From George’spay, Curtis Building Services deducts 20% for federal incometax, and 8% for FICA taxes (up to $90,000 per year). The com-pany also withholds $100 per week for his health insurance.The federal unemployment tax rate is 0.8% up to $7,000 ofemployee earnings per year. The state unemployment tax rateis 5.4% up to $7,000 of employee earnings per year. Curtispays $100 per week for medical insurance premiums hisemployee. (pp. 531–534)

a. Compute the gross pay and the net pay for George North for thecurrent week ending December 18, 2008. George worked 48 hours.Round all amounts to the nearest dollar.

Gross Pay: (40 ! $36) + (8 ! $36 ! 1.5) = $1,440 + $432 = $1,872Net Pay: FICA tax withheld = ($800 ! 0.08) = $64Federal tax withheld = ($1,872 ! 0.20) = $374Health insurance withheld = $100 Total deductions = $64 + $374 + $100 = $538Net pay = $1,872 – $538 = $1,334

b. Journalize the payroll expense on December 18, 2008.c. Journalize the payroll taxes imposed on Curtis Building Services

on December 18, 2008.

360 Chapter 10 | Quick Practice Solutions

Journal Entry:

b. Dec. 18 Salary Expense 1,872 FICA Tax Payable 64 Employee Income Tax Payable 374 Health Insurance Payable 100 Salary Payable 1,334 To record payroll expense.*

PostDate Accounts Ref. Dr. Cr.

*Note: Maximum unemployment taxes have been incurred.

HarrCh10v1.qxd 10/23/06 9:17 AM Page 360

Page 393: polpoa_sg

10-5. Use the data in Quick Exercise 10-4 for the following:(pp. 539–541)

a. Journalize the payment of payroll to the employee on December 18,2008.

b. Journalize the payment of the income tax withheld and FICA forthe employee and employer on December 18, 2008.

c. Journalize the payment of the health insurance premiums with-held on December 31, 2008.

Quick Practice Solutions | Chapter 10 361

Journal Entry:

c. Dec. 18 Payroll Tax Expense 64 FICA Tax Payable 64 To record payroll tax expense. ($90,000 – $89,200) 3 0.08 = $64

PostDate Accounts Ref. Dr. Cr.

Journal Entry:

a. Dec. 18 Salary Payable 1,334 Cash 1,334 To record payroll paid to employees.

PostDate Accounts Ref. Dr. Cr.

Journal Entry:

b. Employee Income Tax Payable 374 FICA Tax Payable 128 Cash 502 To record remittance of income tax and FICA taxes.

PostDate Accounts Ref. Dr. Cr.

Journal Entry:

c. Dec. 31 Health Insurance Payable 100 Cash 100 To record payment of health insurance premium.

PostDate Accounts Ref. Dr. Cr.

HarrCh10v1.qxd 10/23/06 9:17 AM Page 361

Page 394: polpoa_sg

362 Chapter 10 | Do It Yourself! Question 1 Solutions

Journal Entry:

Warranty Expense ($10,000 ! 0.01) 100 Estimated Warranty Payable 100

PostDate Accounts Ref. Dr. Cr.

Journal Entry:

Warranty Payable 60 Cash 60

PostDate Accounts Ref. Dr. Cr.

Journal Entry:

Cash ($10,000 ! 1.08) 10,800 Sales Revenue 10,000 Sales Tax Payable ($10,000 ! 8%) 800

PostDate Accounts Ref. Dr. Cr.

Journal Entry:

Cost of Goods Sold 8,000 Inventory 8,000

PostDate Accounts Ref. Dr. Cr.

Nitro estimates warranty costs to be 1% of the selling price.

During October 2008, Nitro made $60 of repairs under warranty (paidin cash to a repair service).

Do It Yourself! Question 1 Solutions

Requirement

Journalize all of Nitro’s transactions in the month of October 2008.

During October 2008, Nitro sold goods for $10,000 cash. These goodscost $8,000 to manufacture. Nitro is required by law to collect 8%sales tax on all sales.

HarrCh10v1.qxd 10/23/06 9:17 AM Page 362

Page 395: polpoa_sg

On October 31, 2008, Nitro remitted all sales tax collected in Octoberto the state government.

Do It Yourself! Question 1 Solutions | Chapter 10 363

Journal Entry:

Oct. 31 Sales Tax Payable ($10,000 3 8%) 800 Cash 800

PostDate Accounts Ref. Dr. Cr.

HarrCh10v1.qxd 10/23/06 9:17 AM Page 363

Page 396: polpoa_sg

364 Chapter 10 | Do It Yourself! Question 2 Solutions

Do It Yourself! Question 2 Solutions

Requirement

Show the presentation of this note on Pulter’s December 31, 2008, balancesheet.

Payments Already Made " $100 ! 2 " $200

$2,000 – $200 " $1,800 Remaining To Be Repaid12 ! $100 " $1,200 " Current Portion$1,800 – $1,200 " $600 " Long-Term Portion

Interest payable only relates to the month of December (because November’sinterest expense was paid on December 1).

Interest Payable " $1,800 ! 10% ! 1/12 " $15

Current Liabilities

Current portion of note payable $1,200 Interest payable 15

Long-Term Liabilities

Notes payable (net of current portion) $ 600

HarrCh10v1.qxd 10/23/06 9:17 AM Page 364

Page 397: polpoa_sg

Do It Yourself! Question 3 Solutions | Chapter 10 365

Journal Entry:

Salary Expense (gross pay) 200,000 Employee Income Tax Payable ($200,000 3 20%) 40,000 FICA Tax Payable ($200,000 3 8%) 16,000 Pension Contributions Payable ($200,000 3 3%) 6,000 Union Dues Payable ($200,000 3 1%) 2,000 Cash (net/take home pay) 136,000

PostDate Accounts Ref. Dr. Cr.

Journal Entry:

Payroll Tax Expense (to balance) 28,000 FICA Tax Payable ($200,000 3 8%) (matching amount) 16,000 Unemployment Tax Payable ($200,000 3 6%) 12,000

PostDate Accounts Ref. Dr. Cr.

Journal Entry:

Employee Income Tax Payable 40,000 FICA Tax Payable ($16,000 + $16,000 matching amount) 32,000 Federal Unemployment Tax Payable 12,000 Cash 84,000

PostDate Accounts Ref. Dr. Cr.

b. Oxygen’s payroll taxes

c. Oxygen’s payment of all payroll taxes

d. Oxygen’s payment of union dues

Journal Entry:

PostDate Accounts Ref. Dr. Cr. Union Dues Payable 2,000 Cash 2,000

Do It Yourself! Question 3 Solutions

Requirement

For a normal week, journalize the following transactions:

a. Cash payment of employee salaries

HarrCh10v1.qxd 10/23/06 9:17 AM Page 365

Page 398: polpoa_sg

e. Oxygen’s payment of 401(k) plan contributions

366 Chapter 10 | Do It Yourself! Question 3 Solutions

Journal Entry:

PostDate Accounts Ref. Dr. Cr. Pension Contributions Payable 6,000 Cash 6,000

HarrCh10v1.qxd 10/23/06 9:17 AM Page 366

Page 399: polpoa_sg

The Power of PracticeFor more practice using the skills learned in this chapter, visit MyAccountingLab.There you will find algorithmically generated questions that are based on theseDemo Docs and your main textbook’s Review and Accounting Practice sections.

To go to MyAccountingLab and follow these steps:

1. Direct your URL to www.myaccountinglab.com.2. Log in using your name and password.3. Click the MyAccountingLab link.4. Click Study Plan in the left navigation bar.5. From the table of contents, select Chapter 10, Current Liabilities and Payroll.6. Click a link to work on the tutorial exercises.

The Power of Practice | Chapter 10 367

HarrCh10v1.qxd 10/23/06 9:17 AM Page 367

Page 400: polpoa_sg

HarrCh10v1.qxd 10/23/06 9:17 AM Page 368

Page 401: polpoa_sg

Corporations andStockholders’ Equity

WHAT YOU PROBABLY ALREADY KNOW

You probably already know that you can purchase shares of acompany’s stock as an investment. CNBC shows the trading price ofvarious stocks as they take place, and the daily prices are reportedin your financial newspapers. Much of the trading taking place isbetween investors rather than from the issuing corporation.

One way that a corporation issues its shares of stock is in aninitial public offering (IPO). A recent popular IPO is Google. Googlewas doing business for six years before its founders took thecompany public in August 2004. The IPO provided investors anopportunity to purchase Google stock at a stated offer price of $85a share. The market price of the stock rose quickly in trading and ayear and a half after the IPO, the stock traded at more than $300per share. The cash received from the sale of the Google stock andthe shareholders’ equity interest was recorded on the books ofGoogle Corporation. In this chapter, we will see how to account forthe equity transactions of a corporation.

Learning Objectives

Compare the three forms of business organization.

As we know from Chapter 1, the three forms of business organization area proprietorship, a partnership, or a corporation. Even though the threeforms share some similarities, each has its own set of characteristics interms of organizational structure, ability to raise capital, ownerinvolvement in management, and other key areas. Review Exhibit 11-1(p. 571) carefully for a comparison of the three forms of organizations.

Describe stockholders’ equity and classes of stock.

The two main segments of stockholders’ equity are paid-in capital(amounts received from stockholders) and retained earnings (earnedfrom profitable operations). Retained Earnings, which usually carries acredit balance, holds the accumulated earnings or losses of the business,and it is from this account where distributions of earnings (dividends)are taken, thereby reducing the size of the company. Remember, retainedearnings are the cumulative profits earned and kept by the business.

The main categories of paid-in capital—the two basic classes of stock—arepreferred stock and common stock. Common stock, the main source of paid-in capital, is issued by every corporation. In a corporation, the commonstockholders are the owners of the business and have all of the four basicrights of stockholders. Owners of preferred stock also have the four basic

1

2

11

1eSG_C11_0131792075.QXD 10/23/06 9:30 AM Page 369

Page 402: polpoa_sg

370 Chapter 11 | Corporations and Stockholders’ Equity

4

5

3

rights, but some rights, such as the right to vote, are sometimes withheld.Preferred stock dividends, while paid before common stock dividends, arealso usually fixed. Just like common stock, dividends must be declared bythe board of directors for the entity to be legally liable. Preferredstockholders also have the advantage of receiving assets before thecommon stockholders if the corporation should ever liquidate. Review the“Stockholders’ Equity” and “Classes of Stock” sections of the main text, aswell as Exhibit 11-5 (p. 581) to see how the distribution of dividends betweencommon and preferred shareholders works.

Record the issuance of stock.

When a company incorporates, the par or stated value, if any, will beindicated in the articles of incorporation. The par or stated value isusually a nominal amount assigned to a share of stock that representsthe minimum legal stated capital. It does not indicate the value or worthof the stock. When the stock is sold by the corporation, the CommonStock account is credited for the par or stated value. Usually, the stock issold above par, which is considered a premium. The excess of the stocksales price over the par or stated value is the amount credited to theequity account, Paid-In Capital in Excess of Part. Review the accountingfor stock issuances under “Issuing Stock” in the main text.

Account for cash dividends.

If the board of directors declares dividends, Retained Earnings isdecreased (a debit) and Dividends Payable is increased (a credit). On thedate of payment, the liability is decreased (a debit) and Cash is alsodecreased (a credit). The date of record, which falls between the dates ofdeclaration and payment, merely identifies the stockholders who areentitled to receive the dividend. No journal entry needs to be made onthat date. Read “Accounting for Cash Dividends” in the main text toreview the significance and accounting of each of the dividend dates.Review “Dividends on Cumulative and Noncumulative Preferred Stock”to learn about the effect these characteristics have on dividend payments.

Account for stock dividends and stock splits.

The board of directors may declare a stock dividend instead of a cashdividend. A stock dividend gives each shareholder more shares of stockbased upon the number of shares currently owned. A stock splitincreases the number of shares issued and outstanding and reduces thepar or stated value proportionately. A 3-for-1 split means that eachshareholder receives 2 more shares for each 1 currently held, and thepar or stated value is 1/3 of the amount before the split. No journal entryis required for a stock split because the change has no impact on thefinancial position of the company.

A stock dividend increases (credits) the Common Stock and Paid-In Capitalin Excess of Par accounts for the additional shares issued. The dividend, areturn of equity to the shareholders, also decreases (debits) RetainedEarnings. Review the section “Comparison of Cash Dividends, StockDividends, and Stock Splits” in the main text, including Exhibit 11-6(p. 585), which overviews the effects of each of these events on stock-holders’ equity.

1eSG_C11_0131792075.QXD 10/23/06 9:30 AM Page 370

Page 403: polpoa_sg

Corporations and Stockholders’ Equity | Chapter 11 371

6 Account for treasury stock.

Treasury stock is when the company buys back its own shares fromexisting shareholders. The cost of the shares is debited to a contra-equity account, Treasury Stock. If the treasury stock is subsequentlysold, Cash is debited, Treasury Stock is credited for the cost of theshares, and Paid-In Capital from Treasury Stock Transactions is debitedor credited for the difference, if any. Review the impact of treasury stockon stockholders’ equity in Exhibit 11-7 (p. 587).

Report stockholders’ equity.

Remember that different accountants report stockholders’ equity indifferent ways. Because it is important for you to be able to read thefinancial statements of real companies, carefully review Exhibit 11-8(p. 589) and consider the important points about the real-world formatnoted in the main text. Also review Exhibit 11-9 (p. 590) to understandthe format of a statement of stockholders’ equity.

7

1eSG_C11_0131792075.QXD 10/23/06 9:30 AM Page 371

Page 404: polpoa_sg

372 Chapter 2 | Demo Doc 1

Demo Doc 1Common StockLearning Objectives 1–3, 7

Jack, Inc. had the following information at December 31, 2008:

Stockholders’ Equity

Common stock, 1,600,000 authorized,350,000 issued and outstanding shares $ 437,500

Paid-in capital in excess of par 787,500

Retained earnings 4,200,000

Total stockholders’ equity $5,425,000

Requirements

1. What are Jack’s two main sources of corporate capital?

2. What is the par value per share of the common stock?

3. On average, what was the original per-share issue price of the common stock?

4. On February 12, 2009, Jack issued another 20,000 common shares for $5cash per share. Journalize this transaction.

5. Jack earned net income of $150,000 and paid no dividends in 2009. Noother equity transactions took place in 2009. Prepare the stockholders’equity section of Jack’s balance sheet on December 31, 2009.

1eSG_C11_0131792075.QXD 10/23/06 9:30 AM Page 372

Page 405: polpoa_sg

Demo Doc 1 Solutions | Chapter 11 373

Demo Doc 1 Solutions

Part 1 Part 2 Part 3 Part 4 Part 5 Demo DocComplete

Requirement 1

What are Jack’s two main sources of corporate capital?

Corporate capital is another term for the shareholders’ equity.Jack has paid-in capital. This money has been received from the stockhold-

ers. Jack also has retained earnings, which represents profits earned on thestockholders’ behalf (and not yet distributed as dividends).

Every corporation has these two sources of capital.

Requirement 2

What is the par value per share of the common stock?

Part 1 Part 2 Part 3 Part 4 Part 5 Demo DocComplete

Common Stock and Preferred Stock accounts hold only the par value of the issuedshares. So the $437,500 in the Common Stock account represents the par value ofall the issued shares.

Requirement 3

On average, what was the original per-share issue price of the common stock?

Par Valueper Share

Common Stock BalanceNumb

!eer of Issued Common Shares

Par Valueper Sharre

$437,500350,000 shares

$1.25 per share

!

!

Part 1 Part 2 Part 3 Part 4 Part 5 Demo DocComplete

When stock is issued for cash, the Cash account increases (a debit) for cashreceived. The Common Stock account increases (a credit) for the par value, andPaid-In Capital in Excess of Par increases (a credit) for the difference. Because theselling price per share is almost always more than the par value, this excess bal-ancing amount to Paid-In Capital in Excess of Par is usually an increase (a credit).

Compare the threeforms of businessorganization

1

Describe stockholders’equity and classes ofstock

2

Describe stockholders’equity and classes ofstock

2

1eSG_C11_0131792075.QXD 10/23/06 9:30 AM Page 373

Page 406: polpoa_sg

374 Chapter 11 | Demo Doc 1 Solutions

Cash increases (a debit) by:

$5 " 20,000 ! $100,000

Common Stock increases (a credit) by the par value of the new shares:

$1.25 " 20,000 ! $25,000

Paid-In Capital in Excess of Par is the excess cash paid:

$100,000 # $25,000 ! $75,000

The balancing amount appears in the journal entry.

We know that total debits must equal total credits for any transaction. Inthis case, the debit is the cash received and the credits are the increases toCommon Stock and Paid-In Capital in Excess of Par:

So the total cash received from issuance of the common shares:

$437,500 $ $787,500 ! $1,225,000

This amount represents all 350,000 issued shares.

$1,225,000/350,000 shares ! $3.50 average cash received per share

$3.50 received # $1.25 par ! $2.25 additional cash paid per share

$2.25 " 350,000 shares ! $787,500 total additional cash paid

This amount is the balancing credit to Paid-In Capital in Excess of Par.

Requirement 4

On February 12, 2009, Jack issued another 20,000 common shares for $5 cashper share. Journalize this transaction.

Cash Received from ShareIssuance

Common Sto! cck Paid-in Capital in Excess of Par$

Part 1 Part 2 Part 3 Part 4 Part 5 Demo DocComplete

Journal Entry:

Feb. 12 Cash (20,000 ! $5) 100,000 Common Stock (20,000 ! $1.25) 25,000 Paid-in Capital in Excess of Par (to balance) 75,000

PostDate Accounts Ref. Dr. Cr.

Record the issuance ofstock

3

1eSG_C11_0131792075.QXD 10/23/06 9:30 AM Page 374

Page 407: polpoa_sg

Demo Doc 1 Solutions | Chapter 11 375

Because of the stock issuance in Requirement 4, the number of outstanding com-mon shares increased:

350,000 shares $ 20,000 shares ! 370,000 shares

This change must be shown for Common Stock as part of its descriptive line onthe balance sheet.

The dollar amount in the Common Stock account increased to:

$437,500 $ $25,000 ! $462,500

The other impact of this transaction on stockholders’ equity was an increase inPaid-In Capital in Excess of Par:

$787,500 $ $75,000 ! $862,500

The net income earned by Jack increases retained earnings:

$4,200,000 $ $150,000 ! $4,350,000

These new amounts create a new total stockholders’ equity of $5,675,000.

Stockholders’ Equity

Common Stock, 1,600,000 authorized,

370,000 issued and outstanding shares $ 462,500

Paid-in capital in excess of par 862,500

Retained earnings 4,350,000

Total stockholders’ equity $5,675,000

Report stockholders’equity

7

Part 1 Part 2 Part 3 Part 4 Part 5 Demo DocComplete

Part 1 Part 2 Part 3 Part 4 Part 5 Demo DocComplete

Requirement 5

Jack earned net income of $150,000 and paid no dividends in 2009. No otherequity transactions took place in 2009. Prepare the stockholders’ equitysection of Jack’s balance sheet on December 31, 2009.

1eSG_C11_0131792075.QXD 10/23/06 9:30 AM Page 375

Page 408: polpoa_sg

376 Chapter 11 | Demo Doc 2

Demo Doc 2Preferred StockLearning Objectives 2–4

Jill Co. issued 25,000 6%, $100 par cumulative preferred shares on January 1,2008, for $120 cash per share. Jill had never issued preferred shares before thisdate.

Jill paid the following cash dividends (in total, to all shares):

2008 $120,000

2009 $160,000

2010 $200,000

Requirements

1. Journalize the issuance of the preferred shares on January 1, 2008, and thepayment of the preferred share dividends in 2008. (Assume the dividendswere declared and paid on the same day.)

2. How much in dividends is Jill supposed to pay to the preferred shareholderseach year?

3. Did Jill pay all of the required dividends in each year? If not, what happensto the amount not paid? How much in dividends did the preferred andcommon shareholders receive each year?

1eSG_C11_0131792075.QXD 10/23/06 9:30 AM Page 376

Page 409: polpoa_sg

Demo Doc 2 Solutions | Chapter 11 377

Demo Doc 2 Solutions

Part 1 Part 2 Part 3 Demo DocComplete

The issuance of preferred shares is the same as the issuance of common shares,except for the account title.

Cash increases (a debit) by:

$120 " 25,000 ! $3,000,000

Preferred Stock increases (a credit) by the par value of the new shares:

$100 " 25,000 ! $2,500,000

Paid-In Capital in Excess of Par equals the excess cash paid:

$3,000,000 – $2,500,000 ! $500,000

This balancing amount appears in the journal entry.

Journal Entry:

PostDate Accounts Ref. Dr. Cr. Jan. 1 Cash (25,000 ! $120) 3,000,000 Preferred Stock (25,000 ! $100) 2,500,000 Paid-in Capital in Excess of Par (to balance) 500,000

When dividends are paid, Retained Earnings decreases because the share-holders are removing some of their capital from the company. So RetainedEarnings decreases (a debit) by $120,000.

Cash also decreases (a credit) by $120,000.

Journal Entry:

PostDate Accounts Ref. Dr. Cr. Retained Earnings 120,000 Cash 120,000

Describe stockholders’equity and classes ofstock

2

Record the issuance ofstock

3

Account for cashdividends

4

Requirement 1

Journalize the issuance of the preferred shares on January 1, 2008, and thepayment of the preferred share dividends in 2008. (Assume the dividends weredeclared and paid on the same day.)

1eSG_C11_0131792075.QXD 10/23/06 9:30 AM Page 377

Page 410: polpoa_sg

378 Chapter 11 | Demo Doc 2 Solutions

2008

For 2008, $120,000 is less than the “required” $150,000, so we know that Jill didnot pay all of the required dividends.

The preferred shares only received:

$120,000/25,000 shares ! $4.80 per share

The difference of $150,000 " $120,000 = $30,000 is dividends in arrears. Thisamount is not recorded in a transaction because it has not yet been declared andtherefore is not a liability. Dividends in arrears do not appear on the balancesheet. However, they are disclosed in a note to the financial statements.

Because the full $150,000 was not paid, the entire $120,000 goes to the preferredshareholders as dividends. The common shareholders get no dividends in 2008.

2009

For 2009, Jill must not only pay the $150,000 annual “requirement,” but firstmust also “catch up” on the dividends in arrears of $30,000 from 2008.

So in order to completely fulfill its obligation to the preferred shares, Jillmust pay $30,000 + $150,000 = $180,000 in dividends to the preferred shares.

We see that $160,000 is less than the $180,000, so we know that Jill did notpay all of the required dividends in 2009.

The difference of $180,000 " $160,000 = $20,000 is dividends in arrears.Because the full $180,000 was not paid, the entire $160,000 goes to the preferred

shareholders as dividends. The common shareholders get no dividends in 2009.

Part 1 Part 2 Part 3 Demo DocComplete

Each year, every preferred share is supposed to receive:

First, we should calculate the “required” annual dividends per share. In this case, it is:

$100 " 6% ! $6 per share

Because 25,000 preferred shares are outstanding, this annual amount works outto $6 ! 25,000 = $150,000 in dividends per year for all preferred shares.

Requirement 3

Did Jill pay all of the required dividends in each year? If not, what happens tothe amount not paid? How much in dividends did the preferred and commonshareholders receive each year?

“Required” PreferredShare Dividends

Par Val!

uuePer Share

DividendPercentage

"

Part 1 Part 2 Part 3 Demo DocComplete

Account for cash dividends

4

Describe stockholders’equity and classes ofstock

2

Account for cashdividends

4

Requirement 2

How much in dividends is Jill supposed to pay to the preferred shareholderseach year?

1eSG_C11_0131792075.QXD 10/23/06 9:30 AM Page 378

Page 411: polpoa_sg

Demo Doc 2 Solutions | Chapter 11 379

2010

In 2010, Jill is supposed to pay the annual $150,000 of dividends plus the $20,000dividends in arrears from 2009, for a total of $170,000.

The $200,000 paid is greater than $170,000, so we know that Jill did pay allof the required dividends in 2010.

The $170,000 goes to the preferred shareholders, while the rest ($200,000 "$170,000 = $30,000) goes to the common shareholders.

Part 1 Part 2 Part 3 Demo DocComplete

1eSG_C11_0131792075.QXD 10/23/06 9:30 AM Page 379

Page 412: polpoa_sg

380 Chapter 11 | Demo Doc 3

Demo Doc 3Stock Splits and DividendsLearning Objectives 2, 4, 5, 7

On December 31, 2008, Tinker Corp. had 25,000 $1.20 par common shares out-standing with a market price of $9 per share.

Retained Earnings had a balance of $60,000, but Paid-In Capital in Excess ofPar contained no balance.

Requirements

1. On January 1, 2009, Tinker split its common stock 3-for-1. Give the journalentry for the split. What are the par and market values per share after thesplit? How does this split affect stockholders’ equity?

2. On February 1, 2009, Tinker issued a 20% stock dividend. Give the journalentry for this dividend. What is the par value per share after the dividend?How does this dividend affect stockholders’ equity?

3. On March 1, 2009, Tinker declared and paid a cash dividend of $0.60 percommon share. Journalize this dividend. How does this dividend affectstockholders’ equity?

1eSG_C11_0131792075.QXD 10/23/06 9:30 AM Page 380

Page 413: polpoa_sg

Demo Doc 3 Solutions | Chapter 11 381

Demo Doc 3 Solutions

Part 1 Part 2 Part 3 Demo DocComplete

Account for stock divi-dends and stock splits

5Before the split, Tinker has 25,000 issued common shares.A 3-for-1 split creates 3 new shares for every 1 old share.

Another result of the split is that the par value and the market price are also split.

The new shares from the split would increase common stock by:

75,000 shares ! $0.40 par per share ! $30,000

No change occurred in the account balance of Common Stock. It remains thesame, only it is now spread across more shares (resulting in a lower par value pershare, as shown in the preceding calculations).

A stock split is like getting change for a $1 bill—you still have $1, it’s just notin one piece anymore.

The net impact on common stock is zero. Essentially, this means that no jour-nal entry is required for a stock split. However, a stock split is described in thenotes to the financial statements.

Because no journal entry is required, total equity is not affected by the stocksplit.

Par Value Per Shareafter Split

Par Value Pe!

rr ShareBefore Split Split Ratio

Par Value

"1

PPer Shareafter Split $1.20

13

$0.40 per sh

! "

! aare

Market Price Per Shareafter Split

Marke!

tt Price Per ShareBefore Split Split Ratio

"1

MMarket Price Per Shareafter Split $9

13

! "

Number of Shares AfterStock Split

Number of!

Shares BeforeStock Split Split Ratio

Numbe

"

rr of Shares AfterStock Split 25,000 3/1

75

! "

! ,,000 Shares

Requirement 1

On January 1, 2009,Tinker split its common stock 3-for-1. Give the journal entryfor the split.What are the par and market values per share after the split? Howdoes this split affect stockholders’ equity?

1eSG_C11_0131792075.QXD 10/23/06 9:30 AM Page 381

Page 414: polpoa_sg

382 Chapter 11 | Demo Doc 3 Solutions

Part 1 Part 2 Part 3 Demo DocComplete

This is a small stock dividend because the dividend percentage (20%) is less than 25%.Remember that after the stock split of January 1, Tinker has 75,000 common

shares outstanding with a market price of $3 per share and a par value of $0.40per share.

With a stock dividend, new shares are issued to existing shareholders.

Each of these new shares is identical to the shares that existed before the stockdividend. They have the same characteristics as the common shares that existedbefore the stock dividend. The par value of the new shares issued is $0.40 pershare, as it is for the other common shares.

So Common Stock increases (a credit) by:

15,000 new shares " $0.40 par ! $6,000

With any dividend, Retained Earnings decreases (a debit) because the sharehold-ers receive some of their value/equity back from the company.

In the case of a small stock dividend, this value is the market value of thenew shares issued.

So in this case, Retained Earnings decreases (a debit) by:

15,000 new shares " $3 market price per share ! $45,000

The difference between these two amounts is balanced to Paid-In Capital inExcess of Par—in this case, an increase (a credit) of:

$45,000 # $6,000 ! $39,000

Number of New Shares Issuedfor Stock Dividennd

Shares OutstandingBefore Dividend

Stock! " DDividend %

Number of New Shares Issuedfor Sttock Dividend 75,000 20%

15,000 New Shares

! "

!

Describe stockholders’equity and classes ofstock

2

Account for stock divi-dends and stock splits

5

Report stockholders’equity

7

Journal Entry:

PostDate Accounts Ref. Dr. Cr. Feb. 1 Retained Earnings (15,000 ! $3) 45,000 Common Stock (15,000 ! $0.40) 6,000 Paid-in Capital in Excess of Par (to balance) 39,000

All these accounts are part of the equity section. Therefore, equal debit (decrease)and credit (increase) effects occur in the equity section. We simply shift valuefrom Retained Earnings to Paid-In Capital.

Requirement 2

On February 1, 2009,Tinker issued a 20% stock dividend. Give the journal entryfor this dividend.What is the par value per share after the dividend? How doesthis dividend affect stockholders’ equity?

1eSG_C11_0131792075.QXD 10/23/06 9:30 AM Page 382

Page 415: polpoa_sg

Demo Doc 3 Solutions | Chapter 11 383

Part 1 Part 2 Part 3 Demo DocComplete

Cash decreases (a credit) by the amount of dividends paid. After the stock divi-dend of February 1, a total of 75,000 + 15,000 = 90,000 shares are outstanding.Therefore, the cash paid is:

90,000 shares " $0.60 per share ! $54,000

With any dividend, Retained Earnings decreases (a debit). In this case, it is adecrease of the cash paid of $54,000.

Account for cashdividends

4

Report stockholders’equity

7

Journal Entry:

PostDate Accounts Ref. Dr. Cr. Mar. 1 Retained Earnings 54,000 Cash ($90,000 ! $0.60) 54,000

Part 1 Part 2 Part 3 Demo DocComplete

Total equity does not change as a result of this transaction.

Before Stock After StockDividend Dividend

Common Stock $30,000 $36,000

Paid-In Capital in Excess of Par 0 39,000

Retained Earnings 60,000 15,000

Total Equity $90,000 (same) $90,000

Requirement 3

On March 1, 2009, Tinker declared and paid a cash dividend of $0.60 percommon share. Give the journal entry for this dividend. How does thisdividend affect stockholders’ equity?

1eSG_C11_0131792075.QXD 10/23/06 9:30 AM Page 383

Page 416: polpoa_sg

384 Chapter 11 | Demo Doc 4

Demo Doc 4Treasury StockLearning Objective 6

On January 1, 2008, Unter, Inc. purchased 4,000 shares of treasury stock for $10each. At this time, Paid-In Capital, Treasury Stock had a balance of $0.

Unter sold the treasury stock as follows:

April 1, 2008 Sold 1,000 shares for $12.00 cash each.

July 1, 2008 Sold 2,500 shares for $9.50 cash each.

October 1, 2008 Sold 500 shares for $8.25 cash each.

Requirements

1. Journalize all of Unter’s treasury stock transactions.

2. Suppose instead of holding onto the common stock purchased as treasurystock, Unter retired the stock on January 2, 2008. Would it be possible forUnter to later reissue the stock?

1eSG_C11_0131792075.QXD 10/23/06 9:30 AM Page 384

Page 417: polpoa_sg

Demo Doc 4 Solutions | Chapter 11 385

Demo Doc 4 SolutionsRequirement 1

Journalize all of Unter’s treasury stock transactions.

January 1, 2008 Purchased 4,000 shares of treasury stock for $10cash each. At this time, Paid-In Capital fromTreasury Stock transactions had a balance of $0.

Demo DocCompletePart 1 Part 2

The Common Stock account represents all issued common shares. When thecompany purchases treasury stock, these shares are still issued but are nolonger outstanding. To represent this decrease in the number of outstandingshares, the Treasury Stock account has a debit balance. It is a contra-equityaccount.When treasury stock is purchased, the Treasury Stock account increases (adebit) by the cost of the treasury shares:

4,000 ! $10 share ! $40,000

Cash decreases (a credit) by $40,000.

Journal Entry:

PostDate Accounts Ref. Dr. Cr. Jan. 1 Treasury Stock (4,000 ! $10) 40,000 Cash 40,000

April 1, 2008 Sold 1,000 shares for $12 cash each.

Cash increases (a debit) by:

1,000 shares ! $12 ! $12,000

Treasury Stock decreases (a credit) by the original cost of the treasury shares:

1,000 shares ! $10 ! $10,000

The difference between these two amounts is a balancing amount to Paid-InCapital from Treasury Stock Transactions of ($12,000 " $10,000) = $2,000credit (increase).

Journal Entry:

PostDate Accounts Ref. Dr. Cr. Apr. 1 Cash (1,000 ! $12) 12,000 Treasury Stock (1,000 ! $10 cost) 10,000 Paid-In Capital from Treasury Stock Transaction (to balance) 2,000

Account for treasurystock

6

1eSG_C11_0131792075.QXD 10/23/06 9:30 AM Page 385

Page 418: polpoa_sg

386 Chapter 11 | Demo Doc 4 Solutions

July 1, 2008 Sold 2,500 shares for $9.50 cash each.

Cash increases (a debit) by:

2,500 shares " $9.50 ! $23,750

Treasury Stock decreases (a credit) by the original cost of the treasury shares:

2,500 shares " $10 ! $25,000

The difference between these two amounts is a balancing debit (decrease) toPaid-In Capital from Treasury Stock Transactions:

$25,000 # $23,750 ! $1,250

Note that we cannot have a debit/negative balance in Paid-In Capital fromTreasury Stock Transactions. However, from the entry on April 1, we knowthat the account holds a balance of $2,000. This balance is more than enoughto cover a $1,250 debit.

Journal Entry:

PostDate Accounts Ref. Dr. Cr. July 1 Cash (2,500 ! $9.50) 23,750 Paid-In Capital from Treasury Stock Transaction (to balance) 1,250 Treasury Stock (2,500 ! $10 cost) 25,000

After this transaction, Paid-In Capital from Treasury Stock Transactionshas a balance of $750 credit:

Paid-In-Capital from Treasury Stock Transactions Treasury Stock

Bal. 0 Jan. 1 40,000Apr. 1 2,000 Apr. 1 10,000

July 1 1,250 Mar. 1 25,000

Bal. 5,000Bal. 750

October 1, 2008 Sold 500 shares for $8.25 cash each.

Cash increases (a debit) by:

500 shares " $8.25 ! $4,125

Treasury Stock decreases (a credit) by the original cost of the treasury shares:

500 shares " $10 ! $5,000

The difference between these two amounts would normally be a balancingamount to Paid-In Capital from Treasury Stock Transactions, but in thiscase, the difference is a decrease (a debit) of:

$5,000 # $4,125 ! $875

Only $750 remains in the Paid-In Capital from Treasury Stock Transactionsaccount. This amount is not enough to cover the debit that would normallybe required.

1eSG_C11_0131792075.QXD 10/23/06 9:30 AM Page 386

Page 419: polpoa_sg

Demo Doc 4 Solutions | Chapter 11 387

Instead, we take as much as possible from the Paid-In Capital from TreasuryStock Transactions account. We debit the $750 left in the account. Theremaining $875 " $750 = $ 125 is balanced with a decrease (a debit) toRetained Earnings.

Journal Entry:

PostDate Accounts Ref. Dr. Cr. Oct. 1 Cash (500 ! $8.25) 4,125 Paid-In Capital from Treasury Stock Transactions 750 Retained Earnings (to balance) 125 Treasury Stock (500 ! $10 cost) 5,000

Demo DocCompletePart 1 Part 2

Requirement 2

Suppose instead of holding onto the common stock purchased as treasurystock, Unter retired the stock on January 2, 2008.Would it be possible for Unterto later reissue the stock?

When stock is retired, the stock certificates are canceled and the stock ceases toexist.

Retired/canceled stock can no longer be reissued.

Part 1 Part 2 Demo DocComplete

Account for treasurystock

6

1eSG_C11_0131792075.QXD 10/23/06 9:30 AM Page 387

Page 420: polpoa_sg

388 Chapter 11 | Quick Practice Questions

Quick Practice QuestionsTrue/False

_____ 1. Stockholders in a corporation are personally liable for the debts of thecorporation.

_____ 2. Stock dividends increase total stockholders’ equity.

_____ 3. Par value is an arbitrary amount assigned by a company to a share ofits stock.

_____ 4. A credit balance in Retained Earnings is referred to as a deficit.

_____ 5. When a corporation sells par value stock at an amount greater thanpar value, other income is reported on the income statement.

_____ 6. Dividends become a liability of the corporation on the payment date.

_____ 7. The owners of cumulative preferred stock must receive all dividends inarrears plus the current year’s dividends before the common stockhold-ers get a dividend.

_____ 8. A stock split reduces the number of outstanding shares and the parvalue of the stock.

_____ 9. When treasury stock is purchased, the balance in the Common Stockaccount remains unchanged.

_____10. Stock dividends may cause the price of the stock to increase.

Multiple Choice

1. What is the document that is used by a state to grant permissionto form a corporation called? a. Charterb. Proxyc. Stock certificated. Bylaw agreement

2. What is the ownership percentage used as a cutoff point fordistinguishing between a small and a large stock dividend?a. 15%b. 10%c. 25%d. 50%

3. Which of the following best describes paid-in capital?a. Investments by the stockholders of a corporationb. Investments by the creditors of a corporationc. Capital that the corporation has earned through profitable operationsd. All of the above

1eSG_C11_0131792075.QXD 10/23/06 9:30 AM Page 388

Page 421: polpoa_sg

Quick Practice Questions | Chapter 11 389

4. Which of the following best describes retained earnings?a. It is classified as a liability on the corporate balance sheetb. It does not appear on any financial statementc. It represents capital that the corporation has earned through prof-

itable operationsd. It represents investments by the stockholders of a corporation

5. What individual(s) has the authority to obligate the corporationto pay dividends?a. The stockholdersb. The board of directorsc. The president of the companyd. The chief executive officer

6. What entry is made to record a 2-for-1 stock split? a. Credit to Common Stockb. Credit to Retained Earningsc. Debit to Retained Earningsd. No journal entry is made for a stock split

7. Which of the following would be recorded for the issuance of55,000 shares of no-par common stock at $13.50 per share? a. Credit to Paid-In Capital in Excess of No-Par Value—Common for

$742,500b. Credit to Common Stock for $742,500c. Credit to Cash for $742,500d. Debit to Paid-In Capital in Excess of No-Par Value—Common for

$742,500

8. Which of the following is true for dividends?a. Dividends are a distribution of cash to the stockholdersb. Dividends decrease both the assets and the total stockholders’ equity

of the corporationc. Dividends increase retained earningsd. Both (a) and (b) are correct

9. Dividends on cumulative preferred stock of $2,500 are in arrearsfor 2008. During 2009, the total dividends declared amount to$10,000. The corporation has 6,000 shares of $10 par, 10%cumulative preferred stock outstanding and 10,000 shares of $5par common stock outstanding. What is the total amount ofdividends payable to each class of stock in 2009?a. $5,000 to preferred, $5,000 to commonb. $6,000 to preferred, $4,000 to commonc. $8,500 to preferred, $1,500 to commond. $10,000 to preferred, $0 to common

10. What effect does the purchase of treasury stock have on thenumber of a corporation’s shares?a. Issued shares exceed authorized sharesb. Outstanding shares exceed issued sharesc. Outstanding shares exceed authorized sharesd. Outstanding shares are less than issued shares

1eSG_C11_0131792075.QXD 10/23/06 9:30 AM Page 389

Page 422: polpoa_sg

390 Chapter 11 | Quick Practice Questions

Quick Exercises

11-1. Journalize the following transactions.

a. Firm Body Corporation sells 12,000 shares of $10 par commonstock for $13.00 per share.

b. Firm Body Corporation sells 5,000 shares of $50 par, 10%, cumula-tive preferred stock for $59 per share.

c. Received a building with a market value of $115,000, and issued6,400 shares of $10 par common stock in exchange.

Journal Entry:

PostDate Accounts Ref. Dr. Cr.

Journal Entry:

PostDate Accounts Ref. Dr. Cr.

Journal Entry:

PostDate Accounts Ref. Dr. Cr.

11-2. The following is a list of stockholders’ equity accounts appearing on the balance sheet for O’Neil Corporation onDecember 31, 2008:

Common stock, $10 par value $300,000Paid-in capital in excess of par—common 200,000Retained earnings 225,000Preferred stock, $50 par value 125,000Paid-in capital in excess of par—preferred 30,000

Determine the following:

a. How many shares of preferred stock have been issued?

b. What was the average issuance price of the preferred stock pershare?

1eSG_C11_0131792075.QXD 10/23/06 9:30 AM Page 390

Page 423: polpoa_sg

Quick Practice Questions | Chapter 11 391

c. How many shares of common stock have been issued?

d. What is total paid-in capital?

e. What is total stockholders’ equity?

11-3. Papelbon Corporation reports the following transactions for 2009:

Jan. 10 Sold 6,000 shares of 9%, noncumulative $50 par, preferredstock for $85 per share.

Feb. 19 Sold 3,000 shares of $10 par common stock for $15 per share.Oct. 12 The board announced a 15% stock dividend on the common

stock. The current market price of the common stock is $22per share. Papelbon Corporation has 120,000 shares of com-mon stock outstanding on October 12.

Requirement

Journalize these transactions.

Journal Entry:

PostDate Accounts Ref. Dr. Cr.

Journal Entry:

PostDate Accounts Ref. Dr. Cr.

Journal Entry:

PostDate Accounts Ref. Dr. Cr.

1eSG_C11_0131792075.QXD 11/22/06 10:27 AM Page 391

Page 424: polpoa_sg

392 Chapter 11 | Quick Practice Questions

Journal Entry:

PostDate Accounts Ref. Dr. Cr.

Journal Entry:

PostDate Accounts Ref. Dr. Cr.

11-4. Following is the stockholders’ equity section of the balancesheet for Watson Corporation as of December 1, 2009:

Preferred stock, $100 par, 6% cumulative, 10,000 shares authorized, 7,500 shares issued $ 750,000

Common stock, $10 par, 200,000 shares issued,130,000 shares issued 1,300,000

Paid-in capital in excess of par—common 520,000Total paid-in capital $2,570,000

Retained earnings 450,000Total stockholders’ equity $3,020,000

Watson Corporation reports the following transactions forDecember 2009:

Dec. 5 Declared the required cash dividend on the preferred stockand a $0.40 dividend on the common stock.

20 Paid the dividends declared on December 5.

Requirements

1. Journalize these transactions.

1eSG_C11_0131792075.QXD 11/22/06 10:27 AM Page 392

Page 425: polpoa_sg

Quick Practice Questions | Chapter 11 393

2. What is the total stockholders’ equity after posting the preceding entries?

11-5. Victory Corporation reported the following stockholders’equity items on December 31, 2008:

Preferred stock, 5%, cumulative $100 par,7,000 shares authorized, 1,000 shares issued $100,000

Paid-in-capital in excess of par—preferred 55,000Common stock, $50 par, 10,000 shares authorized,

5,000 shares issued 250,000Paid-in-capital in excess of par—common 235,000Retained earnings 455,300Treasury common stock, at cost, 700 shares 96,000

Requirements

1. Compute the:a. Number of shares of common stock outstanding

b. Number of shares of preferred stock outstanding

c. Average issue price of common stock

d. Average issue price of preferred stock

2. Assume that Victory Corporation declares a 4-for-1 stock split.Compute the:a. Number of shares of common outstanding

b. Par value

1eSG_C11_0131792075.QXD 11/22/06 10:27 AM Page 393

Page 426: polpoa_sg

394 Chapter 11 | Do It Yourself! Question 1

Do It Yourself! Question 1

Common StockDinner Co. had the following information at December 31, 2008:

Stockholders’ Equity

Common stock, 500,000 authorized, 50,000 issued and outstanding shares $100,000

Additional paid-in capital $50,000

Retained earnings $400,000

Total stockholders’ equity $550,000

Requirements

1. What is the par value per share of the common stock?

2. On average, what was the original per-share issue price of the commonstock?

3. On January 9, 2009, Dinner issued another 10,000 common shares for $4cash per share. Journalize this transaction.

Describe stockholders’equity and classes ofstock

2

Record the issuance ofstock

3

Record the issuance ofstock

3

Journal Entry:

PostDate Accounts Ref. Dr. Cr.

1eSG_C11_0131792075.QXD 11/22/06 10:27 AM Page 394

Page 427: polpoa_sg

Do It Yourself! Question 2 | Chapter 11 395

Do It Yourself! Question 2

Preferred StockLunch Corp. issued 5,000 8%, $20 par cumulative preferred shares on January 1,2008 for $25 cash per share. Lunch had never had preferred shares before this date.

On December 31, 2008, Lunch paid $5,000 in cash dividends to its share-holders. On December 31, 2009, Lunch paid $15,000 in cash dividends to itsshareholders.

Requirements

1. Journalize the issuance of the preferred shares on January 1, 2008.

2. How much in dividends is Lunch supposed to pay to the preferredshareholders each year?

3. How much of the $5,000 paid as dividends in 2008 went to the preferredand common shareholders?

4. How much of the $15,000 paid as dividends in 2009 went to the preferredand common shareholders?

Record the issuance ofstock

3

Account for cashdividends

4

Account for cashdividends

4

Account for cashdividends

4

Journal Entry:

PostDate Accounts Ref. Dr. Cr.

1eSG_C11_0131792075.QXD 11/22/06 10:27 AM Page 395

Page 428: polpoa_sg

396 Chapter 11 | Do It Yourself! Question 3

2. On January 2, 2009, Garbage split its common stock 2-for-1. Journalize thesplit.What are the par and market values per share after the split?

Do It Yourself! Question 3

Stock Splits and DividendsOn December 31, 2008, Garbage, Inc., had 12,000 common shares outstandingwith a market price of $8 per share and a par value of $2 per share.

Requirements

1. On January 1, 2009, Garbage issued a 15% stock dividend. Journalize thisdividend.

Account for stock divi-dends and stock splits

5

Journal Entry:

PostDate Accounts Ref. Dr. Cr.

Account for stock divi-dends and stock splits

5

Journal Entry:

PostDate Accounts Ref. Dr. Cr.

Journal Entry:

PostDate Accounts Ref. Dr. Cr.

Account for cashdividends

4 3. On January 3, 2009, Garbage declared and paid a cash dividend of $0.30 percommon share. Journalize this dividend.

1eSG_C11_0131792075.QXD 11/22/06 10:27 AM Page 396

Page 429: polpoa_sg

Account for treasurystock

6

Journal Entry:

PostDate Accounts Ref. Dr. Cr.

Journal Entry:

PostDate Accounts Ref. Dr. Cr.

Journal Entry:

PostDate Accounts Ref. Dr. Cr.

Journal Entry:

PostDate Accounts Ref. Dr. Cr.

Do It Yourself! Question 4

Treasury StockOn January 1, 2008, Hartnick Co. purchased 1,000 shares of treasury stock for$7 cash each. At this time, Paid-In Capital from Treasury Stock Transactionshad a balance of $0.

Hartnick sold the treasury stock as follows:

February 1, 2008 Sold 200 shares for $10 cash each.

March 1, 2008 Sold 500 shares for $6 cash each.

April 1, 2008 Sold 300 shares for $6.50 cash each.

Requirement

Journalize all of Hartnick’s treasury stock transactions.

1eSG_C11_0131792075.QXD 11/22/06 10:27 AM Page 397

Page 430: polpoa_sg

398 Chapter 11 | Quick Practice Solutions

Quick Practice Solutions True/False

F 1. Stockholders in a corporation are personally liable for the debts of the corporation.False–Stockholders are not personally liable for the debts of the corporation. (p. 571)

F 2. Stock dividends increase total stockholders’ equity.False–Stock dividends have no effect on total stockholders’ equity.(p. 584)

T 3. Par value is an arbitrary amount assigned by a company to a share of its stock. (p. 575)

F 4. A credit balance in Retained Earnings is referred to as a deficit.False–A debit balance in Retained Earnings is referred to as a deficit. (p. 573)

F 5. When a corporation sells par value stock at an amount greater than par value, other income is reported on the income statement.False–When a corporation sells par value stock at an amount greater than par value, paid-in capital in excess of par value is journalized for the difference. This entry has no effect on the income statement from a company’s stock transactions.(pp. 576–577)

F 6. Dividends become a liability of the corporation on the payment date.False–Dividends become a liability of the corporation on the declaration date. (p. 579)

T 7. The owners of cumulative preferred stock must receive all dividends in arrears plus the current year’s dividends before the common stockholders get a dividend. (p. 581)

F 8. A stock split reduces the number of outstanding shares and the par value of the stock.False–A stock split increases the number of outstanding shares of stock and reduces the par value of the stock. (p. 585)

T 9. When treasury stock is purchased, the balance in the Common Stock account remains unchanged. (pp. 585–586)

F 10. Stock dividends may cause the price of the stock to increase.False–Stock dividends may cause the price of the stock to decreasebecause of the increased supply of the stock. (p. 583)

1eSG_C11_0131792075.QXD 10/23/06 9:30 AM Page 398

Page 431: polpoa_sg

Quick Practice Solutions | Chapter 11 399

Multiple Choice

1. What is the document that is used by a state to grant permissionto form a corporation called? (p. 572)a. Charterb. Proxyc. Stock certificated. Bylaw agreement

2. What is the ownership percentage used as a cutoff point fordistinguishing between a small and a large stock dividend? (p. 583)a. 15%b. 10%c. 25%d. 50%

3. Which of the following best describes paid-in capital? (p. 573)a. Investments by the stockholders of a corporationb. Investments by the creditors of a corporationc. Capital that the corporation has earned through profitable operationsd. All of the above

4. Which of the following best describes retained earnings? (p. 573)a. It is classified as a liability on the corporate balance sheetb. It does not appear on any financial statementc. It represents capital that the corporation has earned through prof-

itable operationsd. It represents investments by the stockholders of a corporation

5. What individual(s) has the authority to obligate the corporationto pay dividends? (p. 579)a. The stockholdersb. The board of directorsc. The president of the companyd. The chief executive officer

6. What entry is made to record a 2-for-1 stock split? (p. 584)a. Credit to Common Stockb. Credit to Retained Earningsc. Debit to Retained Earningsd. No journal entry is made for a stock split

7. Which of the following would be recorded for the issuance of55,000 shares of no-par common stock at $13.50 per share? (p. 576)a. Credit to Paid-In Capital in Excess of No-Par Value—Common for

$742,500b. Credit to Common Stock for $742,500c. Credit to Cash for $742,500d. Debit Paid-In Capital in Excess of No-Par Value—Common for

$742,500

1eSG_C11_0131792075.QXD 10/23/06 9:30 AM Page 399

Page 432: polpoa_sg

400 Chapter 11 | Quick Practice Solutions

8. Which of the following is true for dividends? (p. 573)a. Dividends are a distribution of cash to the stockholdersb. Dividends decrease both the assets and the total stockholders’ equity

of the corporationc. Dividends increase retained earningsd. Both (a) and (b) are correct

9. Dividends on cumulative preferred stock of $2,500 are in arrearsfor 2008. During 2009, the total dividends declared amount to$10,000. The company has 6,000 shares of $10 par, 10% cumulativepreferred stock outstanding and 10,000 shares of $5 par commonstock outstanding. What is the total amount of dividends payableto each class of stock in 2009? (p. 581)a. $5,000 to preferred, $5,000 to commonb. $6,000 to preferred, $4,000 to commonc. $8,500 to preferred, $1,500 to commond. $10,000 to preferred, $0 to common

10. What effect does the purchase of treasury stock have on thenumber of a corporation’s shares? (pp. 585–586)a. Issued shares exceed authorized sharesb. Outstanding shares exceed issued sharesc. Outstanding shares exceed authorized sharesd. Outstanding shares are less than issued shares

1eSG_C11_0131792075.QXD 10/23/06 9:30 AM Page 400

Page 433: polpoa_sg

Quick Practice Solutions | Chapter 11 401

Quick Exercise

11-1. Journalize the following transactions. (pp. 576–578)

a. Firm Body Corporation sells 12,000 shares of $10 par commonstock for $13.00 per share.

b. Firm Body Corporation sells 5,000 shares of $50 par, 10%, cumula-tive preferred stock for $59 per share.

c. Received a building with a market value of $115,000, and issued6,400 shares of $10 par common stock in exchange.

11-2. The following is a list of stockholders’ equity accounts appear-ing on the balance sheet for O’Neil Corporation on December 31,2008:

Common stock, $10 par value $300,000

Paid-in capital in excess of par—common 200,000

Retained earnings 225,000

Preferred stock, $50 par value 125,000

Paid-in capital in excess of par—preferred 30,000

Journal Entry:

PostDate Accounts Ref. Dr. Cr.

Journal Entry:

PostDate Accounts Ref. Dr. Cr.

Journal Entry:

PostDate Accounts Ref. Dr. Cr.

a. Cash 156,000 Common Stock 120,000 Paid-In Capital in Excess of Par—Common 36,000

b. Cash 295,000 Preferred Stock 250,000 Paid-In Capital in Excess of Par—Preferred 45,000

c. Building 115,000 Common Stock 64,000 Paid-In Capital in Excess of Par—Common 51,000

1eSG_C11_0131792075.QXD 10/23/06 9:30 AM Page 401

Page 434: polpoa_sg

Journal Entry:

PostDate Accounts Ref. Dr. Cr.

Journal Entry:

PostDate Accounts Ref. Dr. Cr.

Journal Entry:

PostDate Accounts Ref. Dr. Cr.

Jan. 10 Cash 510,000 Common Stock 300,000 Paid-In Capital in Excess of Par—Common 210,000

Feb. 19 Cash 45,000 Preferred Stock 30,000 Paid-In Capital in Excess of Par—Preferred 15,000

Oct. 12 Retained Earnings 396,000 Common Stock 180,000 Paid-In Capital in Excess of Par–Common 216,000

Determine the following: (p. 578)

a. How many shares of preferred stock have been issued? $125,000/$50 = 2,500

b. What was the average issuance price of the preferred stock pershare? ($125,000 + $30,000)/2,500 = $62

c. How many shares of common stock have been issued? $300,000/$10 = 30,000

d. What is total paid-in capital? $300,000 + $200,000 + $125,000 + $30,000 = $655,000

e. What is total stockholders’ equity?$655,000 + $225,000 = $880,000

11-3. Papelbon Corporation reports the following transactions for2009: (pp. 579–583)

Jan. 10 Sold 6,000 shares of 9%, noncumulative $50 par, preferred stock for $85 per share.

Feb. 19 Sold 3,000 shares of $10 par common stock for $15 per share.

Oct. 12 The board announced a 15% stock dividend on the common stock. The current market price of the common stock is $22 per share. Papelbon Corporation has 120,000 shares of common stock outstanding on October 12.

Requirement

Journalize these transactions.

1eSG_C11_0131792075.QXD 11/22/06 10:28 AM Page 402

Page 435: polpoa_sg

Quick Practice Solutions | Chapter 11 403

Journal Entry:

PostDate Accounts Ref. Dr. Cr. Dec. 5 Retained Earnings 97,000 Dividends Payable 97,000

Journal Entry:

PostDate Accounts Ref. Dr. Cr. Dec. 20 Dividends Payable 97,000 Cash 97,000

Requirement 2

What is the total stockholders’ equity after posting the precedingentries?

$2,923,000 ($3,020,000 – $97,000)

11-4. Following is the stockholders’ equity section of the balancesheet for Watson Corporation as of December 1, 2009:

Preferred stock, $100 par, 6% cumulative, $ 750,00010,000 shares authorized, 7,500 shares issued

Common stock, $10 par, 200,000 shares authorized, 130,000 shares issued 1,300,000

Paid-in capital in excess of par—common 520,000

Total paid-in capital $2,570,000

Retained earnings 450,000

Total stockholders’ equity $3,020,000

Watson Corporation reports the following transactions forDecember 2009: (pp. 578–581)

Dec. 5 Declared the required cash dividend on the preferred stockand a $0.40 dividend on the common stock.

20 Paid the dividends declared on December 5.

Requirement 1

Journalize these transactions.

1eSG_C11_0131792075.QXD 11/22/06 10:28 AM Page 403

Page 436: polpoa_sg

404 Chapter 11 | Quick Practice Solutions

11-5. Victory Corporation reported the following stockholders’equity items on December 31, 2008: (pp. 576–586)

Preferred stock, 5%, cumulative $100 par,7,000 shares authorized, 1,000 shares issued $100,000

Paid-in-capital in excess of par—preferred 55,000 Common stock, $50 par, 10,000 shares authorized,

5,000 shares issued 250,000 Paid-in-capital in excess of par—common 235,000 Retained earnings 455,300 Treasury common stock, at cost, 700 shares 96,000

Requirement 1

Compute the:a. Number of shares of common stock outstanding

$250,000/$50 = 5,000 ! 700 = 4,300b. Number of shares of preferred stock outstanding

$100,000/$100 = 1,000c. Average issue price of common stock

$250,000 + 235,000 = 485,000/5,000 = $97d. Average issue price of preferred stock

$100,000 + $55,000 = $155,000/1,000 = $155

Requirement 2

Assume that Victory Corporation declares a 4-for-1 stock split. Com-pute the:

a. Number of shares of common outstanding4,300 shares ! 4 = 17,200

b. Par value$50/4 = $12.50

1eSG_C11_0131792075.QXD 11/22/06 10:28 AM Page 404

Page 437: polpoa_sg

Do It Yourself! Question 2 Solutions | Chapter 11 405

Do It Yourself! Question 1 Solutions

Requirement 1

What is the par value per share of common stock?

Requirement 2

On average, what was the original per-share issue price of the common stock?

Total cash received from issuance of the common shares (par + additional paid-incapital):

Requirement 3

On January 9, 2009, Dinner issued another 10,000 common shares for $4 cashper share. Journalize this transaction.

$100,000 $50,000 $150,000

$150,00050,000 Sha

! "

rres$3 Cash Received Per Share"

Par ValuePer Share

$100,00050,000 Shares$2

"

" Per Share

Journal Entry:

PostDate Accounts Ref. Dr. Cr. Jan. 9 Cash (10,000 " $4) 40,000 Common Stock (10,000 " $2) 20,000 Paid-in Capital in Excess of Par (to balance) 20,000

1eSG_C11_0131792075.QXD 11/22/06 10:28 AM Page 405

Page 438: polpoa_sg

406 Chapter 11 | Do It Yourself! Question 2 Solutions

Do It Yourself! Question 2 Solutions

Requirement 1

Journalize the issuance of the preferred shares on January 1, 2008.

Journal Entry:

PostDate Accounts Ref. Dr. Cr. Jan. 1 Cash (5,000 " $25) 125,000 Common Stock (5,000 " $20) 100,000 Paid-in Capital in Excess of Par (to balance) 25,000

Requirement 2

How much in dividends is Lunch supposed to pay to the preferred shareholderseach year?

Preferred shareholders are supposed to receive:

$20 par # 8% " $1.60 per share annually$1.60 # 5,000 " $8,000 in dividends per year for all outstanding preferred shares

Requirement 3

How much of the $5,000 paid as dividends in 2008 went to the preferred andcommon shareholders?

The full $8,000 was not paid, therefore, the entire $5,000 goes to the preferredshareholders. Common shareholders get nothing.

$8,000 $ $5,000 " $3,000 dividends in arrears

Requirement 4

How much of the $15,000 paid as dividends in 2009 went to the preferred andcommon shareholders?

Preferred shareholders received:

$8,000 ! $3,000 " $11,000

Common shareholders received:

$15,000 $ $11,000 " $4,000

1eSG_C11_0131792075.QXD 11/22/06 10:28 AM Page 406

Page 439: polpoa_sg

Do It Yourself! Question 3 Solutions | Chapter 11 407

Journal Entry:

PostDate Accounts Ref. Dr. Cr. Jan. 1 Retained Earnings (1,800 " $8) 14,400 Common Stock (1,800 " $2) 3,600 Paid-in Capital in Excess of Par (to balance) 10,800

Journal Entry:

PostDate Accounts Ref. Dr. Cr. Jan. 1 Retained Earnings 8,280 Cash (27,600 3 $0.30) 8,280

Requirement 2

On January 2, 2009, Garbage split its common stock 2-for-1. Journalize thesplit.What are the par and market values per share after the split?

Common shares before split:

12,000 ! 1,800 " 13,800

Number of shares after stock split:

13,800 # 2/1 " 27,600 shares

Par value per share after split:

$2 # 1/2 " $1

Market price per share after split:

$8 # 1/2 " $4

No journal entry is needed for a stock split.

Requirement 3

On January 3, 2009, Garbage declared and paid a cash dividend of $0.30 percommon share. Journalize this dividend.

Do It Yourself! Question 3 Solutions

Requirement 1

On January 1, 2009, Garbage issued a 15% stock dividend. Journalize thisdividend.

Number of new shares issued for stock dividend:

12,000 # 15% " 1,800 new shares

1eSG_C11_0131792075.QXD 11/22/06 10:28 AM Page 407

Page 440: polpoa_sg

408 Chapter 11 | Do It Yourself! Question 4 Solutions

Do It Yourself! Question 4 Solutions

Requirement

Journalize all of Hartnick’s treasury stock transactions.

January 1, 2008 Hartnick Co. purchased 1,000 shares of treasurystock for $7 cash each. At this time, Paid-In Capitalfrom Treasury Stock Transactions had a balance of $0.

Journal Entry:

Jan. 1 Treasury Stock (1,000 " $7) 7,000 Cash 7,000

PostDate Accounts Ref. Dr. Cr.

Journal Entry:

Feb. 1 Cash (200 " $10) 2,000 Paid-In Capital from Treasury Stock Transaction (to balance) 600 Treasury Stock (200 3 $7 cost) 1,400

PostDate Accounts Ref. Dr. Cr.

Journal Entry:

Mar. 1 Cash (500 " $6) 3,000 Paid-In Capital from Treasury Stock Transactions 500 Treasury Stock (500 3 $7 cost) 3,500

PostDate Accounts Ref. Dr. Cr.

Paid-In-Capital from Treasury Stock Transactions Treasury Stock

Bal. 0 Jan. 1 7,000Feb. 1 1,600 Feb. 1 1,000

Mar. 1 500 Mar. 1 3,000Bal. 100 Bal. 2,100

February 1, 2008 Sold 200 shares for $10 cash each.

March 1, 2008 Sold 500 shares for $6 cash each.

After this transaction:

1eSG_C11_0131792075.QXD 11/22/06 10:28 AM Page 408

Page 441: polpoa_sg

Do It Yourself! Question 4 Solutions | Chapter 11 409

Journal Entry:

Apr. 1 Cash (300 ! $6.50) 1,950 Paid-In Capital from Treasury Stock Transactions 100 Retained Earnings (to balance) 50 Treasury Stock (300 3 $7 cost) 2,100

PostDate Accounts Ref. Dr. Cr.

April 1, 2008 Sold 300 shares for $6.50 cash each.

1eSG_C11_0131792075.QXD 10/23/06 9:31 AM Page 409

Page 442: polpoa_sg

The Power of PracticeFor more practice using the skills learned in this chapter, visit MyAccountingLab.There you will find algorithmically generated questions that are based on theseDemo Docs and your main textbook’s Review and Accounting Practice sections.

To go to MyAccountingLab and follow these steps:

1. Direct your URL to www.myaccountinglab.com.2. Log in using your name and password3. Click the MyAccountingLab link.4. Click Study Plan in the left navigation bar.5. From the table of contents, select Chapter 11, Corporations and Stockholders’

Equity.6. Click a link to work on the tutorial exercises.

410 Chapter 11 | The Power of Practice

1eSG_C11_0131792075.QXD 10/23/06 9:31 AM Page 410

Page 443: polpoa_sg

Long-Term Liabilities

WHAT YOU PROBABLY ALREADY KNOW

You probably already know that when you purchase a home, youwill likely need to obtain a mortgage. You can choose from varioustypes of mortgages, but a popular form is the fixed-rate mortgage.If you obtain a 30-year fixed-rate mortgage, you know that you arelocked into the interest rate specified in the mortgage contract for30 years. Mortgage interest rates may increase or decrease insubsequent years, but it won’t affect your fixed monthly payment.If the interest rate decrease is material enough, you may choose torefinance the mortgage to save future interest costs. Refinancingmeans that the old mortgage is paid off with a new mortgage loan.The characteristics of a bond are similar to a mortgage. The issuerof a bond incurs a long-term liability and is committed to payinterest at the fixed interest rate included in the bond agreement.Sometimes an issuer will refinance its debt if the interest ratedecreases by issuing new bonds at the lower interest rate andpaying off the higher-rate bonds. In this chapter, we learn aboutbonds and mortgages and how to account for them.

Learning Objectives

Describe mortgages and leases, and record mortgage and leasetransactions.

A mortgage is a long-term loan taken out for the purchase of land,buildings, or both. The loan is secured by the asset mortgaged ascollateral. When a mortgage loan is issued, the asset account is debitedand the Mortgage Note Payable (a liability) is credited. Payments, whichare the same for each period, consist of principal and interest; however,the portion allocated to principal increases with time and the interestexpense portion decreases. The Interest Expense and Mortgage NotePayable accounts are debited for their respective amounts and Cash iscredited for the periodic payment amount.

With a lease, the lessee agrees to pay to use an asset owned by thelessor. Typical lease payments are recorded with a debit to RentExpense and a credit to Cash. Some leases contain characteristicssimilar to an asset purchase rather than a usual rental. In these cases, itis considered a capital lease and is accounted for like an asset purchasethat is financed; the asset account is debited and Lease Payable iscredited for the total lease commitment. Review the amortizationschedule in Exhibit 12-1 (p. 627) and the entry to record the mortgagepayment.

1

12

1eSG_C12_0131792075.qxd 10/23/06 9:33 AM Page 411

Page 444: polpoa_sg

412 Chapter 12 | Long-Term Liabilities

Describe bonds payable.

A bond is a long-term liability that may be issued by corporations; local,state, or federal governments; and agencies. The principal amount (parvalue) is the amount to be paid to the investor on the maturity date. Thisamount is recorded in the Bonds Payable account. Over the life of thebond, interest will be paid at the stated (fixed) interest rate.

Bond market prices are quoted as a percentage of the principal amount.If the market price of the bond is less than 100 (100% of par value), thebond will be sold for less than the principal amount. The differencebetween the principal amount and the amount received in this case isconsidered a Discount on Bonds Payable. A discount may occur becausethe market or effective rate is higher than the stated interest rate. Thediscount compensates the investor for the interest rate differential byproviding an additional return to the investor. If the market price of the bond is greater than 100, the bond will be sold for more than the principal amount (a premium). A premium may occur because the market or effective rate is lower than the stated interest rate. Thepremium reduces the return to the investor and the interest expenseincurred by the issuer from the stated to the market rate. Review bondfinancing compared to stock in Exhibit 12-2 (p. 630). Review therelationship between the stated and market rates of interest and the bond market price in Exhibit 12-5 (p. 634).

Account for bond issuance and bond interest.

When bonds are issued, the Cash account is debited for the cashreceived and the Bonds Payable account is credited for the principalamount. When bonds are issued at maturity value, a market price of 100,the Cash and Bonds Payable amounts are the same. If a bond is issued ata price of less than 100, the Discount on Bonds Payable (a contra-liability)is debited, in addition to the Cash account, for the difference between thecash received and the bond principal amount. If a bond is issued at aprice of more than 100, the Premium on Bonds Payable (a liability) iscredited, in addition to the Bonds Payable account, for the differencebetween the cash received and the bond principal amount.

The issuer will pay the investors the same amount of bond interest,whether the bonds are issued at maturity value, a discount, or premium.Recall that:

Interest ! Principal " Stated Rate of Interest " Time

When bonds are issued at maturity value, the stated rate of interestequals the market rate of interest and therefore the interest expense isequal to the amount of interest paid. When the bonds are issued at adiscount, the discount amount is the additional interest expense thatmust be recorded over the life of the bonds to record the higher marketrate of interest incurred. The Discount on Bonds Payable amount isamortized evenly over the life of the bond using the straight-linemethod. When the bonds are issued at a premium, the premium amountis the reduction of interest expense that must be recorded over the lifeof the bonds to record the lower market rate of interest incurred. ThePremium on Bonds Payable amount is amortized evenly over the life ofthe bond using the straight-line method. Review “Issuing Bonds Payable

3

2

1eSG_C12_0131792075.qxd 10/23/06 9:33 AM Page 412

Page 445: polpoa_sg

Long-Term Liabilities | Chapter 12 413

and Paying Interest” in the main text for bond issuance and interestexpense journal entries.

Account for bond repayment.

When bonds are repaid, the Bonds Payable liability is reduced (debited)and Cash is reduced (credited) for the maturity value. This entry is thereverse of the entry that journalized the issuance of bonds at maturityvalue. Bonds may be retired before the maturity date. When a companyretires the bonds, it is relieved of the net bond liability. This net amount,called the carrying amount, is equal to the Bonds Payable less Discounton Bonds Payable or plus Premium on Bonds Payable amounts. Thecompany must pay the market price to redeem the bonds. Similar to thedisposal of a plant asset in Chapter 9, a gain or loss may result andneeds to be journalized. If the amount received (elimination of bondcarrying amount) is greater than the amount given up (call or marketprice), then a Gain on Retirement of Bonds Payable must be credited forthe difference. If the amount received (elimination of bond carryingamount) is less than the amount given up (call or market price), then aLoss on Retirement of Bonds Payable must be debited for the difference.Review the example and journal entries in the “Retirement of Bonds”section of the main text.

Report long-term liabilities on the balance sheet.

Mortgages, capital leases, and bonds payable may appear as long-termliabilities on the balance sheet. Recall that the Premium on BondsPayable is a liability account and would be shown with the BondsPayable account as a long-term liability. The Discount on Bonds Payableis a contra-liability account and would be shown as a deduction from theBonds Payable account. The portion of these liabilities that are due withinone year may be classified as current liabilities. Interest payable on theliabilities is also included as a current liability. Review the illustrationunder “Reporting Liabilities on the Balance Sheet” in the main text.

4

5

1eSG_C12_0131792075.qxd 10/23/06 9:33 AM Page 413

Page 446: polpoa_sg

414 Chapter 12 | Demo Doc 1

Demo Doc 1Bonds Payable (Straight-Line Amortization)Learning Objectives 3–5

Blue Co. issued $50,000 maturity value of bonds payable for $51,788 cash onJanuary 1, 2008. The bonds had a stated rate of 12%, but the market rate was10%. Interest is paid semiannually, and the bonds are due in two years.

Blue uses the straight-line method of amortization.

Requirements

1. Are these bonds issued at a discount or premium? How do you know?

2. Journalize Blue’s issuance of the bonds on January 1, 2008.

3. Journalize Blue’s first two interest payments on June 30, 2008, andDecember 31, 2008.

4. Show how the bonds would appear on Blue’s December 31, 2008, balancesheet.

5. On January 2, 2009, 30% of the bonds were converted to 10,000 commonshares, $1 par. Journalize this transaction.

6. On January 3, 2009, Blue purchased the remaining bonds from themarketplace for $36,000 cash. The bonds were immediately retired.Journalize this transaction.

1eSG_C12_0131792075.qxd 10/23/06 9:33 AM Page 414

Page 447: polpoa_sg

Demo Doc 1 Solutions | Chapter 12 415

Journal Entry:

Jan. 1, 2008 Cash 51,788 Premium on Bonds Payable (to balance) 1,788 Bonds Payable (principal amount) 50,000

PostDate Accounts Ref. Dr. Cr.

Part 1 Part 2 Part 3 Part 4 Part 6Part 5 Demo DocComplete

3 Account for bondissuance and bondinterest

Part 1 Part 2 Part 3 Part 4 Part 6Part 5 Demo DocComplete

Part 1 Part 2 Part 3 Part 4 Part 6Part 5 Demo DocComplete

Demo Doc 1 SolutionsRequirement 1

Are these bonds issued at a discount or premium? How do you know?

These bonds were issued at a premium. We know it was a premium because thecash received for the bonds is more than the maturity value and because thestated rate is greater than the market rate.

Requirement 2

Journalize Blue’s issuance of the bonds on January 1, 2008.

Cash increases (a debit) by $51,788.Bonds Payable increases (a credit) by the bonds’ maturity value of $50,000.The difference between these two amounts is balanced to Premium on Bonds

Payable. The balancing amount is a credit of:

$51,788 # $50,000 ! $1,788

Requirement 3

Journalize Blue’s first two interest payments on June 30, 2008, and December 31,2008.

Under straight-line amortization, the premium will be amortized by the sameamount every interest period.

Premium Amortization Each Interest PeriodD

!iiscount or Premium

Number of Interest Periodss

3 Account for bondissuance and bondinterest

1eSG_C12_0131792075.qxd 10/23/06 9:33 AM Page 415

Page 448: polpoa_sg

416 Chapter 12 | Demo Doc 1 Solutions

The bonds are due in two years and have two interest payments per year. Thisresults in 2 ! 2 = 4 interest payment periods.

Every time interest expense is recorded, the premium will be amortized (that is,decreased/debited) by $447.

Cash will decrease by the cash interest paid.

The question states that the bonds are semiannual: that is, they pay interesttwice per year.

Interest Expense increases (a debit) by the balancing amount of:

$3,000 – $447 ! $2,553

Because the cash interest paid and the premium amortization do not change fromperiod to period, the entry to record the interest expense and payment is alwaysthe same. So the entry for June 30, 2008, and December 31, 2008, is:

Cash Interest Paid$50,000 12%

2!

"

! $3,000

Cash Interest PaidMaturity Value Stated Rat

!" ee

Number of Interest Payments per Year

Premium Amortization Each Interest Period!$11 788

4,

! $447

Journal Entry:

Interest Expense (to balance) 2,553 Premium on Bonds Payable ($1,788/4) 447 Cash ($50,000 ! 12%/2) 3,000

PostDate Accounts Ref. Dr. Cr.

Requirement 4

Show how the bonds would appear on Blue’s December 31, 2008, balancesheet.

5 Report long-term liabil-ities on the balancesheet

Part 1 Part 2 Part 3 Part 4 Part 6Part 5 Demo DocComplete

The bonds are reported in the liabilities section of the balance sheet.The premium is added to the bonds payable to create a net value.

Long-Term Liabilities:

Bonds Payable $50,000

Plus: Premium on Bonds Payable 894* $50,894*$1,788 - $447 (June amortization) - $447 (December amortization)

1eSG_C12_0131792075.qxd 10/23/06 9:33 AM Page 416

Page 449: polpoa_sg

Demo Doc 1 Solutions | Chapter 12 417

Premium on Bonds Payable

Bal. 0Jan. 1 1,788

June 30 447Dec. 31 447

Bal. 894

Part 1 Part 2 Part 3 Part 4 Part 6Part 5 Demo DocComplete

Requirement 5

On January 2, 2009, 30% of the bonds were converted to 10,000 commonshares with $1 par per share. Journalize this transaction.

Immediately before this transaction, the Premium account had a balance of $894:

If 30% of the bonds are converted, then 30% of the maturity value is converted:

30% ! $50,000 " $15,000

Also, 30% of the premium is converted:

30% ! $894 " $268

Note that these values are rounded to the nearest dollar.The bonds will not exist after the conversion, so the Bonds Payable account

decreases (a debit) by $15,000. The Premium also decreases (a debit) by $268.New common stock is issued for the conversion, so Common Stock increases

(a credit) by the par value of the new shares:

10,000 ! $1 " $10,000

The remainder is balanced to Paid-In Capital in Excess of Par. This balancingamount is:

$15,000 # $268 $ $10,000 " $5,268

Journal Entry:

Jan. 2, 2009 Bonds Payable ($50,000 ! 30%) 15,000 Premium on Bonds Payable ($894 ! 30%) 268 Common Stock (10,000 ! $1) Paid-In Capital in Excess of Par (to balance) 5,268

PostDate Accounts Ref. Dr. Cr.

10,000

Premium on Bonds Payable Bonds Payable

Bal. 0 Bal. 0Jan. 1 1,788 Jan. 1 50,000

June 30 447 Jan 2 15,000Dec. 31 447Jan. 2 268

Bal. 35,000

Bal. 626

4 Account for bondrepayment

1eSG_C12_0131792075.qxd 11/22/06 10:30 AM Page 417

Page 450: polpoa_sg

418 Chapter 12 | Demo Doc 1 Solutions

4 Account for bondrepayment

Part 1 Part 2 Part 3 Part 4 Part 6Part 5 Demo DocComplete

Part 1 Part 2 Part 3 Part 4 Part 6Part 5 Demo DocComplete

Because 30% of the bonds were converted to common stock, 100% – 30% = 70%remain. These bonds will be retired.

If 70% of the bonds are retired, then 70% of the maturity value is retired:

70% " $50,000 ! $35,000

70% of the premium is also retired:

70% " $894 ! $626

The bonds will not exist after retirement, so the Bonds Payable accountdecreases (a debit) by $35,000. The premium also decreases (a debit) by $626.

Cash decreases (a credit) by $36,000.The remainder is balanced to a gain or loss. In this case, the balancing

amount is a debit, which means a loss of:

$36,000 – $626 – $35,000 ! $374

Journal Entry:

Jan. 3, 2009 Bonds Payable ($50,000 ! 70%) 35,000 Loss of Retirement of Bonds Payable (to balance) 374 Premium on Bonds Payable ($894 ! 70%) 626 Cash 36,000

PostDate Accounts Ref. Dr. Cr.

Requirement 6

On January 3, 2009, Blue purchased the remaining bonds from the marketplacefor $36,000 cash.The bonds were immediately retired. Journalize this transaction.

1eSG_C12_0131792075.qxd 10/23/06 9:33 AM Page 418

Page 451: polpoa_sg

Demo Doc 2 | Chapter 12 419

Demo Doc 2Bonds Payable (Effective-Interest Amortization)Learning Objectives 1–3, 5

Red Co. issued $2,000 maturity (face) value of bonds payable for $1,930 cash onJanuary 1, 2008. The bonds had a stated rate of 8%, but the market rate was 10%.Interest is paid semiannually, and the bonds are due in two years.

Red uses the effective-interest method of amortization.

Requirements

1. Are these bonds issued at a discount or premium? How do you know?

2. Journalize Red’s issuance of the bonds on January 1, 2008.

3. Prepare Red’s effective-interest amortization table for the entire life of thebonds payable.

4. Journalize Red’s first two interest payments on June 30, 2008, andDecember 31, 2008.

5. On January 2, 2009, 20% of the bonds were converted to 120 commonshares with a $2 par value per share. Journalize this transaction.

6. On January 3, 2009, Red purchased the remaining bonds from themarketplace for $1,800 cash.The bonds were immediately retired. Journalizethis transaction.

7. How do interest payments on bonds differ from mortgages?

1eSG_C12_0131792075.qxd 10/23/06 9:33 AM Page 419

Page 452: polpoa_sg

420 Chapter 12 | Demo Doc 2 Solutions

3 Account for bondissuance and bondinterest

Part 1 Part 2 Part 3 Part 4 Part 5 Part 6 Part 7 Part 8 Part 11Part 10Part 9Demo Doc

Complete

Demo Doc 2 SolutionsRequirement 1

Are these bonds issued at a discount or premium? How do you know?

Part 1 Part 2 Part 3 Part 4 Part 5 Part 6 Part 7 Part 8 Part 11Part 10Part 9Demo Doc

Complete

Part 1 Part 2 Part 3 Part 4 Part 5 Part 6 Part 7 Part 8 Part 11Part 10Part 9Demo Doc

Complete

These bonds were issued at a discount. We know because the cash received forthe bonds is less than the maturity value and because the stated rate is less thanthe market rate.

Requirement 2

Journalize Red’s issuance of the bonds on January 1, 2008.

Cash increases (a debit) by $1,930.Bonds Payable increases (a credit) by the bonds’ maturity or face value of $2,000.The difference between these two amounts is balanced to the Discount on

Bonds Payable. The balancing amount is a debit of:

$2,000 – $1,930 ! $70

Journal Entry:

Jan. 1, 2008 Cash 1,930 Discount on Bonds Payable (to balance) 70 Bonds Payable (principal amount) 2,000

PostDate Accounts Ref. Dr. Cr.

Interest Payment

The cash interest payment is the same every interest period and is calculated thesame as under the straight-line method of amortization:

Cash Interest PaidMaturity Value Stated Rat

!" ee

of Interest Payments per YearNumber

3 Account for bondissuance and bondinterest

Requirement 3

Prepare Red’s effective-interest amortization table for the entire life of thebonds payable.

1eSG_C12_0131792075.qxd 10/23/06 9:33 AM Page 420

Page 453: polpoa_sg

Demo Doc 2 Solutions | Chapter 12 421

The question states that the bonds are semiannual: that is, they pay interesttwice per year.

So every line in the Interest Payment column will show $80.

Interest Expense

Cash Interest Paid!"

!

$ , %2 000 82

$80

Part 1 Part 2 Part 3 Part 4 Part 5 Part 6 Part 7 Part 8 Part 11Part 10Part 9Demo Doc

Complete

Part 1 Part 2 Part 3 Part 4 Part 5 Part 6 Part 7 Part 8 Part 11Part 10Part 9Demo Doc

Complete

Part 1 Part 2 Part 3 Part 4 Part 5 Part 6 Part 7 Part 8 Part 11Part 10Part 9Demo Doc

Complete

Under the effective-interest method, the interest expense is calculated as:

The carrying value changes every time interest expense is recorded and the dis-count is amortized, which means the interest expense changes as well.

The interest expense is calculated as:

June 30, 2008 $1,930 ! 10%/2 ! $96Dec. 31, 2008 $1,946 ! 10%/2 ! $97June 30, 2009 $1,963 ! 10%/2 ! $98Dec. 31, 2009 $1,981 ! 10%/2 ! $99

Discount Amortization

Interest ExpenseCarrying Value Market Rate

N!

"

uumber of Interest Payments per Year

Under the effective-interest method, the discount (or premium) is the balancingamount. It is the difference between the cash interest payment and the interestexpense.

The discount amortization is calculated as:

June 30, 2008 $96 – $80 ! $16Dec. 31, 2008 $97 – $80 ! $17June 30, 2009 $98 – $80 ! $18Dec. 31, 2009 $99 – $80 ! $19

Discount Balance

The discount balance is the balance from the Discount on Bonds Payable T-account.By putting the discount amortization into the account, we can calculate the bal-ance after each interest transaction.

1eSG_C12_0131792075.qxd 10/23/06 9:33 AM Page 421

Page 454: polpoa_sg

422 Chapter 12 | Demo Doc 2 Solutions

Bond Carrying Value

Part 1 Part 2 Part 3 Part 4 Part 5 Part 6 Part 7 Part 8 Part 11Part 10Part 9Demo Doc

Complete

Discount on Bonds Payable

1/1/08 706/30/08 16

Bal. 5412/31/08 17

Bal. 376/30/09 18

Bal. 1912/31/09 19

Bal. 0

The carrying value is the maturity value of the bonds minus the discount (or plusthe premium).

The bonds’ carrying value is calculated as the maturity value less the dis-count. After each interest period, this amount is:

Jan. 1, 2008 $2,000 – $70 ! $1,930June 30, 2008 $2,000 – $54 ! $1,946Dec. 31, 2008 $2,000 – $37 ! $1,963

June 30, 2009 $2,000 – $19 ! $1,981Dec. 31, 2009 $2,000 – $ 0 ! $2,000

Combining all this information into the amortization table, we get:

Bonds’ Interest Interest Discount Discount Carrying

Date Payment Expense Amortization Balance Value

Jan. 1,2008 $70 $1,930

June 30,2008 $80 $96 $16 54 1,946

Dec. 31,2008 80 97 17 37 1,963

June 30,2009 80 98 18 19 1,981

Dec. 31,2009 80 99 19 0 2,000

1eSG_C12_0131792075.qxd 10/23/06 9:33 AM Page 422

Page 455: polpoa_sg

Demo Doc 2 Solutions | Chapter 12 423

Part 1 Part 2 Part 3 Part 4 Part 5 Part 6 Part 7 Part 8 Part 11Part 10Part 9Demo Doc

Complete

June 30, 2008

Interest Expense increases (a debit) by $96, as calculated in the table inRequirement 3.

Cash decreases (a credit) by the cash interest paid of $80, as calculated inRequirement 3.

The difference between these two amounts is balanced to Discount on BondsPayable. The balancing amount is a credit of:

$96 – $80 ! $16

This is the discount amortization shown in Requirement 3.

Journal Entry:

June 30, 2008 Interest Expense ($1,930 ! 10%/2) 96 Discount on Bonds Payable (to balance) 16 Cash ($2,000 ! 8%/2) 80

PostDate Accounts Ref. Dr. Cr.

3 Account for bondissuance and bondinterest

December 31, 2008

Interest Expense increases (a debit) by $97, as calculated in the table inRequirement 3.

Cash decreases (a credit) by the cash interest paid of $80, as calculated inRequirement 3.

The difference between these two amounts is balanced to Discount on BondsPayable. The balancing amount is a credit of:

$97 – $80 ! $17

This is the discount amortization shown in Requirement 3.

Journal Entry:

Dec. 31, 2008 Interest Expense ($1,946 ! 10%/2) 97 Discount on Bonds Payable (to balance) 17 Cash ($2,000 ! 8%/2) 80

PostDate Accounts Ref. Dr. Cr.

Requirement 4

Journalize Red’s first two interest payments on June 30, 2008, and December 31,2008.

1eSG_C12_0131792075.qxd 10/23/06 9:33 AM Page 423

Page 456: polpoa_sg

424 Chapter 12 | Demo Doc 2 Solutions

If 20% of the bonds are converted, then 20% of the maturity value is converted:

20% ! $2,000 " $400

Also, 20% of the discount is converted:

20% ! $37 " $7

The bonds will not exist after the conversion, so the Bonds Payable accountdecreases (a debit) by $400. The Discount on Bonds Payable also decreases (acredit) by $7.

New common stock is issued for the conversion, so Common Stock increases(a credit) by the par value of the new shares:

120 ! $2 " $240

The remainder is balanced to Paid-In Capital in Excess of Par. The balancingamount is:

$400 – $7 – $240 " $153

4 Account for bondrepayment

Discount on Bonds Payable Bonds Payable

Bal. 0 Bal. 0Jan. 1 70 Jan. 1 2,000

June 30 16 Jan. 2 400Dec. 31 17Jan. 2 7 Bal. 1,600

Bal. 30

4 Account for bondrepayment

Part 1 Part 2 Part 3 Part 4 Part 5 Part 6 Part 7 Part 8 Part 11Part 10Part 9Demo DocComplete

Journal Entry:

Jan. 2, 2009 Bonds Payable ($2,000 ! 20%) 400 Discount on Bonds Payable ($37 ! 20%) 7 Common Stock (120 ! $2) 240 Paid-In Capital in Excess of Par (to balance) 153

PostDate Accounts Ref. Dr. Cr.

Requirement 6

On January 3, 2009, Red purchased the remaining bonds from the marketplacefor $1,800 cash.The bonds were immediately retired. Journalize this transaction.

Part 1 Part 2 Part 3 Part 4 Part 5 Part 6 Part 7 Part 8 Part 11Part 10Part 9Demo DocComplete

Because 20% of the bonds were converted to common stock, 100% – 20% = 80%remain. These bonds will be retired.

Requirement 5

On January 2, 2009, 20% of the bonds were converted to 120 common shareswith a $2 par value per share. Journalize this transaction.

1eSG_C12_0131792075.qxd 11/22/06 10:32 AM Page 424

Page 457: polpoa_sg

Demo Doc 2 Solutions | Chapter 12 425

If 80% of the bonds are retired, then 80% of the maturity value is retired:

80% " $2,000 ! $1,600

Also, 80% of the discount is retired:

80% " $37 ! $30

The bonds will not exist after retirement, so the Bonds Payable account decreases(a debit) by $1,600. The Discount also decreases (a credit) by $30.

Cash decreases (a credit) by $1,800.The remainder is balanced to a gain or loss. In this case, the balancing

amount is a debit, which means a loss of:

$1,800 $ $30 # $1,600 ! $230

1 Describe mortgagesand leases, and recordmortgage and leasetransactions

2 Describe bondspayable

Part 1 Part 2 Part 3 Part 4 Part 5 Part 6 Part 7 Part 8 Part 11Part 10Part 9Demo Doc

Complete

Journal Entry:

Jan. 3, 2009 Bonds Payable ($2,000 ! 80%) 1,600 Loss on Retirement (to balance) 230 Discount on Bonds Payable ($37 ! 80%) 30 Cash 1,800

PostDate Accounts Ref. Dr. Cr.

Although both bonds and mortgages have set cash payments that are the sameevery period, the nature of these payments differ. Bonds have their principal paidback as one lump sum at their maturity.

So, for the entire life of the bond, the (present value of the) debt is a liabilityfor the company. Because this debt is outstanding for the bond’s entire life, inter-est expense is high each period and remains high over the bond’s life.

Mortgages pay off the principal and interest with each payment. Unlikebonds, the debt and the interest expense are both decreasing over the life of themortgage.

With bonds, large principal payments can lead to bankruptcy if the companyis unable to pay off the debt when the bonds mature. This problem is much lesslikely to happen with mortgage payments.

Part 1 Part 2 Part 3 Part 4 Part 5 Part 6 Part 7 Part 8 Part 11Part 10Part 9Demo Doc

Complete

Requirement 7

How do interest payments on bonds differ from mortgages?

1eSG_C12_0131792075.qxd 10/23/06 9:33 AM Page 425

Page 458: polpoa_sg

Quick Practice QuestionsTrue/False

_____ 1. The journal entry to record selling $200,000 face value bonds at 98 willinvolve a credit to Bonds Payable for $196,000.

_____ 2. When a bond is issued at a discount, the discount has the effect of rais-ing the interest expense on the bonds to the market rate of interest.

_____ 3. The carrying value of bonds will decrease each interest period if thebonds were sold at a discount.

_____ 4. An amortization schedule provides the information to divide mortgagepayments between the interest expense and the decrease in the princi-pal balance.

_____ 5. Callable bonds give the issuer the benefit of being able to take advan-tage of low interest rates by paying off bonds when it is favorable to doso.

_____ 6. If a lease transfers title of the leased asset to the lessee at the end ofthe lease term, it is considered an operating lease.

_____ 7. When reporting a mortgage on the balance sheet, the portion maturingwithin one year is shown as a current liability.

_____ 8. Positive financial leverage can result from using funds borrowed at onerate to earn profit at a lower rate.

_____ 9. If bonds with a carrying value of $540,000 are retired early and were pur-chased at the market price of $550,000, a $10,000 Gain on Retirement ofBonds Payable should be recorded.

_____10. Bondholders are creditors of a corporation.

426 Chapter 12 | Quick Practice Questions

1eSG_C12_0131792075.qxd 10/23/06 9:33 AM Page 426

Page 459: polpoa_sg

Multiple Choice

1. On January 2, 2008, Lot Corporation issues $200,000 par value,6% bonds for $196,000. What can be concluded about theeffective (market) rate of interest?a. It is less than 6%b. It is more than 6%c. It is equal to 6%d. It is impossible to determine from the given data

2. Dalton Corporation issues 50, $1,000 par value, 10% bonds at102.5. The journal entry includes which of the following?a. A debit to Cash for $50,000b. A credit to Premium on Bonds Payable for $1,250c. A debit to Discount on Bonds Payable for $1,250d. A credit to Bonds Payable for $51,250

3. Jones Corporation issues $400,000, 10%, 5-year bonds at 103.What is the total interest expense over the life of the bonds?a. $40,000b. $188,000c. $200,000d. $212,000

4. What are bonds issued on the general credit of the issuingcorporation called?a. Serial bondsb. Term bondsc. Debenture bondsd. Convertible bonds

5. What are bonds called when the maturities are spread overseveral dates?a. Term bondsb. Debenture bondsc. Serial bondsd. Callable bonds

6. What is the interest rate specified in the bond indenture called?a. Stated rateb. Discount ratec. Yield rated. Effective rate

7. Which of the following statements about the discount on bondspayable is correct?a. It is added to bonds payable on the balance sheet.b. It is a contra-asset.c. It is amortized over the life of the bonds.d. Both b and c are correct

Quick Practice Questions | Chapter 12 427

1eSG_C12_0131792075.qxd 10/23/06 10:13 AM Page 427

Page 460: polpoa_sg

8. A corporation may choose to purchase an asset rather thanlease it for which of the following reasons?a. Save cashb. Adapt to changes in technology easilyc. Obtain easier financingd. Avoid recording the expense for lease payments

9. A bond issued with a face value of $200,000 and a carryingamount of $195,500 is called in at 98 1/2 and retired. What is thegain or loss on this transaction?a. $1,500 lossb. $1,500 gainc. $3,000 lossd. $3,000 gain

10. All except which of the following is an advantage of issuingstock?a. The corporation is not required to pay dividendsb. The corporation is not required to repay the amount paid in by the

stockholdersc. Positive financial leverage exists when issuing stockd. Dividends are not recorded as an expense

428 Chapter 12 | Quick Practice Questions

1eSG_C12_0131792075.qxd 10/23/06 10:13 AM Page 428

Page 461: polpoa_sg

Quick Exercises

12-1. For each of the following independent situations, state whetherthe bonds were issued at a premium, at a discount, or at par.

a. Bonds with a maturity value of $50,000 were issued for $53,000. ___________

b. Bonds with a stated rate of 8% were issued to yield 7.5%. ___________

c. Bonds with a maturity value of $75,000 were issued for $75,000. ___________

d. Bonds with a stated rate of 8.25% were issued to yield 8.75%. ___________

e. Bonds with a maturity value of $110,000 were issued for $106,000. ___________

12-2. Fox Corporation issued 10-year, 10%, $1,000,000 bonds onJanuary 1, 2008. The bonds pay interest every June 30 andDecember 31. The bonds were issued for $1,065,000. FoxCorporation uses straight-line amortization for any discountor premium amortization.

a. Journalize the bonds’ issuance on January 1, 2008.b. Journalize the interest payment and amortize the premium or dis-

count on June 30, 2008.

Quick Practice Questions | Chapter 12 429

Journal Entry:

PostDate Accounts Ref. Dr. Cr.

Journal Entry:

PostDate Accounts Ref. Dr. Cr.

c. What is the carrying value of the bonds on June 30, 2008?

1eSG_C12_0131792075.qxd 10/23/06 9:33 AM Page 429

Page 462: polpoa_sg

12-4. Ryan Company issues a $300,000, 7%, 15-year mortgage onJanuary 1, 2008 to purchase a building.

a. Complete the following amortization schedule for Ryan Company.

Semiannual Cash Interest Decrease in PrincipalInterest Period Payment Expense Principal Balance

Jan. 1, 2008 $300,000

Jan. 31, 2008 2,697 1,750 947 299,053

Feb. 28, 2008 2,697 1,745 952 298,101

Mar. 31 , 2008

Apr. 30, 2008

430 Chapter 12 | Quick Practice Questions

Journal Entry:

PostDate Accounts Ref. Dr. Cr.

Journal Entry:

PostDate Accounts Ref. Dr. Cr.

Journal Entry:

PostDate Accounts Ref. Dr. Cr.

12-3. On April 1, 2008, Needy Corporation issued $3,000,000 of 8%,10-year bonds dated April 1, 2008, with interest paymentsmade each October 1 and April 1. The bonds are issued at 95.Needy Corporation amortizes any premium or discount usingthe straight-line method.

a. Journalize the issuance of bonds on April 1, 2008.

b. Journalize the payment of interest and the amortization of any dis-count or premium on October 1, 2008.

c. Journalize the accrued interest and the amortization of any pre-mium or discount on December 31, 2008.

1eSG_C12_0131792075.qxd 10/23/06 9:33 AM Page 430

Page 463: polpoa_sg

Quick Practice Questions | Chapter 12 431

b. Journalize the issuance of mortgage on January 1, 2008, and thefirst cash payment on January 31, 2008.

Journal Entry:

PostDate Accounts Ref. Dr. Cr.

Journal Entry:

PostDate Accounts Ref. Dr. Cr.

Journal Entry:

PostDate Accounts Ref. Dr. Cr.

12-5. Apex Corporation issued $500,000, 10%, 10-year callable bondsat 93 on January 2, 2008. The bonds are callable at 102 any-time after January 2, 2010. On December 31, 2011, ApexCorporation calls the entire issuance.

a. Compute the balance in the Premium or Discount account onDecember 31, 2011. Apex Corporation uses the straight-linemethod of amortization.

b. Prepare the entry to call the bonds on December 31, 2011.

1eSG_C12_0131792075.qxd 10/23/06 9:33 AM Page 431

Page 464: polpoa_sg

432 Chapter 12 | Do It Yourself! Question 1

Do It Yourself! Question 1

Bonds Payable (Straight-Line Amortization)Circle Inc., issued $20,000 maturity value of bonds payable for $19,337 cash onJanuary 1, 2008. The bonds had a stated rate of 14%, but the market rate was16%. Interest is paid semiannually and the bonds are due in 2 years.

Circle uses the straight-line method of amortization.

Requirements

1. Are these bonds issued at a discount or premium?

2. Journalize Circle’s issuance of the bonds on January 1, 2008.

3 Account for bondissuance and bondinterest

3 Account for bondissuance and bondinterest

Journal Entry:

PostDate Accounts Ref. Dr. Cr.

3. Journalize Circle’s first two interest payments on June 30, 2008, andDecember 31, 2008.

Journal Entry:

PostDate Accounts Ref. Dr. Cr.

Journal Entry:

PostDate Accounts Ref. Dr. Cr.

1eSG_C12_0131792075.qxd 10/23/06 9:33 AM Page 432

Page 465: polpoa_sg

4. Show how the bonds would appear on Circle’s December 31, 2008, balancesheet.

Do It Yourself! Question 1 | Chapter 12 433

5 Report long-term liabil-ities on the balancesheet

5. On January 2, 2009, 25% of the bonds were converted to 9,000 commonshares with a $0.50 par value per share. Journalize this transaction.

4 Account for bondrepayment

4 Account for bondrepayment

Journal Entry:

PostDate Accounts Ref. Dr. Cr.

6. On January 3, 2009, Circle purchased the remaining bonds from themarketplace for $14,000 cash. The bonds were immediately retired.Journalize this transaction.

Journal Entry:

PostDate Accounts Ref. Dr. Cr.

1eSG_C12_0131792075.qxd 10/23/06 9:33 AM Page 433

Page 466: polpoa_sg

434 Chapter 12 | Do It Yourself! Question 2

Do It Yourself! Question 2

Bonds Payable (Effective-Interest Amortization)Square Corp. issued $10,000 maturity value of bonds payable for $10,346 cashon January 1, 2008. The bonds had a stated rate of 14% but the market rate was12%. Interest is paid semiannually and the bonds are due in 2 years.

Square Corp. uses the effective-interest method of amortization.

Requirements

1. Are these bonds issued at a discount or premium?

2. Journalize Square’s issuance of the bonds on January 1, 2008.

3 Account for bondissuance and bondinterest

3 Account for bondissuance and bondinterest

Journal Entry:

PostDate Accounts Ref. Dr. Cr.

3. Prepare Square’s effective interest amortization table for the entire life ofthe bonds payable.

4. Journalize Square’s first two interest payments on June 30, 2008, andDecember 31, 2008.

Journal Entry:

PostDate Accounts Ref. Dr. Cr.

3 Account for bondissuance and bondinterest

1eSG_C12_0131792075.qxd 10/23/06 9:33 AM Page 434

Page 467: polpoa_sg

5. On January 2, 2009, 50% of the bonds were converted to 3,000 commonshares with a $1.50 par value per share. Journalize this transaction.

Do It Yourself! Question 2 | Chapter 12 435

Journal Entry:

PostDate Accounts Ref. Dr. Cr.

Journal Entry:

PostDate Accounts Ref. Dr. Cr.

4 Account for bondrepayment

6. On January 3, 2009, Square purchased the remaining bonds from themarketplace for $6,000 cash.The bonds were immediately retired. Journalizethis transaction.

Journal Entry:

PostDate Accounts Ref. Dr. Cr.

4 Account for bondrepayment

1eSG_C12_0131792075.qxd 10/23/06 9:33 AM Page 435

Page 468: polpoa_sg

Quick Practice SolutionsTrue/False

F 1. The journal entry to record selling $200,000 face value bonds at 98 will involve a credit to Bonds Payable for $196,000.False–The journal entry to record selling $200,000 face value bondsat 98 will involve a credit to Bonds Payable for $200,000 (p. 635)

T 2. When a bond is issued at a discount, the discount has the effect of raising the interest expense on the bonds to the market rate of interest. (p. 636)

F 3. The carrying value of bonds will decrease each interest period if the bonds were sold at a discount.False–The carrying value of bonds will increase each interest periodif the bonds were sold at a discount. (p. 637)

T 4. An amortization schedule provides the information to divide mortgage payments between the interest expense and the decrease in the principal balance. (p. 626)

T 5. Callable bonds give the issuer the benefit of being able to take advantage of low interest rates by paying off bonds when it is favorable to do so. (p. 631)

F 6. If a lease transfers title of the leased asset to the lessee at the end of the lease term, it is considered an operating lease.False–If a lease transfers title of the leased asset to the lessee at the end of the lease term, it is considered a capital lease. (p. 629)

T 7. When reporting a mortgage on the balance sheet, the portion maturing within one year is shown as a current liability. (p. 643)

F 8. Positive financial leverage can result from using funds borrowed at one rate to earn profit at a lower rate.False–Positive financial leverage can result from using funds borrowed at one rate to earn a higher rate. (p. 630)

F 9. If bonds with a carrying value of $540,000 are retired early and were purchased at the market price of $550,000, a $10,000 Gain on Retirement of Bonds Payable should be recorded.False–If bonds with a carrying value of $540,000 are retired early and were purchased at the market price of $550,000, a $10,000 Losson Retirement of Bonds Payable should be recorded. (p. 642)

T 10. Bondholders are creditors of a corporation. (p. 630)

436 Chapter 12 | Quick Practice Solutions

1eSG_C12_0131792075.qxd 10/23/06 9:33 AM Page 436

Page 469: polpoa_sg

Multiple Choice

1. On January 2, 2008, Lot Corporation issues $200,000 face value,6% bonds for $196,000. What can be concluded about theeffective (market) rate of interest? (p. 633)a. It is less than 6%b. It is more than 6%c. It is equal to 6%d. It is impossible to determine from the given data

2. Dalton Corporation issues 50, $1,000 par value, 10% bonds at102.5. The journal entry includes which of the following? (p. 638)a. A debit to Cash for $50,000b. A credit to Premium on Bonds Payable for $1,250c. A debit to Discount on Bonds Payable for $1,250d. A credit to Bonds Payable for $51,250

3. Jones Corporation issues $400,000, 10%, 5-year bonds at 103.What is the total interest expense over the life of the bonds? (p. 635)a. $40,000b. $188,000c. $200,000d. $212,000

4. What are bonds issued on the general credit of the issuingcorporation called? (p. 631)a. Serial bondsb. Term bondsc. Debenture bondsd. Convertible bonds

5. What are bonds called when the maturities are spread overseveral dates? (p. 631)a. Term bondsb. Debenture bondsc. Serial bondsd. Callable bonds

6. What is the interest rate specified in the bond indenture called? (p. 633)a. Stated rateb. Discount ratec. Yield rated. Effective rate

7. Which of the following statements about the discount on bondspayable is correct? (p. 636)a. It is added to bonds payable on the balance sheetb. It is a contra-assetc. It is amortized over the life of the bondsd. Both b and c are correct

Quick Practice Solutions | Chapter 12 437

1eSG_C12_0131792075.qxd 10/23/06 10:14 AM Page 437

Page 470: polpoa_sg

8. A corporation may choose to purchase an asset rather thanlease it for which of the following reasons? (p. 628)a. Save cashb. Adapt to changes in technology easilyc. Obtain easier financingd. Avoid recording the expense for lease payments

9. A bond issued with a face value of $200,000 and a carryingamount of $195,500 is called in at 981/2 and retired. What is thegain or loss on this transaction? (p. 636)a. $1,500 lossb. $1,500 gainc. $3,000 lossd. $3,000 gain

10. All except which of the following is an advantage of issuingstock? (p. 630)a. The corporation is not required to pay dividendsb. The corporation is not required to repay the amount paid in by the

stockholdersc. Positive financial leverage exists when issuing stockd. Dividends are not recorded as an expense

438 Chapter 12 | Quick Practice Solutions

1eSG_C12_0131792075.qxd 10/23/06 10:22 AM Page 438

Page 471: polpoa_sg

Quick Exercise

12-1. For each of the following independent situations, statewhether the bonds were issued at a premium, at a discount, orat par. (pp. 635–640)

a. Bonds with a maturity value of $50,000 were issued for $53,000.Premium

b. Bonds with a stated rate of 8% were issued to yield 7.5%.Premium

c. Bonds with a maturity value of $75,000 were issued for $75,000.Par

d. Bonds with a stated rate of 8.25% were issued to yield 8.75%.Discount

e. Bonds with a maturity value of $110,000 were issued for $106,000.Discount

12-2. Fox Corporation issued 10-year, 10%, $1,000,000 bonds onJanuary 1, 2008. The bonds pay interest every June 30 andDecember 31. The bonds were issued for $1,065,000. FoxCorporation uses straight-line amortization for any discountor premium amortization. (pp. 638–639)

a. Journalize the bonds’ issuance on January 1, 2008.b. Journalize the interest payment and amortize the premium or dis-

count on June 30, 2008.

Quick Practice Solutions | Chapter 12 439

c. What is the carrying value of the bonds on June 30, 2008?Carrying Value = $1,000,000 + ($65,000 " $3,250) = $1,061,750

12-3. On April 1, 2008, Needy Corporation issued $3,000,000 of 8%,10-year bonds with interest payments made each October 1and April 1. The bonds are issued at 95. Needy Corporationamortizes any premium or discount using the straight-linemethod. (pp. 635–637)

a. Journalize the issuance of bonds on April 1, 2008.$3,000,000 ! 0.95 = $2,850,000$3,000,000 – $2,850,000 = $150,000

Journal Entry:

a. Cash 1,065,000 Jan. 1 Bonds Payable 1,000,000 Premium on Bonds Payable 65,000

b. Interest Expense 46,750 June 30 Premium on Bonds Payable 3,250 Cash 50,000

PostDate Accounts Ref. Dr. Cr.

1eSG_C12_0131792075.qxd 10/23/06 9:33 AM Page 439

Page 472: polpoa_sg

440 Chapter 12 | Quick Practice Solutions

b. Journalize the payment of interest and the amortization of any dis-count or premium on October 1, 2008.$3,000,000 ! 0.08 ! 6/12 = $120,000$150,000 ! 6/120 months = $7,500

c. Journalize the accrued interest and the amortization of any pre-mium or discount on December 31, 2008.$3,000,000 ! 0.08 ! 3/12 = $60,000$150,000 ! 3/120 months = $3,750

Journal Entry:

a. Cash 2,850,000 Apr. 1 Discount on Bonds Payable 150,000 Bonds Payable 3,000,000

b. Interest Expense 127,500 Oct. 1 Discount on Bonds Payable 7,500 Cash 120,000

c. Interest Expense 63,750 Dec. 31 Discount on Bonds Payable 3,750 Interest Payable 60,000

PostDate Accounts Ref. Dr. Cr.

12-4. Ryan Company issues a $300,000, 7%, 15-year mortgage onJanuary 1, 2008 to purchase a building. (pp. 626–628)

a. Complete the following amortization schedule for Ryan Company.

Semiannual Cash Interest Decrease in PrincipalInterest Period Payment Expense Principal Balance

Jan. 1, 2008 $300,000

Jan. 31, 2008 2,697 1,750 947 299,053

Feb. 28, 2008 2,697 1,745 952 298,101

Mar. 31, 2008 2,697 1,739 958 297,143

Apr. 30, 2008 2,697 1,733 964 296,179

1eSG_C12_0131792075.qxd 10/23/06 9:33 AM Page 440

Page 473: polpoa_sg

Quick Practice Solutions | Chapter 12 441

12-5. Apex Corporation issued $500,000, 10%, 10-year callable bondsat 93 on January 2, 2008. The bonds are callable at 102 any-time after January 2, 2010. On December 31, 2011, ApexCorporation calls the entire issuance. (pp. 641–642)

a. Compute the balance in the Premium or Discount account onDecember 31, 2011. Apex Corporation uses the straight-linemethod of amortization.

$500,000 ! 0.93 = $465,000$500,000 – $465,000 = $35,000 discount$35,000/10 years = $3,500 annual amortization of discount$3,500 ! 4 years = $14,000 amortized discount as of

December 31, 2011$35,000 – $14,000 = $21,000 unamortized discount as of

December 31, 2011

b. Prepare the entry to call the bonds on December 31, 2011.

Journal Entry:

Jan. 1, 2008 Buildings 300,000 Mortgage Notes Payable 300,000 Record purchase of building and insurance of mortgage note.

Jan. 31, 2008 Interest Expense 1,750 Mortgage Notes Payable 947 Cash 2,697 Make monthly mortgage payment.

PostDate Accounts Ref. Dr. Cr.

Journal Entry:

Dec. 31, 2011 Bonds Payable 500,000 Loss on Retirement of Bonds Payable [$510,000 – ($500,000 – $21,000)] 31,000 Discount on Bonds Payable 21,000 Cash ($500,000 3 102%) 510,000

PostDate Accounts Ref. Dr. Cr.

b. Journalize the issuance of mortgage on January 1, 2008, and thefirst cash payment on January 31, 2008.

1eSG_C12_0131792075.qxd 10/23/06 9:33 AM Page 441

Page 474: polpoa_sg

442 Chapter 12 | Do It Yourself! Question 1 Solutions

Do It Yourself! Question 1 Solutions

Bonds Payable (Straight-Line Amortization)

Requirement 1

Are these bonds issued at a discount or premium?

These bonds were issued at a discount.

Requirement 2

Journalize Circle’s issuance of the bonds on January 1, 2008.

Journal Entry:

Jan. 1, 2008 Cash 19,337 Discount on Bonds Payable (to balance) 663 Bonds Payable (principle amount) 20,000

PostDate Accounts Ref. Dr. Cr.

Requirement 3

Journalize Circle’s first two interest payments on June 30, 2008, and December 31,2008.

The entries for both June 30, 2008, and December 31, 2008, would be the same:

Journal Entry:

Interest Expense (to balance) 1,566 Cash ($20,000 ! 14% ! 6/12) 1,400 Discount on Bonds Payable ($663/4) 166

PostDate Accounts Ref. Dr. Cr.

Requirement 4Show how the bonds would appear on Circle’s December 31, 2008, balancesheet.

Long-Term Liabilities:

Bonds Payable $20,000

Less: Discount on Bonds Payable (331) $19,669

1eSG_C12_0131792075.qxd 10/23/06 9:33 AM Page 442

Page 475: polpoa_sg

Do It Yourself! Question 1 Solutions | Chapter 12 443

Discount on Bonds Payable

Bal. 0Jan. 1 663

June 30 166Dec. 31 166

Bal. 331

Journal Entry:

Jan. 2, 2009 Bonds Payable ($20,000 ! 25%) 5,000 Discount on Bonds Payable ($133.50! 25%) 83 Common Stock (9,000 ! $0.50) 4,500 Paid-In Capital in Excess of Par (to balance) 417

PostDate Accounts Ref. Dr. Cr.

Journal Entry:

Jan. 2, 2009 Bonds Payable ($20,000 ! 75%) 15,000 Gain on Retirement (to balance) 751 Discount on Bonds Payable ($331.50 ! 75%) 249 Cash 14,000

PostDate Accounts Ref. Dr. Cr.

Requirement 6

On January 3, 2009, Circle purchased the remaining bonds from the marketplacefor $14,000 cash.The bonds were immediately retired. Journalize this transaction.

Requirement 5

On January 2, 2009, 25% of the bonds were converted to 9,000 common shareswith a $0.50 par value per share. Journalize this transaction.

Immediately before this transaction, the discount had a balance of $331:

1eSG_C12_0131792075.qxd 10/23/06 9:33 AM Page 443

Page 476: polpoa_sg

444 Chapter 12 | Do It Yourself! Question 2 Solutions

Do It Yourself! Question 2 Solutions

Bonds Payable (Effective-Interest Amortization)

Requirement 1

Are these bonds issued at a discount or premium?

These bonds were issued at a premium.

Requirement 2

Journalize Square’s issuance of the bonds on January 1, 2008.

Journal Entry:

Jan. 1, 2008 Cash 10,346 Discount on Bonds Payable (to balance) 346 Bonds Payable (principle amount) 10,000

PostDate Accounts Ref. Dr. Cr.

Requirement 3

Prepare Square’s effective interest amortization table for the entire life of thebonds payable.

Bonds’ Interest Interest Premium Premium Carrying

Date Payment Expense Amortization Balance Value

Jan. 1,2008 $346 $10,346

June 30,2008 $700 $621 $79 267 10,267

Dec. 31,2008 700 617 83 184 10,184

June 30,2009 700 611 89 95 10,095

Dec. 31,2009 700 605 95 0 10,000

1eSG_C12_0131792075.qxd 10/23/06 9:33 AM Page 444

Page 477: polpoa_sg

Do It Yourself! Question 2 Solutions | Chapter 12 445

Journal Entry:

June 30, 2008 Interest Expense ($10,267 ! 12%/2) 617 Premium on Bonds Payable (to balance) 83 Cash ($10,000 ! 14%/2) 700

PostDate Accounts Ref. Dr. Cr.

Journal Entry:

Dec. 31, 2008 Interest Expense ($10,267 ! 12%/2) 617 Premium on Bonds Payable (to balance) 83 Cash ($10,000 ! 14%/2) 700

PostDate Accounts Ref. Dr. Cr.

Journal Entry:

Jan. 2, 2009 Bonds Payable ($10,000 ! 50%) 5,000 Premium on Bonds Payable ($184 ! 50%) 92 Common Stock (3,000 ! $1.50) 4,500 Paid-In Capital in Excess of Par (to balance) 592

PostDate Accounts Ref. Dr. Cr.

Journal Entry:

Jan. 3, 2009 Bonds Payable ($10,000 ! 50%) 5,000 Loss on Retirement of Bonds Payable (to balance) 908 Premium on Bonds Payable ($184 ! 50%) 92 Cash 6,000

PostDate Accounts Ref. Dr. Cr.

Requirement 5

On January 2, 2009, 50% of the bonds were converted to 3,000 common shareswith a $1.50 par value per share. Journalize this transaction.

Requirement 6

On January 3, 2009, Square purchased the remaining bonds from the marketplacefor $6,000 cash.The bonds were immediately retired. Journalize this transaction.

Requirement 4

Journalize Square’s first two interest payments on June 30, 2008, andDecember 31, 2008.

1eSG_C12_0131792075.qxd 10/23/06 9:33 AM Page 445

Page 478: polpoa_sg

446 Chapter 12 | The Power of Practice

The Power of PracticeFor more practice using the skills learned in this chapter, visit MyAccountingLab.There you will find algorithmically generated questions that are based on theseDemo Docs and your main textbook’s Review and Accounting Practice sections.

To go to MyAccountingLab and follow these steps:

1. Direct your URL to www.myaccountinglab.com.2. Log in using your name and password.3. Click the MyAccountingLab link.4. Click Study Plan in the left navigation bar.5. From the table of contents, select Chapter 12, Long-Term Liabilities.6. Click a link to work on the tutorial exercises.

1eSG_C12_0131792075.qxd 10/23/06 9:33 AM Page 446

Page 479: polpoa_sg

Statement of CashFlows13

WHAT YOU PROBABLY ALREADY KNOW

If you find yourself short of cash occasionally, it is not uncommon towonder where all the money has gone. You probably already knowthat you need to keep track of all cash received and spent for aperiod of time to find out the answer. Not only does keeping trackshow you the amount of money coming in and going out, but youwould also identify the source of the cash received and the use ofthe cash spent. Identifying the cash activities in your life helps youto predict your future cash flows based on past history, review thedecisions you made in your financial life that result in the creationand disbursement of cash, and assess your ability to meet futurefinancial obligations. The same issues are important to a business.In this chapter, we see how the statement of cash flows providesthis information for an entity.

Learning Objectives

Explain the purposes of the statement of cash flows and describeits elements.

The statement of cash flows shows the cash receipts and payments for aperiod of time. It helps to predict future cash flows, evaluate managementdecisions, and predict a company’s ability to pay dividends. See Exhibit 13-1(p. 690) to learn how the statement of cash flows fits in with the otherfinancial statements.

Distinguish between operating, investing, and financing cashflows.

The three sections of a statement of cash flows are operating activities,investing activities, and financing activities. Operating activities affectthe income statement and current assets and current liabilities on thebalance sheet. Investing activities affect long-term assets. Financingactivities affect long-term liabilities and owners’ equity. See Exhibit 13-2(p. 692) for the relationship between these activity categories and thebalance sheet classifications.

Prepare a statement of cash flows by the indirect method.

The indirect method reconciles the net income from the incomestatement to the cash flow from operating activities. The investing andfinancing sections of the statement of cash flows are the same under the

1

2

3

1eSG_C13_0131792075.qxd 10/23/06 9:35 AM Page 447

Page 480: polpoa_sg

448 Chapter 13 | Statement of Cash Flows

4

indirect and direct methods. See Exhibits 13-5 and 13-7 (pp. 698) to viewthe format of an indirect method statement of cash flows.

Prepare a statement of cash flows by the direct method.

The direct method lists the amount of cash receipts and cash paymentsfrom operating activities by major category. See Exhibits 13-13 and 13-14 (pp. 705–706) to view the structure of a direct method statement ofcash flows. Review Exhibit 13-15 (p. 706) to see how to calculate cashreceipts and payments under the direct method.

1eSG_C13_0131792075.qxd 10/23/06 9:35 AM Page 448

Page 481: polpoa_sg

Demo Doc 1 | Chapter 13 449

Sales revenue $3,400 Less cost of goods sold (1,750) Gross margin $1,650 Depreciation expense $ (110) Insurance expense (230) Other operating expenses (390) Gain on sale of furniture 80 Net income $1,000

TANKER, INC.Income Statement

Year Ended December 31, 2008

Statement of Cash Flows (Indirect Method)Learning Objectives 2, 3

Tanker, Inc., had the following information at December 31, 2008:

Demo Doc 1

Current: Cash Accounts receivable Inventory Prepaid insurance

Furniture Less: Accum. Depr.Net

Total assets

Current: Accounts payable Long-term notes payableTotal liabilities Common stock (no par)Retained earnings Less: Treasury stockTotal equityTotal liabilities and equity

$ 700300800120

1,500(400)

1,100

$3,020

$1,16042075090

1,400(475)925

$3,345

$(460)(120)

5030

$ 680

660$1,340

1,800880

(1,000)$1,680$3,020

$ 530

815$1,345

1,800200

0$2,000$3,345

$ 150

(155)

2008Assets Liabilities

Owners’ Equity

2007 Change 2008 2007 Change

TANKER, INC.Comparative Balance Sheet December 31, 2008 and 2007

Other important information includes the following:

• Every year, Tanker declares and pays cash dividends.

• During 2008, Tanker sold old furniture for $90 cash. Tanker alsobought new furniture by making a cash down payment and signing a$200 note payable.

• During 2008, Tanker repaid $500 of notes payable in cash, and bor-rowed new long-term notes payable for cash.

1eSG_C13_0131792075.qxd 10/23/06 9:35 AM Page 449

Page 482: polpoa_sg

450 Chapter 13 | Demo Doc 1

• During 2008, Tanker purchased treasury stock for cash. No treasurystock was sold.

Requirement

Prepare Tanker’s statement of cash flows for the year ended December 31,2008, using the indirect method.

1eSG_C13_0131792075.qxd 10/23/06 9:35 AM Page 450

Page 483: polpoa_sg

Demo Doc 1 Solution | Chapter 13 451

Demo Doc 1 SolutionRequirement

Prepare Tanker’s statement of cash flows for the year ended December 31,2008, using the indirect method.

Operating Activities

Part 1 Part 2 Part 3 Demo DocComplete

We first set up the statement of cash flows with the proper title, and then startwith operating activities.

Net Income

The first item is net income. Because net income is positive, it increases tothe Cash balance. Therefore, we add (that is, a positive number) $1,000 on ourcash-flow statement.

Depreciation Expense

Net income includes depreciation expense, which must be removed becauseit is a 100% noncash item. Remember, no cash was “spent” for depreciation, yet itwas still deducted to arrive at the net income number. Because depreciationexpense was subtracted to calculate net income, we add it back to remove it.

Gains and Losses

After depreciation, we must look for gains and losses on disposal of long-term assets. These are treated in a manner similar to the depreciation. No cashwas “earned” for the gain, yet it was still added to arrive at the net income num-ber. The gain on sale of furniture was added to calculate net income, so wesubtract it to remove it.

Accounts Receivable

After looking at net income and the depreciation and gain adjustments, weneed to incorporate the changes in current assets and current liabilities.

The increases and decreases in these accounts do not tell us whether to addor subtract these items on the cash-flow statement.

The first current asset (other than cash) is Accounts Receivable. On the bal-ance sheet we see:

Assets 2008 2007 Change

Current:

Cash $ 700 $ 1,160 $(460)

Accounts Receivable 300 420 (120)

We must add the $120 decrease in accounts receivable. We can reason outthis adjustment in two ways:

1. Accounts Receivable went down. Why? Tanker is collecting more ofthe cash that its customers owe. How does this collection affectCash? It increases Cash, therefore, we should add the number onthe cash-flow statement.

2 Distinguish betweenoperating, investing,and financing cashflows

3 Prepare a statement ofcash flows by the indi-rect method

1eSG_C13_0131792075.qxd 10/23/06 9:35 AM Page 451

Page 484: polpoa_sg

452 Chapter 13 | Demo Doc 1 Solution

2. Accounts Receivable went down. This decrease in an asset appearsas a credit. To balance out this credit would require a debit to Cash,which is an increase. If Cash increases, we should add the numberon the cash flow statement.

Notice that in both cases, we add or subtract on the cash-flow statementbecause of the item’s effect on cash flow. It doesn’t matter whether AccountsReceivable went up or down; what matters is how that change affects cash flow.

Inventory

Let’s try the two ways with the next current asset: inventory. On the balancesheet, we see:

Assets 2008 2007 Change

Current:

Cash $700 $1,160 $(460)

Accounts Receivable 300 420 (120)

Inventory 800 750 50

During the year, inventory increased by $50.

1. Why did Inventory increase? Tanker purchased inventory withcash. Therefore, this transaction has a negative effect on cash flow.

2. An increase in inventory means an increase in an asset, which is adebit. For this debit to be balanced out by cash, the Cash account iscredited, which is a negative effect on cash flow.

Prepaid Expenses

The last current asset is Prepaid Insurance. On the balance sheet, we see:

Assets 2008 2007 Change

Current:

Cash $700 $1,160 $(460)

Accounts Receivable 300 420 (120)

Inventory 800 750 50

Prepaid Insurance 120 90 30

During the year, Prepaid Insurance increased by $30.

1. Why did the Prepaid Insurance account increase? Tanker paid moreinsurance costs in advance, which caused a negative effect on cashflow.

2. An increase in Prepaid Insurance means an increase in an asset,which is a debit. If this debit is balanced out by cash, then the Cashaccount is credited, which is a negative effect on cash flow.

Accounts Payable

The last part of operating activities is to look at the changes in current liabilities.The only current liability in this question is Accounts Payable. On the balancesheet, we see:

1eSG_C13_0131792075.qxd 10/23/06 9:35 AM Page 452

Page 485: polpoa_sg

Liabilities 2008 2007 Change

Current:

Accounts Payable $ 680 $ 530 $150

During the year, Accounts Payable increased by $150.

1. Why did Accounts Payable increase? Tanker is not paying all of itsbills. If they are holding onto their cash by not paying bills, theresult is a positive effect on cash flow.

2. An increase in the Accounts Payable account causes an increase in aliability, which is a credit. If this credit is balanced out by cash, thenthe Cash account is debited, resulting in a positive effect on cash flow.

We total these numbers, and we are finished with operating activities.The completed operating activities section would appear as:

Operating Activities

Net income $1,000

Depreciation Expense 110

Gain on Sale of Furniture (80)

Decrease in Accounts Receivable 120

Increase in Inventory (50)

Increase in Prepaid Insurance (30)

Increase in Accounts Payable 150

Total Cash Flow Provided by Operating Activities $1,220

Investing Activities

Part 1 Part 2 Part 3 Demo DocComplete

Investing activities involve cash purchases and cash disposals of long-termassets. We need to know how much cash was paid to purchase new furniture, andhow much cash was received when Tanker sold some of the old furniture. Do wehave any of these numbers right away? Yes, we are told in the question thatTanker signed a $200 note payable to purchase new furniture. We also know thatthe old furniture was sold for $90 cash.

Before we do anything else, we should point out that the $200 note payableis a noncash transaction. Although we will need to use it in our analysis, it willnot appear on the main body of the cash-flow statement. Instead, it will appear ina note for noncash investing and financing activities:

Noncash investing and financing activities

Purchase of furniture with note payable $ 200

We need to calculate the cash Tanker paid to purchase new furniture. For thisanalysis, we need to look at the Furniture (Net) T-account:

2 Distinguish betweenoperating, investing,and financing cashflows

3 Prepare a statement ofcash flows by the indi-rect method

Furniture (Net)Bal. 12/31/07 925

increases decreasesBal. 12/31/08 1,100

1eSG_C13_0131792075.qxd 10/23/06 9:35 AM Page 453

Page 486: polpoa_sg

454 Chapter 13 | Demo Doc 1 Solution

We know that the Furniture (Net) account increased and decreased. Whatcaused that account to increase? Well, it would increase if Tanker bought newfurniture. So obviously the cash paid and the note signed for new furniture wentinto this account.

What would cause the Furniture (Net) account to decrease?If Tanker sold furniture, we would decrease the account, but it would

decrease by the book value (that is, the net amount) of the furniture sold.Remember, the book value is another term for net value. We are looking at netvalue in the T-account, so the Furniture (Net) account decreases by its net/bookvalue.

We know that some furniture was sold, so obviously this decrease occurred.We know that this furniture was sold for $90 cash, but this amount is not the

net book value (NBV) of the furniture sold. This amount is still unknown.However, we can calculate this amount using the gain/loss formula:

Gain or Loss on Sale of Fixed Assets ! Cash Received on Sale of Fixed Assets " NBV of Fixed Assets Sold

For this example, this formula becomes:

Gain on Sale of Furniture ! Cash Received on Sale of Furniture " NBV of Furniture Sold

Therefore,

$80 ! $90 " NBV of Furniture Sold

NBV of Furniture Sold ! $10

What else would decrease Furniture (Net)? Well, when Tanker takes depreciationexpense, don’t we decrease the net value of its assets? We know from the incomestatement that depreciation expense is $110. Let’s now put all of the numbers inand see what comes out:

Furniture (Net)

Bal. 12/31/07 925Cash purchases XNoncash purchases 200

NBV furniture sold 10Depreciation expense 110

Bal. 12/31/08 1,100

So

$925 # X # $200 " $10 " $110 ! $1,100

X ! Cash paid to purchase furniture ! $95

The following summary shows how we find missing information for long-termassets:

1. Set up a T-account for the net value of the asset.

2. Fill in as much information as you can in the T-account (such asbeginning and ending balances, depreciation expense, and pur-chases or net book value of disposals).

3. Solve for any missing information.

4. If more than one number is missing, or if the missing information isnot the number you need, use the gain/loss formula to calculate anyremaining information.

1eSG_C13_0131792075.qxd 10/23/06 9:35 AM Page 454

Page 487: polpoa_sg

Demo Doc 1 Solution | Chapter 13 455

Notes Payable

Bal. 12/31/07 815Note repayments 500

New cash notes XNew noncash notes 200Bal. 12/31/08 660

Now we can put our two numbers, $90 and $95, into the statement of cash flows.Cash purchases of equipment were $95. Did this amount cause Cash to increaseor decrease? Obviously it is a decrease because Tanker paid cash, so we subtractit. Cash received on sale of equipment is $90, which is an increase to Cash, so weadd it.

Remember that for investing activities, we cannot combine these two items.They must be listed separately because they are two separate transactions.

Totaling these numbers completes investing activities. The completedinvesting activities section would appear as:

Investing Activities

Cash paid to purchase new furniture $ (95)

Cash proceeds from sale of furniture 90

Total cash flow provided by investing activities $ (5)

Financing Activities

Part 1 Part 2 Part 3 Demo DocComplete

The financing activities section deals with long-term liabilities (debt) and equityaccounts. First, we will look at long-term liabilities.

Long–Term Liabilities

Tanker signed new notes payable (for which it received cash) and repaidsome other notes.

We need the cash numbers involved so that we can put them into the cash-flow statement. Do we have any of them immediately available to us?

Yes, we are told that Tanker repaid $500 of notes payable.We also know that Tanker took out a noncash note (to purchase furniture) of

$200. This noncash transaction has already been recorded in the note to the cashflow statement (discussed under investing activities).

2 Distinguish betweenoperating, investing,and financing cashflows

3 Prepare a statement ofcash flows by the indi-rect method

Notes Payable

Bal. 12/31/07 815decreases increases

Bal. 12/31/08 660

Now let’s analyze the Notes Payable T-account:What would cause this account to increase? Well, it would increase if Tanker tookout new notes payable. What would cause it to decrease? It would decrease ifTanker paid off some of the notes. Let’s put that information into the T-account:

1eSG_C13_0131792075.qxd 10/23/06 9:35 AM Page 455

Page 488: polpoa_sg

456 Chapter 13 | Demo Doc 1 Solution

So we can calculate new cash notes:

$815 " $500 # X # $200 ! $660 X ! $145

Now we can put these numbers into the cash flow statement. Cash received fromnew notes was $145. This transaction increased Cash, so it has a positive effecton cash flow. Cash paid to repay old notes was $500. This transaction decreasedCash, so it has a negative effect on cash flow.

Treasury Stock

Now we must analyze the changes in Tanker’s equity. Tanker had some activitywith treasury stock during the year.We know that Tanker purchased treasury stock.

Treasury Stock

Bal. 12/31/07 0increases decreases

Bal. 12/31/08 1,000

What could cause this account to go up? It would go up if Tanker purchasedtreasury stock. What could cause it to go down? It would go down if treasurystock were sold. We know that no treasury stock was sold:

Treasury Stock

Bal. 12/31/07 0Treasury stock purchased X

Treasury stock sold 0Bal. 12/31/08 1,000

So we can calculate that treasury stock purchased:

$0 # X " 0 ! $1,000X ! $1,000

This balance means that cash was paid by Tanker, which is a negative effect oncash flow.

Dividends

The other account in equity that changed is Retained Earnings. The twomajor transactions affecting Retained Earnings are net income and dividends.

Net income was already listed in the operating activities section, so all thatremains to be included in the financing activities section is dividend activity.

The Retained Earnings account looks like this:

Retained Earnings

Bal. 12/31/07 200decreases increases

Bal. 12/31/08 880What makes Retained Earnings go up? It goes up when Tanker earns net income.What makes it go down? It goes down when Tanker pays dividends:

Retained Earnings

Bal. 12/31/07 200Cash dividends paid X

Net income 1,000Bal. 12/31/08 880

1eSG_C13_0131792075.qxd 10/23/06 9:35 AM Page 456

Page 489: polpoa_sg

So cash dividends declared and paid are calculated as:

$200 # $1,000 " X ! $880 X ! $320

These dividends were paid in cash so this transaction has a negative effect on cash.Totaling these numbers completes financing activities. The completed

financing activities section would appear as:

Financing Activities

Cash Proceeds from New Notes $ 145Cash Repayment of Old Notes (500)Cash Purchase of Treasury Stock (1,000)Cash Dividends Paid (320)Total Cash Flow Provided by Financing Activities $(1,675)

Now we must combine operating activities, investing activities, and financingactivities to get the total cash flow (the change in Cash during the year).

Next, we show the Cash balance from the prior year (December 31, 2007)and add it to total cash flow to get this year’s Cash balance (December 31, 2008).

Operating Activities Net income $ 1,000 + Depreciation expense 110 – Gain on sale of furniture (80) + Decrease in accounts receivable 120 – Increase in inventory (50) – Increase in prepaid insurance (30) + Increase in accounts payable 150 Total cash flow provided by operating activities $ 1,220

Investing Activities Cash paid to purchase new furniture $ (95) Cash proceeds from sale of furniture 90 Total cash flow provided by investing activities $ (5)

Financing Activities Cash proceeds from new notes $ 145 Cash repayment of old notes (500) Cash purchase of treasury stock (1,000) Cash dividends paid (320) Total cash flow provided by financing activities $(1,675) Total cash flow (change in Cash balance) $ (460) Cash, December 31, 2007 $1,160 Cash, December 31, 2008 $ 700

Noncash Investing and Financing Activities Purchase of furniture with note payable $ 200

TANKER, INC.Statement of Cash Flows

Year Ended December 31, 2008

Part 1 Part 2 Part 3 Demo DocComplete

1eSG_C13_0131792075.qxd 10/23/06 9:35 AM Page 457

Page 490: polpoa_sg

458 Chapter 13 | Demo Doc 2

Statement of Cash Flows (Direct Method)Learning Objectives 1, 2, 4

Use the information for Tanker Inc. in the previous question:

Sales revenues $3,400 Less cost of goods sold (1,750) Gross margin $1,650 Depreciation expense $ (110) Insurance expense (230) Other operating expenses (390) Gain on sale of furniture 80 Net income $1,000

TANKER, INC.Income Statement

Year Ended December 31, 2008

Demo Doc 2

Current: Cash Accounts receivable Inventory Prepaid insurance

Furniture Less: Accum. Depr.Net

Total assets

Current: Accounts payable Long-term notes payableTotal liabilities Common stock (no par)Retained earnings Less: Treasury stockTotal equityTotal liabilities and equity

$ 700300800120

1,500(400)

1,100

$3,020

$1,16042075090

1,400(475)925

$3,345

$(460)(120)

5030

$ 680

660$1,340

1,800880

(1,000)$1,680$3,020

$ 530

815$1,345

1,800200

0$2,000$3,345

$ 150

(155)

2008Assets Liabilities

Owners’ Equity

2007 Change 2008 2007 Change

TANKER, INC.Comparative Balance Sheet December 31, 2008 and 2007

Requirements

1. Prepare the operating activities section of Tanker’s statement of cash flowsusing the direct method.

2. How is the information a cash flow statement provides different from theinformation an income statement provides?

1eSG_C13_0131792075.qxd 10/23/06 9:35 AM Page 458

Page 491: polpoa_sg

Demo Doc 2 Solutions | Chapter 13 459

2 Distinguish betweenoperating, investing,and financing cashflows

4 Prepare a statement ofcash flows by thedirect method

Demo Doc 2 SolutionsRequirement 1

Prepare the operating activities section of Tanker’s statement of cash flowsusing the direct method.

Demo DocCompletePart 1 Part 2

We need to list all of the cash transactions involved in Tanker’s day-to-day busi-ness operations. For this task, we look at the income statement to get an idea ofwhat these transactions are.

Cash Received from Customers

What is the first item on the income statement? Revenues. How does thisitem translate into a cash transaction? Revenues should result in customers giv-ing Tanker cash, so the appropriate line on the direct method cash flow statementis “cash received from customers.”

For each income statement account that is not 100% cash, there is always a bal-ance sheet account to record the related accrual. In this case, Accounts Receivable(on the balance sheet) takes care of revenues when cash has not yet been collected.

Accounts Receivable

Bal. 12/31/07 420increases decreases

Bal. 12/31/08 300

Accounts Receivable

Bal. 12/31/07 420Sales revenue 3,400 Cash collected XBal. 12/31/08 300

What could cause this account to increase? It would increase if Tanker had moresales. What could cause it to decrease? It would decrease if Tanker collected thecash. We know from the income statement that sales were $3,400, so:

$420 # $3,400 " X ! $300X ! Cash collected from customers ! $3,520

This amount appears on the direct method cash-flow statement.

Cash Paid to Suppliers

The next line on the income statement is cost of goods sold. How does thisitem relate to a cash transaction? In order to get the goods Tanker sold, it mustbuy the items from a supplier and pay for them. So the appropriate line on thecash flow statement is “cash paid to suppliers.” Accounts Payable is the balancesheet account that takes care of bills to suppliers that have not yet been paid.

Accounts Payable

Bal. 12/31/07 530decreases increases

Bal. 12/31/08 680

1eSG_C13_0131792075.qxd 10/23/06 9:35 AM Page 459

Page 492: polpoa_sg

460 Chapter 13 | Demo Doc 2 Solutions

$530 # $1,800 " X ! $680X ! Cash payments to suppliers ! $1,650

This amount appears on the direct method cash-flow statement.

Cash Paid for Expenses

The next item on the income statement is depreciation expense. Because weknow that it is 100% noncash, we can ignore it for the direct method.

Next is insurance expense.This item would result in “cash paid as insurance.”To calculate this number, we need to analyze the Prepaid Insurance account.

What could cause this account to increase? It would increase if Tanker hadmore bills (that is, if Tanker were to purchase inventory from its suppliers). Whatcould cause it to decrease? It would decrease if Tanker paid the cash it owed tothe suppliers. However, we don’t know how much inventory was purchased.

We can figure this out using the inventory formula from Chapters 5 and 8:

Putting this amount into the Accounts Payable T-account:

COGS BeginningInventory

Purchases EndingInv

! # "eentory

$1,750 $750 Purchases $800

Purchase

! # "

ss $1,800!

Accounts Payable

Bal. 12/31/07 530Cash payments X

Inventory purchases 1,800Bal. 12/31/08 680

Prepaid InsuranceBal. 12/31/07 90

increases decreasesBal. 12/31/08 120

What could cause this account to increase? It would increase if Tanker paid moreinsurance in advance. What could cause it to decrease? It would decrease ifTanker incurred that insurance expense. We know from the income statementthat insurance expense was $230.

Prepaid InsuranceBal. 12/31/07 90Cash payments X

Insurance expense 230Bal. 12/31/08 120

$90 # X " $230 ! $300X ! Cash paid as insurance ! $260

This amount applies to the direct method statement of cash flows.Following insurance expense are other expenses. Let’s leave this item until

the end.The gain on sale of furniture is a noncash transaction that does not affect a

direct method cash-flow statement.

1eSG_C13_0131792075.qxd 10/23/06 9:35 AM Page 460

Page 493: polpoa_sg

Demo Doc 2 Solutions | Chapter 13 461

Now, we come back to other expenses. Do any other current asset or current lia-bility accounts remain with which we have not yet dealt? No, we have analyzed allof them, which means they have no accrual portion (that is, no noncash portion). Sowe can just assume that they were all paid in cash. Therefore, the last line in theoperating activities section is “other cash expenses” of $390.

Operating Activities

Cash Collected from Customers $3,520

Cash Paid to Suppliers (1,650)

Cash Paid for Insurance (260)

Other Cash Expenses (390)

Cash Flow Provided by Operating Activities $1,220

Notice that the “cash flow provided by operating activities” of $1,220 is the sametotal we calculated under the indirect method. It is always the case that cash flowfrom operating activities is the same under the direct and indirect methods. Thischeck is a good way to confirm that our calculations are correct.

Remember that the investing and financing activities are the same underboth methods. So the rest of Tanker’s cash flow statement (investing activities tothe end) would be identical to what is shown in Demo Doc 1.

Requirement 2

How is the information provided by a cash flow statement different from theinformation provided by an income statement?

Demo DocCompletePart 1 Part 2

The income statement shows the determination of net income. Net income is cal-culated on an accrual basis.

Net income not only includes cash transactions; it also includes noncashtransactions. We record revenue earned and expenses incurred regardless ofwhether cash has been received or paid.

The cash flow statement shows the determination of cash flow (that is, thechange in the cash balance during the year). Because the cash flow statementdistills all transactions down to their cash components only, it is missing certainnoncash transactions that are included in net income. Cash flow is actually netincome under the cash basis of accounting.

So the primary difference is that the income statement is prepared underthe accrual basis of accounting whereas the cash-flow statement is preparedunder the cash basis of accounting.

1 Explain the purposesthe statement of cashflows and describe itselements

Part 1 Part 2 Demo DocComplete

1eSG_C13_0131792075.qxd 10/23/06 9:35 AM Page 461

Page 494: polpoa_sg

Quick Practice QuestionsTrue/False

_____ 1. The statement of cash flows helps to inform the reader about all of thedifferences between net income and cash flows from operations.

_____ 2. A company may have net income but still have a net cash outflow.

_____ 3. Cash payments for interest expense would be classified as a financingactivity.

_____ 4. Under the direct method, cash received from customers is calculated asthe sum of Sales Revenue plus the increase in Accounts Receivable.

_____ 5. Purchases of plant assets for cash would be classified as a financingactivity.

_____ 6. Under the indirect method, depreciation expense would be subtractedfrom net income in the operating activities.

_____ 7. The majority of U.S. corporations use the direct method in preparingthe statement of cash flows.

_____ 8. Under the indirect method, the acquisition of land through theissuance of common stock would be an investing activity on the state-ment of cash flows.

_____ 9. When using the indirect method, a loss on sale of equipment is addedto net income under the operating activities.

_____10. Interest received on a bond investment would be shown as an investingcash inflow.

462 Chapter 13 | Quick Practice Questions

1eSG_C13_0131792075.qxd 10/23/06 9:35 AM Page 462

Page 495: polpoa_sg

Multiple Choice

1. Which of the following statements is incorrect? a. A statement of cash flows is a basic financial statement required by

GAAPb. A statement of cash flows is dated for a period of time as opposed to a

point in timec. One purpose of a statement of cash flows is to predict future cash

flowsd. The statement of cash flows may be combined with the stockholders’

equity section of the balance sheet

2. Operating activities have a relationship with which part of thebalance sheet?a. Current assets and current liabilitiesb. Long-term assetsc. Owners’ equity and all liabilitiesd. Owners’ equity and long-term liabilities

3. Dividend payments would be included in which section of thestatement of cash flows?a. Operating activitiesb. Financing activitiesc. Investing activitiesd. Dividend payments are not included on the statement of cash flows

4. Cash dividends received would be included in which section ofthe statement of cash flows?a. Operating activitiesb. Financing activitiesc. Investing activitiesd. Cash dividends received is not included on the statement of cash

flows

5. The purchase of treasury stock would be included in whichsection of the statement of cash flows?a. Operating activitiesb. Financing activitiesc. Investing activitiesd. The purchase of treasury stock is not included on the statement of

cash flows

6. Activities that create revenues and expenses are included inwhich section of the statement of cash flows? a. Investing activitiesb. Operating activitiesc. Financing activitiesd. Noncash investing and financing activities

7. Where are noncash investing and financing activities reported? a. The financing activities section of the statement of cash flowsb. The investing activities section of the statement of cash flowsc. Both a and b are correctd. An accompanying schedule to the statement of cash flows

Quick Practice Questions | Chapter 13 463

1eSG_C13_0131792075.qxd 10/23/06 9:35 AM Page 463

Page 496: polpoa_sg

8. Where is the gain resulting from the sale of equipment shownunder the indirect method? a. In the operating activities section as a deductionb. In the operating activities section as an additionc. In the investing activities section as an additiond. In the financing activities section as a deduction

9. Wilson Company’s 2008 income statement reports depreciationexpense of $25,000. How would depreciation be shown on thestatement of cash flows using the direct method for 2008?a. As an addition under financing activitiesb. As a deduction under operating activitiesc. As an addition under operating activitiesd. It would not be reported

10. Which of the following would be shown as a deduction to netincome under the operating activities section using the indirectmethod? a. Depletion expenseb. Increase in Accounts Payable account balance for the periodc. Increase in Inventory balance for the periodd. Decrease in Accounts Receivable account balance for the period

464 Chapter 13 | Quick Practice Questions

1eSG_C13_0131792075.qxd 10/23/06 9:35 AM Page 464

Page 497: polpoa_sg

Quick Practice Questions | Chapter 13 465

Quick Exercises

13-1. Your best friend just lost his job because the company he wasworking for went bankrupt. He complains to you that eventhough the company had been profitable for three years in arow, it still went out of business. He wonders how such a thingcan happen.

a. Explain the most likely reason for the company’s declaring bank-ruptcy. Could your friend have seen it coming? How?

b. Discuss the four purposes of the statement of cash flows.

13-2. State whether each of the following events should be classi-fied as an operating activity (O), investing activity (I), financ-ing activity (F), shown in a separate schedule of noncashinvesting and financing activities (N), or not disclosed on thestatement of cash flows (NA).

________ a. Received cash dividends.________ b. Retired bonds payable by issuing common stock.________ c. Paid for merchandise purchased on account.________ d. Paid interest on a short-term note payable.________ e. Received stock dividends.________ f. Paid for a three-year insurance policy on property.________ g. Issued preferred stock in exchange for land.________ h. Issued common stock for cash.________ i. Received cash from sale of land.________ j. Purchased equipment for cash.

1eSG_C13_0131792075.qxd 10/23/06 9:35 AM Page 465

Page 498: polpoa_sg

13-3. Using the following data, prepare the operating activities sec-tion of a statement of cash flows for Washington Corporationfor the year ended December 31, 2008. Assume the indirectmethod is used.

Increase in Salary Payable $ 1,500

Decrease in Accounts Payable 2,000

Increase in Accounts Receivable 3,500

Net Income 98,000

Decrease in Inventory 5,800

Increase in Prepaid Expenses 1,200

Depreciation Expense, Equipment 5,000

Depreciation Expense, Buildings 7,500

Gain on Sale of Equipment 1,300

Loss on Sale of Patent 2,500

466 Chapter 13 | Quick Practice

WASHINGTON CORPORATIONPartial Statement of Cash Flows

Year Ended December 31, 2008

1eSG_C13_0131792075.qxd 10/23/06 9:35 AM Page 466

Page 499: polpoa_sg

Quick Practice Questions | Chapter 13 467

13-4. For each of the following events, determine whether it shouldbe classified as an operating activity (O), investing activity (I),or financing activity (F). Then determine the cash inflow or(outflow).

Transaction Type of Cash InflowDescription Activity (Outflow)

a. Declared cash dividends of $21,000 during the currentperiod. Dividends payable onJanuary 1 were $1,500,the December 31 balance was $2,300. ___________ _____________

b. Interest income on the income statement for the current period is $22,000.Interest receivable on January 1 was $2,700; the December 31 balance was $2,250. ___________ _____________

c. Issued $1,000,000,10-year, 10% bonds at 102. ___________ _____________

d. Sales on account for the current period amount to $160,000. The January 1 balance in Accounts Receivable was $95,000;the December 31 balance was $106,000. ___________ _____________

e. Purchased equipment for $215,000 cash. ___________ _____________

f. Sold 1,000 shares of $20 par common stock for cash at $29. ___________ _____________

g. Salary expense on the income statement for the current year is $151,500.The Salary Payable balance on January 1 was $20,300;the December 31 balance was $17,800. ___________ _____________

1eSG_C13_0131792075.qxd 10/23/06 9:35 AM Page 467

Page 500: polpoa_sg

468 Chapter 13 | Quick Practice Questions

AYCOTH INC.Partial Statement of Cash Flows

Year Ended December 31, 2008

13-5. Aycoth, Inc., gathered the following data from its accountingrecords for the year ended December 31, 2008:

Depreciation Expense $ 15,900

Payment of Income Taxes 24,500

Collections of Accounts Receivable 166,700

Purchase of Treasury Stock 40,000

Declaration of Stock Dividend 65,000

Loss on Sale of Plant Assets 8,400

Collection of Dividend Revenue 13,800

Payments of Salaries and Wages 83,600

Cash Sales 102,900

Net Income 61,200

Acquisition of Land 73,500

Payment of Interest 19,400

Interest Received on Investments 3,100

Issuance of Bonds Payable 500,000

Increase in Accounts Payable 20,300

Payments to Suppliers 170,300

Acquisition of Equipment by IssuingLong-Term Note Payable 50,000

Prepare the operating activities section of the statement of cash flowsusing the direct method.

1eSG_C13_0131792075.qxd 10/23/06 9:35 AM Page 468

Page 501: polpoa_sg

Do It Yourself! Question 1 | Chapter 13 469

Do It Yourself! Question 1

Indirect Method

Clean Co. had the following information at December 31, 2008:

Sales revenue $1,800 Less cost of goods sold (960) Gross margin $ 840

Depreciation expense $ (90) Rent expense (140) Other operating expenses (195) Loss on sale of equipment (55) Net income $ 360

CLEAN CO.Income Statement

Year Ended December 31, 2008

Current: Cash Accounts receivable Inventory Prepaid rent

Equipment Less: Accum. Depr.Net

Total assets

Current: Accounts payable Long-term notes payableTotal liabilities Common stock (no par)Retained earnings Total equityTotal liabilities and equity

$ 460510710170

1,350(400)950

$2,800

$320420750250

1,500(650)850

$2,590

$14090

(40)(80)

$ 800

600$1,400

$ 2001,200

$1,400$2,800

$ 540

900$1,440

1501,000

$1,150$2,590

$ 260

(300)

2008Assets Liabilities

Owners’ Equity

2007 Change 2008 2007 Change

CLEAN CO.Comparative Balance Sheet December 31, 2008 and 2007

Other important information includes the following:

• Every year, Clean declares and pays cash dividends.

• During 2008, Clean sold old equipment for cash. Clean also boughtnew equipment for $120 cash and a $140 note payable.

• During 2008, Clean repaid $600 of notes payable in cash and borrowednew long-term notes payable for cash.

• During 2008, new common stock was issued. No stock was retired.

1eSG_C13_0131792075.qxd 10/23/06 9:35 AM Page 469

Page 502: polpoa_sg

470 Chapter 13 | Do It Yourself! Question 1

Requirement

Prepare Clean’s statement of cash flows for the year ended December 31, 2008,using the indirect method.

Notes Payable

Common Stock

Retained Earnings

Equipment (Net)

1eSG_C13_0131792075.qxd 10/23/06 9:35 AM Page 470

Page 503: polpoa_sg

Do It Yourself! Question 1 | Chapter 13 471

1eSG_C13_0131792075.qxd 10/23/06 9:35 AM Page 471

Page 504: polpoa_sg

472 Chapter 13 | Do It Yourself! Question 2

Accounts Receivable

Accounts Payable

Do It Yourself! Question 2

Direct MethodUse the information for Clean, Inc., in the previous question.

Requirement

Prepare the operating activities section of Clean’s statement of cash flowsusing the direct method.

Prepaid Rent

1eSG_C13_0131792075.qxd 10/23/06 9:35 AM Page 472

Page 505: polpoa_sg

Quick Practice Solutions | Chapter 13 473

Quick Practice SolutionsTrue/False

T 1. The statement of cash flows helps to inform the reader about all of the differences between net income and cash flows from operations.(p. 690)

T 2. A company may have net income but still have a net cash outflow.(p. 691)

F 3. Cash payments for interest expense would be classified as a financing activity.False–Cash payments for interest expense would be classified as an operating activity. (p. 691)

F 4. Under the direct method, cash received from customers is calculated as the sum of Sales Revenue plus the increase in Accounts Receivable.False–Under the direct method, cash received from customers is calculated as Sales Revenue minus the increase in Accounts Receivable. (p. 692)

F 5. Purchases of plant assets for cash would be classified as a financingactivity.False–Purchases of plant assets for cash would be classified as an investing activity. (pp. 692–693)

F 6. Under the indirect method, depreciation expense would be subtracted from net income in the operating activities.False–Under the indirect method, depreciation expense would be added to net income in the operating activities. (p. 692)

F 7. The majority of U.S. corporations use the direct method in preparing the statement of cash flows.False–The majority of U.S. corporations use the indirect method in preparing the statement of cash flows. (p. 702)

F 8. Under the indirect method, the acquisition of land through the issuance of common stock would be an investing activity on the statement of cash flows.False–Under the indirect method, the acquisition of land through the issuance of common stock would be a noncash investing and financing activity. (p. 702)

T 9. When using the indirect method, a loss on sale of equipment is added to net income under the operating activities. (pp. 697–698)

F 10. Interest received on a bond investment would be shown as an investing cash inflow.False–Interest received on a bond investment would be shown as anoperating cash inflow. (p. 704)

1eSG_C13_0131792075.qxd 10/23/06 9:35 AM Page 473

Page 506: polpoa_sg

474 Chapter 13 | Quick Practice Solutions

Multiple Choice

1. Which of the following statements is incorrect? (p. 690)a. A statement of cash flows is a basic financial statement required by

GAAPb. A statement of cash flows is dated for a period of time as opposed to a

point in timec. One purpose of a statement of cash flows is to predict future cash

flowsd. The statement of cash flows may be combined with the stockholders’

equity section of the balance sheet

2. Operating activities have a relationship with which part of thebalance sheet? (p. 691)a. Current assets and current liabilitiesb. Long-term assetsc. Owners’ equity and all liabilitiesd. Owners’ equity and long-term liabilities

3. Dividend payments would be included in which section of thestatement of cash flows? (p. 692)a. Operating activitiesb. Financing activitiesc. Investing activitiesd. Dividend payments are not included on the statement of cash flows

4. Cash dividends received would be included in which section ofthe statement of cash flows? (p. 691)a. Operating activitiesb. Financing activitiesc. Investing activitiesd. Cash dividends received is not included on the statement of cash

flows

5. The purchase of treasury stock would be included in whichsection of the statement of cash flows? (pp. 691– 692)a. Operating activitiesb. Financing activitiesc. Investing activitiesd. The purchase of treasury stock is not included on the statement of

cash flows

6. Activities that create revenues and expenses are included inwhich section of the statement of cash flows? (p. 691)a. Investing activitiesb. Operating activitiesc. Financing activitiesd. Noncash investing and financing activities

7. Where are noncash investing and financing activities reported? (p. 693)a. The financing activities section of the statement of cash flowsb. The investing activities section of the statement of cash flowsc. Both a and b are correctd. An accompanying schedule to the statement of cash flows

1eSG_C13_0131792075.qxd 10/23/06 9:35 AM Page 474

Page 507: polpoa_sg

Quick Practice Solutions | Chapter 13 475

8. Where is the gain resulting from the sale of equipment shownunder the indirect method? (p. 694)a. In the operating activities section as a deductionb. In the operating activities section as an additionc. In the investing activities section as an additiond. In the financing activities section as a deduction

9. Wilson Company’s 2008 income statement reports depreciationexpense of $25,000. How would depreciation be shown on thestatement of cash flows using the direct method for 2008? (p. 696)a. As an addition under financing activitiesb. As a deduction under operating activitiesc. As an addition under operating activitiesd. It would not be reported

10. Which of the following would be shown as a deduction to netincome under the operating activities section using the indirectmethod? (p.697)a. Depletion expenseb. Increase in Accounts Payable account balance for the periodc. Increase in Inventory balance for the periodd. Decrease in Accounts Receivable account balance for the period

1eSG_C13_0131792075.qxd 10/23/06 9:35 AM Page 475

Page 508: polpoa_sg

476 Chapter 13 | Quick Practice Solutions

Quick Exercise

13-1. Your best friend just lost his job because the company he wasworking for went bankrupt. He complains to you that eventhough the company had been profitable for three years in arow, it still went out of business. He wonders how such a thingcan happen. (pp. 690–692)

a. Explain the most likely reason for the company’s declaring bank-ruptcy. Could your friend have seen it coming? How?A profitable company is one in which revenues exceed expenses onan accrual basis. It does not necessarily mean that the company isgenerating enough cash to pay its bills. The most likely reason yourfriend’s company went bankrupt is lack of cash. If your friend hadaccess to the statement of cash flows, the cash flow problems wouldlikely have been evident.

b. Discuss the four purposes of the statement of cash flows.1. To help predict future cash flows2. To evaluate management decisions3. To determine the company’s ability to pay dividends to stock-

holders and interest and principal to creditors4. To show the relationship of net income to changes in the busi-

ness’s cash

13-2. State whether each of the following events should be classi-fied as an operating activity (O), investing activity (I), financ-ing activity (F), shown in a separate schedule of noncashinvesting and financing activities (N), or not disclosed on thestatement of cash flows (NA). (pp. 691–692)

O a. Received cash dividends.N b. Retired bonds payable by issuing common stock.O c. Paid for merchandise purchased on account.O d. Paid interest on a short-term note payable.

NA e. Received stock dividends.O f . Paid for a three-year insurance policy on property.N g. Issued preferred stock in exchange for land.F h. Issued common stock for cash.I i . Received cash from sale of land.I j . Purchased equipment for cash.

1eSG_C13_0131792075.qxd 10/23/06 9:35 AM Page 476

Page 509: polpoa_sg

Quick Practice Solutions | Chapter 13 477

Cash flows from operating activities: Net income $ 98,000 Adjustments to reconcile net income to net cash provided by operating activities: Depreciation on equipment $5,000 Depreciation on buildings 7,500 Loss on sale of patent 2,500 Gain on sale of equipment (1,300) Increase in accounts receivable (3,500) Increase in prepaid expenses (1,200) Decrease in inventory 5,800 Increase in salary payable 1,500 Decrease in accounts payable (2,000) 14,300 Net cash inflow from operating activities $112,300

WASHINGTON CORPORATIONPartial Statement of Cash Flows

Year Ended December 31, 2008

13-3. Using the following data, prepare the operating activities sec-tion of a statement of cash flows for Washington Corporationfor the year ended December 31, 2008. Assume the indirectmethod is used. (pp. 696–697)

Increase in Salary Payable $ 1,500

Decrease in Accounts Payable 2,000

Increase in Accounts Receivable 3,500

Net Income 98,000

Decrease in Inventory 5,800

Increase in Prepaid Expenses 1,200

Depreciation Expense, Equipment 5,000

Depreciation Expense, Buildings 7,500

Gain on Sale of Equipment 1,300

Loss on Sale of Patent 2,500

1eSG_C13_0131792075.qxd 10/23/06 9:35 AM Page 477

Page 510: polpoa_sg

478 Chapter 13 | Quick Practice Solutions

13-4. For each of the following events, determine whether it shouldbe classified as an operating activity (O), investing activity (I),or financing activity (F). Then determine the cash inflow or(outflow). (pp. 704–707)

Transaction Type of Cash InflowDescription Activity (Outflow)

a. Declared cash dividends of $21,000 during the current period. Dividends payable on January 1 were $1,500, the December 31 balance was $2,300. F $ (20,200)

b. Interest income on the income statement for the current periodis $22,000. Interest receivable on January 1 was $2,700; the December 31 balance was $2,250. O $22,450

c. Issued $1,000,000, 10-year,10% bonds at 102. F $1,020,000

d. Sales on account for the currentperiod amount to $160,000. TheJanuary 1 balance in AccountsReceivable was $95,000; the December 31 balance was $106,000. O $ 149,000

e. Purchased equipment for $215,000 cash. I $(215,000)

f. Sold 1,000 shares of $20 par common stock for cash at $29. F $ 29,000

g. Salary expense on the income statement for the current year is $151,500. The Salary Payablebalance on January 1 was $20,300; the December 31 balance was $17,800. O $(154,000)

1eSG_C13_0131792075.qxd 10/23/06 9:35 AM Page 478

Page 511: polpoa_sg

Quick Practice Solutions | Chapter 13 479

Cash flows from operating activities: Receipts Collections from customers $ 269,600* Interest received 3,100 Dividends received 13,800 Total cash receipts $286,500 Payments: To suppliers $(170,300) To employees (83,600) For interest (19,400) For income tax (24,500) Total cash payments (297,800) Net cash outflow from operating activities $ (11,300) *($166,700 + $102,900 = $269,600)

AYCOTH, INC.Partial Statement of Cash Flows

Year Ended December 31, 2008

13-5. Aycoth, Inc., gathered the following data from its accountingrecords for the year ended December 31, 2008:

Depreciation Expense $ 15,900

Payment of Income Taxes 24,500

Collections of Accounts Receivable 166,700

Purchase of Treasury Stock 40,000

Declaration of Stock Dividend 65,000

Loss on Sale of Plant Assets 8,400

Collection of Dividend Revenue 13,800

Payments of Salaries and Wages 83,600

Cash Sales 102,900

Net Income 61,200

Acquisition of Land 73,500

Payment of Interest 19,400

Interest Received on Investments 3,100

Issuance of Bonds Payable 500,000

Increase in Accounts Payable 20,300

Payments to Suppliers 170,300

Acquisition of Equipment by IssuingLong-Term Note Payable 50,000

Prepare the operating activities section of the statement of cash flows using the direct method. (pp. 702–706)

1eSG_C13_0131792075.qxd 10/23/06 9:35 AM Page 479

Page 512: polpoa_sg

480 Chapter 13 | Do It Yourself! Question 1 Solutions

2 Distinguish betweenoperating, investing,and financing cashflows

3 Prepare a statement ofcash flows by the indi-rect method

Notes Payable

Bal. 12/31/07 900Note repayments 600

New cash notes XNew noncash notes 140

Bal. 12/31/08 600

Common Stock

Bal. 12/31/07 150Retirements 0

New stock issued X

Bal. 12/31/08 200

Retained Earnings

Bal. 12/31/07 1,000Cash dividends paid X

Net Income 360

Bal. 12/31/08 1,200

Equipment (Net)

Bal. 12/31/07 850Cash purchases 120Noncash purchases 140

NBV equipment sold XDepreciation expense 90

Bal. 12/31/08 950

Do It Yourself! Question 1 Solution

Requirement

Prepare Clean’s statement of cash flows for the year ended December 31, 2008,using the indirect method.

Calculations: Investing Activities

The $140 note payable is a noncash transaction

X ! NBV of Equipment Sold ! $70Loss ! – $55 ! Cash Received – $70Cash Received on Sale of Equipment ! $15

Calculations: Financing Activities

New Cash Notes ! X ! $160

2 Distinguish betweenoperating, investing,and financing cashflows

3 Prepare a statement ofcash flows by the indi-rect method

New Stock Issued ! X ! $50

Cash Dividends Paid ! X ! $160

1eSG_C13_0131792075.qxd 10/23/06 9:35 AM Page 480

Page 513: polpoa_sg

Do It Yourself! Question 1 Solutions | Chapter 13 481

Operating Activities Net income $360 + Depreciation expense 90 + Loss on sale of equipment 55 – Increase in accounts receivable (90) + Decrease in inventory 40 + Decrease in prepaid rent 80 + Increase in accounts payable 260 Total cash flow provided by operating activities $795

Investing Activities Cash paid to purchase new equipment $(120) Cash proceeds from sale of equipment 15 Total cash flow provided by investing activities $(105)

Financing Activities Cash proceeds from new notes $160 Cash repayment of old notes (600) Cash proceeds from new stock issue 50 Cash dividends paid (160) Total cash flow provided by financing activities $(550) Total cash flow (change in Cash during year) $140 Cash, December 31, 2007 $320 Cash, December 31, 2008 $460

Noncash Investing and Financing Activities Purchase of equipment with note payable $140

CLEAN CO.Statement of Cash Flows

Year Ended December 31, 2008

1eSG_C13_0131792075.qxd 10/23/06 9:35 AM Page 481

Page 514: polpoa_sg

482 Chapter 13 | Do It Yourself! Question 2 Solutions

Accounts Payable

Bal. 12/31/07 540Cash payments X

Inventory purchases 920

Bal. 12/31/08 800

Prepaid Rent

Bal. 12/31/07 250Cash payments X

Rent expense 140

Bal. 12/31/08 170

Do It Yourself! Question 2 Solutions

Requirement

Prepare the operating activities section of Clean’s statement of cash flowsusing the direct method.

2 Distinguish betweenoperating, investing,and financing cashflows

4 Prepare a statement ofcash flows by thedirect method

Accounts Receivable

Bal. 12/31/07 420Sales revenue 1,800

Cash collected X

Bal. 12/31/08 510

X ! Cash Collected from Customers ! $1,710

COGS ! $960 ! $750 # Purchases – $710

Purchases ! $920

X ! Cash Payments to Suppliers ! $660

X ! Cash Paid as Rent ! $60

Operating Activities

Cash Collected from Customers $1,710

Cash Paid to Suppliers (660)

Cash Paid for Rent (60)

Other Cash Expenses (195)

Cash Flow from Operating Activities $ 795

1eSG_C13_0131792075.qxd 10/23/06 9:35 AM Page 482

Page 515: polpoa_sg

The Power of Practice | Chapter 13 483

The Power of PracticeFor more practice using the skills learned in this chapter, visit MyAccountingLab.There you will find algorithmically generated questions that are based on theseDemo Docs and your main textbook’s Review and Accounting Practice sections.

To go to MyAccountingLab and follow these steps:

1. Direct your URL to www.myaccountinglab.com.2. Log in using your name and password.3. Click the MyAccountingLab link.4. Click Study Plan in the left navigation bar.5. From the table of contents, select Chapter 13, The Statement of Cash Flows.6. Click a link to work on the tutorial exercises.

1eSG_C13_0131792075.qxd 10/23/06 9:35 AM Page 483

Page 516: polpoa_sg

1eSG_C13_0131792075.qxd 10/23/06 9:35 AM Page 484

Page 517: polpoa_sg

Financial StatementAnalysis14

WHAT YOU PROBABLY ALREADY KNOW

For years now, you have been a student and have taken manyexams. You probably already know that receiving your gradesevokes some typical responses. Your first reaction may be the levelof satisfaction you have with your grade compared to your previousgrades received in that class and the established grading norms foryour institution. You may then ask your friends what grade theyreceived so that you can compare your results with theirs. Theinstructor may announce the average exam results and you couldthen determine whether you performed better or worse than theaverage. Students often like to assess their performance bycomparing their grade to a standard, their peers, and the average.Businesses do the same thing. In this chapter, you study varioustechniques and ratios that a business can use to assess itsperformance using comparisons to previous results, competitors,and the industry average.

Learning Objectives

Perform a horizontal analysis of comparative financial statements.

Horizontal analysis provides comparisons of financial information overtime. To analyze a line item in the financial statements, the differencebetween the current and earlier time period amounts is computed. Thedollar amount change of the line item between the periods is useful, butit is more informative to determine the percentage change by dividingthe dollar change (current period amount, or this year’s balance, minusthe earlier period amount, or last year’s balance) by the earlier periodamount. Review the horizontal analysis of the income statement and thebalance sheet in Exhibits 14-1 and 14-2 (pp. 750–751).

Perform a vertical analysis of financial statements.

Vertical analysis provides comparisons of individual items on a financialstatement to a relative base. The base, which serves as the denominator,is usually net sales for the income statement and total assets for thebalance sheet. The vertical analysis percentage is calculated by dividingeach financial statement item amount by the relevant base of net salesor total assets.

The vertical analysis percentage is shown next to the item amount onthe financial statement. Review the vertical analysis of the incomestatement and the balance sheet in Exhibits 14-4 and 14-5 (pp. 753–754).

1

2

1eSG_C14_0131792075.Qxd 10/23/06 9:36 AM Page 485

Page 518: polpoa_sg

486 Chapter 14 | Financial Statement Analysis

4

5

3 Prepare and use common-size financial statements.

A common-size statement is similar to the vertical analysis but showsonly the vertical analysis percentages of each item in the financialstatement. This presentation permits ready comparisons betweencompanies of various sizes. Review the common-size comparisons ofTarget versus the industry average and Wal-Mart in in Exhibits 14-6(p. 755 ) and 14-7 (p. 755).

Compute the standard financial ratios.

Financial ratios are helpful to assess a company’s performance andfinancial position. Trends can be determined and comparisons tocompeting companies can be made. Various ratios are presented tomeasure the following:

• Ability to pay current liabilities• Ability to sell inventory and collect receivables• Ability to pay long-term debt• Profitability• Return on stock investment

Review “Using Ratios to Make Decisions” in the main text for descriptionsand formulas for the financial ratios.

Measure economic value added.

Economic value added, which is used to evaluate a company’s operatingperformance, is equal to Net income plus Interest expense minusCapital charge. Capital charge is the amount that stockholders andlenders charge a company for the use of their money. It is calculated byadding notes payable, loans payable, long-term debt, and stockholders’equity, then multiplying that sum by the cost of capital (a weightedaverage of the returns demanded by the company’s stockholders andlenders). Review the section entitled “Economic Value Added” in themain textbook.

1eSG_C14_0131792075.Qxd 10/23/06 9:36 AM Page 486

Page 519: polpoa_sg

Demo Doc 1 | Chapter 14 487

AssetsCashAccounts receivableInventoryTotal assetsLiabilitiesAccounts payableBonds payableTotal liabilitiesStockholder’s equityCommon stockRetained earningsTotal stockholder’s equityTotal liabilities and stockholder’s equity

$15080

130$360

$ 90140

$230

20110

$130$360

$130145190

$465

$140220

$360

1095

$105$465

2008(Dollar amounts in millions) 2007

MEME CO.Comparative Balance Sheet December 31, 2008 and 2007

Sales revenueLess: Cost of goods soldGross profit

Salary expenseInterest expense Total expenseNet income

$650430

$220

12070

$190$ 30

$580350

$230

14080

$220$ 10

2008(Dollar amounts in millions) 2007

MEME CO.Comparative Income Statement

December 31, 2008 and 2007

Financial Statement AnalysisLearning Objectives 1–5

MeMe Co. had the following information at December 31, 2008:

Demo Doc 1

At December 31, 2006, MeMe’s inventory was $160,000,000 and total equitywas $95,000,000.

Requirements

1. Prepare horizontal and vertical analyses for MeMe’s financial statements.

2. Calculate MeMe’s inventory turnover and rate of return on stockholders’equity ratios for both years.

3. Assuming a 10% cost of capital, what is MeMe’s economic value added for2008?

1eSG_C14_0131792075.Qxd 10/23/06 9:36 AM Page 487

Page 520: polpoa_sg

488 Chapter 14 | Demo Doc 1 Solutions

1 Perform a horizontalanalysis of comparativefinancial statements

Demo Doc 1 SolutionsRequirement 1

Prepare horizontal and vertical analyses for MeMe’s financial statements.

Horizontal Analysis

Part 1 Part 2 Part 3 Part 4 Demo DocComplete

As its name implies, horizontal analysis goes across the rows of the financialstatements, looking at one account and how it has changed.

For each number on the balance sheet and income statement, we calculatethe dollar change and the percent change.

For example, the dollar changes in Accounts Receivable and Sales Revenue:

Accounts Receivable ! $800,000,000 – $145,000,000 ! $(65,000,000) ChangeSales Revenue ! $650,000,000 – $580,000,000 ! $70,000,000 Change

Notice that the parentheses around the change in Accounts Receivable indicatesa decrease, while the positive value for the change in Sales Revenue indicatesthat it increased.

Extra care must be taken when using this calculation on expenses (becausethey are assumed to be subtracted/negative numbers on the income statement).The absolute value of the expense (that is, ignoring the fact that it is subtracted)must be used to calculate dollar change. For example, the dollar change of COGSand Interest Expense is:

COGS ! $430,000,000 – $350,000,000 ! $80,000,000 ChangeInterest Expense ! $70,000,000 – $80,000,000 ! $(10,000,000) Change

Again, the positive value indicates that COGS increased and the negative valueindicates that Interest Expense decreased.

For example, the percent change of Accounts Receivable and Sales Revenue is:

Again, the percent change numbers are negative for Accounts Receivable (whichdecreased in 2008) and positive for Sales Revenue (which increased in 2008).

PercentChange

Dollar ChangeLast Year's Bala

!nnce

AccountsReceivable

$(65,000,000)$145,00

!00,000

(44.8)% Change

Sales Revenue$70,000,

!

!0000

$580,000,00012.1% Change!

PercentChange

Dollar ChangeLast Year s Bala

!�’ nnce

DollarChange

This Year sBalance

Last Year s! "

�’ �’BBalance

1eSG_C14_0131792075.Qxd 10/23/06 9:36 AM Page 488

Page 521: polpoa_sg

Demo Doc 1 Solutions | Chapter 14 489

The percent change is calculated the same way for expenses, again using theabsolute value of the expenses. For example, the percent change of COGS andInterest Expense is:

COGS$80,000,000$350,000,00022.9% Change

In

!

!

tterestExpense

$(10,000,000)$80,000,000

(12

!

! ..5)% Change

AssetsCashAccounts receivableInventoryTotal assetsLiabilitiesAccounts payableBonds payableTotal liabilitiesStockholder’s equityCommon stockRetained earningsTotal stockholder’s equityTotal liabilities and stockholder’s equity

$15080

130$360

$ 90140

$230

20110

$130$360

$130145190

$465

$140220

$360

1095

$105$465

$ 20(65)(60)

$(105)

$ 5080

$(130)

1015

$ 25$(105)

15.4(44.8)(31.6)(22.6)

(35.7)(36.4)(36.1)

100.015.823.8

(22.6)

%

%

%

%

2008(Dollar amounts in millions) 2007 Increase (Decrease)Amount Percent

MEME CO.Horizontal Analysis of Comparative Balance Sheet

December 31, 2008 and 2007

Sales revenueLess: Cost of goods soldGross profit

Salary expenseInterest expense Total expensesNet income

$650430

$220

12070

$190$ 30

$580350

$230

14080

$220$ 10

$ 7080

$(10)

(20)(10)

$(30)$ 20

12.122.9(4.3)

(14.3)(12.5)(13.6)200.0

%

%

2008(Dollar amounts in millions) 2007 Increase (Decrease)Amount Percent

MEME CO.Horizontal Analysis of Comparative Income Statement

Years Ended December 31, 2008 and 2007

1eSG_C14_0131792075.Qxd 10/23/06 9:36 AM Page 489

Page 522: polpoa_sg

490 Chapter 14 | Demo Doc 1 Solutions

As its name implies, vertical analysis takes each number on the financial state-ment and compares it to others in the same year (that is, down the columns of thefinancial statements). Vertical analysis is sometimes called common-size analysisbecause it allows two companies of different sizes to be compared (through theuse of percentages).

Balance Sheet Vertical Analysis

On the balance sheet, each number, whether it be an asset, a liability, or an equityaccount, is calculated as a percentage of total assets.

So in the case of accounts receivable:

In other words, about 22% of all the assets in 2008 are in Accounts Receivable.

Income Statement Vertical Analysis

On the income statement, each number is calculated as a percentage of net salesrevenues.

So in the case of gross profit:

This figure means that for every dollar in sales revenues, about $0.34 went togross profit.

For expenses, the calculation is the same. So in the cases of COGS and inter-est expense:

Vertical Analysis Percent(2008 COGS)

$430,0!

000,000$650,000,000

Vertical Analysis

! 66.2%

PPercent(2008 Interest Expense)

$70,000,000!

$$650,000,000! 10.8%

Vertical Analysis Percent(2008 Gross Profit))

$220,000,000$650,000,000

!

! 33.8%

Vertical Analysis Percent(Income Statement)

!! Account BalanceNet Sales Revenues

Vertical Analysis Percent(2008 Accounts Receeivable)

$80,000,000$360,000,000

!

! 22.2%

Vertical Analysis Percent(Balance Sheet)

Ac!

ccount BalanceTotal Assets

Perform a verticalanalysis of financialstatements

2

Prepare and use common-size financial statements

3

Vertical Analysis

Part 1 Part 2 Part 3 Part 4 Demo DocComplete

1eSG_C14_0131792075.Qxd 10/23/06 9:36 AM Page 490

Page 523: polpoa_sg

Demo Doc 1 Solutions | Chapter 14 491

AssetsCashAccounts receivableInventoryTotal assetsLiabilitiesAccounts payableBonds payableTotal liabilitiesStockholder’s equityCommon stockRetained earningsTotal stockholder’s equityTotal liabilities and stockholder’s equity

$15080

130$360

$ 90140

$230

$ 20110

$130$360

2008(Dollar amounts in millions) 2008%

$130145190

$465

$140220

$360

$ 1095

$205$465

%

%

%

%

%

%%

28.031.140.9

100.0

30.147.377.4

2.220.433.6

100.0

2007 2007%

41.722.236.1

100.0

25.038.963.9

5.530.636.2

100.0

%

%

%

%

%

%%

*

*

*Rounded down to balance

MEME CO.Vertical Analysis of Balance Sheet

December 31, 2008 and 2007

Requirement 2

Calculate MeMe’s inventory turnover and rate of return on stockholders’ equityratios for both years.

Sales revenueLess: Cost of goods soldGross profit

Salary expenseInterest expenseNet income

$650430

$220

12070

$ 30

2008(Dollar amounts in millions) 2008%$580350

$230

14080

$ 10

2007 2007%100.066.233.8

18.410.84.6

%

%

%*Rounded to balance

100.060.439.6

24.113.8

1.7

%

%

%

**

MEME CO.Vertical Analysis of Income StatementYears Ended December 31, 2008 and 2007

Compute the stan-dard financial ratios

4

Part 1 Part 2 Part 3 Part 4 Demo DocComplete

Remember that average (when used in a financial ratio) generally means thebeginning balance plus the ending balance divided by two.

InventoryTurnover

COGSAverage Inventory

!

1eSG_C14_0131792075.Qxd 10/23/06 9:36 AM Page 491

Page 524: polpoa_sg

492 Chapter 14 | Demo Doc 1 Solutions

Requirement 3

Assuming a 10% cost of capital, what is MeMe’s economic value added for 2008?

2008InventoryTurnover

$430,000,000($190,000

!,,000 $130,000,000)/2

2007Inventor

#

! 2.7 times

yyTurnover

$350,000,000($160,000,000 $190,0

!# 000,000)/2

Rate of Returnon Stockhol

! 2 times

dder sEquity

Net income Preferred Dividends�’ !

"

AAverage Common Share Equity

2008 Rate of Retturn onStockholders Equity

$30,000,000 $0�’

! "

(($105,000,000 $130,000,000)/2

20

#

! 25.5%

007 Rate of Return onStockholders Equity

$�’

! 110,000,000 $0($95,000,000 $105,000,000)/2

"

#! 110%

Part 1 Part 2 Part 3 Part 4 Demo DocComplete

Economic Value Added (EVA) ! Net Income # Interest Expense – Capital Charge

Where, in this case, the capital charge is:

Capital Charge ! (Bonds Payable # Equity) $ Cost of CapitalMeMe’s 2008 Capital Charge ! ($140,000,000 # $130,000,000) $ 10% ! $27,000,000

So, MeMe’s 2008 EVA:

! $30,000,000 Net Income # $70,000,000 Interest Expense – $27,000,000 CapitalCharge ! $73,000,000

Part 1 Part 2 Part 3 Part 4 Demo DocComplete

Measure economicvalue added

5

1eSG_C14_0131792075.Qxd 10/23/06 9:36 AM Page 492

Page 525: polpoa_sg

Quick Practice Questions | Chapter 14 493

Quick Practice QuestionsTrue/False

_____ 1. It is generally considered more useful to know the percentage changein financial statement amounts from year to year than to know theabsolute dollar amount of their change.

_____ 2. Benchmarking may be done against an industry average or against akey competitor.

_____ 3. Vertical analysis of financial statements reveals changes in items onthe financial statements over time.

_____ 4. Inventory turnover is the ratio of average inventory to cost of goods sold.

_____ 5. Dividend yield is the ratio that measures the percentage of a stock’smarket value that is returned annually as dividends.

_____ 6. A high current ratio means that a company’s current assets representa relatively large portion (or ratio) of total liabilities.

_____ 7. The debt ratio measures the ability to pay current liabilities.

_____ 8. The acid-test (quick) ratio includes the sum of Cash, Net AccountsReceivable, and Inventory accounts in the numerator.

_____ 9. Earnings per share indicate the net income earned for each share ofcommon and preferred stock.

_____10. There is a direct relationship between leverage and the debt ratio; thehigher the debt ratio, the greater the leverage.

Multiple Choice

1. Horizontal analysis can be described as which of the following? a. Percentage changes in various financial statement amounts from year

to year b. The changes in individual financial statement amounts as a percent-

age of some related total c. The change in key financial statement ratios over a certain time

frame or horizon d. None of the above

2. Trend percentages can be considered a form of which of thefollowing?a. Ratio analysis b. Vertical analysis c. Profitability analysis d. Horizontal analysis

1eSG_C14_0131792075.Qxd 10/23/06 9:36 AM Page 493

Page 526: polpoa_sg

494 Chapter 14 | Quick Practice Questions

3. In 2008, net sales were $1,600,000 and in 2009, net sales were$1,750,000. How is the percent change calculated? a. Divide $1,600,000 by $1,750,000 b. Divide $1,750,000 by $1,600,000 c. Divide $150,000 by $1,750,000 d. Divide $150,000 by $1,600,000

4. Vertical analysis can be described as which of the following? a. Percentage changes in the balances shown in comparative financial

statements b. The change in key financial statement ratios over a specified period of

time c. The dollar amount of the change in various financial statement

amounts from year to year d. Individual financial statement items expressed as a percentage of a

base (which represents 100%)

5. What is the base that is used when performing vertical analysison an income statement?a. Net sales b. Gross sales c. Gross profit d. Total expenses

6. What is the base that is used when performing vertical analysison a balance sheet?a. Total assets b. Stockholders’ equity c. Total liabilities d. Net assets

7. Which ratio measures the ability to pay long-term debt? a. Rate of return on net salesb. Earnings per sharec. Times-interest-earned ratiod. Acid-test (quick) ratio

8. Which of the following would be most helpful in the comparisonof different-sized companies?a. Performing horizontal analysis b. Looking at the amount of income earned by each company c. Comparing working capital balances d. Preparing common-size financial statements

9. Which ratio(s) help(s) in the analysis of working capital? a. Current ratio b. Acid-test ratio c. Debt ratio d. Both a and b are correct

10. Assume that collections from customers on account are beingreceived faster. Which of the following would be true?a. The accounts receivable turnover would be higherb. The days’ sales in receivables would be higher c. The current ratio would be higher d. None of the above

1eSG_C14_0131792075.Qxd 10/23/06 9:36 AM Page 494

Page 527: polpoa_sg

Quick Practice Questions | Chapter 14 495

Quick Exercises

14-1. Selected items from the balance sheet and income statementare shown as follows for the Brothers Company for 2007 and2008. Calculate the amount of the change and the percentageof change for each item.

2008 2007 $ Change % Change

Cash $121,000 $100,000

Accounts receivable 117,000 125,000

Merchandise inventory 70,000 85,000

Accounts payable 63,500 50,000

Sales 144,000 135,000

Cost of goods sold 74,000 67,500

14-2. The income statement for Commerce Corporation for the yearended December 31, 2007, follows.

Net salesExpenses Cost of goods sold General expenses Selling expenses Interest expense Income tax expenseTotal expensesNet income

$268,50049,30045,00035,00030,000

$661,000

427,800$233,200

COMMERCE CORPORATIONIncome Statement

Year Ended December 31, 2007

Net salesExpensesCost of goods soldGeneral expensesSelling expensesInterest expenseIncome tax expenseTotal expensesNet income

COMMERCE CORPORATIONVertical Analysis Income Statement For theYear Ended December 31, 2007

Amount Percentage

a. Prepare a vertical analysis of the income statement showing appro-priate percentages for each item listed in the income statement.

1eSG_C14_0131792075.Qxd 10/23/06 9:36 AM Page 495

Page 528: polpoa_sg

496 Chapter 14 | Quick Practice Questions

b. What additional information would you need to determine whetherthese percentages are good or bad?

14-3. Financial ratios and analytical functions are listed as follows.Match the function with the appropriate ratio.

Functions:

a. Gives the amount of net income earned for each share of the company’s common stock

b. Measures the number of times operating income can cover interest expense

c. Shows ability to pay all current liabilities if they come due immediately

d. Shows the percentage of a stock’s market value returned to stockholders as dividends each period

e. Measures ability to collect cash from credit customers f. Measures ability to pay current liabilities with current assets g. Indicates the market price of $1 of earnings h. Measures the difference between current assets and current

liabilitiesi. Indicates percentage of assets financed with debt j. Shows the percentage of each sales dollar earned as net income

Ratios:

1. ________ Dividend yield 2. ________ Rate of return on net sales 3. ________ Accounts receivable turnover 4. ________ Working capital5. ________ Debt ratio 6. ________ Current ratio 7. ________ Price/earnings ratio 8. ________ Times-interest-earned ratio 9. ________ Acid-test ratio

10. ________ Earnings per share of common stock

1eSG_C14_0131792075.Qxd 10/23/06 9:36 AM Page 496

Page 529: polpoa_sg

Quick Practice Questions | Chapter 14 497

14-4. Following are the data for Dream Corporation for 2008:

Market price per share of common stock at 12/31/08 $ 9.00

Net income $50,000.00

Number of common shares outstanding 25,000.00

Dividend per share of common stock $ 0.71

Using the data, calculate the following ratios for DreamCorporation:

a. Earnings per share of common stock

b. Price/earnings ratio

c. Dividend yield

1eSG_C14_0131792075.Qxd 10/23/06 9:36 AM Page 497

Page 530: polpoa_sg

14-5. Following are selected data from the comparative incomestatement and balance sheet for Deerfield Corporation for theyears ended December 31, 2008 and 2007:

2008 2007

Net sales (all on credit) $97,600 $93,000

Cost of goods sold 53,500 52,500

Gross profit 44,700 40,500

Income from operations 16,300 15,000

Interest expense 3,100 3,500

Net income 9,800 9,000

Cash 7,700 7,500

Accounts receivable, net 10,700 12,500

Inventory 20,000 26,000

Prepaid expenses 1,000 900

Total current assets 39,400 46,900

Total long-term assets 50,000 67,000

Total current liabilities 32,000 44,500

Total long-term liabilities 11,000 39,800

Common stock, no par* 10,000 10,000

Retained earnings 25,400 19,600

*Note: 2,000 shares of common stock have been issued and outstanding since the companystarted operations. During the entire fiscal year ended December 31, 2008, the stock wasselling for $45 per share.

Calculate the following ratios at December 31, 2008:

a. Acid-test ratio

b. Inventory turnover

c. Days’ sales in receivables

d. Price/earnings ratio

e. Rate of return on total assets

f. Times-interest-earned ratio

g. Current ratio

h. Debt ratio

1eSG_C14_0131792075.Qxd 10/23/06 9:36 AM Page 498

Page 531: polpoa_sg

Do It Yourself! Question 1 | Chapter 14 499

Do It Yourself! Question 1

Tykes, Inc., had the following information at December 31, 2008:

AssetsCashAccounts receivableInventoryTotal assetsLiabilitiesAccounts payableLoans payableTotal liabilitiesStockholder’s equityCommon stockRetained earningsTotal stockholder’s equityTotal liabilities and stockholder’s equity

$400290150

$840

$140450

$590

40210

$250$840

$300350220

$870

$ 75600

$675

40155

$195$870

2008(Dollar amounts in millions) 2007

TYKES, INC.Comparative Balance Sheet December 31, 2008 and 2007

Sales revenueLess: Cost of goods soldGross profit

Insurance expenseInterest expenseNet income

$1,200800

$ 400

20060

$ 140

$1,000600

$ 400

19080

$ 130

2008 2007

TYKES INC.Comparative Income Statement

Years Ended December 31, 2008 and 2007

At December 31, 2006, Tykes’ inventory was $200,000,000 and total equitywas $165,000,000.

1eSG_C14_0131792075.Qxd 10/23/06 9:36 AM Page 499

Page 532: polpoa_sg

500 Chapter 14 | Do It Yourself! Question 1

AssetsCashAccounts receivableInventoryTotal assetsLiabilitiesAccounts payableLoans payableTotal liabilitiesStockholder’s equityCommon stockRetained earningsTotal stockholder’s equityTotal liabilities and stockholder’s equity

$400290150

$840

$140450

$590

40210

$250$840

$300350220

$870

$ 75600

$675

40155

$195$870

2008(Dollar amounts in millions) 2007 Increase (Decrease)Amount Percent

TYKES, INC.Horizontal Analysis of Comparative Balance Sheet

December 31, 2008 and 2007

Sales revenueLess: Cost of goods soldGross profit

Insurance expenseInterest expenseNet income

$1,200800

$ 400

20060

$ 140

$1,000600

$ 400

19080

$ 130

2008(Dollar amounts in millions) 2007 (Decrease)PercentAmount

Increase

TYKES, INC.Horizontal Analysis of Comparative Income Statement

Years Ended December 31, 2008 and 2007

Perform a horizontalanalysis of comparativefinancial statements

1 Requirements

1. Prepare a horizontal analysis of Tykes’ financial statements.

1eSG_C14_0131792075.Qxd 10/23/06 9:36 AM Page 500

Page 533: polpoa_sg

Do It Yourself! Question 1 | Chapter 14 501

3. Calculate Tykes’ inventory turnover and rate of return on stockholders’equity ratios for both years.

AssetsCashAccounts receivableInventoryTotal assetsLiabilitiesAccounts payableLoans payableTotal liabilitiesStockholder’s equityCommon stockRetained earningsTotal stockholder’s equityTotal liabilities and stockholder’s equity

$400290150

$840

$140450

$590

$ 40210

$250$840

2008(Dollar amounts in millions) 2008%

$300350220

$870

$ 75600

$675

$ 40155

$195$870

2007 2007%

TYKES, INC.Vertical Analysis of Comparative Balance Sheet

December 31, 2008 and 2007

Net sales revenueLess: Cost of goods soldGross profit

Insurance expenseInterest expenseNet income

$1,200800

$ 400

20060

$ 140

2008(Dollar amounts in millions) 2008%$1,000

600$ 400

19080

$ 130

2007 2007%

TYKES, INC.Vertical Analysis of Comparative Income Statement

Years Ended December 31, 2008 and 2007

Perform a verticalanalysis of financialstatements

2

Prepare and use common-size financialstatements

3

Compute the stan-dard financial ratios

4

2. Prepare a vertical analysis of Tykes’ financial statements.

1eSG_C14_0131792075.Qxd 10/23/06 9:36 AM Page 501

Page 534: polpoa_sg

Quick Practice SolutionsTrue/False

T 1. It is generally considered more useful to know the percentage change in financial statement amounts from year to year than to know the absolute dollar amount of their change. (p. 749)

T 2. Benchmarking may be done against an industry average or against a key competitor. (p. 753)

F 3. Vertical analysis of financial statements reveals changes in items on the financial statements over time.False–Horizontal analysis of financial statements reveals changes in items on the financial statements over time. (p. 749)

F 4. Inventory turnover is the ratio of average inventory to cost of goods sold.False–Inventory turnover is the ratio of cost of goods sold to average inventory. (p. 760)

T 5. Dividend yield is the ratio that measures the percentage of a stock’s market value that is returned annually as dividends. (p. 763)

F 6. A high current ratio means that a company’s current assets represent a relatively large portion (or ratio) of total liabilities.False–A high current ratio means that a company’s current assets represent a relatively large portion (or ratio) of total current liabilities. (p. 757)

F 7. The debt ratio measures the ability to pay current liabilities.False–The debt ratio measures the percentage of assets financed with debt. (p. 761)

F 8. The acid-test (quick) ratio includes the sum of Cash, Net Accounts Receivable, and Inventory accounts in the numerator.False–The acid-test (quick) ratio includes the sum of Cash, Short-Term Investments, and Net Receivables in the numerator. (p. 757)

F 9. Earnings per share indicate the net income earned for each share of common and preferred stock.False–Earnings per share indicate the net income earned for each share of the company’s common stock. (p. 759)

T 10. There is a direct relationship between leverage and the debt ratio;the higher the debt ratio, the greater the leverage. (p. 757)

502 Chapter 14 | Quick Practice Solutions

1eSG_C14_0131792075.Qxd 10/23/06 9:36 AM Page 502

Page 535: polpoa_sg

Quick Practice Solutions | Chapter 14 503

Multiple Choice

1. Horizontal analysis can be described as which of the following? (p. 749)a. Percentage changes in various financial statement amounts from year

to yearb. The changes in individual financial statement amounts as a percent-

age of some related total c. The change in key financial statement ratios over a certain time

frame or horizon d. None of the above

2. Trend percentages can be considered a form of which of thefollowing? (p. 751)a. Ratio analysis b. Vertical analysis c. Profitability analysis d. Horizontal analysis

3. In 2008, net sales were $1,600,000 and in 2009, net sales were$1,750,000. How is the percent change calculated? (p. 750)a. Divide $1,600,000 by $1,750,000 b. Divide $1,750,000 by $1,600,000 c. Divide $150,000 by $1,750,000 d. Divide $150,000 by $1,600,000

4. Vertical analysis can be described as which of the following? (p. 752)a. Percentage changes in the balances shown in comparative financial

statements b. The change in key financial statement ratios over a specified period of

time c. The dollar amount of the change in various financial statement

amounts from year to year d. Individual financial statement items expressed as a percentage of a

base (which represents 100%)

5. What is the base that is used when performing vertical analysison an income statement? (p. 752)a. Net salesb. Gross sales c. Gross profit d. Total expenses

6. What is the base that is used when performing vertical analysison a balance sheet? (p. 752)a. Total assetsb. Stockholders’ equity c. Total liabilities d. Net assets

1eSG_C14_0131792075.Qxd 10/23/06 9:36 AM Page 503

Page 536: polpoa_sg

504 Chapter 14 | Quick Practice Solutions

7. Which ratio measures the ability to pay long-term debt? (p. 762)a. Rate of return on net salesb. Earnings per sharec. Times-interest-earned ratiod. Acid-test (quick) ratio

8. Which of the following would be most helpful in the comparisonof different-sized companies? (p. 754)a. Performing horizontal analysis b. Looking at the amount of income earned by each company c. Comparing working capital balances d. Preparing common-size financial statements

9. Which ratio(s) help(s) in the analysis of working capital? (p. 757)a. Current ratio b. Acid-test ratio c. Debt ratio d. Both a and b are correct

10. Assume that collections from customers on account are beingreceived faster. Which of the following would be true? (p. 760)a. The accounts receivable turnover would be higherb. The days’ sales in receivables would be higher c. The current ratio would be higher d. None of the above

1eSG_C14_0131792075.Qxd 10/23/06 9:36 AM Page 504

Page 537: polpoa_sg

Quick Practice Solutions | Chapter 14 505

Quick Exercise Solutions

14-1. Selected items from the balance sheet and income statementare shown as follows for the Brothers Company for 2007 and2008. Calculate the amount of the change and the percentageof change for each item. (p. 749)

2008 2007 $ Change % Change

Cash $121,000 $100,000 $21,000 21.0%

Accounts receivable 117,000 125,000 (8,000) (6.4)

Merchandise 70,000 85,000 (15,000) (17.6)inventory

Accounts payable 63,500 50,000 13,500 27.0

Sales 144,000 135,000 9,000 6.7

Cost of goods sold 74,000 67,500 6,500 9.6

14-2. The income statement for Commerce Corporation for the yearended December 31, 2007, follows: (p. 752)

Net salesExpenses Cost of goods sold General expenses Selling expenses Interest expense Income tax expenseTotal expensesNet income

$268,50049,30045,00035,00030,000

$661,000

427,800$233,200

COMMERCE CORPORATIONIncome Statement

For theYear Ended December 31, 2007

a. Prepare a vertical analysis of the income statement showing appro-priate percentages for each item listed in the income statement.

Net salesExpensesCost of goods soldGeneral expensesSelling expensesInterest expenseIncome tax expenseTotal expensesNet income

$661,000

268,50049,30045,00035,00030,000

427,800$233,200

100.0

40.67.56.85.34.5

64.735.3

%

%

COMMERCE CORPORATIONVertical Analysis Income Statement For theYear Ended December 31, 2007

Amount Percentage

1eSG_C14_0131792075.Qxd 10/23/06 9:36 AM Page 505

Page 538: polpoa_sg

506 Chapter 14 | Quick Practice Solutions

b. What additional information would you need to determine whetherthese percentages are good or bad?Additional information to determine whether these percentages aregood or bad might include:• industry averages to compare to Commerce Corporation, and• the change in each line item percentage over a relevant period of

time

14-3. Financial ratios and analytical functions are listed as follows.Match the function with the appropriate ratio. (pp. 756–762)

Functions:

a. Gives the amount of net income earned for each share of the com-pany’s common stock

b. Measures the number of times operating income can cover interestexpense

c. Shows ability to pay all current liabilities if they come due immediately

d. Shows the percentage of a stock’s market value returned to stock-holders as dividends each period

e. Measures ability to collect cash from credit customers f. Measures ability to pay current liabilities with current assets g. Indicates the market price of $1 of earnings h. Measures the difference between current assets and current liabilitiesi. Indicates percentage of assets financed with debt j. Shows the percentage of each sales dollar earned as net income

Ratios:

1. d Dividend yield 2. j Rate of return on net sales 3. e Accounts receivable turnover 4. h Working capital 5. i Debt ratio 6. f Current ratio 7. g Price/earnings ratio 8. b Times-interest-earned ratio 9. c Acid-test ratio

10. a Earnings per share of common stock

14-4. Following are the data for Dream Corporation for 2008:(pp. 759–763)

Market price per share of common stock at 12/31/08 $ 9.00

Net income $50,000

Number of common shares outstanding 25,000

Dividend per share of common stock $ 0.71

1eSG_C14_0131792075.Qxd 10/23/06 9:36 AM Page 506

Page 539: polpoa_sg

Quick Practice Solutions | Chapter 14 507

Using the data, calculate the following ratios for DreamCorporation:

a. Earnings per share of common stock $50,000/25,000 = $2.00

b. Price/earnings ratio $9.00/$2.00 = 4.5

c. Dividend yield $0.71/$9.00 = 0.08

14-5. Following are selected data from the comparative incomestatement and balance sheet for Deerfield Corporation for theyears ended December 31, 2008 and 2007: (pp. 756–762)

2008 2007

Net sales (all on credit) $97,600 $93,000

Cost of goods sold 53,500 52,500

Gross profit 44,700 40,500

Income from operations 16,300 15,000

Interest expense 3,100 3,500

Net income 9,800 9,000

Cash 7,700 7,500

Accounts receivable, net 10,700 12,500

Inventory 20,000 26,000

Prepaid expenses 1,000 900

Total current assets 39,400 46,900

Total long-term assets 50,000 67,000

Total current liabilities 32,000 44,500

Total long-term liabilities 11,000 39,800

Common stock, no par* 10,000 10,000

Retained earnings 25,400 19,600

*NOTE: 2,000 shares of common stock have been issued and outstanding since the company started operations. During the entire fiscal year ended December 31, 2008, the stock was selling for $45 per share.

Calculate the following ratios at December 31, 2008:

a. Acid-test ratio

($7,700 # $10,700)/$32,000 ! 0.58

b. Inventory turnover

c. Days’ sales in receivables

($10,700 # $12,500)/2! 43.4 days

($97,500/365)

$53,500! 2.33

($20,000 # $26,000)/2

1eSG_C14_0131792075.Qxd 10/23/06 9:36 AM Page 507

Page 540: polpoa_sg

508 Chapter 14 | Quick Practice Solutions

d. Price/earnings ratio

$45! 9.18

($9,800 # 2000)

e. Rate of return on total assets

$9.800 # $3,100 ! 0.13($39,400 # $50,000 # $46,900 # $67,000)/2

f. Times-interest-earned ratio

$16,300! 5.26 times

$3,100

g. Current ratio

$39,400! 1.23

$32,000

h. Debt ratio

$ 32,000 # $11,000! 0.48

$ 39,400 # $50,000

1eSG_C14_0131792075.Qxd 10/23/06 9:36 AM Page 508

Page 541: polpoa_sg

Do It Yourself! Question 1 Solutions | Chapter 14 509

AssetsCashAccounts receivableInventoryTotal assetsLiabilitiesAccounts payableLoans payableTotal liabilitiesStockholder’s equityCommon stockRetained earningsTotal stockholder’s equityTotal liabilities and stockholder’s equity

$400290150

$840

$140450

$590

40210

$250$840

$300350220

$870

$ 75600

$675

40155

$195$870

$100(60)(70)

$ (30)

$ 65(150)$ (85)

055

$ 55$ (30)

33.3(17.1)(31.8)(3.4)

86.7(25.0)(12.6)

0.035.528.2(3.4)

%

%

%

%%

2008(Dollar amounts in millions) 2007 (Decrease)Percent

IncreaseAmount

TYKES, INC.Horizontal Analysis of Comparative Balance Sheet

December 31, 2008 and 2007

Sales revenueLess: Cost of goods soldGross profit

Insurance expenseInterest expenseNet income

$1,200800

$ 400

20060

$ 140

$1,000600

$ 400

19080

$ 130

$200200

$ 0

10(20)

$ 10

20.033.3

0.0

5.3(25.0)

7.7

%

%

%

2008(Dollar amounts in millions) 2007 (Decrease)Percent

IncreaseAmount

TYKES, INC.Horizontal Analysis of Income Statement

Years Ended December 31, 2008 and 2007

Do It Yourself! Question 1 Solutions

Requirement 1

Prepare a horizontal analysis of Tykes’ financial statements.

1eSG_C14_0131792075.Qxd 10/23/06 9:36 AM Page 509

Page 542: polpoa_sg

510 Chapter 14 | Do It Yourself! Question 1 Solutions

AssetsCashAccounts receivableInventoryTotal assetsLiabilitiesAccounts payableLoans payableTotal liabilitiesStockholder’s equityCommon stockRetained earningsTotal stockholder’s equityTotal liabilities and stockholder’s equity

$400290150

$840

$140450

$590

$ 40210

$250$840

47.634.517.9

100.0

16.753.570.2

4.825.029.8

100.0

%

%

%

%

%%

34.540.225.3

100.0

8.669.077.6

4.617.822.4

100.0

%

%

%

%

%%

2008(Dollar amounts in millions) 2008%

$300350220

$870

$ 75600

$675

$ 40155

$195$870

2007 2007%

TYKES, INC.Vertical Analysis of Comparative Balance Sheet

December 31, 2008 and 2007

Net sales revenueLess: Cost of goods soldGross profit

Insurance expenseInterest expenseNet income

$1,200800

$ 400

20060

$ 140

2008(Dollar amounts in millions) 2008%$1,000

600$ 400

19080

$ 130

100.066.733.3

16.65.0

11.7

%

%

%

100.060.040.0

19.08.0

13.0

%

%

%

2007 2007%

TYKES, INC.Vertical Analysis of Comparative Income Statement

Years Ended December 31, 2008 and 2007

Requirement 2

Prepare a vertical analysis of Tykes’ financial statements.

1eSG_C14_0131792075.Qxd 10/23/06 9:36 AM Page 510

Page 543: polpoa_sg

Do It Yourself! Question 1 Solutions | Chapter 14 511

Requirement 3

Calculate Tykes’ inventory turnover and rate of return on stockholders’ equityratios for both years.

2008InventoryTurnover

$800,000,000($220,000

!,,000 150,000,000)/2

2007Inventor

#

!

$

4.3 times

yyTurnover

$600,000,000($200,000,000 220,0

!#$ 000,000)/2

2008 Rate of Return onS

! 2.9 times

ttockholders Equity$140,000,000 $0)

($195,�’!

"

0000,000 $250,000,000)/2

2007 Rate of

#

! 62.9%

RReturn onStockholders Equity

$130,000,000�’

!""

#

!

$0($165,000,000 $195,000,000)/2

72.2%

1eSG_C14_0131792075.Qxd 10/23/06 9:36 AM Page 511

Page 544: polpoa_sg

512 Chapter 1 | The Power of Practice

The Power of PracticeFor more practice using the skills learned in this chapter, visit MyAccountingLab.There you will find algorithmically generated questions that are based on theseDemo Docs and your main textbook’s Review and Accounting Practice sections.

To go to MyAccountingLab and follow these steps:

1. Direct your URL to www.myaccountinglab.com.2. Log in using your name and password.3. Click the MyAccountingLab link.4. Click Study Plan in the left navigation bar.5. From the table of contents, select Chapter 14, Financial Statement Analysis.6. Click a link to work on the tutorial exercises.

1eSG_C14_0131792075.Qxd 10/23/06 9:36 AM Page 512

Page 545: polpoa_sg

Introduction to ManagementAccounting

15WHAT YOU PROBABLY ALREADY KNOW

If you have ever baked a cake, you probably already know thatcertain ingredients are required to produce the desired result. It maybe a mix that only requires eggs and water, or you may follow a recipewhere you must add in all of the ingredients separately. Whichever itmay be, you know that the more ingredients that are necessary tomake the cake, the more costly it is. Assume you ask your sister toapply the icing because you’re short on time and offer to pay her $5.The amount paid for her services, labor, adds to the cost of the cake. Itis certain that without utilities to run the mixer and the oven, youcould not make the cake. In business, these inputs would be referredto as overhead. So, it seems that the costs of materials, labor, andoverhead are involved in making the cake. In this chapter, we willstudy these three components of cost for manufacturers.

Learning Objectives

Identify trends in the business environment and the role ofmanagement accountability.

Some of the changes that have taken place over recent years include anincreasing shift toward a service economy, global competition andincreasing opportunities for worldwide expansion, time-based competition(including ERP, e-commerce, and JIT management), and an increased focuson promoting continuous improvement in the quality of goods andservices produced (total quality management). Review the section called“Today’s Business Environment” in the main text, as well as management’saccountability to stakeholders in Exhibit 15-2 (p. 803).

Distinguish management accounting from financial accounting.

Management accounting provides financial and nonfinancial informationto managers and other internal users of information. The data helpmanagement plan and control the operations of the business. Financialaccounting provides financial information to users outside of thebusiness such as creditors, investors, and governmental agencies.Review Management Accounting versus Financial Accounting in Exhibit15-3 (p. 804).

Classify costs and prepare an income statement for a servicecompany.

All of the costs of a service company are considered period costs. Periodcosts include selling, general, and administrative costs that are includedas expenses on the income statement in the period incurred. A service

1

2

3

1eSG_C15_0131792075.Qxd 10/24/06 1:58 PM Page 513

Page 546: polpoa_sg

514 Chapter 15 | Introduction to Management Accounting

4

company has no inventory, and therefore none of the costs incurred areinventoriable. Similarly, no cost of goods sold appears on the incomestatement. Review the income statement for a service company inExhibit 15-4 (p. 807).

Classify costs and prepare an income statement for amerchandising company.

Some costs are inventoriable product costs, included in the cost ofinventory on the balance sheet, until sold. These costs include the totalcost of purchasing the inventory, plus the freight that may be required toobtain the goods. When the inventory is sold, it becomes cost of goodssold, one of the largest expenses on the income statement for amerchandising company. Recall from Chapter 5 the calculation of cost ofgoods sold for a periodic inventory system is to add beginning inventory,purchases, and freight-in and subtract ending inventory. Review theincome statement for a merchandising company in Exhibit 15-5 (p. 808).

Classify costs and prepare an income statement for a manufacturingcompany.

Manufacturers use three stages and accounts for inventory: rawmaterials (including the components, ingredients, or parts used inmanufacturing), work in process (including raw materials that havesome degree of work done but not completed), and finished goods(completed and ready for sale). The inventoriable product costs infinished goods for a manufacturing company include the elements ofcost required to make the goods, including direct materials, direct labor,and manufacturing overhead.

As described under Objective 4, merchandisers consider the cost ofpurchases and freight-in to determine the cost of goods sold. Becausemanufacturers don’t purchase their inventory, the cost of the goodsmanufactured must be considered to calculate the cost of goods sold formanufacturers. The approach, similar to that used to calculate the costof goods sold for a merchandiser, is to start with beginning inventory,add direct materials, labor, and manufacturing overhead, and subtractending inventory. Review the inventoriable product and period costsanalysis in Exhibit 15-8 (p. 811). Also, review the income statement for amanufacturing company in Exhibit 15-7 (p. 811) and carefully study theschedule of cost of goods manufactured, which can be particularlytroublesome, in Exhibit 15-10 (p. 813).

Use reasonable standards to make ethical judgments.

It is more important than ever to be mindful of making ethicaljudgments. Professional accounting associations have standards ofethical conduct, as do most other professions. In addition, employeesare often provided with a code of ethics from their employer.Management accountants are required to maintain their professionalcompetence, preserve the confidentiality of the information theyhandle, and act with integrity and objectivity. Review the excerpt of theInstitute of Management Accountants’ Standards of ethical conduct inExhibit 15-12 (p. 815).

6

5

1eSG_C15_0131792075.Qxd 10/24/06 1:58 PM Page 514

Page 547: polpoa_sg

Demo Doc 1 | Chapter 15 515

*Assume that 100% of Sales Revenue isfrom selling tanning spray.

Demo Doc 1Introduction to Management AccountingLearning Objectives 1–4

Dark Spray Tanning hired you as its new management accountant. Data for themonth ended April 30 are as follows:

Wages expense $ 8,000

Supply expense 3,000

Utility expense 2,000

Rent expense 1,000

Service revenue 17,000

Requirements

1. Your new boss has heard of the term management accountability, but doesn’treally understand what it means. Explain the concept of managementaccountability.

2. Explain to your boss the scope of information you can produce as amanagement accountant.

3. Prepare an income statement for Dark Spray Tanning for the month endedApril 30, 2010.

4. In the month of May, Dark Spray decided to sell tanning spray. Based on thefollowing data, prepare Dark Spray’s income statement for the monthended May 31, 2010.

Wages expense $11,000Supply expense 3,000Utility expense 2,000Rent expense 1,000Sales revenue 30,000*

Purchases 15,000Ending inventory 6,000

5. Suppose Dark Spray sold 900 bottles of tanning spray in the month of May.What is its cost per bottle?

1eSG_C15_0131792075.Qxd 10/24/06 1:58 PM Page 515

Page 548: polpoa_sg

516 Chapter 15 | Demo Doc 1 Solutions

1 Identify trends in thebusiness environmentand the role of man-agement accountability

1 Distinguish manage-ment accounting fromfinancial accounting

Part 1 Part 2 Part 3 Part 4 Part 5 Demo DocComplete

Part 1 Part 2 Part 3 Part 4 Part 5 Demo DocComplete

Demo Doc 1 SolutionsRequirement 1

Your new boss has heard of the term management accountability, but doesn’treally understand what it means. Explain the concept of managementaccountability.

Management accountability is the idea that the manager is responsible to man-age the resources of the organization. Many different groups and individuals,called stakeholders, have an interest in an organization (that is, owners, credi-tors, customers, suppliers, and the various government organizations). To satisfythe needs of these stakeholders, managers provide information that communi-cates the decisions made and the results obtained from these decisions.Management accountability requires two forms of accounting:

• Financial accounting for external reporting

• Management accounting for internal planning, controlling, and decisionmaking

Remember, planning means choosing goals and deciding how to achievethem. Controlling means evaluating the results of business operations by com-paring the actual results to the plan.

Requirement 2

Explain to your boss the scope of information you can produce as amanagement accountant.

Management accounting provides more detailed and timely information thandoes financial accounting. Managers use this information in the following ways:

• Identify ways to cut costs

• Set prices that will be competitive and yet yield profits

• Identify the most profitable products and customers so the sales force canfocus on key profit makers

• Evaluate employees’ job performance

1eSG_C15_0131792075.Qxd 10/24/06 1:58 PM Page 516

Page 549: polpoa_sg

Demo Doc 1 Solutions | Chapter 15 517

Service revenue $17,000 100% Expenses: Wages expense $ 8,000 Supply expense 3,000 Utility expense 2,000 Rent expense 1,000 Total expenses 14,000 82% Operating income $ 3,000 18%

DARK SPRAY TANNINGIncome Statement

Month Ended April 30, 2010

As you recall from earlier chapters, an income statement reports the organiza-tion’s revenues and expenses for a period of time.

Dark Spray is a service company, so all of its costs are period costs. Because itdoesn’t carry an inventory of product, no cost of goods sold appears on its incomestatement. Period costs are costs incurred and expensed in the current account-ing period.

Dark Spray’s period costs include all expenses for the month of April (wages,supplies, utilities, and rent). Dark Spray’s period costs are $8,000 + $3,000 +$2,000 + $1,000 = $14,000 for the month of April 2010.

Here is the income statement for Dark Spray for the month of April 2010.

3 Classify costs and pre-pare an income state-ment for a servicecompany

*Assume that 100% of Sales Revenue isfrom selling tanning spray.

Notice that Dark Spray includes no cost of goods sold because it is a servicecompany. Its largest expense on the income statement is for employee salaries.They also show an 18% profit margin for the month of April.

Requirement 4

In the month of May, Dark Spray decided to sell tanning spray. Based on thefollowing data, prepare Dark Spray’s income statement for the month endedMay 31, 2010.

Wages expense $11,000

Supply expense 3,000

Utility expense 2,000

Rent expense 1,000

Sales revenue 30,000*

Purchases 15,000

Ending inventory 6,000

Part 1 Part 2 Part 3 Part 4 Part 5 Demo DocComplete

Requirement 3

Prepare an income statement for Dark Spray Tanning for the month ended April 30,2010.

1eSG_C15_0131792075.Qxd 10/24/06 1:58 PM Page 517

Page 550: polpoa_sg

518 Chapter 15 | Demo Doc 1 Solutions

Because Dark Spray has purchased products from a supplier that they nowresell, Dark Spray is now a merchandiser (retailer). Dark Spray now maintainsan inventory.

Dark Spray’s income statement will now include cost of goods sold as a majorexpense. Any product not sold in one period will be shown as an asset called mer-chandise inventory. Merchandise inventory is NOT an expense until it is sold—then the expense becomes cost of goods sold.

The goods available for sale are computed by adding purchases during theperiod to beginning inventory. Ending inventory is subtracted from cost of goodsavailable for sale to get cost of goods sold. Cost of goods sold is subtracted fromsales revenue to get gross profit. Consider the following formula:

Beginning Inventory! Purchases

Goods Available for Sale– Ending Inventory

Cost of Goods Sold

So in the case of Dark Spray:

Subtract $9,000 from sales revenue of $30,000 for a gross profit of $21,000.Here is Dark Spray’s income statement for the month ended May 31, 2010:

Beginning Inventory $ 0Purchases 15,00! 00

Goods Available for Sale 15,000Ending In" vventory 6,000

Cost of Goods Sold $ 9,0000

4 Classify costs andprepare an incomestatement for a mer-chandising company.

Sales revenue $30,000 100% Cost of goods sold: Beginning inventory $ 0 Purchases 15,000 Cost of goods available for sale 15,000 Ending inventory (6,000) Cost of goods sold 9,000 30% Gross profit 21,000 70% Operating expenses: Wages expense 11,000 Supply expense 3,000 Utility expense 2,000 Rent expense 1,000 17,000 57% Operating income $ 4,000 13%

DARK SPRAY TANNINGIncome Statement

Month Ended May 31, 2010

Part 1 Part 2 Part 3 Part 4 Part 5 Demo DocComplete

1eSG_C15_0131792075.Qxd 10/24/06 1:58 PM Page 518

Page 551: polpoa_sg

Demo Doc 1 Solutions | Chapter 15 519

Gross profit is what remains from sales after the cost of goods sold is sub-tracted from sales revenue. Gross profit is then used to cover operating expensesand yield a profit (or loss). Managers want to keep an eye on the gross profit per-centage, which is a measure of profitability, to make sure that it doesn’t fluctuatetoo much from period to period.

In this case, Dark Spray had a gross profit percentage of 70% and a 13%profit margin. These figures are calculated as follows:

In other words, Dark Spray retains $0.70 from each dollar of revenue for theperiod, which is used to cover their operating expenses and then generate operat-ing income.

Profit margin represents the percentage of revenue that the company keeps asearnings. In this case, Dark Spray retains $0.13 from every revenue dollar asincome. Managers watch this percentage as well to make sure that it doesn’t dropfrom period to period. Even if overall earnings increase, it’s still possible for acompany’s profit margin to diminish if, for example, costs increase at a rategreater than sales. Changes in this ratio would signal to managers that they mayneed to exercise greater control over costs.

Requirement 5

Suppose Dark Spray sold 900 bottles of tanning spray in the month of May.What is its cost per bottle?

Profit MarginGross Profit

Sales Revenue

Dark

#

Spray's Profit Margin$4,000$30,00013%

#

#

Gross Profit PercentageGross Profit

Sales R#

eevenueDark Spray's Gross Profit

Percentage #$$21,000$30,00070%#

Part 1 Part 2 Part 3 Part 4 Part 5 Demo DocComplete

Cost per unit is determined by dividing total cost by the number of units.Knowing the costs the company incurs per unit it sells helps managers makepricing decisions. They obviously want to make sure they are charging their cus-tomers enough to cover their own costs and generate a profit. To calculate thecost per unit:

Cost per UnitTotal Cost of Goods Sold

Total#

Number of Units Sold

1eSG_C15_0131792075.Qxd 10/24/06 1:58 PM Page 519

Page 552: polpoa_sg

520 Chapter 15 | Demo Doc 1 Solutions

Dark Spray wants to know its cost for each bottle of tanning spray it sells. In thiscase, the total cost of selling 900 bottles is $9,000.

Managers will use the per-unit cost information to help them make better deci-sions. Knowing that its per-unit cost is $10, Dark Spray managers may decidethey need to adjust the price.

Cost per Bottle$9,000

900 bottles$10 per b

#

# oottle

Part 1 Part 2 Part 3 Part 4 Part 5 Demo DocComplete

1eSG_C15_0131792075.Qxd 10/24/06 1:58 PM Page 520

Page 553: polpoa_sg

Demo Doc 2 | Chapter 15 521

Demo Doc 2Manufacturing CompaniesLearning Objectives 5, 6

Chase Toys produces toys for dogs. The following information was available forthe month ended November 30, 2010 (assume no beginning or ending raw mate-rials inventory).

Sales revenue $106,500

Direct materials used 32,000

Direct labor 16,000

Manufacturing overhead* 17,000

Beginning work in process 8,000

Ending work in process 6,000

Operating expenses 32,000

Finished goods, Nov. 1, 2010 4,000

Finished goods, Nov. 30, 2010 7,200

*All indirect production costs are included in ManufacturingOverhead.

Requirements

1. Prepare a schedule of cost of goods manufactured for the month endedNovember 30, 2010.

2. Prepare an income statement for the month ended November 30, 2010.

3. At an internal Chase Toys meeting, you learned that Chase Toys hasdeveloped a new product that is expected to produce record profits. BeforeChase Toys went public with this product, you advised your girlfriend toinvest in Chase Toys. Which standard of ethical conduct for managementaccountants did you violate? Explain.

1eSG_C15_0131792075.Qxd 10/24/06 1:58 PM Page 521

Page 554: polpoa_sg

522 Chapter 15 | Demo Doc 2 Solutions

Demo Doc 2 SolutionsRequirement 1

Prepare a schedule of cost of goods manufactured for the month endedNovember 30, 2010.

Part 1 Part 2 Part 3 Demo DocComplete

The cost of goods manufactured summarizes the manufacturing activities thattook place for the month of November 2010.

The three manufacturing costs—direct materials, direct labor, and manufac-turing overhead—are added together to get the total manufacturing costsincurred during November ($32,000 + $16,000 + $17,000). These costs are addedto beginning inventory to yield the total accountable manufacturing costs.Ending inventory is then subtracted from total accountable manufacturing coststo get cost of goods manufactured. Consider the following formula:

So for Chase Toys:

Here is Chase Toys’ schedule of cost of goods manufactured for the month endedNovember 30, 2010:

Beginning Work in Progress $ 3,000Direct Materials Used! 32,000Direct! Labor Used 16,000Factory/Manufacturing Ov! eerhead Applied 17,000

Current Manufacturingg Costs 73,000Endin" gg Work in Progress 6,000

Cost of Goods Manufactured $67,000

Beginning Work in ProcessDirect Materials! UUsedDirect Labor UsedFactory/Manufacturi

!

! nng Overhead Applied

Current Manufacturingg CostsEnding Work in Process"

Cost of Goods Manufactureed

5 Classify costs andprepare an incomestatement for a manu-facturing company

1eSG_C15_0131792075.Qxd 10/24/06 1:58 PM Page 522

Page 555: polpoa_sg

Demo Doc 2 Solutions | Chapter 15 523

5 Classify costs andprepare an incomestatement for a manu-facturing company

Part 1 Part 2 Part 3 Demo DocComplete

Beginning work in process inventory $ 8,000 Add: Direct materials used $32,000 Direct labor 16,000 Manufacturing overhead 17,000 Total manufacturing costs incurred during the period 65,000 Total accountable manufacturing costs 73,000 Less: Ending work in process (6,000) Cost of goods manufactured $67,000

CHASE TOYSSchedule of Cost of Goods Manufactured

Month Ended November 30, 2010

Notice the similarity between calculating cost of goods manufactured and cal-culating cost of goods sold for a merchandiser: Start with the beginning inventorybalance, increase it for the additions during the period, and subtract the endinginventory balance.

The cost of goods manufactured becomes part of the finished goods inventory,which will be shown as an asset until the period in which it is sold, when it flowsto cost of goods sold.

Note that the inventoriable product costs in finished goods for a manufactur-ing company include the elements of cost required to make the goods:

• Direct materials. Physical components required to manufacture the prod-uct, which can be traced directly to the finished good.

• Direct labor. Labor of employees who work directly on the finished product.

• Manufacturing overhead. Manufacturing costs other than direct materi-als and direct labor, including factory costs such as insurance, depreciation,and utilities. Overhead also includes indirect materials (low value materi-als that cannot be traced directly to a finished product) and indirect labor(supportive factory labor of janitors, managers, and equipment operatorsthat cannot be traced directly to a finished product).

Requirement 2

Prepare an income statement for the month ended November 30, 2010.

The Cost of Goods Manufactured account summarizes the activities that takeplace in a manufacturing plant over a period of time. It represents the manufac-turing cost of goods that Chase Toys finished during November.

Cost of goods sold is computed as follows:

Beginning Inventory $ 4,0000Cost of Goods Manufactured 67,000!

CCost of Goods Available for Sale 71,000End" iing Inventory 7,200

Cost of Goods Sold $63,800

1eSG_C15_0131792075.Qxd 10/24/06 1:58 PM Page 523

Page 556: polpoa_sg

524 Chapter 15 | Demo Doc 2 Solutions

Following is Chase Toys’ income statement for the month of November:

6 Use reasonable stan-dards to make ethicaljudgments

Part 1 Part 2 Part 3 Demo DocComplete

Part 1 Part 2 Part 3 Demo DocComplete

Sales revenue $106,500 100% Cost of goods sold: Beginning finished goods inventory $ 4,000 Cost of goods manufactured 67,000 Cost of goods available for sale 71,000 Ending finished goods inventory (7,200) Cost of goods sold 63,800 60% Gross profit 42,700 40% Operating expenses 32,000 30% Operating income $ 10,700 10%

CHASE TOYSIncome Statement

Month Ended November 30, 2010

Notice how the cost of goods manufactured amount computed on the schedulein Requirement 1 is part of finished goods here—the only inventory that is readyto sell. It becomes part of cost of goods sold on the income statement.

Note that Finished Goods for a manufacturer is like the Inventory accountfor a merchandiser. In both cases, these accounts represent the inventory that iscomplete and available to be sold.

Requirement 3

At an internal Chase Toys meeting, you learned that Chase Toys has developeda new product that is expected to produce record profits. Before Chase Toyswent public with this product, you advised your girlfriend to invest in ChaseToys. Which standard of ethical conduct for management accountants did youviolate? Explain.

Providing confidential company information to your girlfriend is a clear violationof the confidentiality standard. Employees must refrain from disclosing confi-dential information acquired in the course of work except when authorized,unless legally obligated to do so.

1eSG_C15_0131792075.Qxd 10/24/06 1:58 PM Page 524

Page 557: polpoa_sg

Quick Practice QuestionsTrue/False

_____ 1. A system that integrates all of a company’s worldwide functions,departments, and data is called supply-chain management.

_____ 2. A budget is a quantitative expression of a plan that helps managerscoordinate and implement the plan.

_____ 3. Goods that are partway through the manufacturing process, but notyet complete, are referred to as materials inventory.

_____ 4. Manufacturers use labor, plant, and equipment to convert raw materi-als into new finished products.

_____ 5. Period costs are operating costs that are expensed in the period inwhich the goods are sold.

_____ 6. Indirect labor and indirect materials are part of manufacturingoverhead.

_____ 7. Trends in the modern business environment include the shift to a ser-vice economy and the rise of the global marketplace.

_____ 8. Management has a responsibility to meet regulatory obligations to fed-eral and local government agencies.

_____ 9. Total quality management applies only to manufacturers and promotesthe creation of superior products.

_____10. The cost of goods manufactured is equal to the sum of direct materialsused, direct labor, and manufacturing overhead.

Quick Practice Questions | Chapter 15 525

1eSG_C15_0131792075.Qxd 10/24/06 1:58 PM Page 525

Page 558: polpoa_sg

Multiple Choice

1. The primary goal of financial accounting is to provideinformation to which of the following?a. Investorsb. Creditorsc. Company managersd. Both a and b

2. Which of the following is true about management accounting? a. Management accounting provides information to customers b. Management accounting provides information that is required to be

audited by certified public accountants c. Management accounting primarily focuses on reporting on the com-

pany as a whole on a quarterly or annual basisd. Management accounting is not restricted by GAAP

3. Manufacturers may have which accounts on their balancesheet? a. Materials, Work in Process, and Finished Goods b. Merchandise, Materials, and Finished Goodsc. Direct Materials, Direct Labor, and Manufacturing Overhead d. Work in Process, Materials, and Manufacturing Overhead

4. In which category would glue or fasteners to manufacture atable be included?a. Direct materialsb. Manufacturing overheadc. Period costsd. Indirect labor

5. Inventoriable product costs include which of the following?a. Marketing costs b. Costs of direct materials, direct labor, and manufacturing overhead

used to produce a product c. Costs of direct materials and direct labor used to produce a product d. Period costs, overhead, and direct labor

6. When do inventoriable costs become expenses? a. When the manufacturing process begins b. When the manufacturing process is completed c. When the direct materials are purchased d. When the units in inventory are sold

7. In which category would selling and administrative costs beincluded? a. Direct materials b. Manufacturing overhead c. Period costs d. Work in process

526 Chapter 15 | Quick Practice Questions

1eSG_C15_0131792075.Qxd 10/24/06 1:58 PM Page 526

Page 559: polpoa_sg

8. All except which of the following are manufacturing overheadcosts? a. Materials used directly in the manufacturing process of the product b. Insurance on factory equipment c. Salaries of production supervisors d. Property tax on factory building

9. Cost of goods sold for a manufacturer equals cost of goodsmanufactured plus which of the following? a. Beginning work in process inventory less ending work in process

inventory b. Ending work in process inventory less beginning work in process

inventory c. Beginning finished goods inventory less ending finished goods inventoryd. Ending finished goods inventory less beginning finished goods inventory

10. At the beginning of 2008, the Taylor Company’s Work in ProcessInventory account had a balance of $30,000. During 2008, $68,000of direct materials were used in production, and $66,000 ofdirect labor costs were incurred. Manufacturing overhead in2008 amounted to $90,000. The cost of goods manufactured was$220,000 in 2008. What is the balance in the work in processinventory on December 31, 2008? a. $34,000 b. $24,000 c. $66,000 d. $6,000

Quick Exercises

15-1. Use the correct number to categorize each item that follows:

1. Direct materials2. Selling and general expenses3. Manufacturing overhead4. Direct labor

a. __________ Rent expense on factory building b. __________ Sales supplies used c. __________ Factory supplies used d. __________ Indirect materials used e. __________ Wages of assembly line personnel f. __________ Cost of primary material used to make product g. __________ Depreciation on office equipment h. __________ Rent on office facilities i. __________ Insurance expired on factory equipment j. __________ Utilities incurred in the office

k. __________ Advertising expense l. __________ Taxes paid on factory building

Quick Practice Questions | Chapter 15 527

1eSG_C15_0131792075.Qxd 10/24/06 1:58 PM Page 527

Page 560: polpoa_sg

15-2. The Carter Company reports the following information for 2007:

Sales $70,600 Direct materials used 7,300 Depreciation on factory equipment 4,700 Indirect labor 5,900 Direct labor 11,300 Factory rent 4,200 Factory utilities 1,200 Sales salary expense 16,300 Office salary expense 8,900 Indirect materials 1,200

Compute the following:

a. Inventoriable product costs

b. Period costs

15-3. Indicate whether each of the following costs is a product costor a period cost:

a. __________ Direct materials used b. __________ Factory utilities c. __________ Salespersons’ commissions d. __________ Plant manager’s salary e. __________ Indirect materials used f. __________ Depreciation on store equipment g. __________ Indirect labor incurred h. __________ Advertising expense i. __________ Direct labor incurred j. __________ Factory machinery repairs and maintenance

k. __________ Depreciation on factory machinery l. __________ Office supplies used

m. __________ Plant insurance expired

528 Chapter 15 | Quick Practice Questions

1eSG_C15_0131792075.Qxd 10/24/06 1:58 PM Page 528

Page 561: polpoa_sg

15-4. South State Company financial information for the yearended December 31, 2008, follows:

Direct materials used $71,000Direct labor incurred 37,000Indirect labor 2,700Indirect materials used 1,600Other factory costs:Utilities 3,100Maintenance 4,500Supplies 1,800Depreciation 7,900Property taxes 2,600

South State had no beginning or ending finished goods inventory, butwork in process inventory began the year with a $5,500 balance andended the year with a $7,500 balance.

Prepare a schedule of cost of goods manufactured for South StateCompany for the year ending December 31, 2008.

Quick Practice Questions | Chapter 15 529

SOUTH STATE COMPANYSchedule of Cost of Goods Manufactured

Year Ended December 31, 2008

15-5. Briefly describe a just-in-time management philosophy.

1eSG_C15_0131792075.Qxd 10/24/06 1:58 PM Page 529

Page 562: polpoa_sg

530 Chapter 15 | Do It Yourself! Question 1

Do It Yourself! Question 1

Introduction to Management Accounting

Stay Fit Exercise Company has hired you as their new management accountant.Data for the month ended February 28 is as follows:

Wages expense 22,000Supply expense 3,000Utility expense 1,000Rent expense 4,500Service revenue 32,000

Requirements

1. Your new boss has heard of the term total quality management, butdoesn’t really understand what it means. Explain the concept of totalquality management.

2. Prepare an income statement for Stay Fit for the month ended February 28,2010.

1eSG_C15_0131792075.Qxd 11/22/06 10:34 AM Page 530

Page 563: polpoa_sg

Do It Yourself! Question 1 | Chapter 15 531

4. Suppose Stay Fit sold 375 exercise balls in the month of March.What is itscost per ball?

*Assume that 100% of Sales Revenue is from selling exercise balls.

3. In the month of March, Stay Fit decided to sell exercise balls. Based on thefollowing data, prepare Stay Fit’s income statement for the month endedMarch 31, 2010. Calculate Stay Fit’s gross profit percentage and profitmargin (round to the nearest percentage point).

Wages expense 24,000Supply expense 3,000Utility expense 1,000Rent expense 4,500Sales revenue 42,000*Purchases 8,000Ending inventory 2,000

1eSG_C15_0131792075.Qxd 10/24/06 1:58 PM Page 531

Page 564: polpoa_sg

532 Chapter 15 | Do It Yourself! Question 2

Do it Yourself! Question 2

Manufacturing CompaniesTheme Cans Company produces metal popcorn cans. The following informationwas available for the month ended August 31, 2010.

Sales $104,250Direct materials used 36,000Direct labor 25,000Manufacturing overhead 12,000Beginning work in process 5,000Ending work in process 3,000Operating expenses 28,400Finished goods, August 1, 2010 2,500Finished goods, August 30, 2010 5,200

Requirements

1. Prepare a schedule of cost of goods manufactured for the month endedAugust 31, 2010.

1eSG_C15_0131792075.Qxd 11/22/06 10:35 AM Page 532

Page 565: polpoa_sg

3. As the management accountant for Theme Cans, you are in the process ofpurchasing new software for the company. One of the suppliers of softwaregave you a brand new set of expensive golf clubs. Was the acceptance ofthe golf clubs a violation of any management accountant standard ofethical conduct? Explain.

Do It Yourself! Question 2 | Chapter 15 533

2. Prepare Theme Cans’ income statement for the month ended August 31, 2010.

1eSG_C15_0131792075.Qxd 10/24/06 1:58 PM Page 533

Page 566: polpoa_sg

534 Chapter 15 | Quick Practice Solutions

Quick Practice SolutionsTrue/False

F 1. A system that integrates all of a company’s worldwide functions,departments, and data is called supply-chain management.False–A system that integrates all of a company’s worldwide functions, departments, and data is called enterprise resource planning. (p. 805)

T 2. A budget is a quantitative expression of a plan that helps managerscoordinate and implement the plan. (p. 804)

F 3. Goods that are partway through the manufacturing process, but notyet complete, are referred to as materials inventory.False–Goods that are partway through the manufacturing process,but not yet complete, are referred to as work in process. (p. 809)

T 4. Manufacturers use labor, plant, and equipment to convert raw materials into new finished products. (p. 808)

F 5. Period costs are operating costs that are expensed in the period in which the goods are sold.False–Period costs are operating costs that are expensed in the period incurred. (p. 806)

T 6. Indirect labor and indirect materials are part of manufacturing overhead. (p. 809)

T 7. Trends in the modern business environment include the shift to a service economy and the rise of the global marketplace. (p. 805)

T 8. Management has a responsibility to meet regulatory obligations to federal and local government agencies. (p. 803)

F 9. Total quality management applies only to manufacturers and promotes the creation of superior products.False–Total quality management promotes the creation of superior products and services. It applies to entities other thanmanufacturers. (p. 806)

F 10. The cost of goods manufactured is equal to the sum of direct materials used, direct labor, and manufacturing overhead.False–The cost of goods manufactured is equal to the sum of direct materials used, direct labor, and manufacturing overhead plusbeginning work in process inventory minus ending work in process inventory. (p. 813)

1eSG_C15_0131792075.Qxd 10/24/06 1:58 PM Page 534

Page 567: polpoa_sg

Quick Practice Solutions | Chapter 15 535

Multiple Choice

1. The primary goal of financial accounting is to provideinformation to which of the following? (p. 803)a. Investorsb. Creditorsc. Company managersd. Both a and b

2. Which of the following is true about management accounting? (p. 804)a. Management accounting provides information to customers.b. Management accounting provides information that is required to be

audited by certified public accountants.c. Management accounting primarily focused on reporting on the com-

pany as a whole on a quarterly or annual basis.d. Management accounting is not restricted by GAAP

3. Manufacturers may have which accounts on their balancesheet? (pp. 808–809)a. Materials, Work in Process, and Finished Goodsb. Merchandise, Materials, and Finished Goodsc. Direct Materials, Direct Labor, and Manufacturing Overhead d. Work in Process, Materials, and Manufacturing Overhead

4. In which category would glue or fasteners to manufacture atable be included? (pp. 809–810)a. Direct materialsb. Manufacturing overheadc. Period costsd. Indirect labor

5. Inventoriable product costs include which of the following? (pp. 810–811)a. Marketing costs b. Costs of direct materials, direct labor, and manufacturing overhead

used to produce a productc. Costs of direct materials and direct labor used to produce a product d. Period costs, overhead, and direct labor

6. When do inventoriable costs become expenses? (p. 810)a. When the manufacturing process begins b. When the manufacturing process is completed c. When the direct materials are purchased d. When the units in inventory are sold

7. In which category would selling and administrative costs beincluded? (p. 811)a. Direct materials b. Manufacturing overhead c. Period costsd. Work in process

1eSG_C15_0131792075.Qxd 10/24/06 1:58 PM Page 535

Page 568: polpoa_sg

536 Chapter 15 | Quick Practice Solutions

8. All except which of the following are manufacturing overheadcosts? (pp. 809–811)a. Materials used directly in the manufacturing process of the productb. Insurance on factory equipment c. Salaries of production supervisors d. Property tax on factory building

9. Cost of goods sold for a manufacturer equals cost of goodsmanufactured plus which of the following? (p. 812)a. Beginning work in process inventory less ending work in process

inventory b. Ending work in process inventory less beginning work in process

inventory c. Beginning finished goods inventory less ending finished goods inventoryd. Ending finished goods inventory less beginning finished goods inventory

10. At the beginning of 2008, the Taylor Company’s Work in ProcessInventory account had a balance of $30,000. During 2008, $68,000of direct materials were used in production, and $66,000 ofdirect labor costs were incurred. Manufacturing overhead in2008 amounted to $90,000. The cost of goods manufactured was$220,000 in 2008. What is the balance in the work in processinventory on December 31, 2008? (p. 813)a. $34,000b. $24,000 c. $66,000 d. $6,000

1eSG_C15_0131792075.Qxd 10/24/06 1:58 PM Page 536

Page 569: polpoa_sg

Quick Practice Solutions | Chapter 15 537

Quick Exercise

15-1. Use the correct number to categorize each item that follows:(p. 809)

1. Direct materials2. Selling and general expenses3. Manufacturing overhead4. Direct labor

a. 3 Rent expense on factory building b. 2 Sales supplies used c. 3 Factory supplies used d. 3 Indirect materials used e. 4 Wages of assembly line personnel f. 1 Cost of primary material used to make product g. 2 Depreciation on office equipment h. 2 Rent on office facilities i. 3 Insurance expired on factory equipment j. 2 Utilities incurred in the office

k. 2 Advertising expense l. 3 Taxes paid on factory building

15-2. The Carter Company reports the following information for2007: (pp. 810–811)

Sales $70,600 Direct materials used 7,300 Depreciation on factory equipment 4,700 Indirect labor 5,900 Direct labor 11,300 Factory rent 4,200 Factory utilities 1,200 Sales salary expense 16,300 Office salary expense 8,900 Indirect materials 1,200

Compute the following:

a. Inventoriable product costs

$7,300 ! $4,700 ! $5,900 ! $11,300 ! $4,200 ! $1,200 ! $1,200 # $35,800

b. Period costs

$16,300 ! $8,900 # $25,200

15-3. Indicate whether each of the following costs is a product costor a period cost: (pp. 810–811)

a. product Direct materials used b. product Factory utilities c. period Salespersons’ commissions d. product Plant manager’s salary e. product Indirect materials used f. period Depreciation on store equipment g. product Indirect labor incurred

1eSG_C15_0131792075.Qxd 10/24/06 1:58 PM Page 537

Page 570: polpoa_sg

538 Chapter 15 | Quick Practice Solutions

h. period Advertising expense i. product Direct labor incurred j. product Factory machinery repairs and maintenance

k. product Depreciation on factory machinery l. period Office supplies used

m. product Plant insurance expired

15-4. South State Company financial information for the yearended December 31, 2008, follows: (p. 813)

Direct materials used $71,000Direct labor incurred 37,000Indirect labor 2,700Indirect materials used 1,600Other factory costs:Utilities 3,100Maintenance 4,500Supplies 1,800Depreciation 7,900Property taxes 2,600

South State had no beginning or ending finished goods inventory, butwork in process inventory began the year with a $5,500 balance andended the year with a $7,500 balance.

Prepare a schedule of cost of goods manufactured for South StateCompany for the year ending December 31, 2008.

Beginning work in process inventory $ 5,500 Add: Direct materials used $71,000 Direct labor 37,000 Manufacturing overhead: Indirect labor $2,700 Indirect materials 1,600 Utilities 3,100 Maintenance 4,500 Suppplies 1,800 Depreciation 7,900 Property taxes 2,600 24,200 Total manufacturing costs incurred during the year 132,200 Total manufacturing costs to account for 137,700 Less: Ending work in process inventory (7,500) Cost of goods manufactured $130,200

SOUTH STATE COMPANYSchedule of Cost of Goods Manufactured

Year Ended December 31, 2008

1eSG_C15_0131792075.Qxd 10/24/06 1:58 PM Page 538

Page 571: polpoa_sg

15-5. Briefly describe a just-in-time management philosophy. (p. 805)

In a just-in-time system, an organization purchases materials andproduces products just when they are needed in the productionprocess. Goods are not produced until it is time for them to be shippedto a customer. The goal is to have zero inventory because holdinginventory does not add value to the product. Reducing inventory andspeeding the production process reduces throughput time—the timebetween buying raw materials and selling the finished products.

Manufacturers adopting just-in-time depend on their suppliers tomake on-time deliveries of perfect-quality raw materials. JIT requiresclose communication with suppliers.

Companies that adopt JIT must strive for perfect quality. Defectsstop production lines. To avoid disrupting production, defects must be rare.

Quick Practice Solutions | Chapter 15 539

1eSG_C15_0131792075.Qxd 10/24/06 1:58 PM Page 539

Page 572: polpoa_sg

540 Chapter 15 | Do It Yourself! Question 1 Solutions

Service revenue $32,000 100% Expenses: Wages expense $ 22,000 Supply expense 3,000 Utility expense 2,000 Rent expense 4,500 Total expenses 30,500 95% Operating income $ 1,500 5%

STAY FIT EXERCISE COMPANYIncome Statement

Month Ended February 28, 2010

*Assume that 100% of Sales Revenue is fromselling exercise balls.

1 Identify trends in thebusiness environmentand the role of man-agement accountability

3 Classify costs and pre-pare an income state-ment for a servicecompany

Do It Yourself! Question 1 Solutions

Requirement 1

Your new boss has heard of the term total quality management, butdoesn’t really understand what it means. Explain the concept of totalquality management.

Total quality management is a management philosophy that promotes the goal ofproviding customers with superior products and services. Companies achieve thisgoal by continuously improving quality and reducing or eliminating defects andwaste. Companies design and build quality into their products and servicesrather than depending on finding and fixing defects later.

Requirement 2

Prepare an income statement for Stay Fit for the month ended February 28,2010.

Requirement 3

In the month of March, Stay Fit decided to sell exercise balls. Based on thefollowing data, prepare Stay Fit’s income statement for the month endedMarch 31, 2010. Calculate Stay Fit’s gross profit percentage and profitmargin (round to the nearest percentage point).

Wages expense $24,000Supply expense 3,000Utility expense 1,000Rent expense 4,500Sales revenue 42,000*Purchases 8,000Ending inventory 2,000

1eSG_C15_0131792075.Qxd 11/22/06 10:35 AM Page 540

Page 573: polpoa_sg

Do It Yourself! Question 1 Solutions | Chapter 15 541

Sales revenue $42,000 100% Cost of goods sold: Beginning inventory $ 0 Purchases 8,000 Cost of goods available for sale 8,000 Ending inventory (2,000) Cost of goods sold 6,000 14% Gross profit 36,000 86% Operating expenses: Wages expense 24,000 Supply expense 3,000 Utility expense 1,000 Rent expense 4,500 32,500 77% Operating income $ 3,500 8%

STAY FIT EXERCISE COMPANYIncome Statement

Month Ended March 31, 2010

Requirement 4

Suppose Stay Fit sold 375 exercise balls in the month of May. What is itscost per ball?

Cost per UnitTotal Cost of Goods Sold

Total!

Number of Units Sold

Cost per Ball$6,000

3!

775 Balls

$16 per Ball!

Gross Profit PercentageGross Profit

Sales R! eevenue

Stay Fit’s Gross ProfitPercentage

$3!

66,000$42,00086%

Profit MarginOperating In

!

!ccome

Sales Revenue

Stay Fit’s Profit Margin !$$3,500$42,0008%!

4 Classify costs andprepare an incomestatement for a mer-chandising company

1eSG_C15_0131792075.Qxd 11/22/06 10:35 AM Page 541

Page 574: polpoa_sg

542 Chapter 15 | Do It Yourself! Question 2 Solutions

Beginning work in process inventory $ 5,000 Add: Direct materials used $36,000 Direct labor 25,000 Manufacturing overhead 12,000 Total manufacturing costs incurred during the period 73,000 Total accountable manufacturing costs 78,000 Less: Ending work in process (3,000) Cost of goods manufactured $75,000

THEME CANSSchedule of Cost of Goods Manufactured

For the Month Ended August 31, 2010

Sales revenue $104,250 100% Cost of goods sold: Beginning finished goods inventory 2,500 Cost of goods manufactured 75,000 Cost of goods available for sale 77,500 Ending finished goods inventory (5,200) Cost of goods sold 72,300 69% Gross profit 31,950 31% Operating expenses 28,400 27% Operating income $ 3,550 3%

THEME CANSIncome Statement

Month Ended August 31 2010

5 Classify costs andprepare an incomestatement for a manu-facturing company

5 Classify costs andprepare an incomestatement for a manu-facturing company

Do It Yourself! Question 2 Solutions

Requirement 1

Prepare a schedule of cost of goods manufactured for the month endedAugust 31, 2010.

Requirement 2

Prepare Theme Cans’ income statement for the month ended August 31, 2010.

1eSG_C15_0131792075.Qxd 11/22/06 10:35 AM Page 542

Page 575: polpoa_sg

Do It Yourself! Question 2 Solutions | Chapter 15 543

6 Use reasonable stan-dards to make ethicaljudgments

Requirement 3

As the management accountant for Theme Cans, you are in the process ofpurchasing new software for the company. One of the suppliers of softwaregave you a brand new set of expensive golf clubs. Was the acceptance ofthe golf clubs a violation of any management accountant standard ofethical conduct? Explain.

The integrity standard indicates that the accountant must refuse any gift,favor, or hospitality that would influence or would appear to influence actions.The acceptance of a new set of golf clubs is a gift that would appear to influenceactions.

1eSG_C15_0131792075.Qxd 11/22/06 10:35 AM Page 543

Page 576: polpoa_sg

The Power of PracticeFor more practice using the skills learned in this chapter, visit MyAccountingLab.There you will find algorithmically generated questions that are based on theseDemo Docs and your main textbook’s Review and Accounting Practice sections.

To go to MyAccountingLab and follow these steps:

1. Direct your URL to www.myaccountinglab.com.2. Log in using your name and password.3. Click the MyAccountingLab link.4. Click Study Plan in the left navigation bar.5. From the table of contents, select Chapter 15, Introduction to Management

Accounting.6. Click a link to work on the tutorial exercises.

544 Chapter 15 | The Power of Practice

1eSG_C15_0131792075.Qxd 10/24/06 1:58 PM Page 544

Page 577: polpoa_sg

Job Order Costing16WHAT YOU PROBABLY ALREADY KNOW

If you own a car, you may have already had the unpleasant task oftaking your car to a repair shop. Then you probably already knowthat before work is performed, a cost estimate is usually stated asa certain amount for parts and an hourly amount for labor. Thehourly labor charge is much higher than the employees are paid.The charge must be sufficient to cover the overhead costs ofrunning the shop such as rent, utilities, maintenance, and supplies.An additional amount is added on top of the costs to create a profit.It is important to be able to accurately identify all of the projectedcosts and estimated hours of work that will take place to calculatethe cost per hour. We will study in this chapter how an overheadrate is calculated and allocated to jobs performed.

Learning Objectives

Distinguish between job order costing and process costing.

Job order costing accumulates costs for each unique job, assignment,or batch. A construction company, photographer, and law firm may usejob order costing because the required work may vary amongstcustomers and clients. Other companies operate by performing asimilar set of production steps or processes. These companies woulduse a process costing system, which accumulates the costs for eachprocess or department. A cereal manufacturer, bank, and automotivemanufacturer might all use process costing.

Record materials and labor transactions in a manufacturer’s joborder costing system.

A job cost record is created when the job is started. All of the costs ofproduction will be recorded on this record: direct materials used, directlabor used, and manufacturing overhead. In Demo Doc 1, you will seehow materials and labor transactions are recorded in a manufacturer’sjob order costing system. Review the “Accounting in Action—Job OrderCosting: Tracing Direct Materials and Direct Labor” in the main text.

Record manufacturing overhead transactions in a manufacturer’sjob order costing system.

Manufacturing overhead includes factory depreciation, repairs, insurance,utilities and other factory costs. The accumulation of these costs is debited

2

3

1

1eSG_C16_0131792075.Qxd 10/24/06 2:01 PM Page 545

Page 578: polpoa_sg

546 Chapter 16 | Job Order Costing

to Manufacturing Overhead and credited to the appropriate accounts, asyou will see in Demo Doc 2.

Record transactions for completion and sales of finished goodsand adjustment for under- or overallocated manufacturingoverhead.

As goods continue to be worked on and completed, the costs willtransfer from the Work in Process Inventory account to the FinishedGoods inventory account. The inventory costs remain in the FinishedGoods asset account until they are sold. When the goods are sold, twoentries are required under the perpetual inventory system, as welearned in Chapter 5. One entry is made to record the sale on account orfor cash. The other entry removes the cost of inventory and charges Costof Goods Sold. Manufacturing Overhead is debited for the actualoverhead costs and credited for the assigned overhead costs. At year-end, as you will see in Demo Doc 2, the balance in ManufacturingOverhead should be transferred into Cost of Goods Sold.

Calculate unit costs for service companies.

The costs for service companies include the labor component and theother indirect office costs. The hourly cost per employee can becalculated as follows:

Other indirect office costs may include rent, utilities, taxes, and supportsalaries. An hourly cost for indirect costs can be calculated as follows:

To determine the cost of a job, the actual number of hours applied to thejob should be multiplied by the hourly labor cost and the predeterminedindirect cost allocation rate.

PredeterminedIndirect Cost

Allocation Rate

T!

ootal Expected Indirect CostsTotal Direct Laabor Hours

Hourly Labor CostSalary and Fringe Benefit

!ss

Total Number of Hours Worked

4

5

1eSG_C16_0131792075.Qxd 10/24/06 2:01 PM Page 546

Page 579: polpoa_sg

Demo Doc 1 | Chapter 16 547

Demo Doc 1Job Order Costing for ManufacturersLearning Objectives 1, 2

Clearance Douglas manufactures specialized art for his customers. SupposeDouglas has the following transactions during March:

a. Purchased raw materials on account, $67,000.b. Materials costing $45,000 were requisitioned for production. Of this total,

$5,000 were indirect materials. c. Labor time records show that direct labor of $30,000 and indirect labor of

$2,000 were incurred, but not yet paid.

Requirements

1. Why would Douglas use the job order costing system?

2. What document would Douglas use to accumulate direct materials, directlabor, and manufacturing overhead costs assigned to each individual job?How do managers use this document to direct and control operations?

3. Prepare summary journal entries for each transaction.

1eSG_C16_0131792075.Qxd 10/24/06 2:01 PM Page 547

Page 580: polpoa_sg

548 Chapter 16 | Demo Doc 1 Solutions

Demo Doc 1 Solutions

Part 1 Part 2 Part 3 Demo DocComplete

Part 1 Part 2 Part 3 Demo DocComplete

1 Distinguish betweenjob order costing andprocess costing

2 Record materials and labor in a manufacturer’s joborder costing system

Companies that manufacture batches of unique or specialized products woulduse a job order costing system to accumulate costs for each job or batch. BecauseDouglas manufactures specialized art for his customers and the required workmay vary from customer to customer, Douglas would use the job order costingsystem. You will learn about the process costing system, which accumulates costsfor production processes as opposed to individual jobs, in Chapter 17.

Requirement 2

What document would Douglas use to accumulate direct materials, directlabor, and manufacturing overhead costs assigned to each individual job?How do managers use this document to direct and control operations?

Requirement 1

Why would Douglas use the job order costing system?

Douglas would use a job cost record to accumulate direct materials, direct labor,and manufacturing overhead costs assigned to each individual job. Managers usejob cost records to determine how much each job (and each unit in the job) coststo produce. This cost information helps managers set prices and control costs.Cost data allow managers to identify their most profitable products so that mar-keting can concentrate on selling these products. Managers also use cost data tomake outsourcing decisions and to prepare the company’s financial statements.

Managers also use the job cost record to see how they can use materials andlabor more efficiently. For example, if a job’s costs exceed its budget, managersmust do a better job controlling costs on future jobs, or raise the sale price on sim-ilar jobs, to be sure that the company remains profitable.

Similarly, managers use labor time records to control labor costs. Together,labor time records and job cost records help managers determine whetheremployees are working efficiently. If they spend longer than expected on a job, itmay not yield a profit.

Managers also use the materials inventory subsidiary ledger to controlinventory levels.

1eSG_C16_0131792075.Qxd 10/24/06 2:01 PM Page 548

Page 581: polpoa_sg

Demo Doc 1 | Chapter 16 549

2 Record materials andlabor in a manufac-turer’s job order cost-ing system

Journal Entry:

Materials Inventory 67,000 Accounts Payable 67,000 To record the purchase of materials on account.

PostDate Accounts Ref. Dr. Cr.

Journal Entry:

Work in Process Inventory (direct material) 40,000 Manufacturing Overhead (indirect material) 5,000 Materials Inventory 45,000 To record direct and indirect materials used.

PostDate Accounts Ref. Dr. Cr.

b. Materials costing $45,000 were requisitioned for production. Of this total,$5,000 were indirect materials.

When materials are requisitioned, it means that they move from materialsinventory into production to be used. You want to record this movement inthe appropriate accounts.If the materials can be directly traceable to a job, Work in Process will bedebited. If the materials do not represent a major component and cannot betraced directly to a job, they are considered indirect materials and are deb-ited to Manufacturing Overhead.Because $40,000 ($45,000 total materials less $5,000 indirect materials) ofthe materials can be traced to specific jobs, this amount goes directly intoWork in Process Inventory, increasing that asset by $40,000.The $5,000 in indirect materials is debited (an increase) to ManufacturingOverhead.Because we are taking all materials out of the Materials Inventory, wereduce this asset with a credit for the total, $45,000.

2 Record materials andlabor in a manufac-turer’s job order cost-ing system

a. Purchased raw materials on account, $67,000.

When materials are purchased on account, you want to record the increasein materials inventory, so you would debit Materials Inventory (an asset) bythe cost of the materials, $67,000.

You also want to record the increase in liability, so you would credit AccountsPayable (a liability) by $67,000 because this amount is still payable.

Part 1 Part 2 Part 3 Demo DocComplete

Requirement 3

Prepare summary journal entries for each transaction.

1eSG_C16_0131792075.Qxd 10/24/06 2:01 PM Page 549

Page 582: polpoa_sg

550 Chapter 16 | Demo Doc 1 Solutions

Part 1 Part 2 Part 3 Demo DocComplete

The manufacturing wages need to be assigned to the appropriate accounts. Alabor time record is completed by each employee who works directly on a job.Each of the jobs and hours worked are identified on the record.

In this case, some of the labor, $30,000, can be traced to specific jobs. Thisamount, called direct labor, is assigned as a debit (increase) to the assetWork in Process Inventory. The rest of the labor, $2,000, is for indirect laborsuch as maintenance and janitorial services, and cannot be traced to specificjobs. Therefore, it is debited to Manufacturing Overhead.

Manufacturing Wages is credited for the full amount (a decrease), bringingits balance to zero.

Journal Entry:

Manufacturing Wages 32,000 Wages Payable 32,000 To record the direct and indirect labor incurred.

PostDate Accounts Ref. Dr. Cr.

Journal Entry:

Work in Process Inventory (direct labor) 30,000 Manufacturing Overhead (indirect labor) 2,000 Materials Inventory 32,000 To record direct and indirect labor used.

PostDate Accounts Ref. Dr. Cr.

2 Record materials andlabor in a manufac-turer’s job order cost-ing system

c. Labor time records show that direct labor of $30,000 and indirect labor of$2,000 were incurred, but not yet paid.

First, we debit Manufacturing Wages for the full amount of labor, direct andindirect, to accumulate total labor costs. We then credit Wages Payable toshow the increased liability to our employees.

1eSG_C16_0131792075.Qxd 10/24/06 2:01 PM Page 550

Page 583: polpoa_sg

Demo Doc 2 | Chapter 1 551

Demo Doc 2Allocating Manufacturing OverheadLearning Objectives 3, 4

Macho Mike Machine Shop manufactures specialized metal products per its cus-tomers’ specifications. Macho Mike uses direct labor cost to allocate manufactur-ing overhead. Macho Mike expects to incur $160,000 of manufacturing overheadcosts and to use $400,000 of direct labor cost during 2010.

During November 2010, Macho Mike’s Machine Shop had the followingselected transactions:

a. Actual indirect manufacturing labor incurred, $4,200.

b. Actual indirect materials used, $3,000.

c. Other manufacturing overhead incurred, $2,800 (credit Accounts Payable).

d. Allocated overhead for November (the machine shop incurred $36,000 ofdirect labor cost during the month).

e. Finished jobs that totaled $6,500 on their job cost records.

f. Sold inventory for $70,000 (on account) that cost $42,000 to produce.

Requirements

1. Compute the predetermined manufacturing overhead rate for Macho Mike.

2. Journalize the transactions.

3. Prepare the journal entry to close the ending balance of manufacturingoverhead.

1eSG_C16_0131792075.Qxd 10/24/06 2:01 PM Page 551

Page 584: polpoa_sg

552 Chapter 16 | Demo Doc 2 Solutions

Demo Doc 2 Solutions

Part 1 Part 2 Part 3 Demo DocComplete

3 Record manufacturingoverhead transactionsin a job order costingsystem

3 Record manufacturingoverhead transactionsin a job order costingsystem.

Part 1 Part 2 Part 3 Demo DocComplete

a. Actual indirect manufacturing labor incurred, $4,200.

b. Actual indirect materials used, $3,000.

c. Other manufacturing overhead incurred, $2,800 (credit Accounts Payable).

In this case, all actual manufacturing overhead costs incurred during theperiod are debited to Manufacturing Overhead because they cannot betraced to any specific job (that is, they are indirect costs). So ManufacturingOverhead is debited by $4,200 + $3,000 + $2,800 = $10,000.Indirect manufacturing labor results in a decrease of $4,200 to ManufacturingWages (a credit).Indirect materials used results in a decrease of $3,000 to Materials Inventory(a credit), because the materials have been used and are therefore removedfrom materials inventory.Other manufacturing overhead of $2,800 is credited to Accounts Payable, asindicated in the question.

Requirement 1

1. Compute the predetermined manufacturing overhead rate for Macho Mike.

Because Macho Mike uses direct labor cost to allocate overhead to jobs, the prede-termined manufacturing overhead rate is computed as follows:

In this case, the estimated manufacturing overhead cost is $160,000 and esti-mated direct labor cost is $400,000. Dividing $160,000 by $400,000 equals 0.40,or a predetermined manufacturing overhead rate of 40% of actual direct laborcost. Another way to think of this is that for every $1 spent on direct labor, weincur $0.40 of manufacturing overhead.

It’s important to remember that this rate is determined at the beginning ofthe period, before any production has started because actual overhead costs andthe actual quantity of the allocation base are not known until the end of theperiod. Managers need this estimate to make decisions and allocate overhead toindividual jobs throughout the period.

Because the allocation base used may be direct labor hours, direct labor cost,machine hours, and other bases, it’s important to label this rate accordingly. Thepredetermined manufacturing overhead rate is multiplied by the allocation baseactivity to determine the amount of overhead applied to each of the jobs.

Requirement 2

Journalize the transactions.

Total Estimated Manufacturing Overhead CosttsTotal Estimated Direct Labor CostManufactturing Overhead Rate !

1eSG_C16_0131792075.Qxd 10/24/06 2:01 PM Page 552

Page 585: polpoa_sg

Demo Doc 2 Solutions | Chapter 16 553

4 Record transactions forcompletion and salesof finished goods andadjustment for under-or overallocatedoverhead

d. Allocate overhead for November (the machine shop incurred $36,000 ofdirect labor cost during the month).

To determine the total overhead allocated to jobs in November, multiply theactual direct labor cost ($36,000) by the predetermined allocation rate of40% (from Requirement 1):

Allocate the overhead to work in process by debiting Work in ProcessInventory and crediting Manufacturing Overhead by $14,400.

$36,000 (directlabor cost)

0.40 (predetermi"

nnedoverhead rate)

$14,400 allocatedoverhea! dd forNovember

Journal Entry:

Manufacturing Overhead 10,000 Manufacturing Wages 4,200 Materials Inventory 3,000 Accounts Payable 2,800

PostDate Accounts Ref. Dr. Cr.

Journal Entry:

Work in Process Inventory 14,400 Manufacturing Overhead 14,400

PostDate Accounts Ref. Dr. Cr.

e. Finished jobs that totaled $6,500 on their job cost records.

When the goods are completed, they are transferred from Work in Process toFinished Goods. This transaction reflects how work in process leaves theplant floor and is moved into the finished goods storage area.

Journal Entry:

Finished Goods Inventory 6,500 Work in Process Inventory 6,500

PostDate Accounts Ref. Dr. Cr.

f. Sold inventory for $70,000 (on account) that cost $42,000 to produce.

Debit the asset Accounts Receivable to record the increased amount of$70,000 that is owed to Macho Mike. Credit the revenue account, SalesRevenue, by the same amount.

4 Record transactions forcompletion and salesof finished goods andadjustment for under-or overallocatedoverhead

3 Record manufacturingoverhead transactionsin a job order costingsystem

1eSG_C16_0131792075.Qxd 10/24/06 2:01 PM Page 553

Page 586: polpoa_sg

554 Chapter 16 | Demo Doc 2 Solutions

Debit the expense Cost of Goods Sold to record the cost of the sale. Becausethe goods are no longer in the finished goods inventory, we must creditFinished Goods Inventory to reduce that asset account.

Journal Entry:

Accounts Receivable 70,000 Sales Revenue 70,000

Cost of Goods Sold 42,000 Finished Goods Inventory 42,000

PostDate Accounts Ref. Dr. Cr.

Part 1 Part 2 Part 3 Demo DocComplete

The balance of the Manufacturing Overhead account should be zero at the end ofthe accounting period. To zero out the account when Manufacturing Overheadhas a debit balance, you would credit Manufacturing Overhead and debit Cost ofGoods Sold. If Manufacturing Overhead has a credit balance, then you woulddebit Manufacturing Overhead and credit Cost of Goods Sold.

In this case, manufacturing overhead was overallocated, because the over-head allocated to Work in Process Inventory ($14,400 from Requirement 2, trans-action d) is more than the amount actually incurred ($4,200 + $3,000 + $2,800 =$10,000 from transactions a, b, and c).

The result is a credit balance of $4,400 in Manufacturing Overhead. To closethe ending balance, then, we debit Manufacturing Overhead by $4,400 and creditCost of Goods Sold by $4,400.

Journal Entry:

Manufacturing Overhead 4,400 Cost of Goods Sold 4,400

PostDate Accounts Ref. Dr. Cr.

Why? Because Macho Mike allocated too much manufacturing overhead toeach job, resulting in cost of goods sold being too high (meaning the jobs werecharged too much overhead during the period). To close the balance inManufacturing Overhead, Macho Mike applies a decrease to Cost of Goods Sold.

Part 1 Part 2 Part 3 Demo DocComplete

4 Record transactions forcompletion and salesof finished goods andadjustment for under-or overallocated manu-facturing overhead

Requirement 3

Prepare the journal entry to close the ending balance of manufacturingoverhead.

1eSG_C16_0131792075.Qxd 10/24/06 2:01 PM Page 554

Page 587: polpoa_sg

Quick Practice QuestionsTrue/False

_____ 1. Process costing is used by companies that produce large numbers ofidentical units in a continuous fashion.

_____ 2. A food and beverage company would most likely use a job order costingsystem.

_____ 3. A job cost record is a document that accumulates direct materials,direct labor, and manufacturing overhead costs assigned to each indi-vidual job.

_____ 4. For jobs that the company has started but not yet finished, the job costrecords form the subsidiary ledger for the general ledger account Workin Process Inventory.

_____ 5. When materials are requisitioned for a job, the Materials Inventoryaccount is debited.

_____ 6. A labor time record identifies the employee, the amount of time spenton a particular job, and the labor cost charged to the job.

_____ 7. Manufacturing Overhead is credited for actual manufacturing over-head costs incurred throughout the year.

_____ 8. The allocation base is a common denominator that links indirect man-ufacturing overhead costs to the cost objects.

_____ 9. Work in Process Inventory is credited for the cost of direct labor in ajob order costing system.

_____10. The required adjustment for an underallocation of manufacturing over-head results in a credit to Cost of Goods Sold.

Quick Practice Questions | Chapter 16 555

Multiple Choice1. What are the two basic types of costing systems?

a. Job order costing and process costingb. Periodic costing and perpetual costingc. Product costing and materials inventory costingd. Periodic costing and process costing

2. Which type of business can use a job order costing system? a. Service and manufacturing businesses b. Manufacturing and merchandising businesses c. Service and merchandising businesses d. Service, merchandising, and manufacturing businesses

3. Which of the following industries is most likely to use a processcosting system? a. Paint b. Aircraft c. Construction d. Unique furniture accessories

1eSG_C16_0131792075.Qxd 10/24/06 2:01 PM Page 555

Page 588: polpoa_sg

4. Which of the following companies is most likely to use job ordercosting? a. Kellogg’s Cereal Company b. Elizabeth’s Custom Furniture Company c. ExxonMobil Oil Refinery d. DuPont Chemical Company

5. Which of the following would be necessary to record thepurchase of materials on account using a job order costingsystem? a. Credit to Work in Process Inventory b. Debit to Accounts Payable c. Debit to Materials Inventory d. Debit to Work in Process Inventory

6. Which of the following would be debited to record the directmaterials used? a. Finished Goods Inventory b. Materials Inventory c. Work in Process Inventory d. Cost of Goods Manufactured

7. Which of the following would be debited to assign direct laborcosts actually incurred? a. Finished Goods Inventory b. Manufacturing Overhead c. Wages Payable d. Work in Process Inventory

8. Which of the following would be debited to assign the costs ofindirect labor? a. Manufacturing Overheadb. Work in Process Inventoryc. Finished Goods Inventoryd. Wages Payable

9. What is the document that is prepared by manufacturingpersonnel to request materials for the production process?a. Materials requisition b. Cost ticket c. Job order card d. Manufacturing ticket

10. Opaque Corporation uses a job order costing system. The Workin Process Inventory balance on December 31, 2007 consists ofJob No. 120, which has a balance of $19,000. Job No. 120 has beencharged with manufacturing overhead of $5,100. Opaqueallocates manufacturing overhead at a predetermined rate of85% of direct labor cost. What is the amount of direct materialscharged to Job No. 120? a. $7,565b. $5,900c. $7,000d. $7,900

556 Chapter 16 | Quick Practice Questions

1eSG_C16_0131792075.Qxd 10/24/06 2:01 PM Page 556

Page 589: polpoa_sg

Quick Exercises

16-1. State whether each of the following companies would be morelikely to use a job order costing system or a process costingsystem.

a. Custom furniture manufacturer _______________b. Paint manufacturer _______________c. Carpet manufacturer _______________d. Concrete manufacturer _______________e. Home builder _______________f. Soft drink bottler _______________g. Custom jewelry manufacturer _______________

16-2. Gadgets Company has two departments, X and Y. Manufacturingoverhead is allocated based on direct labor cost in Department Xand direct labor hours in Department Y. The following addi-tional information is available:

Estimated Amounts

Department X Department Y

Direct labor costs $249,600 $427,500

Direct labor hours 24,960 45,000

Manufacturing overhead costs 259,000 262,000

Actual data for completed Job No. 140 is as follows:

Direct materials requisitioned $23,700 $48,600

Direct labor cost 34,400 38,800

Direct labor hours 4,300 3,800

a. Compute the predetermined manufacturing overhead rate forDepartment X.

b. Compute the predetermined manufacturing overhead rate forDepartment Y.

c. What is the total manufacturing overhead cost for Job No. 140?

d. If Job No. 140 consists of 350 units of product, what is the aver-age unit cost of this job?

Quick Practice Questions | Chapter 16 557

1eSG_C16_0131792075.Qxd 10/24/06 2:01 PM Page 557

Page 590: polpoa_sg

16-3. Peterson Corporation uses a job order costing system.Journalize the following transactions in Peterson’s generaljournal for the current month:

a. Purchased materials on account, $74,000.b. Requisitioned $47,700 of direct materials and $6,500 of indirect

materials for use in production.c. Factory payroll incurred and due to employees, $72,000.d. Allocated factory payroll, 85% direct labor, 15% indirect labor.e. Recorded depreciation on factory equipment, $10,500, and other

manufacturing overhead of $45,900 (credit Accounts Payable).f. Allocated manufacturing overhead based on 130% of direct labor

cost.g. Cost of completed production for the current month, $155,000.h. Cost of finished goods sold, $131,000; selling price, $183,000 (all

sales on account).

General Journal

Date Accounts Dr. Cr.

General Journal

Date Accounts Dr. Cr.

General Journal

Date Accounts Dr. Cr.

General Journal

Date Accounts Dr. Cr.

General Journal

Date Accounts Dr. Cr.

1eSG_C16_0131792075.Qxd 10/24/06 2:01 PM Page 558

Page 591: polpoa_sg

Quick Practice Questions | Chapter 16 559

General Journal

Date Accounts Dr. Cr.

General Journal

Date Accounts Dr. Cr.

General Journal

Date Accounts Dr. Cr.

General Journal

Date Accounts Dr. Cr.

16-4. The following activities took place in the Work in ProcessInventory account during April:

Work in process balance, April 1 $ 15,000Direct materials used 123,000

Total manufacturing labor incurred in April was $163,500. Directlabor represents 75% of manufacturing labor. The predeterminedmanufacturing overhead rate is 120% of direct labor cost. Actualmanufacturing overhead costs for April amounted to $150,000.

Two jobs were completed with total costs of $118,000 and $85,000, respec-tively. They were sold on account for $258,000 and $150,000, respectively.

Requirements

1. Compute the balance in Work in Process Inventory on April 30.

1eSG_C16_0131792075.Qxd 11/22/06 10:37 AM Page 559

Page 592: polpoa_sg

General Journal

Date Accounts Dr. Cr.

General Journal

Date Accounts Dr. Cr.

General Journal

Date Accounts Dr. Cr.

2. Journalize the following:

a. Direct materials used in April.b. The total manufacturing labor incurred in April.c. The entry to assign manufacturing labor to the appropriate accounts.d. The allocated manufacturing overhead for April.e. The entry to move the completed jobs into finished goods inventory.f. The entry to sell the two completed jobs on account.

General Journal

Date Accounts Dr. Cr.

General Journal

Date Accounts Dr. Cr.

General Journal

Date Accounts Dr. Cr.

1eSG_C16_0131792075.Qxd 10/24/06 2:01 PM Page 560

Page 593: polpoa_sg

Quick Practice Questions | Chapter 16 561

16-5. The following account balances as of January 1, 2008, wereselected from the general ledger of Browning ManufacturingCompany:

Work in process inventory $ 0 Materials inventory 21,000 Finished goods inventory 44,000

Additional data:

Actual manufacturing overhead for January, $59,000.Total direct labor cost for January, $56,000.The predetermined manufacturing overhead rate is based on direct laborcost. The budget for 2008 called for $300,000 of direct labor cost and$369,000 of manufacturing overhead costs.The only job unfinished on January 31, 2008, was Job No. 410, for whichtotal labor charges were $5,600 (700 direct labor hours) and total directmaterial charges were $10,000.Cost of direct materials placed in production during January, $100,000.No indirect material requisitions were made during January.January 31 balance in Materials Inventory, $29,000.Finished Goods Inventory balance on January 31, $30,000.

a. Calculate the predetermined manufacturing overhead rate.

b. Determine the amount of materials purchased during January.

c. Determine cost of goods manufactured for January.

d. Determine the Work in Process Inventory balance on January 31.

e. Calculate cost of goods sold for January.

f. Is manufacturing overhead overallocated or underallocated?What is the account balance at January 31?

1eSG_C16_0131792075.Qxd 10/24/06 2:01 PM Page 561

Page 594: polpoa_sg

562 Chapter 16 | Do It Youself! Question 1

General Journal

Date Accounts Dr. Cr.

General Journal

Date Accounts Dr. Cr.

General Journal

Date Accounts Dr. Cr.

Do It Yourself! Question 1

Bell Boxers is a customized clothing manufacturer. Bell has the following transactions:

a. Purchased raw materials on account, $20,000.

b. Materials costing $15,000 were requisitioned for production. Of thistotal, $1,500 worth were indirect materials.

c. Labor time records show that direct labor of $24,000 and indirectlabor of $3,000 were incurred, but not yet paid.

Requirements

1. Why would Bell Boxers use the job order costing system?

2. What document would Bell Boxers use to accumulate direct materials,direct labor, and manufacturing overhead costs assigned to each job?

3. Journalize each transaction.

1eSG_C16_0131792075.Qxd 11/22/06 10:38 AM Page 562

Page 595: polpoa_sg

Do ItYouself! Question 2 | Chapter 16 563

General Journal

Date Accounts Dr. Cr.

Do It Yourself Question 2

Quality Cabinet Maker manufactures specialized cabinets. Because of its uniquespecialization, Quality Cabinet is labor intensive, so it uses direct labor cost toallocate its overhead. Quality Cabinet expects to incur $240,000 of manufactur-ing overhead costs and to use $300,000 of direct labor cost during 2010.

During May 2010, Quality Cabinet actually incurred $22,000 of direct labor costand recorded the following transactions:

a. Indirect actual manufacturing labor, $5,100.

b. Indirect actual materials used, $6,200.

c. Other manufacturing overhead incurred, $7,000 (credit accountspayable).

d. Allocated overhead for May.

e. Transferred 45,000 of product to finished goods.

f. Sold product on account, $60,000; cost of the product, $33,000.

Requirements

1. Compute the predetermined manufacturing overhead for Quality.

2. Journalize the transactions in the general journal.

General Journal

Date Accounts Dr. Cr.

General Journal

Date Accounts Dr. Cr.

1eSG_C16_0131792075.Qxd 11/22/06 10:38 AM Page 563

Page 596: polpoa_sg

564 Chapter 16 | Do It Youself! Question 2

3. Prepare the journal entry to close the ending balance of ManufacturingOverhead.

General Journal

Date Accounts Dr. Cr.

General Journal

Date Accounts Dr. Cr.

General Journal

Date Accounts Dr. Cr.

General Journal

Date Accounts Dr. Cr.

1eSG_C16_0131792075.Qxd 10/24/06 2:01 PM Page 564

Page 597: polpoa_sg

Quick Practice Solutions | Chapter 16 565

Quick Practice SolutionsTrue/False

T 1. Process costing is used by companies that produce large numbers of identical units in a continuous fashion. (p. 849)

F 2. A food and beverage company would most likely use a job order costing system.False–A food and beverage company would most likely use a processcosting system. (p. 849)

T 3. A job cost record is a document that accumulates direct materials,direct labor, and manufacturing overhead costs assigned to each individual job. (p. 849)

T 4. For jobs that the company has started but not yet finished, the job cost records form the subsidiary ledger for the general ledger account Work in Process Inventory. (p. 850)

F 5. When materials are requisitioned for a job, the Materials Inventory account is debited.False–When materials are requisitioned for a job, the Materials Inventory account is credited. (p. 851)

T 6. A labor time record identifies the employee, the amount of time spent on a particular job, and the labor cost charged to the job. (p. 852)

F 7. Manufacturing Overhead is credited for actual manufacturing overhead costs incurred throughout the year.False–Manufacturing Overhead is debited for actual manufacturingoverhead costs incurred throughout the year. (p. 857)

T 8. The allocation base is a common denominator that links indirect manufacturing overhead costs to the cost objects. (p. 856)

F 9. Work in Process Inventory is credited for the cost of direct labor in a job order costing system.False–Work in process inventory is debited for the cost of direct labor in a job order costing system. (p. 857)

F 10. The required adjustment for an underallocation of manufacturing overhead results in a credit to Cost of Goods Sold.False–The required adjustment for an underallocation of manufacturing overhead results in a debit to Cost of Goods Sold.(p. 860)

1eSG_C16_0131792075.Qxd 10/24/06 2:01 PM Page 565

Page 598: polpoa_sg

566 Chapter 16 | Quick Practice Solutions

Multiple Choice

1. What are the two basic types of costing systems? (pp. 848–849)a. Job order costing and process costingb. Periodic costing and perpetual costingc. Product costing and materials inventory costingd. Periodic costing and process costing

2. Which type of business can use a job order costing system? (pp. 848–849)a. Service and manufacturing businesses b. Manufacturing and merchandising businesses c. Service and merchandising businesses d. Service, merchandising, and manufacturing businesses

3. Which of the following industries is most likely to use a processcosting system? (p. 849)a. Paint b. Aircraft c. Construction d. Unique furniture accessories

4. Which of the following companies is most likely to use job ordercosting? (pp. 848–849)a. Kellogg’s Cereal Company b. Elizabeth’s Custom Furniture Companyc. ExxonMobil Oil Refinery d. DuPont Chemical Company

5. Which of the following would be necessary to record thepurchase of materials on account using a job order costingsystem? (p. 850)a. Credit to Work in Process Inventory b. Debit to Accounts Payable c. Debit to Materials Inventoryd. Debit to Work in Process Inventory

6. Which of the following would be debited to record the directmaterials used? (pp. 850–851)a. Finished Goods Inventory b. Materials Inventory c. Work in Process Inventoryd. Cost of Goods Manufactured

7. Which of the following would be debited to assign direct laborcosts actually incurred? (p. 853)a. Finished Goods Inventory b. Manufacturing Overhead c. Wages Payable d. Work in Process Inventory

1eSG_C16_0131792075.Qxd 10/24/06 2:01 PM Page 566

Page 599: polpoa_sg

Quick Practice Solutions | Chapter 16 567

8. Which of the following would be debited to assign the costs ofindirect labor? (p. 853)a. Manufacturing Overheadb. Work in Process Inventoryc. Finished Goods Inventoryd. Wages Payable

9. What is the document that is prepared by manufacturingpersonnel to request materials for the production process? (p. 851)a. Materials requisitionb. Cost ticket c. Job order card d. Manufacturing ticket

10. Opaque Corporation uses a job costing system. The Work inProcess Inventory balance on December 31, 2007, consists of JobNo. 120, which has a balance of $19,000. Job No. 120 has beencharged with manufacturing overhead of $5,100. Opaqueallocates manufacturing overhead at a predetermined rate of85% of direct labor cost. What is the amount of direct materialscharged to Job No. 120? (p. 855)a. $7,565b. $5,900c. $7,000d. $7,900

Quick Exercise

16-1. State whether each of the following companies would be morelikely to use a job order costing system or a process costingsystem: (pp. 848–849)

a. Custom furniture manufacturer job costing

b. Paint manufacturer process costing

c. Carpet manufacturer process costing

d. Concrete manufacturer process costing

e. Home builder job costing

f. Soft drink bottler process costing

g. Custom jewelry manufacturer job costing

1eSG_C16_0131792075.Qxd 10/24/06 2:01 PM Page 567

Page 600: polpoa_sg

568 Chapter 16 | Quick Practice Solutions

16-2. Gadgets Company has two departments, X and Y.Manufacturing overhead is allocated based on direct laborcost in Department X and direct labor hours in Department Y.The following additional information is available: (p. 855)

Estimated Amounts

Department X Department Y

Direct labor costs $249,600 $427,500

Direct labor hours 24,960 45,000

Manufacturing overhead costs 259,000 262,000

Actual data for completed Job No. 140 is as follows:

Direct materials requisitioned $23,700 $48,600

Direct labor cost 34,400 38,800

Direct labor hours 4,300 3,800

a. Compute the predetermined manufacturing overhead rate forDepartment X.

$259,000/$249,600 ! 104% of direct labor cost

b. Compute the predetermined manufacturing overhead rate forDepartment Y.

$262,000/45,000 hours ! $5.82 per direct labor hour

c. What is the total manufacturing overhead cost for Job. No.140?

Dept. X ! $34,400 " 104% ! $35,776

Dept. Y ! $5.82 " 3,800 ! 22,116

$57,892

d. If Job No. 140 consists of 350 units of product, what is the aver-age unit cost of this job?$23,700 # $34,400 # $35,776 # $48,600 # $38,800 # $22,116 ! $203,392

$203,392/350 units ! $581.12

16-3. Peterson Corporation uses a job costing system. Journalizethe following transactions in Peterson’s general journal forthe current month: (pp. 850–859)

a. Purchased materials on account, $74,000.b. Requisitioned $47,700 of direct materials and $6,500 of indirect

materials for use in production.c. Factory payroll incurred and due to employees, $72,000.d. Allocated factory payroll, 85% direct labor, 15% indirect labor.e. Recorded depreciation on factory equipment $10,500 and other

manufacturing overhead of $45,900 (credit Accounts Payable).f. Allocated manufacturing overhead based on 130% of direct labor

cost.g. Cost of completed production for the current month, $155,000.h. Cost of finished goods sold, $131,000; selling price, $183,000 (all

sales on account).

1eSG_C16_0131792075.Qxd 10/24/06 2:01 PM Page 568

Page 601: polpoa_sg

Quick Practice Solutions | Chapter 16 569

General Journal

a. Materials Inventory 74,000 Accounts Payable 74,000

Date Accounts Dr. Cr.

General Journal

b. Work in Process Inventory 47,700 Manufacturing Overhead 6,500 Materials Inventory 54,200

Date Accounts Dr. Cr.

General Journal

c. Manufacturing Wages 72,000 Wages Payable 72,000

Date Accounts Dr. Cr.

General Journal

d. Work in Process Inventory 61,200 Manufacturing Overhead 10,800 Manufacturing Wages 72,000

Date Accounts Dr. Cr.

General Journal

e. Manufacturing Overhead 56,400 Accumulated Depreciation, Factory Equip. 10,500 Accounts Payable 45,900

Date Accounts Dr. Cr.

General Journal

f. Work in Process Inventory 79,560 Manufacturing Overhead 79,560

Date Accounts Dr. Cr.

1eSG_C16_0131792075.Qxd 10/24/06 2:01 PM Page 569

Page 602: polpoa_sg

570 Chapter 16 | Quick Practice Solutions

16-4. The following activities took place in the Work in ProcessInventory account during April: (pp. 850–859)

Work in process balance, April 1 $ 15,000Direct materials used 123,000

Total manufacturing labor incurred in April was $163,500. Directlabor represents 75% of manufacturing labor. The predeterminedmanufacturing overhead rate is 120% of direct labor cost. Actual man-ufacturing overhead costs for April amounted to $150,000.

Two jobs were completed with total costs of $118,000 and $85,000,respectively. They were sold on account for $258,000 and $150,000,respectively.

Requirement 1

Compute the balance in Work in Process Inventory on April 30.

$15,000 ! $123,000 ! (75% ! $163,500) ! (120% ! $122,625) – $118,000 – $85,000 "$204,775

Requirement 2

Journalize the following:

a. Direct materials used in April.b. The total manufacturing labor incurred in April.c. The entry to assign manufacturing labor to the appropriate

accounts.d. The allocated manufacturing overhead for April.e. The entry to move the completed jobs into finished goods inventory.f. The entry to sell the two completed jobs on account.

General Journal

g. Finished Goods Inventory 155,000 Work in Process Inventory 155,000

Date Accounts Dr. Cr.

General Journal

h. Accounts Receivable 183,000 Sales Revenue 183,000 Cost of Goods Sold 131,000 Finshed Goods Inventory 131,000

Date Accounts Dr. Cr.

1eSG_C16_0131792075.Qxd 11/22/06 10:38 AM Page 570

Page 603: polpoa_sg

Quick Practice Solutions | Chapter 16 571

General Journal

a. Work in Process Inventory 123,000 Materials Inventory 123,000

Date Accounts Dr. Cr.

General Journal

b. Manufacturing Wages 163,500 Wages Payable 163,500

Date Accounts Dr. Cr.

General Journal

c. Work in Process Inventory 122,625 Manufacturing Overhead 40,875 Manufacturing Wages 163,500

Date Accounts Dr. Cr.

General Journal

d. Work in Process Inventory 147,150 Manufacturing Overhead 147,150

Date Accounts Dr. Cr.

General Journal

e. Finished Goods Inventory 203,000 Work in Process Inventory 203,000

Date Accounts Dr. Cr.

General Journal

f. Accounts Receivable 408,000 Sales Revenue 408,000

Cost of Goods Sold 203,000 Finshed Goods Inventory 203,000

Date Accounts Dr. Cr.

1eSG_C16_0131792075.Qxd 10/24/06 2:01 PM Page 571

Page 604: polpoa_sg

572 Chapter 16 | Quick Practice Solutions

16-5. The following account balances as of January 1, 2008, wereselected from the general ledger of Browning ManufacturingCompany: (pp. 850–860)

Work in process inventory $ 0 Materials inventory 21,000 Finished goods inventory 44,000

Additional data:

Actual manufacturing overhead for January, $59,000.Total direct labor cost for January, $56,000.

The predetermined manufacturing overhead rate is based on directlabor cost. The budget for 2008 called for $300,000 of direct labor costand $369,000 of manufacturing overhead costs.

The only job unfinished on January 31, 2008, was Job No. 410, forwhich total labor charges were $5,600 (700 direct labor hours) andtotal direct material charges were $10,000.

Cost of direct materials placed in production during January,$100,000. No indirect material requisitions were made duringJanuary.

January 31 balance in Materials Inventory, $29,000.Finished Goods Inventory balance on January 31, $30,000.

a. Calculate the predetermined manufacturing overhead rate.

$369,000/$300,000 ! 123% of direct labor cost

b. Determine the amount of materials purchased during January.

$21,000 # X - $100,000 ! $29,000$79,000 # X ! $29,000

X ! $108,000

c. Determine cost of goods manufactured for January.

$56,000 # (123% " $56,000) # $100,000 – $5,600 – $10,000 – (123% " $5,600) ! $202,392

d. Determine the Work in Process Inventory balance on January 31.

$5,600 # (123% " $5,600) # $10,000 ! $22,488

e. Calculate cost of goods sold for January.

$44,000 # $202,392 – $30,000 ! $216,392

f. Is manufacturing overhead overallocated or underallocated?What is the account balance at January 31?

Actual manufacturing overhead ! $59,000Allocated manufacturing overhead ! $68,880

$68,880-$59,00 ! $9,880 overallocated

1eSG_C16_0131792075.Qxd 10/24/06 2:01 PM Page 572

Page 605: polpoa_sg

Do ItYouself! Question 1 Solutions | Chapter 16 573

General Journal

a. Materials Inventory 20,000 Accounts Payable 20,000

Date Accounts Dr. Cr.

General Journal

b. Work in Process Inventory 13,500 Manufacturing Overhead 1,500 Materials Inventory 15,000

Date Accounts Dr. Cr.

General Journal

c. Manufacturing Wages 27,000 Wages Payable 27,000 Work in Process Inventory 24,000 Manufacturing Overhead 3,000 Manufacturing Wages 27,000

Date Accounts Dr. Cr.

1 Distinguish betweenjob order costing andprocess costing

2 Record materials andlabor transactions in ajob order costingsystem

2 Record materials andlabor transactions in ajob order costingsystem

Do It Yourself! Question 1 Solutions

Requirement 1

Why would Bell Boxers use the job order costing system?

Companies that manufacture batches of unique or specialized products woulduse a job order costing system to accumulate costs for each job or batch. BecauseBell Boxers manufactures specialized clothing for customers and the requiredwork may vary from customer to customer, Bell would use the job order costingsystem.

Requirement 2

What document would Bell use to accumulate direct materials, direct labor,and manufacturing overhead costs assigned to each job?

Bell would use a job cost record to accumulate direct materials, direct labor, andmanufacturing overhead costs assigned to each individual job.

Requirement 3Journalize each transaction.

1eSG_C16_0131792075.Qxd 11/22/06 10:39 AM Page 573

Page 606: polpoa_sg

574 Chapter 16 | The Power of Practice

General Journal

a. Manufacturing Overhead 5,100 Manufacturing Wages 5,100

Date Accounts Dr. Cr.

3 Record manufacturingoverhead in a manu-facturer’s job ordercosting system

Do It Yourself! Question 2 Solutions

Requirement 1

Compute the predetermined manufacturing overhead for Quality.

$240,000/$300,000 ! 0.80, or 80% of direct labor cost

Requirement 2Journalize the transactions in the general journal.

General Journal

b. Manufacturing Overhead 6,200 Materials Inventory 6,200

Date Accounts Dr. Cr.

General Journal

c. Manufacturing Overhead 7,000 Accounts Payable 7,000

Date Accounts Dr. Cr.

General Journal

d. Work in Process Inventory 17,600 Manufacturing Overhead 17,600

Date Accounts Dr. Cr.

General Journal

e. Finished Goods Inventory 45,000 Work in Process Inventory 45,000

Date Accounts Dr. Cr.

1eSG_C16_0131792075.Qxd 11/22/06 10:39 AM Page 574

Page 607: polpoa_sg

Do It Yourself Question 2 Solutions | Chapter 16 575

General Journal

Cost of Goods Sold 700 Manufacturing Overhead 700

Date Accounts Dr. Cr.

4 Record transactionsfor completion andsales of finishedgoods and adjustmentfor under- or overallo-cated overhead

Requirement 3

Prepare the journal entry to close the ending balance of ManufacturingOverhead.

General Journal

f. Accounts Receivable 60,000 Sales Revenue 60,000 Cost of Goods Sold 33,000 Finished Goods Inventory 33,000

Date Accounts Dr. Cr.

1eSG_C16_0131792075.Qxd 11/22/06 10:39 AM Page 575

Page 608: polpoa_sg

576 Chapter 16 | The Power of Practice

The Power of PracticeFor more practice using the skills learned in this chapter, visit MyAccountingLab.There you will find algorithmically generated questions that are based on theseDemo Docs and your main textbook’s Review and Accounting Practice sections.

To go to MyAccountingLab and follow these steps:

1. Direct your URL to www.myaccountinglab.com.2. Log in using your name and password.3. Click the MyAccountingLab link.4. Click Study Plan in the left navigation bar.5. From the table of contents, select Chapter 16, Job Order Costing.6. Click a link to work on the tutorial exercises.

1eSG_C16_0131792075.Qxd 10/24/06 2:01 PM Page 576

Page 609: polpoa_sg

Process Costing17WHAT YOU PROBABLY ALREADY KNOW

Sometimes when you have the opportunity, like on vacation, youmay want to catch up on reading several books of interest. Assumethat you started reading three books and were approximately two-thirds done reading each of them at the end of the week. If you wereasked how many books you finished reading, you would have to saynone. But that answer would not indicate the amount of time spentduring the week reading. You read two-thirds of each of the threebooks, which is equivalent to reading two complete books (2/3 ! 3 = 2)over the course of the week. Using an equivalent number of booksaccurately quantifies the amount of work completed during theweek. In this chapter, we study the computation of inventory costs.Todetermine the cost of work in process, we use this concept ofequivalent units for those inventory items that are partiallycomplete.

Learning Objectives

Distinguish between the flow of costs in process costing and joborder costing.

In Chapter 16, we learned that job order costing is an appropriate systemfor companies that produce unique jobs, batches, or assignments. Processcosting is an appropriate system for companies that mass-produce manysimilar products in a continuous fashion such as food and beverage, paint,or chemical manufacturers. The flow of costs through both of thesesystems is similar.The three components of cost—direct materials, directlabor, and manufacturing overhead—are included in the product cost.After the goods are produced, they are transferred fromWork in Process toFinished Goods. When the goods are sold, the costs are transferred fromFinished Goods to Cost of Goods Sold.The difference between the twosystems occurs during the processing stage.When direct materials, directlabor, and overhead are applied, the Work in Process account is debitedunder both systems.The job order costing system includes a subsidiaryledger of job cost records.The sum of the job cost records, which includethe direct materials, direct labor, and manufacturing overhead for eachjob, should always equal the one Work in Process account balance. Underprocess costing, each of the processing departments may have a Work inProcess account.The Work in Process costs will flow from department todepartment while in production until the product is complete and the costsare transferred into Finished Goods. Review Exhibit 17-1 (p. 900) for acomparison between the cost flows of job order and process costingsystems.

1

1eSG_C17_0131792075.Qxd 10/24/06 2:03 PM Page 577

Page 610: polpoa_sg

578 Chapter 17 | Process Costing

Compute equivalent units.

Equivalent units express the amount of work done during a period interms of fully complete units of output in order to calculate the costincurred for each unit. At the end of the period, some of those costs willbe transferred either to the next Work in Process account or to FinishedGoods. To calculate equivalent units, the department’s physical flow ofinventory is first determined. Inputs must equal outputs. Beginning unitsin process plus those started in the department are the inputs oraccountable units. The ending units in process and those completed andtransferred out are the outputs, or units accounted for. Equivalent unitsare determined for direct materials and conversion costs separately forthe outputs. Those units completed and transferred out are 100%complete for direct materials and conversion costs; the physical units andequivalent units are the same. For ending Work in Process units, adetermination of the percentage complete for direct materials andconversion costs must be made. The physical units multiplied by thepercentage complete equals the equivalent units. Review Exhibit 17-3(p. 903) and Exhibits 17-5 and 17-6 (pp. 906–907) for an illustration andcalculation of equivalent units.

Use process costing to assign costs to units completed and tounits in ending work in process inventory.

The direct materials and conversion costs are determined by adding thecosts from beginning inventory, if any, to the costs added to thedepartment for the period. The total costs are divided by the respectiveequivalent units to determine the cost per equivalent units, as shown inDemo Doc 1. The units completed will be assigned the full unit cost—direct materials plus conversion cost. The equivalent units of ending workin process are allocated the respective cost per equivalent unit todetermine the cost of ending work in process inventory. Review Ex-hibits 17-8 and 17-9 (pp. 908–909) for the calculation of costs to unitscompleted and in ending work in process inventory.

Use the weighted-average method to assign costs to unitscompleted and to units in ending work in process inventory in asecond department.

The procedure to calculate the cost of units completed and of units inending work in process in a second department is similar to thatdescribed in objective 3. The difference is a third column for thetransferred in equivalent units and costs. All of the units completed and inending work in process will be allocated the transferred in cost per unit.Review Exhibits 17-12 to 17-16 (pp. 913–917) for illustrations of calculatingand assigning costs to units completed and to units in ending work inprocess inventory in a second department.

2

4

3

1eSG_C17_0131792075.Qxd 10/24/06 2:03 PM Page 578

Page 611: polpoa_sg

Demo Doc 1 | Chapter 17 579

Demo Doc 1Illustrating Process CostingLearning Objectives 2, 3

Clear Bottled Water produces packaged water. Clear has two production depart-ments: blending and packaging. In the blending department, materials are addedat the beginning of the process. Conversion costs are added throughout theprocess for blending. Data for the month of April for the blending department areas follows:

Requirement

Use the four-step process to calculate (1) the cost of the units completed andtransferred out to the packaging department, and (2) the total cost of the units inthe blending department ending work in process inventory.

CLEAR BOTTLED WATERBlending Department

Month Ended April 30, 2010

Units: Beginning work in process Started in production during April Completed and transferred out to Packaging in April Ending work in process inventory (70% completed)Costs: Beginning work in process Costs added during April: Direct materials Conversion costs Total costs added during April

0 units116,000 units98,000 units18,000 units

$ 0

54,52032,074

$86,594

1eSG_C17_0131792075.Qxd 10/24/06 2:03 PM Page 579

Page 612: polpoa_sg

580 Chapter 17 | Demo Doc 1 Solutions

CLEAR BOTTLED WATERBlending Department

Month Ended April 30, 2010

Flow of Production

Step 1Flow of

Physical Units

Step 2: Equivalent UnitsDirect

MaterialsConversion

CostsUnits to account for: Beginning work in process, March 31 Started in production during April Total accountable physical units

0116,000116,000

Materials are added at the beginning of the blending production process, soequivalent units for materials are the same as the total units.

Completed units are 100% for conversion costs (98,000).Conversion costs are added evenly throughout the blending process, so the

conversion equivalent units for ending work in process inventory are the totalunits in ending work in process multiplied by the percent complete.

18,000 ! 70% " 12,600

Remember, conversion costs include both direct labor and manufacturing overhead.

Demo Doc 1 Solution

Part 1 Part 2 Part 3 Demo DocComplete

Total units to account for is the equivalent of units completed and transferred outof packaging in April (98,000) plus the ending work in process inventory of April 30(18,000):

2 Compute equivalentunits

Requirement

Use the four-step process to calculate (1) the cost of the units completed andtransferred out to the packaging department, and (2) the total cost of the units inthe blending department ending work in process inventory.

Steps 1 and 2: Summarize the flow of physical units and compute output interms of equivalent units.

1eSG_C17_0131792075.Qxd 10/24/06 2:03 PM Page 580

Page 613: polpoa_sg

Demo Doc 1 Solutions | Chapter 17 581

We can see that the total units accounted for is 116,000 units, with 98,000completed units + 18,000 work in process units = 116,000 equivalent units fordirect materials, and 98,000 completed units + 12,600 work in process units =110,600 equivalent units for conversion costs.

Step 3: Compute the cost per equivalent unit.

3 Use process costingto assign costs tounits completed andto units in ending workin process inventory

Part 1 Part 2 Part 3 Demo DocComplete

CLEAR BOTTLED WATERBlending Department

Month Ended April 30, 2010

Flow of Production

Step 1Flow of

Physical Units

Step 2: Equivalent UnitsDirect

MaterialsConversion

CostsUnits to account for: Beginning work in process, March 31 Started in production during April Total accountable physical units

Units accounted for: Completed and transferred out during April Ending work in process, April 30 Total physical units accounted for

Equivalent units

0116,000116,000

98,00018,000

116,000

98,00018,000

116,000

98,00012,600

110,600

The cost per equivalent unit is computed by dividing the costs added during theperiod by the equivalent units:

We know from Step 1 that Clear has 116,000 equivalent units. From the question,we know that Clear has $54,520 of total direct materials costs in April.

Using the formula for cost per equivalent unit for direct materials, we dividethe total direct materials costs by the equivalent units of materials, as determinedin Step 2.

$54,520/116,000 units " $0.47 per equivalent unit for direct materials

By using equivalent units, we indicate that $32,074 of conversion costs will blend110,600 units from the start of the blending process to the end of the blendingprocess. To calculate costs per equivalent unit for conversion costs, we must dividethe total conversion costs by the number of equivalent units for conversion, asdetermined in Step 2.

$32,074/110,600 " $0.29 per equivalent units for conversion costs

Cost per Equivalent Unit forDirect Materialss

Total Direct Material CostEquivalent Uni

"tts for Direct Material Cost

Cost per Equivallent Unit forConversion Cost

Total Convers"

iion CostEquivalent Units for Conversion Cosst

1eSG_C17_0131792075.Qxd 10/24/06 2:03 PM Page 581

Page 614: polpoa_sg

582 Chapter 17 | Demo Doc 1 Solutions

Step 4: Assign costs to units completed and to units in ending work in processinventory.

Because the units completed and transferred out were started and finished in themonth of April, their cost is the full unit cost of $0.76. Shown another way:

The ending work in process is complete regarding materials because they areadded in their entirety at the beginning of the mixing process.

The conversion costs in ending work in process are only 70% complete becauseconversion costs occur evenly throughout the mixing process. Multiplying each bytheir respective per-unit cost:

18,000 $0.47 $ 8,460 direct materials (100! " %%)

12,600 $0.29 $ 3,654 conversion costs (! " 770%)

$12,114"

98,000 $0.47 $46,060 direct materials (100%! " ))

98,000 $0.29 $28,420 conversion costs (10! " 00%)

98,000 $0.76 $74,480! "

3 Use process costingto assign costs tounits completed andto units in ending workin process inventory

Part 1 Part 2 Part 3 Demo DocComplete

CLEAR BOTTLED WATERBlending Department

Month Ended April 30, 2010

Step 3:Cost per Equivalent Unit

DirectMaterials

ConversionCosts

Beginning work in process, March 31Cost added during AprilTotal costs for AprilDivide by equivalent unitsCost per equivalent unit

$ 054,520

$ 54,520"116,000$ 0.47

$ 032,074

$ 32,074"110,600$ 0.29

1eSG_C17_0131792075.Qxd 10/24/06 2:03 PM Page 582

Page 615: polpoa_sg

Demo Doc 1 Solutions | Chapter 17 583

CLEAR BOTTLED WATERBlending Department

Month Ended April 30, 2010

Step 4: Assign CostsDirect

MaterialsConversion

Costs TotalUnits completed and transferred out to Packaging in AprilEnding work in process, April 30: Direct materials Conversion costs Total ending work in process, April 30Total costs accounted for

[98,000 3 ($0.47 + $0.29)] = $74,480

8,4603,654

$12,114$86,594

[18,000 3 0.47][12,600 3 0.29]

Part 1 Part 2 Part 3 Demo DocComplete

The solution to the problem is the $74,480 cost of the goods completed and trans-ferred out of the blending department to the packaging department during Apriladded to the $12,114 cost of the ending work in process in the blending departmentas of the April 30.

$74,480 # $12,114 " $86,594 total costs

1eSG_C17_0131792075.Qxd 10/24/06 2:03 PM Page 583

Page 616: polpoa_sg

584 Chapter 17 | Demo Doc 2

Weighted-Average Process CostingLearning Objectives 2, 3, 4

Easy Flow produces tubes for toothpaste. Easy has two departments: molding andpackaging. In the second department, packaging, conversion costs are incurredevenly throughout the process. Packaging materials are not added until the end ofthe packaging process. Costs in beginning work in process inventory includetransferred in costs of $28,360, direct labor of $12,369, and manufacturing over-head of $10,000. October data from the packaging department are as follows:

Demo Doc 2

EASY FLOWPackaging Department

Month Ended October 31, 2010

Units DollarsBeginning inventory, Sept. 30 (60% complete)Production started: Transferred in Direct materials Conversion costs: Direct labor Manufacturing overhead Total conversion costsTotal to account forTransferred outEnding inventory (30% complete)

5,700

120,000

125,700110,00015,700

$ 50,729

575,000155,100

240,000185,000

$ 425,000$1,205,829$ ?$ ?

Easy Flow uses weighted-average process costing.

Requirements

1. Compute Easy Flow’s equivalent units for the month of October.

2. Compute the cost per equivalent unit for October.

3. Assign the costs to units completed and transferred out and to endinginventory.

1eSG_C17_0131792075.Qxd 10/24/06 2:03 PM Page 584

Page 617: polpoa_sg

Demo Doc 2 Solutions | Chapter 17 585

Part 1 Part 2 Part 3 Part 4 Demo DocComplete

Demo Doc 2 Solutions

Part 1 Part 2 Part 3 Part 4 Demo DocComplete

The 110,000 units completed and transferred out are 100% completed regardingtransferred in, direct materials, and conversion costs, so the equivalent units are100%.

The ending work in process inventory is 100% complete regarding transferredin. In other words, Easy Flow doesn’t need to transfer any additional costs fromanother department to complete the units in ending inventory.

Because direct materials are added at the end of the packaging process, theyare zero complete in terms of work in process.

Units in ending inventory are only 30% finished in terms of conversion costs.Another way to think of this situation is that the conversion costs applied to the end-ing inventory so far will complete 15,700 units 30% of the way (15,700 ! 0.30 =4,710), or those same conversion costs would complete 4,710 units 100% of the way.

EASY FLOWPackaging Department

Month Ended October 31, 2010

Flow of ProductionTransferred

In

Equivalent UnitsDirect

MaterialsConversion

CostsUnits accounted for: Completed and transferred out during October Ending work in process inventory, October 31Equivalent units

110,00015,700

125,700

110,0000

110,000

110,0004,710

114,710

2 Compute equivalentunits

4 Use the weighted-average method toassign costs to unitscompleted and tounits in ending work inprocess inventory in asecond department

Determine total costs for each of the three categories. We know from the questionthat we have beginning inventory of $28,360 transferred in costs and $22,369 inconversion costs ($12,369 for direct labor and $10,000 manufacturing overhead),

4 Use the weighted-average method toassign costs to unitscompleted and tounits in ending work inprocess inventory in asecond department

So the total equivalent units for the packaging department during October is125,700 transferred-in units, 110,000 direct material units, and 114,710 conversioncosts units.

Requirement 2

Compute the cost per equivalent unit for October.

Requirement 1

Compute Easy Flow’s equivalent units for the month of October.

1eSG_C17_0131792075.Qxd 10/24/06 2:03 PM Page 585

Page 618: polpoa_sg

586 Chapter 17 | Demo Doc 2 Solutions

for a total of $50,729. The direct materials are zero for beginning work in processbecause they are added at the end of the packaging process.

During October, we saw $575,000 in costs transferred in from the moldingdepartment, $155,100 in direct materials costs, and $425,000 in conversion costs,for a total of $1,155,100 in costs added during October. As a check, make sure thatthe sum of the totals of the transferred in, direct materials, and conversion costsequals the sum of the total beginning work in process and the total of costs addedduring October.

EASY FLOWPackaging Department

Month Ended October 31, 2010

Flow of ProductionTransferred

In

Equivalent UnitsDirect

MaterialsConversion

Costs TotalBeginning work in process inventory, Sept. 30 $ 28,360 $ 0 $ 22,369 $ 50,729

EASY FLOWPackaging Department

Month Ended October 31, 2010

Flow of ProductionTransferred

In

Equivalent UnitsDirect

MaterialsConversion

Costs TotalBeginning work in process inventory, Sept. 30Costs added during OctoberTotal costs

$ 28,360575,000

$603,360

$ 0155,100

$155,100

$ 22,369425,000

$447,369

$ 50,7291,155,100

$1,205,829

Now that we have our total costs, we must divide total costs for each of the cat-egories by the equivalent units for each respective category (as determined inRequirement 1) to determine the cost per equivalent unit.The transferred-in costs are:

Direct materials costs are:

Conversion costs are:

$447,369114,710 units

$3.90 per unit"

$155,100110,000 units

$1.41 per unit"

$603,360125,700 units

$4.80 per unit"

1eSG_C17_0131792075.Qxd 10/24/06 2:03 PM Page 586

Page 619: polpoa_sg

Demo Doc 2 Solutions | Chapter 17 587

Completed and Transferred Out

Finished goods inventory consists of all three cost categories, so Easy Flow multi-plies the total units transferred out by the sum of the three costs.

In this case, 110,000 units have been completed and transferred out duringOctober. We know from Requirement 2 that cost per transferred-in unit is $4.80,cost per unit for direct materials is $1.41, and the per-unit conversion cost is $3.90.The 110,000 completed units receive 100% of their transferred-in, direct material,and conversion costs. Shown another way:

The $1,112,100 will be transferred out to finished goods because the packagingdepartment is the last process in Easy Flow production (debit Finished Goods,credit Work in Process).

110 000 4 80 528 000, $ . $ ,! " Transferred-inDirect Mate110 000 1 41 155 100, $ . $ ,! " rrials

110,000 Convers! "$ . $ ,3 90 429 000 iion Cost110,000 Total Tr! "$ . $ , ,10 11 1 112 100 aansferred

EASY FLOWPackaging Department

Month Ended October 31, 2010

TransferredIn

Assign CostsDirect

MaterialsConversion

Costs TotalUnits completed and transferred out to Finished Goods Inventory $1,112,100[110,000 3 ($4.80 + $1.41 + $3.90)] =

EASY FLOWPackaging Department

Month Ended October 31, 2010

TransferredIn

Costs per Equivalent UnitsDirect

MaterialsConversion

Costs TotalBeginning work in process inventory, Sept. 30Costs added during OctoberTotal costsDivide by equivalent unitsCost per equivalent unitTotal costs to account for

$ 28,360575,000

$603,360"125,700$ 4.80

$ 0155,100

$155,100"110,000$ 1.41

$ 22,369425,000

$447,369"114,710$ 3.90

$0,050,7291,155,100

$1,205,829

Part 1 Part 2 Part 3 Part 4 Demo DocComplete

3 Use process costingto assign costs tounits completed andto units in ending workin process inventory

4 Use the weighted-average method toassign costs to unitscompleted and tounits in ending work inprocess inventory in asecond department

We will use these calculations to assign total costs for October in the packagingdepartment to units completed and to units in ending work in process inventory.

Requirement 3

Assign the costs to units completed and transferred out and to endinginventory.

1eSG_C17_0131792075.Qxd 10/24/06 2:03 PM Page 587

Page 620: polpoa_sg

588 Chapter 17 | Demo Doc 2 Solutions

Ending Inventory

Make sure the total costs to account for, $1,112,100 + $93,729 = $1,205,829,match with the total costs from Requirement 2.

EASY FLOWPackaging Department

Month Ended October 31, 2010

TransferredIn

Assign CostsDirect

MaterialsConversion

Costs TotalUnits completed and transferred out to Finished Goods Inventory Ending work in process inventory, October 31: Transferred in costs Direct materials Conversion costs Total ending work in process inventory, October 31Total costs accounted for

$1,112,100[110,000 3 ($4.80 + $1.41 + $3.90)] =

[15,700 3 4.80]

0[4,710 3 3.90]

$1,175,360

018,369

$ 93,729$1,205,829

Part 1 Part 2 Part 3 Part 4 Demo DocComplete

Part 1 Part 2 Part 3 Part 4 Demo DocComplete

Because direct materials are added at the end of the packaging process, they havenot yet been added to the units in ending work in process.

We know from Requirement 1 that the ending work in process inventory is15,700 transferred-in units, 0 direct materials, and 4,710 conversion cost equiva-lent units. Shown another way:

The $93,729 ending work in process inventory will be listed as an asset on EasyFlow’s balance sheet (both Work in Process and Finished Goods are inventory and,therefore, assets).

15,700 $4.80 $75,360 Tansferred-in! "

0 $1.41 $ 0 Direct Material! " ss4,710 $3.90 $18,369 Conversion Costs! "

$$93,729

1eSG_C17_0131792075.Qxd 10/24/06 2:03 PM Page 588

Page 621: polpoa_sg

Quick Practice Questions | Chapter 17 589

Quick Practice QuestionsTrue/False

_____ 1. In a process costing system, a separate Work in Process Inventoryaccount is maintained for each process.

_____ 2. In a process costing system, costs flow into Finished Goods Inventoryonly from the Work in Process Inventory of the last manufacturingprocess.

_____ 3. Wong Corporation had 25,000 units completed and transferred out and8,000 units that were 35% complete. The equivalent units total 33,000.

_____ 4. The entry to transfer goods in process from Department A toDepartment B includes a debit to Work in Process, Department A.

_____ 5. The cost per equivalent unit must be computed for direct materials, con-version, and transferred-in costs in a subsequent department.

_____ 6. The number of equivalent units may be greater than the number ofaccountable physical units.

_____ 7. The cost per equivalent unit is calculated separately for each of the threecomponents of cost—direct materials, direct labor, and manufacturingoverhead.

_____ 8. Unique or custom-made goods would be accounted for by using a processcosting system.

_____ 9. Conversion costs are generally added evenly throughout the process.

_____10. If a department has beginning inventory of 2,000 units, 23,000 units arestarted into production, and ending inventory is 1,500 units, then 22,500units are completed.

Multiple Choice

1. In a process costing system, the number of Work in ProcessInventory accounts is equal to what amount?a. The number of products producedb. The number of production departmentsc. The number used in a job order costing systemd. Cannot be determined without additional information

2. In a process costing system, the entry to record the use of directmaterials in production would include which of the following?a. Debit to Work in Process Inventoryb. Debit to Materials Inventoryc. Debit to Finished Goods Inventoryd. Credit to Finished Goods Inventory

1eSG_C17_0131792075.Qxd 10/24/06 2:03 PM Page 589

Page 622: polpoa_sg

590 Chapter 17 | Quick Practice Questions

3. The entry to record a $24,000 transfer from the assemblydepartment to the finishing department would include which ofthe following?a. Debit to Work in Process Inventory, Assemblyb. Debit to Finished Goods Inventoryc. Credit to Work in Process Inventory, Assemblyd. Credit to Materials Inventory

4. During the period, 50,000 units were completed, and 3,600 unitswere on hand at the end of the period. If the ending work inprocess inventory was 75% complete as to direct materials and25% complete as to conversion costs, the equivalent units fordirect materials under the weighted-average method would bewhat amount?a. 45,900b. 47,700c. 48,000d. 52,700

5. Beginning work in process is 900 units, units completed andtransferred out in October are 3,200 units, and ending work inprocess is 500 units. Under weighted-average costing, what areunits started into production in October?a. 2,300b. 2,800c. 3,200d. 3,600

6. Conversion costs consist of which of the following?a. Direct materials and direct laborb. Direct labor and manufacturing overheadc. Direct materials and manufacturing overheadd. Product costs and period costs

7. The Lloyd Company uses a process costing system. No units werein beginning work in process, 1,400 were started, and 1,000 unitswere completed and transferred out. The units at the end of theperiod were 60% complete regarding materials and 40% completeregarding conversion. The cost of materials added during thecurrent period amounted to $31,944; the cost of conversion addedduring the current period amounted to $30,016. What are theequivalent units for materials?a. 1,200b. 1,240c. 1,320d. 1,400

8. Refer to Question 7. What are equivalent units for conversioncosts?a. 1,160b. 1,280c. 1,300d. 1,320

1eSG_C17_0131792075.Qxd 10/24/06 2:03 PM Page 590

Page 623: polpoa_sg

Quick Practice Questions | Chapter 17 591

9. Refer to Question 7. What is the cost per equivalent unit formaterials?a. $22.50b. $22.82c. $24.20d. $25.76

10. Refer to Question 7. What is the cost per equivalent unit forconversion?a. $22.74b. $23.45c. $25.01d. $25.88

Quick Exercises

17-1. Identify which of the following products or services would usea process costing system.

a. Paintb. Surgical operationc. Custom kitchen cabinetsd. Cellular telephonese. Cerealf. Airplanesg. Soft drinksh. Office buildingsi. Custom swimming poolsj. Personal computers

17-2. Department A has no beginning work in process inventory.During the current period, 13,500 units were placed into pro-duction. At the end of the current period, 12,000 units weretransferred to Department B. The ending units in Department Awere 90% complete regarding direct materials and 65% com-plete regarding conversion costs. Compute the equivalent unitsfor direct materials and conversion costs.

17-3. Journalize the following transactions:

a. Issued $8,800 of direct materials to production in the Carvingdepartment.

b. Manufacturing labor in the Carving department amounted to $8,000.c. Allocated manufacturing labor to the appropriate accounts: 90%

direct labor; 10% indirect labor. The pay rate for all direct laboris $20 per hour.

d. Allocated manufacturing overhead in the Carving department at$15 per direct labor hour.

e. Transferred $8,600 of product from the Carving department to finished goods inventory.

1eSG_C17_0131792075.Qxd 10/24/06 2:03 PM Page 591

Page 624: polpoa_sg

17-4. Holland Company makes a variety of chemicals. Its Grindingdepartment reports the following information for June of thecurrent year:

General Journal

Date Accounts Dr. Cr.

General Journal

Date Accounts Dr. Cr.

General Journal

Date Accounts Dr. Cr.

General Journal

Date Accounts Dr. Cr.

General Journal

Date Accounts Dr. Cr.

HOLLAND COMPANYGrinding Department

Month Ended June 30, 2010

Units:Completed and transferred outUnfinished units, work in process, June 30

Costs:Direct materialsDirect laborManufacturing overhead

7,4003,500

$110,05555,00079,845

*100% complete for direct materials and 30% complete for conversion costs incurred.

*

1eSG_C17_0131792075.Qxd 10/24/06 2:03 PM Page 592

Page 625: polpoa_sg

a. Compute the equivalent units for direct materials and conversioncosts.

b. Compute the cost per equivalent unit for direct materials and con-version costs.

c. Compute the cost of the goods completed and transferred out.

d. Compute the cost of the work in process at June 30.

17-5. Comfort Corporation manufactures air mattresses. The com-pany uses the weighted-average method of process costing.Information for the Assembly department for the month ofJuly is as follows:

COMFORT CORPORATIONAssembly Department

Month Ended July 31, 2010

Units:Work in process inventory, July 1 (75% complete for direct materials, 40% complete for conversion costs)Units transferred in from Cutting departmentUnits completed and transferred outWork in process inventory, July 31 (55% complete for direct materials, 20% complete for conversion costs)Costs:Direct materials: Work in process inventory, July 1 Added during JulyConversion costs: Work in process inventory, July 1 Added during JulyTransferred-in costs: Work in process inventory, July 1 Units transferred in from Cutting department during July

30,000 units150,000 units140,000 units

40,000 units

$ 90,000567,000

61,800721,000

315,000$1,575,000

a. Compute the total cost of units completed and transferred out.

b. Compute the total cost of work in process inventory on July 31.

1eSG_C17_0131792075.Qxd 10/24/06 2:03 PM Page 593

Page 626: polpoa_sg

594 Chapter 17 | Do It Yourself! Question 1

JIGGLING JELLYBlending Department

Month Ended June 30, 2010

Do It Yourself! Question 1

Jiggling Jelly produces packaged jelly. Jiggling has two production departments:blending and packaging. In the blending department, materials are added at thebeginning of the process. Conversion costs are added throughout the process forblending. Data for the month of June for the blending department are as follows:

JIGGLING JELLYBlending Department

Month Ended June 30, 2010

Units: Beginning work in process Started in production during June Completed and transferred out to Packaging in June Ending work in process inventory (20% completed)Costs: Beginning work in process Costs added during June: Direct materials Conversion costs Total costs added during June

0 units32,500 units28,000 units

4,500 units

$ 0

15,92524,998

$40,923

Use the four-step process to calculate (1) the cost of the units completed andtransferred out to the packaging department, and (2) the total cost of the units inthe blending department ending work in process inventory.

Steps 1 and 2

1eSG_C17_0131792075.Qxd 10/24/06 2:03 PM Page 594

Page 627: polpoa_sg

Do It Yourself! Question 1 | Chapter 17 595

Step 4

Step 3

JIGGLING JELLYBlending Department

Month Ended June 30, 2010

JIGGLING JELLYBlending Department

Month Ended June 30, 2010

1eSG_C17_0131792075.Qxd 10/24/06 2:03 PM Page 595

Page 628: polpoa_sg

596 Chapter 17 | Do ItYourself! Question 2

Do It Yourself! Question 2

Quality Chemicals produces a chemical that it sells to hospitals. Quality has twodepartments: mixing and packaging. In the second department, packaging, conver-sion costs are incurred evenly throughout the process. Packaging materials are notadded until the end of the packaging process. Costs in beginning work in processinventory include transferred-in costs of $88,000, direct labor of $44,000, and man-ufacturing overhead of $21,000. July data from the packaging department are asfollows:

QUALITY CHEMICALSPackaging Department

Month Ended July 31, 2010

Units DollarsBeginning inventory, June 30 (80% complete)Production started: Transferred in Direct materials Conversion costs: Direct labor Manufacturing overhead Total conversion costsTotal to account forTransferred outEnding inventory (30% complete)

55,000

230,000

285,000220,00065,000

$153,000

293,900250,800

189,250105,000294,250

$991,950?

$ ?

QUALITY CHEMICALSPackaging Department

Month Ended July 31, 2010

Quality uses the weighted-average method for process costing.

Requirement 1

Compute Quality’s equivalent units for the month of July.

1eSG_C17_0131792075.Qxd 11/22/06 10:40 AM Page 596

Page 629: polpoa_sg

Do ItYourself! Question 2 | Chapter 17 597

QUALITY CHEMICALSPackaging Department

Month Ended July 31, 2010

QUALITY CHEMICALSPackaging Department

Month Ended July 31, 2010

Requirement 2

Compute the cost per equivalent unit for July.

Requirement 3

Assign the costs to units completed and transferred out and to ending workin process inventory.

1eSG_C17_0131792075.Qxd 11/22/06 10:40 AM Page 597

Page 630: polpoa_sg

598 Chapter 17 | Quick Practice Solutions

Quick Practice SolutionsTrue/False

T 1. In a process costing system, a separate Work in Process Inventory account is maintained for each process (p. 899)

T 2. In a process costing system, costs flow into Finished Goods Inventory only from the Work in Process Inventory of the last manufacturing process. (p. 899)

F 3. Wong Corporation had 25,000 units completed and transferred out and 8,000 units that were 35% complete. The equivalent units total 33,000.False–Equivalent units would be 27,800 = 25,000 + (8,000 3 0.35).(p. 902)

F 4. The entry to transfer goods in process from Department A to Department B includes a debit to Work in Process, Department A.False–The entry to transfer goods in process from Department A to Department B includes a credit to Work in Process, Department A.(p. 909)

T 5. The cost per equivalent unit must be computed for direct materials,conversion, and transferred-in costs in a subsequent department.(p. 910)

F 6. The number of equivalent units may be greater than the number of accountable physical units.False–The number of equivalent units cannot be greater than the number of accountable physical units. (p. 902)

F 7. The cost per equivalent unit is calculated separately for each of the three components of cost—direct materials, direct labor, and manufacturing overhead.False–The cost per equivalent unit is calculated separately for direct materials, conversion costs, and transferred-in costs as applicable.(p. 907)

F 8. Unique or custom-made goods would be accounted for by using a process costing system.False–Unique or custom-made goods would be accounted for by using a job order costing system. (p. 898)

T 9. Conversion costs are generally added evenly throughout the process. (p. 902)

F 10. If a department has beginning inventory of 2,000 units, 23,000 units are started into production, and ending inventory is 1,500 units, then22,500 units are completed.False–If a department has beginning inventory of 2,000 units,23,000 units are started into production, and ending inventory is 1,500 units, then 2,000 + 23,000 – 1,500 = 23,500 units are completed. (p. 905)

1eSG_C17_0131792075.Qxd 10/24/06 2:03 PM Page 598

Page 631: polpoa_sg

Quick Practice Solutions | Chapter 17 599

Multiple Choice

1. In a process costing system, the number of Work in ProcessInventory accounts is equal to what amount? (p. 899)a. The number of products producedb. The number of production departmentsc. The number used in a job order costing systemd. Cannot be determined without additional information

2. In a process costing system, the entry to record the use of directmaterials in production would include which of the following? (p. 909)a. Debit to Work in Process Inventoryb. Debit to Materials Inventoryc. Debit to Finished Goods Inventoryd. Credit to Finished Goods Inventory

3. The entry to record a $24,000 transfer from the assemblydepartment to the finishing department would include which ofthe following? (p. 909)a. Debit to Work in Process Inventory, Assemblyb. Debit to Finished Goods Inventoryc. Credit to Work in Process Inventory, Assemblyd. Credit to Materials Inventory

4. During the period, 50,000 units were completed, and 3,600 unitswere on hand at the end of the period. If the ending work inprocess inventory was 75% complete as to direct materials and25% complete as to conversion costs, the equivalent units fordirect materials under the weighted-average method would bewhat amount? (p. 911)a. 45,900b. 47,700c. 48,000d. 52,700

5. Beginning work in process is 900 units, units completed andtransferred out in October are 3,200 units, and ending work inprocess is 500 units. Under weighted-average costing, what areunits started into production in October? (p. 911)a. 2,300b. 2,800c. 3,200d. 3,600

6. Conversions costs consist of which of the following? (p. 902)a. Direct materials and direct laborb. Direct labor and manufacturing overheadc. Direct materials and manufacturing overheadd. Product costs and period costs

1eSG_C17_0131792075.Qxd 10/24/06 2:03 PM Page 599

Page 632: polpoa_sg

600 Chapter 17 | Quick Practice Solutions

7. The Lloyd Company uses a process costing system. No units werein beginning work in process, 1,400 were started, and 1,000 unitswere completed and transferred out. The units at the end of theperiod were 60% complete regarding materials and 40% completeregarding conversion. The cost of materials added during thecurrent period amounted to $31,944; the cost of conversion addedduring the current period amounted to $30,016. What are theequivalent units for materials? (p. 906)a. 1,200b. 1,240c. 1,320d. 1,400

8. Refer to Question 7. What are equivalent units for conversioncosts? (p. 906)a. 1,160b. 1,280c. 1,300d. 1,320

9. Refer to Question 7. What is the cost per equivalent unit formaterials? (p. 907)a. $22.50b. $22.82c. $24.20d. $25.76

10. Refer to Question 7. What is the cost per equivalent unit forconversion? (p. 907)a. $22.74b. $23.45c. $25.01d. $25.88

Quick Exercise Solutions

17-1. Identify which of the following products or services would usea process costing system. (p. 898)

a. Paintb. Surgical operationc. Custom kitchen cabinetsd. Cellular telephonese. Cerealf. Airplanesg. Soft drinksh. Office buildingsi. Custom swimming poolsj. Personal computers

1eSG_C17_0131792075.Qxd 10/24/06 2:03 PM Page 600

Page 633: polpoa_sg

Quick Practice Solutions | Chapter 17 601

17-2. Department A has no beginning work in process inventory.During the current period, 13,500 units were placed into pro-duction. At the end of the current period, 12,000 units weretransferred to Department B. The ending units in DepartmentA were 90% complete regarding direct materials and 65% com-plete regarding conversion costs. Compute the equivalent unitsfor direct materials and conversion costs. (p. 902)

Direct materials12,000 # (1,500 ! 0.90) " 13,350

Conversion costs12,000 # (1,500 ! 0.65) " 12,975

17-3. Journalize the following transactions. (p. 909)

a. Issued $8,800 of direct materials to production in the Carvingdepartment.

b. Manufacturing labor in the Carving department amounted to$8,000.

c. Allocated manufacturing labor to the appropriate accounts: 90%direct labor; 10% indirect labor. The pay rate for all direct labor is$20 per hour.

d. Allocated manufacturing overhead in the Carving department at$15 per direct labor hour.

e. Transferred $8,600 of product from the Carving department tofinished goods inventory.

General Journal

a. Work in Process, Carving 8,800 Materials Inventory 8,800

Date Accounts Dr. Cr.

General Journal

b. Manufacturing Wages 8,000 Wages Payable 8,000

Date Accounts Dr. Cr.

General Journal

c. Work in Process Inventory, Carving 7,200 Manufacturing Overhead 800 Manufacturing Wages 8,000

Date Accounts Dr. Cr.

1eSG_C17_0131792075.Qxd 10/24/06 2:03 PM Page 601

Page 634: polpoa_sg

17-4. Holland Company makes a variety of chemicals. Its Grindingdepartment reports the following information for June of thecurrent year: (pp. 906–909)

602 Chapter 17 | Quick Practice Solutions

General Journal

e. Finished Goods Inventory 8,600 Work in Process Inventory, Carving 8,600

Date Accounts Dr. Cr.

General Journal

d. Work in Process Inventory, Carving 6,000 Manufacturing Overhead 6,000

Date Accounts Dr. Cr.

HOLLAND COMPANYGrinding Department

Month Ended June 30, 2010

Units:Completed and transferred outUnfinished units, work in process, June 30

Costs:Direct materialsDirect laborManufacturing overhead

7,4003,500

$110,05555,00079,845

*100% complete for direct materials and 30% complete for conversion costs incurred.

*

a. Compute the equivalent units for direct materials and conversion costs.

Direct materials7,400 # 3,500 " 10,900 equivalent units

Conversion costs7,400 # (3,500 ! 0.30) " 8,450 equivalent units

b. Compute the cost per equivalent unit for direct materials and con-version costs.

Direct materials$110,055/10,900 " $10.10

Conversion costs$134,845/8,450 " $15.96

c. Compute the cost of the goods completed and transferred out.

7,400 ! ($10.10 # $15.96) " $192,844

d. Compute the cost of the work in process at June 30.

(3,500 ! 1.00) ! $10.10 " $35,350(3,500 ! 0.30) ! $15.96 " 16,758

$52,108

1eSG_C17_0131792075.Qxd 10/24/06 2:03 PM Page 602

Page 635: polpoa_sg

17-5. Comfort Corporation manufactures air mattresses. The com-pany uses the weighted-average method of process costing.Information for the Assembly department for the month of Julyis as follows: (pp. 910–915)

Quick Practice Solutions | Chapter 17 603

COMFORT CORPORATIONAssembly Department

Month Ended July 31, 2010

Units:Work in process inventory, July 1 (75% complete for direct materials, 40% complete for conversion costs)Units transferred in from Cutting departmentUnits completed and transferred outWork in process inventory, July 31 (55% complete for direct materials, 20% complete for conversion costs)Costs:Direct materials: Work in process inventory, July 1 Added during JulyConversion costs: Work in process inventory, July 1 Added during JulyTransferred-in costs: Work in process inventory, July 1 Units transferred in from Cutting department during July

30,000 units150,000 units140,000 units

40,000 units

$ 90,000567,000

61,800721,000

315,000$1,575,000

a. Compute the total cost of units completed and transferred out.

Direct materials equivalent units 140,000 # (40,000 ! 0.55) " 162,000 Conversion costs equivalent units 140,000 # (40,000 ! 0.20) " 148,000 Transferred-in equivalent units 30,000 # 150,000 " 180,000

Cost per equivalent unit:Direct materials $657,000/162,000 " $4.06

Conversion costs $782,800/148,000 " $5.29

Transferred-in $1,890,000/180,000 " $10.50

Costs of units completed and transferred out 140,000 ! ($4.06 # $5.29 # $10.50) " $2,779,000

b. Compute the total cost of work in process inventory on July 31.

Cost of July 31 Work in Process " (22,000 ! $4,06) # (8,000 ! $5,29) #(40,000 ! $10.50)

" $89,320 # $42,320 # $420,000" $551,640

1eSG_C17_0131792075.Qxd 10/24/06 2:03 PM Page 603

Page 636: polpoa_sg

604 Chapter 17 | Do It Yourself! Question 1 Solutions

Do It Yourself! Question 1 SolutionsUse the four-step process to calculate (1) the cost of the units completed andtransferred out to the packaging department and (2) the total cost of the units inthe blending department ending work in process inventory.

Step 1: Summarize the flow of physical units.

Step 2: Compute output in terms of equivalent units.

Step 3: Compute the cost per equivalent unit.

JIGGLING JELLYBlending Department

Month Ended June 30, 2010

Flow of Production

Step 1Flow of

Physical Units

Step 2: Equivalent UnitsDirect

MaterialsConversion

CostsUnits to account for: Beginning work in process, May 31 Started in production during June Total accountable physical units

Units accounted for: Completed and transferred out during June Ending work in process, June 30 Total physical units accounted for

Equivalent units

032,50032,500

28,0004,500

32,500

28,0004,500

32,500

28,000900

28,900

JIGGLING JELLYBlending Department

Month Ended June 30, 2010

Step 3:Cost per Equivalent Unit

DirectMaterials

ConversionCosts

Beginning work in process, May 31Cost added during JuneTotal costs for JuneDivide by equivalent unitsCost per equivalent unit

$ 015,925

$15,925"32,500$ 0.49

$ 024,998

$24,998"28,900$ 0.865

2 Compute equivalentunits

3 Use process costingto assign costs tounits completed andto units in ending workin process inventory

1eSG_C17_0131792075.Qxd 10/24/06 2:03 PM Page 604

Page 637: polpoa_sg

Do ItYourself! Question 1 Solutions | Chapter 17 605

Step 4: Assign costs to units completed and to units in ending work in processinventory.

JIGGLING JELLYBlending Department

Month Ended June 30, 2010

Step 4: Assign CostsDirect

MaterialsConversion

Costs TotalUnits completed and transferred out to Packaging in April Ending work in process, May 31: Direct materials Conversion costs Total ending work in process, June 30Total costs accounted for

$37,940

2,205

[28,000 ! ($0.49 + $0.865)] =

778$ 2,983$40,923

[900 ! 0.865]

*900 ! $0.865 = $778.50, rounded to $778.

*4,500 ! 0.49

3 Use process costingto assign costs tounits completed andto units in ending workin process inventory

1eSG_C17_0131792075.Qxd 10/24/06 2:03 PM Page 605

Page 638: polpoa_sg

606 Chapter 17 | Do ItYourself! Question 2 Solutions

QUALITY CHEMICALSPackaging Department

Month Ended July 31, 2010

Flow of ProductionTransferred

In

Equivalent UnitsDirect

MaterialsConversion

CostsUnits accounted for: Completed and transferred out during July Ending work in process inventory, July 31Equivalent units

220,00065,000

285,000

220,0000

220,000

220,00019,500

239,500

4 Use the weighted-average method toassign costs to unitscompleted and tounits in ending work inprocess inventory in asecond department

4 Use the weighted-average method toassign costs to unitscompleted and tounits in ending work inprocess inventory in asecond department

QUALITY CHEMICALSPackaging Department

Month Ended July 31, 2010

TransferredIn

Costs per Equivalent UnitsDirect

MaterialsConversion

Costs TotalBeginning work in process inventory, June 30Costs added during JulyTotal costsDivide by equivalent unitsCost per equivalent unitTotal costs to account for

$ 88,000293,900

$381,900!285,000$ 1.34

$ 0250,800

$250,800!220,000$ 1.14

$ 65,000294,250

$359,250!239,500$ 1.50

$153,000838,950

$991,950

Do It Yourself! Question 2 Solutions

Requirement 1

Compute Quality’s equivalent units for the month of July.

Requirement 2

Compute the cost per equivalent unit for July.

1eSG_C17_0131792075.Qxd 11/22/06 10:41 AM Page 606

Page 639: polpoa_sg

Do ItYourself! Question 2 Solutions | Chapter 17 607

QUALITY CHEMICALSPackaging Department

Month Ended July 31, 2010

TransferredIn

Assign CostsDirect

MaterialsConversion

Costs TotalUnits completed and transferred out to Finished Goods Inventory Ending work in process inventory, July 31: Transferred in costs Direct materials Conversion costs Total ending work in process inventory, July 31Total costs accounted for

$875,600

87,1000

29,250$116,350$991,950

0

[220,000 " ($1.34 + $1.14 + $1.50)] =

65,000 " 1.34 =

19,500 " 1.50 =

4 Use the weighted-average method toassign costs to unitscompleted and tounits in ending work inprocess inventory in asecond department

Requirement 3

Assign the costs to units completed and transferred out and to endinginventory.

1eSG_C17_0131792075.Qxd 11/22/06 10:42 AM Page 607

Page 640: polpoa_sg

The Power of PracticeFor more practice using the skills learned in this chapter, visit MyAccountingLab.There you will find algorithmically generated questions that are based on theseDemo Docs and your main textbook’s Review and Accounting Practice sections.

To go to MyAccountingLab and follow these steps:

1. Direct your URL to www.myaccountinglab.com.2. Log in using your name and password.3. Click the MyAccountingLab link.4. Click Study Plan in the left navigation bar.5. From the table of contents, select Chapter 17, Process Costing.6. Click a link to work on the tutorial exercises.

608 Chapter 17 | The Power of Practice

1eSG_C17_0131792075.Qxd 10/24/06 2:03 PM Page 608

Page 641: polpoa_sg

Cost-Volume-ProfitAnalysis18

WHAT YOU PROBABLY ALREADY KNOW

If you decide to sign up for cell phone service, you know you have achoice of providers and plans. Assume that you are debatingwhether to enroll in one of two plans. The first plan has 450 freeanytime minutes, which costs $40 a month plus $0.45/minutebeyond 450. The second plan has 900 free anytime minutes, whichcosts $60 a month plus $0.40/minute beyond 900. Your objective isto minimize your cost. To accomplish this evaluation, you are usingconcepts like fixed and variable costs that we will study in thischapter. The monthly amount of $40 or $60 is fixed; once the plan isselected, you will be charged that monthly amount regardless ofthe number of minutes you use. In addition, you will be charged avariable cost per minute if you exceed the allowed number ofminutes. If you estimate that you will be using about 600 anytimeminutes, which plan is cheaper? Plan one would be $40 + [(600 –450) ! $0.45] = $107.50; plan two would be $60 + 0 = $60. Thesecond plan is cheaper.

Learning Objectives

Identify how changes in volume of activity affect costs.

There are three types of costs: fixed, variable, and mixed. Fixed costsare costs that do not change over wide ranges of volume. The range ofvolume over which fixed costs are constant is called the relevant range.Building rent, furniture depreciation, and the salary of an office managerwould be examples of fixed costs. If fixed costs are $100,000 and 10,000units are produced, the fixed cost per unit is $100,000/10,000 = $10. Ifthe volume doubles to 20,000 units, the fixed cost per unit is reduced to$5 per unit. Variable costs are costs that increase in total as the volumeof activity increases and decreases as the volume of activity decreases.Total variable costs change in direct proportion to changes in volume.Direct materials and direct labor are examples of variable costs. If itcosts $3 per unit for direct materials, the total variable costs when10,000 units are produced is $30,000. If the volume doubles to 20,000,the total variable cost is now $60,000, but the variable cost per unit isunchanged at $3 per unit. Mixed costs have both variable and fixedcomponents. The high-low method is used to separate the mixed costinto the variable and fixed elements. Review Exhibits 18-1 and 18-2 (p. 959) to see the impact of varying volume amounts on total variableand fixed costs.

1

1eSG_C18_0131792075.Qxd 10/24/06 2:05 PM Page 609

Page 642: polpoa_sg

610 Chapter 18 | Cost-Volume-Profit Analysis

Use CVP analysis to compute breakeven points.

The breakeven point is the sales level at which operating income is zero.Sales revenue equals total fixed and variable costs. You can use threeapproaches to compute the breakeven point. The income statementapproach uses the income statement to facilitate the calculation ofbreakeven units sold (see Demo Doc 1). Contribution margin is thedifference between sales revenue and variable costs. The contributionmargin approach is a shortcut to the income statement approach. Thecontribution margin ratio approach is used when you don’t havedetailed information on individual products. You will use these threeapproaches in Demo Doc 1.

Rounding Note: When calculating the breakeven sales units or sales dollars, it ispossible to obtain a fractional amount. To be conservative, you should alwaysround up to the next higher unit or dollar.

Use CVP analysis for profit planning, and graph the cost-volume-profit relations.

As you will see in Demo Doc 1, you can use the three approaches tocompute breakeven to also determine the number of units that must besold to create a desired net income.

The cost-volume-profit relationship can be graphed using increments ofvolume within the relevant range as the horizontal axis and dollars asthe vertical axis. Total sales revenue is zero at zero volume and wouldincrease linearly $35 for each unit of volume using the example in thesection called Using CVP to Plan Profits of the main textbook. Total costswould start on the vertical axis at $7,000, the total fixed costs, and wouldincrease linearly $21 for variable costs for each unit of volume. The pointat which the total sales revenue line equals total costs is the breakevenpoint. Drawing a vertical line from that point would indicate thebreakeven volume in units. Review the example in the main textbook forcomputations of sales units and dollars to achieve desired results.Review the cost-volume-profit graph in Exhibit 18-6 (p. 968).

Use CVP methods to perform sensitivity analyses.

Sensitivity analysis is a “what if” technique that shows how profits willbe affected if sales prices, costs, or underlying assumptions change.Using technology tools with an understanding of the interrelationshipbetween the elements of cost, volume, and profit you can determinewhat the effect may be on net income of making business decisions.

Calculate the breakeven point for multiple product lines or services.

Most companies sell multiple product lines or services. Each product orservice may have a different contribution margin. It is necessary tocalculate the weighted-average contribution margin of all of the salesusing a sales mix, as you will see in Demo Doc 2. A sales mix is thecombination of products that make up total sales.

2

3

4

5

1eSG_C18_0131792075.Qxd 10/24/06 2:05 PM Page 610

Page 643: polpoa_sg

Demo Doc 1 | Chapter 18 611

Demo Doc 1Use CVP to Plan ProfitsLearning Objectives 2, 3

Crew Cut Mowing Service mows residential lawns. The average amount it chargeto mow a single lawn is $30. Crew Cut calculated the average variable cost ofmowing a lawn to be about $18. Its monthly fixed cost is $1,200.

Requirements

1. Use the contribution margin approach to calculate how many lawns CrewCut must mow in a month to break even.

2. Use the contribution margin ratio approach to determine Crew Cut’sbreakeven point in sales dollars.

3. Use the income statement approach to prove that your solutions toRequirements 1 and 2 are correct.

4. The owner of Crew Cut currently works for another lawn service and earns$2,800 per month. He doesn’t want to incur the risk of owning his ownbusiness unless he believes that he can have a profit of at least the amounthe currently earns. Determine the number of lawns Crew Cut must mow ina month to earn a profit of $2,800.

1eSG_C18_0131792075.Qxd 10/24/06 2:05 PM Page 611

Page 644: polpoa_sg

612 Chapter 18 | Demo Doc 1 Solutions

Demo Doc 1 Solutions

Part 1 Part 2 Part 3 Part 4 Demo DocComplete

2 Use CVP analysis tocompute breakevenpoints

Requirement 1

Use the contribution margin approach to calculate how many lawns CrewCut must mow in a month to break even.

The contribution margin tells managers how much revenue is left after payingvariable costs. That revenue is used for contributing toward first covering fixedcosts and then generating a profit. The contribution margin is calculated by sub-tracting variable costs from the sales revenue. Therefore:

Sales Price per Unit – Variable Cost per Unit

Contribution Margin per Unit

Crew Cut’s variable cost per lawn (unit) is $18. Therefore, its unit contributionmargin is:

After variable costs are covered, Crew Cut has $12 per lawn that then con-tributes toward fixed costs until fixed costs are covered, after which point CrewCut will begin to generate a $12 profit per lawn.

Breakeven is the level of sales at which total operating revenues is equal tototal expenses (fixed and variable). In other words, the level at which income iszero. To compute breakeven using the contribution margin approach:

In this case, Crew Cut must be able to mow 100 lawns per month to break even.That’s because at 100 lawns, Crew Cut earns just enough contribution margin tocover total fixed costs. Every lawn mowed after the breakeven point contributesthe unit contribution margin to profit. For example, if Crew Cut mows 101 lawns,then it would earn a $12 profit.

BreakevenFixed Cost

Contribution Margin per!

Unit$1,200

$12L

!

! 100 awns

Sales Price per Lawn $30V" aariable Cost per Lawn (18)

Contribution Marrgin per Lawn $12

1eSG_C18_0131792075.Qxd 10/24/06 2:05 PM Page 612

Page 645: polpoa_sg

Demo Doc 1 Solutions | Chapter 18 613

2 Use CVP analysis tocompute breakevenpoints

So far, we have seen that computing the breakeven point for a simple businessthat sells only one product is pretty straightforward. Larger companies that don’thave detailed information on individual products use its contribution marginratio to predict profits, rather than using individual unit contribution margins oneach of its products.

The contribution margin ratio is the ratio of contribution margin to salesrevenue. In other words, it computes its breakeven point in terms of sales dollars.This calculation enables managers to do CVP analysis with aggregated informa-tion across many products with varied selling prices. We calculate the contribu-tion margin ratio as follows:

So, in this case, the contribution margin ratio is equal to the contribution marginper unit (determined in Requirement 1 to be $12 per lawn) divided by the salesrevenue per unit ($30 per lawn).

Each dollar of sales revenue contributes 40% ($0.40) toward fixed costs andprofit. To compute breakeven using the contribution margin ratio:

We know that Crew Cut’s fixed cost is $1,200 and its contribution margin ratio is40%, so:

Crew Cut must produce revenue of $3,000 per month to cover its fixed costs andvariable costs (that is, break even). This result is consistent with our previous cal-culations for breakeven (that is, 100 lawns ! $30 per lawn = $3,000).

Breakeven Sales in Dollars$1,20040%

!

! $3,000

Breakeven Sales in DollarsFixed Cost

Contri!

bbution Margin Ratio

Contribution Margin Ratio$12$30$

!

! 0.400 (or 40%)

Contribution Margin RatioContribution Marg

!iin per Unit

Sales Revenue per Unit

Part 1 Part 2 Part 3 Part 4 Demo DocComplete

Requirement 2

Use the contribution margin ratio approach to determine Crew Cut’sbreakeven point in sales dollars.

1eSG_C18_0131792075.Qxd 10/24/06 2:05 PM Page 613

Page 646: polpoa_sg

614 Chapter 18 | Demo Doc 1 Solutions

Up until now, we’ve computed how many lawns (or sales revenue) Crew Cutneeds to mow in order to break even. We know that it must mow 100 lawns, or theequivalent of $3,000 in sales, to break even. Anything less would be a loss.Anything more would be profit.

Now, Crew Cut wants to know how many lawns it needs to mow to generate$2,800 in profit. Because Crew Cut wants to know the number of lawns (units),we’ll use the formula based on the unit contribution margin.

Using the contribution margin approach, use the desired profit of $2,800 asfixed cost (in CVP analysis, always think of desired profit as a fixed cost):

Desired Profit Sales LevelFixed Cost Desir

!# eed Profit

Contribution Margin per Unit

3 Use CVP analysis forprofit planning, andgraph the cost-volume-profit relations

Part 1 Part 2 Part 3 Part 4 Demo DocComplete

By multiplying Crew Cut’s breakeven in units, 100, by what it charges to moweach lawn, $30, we’ve proven that our answers to Requirements 1 and 2 are thesame because the result equals the same as what we calculated in Requirement 2,$3,000.

After deducting the variable cost of $1,800 for 100 lawns and the fixed cost of$1,200, the income statement illustrates that Crew Cut would produce $0 operat-ing income at a breakeven level of 100 units.

Requirement 4

The owner of Crew Cut currently works for another lawn service and earns$2,800 per month. He doesn’t want to incur the risk of owning his ownbusiness unless he believes that he can have a profit of at least the amounthe currently earns. Determine the number of lawns Crew Cut must mow ina month to earn a profit of $2,800.

Sales Revenue (100 Units $30) $3,000Less: Va

$

rriable Cost (100 Units $18) 1,800Total Co

$

nntribution Margin $1,200Less: Fixed Cost 1,,200Operating Income $ 0

Part 1 Part 2 Part 3 Part 4 Demo DocComplete

Requirement 3

Use the income statement approach to prove that your solutions toRequirements 1 and 2 are correct.

1eSG_C18_0131792075.Qxd 10/24/06 2:05 PM Page 614

Page 647: polpoa_sg

Demo Doc 1 Solutions | Chapter 18 615

In this way, it’s similar to calculating breakeven, except now the desired profit istreated as a fixed cost in our calculations (remember, the contribution margin perunit is equal to selling price per unit minus variable cost per unit):

Once you know the breakeven, another way to think of it is to divide desiredprofit by contribution margin and add the difference to the breakeven:

$2,800/$12 ! 234 lawns (rounded)234 lawns # 100 breakeven ! 334 lawns

This analysis shows that Crew Cut must mow 334 lawns to earn a profit of$2,800 (remember, we round up in this case to avoid partial units). If Crew Cutalso wanted to know how much sales revenue it would need to earn $2,800 profit,it could use this figure to do the calculation:

334 lawns ! $30 per lawn ! $10,020 sales revenue

The desired profit is treated as a fixed cost in our calculations. Using 334 lawnsto achieve the desired profit, the owner of Crew Cut can decide whether it isworth leaving his current job to start Crew Cut. These data can also be used as amanagement tool to help determine marketing strategy, hiring policy, and aid inother types of decision making.

As before, you can use the income statement approach to prove these figures:

*The $8 operating income results from rounding a lawn [($334 – 333.33) ! $12] ! $8.

Whenever rounding must occur in a problem, such as when we rounded up from333.33 to 334 lawns to avoid a partial unit, a small difference can result whenproving the numbers in this way.

Sales Revenue (334 $30) $10,020Less: Total V

$

aariable Costs (334 $18) 6,012

Total Contrib

$

uution Margin (334 $12) $ 4,008Less: Total Fi

$

xxed Costs 1,200

Operating Income $ 2,808*

Desired Profit Sales Level$1,200 $2,800

$30!

#

""

!

!

$18$4,000

$12333.33, rounded to 334 lawnss

Part 1 Part 2 Part 3 Part 4 Demo DocComplete

1eSG_C18_0131792075.Qxd 10/24/06 2:05 PM Page 615

Page 648: polpoa_sg

616 Chapter 18 | Demo Doc 2

Demo Doc 2Using CVP for Sensitivity AnalysisLearning Objectives 2–4

Hacker Golf developed a unique swing trainer golf club. It currently has a pro-duction company produce the golf club at a cost of $22. Other variable costs total$6 while monthly fixed costs are $16,000. Hacker currently sells the trainer golfclub for $48.

Requirements Note: Solve each requirement as a separate situation.

1. Calculate Hacker’s breakeven point in units.

2. Hacker is considering raising its selling price to $49.95. Calculate the newbreakeven in units.

3. Hacker found a new company to produce the golf clubs at a lower cost of$19 each. Calculate the new breakeven in units.

4. Because many customers requested a golf glove to go along with thetrainer club, Hacker is considering selling gloves. It only expects to sell oneglove for every four trainer clubs it sells. Hacker can purchase the gloves for$5 each and sell them for $9 each.Total fixed costs should remain the sameat $16,000 per month. Calculate the breakeven point in units for trainerclubs and golf gloves.

5. Use a contribution margin income statement to prove the breakeven pointcalculated in Requirement 4.

1eSG_C18_0131792075.Qxd 10/24/06 2:05 PM Page 616

Page 649: polpoa_sg

Demo Doc 2 Solutions | Chapter 18 617

4 Use CVP methods toperform sensitivityanalyses

Demo Doc 2 Solutions

2 Use CVP analysis tocompute breakevenpoints

Part 1 Part 2 Part 3 Part 4 Part 5 Part 6 Part 7Demo DocComplete

Part 1 Part 2 Part 3 Part 4 Part 5 Part 6 Part 7Demo DocComplete

In this case, the selling price is changing, but Hacker’s variable and fixed costsare staying the same as in the original question ($28 and $16,000, respectively).The new selling price for the club is going to be $1.95 higher than the originalprice: from $48.00 to $49.95.

Requirement 1

Calculate Hacker’s breakeven point in units.

To determine how changes in sales prices, costs, or volume affect profits, let’s firststart by calculating the current breakeven point.

To determine the breakeven point, we first must calculate the contributionmargin per unit. The contribution margin per unit is calculated by subtractingvariable costs from the sales revenue. Therefore:

Contribution Margin per Unit ! Sales Price per Unit – Variable Cost per Unit

Hacker’s variable cost per club (unit) is the price it pays for each club ($22) plusits additional variable costs ($6). Therefore, its unit contribution margin is:

The contribution margin represents the amount from each unit sold that is avail-able to recover fixed costs. That means that after variable costs are covered,Hacker earns $20 per club, which then contributes toward fixed costs until fixedcosts are covered, after which point Hacker will begin to generate $20 profit perclub sold.

Breakeven is the level of sales at which income is zero. To computebreakeven using the contribution margin approach:

Requirement 2

Hacker is considering raising its selling price to $49.95. Calculate the newbreakeven in units.

BreakevenFixed Cost

Contribution Margin per!

Unit$16,000

$20!

! 800 trainer clubs

Selling Price per Club $48Variable Cost pe rr Club ($22 $6) (28)Contribution Margin pe

#

rr Club $20

1eSG_C18_0131792075.Qxd 10/24/06 2:05 PM Page 617

Page 650: polpoa_sg

618 Chapter 18 | Demo Doc 2 Solutions

Once we update the original data to reflect the changes, the data are thenprocessed with the same calculations. First, calculate the new contribution margin:

Using the contribution margin approach:

Again, we round because Hacker cannot sell a partial unit (the .93 in the actualcalculation).

With the increased selling price, breakeven has been reduced from 800 clubsto 729 clubs. The higher price means that each club contributes more to fixed costs.

You can prove this using the income statement approach:

*The $2 profit results from rounding 728.93 clubs to 729 (0.07 ! $21.95 ! $2).

We round the sales revenue total in this case to make sure that our figures balance.Remember that as selling prices increase (provided all costs remain the

same), the volume required to break even or achieve target profit goals decreases.Conversely, as selling prices decrease, the volume required to break even orachieve target profit goals increases.

Consider the following:

Selling price goes from $50 to $60, variable costs stay at $20, and totalfixed costs are $60,000.

Old contribution margin was $50 – $20 = $30.

Old breakeven point in units was $60,000/$30 = 2,000 units.

New contribution margin is $60 – $20 = $40.

New breakeven point in units is $60,000/$40 = 1,500 units.

The relationship between contribution margin (an increase of $10 in thiscase) and breakeven in units (a decrease of 500 units in this case) is aninverse relationship.

Sales Revenue (729 $49.95) $36,414Less: Vari

$

aable Costs (729 $28) 20,412Total Contribu

$

ttion Margin 16,002Less: Fixed Costs 16,000OOperating Income $ 2*

Breakeven in UnitsFixed Cost

Contribution M!

aargin per Unit$16,000.00

$21.95728.93,

!

! rouunded up to 729 trainer clubs

Selling Price per Club $49.95Variable Cost peer Club ($22 $6) (28.00)

Contribution Marg

#

iin per Club $21.95

1eSG_C18_0131792075.Qxd 10/24/06 2:05 PM Page 618

Page 651: polpoa_sg

Demo Doc 2 Solutions | Chapter 18 619

4 Use CVP methods toperform sensitivityanalyses

Part 1 Part 2 Part 3 Part 4 Part 5 Part 6 Part 7Demo DocComplete

Requirement 3

Hacker found a new company to produce the golf club at a lower cost of$19 each. Calculate the new breakeven in units.

Once costs begin to change, a new breakeven must be calculated to determine theeffects of the changes. In this case, the variable cost changes, yet fixed costs and thesales price stay the same as in the original question ($16,000 and $48, respectively).

In this case, we calculate as we normally would, except that our contributionmargin will be different:

Contribution Margin per Unit ! Sales Price per Unit – Variable Cost per Unit

Hacker’s variable cost per club (unit) is the price it pays for each club (now $19)plus its additional variable costs ($6). Therefore, its unit contribution margin is:

Using the contribution margin approach:

With the reduced variable cost, Hacker’s breakeven in units decreases from 800clubs to 696 clubs. Using this information, Hacker’s management must decide ifit is worth the risk to switch to a new producer.

You can also prove this result using the income statement approach:

Sales Revenue (696 $ $48) $33,408

Less:Variable Cost (696 $ $25) 17,400

Total Contribution Margin 16,008

Less: Fixed Costs 16,000

Operating Income $ 8*

*The $8 profit results from rounding [(696 – 695.65) ! $23] ! $8.

With both fixed and variable costs, remember that as these costs increase, so doesthe volume needed to break even or achieve target profits. Conversely, as these costsdecrease, the volume needed to break even or achieve target profits also decreases.

Selling price per club $48Variable cost" per club ($19 $6) (25)Contribution m

#

aargin per club $23

Total Salesin Units

Fixed CostWeighted

! --Average Contribution Margin per Unit

1eSG_C18_0131792075.Qxd 10/24/06 2:05 PM Page 619

Page 652: polpoa_sg

620 Chapter 18 | Demo Doc 2 Solutions

Part 1 Part 2 Part 3 Part 4 Part 5 Part 6 Part 7Demo DocComplete

5 Calculate thebreakeven point formultiple product linesor services

Calculating the breakeven point is fairly straightforward when a company isonly dealing with one product. But Hacker is now considering selling two prod-ucts rather than just the one. Now breakeven becomes a little more complicated.Different products will have different effects on the contribution marginsbecause of different costs and selling prices. So the company needs to considerthe sales mix (a combination of products that make up total sales) in figuringCVP relationships.

You can use the same formulas to determine the breakeven point consider-ing the sales mix, but before calculating breakeven, you must calculate theweighted-average contribution margin of all the products first. You saw anotherweighted-average for process costing in Chapter 17. In this case, the sales mixprovides the weights.

Step 1: Calculate the weighted-average contribution margin.

Hacker believes that it can sell one glove for every four trainer clubs that itsells, for a 4-to-1 sales mix. So it expect that 4/5 (or 80%) of its sales will betrainer clubs, and 1/5 (or 20%) of its sales will be gloves.

Recall that Hacker pays $28 in variable costs for its clubs and sells them for$48, for a contribution margin of $20 per unit. The gloves will cost $5 perpair and sell for $9, for a contribution margin of $4 per unit:

Clubs Gloves Total

Sales price per unit $48 $9

Less: Variable cost per unit (28) (5)

Contribution margin per unit $20 $4

The weighted-average contribution margin is calculated by multiplying thecontribution margin per unit by the sales mix expected for each. Once wehave a total contribution margin ($80 + $4 = $84, in this case), we divide thetotal contribution margin by the total sales mix in units (5), as follows:

Requirement 4

Because many customers requested a golf glove to go along with thetrainer club, Hacker is considering selling gloves. It only expects to sell oneglove for every four trainer clubs it sells. Hacker can purchase the gloves for$5 each and sell them for $9 each.Total fixed costs should remain the sameat $16,000 per month. Calculate the breakeven point in units for trainerclubs and golf gloves.

1eSG_C18_0131792075.Qxd 10/24/06 2:05 PM Page 620

Page 653: polpoa_sg

Demo Doc 2 Solutions | Chapter 18 621

Part 1 Part 2 Part 3 Part 4 Part 5 Part 6 Part 7Demo DocComplete

Sales Mix Percentage

20% 80%Clubs Gloves Total

Sales price per unit $48 $9

Less: Variable cost per unit (28) (5)

Contribution margin per unit $20 $4

Sales mix in units 4 1 5

Contribution margin per product $80 $4 $84.00

Weighted-average contribution margin ($84/5) $16.80

Another way to calculate this is to multiply each product’s contribution mar-gin by its sales mix percentage:

Clubs: $20 ! 80% " $16.00

Gloves: $4 ! 20% " $ 0.80

$16.80

The $16.80 represents an average contribution margin for all theproducts Hacker sells. The golf clubs are weighted more heavily because Hacker

expects to sell four times as many clubs compared to the gloves.The next step is to calculate the breakeven in units for the bundle of proucts.

Step 2: Calculate the breakeven point in units for the total of both prod-ucts combined using the following formula:

We know from the question that fixed costs will not be affected, so theyshould remain at $16,000. The weighted-average contribution margin, aswe just calculated, is $16.80 per unit. So we compute as follows:

Recall that we round up because Hacker cannot sell a partial unit.Hacker must sell a combined 953 clubs and gloves to break even. Managementneeds to know how many units of each product must be sold to break even.The next step is to determine the breakeven point in units for each product.

Total Salesin Units

$16,000$16.80

952.38,

"

" roounded to 953

Total Sales in Units Fixed CostWeighted-Ave

"rrage Contribution Margin per Unit

Part 1 Part 2 Part 3 Part 4 Part 5 Part 6 Part 7Demo DocComplete

1eSG_C18_0131792075.Qxd 11/22/06 10:46 AM Page 621

Page 654: polpoa_sg

622 Chapter 18 | Demo Doc 2 Solutions

5 Calculate thebreakeven point formultiple product linesor services

Sales revenue: Trainer clubs (763 3 $48) Gloves (191 3 $9)Variable costs: Trainer clubs (763 3 $28) Gloves (191 3 $5)Contribution marginFixed costsOperating income

$1,719

955$ 764

$36,624

21,364

$15,260

$38,343

22,319$16,024(16,000)$ 24

GlovesClubs Total

HACKER GOLFContribution Margin Income Statement

Part 1 Part 2 Part 3 Part 4 Part 5 Part 6 Part 7Demo DocComplete

To test the calculation of the breakeven point, you would add together the rev-enue generated from all sales, subtract the variable costs for each of the clubsand gloves, then subtract the total fixed costs. The result should balance to zero(or close to zero, in cases in which rounding occurs).

Part 1 Part 2 Part 3 Part 4 Part 5 Part 6 Part 7Demo DocComplete

A slight $24 profit at the breakeven level occurs because of rounding towhole units.

Step 3: Calculate the breakeven in units for each product line.

Because Hacker believes that it will sell four trainer clubs for every glove,the total breakeven, 953, is multiplied by each product’s respective percentof expected total sales:

So from this analysis, we know that Hacker needs to sell 763 trainer clubsand 191 gloves to break even. The breakeven point in sales dollars is:

Requirement 5

Use a contribution margin income statement to prove the breakeven pointcalculated in Requirement 4.

763 clubs $48 $36,624191 gloves $9 $ 1,719

$ !# $ !

TTotal $38,343!

Breakeven Sales of Clubs (953 80%) 762.4,$ ! roounded to 763Breakeven Sales of Gloves (9533 20%) 190.6,

954 total uni$ !

!rounded to191

tts

1eSG_C18_0131792075.Qxd 10/24/06 2:05 PM Page 622

Page 655: polpoa_sg

Quick Practice Questions | Chapter 18 623

Quick Practice QuestionsTrue/False

_____ 1. Total fixed costs don’t change as production levels decrease.

_____ 2. On a CVP graph, the vertical distance between the total expense lineand the total sales revenue line equals the operating income or loss.

_____ 3. Sensitivity analysis is a “what if” technique that asks what a resultwill be if a predicted amount is not achieved or if an underlyingassumption changes.

_____ 4. The margin of safety is the excess of breakeven sales over expectedsales.

_____ 5. The contribution margin is the band of volume where total fixed costsremain constant and the variable cost per unit remains constant.

_____ 6. Gray Company sells two products, X and Y. For the coming year, Graypredicts the sale of 5,000 units of X and 10,000 units of Y. The contribu-tion margins of the two products are $2 and $3, respectively. Theweighted-average contribution margin would be $2.50.

_____ 7. An easy method to separate mixed costs into variable and fixed compo-nents is the high-low method.

_____ 8. Sensitivity analysis is the combination of products that make up totalsales.

_____ 9. If a mixed cost has a high level of activity of 1,200 hours and a cost of$16,500 and a low level of activity of 800 hours and a cost of 12,500,the variable cost per unit is $5.

_____10. Using the information in question 9, the fixed cost would be $4,000.

Multiple Choice

1. A cost whose total amount changes in direct proportion to achange in volume is what type of cost? a. Fixed cost b. Variable cost c. Mixed cost d. Irrelevant cost

2. Which of the following is a fixed cost?a. Salary of plant managerb. Sales commissionsc. Direct materialsd. Delivery costs

1eSG_C18_0131792075.Qxd 10/24/06 2:05 PM Page 623

Page 656: polpoa_sg

624 Chapter 18 | Quick Practice Questions

3. What is the effect on total variable costs of changes inproduction?a. Remain the same as production levels changeb. Decrease as production increases c. Decrease as production decreases d. Increase as production decreases

4. What is the effect on fixed costs per unit of changes inproduction? a. Increase as production increases b. Decrease as production decreases c. Increase as production decreases d. Remain the same as production levels change

5. What is contribution margin? a. Fixed expenses plus variable expenses b. Sales revenues minus variable expenses c. Fixed expenses minus variable expenses d. Sales revenues minus fixed expenses

6. If the sale price per unit is $75, variable expenses per unit are$40, target operating income is $22,000, and total fixed expensesare $20,000, what is the total number of units that must be soldto reach the target operating income?a. 571b. 629c. 1,050d. 1,200

7. Canine Company produces and sells dog treats fordiscriminating pet owners. The unit selling price is $10, unitvariable costs are $7, and total fixed costs are $3,300. How manydog treats must Canine Company sell to break even? a. 194b. 330c. 471d. 1,100

8. Fixed Company produces a single product selling for $30 perunit. Variable costs are $12 per unit and total fixed costs are$4,000. What is the contribution margin ratio? a. 0.40 b. 0.60c. 1.67 d. 2.50

9. Which of the following will decrease the breakeven pointassuming no other changes in the cost-volume-profitrelationship? a. A decrease in the sale price per unit b. An increase in the sale price per unitc. An increase in total fixed costs d. An increase in the variable costs per unit

1eSG_C18_0131792075.Qxd 10/24/06 2:05 PM Page 624

Page 657: polpoa_sg

Quick Practice Questions | Chapter 18 625

10. Gould Enterprises sells computer disks for $1.50 per disk. Unitvariable expenses total $0.90. The breakeven sales in units are3,000 and budgeted sales in units are 4,300. What is the marginof safety?a. $ 780b. $1,950c. $2,580d. $4,500

Quick Exercises

18-1. Indicate whether each cost is typically a fixed cost (F), a vari-able cost (V), or a mixed cost (M).

a. __________ Packing materials b. __________ Executive salaries c. __________ Sales commissions d. __________ Direct materials e. __________ Units-of-production depreciation f. __________ Insurance expense g. __________ Building rent ($2,000 plus 5% of sales revenue

per month) h. __________ Property taxes i. __________ Delivery expenses j. __________ Photocopying machine rent (X amount per

month plus Y amount per copy)

18-2. Calculate the unknowns for the following situations based onthe given data.

Calculate the following items.

a. Breakeven point in dollars

b. Unit selling price

c. Unit contribution margin

Actual total sales revenueTotal fixed costUnnit variable costContribution margin ratio

$$400,00076,000

1540%

1eSG_C18_0131792075.Qxd 10/24/06 2:05 PM Page 625

Page 658: polpoa_sg

626 Chapter 18 | Quick Practice Questions

d. Breakeven point in units

e. Margin of safety

18-3. Robinson Company produces swim goggles and has gatheredthe following information:

Assuming breakeven sales in units of 30,000, compute:

a. Sales price per unit

b. Contribution margin ratio

c. Breakeven sales in dollars

18-4. Calculate the unknowns for the following situations based onthe following data. All situations are independent of each other.

a. Calculate the breakeven point in units.

b. Calculate the breakeven point in dollar sales.

Total fixed costsUnit sale priceUnit variablle cost

$180,00010040

Total fixed costsUnit variable costPlanned ssales in units

$156,0006

32,000

1eSG_C18_0131792075.Qxd 10/24/06 2:05 PM Page 626

Page 659: polpoa_sg

Quick Practice Questions | Chapter 18 627

c. Assume the unit sale price increases by 10%. Other data areunchanged. Calculate the breakeven point in units.

d. Assume the unit variable cost increases by 10%. Other data areunchanged. Calculate the breakeven point in units.

e. Assume total fixed costs increase by $5,000. Other data areunchanged. Calculate the breakeven point in units.

18-5. Ultimate Jelly Company manufactures two different types ofjelly, one with sugar (Jelly) and one without sugar (SimplyJelly). The following information is available for the two products:

Jelly Simply Jelly

Sale price per unit $5 $7

Variable expenses per unit $3 $6

Total fixed expenses are estimated at $381,500. Two jars of Jelly aresold for every three jars of Simply Jelly.

a. Determine the breakeven sales in units of both products.

b. Compute the target sales in dollars if Ultimate Jelly wants toearn $70,000 in operating income.

c. Prove the solution to Requirement 2 by calculating net incomeusing the targeted sales.

1eSG_C18_0131792075.Qxd 10/24/06 2:05 PM Page 627

Page 660: polpoa_sg

628 Chapter 18 | Do It Yourself! Question 1

Do It Yourself! Question 1

Easy Wear T-shirts prints T-shirts for local organizations. The average amount itcharges for a printed T-shirt is $10. Easy Wear calculated the average variablecost of a printed T-shirt to be $6. Its monthly fixed cost is $18,000.

Requirements

1. Use the contribution margin approach to calculate how many T-shirts EasyWear must sell in a month to break even.

2. Use the contribution margin ratio approach to determine Easy Wear’sbreakeven point in sales dollars.

3. Use the income statement approach to prove that the solutions toRequirement 1 and 2 are correct.

4. The owner of Easy Wear currently works for another T-shirt company andearns $3,200 per month. She doesn’t want to incur the risk of owning herown business unless she believes that she can have a profit of at least theamounts she currently earns. Determine the number of T-shirts Easy Wearmust print in a month to earn a profit of $3,200.

1eSG_C18_0131792075.Qxd 10/24/06 2:05 PM Page 628

Page 661: polpoa_sg

Do It Yourself! Question 2

Cool Board sells a snowboard for $240 that it can purchase for $100. It has addi-tional variable costs of $40 and a monthly fixed cost of $22,000.

Requirements Note: Solve each requirement as a separate and independent situation.

1. Calculate Cool Board’s breakeven point in units.

2. Cool Board is considering raising its selling price to $249. Calculate the newbreakeven in units.

3. Cool Board has found a new supplier for the snowboards, who will sell theboard to Cool for $95. Calculate the new breakeven in units.

4. Cool Board has had many requests from customers for bindings to goalong with the board. Cool believes that for every three boards it sells, itcould sell two bindings. Cool can purchase the bindings for $25 and wouldincur another $5 in other variable costs for a total variable cost on thebindings of $30. Cool can sell the bindings for $55.Total fixed costs shouldremain the same at $22,000 per month. Calculate the breakeven point inunits for snowboards and bindings.

Do It Yourself! Question 2 | Chapter 18 629

1eSG_C18_0131792075.Qxd 10/24/06 2:05 PM Page 629

Page 662: polpoa_sg

630 Chapter 18 | Do It Yourself! Question 2

5. Use a contribution margin income statement to prove the breakeven pointcalculated in Requirement 4.

1eSG_C18_0131792075.Qxd 10/24/06 2:05 PM Page 630

Page 663: polpoa_sg

Quick Practice SolutionsTrue/False

T 1. Total fixed costs don’t change as production levels decrease. (p. 959)

T 2. On a CVP graph, the vertical distance between the total expense line and the total sales revenue line equals the operating income or loss.(p. 968)

T 3. Sensitivity analysis is a “what if” technique that asks what a result will be if a predicted amount is not achieved or if an underlying assumption changes. (p. 969)

F 4. The margin of safety is the excess of breakeven sales over expected sales.False–The margin of safety is the excess of actual sales over breakeven sales. (p. 971)

F 5. The contribution margin is the band of volume where total fixed costs remain constant and the variable cost per unit remains constant.False - The relevant range is the band of volume where total fixed costsremain constant and the variable cost per unit remains constant.(p. 965)

F 6. Gray Company sells two products, X and Y. For the coming year,Gray predicts the sale of 5,000 units of X and 10,000 units of Y. The contribution margins of the two products are $2 and $3, respectively.The weighted-average contribution margin would be $2.50.False–The weighted-average contribution margin would be $2.67 (p. 965)

Sales Mix: 5,000 units/15,000 units ! 1/3; 10,000 units/15,000 units ! 2/3;Sales Mix ! 1:2 (1 ! $2) # (2 ! $3) ! $8; $8/3 ! $2.67/unit

T 7. An easy method to separate mixed costs into variable and fixed components is the high-low method. (p. 961)

F 8. Sensitivity analysis is the combination of products that make up total sales.False–Sales mix is the combination of products that make up total sales. (p. 972)

F 9. If a mixed cost has a high level of activity of 1,200 hours and a cost of $16,500 and a low level of activity of 800 hours and a cost of 12,500, the variable cost per unit is $5.False–If a mixed cost has a high level of activity of 1,200 hours and a cost of $16,500 and a low level of activity of 800 hours and a cost of12,500, the variable cost/unit is $10. ($16,500 – $12,500)/(1,200 – 800) = $10/hour. (p. 960)

T 10. Using the information in question 9, the fixed cost would be $4,000.(p. 960)

Quick Practice Solutions | Chapter 18 631

1eSG_C18_0131792075.Qxd 10/24/06 2:05 PM Page 631

Page 664: polpoa_sg

632 Chapter 18 | Quick Practice Solutions

Multiple Choice

1. A cost whose total amount changes in direct proportion to achange in volume is what type of cost? (p. 958)a. Fixed cost b. Variable costc. Mixed cost d. Irrelevant cost

2. Which of the following is a fixed cost? (p. 959)a. Salary of plant managerb. Sales commissionsc. Direct materialsd. Delivery costs

3. What is the effect on total variable costs of changes inproduction? (p. 958)a. Remain the same as production levels change b. Decrease as production increases c. Decrease as production decreasesd. Increase as production decreases

4. What is the effect on fixed costs per unit of changes inproduction? (p. 959)a. Increase as production increases b. Decrease as production decreases c. Increase as production decreasesd. Remain the same as production levels change

5. What is contribution margin? (p. 964)a. Fixed expenses plus variable expenses b. Sales revenues minus variable expensesc. Fixed expenses minus variable expenses d. Sales revenues minus fixed expenses

6. If the sale price per unit is $75, variable expenses per unit are$40, target operating income is $22,000, and total fixed expensesare $20,000, what is the total number of units that must be soldto reach the target operating income? (p. 967)a. 571b. 629c. 1,050d. 1,200

7. Canine Company produces and sells dog treats fordiscriminating pet owners. The unit selling price is $10, unitvariable costs are $7, and total fixed costs are $3,300. How manydog treats must Canine Company sell to break even? (p. 963)a. 194b. 330c. 471d. 1,100

1eSG_C18_0131792075.Qxd 10/24/06 2:05 PM Page 632

Page 665: polpoa_sg

Quick Practice Solutions | Chapter 18 633

8. Fixed Company produces a single product selling for $30 perunit. Variable costs are $12 per unit and total fixed costs are$4,000. What is the contribution margin ratio? (p. 966)a. 0.40 b. 0.60c. 1.67d. 2.50

9. Which of the following will decrease the breakeven pointassuming no other changes in the cost-volume-profitrelationship? (pp. 963–966)a. A decrease in the sale price per unitb. An increase in the sale price per unitc. An increase in total fixed costs d. An increase in the variable costs per unit

10. Gould Enterprises sells computer disks for $1.50 per disk. Unitvariable expenses total $.90. The breakeven sales in units are3,000 and budgeted sales in units are 4,300. What is the marginof safety? (p. 971)a. $ 780b. $1,950c. $2,580d. $4,500

Quick Exercise Solutions

18-1. Indicate whether each cost is typically a fixed cost (F), a vari-able cost (V), or a mixed cost (M). (pp. 958–960)

a. V Packing materials b. F Executive salaries c. V Sales commissions d. V Direct materials e. V Units-of-production depreciation f. F Insurance expense g. M Building rent ($2,000 plus 5% of sales revenue

per month) h. F Property taxes i. V Delivery expenses j. M Photocopying machine rent (X amount per

month plus Y amount per copy)

18-2. Calculate the unknowns for the following situations based onthe given data. (pp. 958–966)

Actual total sales revenueTotal fixed costUnnit variable costContribution margin ratio

$$400,00076,000

1540%

1eSG_C18_0131792075.Qxd 10/24/06 2:05 PM Page 633

Page 666: polpoa_sg

634 Chapter 18 | Quick Practice Solutions

Calculate the following items.

a. Breakeven point in dollars$76,000/0.40 = $190,000

b. Unit selling price $15/(100% – 40%) = $25

c. Unit contribution margin $25 ! 0.40 = $10, or $25 – $15 = $10

d. Breakeven point in units $76,000/$10 = 7,600 units

e. Margin of safety $400,000 – $190,000 = $210,000

18-3. Robinson Company produces swim goggles and has gatheredthe following information: (pp. 958–966)

Assuming breakeven sales in units of 30,000, compute:

a. Sales price per unit Let X = contribution margin per unit $156,000/X = 30,000 $156,000 = 30,000XX = $156,000/30,000 = $5.20 $5.20 + $6 = $11.20

b. Contribution margin ratio $5.20/$11.20 = 0.4643

c. Breakeven sales in dollars 30,000 ! $11.20 = $336,000, or $156,000/0.4643 = $336,000,rounded

18-4. Calculate the unknowns for the following situations based onthe following data. All situations are independent of eachother. (pp. 963–966)

a. Calculate the breakeven point in units.$180,000/($100 – $40) = 3,000

b. Calculate the breakeven point in dollar sales.3,000 units ! $100 = $300,000, or $180,000/0.60 = $300,000

c. Assume the unit sale price increases by 10%. Other data areunchanged. Calculate the breakeven point in units.$100 ! 1.10 = $110 – $40 = $70 $180,000/$70 = 2,571 units (rounded)

Total fixed costsUnit sale priceUnit variablle cost

$180,00010040

Total fixed costsUnit variable costPlanned ssales in units

$156,0006

32,000

1eSG_C18_0131792075.Qxd 10/24/06 2:05 PM Page 634

Page 667: polpoa_sg

Quick Practice Solutions | Chapter 18 635

d. Assume the unit variable cost increases by 10%. Other data areunchanged. Calculate the breakeven point in units.$40 ! 1.10 = $44; $100 – $44 = $56 $180,000/$56 = 3,214 units (rounded)

e. Assume total fixed costs increase by $5,000. Other data areunchanged. Calculate the breakeven point in units.$185,000/$60 = 3,083 units (rounded)

18-5. Ultimate Jelly Company manufactures two different types ofjelly, one with sugar (Jelly) and one without sugar (SimplyJelly). The following information is available for the twoproducts: (pp. 963–966)

Jelly Simply Jelly

Sale price per unit $5 $7

Variable expenses per unit $3 $6

Total fixed expenses are estimated at $381,500. Two jars of Jelly aresold for every three jars of Simply Jelly.

a. Determine the breakeven sales in units of both products.$5 – $3 = $2$2 ! 2 = $4

$7 – $6 = $1$1 ! 3 = $3

$4 + $3 = $7$7/5 = $1.40 weighted-average contribution margin

$381,500/$1.40 = 272,500 sets 272,500 ! 2/(2 + 3) = 109,000 units of Jelly

272,500 ! 3/(2 + 3) = 163,500 units of Simply Jelly

b. Compute the target sales in dollars if Ultimate Jelly wants toearn $70,000 in operating income.

$381,500 + $70,000 = $451,500$451,500/$1.40 = 322,500 sets

322,500 ! 2/(2 + 3) = 129,000 units129,000 units ! $5 = $645,000 Jelly

322,500 ! 3/(2 + 3) = 193,500 units193,500 units ! $7 = $1,354,500 Simply Jelly

$645,000 + $1,354,500 = $1,999,500

c. Prove the solution to Requirement 2 by calculating net incomeusing the targeted sales.

Revenue – Variable Costs – Fixed Costs = Net Income$1,999,500 – (129,000 ! $3) – (193,500 ! $6) – $381,500 = $70,000

1eSG_C18_0131792075.Qxd 10/24/06 2:05 PM Page 635

Page 668: polpoa_sg

636 Chapter 18 | Do It Yourself! Question 1 Solution

2 Use CVP analysis tocompute breakevenpoints

2 Use CVP analysis tocompute breakevenpoints

2 Use CVP analysis tocompute breakevenpoints

3 Use CVP analysis forprofit planning, andgraph the cost-volume-profit relations

Do It Yourself! Question 1 Solution

Requirement 1

Use the contribution margin approach to calculate how many T-shirts EasyWear must sell in a month to break even.

Requirement 2

Use the contribution margin ratio approach to determine Easy Wear’sbreakeven point in sales dollars.

Contribution Margin Ratio ! Contribution Margin per Unit/Sales Revenue per Unit

Contribution Margin Ratio ! $4/$10 ! 0.40 (or 40%)Breakeven in Sales Dollars ! Fixed Cost/Contribution Margin RatioBreakeven in Sales Dollars ! $18,000/0.40 ! $45,000

Requirement 3

Use the income statement approach to prove that the solutions toRequirements 1 and 2 are correct.

Requirement 4

The owner of Easy Wear currently works for another T-shirt company andearns $3,200 per month. She doesn’t want to incur the risk of owning herown business unless she believes that she can have a profit of at least theamount she currently earns. Determine the number of T-shirts Easy Wearmust print in a month to earn a profit of $3,200.

Desired Profit Sales Level in Units ! (Fixed Cost # Desired Profit)/Contribution Margin

Desired Profit Sales Level in Units ! ($18,000 # $3,200)/($10 – $6) Desired Profit Sales Level in Units ! $21,200/$4 ! 5,300 Units

Another way to think of this calculation is to divide your desired profit by yourcontribution margin and add the difference to your breakeven:

$3,200/$4 800 units800 units 4,500 breakeve

!# nn 5,300 units!

Sales Revenue ($10 4,500 Units) $45,000Less:

$

Variable Costs (4,500 $6) 27,000Total Co

$

nntribution Margin 18,000Less: Fixed Costs 188,000Operating Income $ 0

Contribution Margin per Unit Sales Price pe! rr Unit Variable Cost per UnitSales Price p

"

eer UnitVariable Cost per UnitContribution MMargin per Unit

$10(6)

$ 4

Breakeven in Unnits Fixed Cost/Contribution Margin per Un! iitBreakeven in Units $18,000/$4! ! 4,500 T-shiirts

1eSG_C18_0131792075.Qxd 10/24/06 2:05 PM Page 636

Page 669: polpoa_sg

Do It Yourself! Question 2 Solution | Chapter 18 637

2 Use CVP analysis tocompute breakevenpoints

4 Use CVP methods toperform sensitivityanalyses

4 Use CVP methods toperform sensitivityanalyses

5 Calculate thebreakeven point formultiple product linesor services

Do It Yourself! Question 2 Solution

Requirement 1

Calculate Cool Board’s breakeven point in units.

Breakeven in Units ! Fixed Cost/(Selling Price per Unit – Variable Cost per Unit)Breakeven in Units ! $22,000/($240 – $140) ! 220 Boards

Requirement 2

Cool Board is considering raising its selling price to $249. Calculate the newbreakeven in units.

Breakeven in Units ! Fixed Cost/(Selling Price per Unit – Variable Cost per Unit)

Breakeven in Units ! $22,000/($249 – $140) ! 201.83, Rounded to 202 Snowboards

Requirement 3

Cool Board has found a new supplier for the snowboards, who will sell theboard to Cool Board for $95. Calculate the new breakeven in units.

Breakeven in Units ! Fixed Cost/(Selling Price per Unit – Variable Cost per Unit)Breakeven in Units ! $22,000/($240 – $135) ! 209.52, Rounded to 210 Snowboards

Requirement 4

Cool Board has had many requests from customers for bindings to goalong with the board. Cool Board believes that for every three boards itsells, it could sell two bindings. Cool Board can purchase the bindings for$25 and would incur another $5 in other variable cost for a total variablecost on the bindings of $30. Cool Board can sell the bindings for $55.Totalfixed costs should remain the same at $22,000 per month. Calculate thebreakeven point in units for snowboards and bindings.

Step 1: Calculate the weighted-average contribution margin.

Boards Bindings Total

Sales price per unit $240 $55

Less: Variable cost per unit (140) (30)

Contribution margin per unit $100 $25

Sales mix in units ! 3 ! 2 5

Contribution margin $300 $50 $350

Weighted-average contribution margin per unit ($350/5) $70

Step 2: Calculate the breakeven point in units for the total of bothproducts combined.

Total Sales in Units ! Fixed Cost/Weighted-Average Contribution Margin per Unit

Total Sales in Units ! $22,000/$70 ! 314.28, rounded to 315

1eSG_C18_0131792075.Qxd 10/24/06 2:05 PM Page 637

Page 670: polpoa_sg

638 Chapter 18 | Do It Yourself! Question 2 Solution

5 Calculate thebreakeven point formultiple product linesor services

Sales revenue: Boards (189 3 $240) Bindings (126 3 $55)Variable costs: Boards (189 3 $140) Bindings (126 3 $30)Contribution marginFixed costsOperating income

$6,930

3,780$3,150

$45,360

26,460

$18,900

$52,290

$30,240$22,050(22,000)$ 50

BindingsBoards Total

COOL BOARDSContribution Margin Income Statement

Step 3: Calculate the breakeven in units for each product line.

Requirement 5

Use a contribution margin income statement to prove the breakeven pointcalculated in Requirement 4.

A slight $50 profit occurs at the breakeven level because of rounding to wholeunits.

Breakeven Sales of Boards 315 (3/5) S$ !189 nowwboardsBreakeven Sales of Gloves 315 (2/5)$ !!126 indingsB

315 Total Units

1eSG_C18_0131792075.Qxd 10/24/06 2:05 PM Page 638

Page 671: polpoa_sg

The Power of PracticeFor more practice using the skills learned in this chapter, visit MyAccountingLab.There you will find algorithmically generated questions that are based on theseDemo Docs and your main textbook’s Review and Accounting Practice sections.

To go to MyAccountingLab and follow these steps:

1. Direct your URL to www.myaccountinglab.com.2. Log in using your name and password.3. Click the MyAccountingLab link.4. Click Study Plan in the left navigation bar.5. From the table of contents, select Chapter 18, Cost-Volume-Profit Analysis.6. Click a link to work on the tutorial exercises.

The Power of Practice | Chapter 18 639

1eSG_C18_0131792075.Qxd 10/24/06 2:05 PM Page 639

Page 672: polpoa_sg

1eSG_C18_0131792075.Qxd 10/24/06 2:05 PM Page 640

Page 673: polpoa_sg

The Master Budgetand ResponsibilityAccounting

19

1

2

WHAT YOU PROBABLY ALREADY KNOW

You may have wanted to purchase something you’ve had to save forover a period of time. To project how you would accomplish yourgoal, you might have made a plan. You would consider the amountof money available at the beginning of your plan plus the forecastedcash receipts less cash disbursements over the period to reach yourgoal. You probably already know that by addressing your financialgoal and creating a cash budget, you are able to plan for the futureand make decisions to facilitate achieving your goal. If the period oftime is sufficiently long, you can compare your actual to yourbudgeted cash flows during interim periods and make changes aswarranted. Some of these may include:

• Initiate steps to increase cash inflows, perhaps work more hours.• Initiate steps to decrease cash outflows, cut back on lesser-

important spending.• Initiate steps to finance the shortfall, pursue borrowing

opportunities.• Postpone or abandon the purchase plan.

In this chapter, we will study the various budgets thatcompanies create and look at the usefulness of the budgetingprocess for all financial entities.

Learning Objectives

Learn how to use a budget.

Budgeting helps managers plan and control their actions. Managementcreates organizational goals. Action steps are planned to achieve thosegoals. The budget is the anticipated financial results of taking thoseaction steps. The actual results can be compared to the budget andcorrective action taken where necessary. Review Exhibits 19-2 and 19-3(pp. 1009–-1010) for the usefulness and benefits of budgets.

Prepare an operating budget.

The sales budget is the first part of the operating budget that must beprepared. Expected sales units and future sales prices are used in thebudget. This budget will drive the remainder of the operating budget,such as the cost of goods sold, inventory, purchase, and operatingexpense budgets. Review Exhibits 19-6 through 19-8 (p. 1013) forexamples of operating budgets.

1eSG_C19_0131792075.Qxd 10/24/06 2:07 PM Page 641

Page 674: polpoa_sg

642 Chapter 19 | The Master Budget and Responsibility Accouting

4

3 Prepare a financial budget.

The financial budget includes the cash and balance sheet budgets. Thecash budget contains the beginning cash balance, projected cashcollections, cash payments, financing required, and the ending cashbalance. The period in which items as sales and purchases on accountresult in cash flows must be projected and integrated into the cashbudget. After completing all of the operating and cash budgets, aprojection of the balance sheet account balances can be determined.This information is used to prepare the balance sheet budget. ReviewExhibits 19-10 to 19-13 (pp. 1017–1019) for components of a cash budget.Review Exhibit 19-15 (p. 1023) for the computation of balance sheetprojected account balances and Exhibit 19-14 (p. 1022) for a budgetedbalance sheet.

Prepare performance reports for responsibility centers.

A responsibility center is a part or subunit of an organization whosemanager is accountable for specific activities. Performance reportscompare the budget to the actual results for each responsibility center.The performance of the managers of each center can then be evaluated.There are four responsibility centers:

• Cost center. Only costs or expenses are incurred. The humanresources, acounting, and information technology departments areexamples of cost centers. The goal of the manager is to minimizecosts.

• Revenue center. Primarily revenues are incurred although somerelated expenses may also be relevant. A sales department would bea revenue center. Managers are assessed based upon the revenuesgenerated. The goal of the manager is to maximize revenues.

• Profit center. Responsible for revenues and expenses, and ultimatelyprofits or net income. The goal of the manager is to exceed the profitprojection. The sales and costs related to the Lipton Cup-a-Soupproduct line would be an example of a profit center.

• Investment center. Responsible for the results of the investment aswell as the profits of the entity. The foods division of Lipton would beconsidered an investment center. Managers are responsible for mak-ing sales, maintaining expenses, and managing the investmentrequired to generate profits.

Review Exhibit 19-17 (p. 1028) for examples of responsibility centers andresponsibility accounting performance reports in Exhibit 19-19 (p. 1027).

1eSG_C19_0131792075.Qxd 10/24/06 2:07 PM Page 642

Page 675: polpoa_sg

Demo Doc 1 | Chapter 19 643

Demo Doc 1Master BudgetLearning Objectives 2

Joe University sells college sweatshirts. Actual sales for the month endedSeptember 30, 2008, were $20,000. Joe expects sales to increase 8% in Octoberand increase another 4% over October sales in November. Cash sales areexpected to be 60% of total sales and credit sales about 40% of sales. Cost of goodssold should be 60% of total sales. Joe doesn’t want inventory to fall below $4,000plus 10% of cost of goods sold for the next month. Sales of $25,000 are expectedfor December. Inventory on September 30 is $6,000.

Operating expenses include sales commission, 10% of sales; rent expense of$1,000; depreciation expense of $1,200; utility expense of $800; and insuranceexpense of $400.

Round all figures to the nearest dollar.

Requirement

1. Prepare the following budgets for October and November:

a. Sales budget

b. Inventory, purchases, and cost of goods sold budget

c. Operating expense budget

d. Budgeted income statement

1eSG_C19_0131792075.Qxd 10/24/06 2:08 PM Page 643

Page 676: polpoa_sg

644 Chapter 19 | Demo Doc 1 Solution

We prepare the sales budget first because sales affect most elements of theother budgets we will be preparing for this period.In order to complete the sales budget, we start by calculating the total salesfor each month. We will then compute the split between cash sales and creditsales for each month based on Joe’s estimation that cash sales will be 60% ofthe total sales for each month and credit sales will be 40% of total sales foreach month.Let’s begin by calculating Joe’s total sales for October and November. Thequestion tells us that actual sales for the month ended September 30 were$20,000, and that Joe expects sales to increase by 8% over that in October andanother 4% over October’s sales in November:

So we begin to build our sales budget with these data:

October Total SalesOctober Total Sales

Novembber Total SalesNovember Total Sales

Se! pptember Sales 108%$20,000 108% $21,60

"

! " ! 00

October Sales 104%$21,600 104% $2

! "

! " ! 22,464

2 Prepare an operatingbudget

Demo Doc 1 Solution

Part 1 Part 2 Part 3 Part 4 Demo DocComplete

Cash sales, 60%Credit sales, 40%Total sales $22,464$21,600 $44,064

NovemberOctober Total

JOE UNIVERSITYSales Budget

Now we work backwards to calculate the split between cash and credit salesfor each month. In this case, cash sales are 60% of total sales and credit sales are40% of total sales for the current months:

Cash Sales To! ttal sales 60%October Cash Sales

November Ca

"

ssh Sales$21,600 60% $12,960! " !

! $$22,464 60% $13,478.40 (rounded to $13,4" ! 778)

Credit Sales ! Total sales 40%October Credit Sales

Novem

"

bber Credit Sales$21,600 40% $8,640! !!

$22,464 40% $8,985.60 (rounded to! !! $8,986)

Requirement

1. Prepare the following budgets for October and November:

a. Sales budget

1eSG_C19_0131792075.Qxd 10/24/06 2:08 PM Page 644

Page 677: polpoa_sg

Demo Doc 1 Solution | Chapter 19 645

Following is the completed sales budget:

Part 1 Part 2 Part 3 Part 4 Demo DocComplete

Cash sales, 60%Credit sales, 40%Total sales

$13,4788,986

$22,464

$12,9608,640

$21,600

$26,43817,626

$44,064

NovemberOctober Total

JOE UNIVERSITYSales Budget

A total sales budget for October and November comes to $44,064, with 60% ofthat ($26,438.40, rounded to $26,438) from cash and 40% ($17,625.60,rounded to $17,626) from credit.Because the sales budget calculates values that you will use when preparingother budgets, it’s always a good idea to check your work. These calculationscan be performed in a number of ways. Here’s one alternative:

b. Inventory, purchases, and cost of goods sold budget

October Total Sales Previous Month! ""

! "

!

108%$20,000 108%$21,600

October Caash Sales October Expected Sales! "660%$21,600 60%$12,960

October Credi

! "

!tt Sales October Expected Sales 40%! "

!! "

!

$21,600 40%$8,640

November Total Salees Previous Month Sales 104%$21,6

! "

! 000 104%$22,464

November Cash Sales

"

!! November Expected Sales 60%

$22,464 60"

! " %%$13,478

November Credit Sales Novem!! bber Expected sales 40%

$22,464 40%$

"

! "

! 88,986

The inventory, purchases, and cost of goods sold budget statement takes thefollowing format:

Cost of Goods Sold# Desired Ending Inventory

Total Inventory Required– Beginning Inventory

Purchases

2 Prepare an operatingbudget

1eSG_C19_0131792075.Qxd 10/24/06 2:08 PM Page 645

Page 678: polpoa_sg

646 Chapter 19 | Demo Doc 1 Solutions

So we first calculate the cost of goods sold. We know from the question thatcost of goods sold is expected to be 60% of total sales for the period. From thesales budget, we know that total sales for October are expected to be $21,600,and total sales for November are expected to be $22,464. So we can calculatecost of goods sold as follows:

So here’s our budget so far:

Cost of Goods Sold 60% of Budgeted Sal! ees from the Sales BudgetOctober $21! ,,600 60% $12,960

November $22,46" !

! 44 60% $13,478" !

Next, we need to add the desired ending inventory for each month. The ques-tion states that Joe doesn’t want inventory to fall below $4,000 plus 10% ofcost of goods sold for the next month. So in order to calculate the desired end-ing inventory for November, we need to know the cost of goods sold forDecember. The question tells us that December’s sales are expected to be$25,000. Returning to our calculation for cost of goods sold:

Desired ending inventory is now calculated as follows:

We can now calculate the total ending inventory required:

Desired Ending Inventory $4,000 10% o! # ff Cost of Goods Sold for theNext MonthOctober $4,000! # (10% $13,478) $5,348

November $4,00" !

! 00 (10% $15,000) $5,500# " !

Cost of Goods Sold 60% of Budgeted Sales! ffrom the Sales BudgetDecember ! $$25,000 60% $15,000" !

Cost of goods sold! Desired ending inventory Total inventory required" Beginning inventory Purchases

$13,478$12,960NovemberOctober

JOE UNIVERSITYInventory, Purchases, and Cost of Goods Sold Budget

Cost of goods sold! Desired ending inventory Total inventory required" Beginning inventory Purchases

$13,4785,500

$18,978

$12,9605,348

$18,308

NovemberOctober

JOE UNIVERSITYInventory, Purchases, and Cost of Goods Sold Budget

1eSG_C19_0131792075.Qxd 10/24/06 2:08 PM Page 646

Page 679: polpoa_sg

Demo Doc 1 Solution | Chapter 19 647

2 Prepare an operatingbudget

Cost of goods sold! Desired ending inventory Total inventory required" Beginning inventory Purchases

$13,4785,500

$18,9785,348

$13,630

$12,9605,348

$18,3086,000

$12,308

NovemberOctober

JOE UNIVERSITYInventory, Purchases, and Cost of Goods Sold Budget

c. Operating expense budget

With the exception of the sales commission, which we know from the questionto be 10% of sales, all expenses remain constant between October andNovember, as follows:

Part 1 Part 2 Part 3 Part 4 Demo DocComplete

Sales commissionDepreciation expenseRent expenseUtility expenseInsurance expenseTotal operating expenses

1,2001,000

800400

1,2001,000

800400

2,4002,0001,600

800

NovemberOctober Total

JOE UNIVERSITYOperating Expense Budget

Beginning inventory is equal to the previous month’s desired ending inven-tory. We are told in the question that the inventory on September 30 is $6,000,so this figure becomes October’s beginning inventory. Once we determinebeginning inventory, we subtract it from the total inventory required to deter-mine total purchases for the period:

So the only calculation to perform here is sales commission. We can computesales commissions for October and November using the respective sales com-putations ($21,600 and $22,464) from the sales budget:

So here’s our completed operating expense budget for October and November:

Sales Commission Ex! ppected Sales 10%October Sales Commission

"

!! " !$21,600 10% $2,160November Sales Commisssion $22,464 10% $2,246.40 (rounded t! " ! oo $2,246)

1eSG_C19_0131792075.Qxd 10/24/06 2:08 PM Page 647

Page 680: polpoa_sg

648 Chapter 19 | Demo Doc 1 Solution

d. Budgeted income statement

The results of the budgets you’ve created so far are carried over into thefourth element: the budgeted income statement.Sales revenue is traced from the sales budget in part a.Cost of goods sold is traced from the inventory, purchases, and cost of goodssold budget in part b.We compute gross profit by subtracting the cost of goods sold from sales revenue:

Operating expenses are traced from the operating expenses budget from part c,We compute net income (loss) by subtracting operating expenses from grossprofit. So our completed budgeted income statement looks like this:

2 Prepare an operatingbudget

Sales commissionDepreciation expenseRent expenseUtility expenseInsurance expenseTotal operating expenses

NovemberOctober Total

JOE UNIVERSITYOperating Expense Budget

$2,2461,2001,000

800400

$5,646

$2,1601,2001,000

800400

$5,560

$ 4,4062,4002,0001,600

800$11,206

Part 1 Part 2 Part 3 Part 4 Demo DocComplete

NovemberOctober Total

JOE UNIVERSITYOperating Expense Budget

Sales revenueCost of goods soldGross profitOperating expensesNet income

$22,46413,4788,986

$21,60012,9608,640

$44,06426,43817,626

NovemberOctober Total

JOE UNIVERSITYOperating Expense Budget

Sales revenueCost of goods soldGross profitOperating expensesNet income

$22,46413,4788,9865,646

$ 3,340

$21,60012,9608,6405,560

$ 3,080

$44,06426,43817,62611,206

$ 6,420

1eSG_C19_0131792075.Qxd 10/24/06 2:08 PM Page 648

Page 681: polpoa_sg

Demo Doc 1 Solution | Chapter 19 649

Part 1 Part 2 Part 3 Part 4 Demo DocComplete

Sales revenueCost of goods soldGross profitOperating expenses: Salary and commissions Depreciation expense Rent expense Utility expense Insurance expenseOperating income

$4,4062,4002,0001,600

800

$44,06426,43817,626

11,206$ 6,420

JOE UNIVERSITYBudgeted Income Statement

Two months Ending November 30, 2008

So, for the period, our totals are as follows:

1eSG_C19_0131792075.Qxd 10/24/06 2:08 PM Page 649

Page 682: polpoa_sg

650 Chapter 19 | Demo Doc 2

Demo Doc 2Financial BudgetLearning Objectives 3

Joe University prepared its sales budget; inventory, purchases, and cost of goodssold budget; operating expense budget; and budgeted income statement. Joewould now like to prepare the cash budget for the months of October andNovember.

Actual sales for the month ended September 30, 2008, were $20,000. Actualsales for the month ended August 31 were $16,000. Joe believes that sales willincrease 8% in October and increase another 4% over October sales in November.Cash sales are expected to be 60% of sales and credit sales about 40% of sales.Cost of goods sold is expected to be 60% of total sales. Joe doesn’t want inventoryto fall below $4,000 plus 10% of cost of goods sold for the next month. Sales of$25,000 are expected for December. Joe purchased $11,000 inventory duringSeptember and ended the month with $6,000 in ending inventory.

Operating expenses include sales commission, 10% of sales; rent expense of$1,000; depreciation expense of $1,200; utility expense of $800; and insuranceexpense of $400.

September 30 cash balance was $4,000.Of the credit sales, Joe expects to collect 70% in the month following the sale

and the remaining 30% in the next month. Purchases made by Joe University arepaid for in the month after the purchase. Sales commissions are paid 50% in themonth incurred and 50% in the next month. Rent, utility, and insurance are paidin the month incurred.

Requirements

1. For the months of October and November, prepare the following:

a. Budgeted cash collections from customers

b. Budgeted cash payments for purchases

c. Budgeted cash payments for operating expenses

d. The cash budget

1eSG_C19_0131792075.Qxd 10/24/06 2:08 PM Page 650

Page 683: polpoa_sg

Demo Doc 2 Solution | Chapter 19 651

Cash sales, 60%Credit sales, 40%Total sales

$12,0008,000

$20,000

$ 9,6006,400

$16,000

September$12,960

8,640$21,600

October$13,478

8,986$22,464

NovemberAugust

JOE UNIVERSITYSales Budget

Demo Doc 2 Solution

Part 1 Part 2 Part 3 Part 4 Demo DocComplete

Armed with the budget data we calculated in Demo Doc 1, we can start toprepare the cash budget. We begin with the budgeted cash collections fromcustomers, which we’ll use in part d of this Demo Doc to calculate total cashavailable for the period.We computed the cash sales for October and November on the sales budgetas $12,960 and $13,478, respectively. To cash sales we add any credit collec-tions that Joe makes in this period.Joe expects to collect on credit sales at a rate of 70% in the month followingthe sale and 30% in the next month. In October, then, Joe expects to collect70% of any credit sales made in September and 30% of credit sales made inAugust. Likewise, in November, Joe will collect 70% of October’s credit salesand 30% of September’s credit sales.Before we can calculate how much in credit sales Joe will collect in Octoberand November, we need to see the sales data for August and Septemberbecause collections are being made from those two months. We know fromthe question that total August sales were $16,000 and total September saleswere $20,000. We also know that Joe expects 40% of total sales each monthto be credit sales. So if we were to figure Joe’s sales budget data for Augustand September, credit sales would look like this:

Given these data, Joe’s sales budget for the period August through Novemberwould look like the following (although note that the only data you’re inter-ested in at this point are the highlighted data):

August Credit Sales August Total S! aales 40%$16,000 40%$6,400

Sep

"

! "

!

ttember Credit Sales September TotalSale! ss 40%$20,000 40%$8,000

"

! "

!

3 Prepare a financialbudget

Requirement

1. For the months of October and November, prepare the following:

a. Budgeted cash collections from customers

1eSG_C19_0131792075.Qxd 10/24/06 2:08 PM Page 651

Page 684: polpoa_sg

652 Chapter 19 | Demo Doc 2 Solution

Remember that credit sales for October and November came from the salesbudget in Demo Doc 1.So for October, Joe expects to collect 30% of credit sales made in August and70% of credit sales made in September:

October Collections from August Credit Sales ! $6,400 " 30% ! $1,920October Collections from September Credit Sales ! $8,000 " 70% ! $5,600

For November, Joe expects to collect 30% of credit sales made in Septemberand 70% of credit sales made in October ($8,640, taken from the sales budgetcreated in Demo Doc 1, part a):

Cash sales on the statement come directly from the sales budget. So the com-pleted budget looks like this:

November Collections from September Credit SSales $8,000 30%$2,400

November Collection

! "

!

ss from October Credit Sales $8,640 7! " 00%$6,048!

Cash salesCollections of previous month’s credit salesCollections of two-months previous credit salesTotal collections

$13,4786,0482,400

$21,926

$12,9605,6001,920

$20,480

$26,43811,648

4,320$42,406

NovemberOctober Total

JOE UNIVERSITYBudgeted Cash Collections from Customers

Shown another way:

October November Total

Month of Sale:

August $16,000

CashCredit 16,000 # 40% # 30% 1,920 1,920

September $20,000Cash 20,000 # 40% # 70% 5,600Credit 20,000 # 40% # 30% 2,400 8,000

October $21,600Cash 60% # 21,600 12,960Credit 21,600 # 40% # 70% 6,048 19,008

November $22,464Cash 60% # 22,464 13,478 13,478CreditTOTALS 20,480 21,926 42,406

1eSG_C19_0131792075.Qxd 10/24/06 2:08 PM Page 652

Page 685: polpoa_sg

Demo Doc 2 Solution | Chapter 19 653

b. Budgeted cash payments for purchases

Purchases made by Joe are paid in the month following the purchase. So inOctober, Joe paid for the purchases made in September and in November, Joepaid for the purchases made in October.The question states that Joe purchased $11,000 of inventory in September(which he will pay for in October). We know from the inventory, purchases,and cost of goods sold budget that we prepared in Demo Doc 1 (part b) of thischapter that Joe’s purchases in October were $12,308 (which he will pay forin November):

c. Budgeted cash payments for operating expenses

Following is Joe’s operating expense budget, which we prepared in Demo Doc 1,part c:

3 Prepare a financialbudget

Purchases paid from previous monthTotal cash payments for purchases

$12,308$12,308

$11,000$11,000

$23,308$23,308

NovemberOctober Total

JOE UNIVERSITYBudgeted Cash Payments for Purchases

Part 1 Part 2 Part 3 Part 4 Demo DocComplete

NovemberOctober TotalSales commissionDepreciation expenseRent expenseUtility expenseInsurance expenseTotal operating expenses

$2,2461,2001,000

800400

$5,646

$2,1601,2001,000

800400

$5,560

$ 4,4062,4002,0001,600

800$11,206

JOE UNIVERSITYOperating Expense Budget

Part 1 Part 2 Part 3 Part 4 Demo DocComplete

The question tells us that sales commissions are paid 50% in the monthincurred and 50% in the next month. Other expenses are paid in the monthincurred. So we must calculate the sales commission payments for the cur-rent month and for the previous month each for October and November.

3 Prepare a financialbudget

1eSG_C19_0131792075.Qxd 10/24/06 2:08 PM Page 653

Page 686: polpoa_sg

654 Chapter 19 | Demo Doc 2 Solution

We can calculate commissions from the current month for October andNovember as:

So far, we have sales commissions from the current month and all other oper-ating expenses, as follows:

October $2,160 50% $1,080November $2,2

! " !! 446 50% $1,123" !

The question tells us that actual sales for the month ending September 30were $20,000 and that sales commissions amount to 10% of actual sales. Sototal sales commissions for September are $20,000 # 10% = $2,000.Knowing these numbers, we can compute the sales commission paymentsfrom the previous month for October and November as:

October ! $2,000 " 50% ! $1,000November ! $2,160 " 50% ! $1,080

Our final statement now looks like this:

NovemberOctober TotalSales commission from current monthSales commission from previous monthRent expenseUtility expenseInsurance expenseTotal operating expenses

$1,123

1,000800400

$1,080

1,000800400

$ 2,203

2,0001,600

800

JOE UNIVERSITYBudgeted Cash Payments for Operating Expenses

NovemberOctober Total

JOE UNIVERSITYBudgeted Cash Payments for Operating Expenses

Sales commission from current monthSales commission from previous monthRent expenseUtility expenseInsurance expenseTotal operating expenses

$1,1231,0801,000

800400

$4,403

$1,0801,0001,000

800400

$4,280

$ 2,2032,0802,0001,600

800$8,683

Shown another way, we can also compute sales commission payments fromthe current and previous months using each month’s actual sales:

1eSG_C19_0131792075.Qxd 10/24/06 2:08 PM Page 654

Page 687: polpoa_sg

Demo Doc 2 Solution | Chapter 19 655

Rent, utilities, and insurance expense are fixed amounts from the operatingexpense budgetRemember that depreciation expense is a noncash expense and therefore isnot included in the cash budget.

d. The cash budget

Sales Commission from Current MonthCurre

!nnt Month Sales 10% Sales Commission 50% P " " aayment this Month

October $21,600 10% 50%! " "

!!

! " "

!

$1,080

November $22,464 10% 50%$1,123..20 (rounded to $1,123)

Sales Commission froom Previous MonthPrevious Month Sales

! "110% Sales Commission 50% Payment this Mont" hh

October $20,000 10% 50%$1,000

November

! " "

!

!! " "

!

$21,600 10% 50%$1,080

To prepare the cash budget, you start with the beginning cash balance andadd the budgeted cash collections that you computed in part a. Cash pay-ments for purchases and operating expenses are then subtracted to achievethe ending cash balance.Beginning cash balance is from the ending cash balance of the previousmonth. September’s ending balance as given in the question becomesOctober’s beginning cash balance, $4,000.Cash collections are from budgeted cash collections from customers. Addingthe beginning cash balance to the collections gives you the available cash forthe month:

3 Prepare a financialbudget

Part 1 Part 2 Part 3 Part 4 Demo DocComplete

Beginning cash balanceCash collectionsCash availableCash payments Purchases of inventory Operating expensesTotal cash paymentsEnding cash balance

$21,926$ 4,00020,48024,480

November TotalOctober

JOE UNIVERSITYCash Budget

1eSG_C19_0131792075.Qxd 10/24/06 2:08 PM Page 655

Page 688: polpoa_sg

656 Chapter 19 | Demo Doc 2 Solution

Beginning cash balanceCash collectionsCash availableCash payments Purchases of inventory Operating expensesTotal cash paymentsEnding cash balance

$21,926

12,3084,403

16,711

$ 4,00020,48024,480

11,0004,280

15,280

NovemberOctober

JOE UNIVERSITYCash Budget

Part 1 Part 2 Part 3 Part 4 Demo DocComplete

Beginning cash balanceCash collectionsCash availableCash payments Purchases of inventory Operating expensesTotal cash paymentsEnding cash balance

$ 9,20021,92631,126

12,3084,403

16,711$14,415

$ 4,00020,48024,480

11,0004,280

15,280$ 9,200

NovemberOctober

JOE UNIVERSITYCash Budget

Ending cash balance is equal to cash available less total cash payments.October’s ending cash balance then becomes November’s beginning cashbalance, and the calculations continue in this manner until the budget iscomplete:

Cash payments for purchases of inventory are from the budgeted cash pay-ments for purchases. Cash payments for operating expenses are from the bud-geted cash payments for operating expenses, giving us the total cash payments:

1eSG_C19_0131792075.Qxd 10/24/06 2:08 PM Page 656

Page 689: polpoa_sg

Quick Practice Questions | Chapter 19 657

Quick Practice QuestionsTrue/False

_____ 1. Budgets provide a benchmark that helps managers evaluate performance.

_____ 2. Managers should build “slack” into the budget.

_____ 3. A company’s plan for purchases of property, plant, equipment, andother long-term assets is part of the financial budget.

_____ 4. The cash budget is a projection of the cash inflows and cash outflowsfor a future period.

_____ 5. The cash budget is prepared before the budgeted balance sheet is prepared.

_____ 6. Sensitivity analysis is a what-if technique that asks what a result willbe if a predicted amount is not achieved or if an underlying assumptionchanges.

_____ 7. Responsibility accounting is a system for evaluating the performance ofeach responsibility center and its manager.

_____ 8. Management by exception directs executives’ attention to all differ-ences between actual and budgeted variable cost amounts.

_____ 9. Most companies consider company divisions as cost centers.

_____10. The research and development department would be considered aprofit center.

1eSG_C19_0131792075.Qxd 10/24/06 2:08 PM Page 657

Page 690: polpoa_sg

658 Chapter 19 | Quick Practice Questions

Multiple Choice1. Which of the following statements regarding the budgeting

process is (are) true?a. The budget should be designed from the bottom up, with input from

employees at all levelsb. The budget should be approved by the company’s external auditorsc. The budget should be designed by top management and communi-

cated to lower-level personneld. All the preceding statements are true regarding the budgeting

process

2. Which of the following budgets is an operating budget? a. Capital expenditures budgetb. Budgeted balance sheetc. Budgeted income statementd. Cash budget

3. Which of the following alternatives reflects the proper order ofpreparing components of the master budget? 1. Financial budget2. Operating budget3. Capital expenditures budget

a. 2, 3, 1b. 1, 3, 2c. 1, 2, 3d. 3, 1, 2

4. Desired ending inventory is 80% of beginning inventory. If costof goods sold is $300,000, purchases will always have whatrelationship with cost of goods sold? a. Be more than cost of goods soldb. Be 80% of cost of goods soldc. Equal cost of goods soldd. Be less than cost of goods sold

5. During April, Cherry Company had actual sales of $180,000compared to budgeted sales of $195,000. Actual cost of goodssold was $135,000, compared to a budget of $136,500. Monthlyoperating expenses, budgeted at $28,000, totaled $25,000.Interest revenue of $2,500 was earned during April but had notbeen included in the budget. What is the net income variance onthe performance report for April?a. $( 8,000)b. $(13,000)c. $ 8,000d. $ 13,000

6. Heath Company has beginning inventory of 21,000 units andexpected sales of 48,000 units. If the desired ending inventory is15,500 units, how many units should be produced? a. 27,000b. 42,500c. 45,000d. 53,500

1eSG_C19_0131792075.Qxd 10/24/06 2:08 PM Page 658

Page 691: polpoa_sg

Quick Practice Questions | Chapter 19 659

7. York Enterprises recorded sales of $160,000 during March.Management expects sales to increase 5% in April, 3% over Aprilin May, and 5% over May in June. Cost of goods sold is expectedto be 70% of sales. What is the budgeted gross profit for June?a. $ 54,508b. $112,000c. $127,184d. $181,692

8. Which of the following is an example of a financial budget? a. Sales budgetb. Budgeted balance sheetc. Budgeted income statementd. Operating expense budget

9. Jay Corporation desires a December 31 ending inventory of1,500 units. Budgeted sales for December are 2,300 units. TheNovember 30 inventory was 850 units. What are budgetedpurchases? a. 2,350b. 2,950c. 3,150d. 3,800

10. Bolin’s, an elite clothier, expects its November sales to be 30%higher than its October sales of $200,000. Purchases were$100,000 in October and are expected to be $150,000 inNovember. All sales are on credit and are collected as follows:30% in the month of the sale and 70% in the following month.Purchases are paid 25% in the month of purchase and 75% in thefollowing month. The beginning cash balance on November 1 is$9,000. What is the ending cash balance on November 30? a. $ 87,000b. $114,500c. $140,000d. $149,000

1eSG_C19_0131792075.Qxd 10/24/06 2:08 PM Page 659

Page 692: polpoa_sg

660 Chapter 19 | Quick Practice Questions

Quick Exercises

19-1. Solve for the following independent situations:

a. A sporting goods store budgeted September purchases of ski jacketsat $17,900. The store had ski jackets costing $1,200 on hand at thebeginning of September, and to cover part of anticipated Octobersales, they expect to have $4,200 of ski jackets on hand at the end ofSeptember. What was the budgeted cost of goods sold for September?

b. A department store has budgeted cost of goods sold for October of$29,300 for its women’s coats. Management also wants to have$8,000 of coats in inventory at the end of the month to prepare forthe winter season. Beginning inventory in October was $6,000. Whatdollar amount of coats should be purchased to meet these objectives?

19-2. Redfield Company prepared the following forecasts ofmonthly sales:

January February March April

Sales (in units) 4,500 5,200 4,700 2,800

Redfield Company decided that the number of units in its inventoryat the end of each month should equal 75% of next month’s sales. Thebudgeted cost per unit is $20.

a. How many units should be in January’s beginning inventory?

b. What amount should be budgeted for the cost of merchandise pur-chases in January?

c. How many units should be purchased in February?

1eSG_C19_0131792075.Qxd 10/24/06 2:08 PM Page 660

Page 693: polpoa_sg

Quick Practice Questions | Chapter 19 661

19-3. The sales budget of Mulls Company for the fourth quarter of2008 is as follows:

October November December

Sales $91,000 $76,000 $108,000

Sales are 20% cash, 80% credit.

Cost of goods sold is 70% of total sales.

Desired ending inventory for each month is equal to 25% of cost ofgoods sold for the following month.

Collections on credit sales are as follows:

50% in the month of sale

30% in the month following sale

15% in the second month following sale

5% uncollectible

October 1 inventory is $16,000.

Expected sales for January 2009, are $84,000.

Payments for inventory are 70% in the month following purchase and30% two months following purchase.

a. Compute the cash collections for December.

b. Compute the cash disbursements for purchases during December.

19-4. Soccer Forever gathered the following information as of May 31,2007:

May 31 inventory balance $11,100

May payments for inventory 8,300

May payments of accounts payable and accrued liabilities 9,800

May 31 accounts payable balance 5,400

April 30 equipment balance 37,500

April 30 accumulated depreciation, equipment balance 20,900

Cash purchase of equipment in May 2,700

May operating expenses, excluding depreciation(75% paid in May, 25% accrued on May 31) 4,200

May depreciation expense 600

April 30 stockholders’ equity 42,485

April 30 cash balance 27,040

May budgeted sales 17,300

May cash receipts from sales on account 12,110

1eSG_C19_0131792075.Qxd 10/24/06 2:08 PM Page 661

Page 694: polpoa_sg

662 Chapter 19 | Quick Practice Questions

Cost of goods sold is 65% of sales.

May 31 accounts receivable balance is 30% of May sales.

Prepare the budgeted balance sheet on May 31, 2007.

19-5. Answer the following completely.

a. Define the term budget.

b. Identify four benefits of budgeting.

c. Should employees participate in the budgeting process or shouldmanagement prepare the budgets alone? Explain.

1eSG_C19_0131792075.Qxd 10/24/06 2:08 PM Page 662

Page 695: polpoa_sg

Do It Yourself! Question 1 | Chapter 19 663

b. Inventory, purchases, and cost of goods sold budget

Do It Yourself! Question 1

Blake’s Brakes sells highly specialized brakes for travel trailers. Actual sales forthe month ended June 30 were $80,000. Blake believes that sales will increase2% in July and an additional 3% over July in August. Cash sales are expected tobe 10% of sales and credit sales about 90% of sales.

Cost of goods sold is expected to be 75% of total sales. Blake doesn’t want inven-tory to fall below $12,000 plus 5% of cost of goods sold for the next month. Salesof $85,000 are expected for September. Inventory on June 30 is $13,000.

Operating expenses include sales commission, 5% of sales; rent expense of$6,000; depreciation expense of $4,000; utility expense of $1,500; and insuranceexpense of $1,400.

Round all figures to the nearest dollar.

Requirement

Prepare the following budgets for July and August:

a. Sales budget

1eSG_C19_0131792075.Qxd 10/24/06 2:08 PM Page 663

Page 696: polpoa_sg

664 Chapter 19 | Do It Yourself! Question 1

c. Operating expense budget

d. Budgeted income statement.

1eSG_C19_0131792075.Qxd 10/24/06 2:08 PM Page 664

Page 697: polpoa_sg

Do It Yourself! Question 2 | Chapter 19 665

Do It Yourself! Question 2

Blake’s Brakes prepared its sales budget; inventory, purchases, and cost of goodssold budget; operating expense budget; and budgeted income statement for Julyand August. Blake would now like to prepare the cash budget for the months ofJuly and August.

Actual sales for the month ended June 30 were $80,000. Actual sales for themonth ended May 30 were $78,000. Blake believes that sales will increase 2% inJuly and increase another 3% over July sales in August. Cash sales are expectedto be 10% of sales and credit sales about 90% of sales.

Cost of goods sold is expected to be 75% of total sales. Blake doesn’t want inven-tory to fall below $12,000 plus 5% of cost of goods sold for the next month. Salesof $85,000 are expected for September. Blake purchased $56,000 of inventoryduring June and ended the month with $13,000 in ending inventory.

Operating expenses include sales commission, 5% of sales; rent expense of$6,000; depreciation expense of $4,000; utility expense of $1,500; and insuranceexpense of $1,400.

June 30 cash balance is $14,000.

Of the credit sales, Blake expects to collect 20% in the month of the sale, 50% inthe month following the sale, and the remaining 30% in the next month.Purchases are paid for in the month after the purchase. Sales commissions arepaid 50% in the month incurred and 50% in the next month. Rent, utility, andinsurance are paid in the month incurred.

Requirement

For the months of July and August, prepare the following:

a. Budgeted cash collections from customers

1eSG_C19_0131792075.Qxd 10/24/06 2:08 PM Page 665

Page 698: polpoa_sg

c. Budgeted cash payments for operating expenses

666 Chapter 19 | Do It Yourself! Question 2

b. Budgeted cash payments for purchases

d. The cash budget

1eSG_C19_0131792075.Qxd 10/24/06 2:08 PM Page 666

Page 699: polpoa_sg

Quick Practice Solutions | Chapter 19 667

Quick Practice SolutionsTrue/False

T 1. Budgets provide a benchmark that helps managers evaluate performance. (p. 1009)

F 2. Managers should build “slack” into the budget.False–Managers should not build “slack” into the budget, it makes the budget less accurate. (pp. 1022–1023)

F 3. A company’s plan for purchases of property, plant, equipment, and other long-term assets is part of the financial budget.False–A company’s plan for purchases of property, plant,equipment, and other long-term assets is part of the capital expenditures budget. (p. 1011)

T 4. The cash budget is a projection of the cash inflows and cash outflows for a future period. (p. 1011)

T 5. The cash budget is prepared before the budgeted balance sheet is prepared. (p. 1011)

T 6. Sensitivity analysis is a what-if technique that asks what a result will be if a predicted amount is not achieved or if an underlying assumption changes. (p. 1024)

T 7. Responsibility accounting is a system for evaluating the performance of each responsibility center and its manager. (p. 1025)

F 8. Management by exception directs executives’ attention to all differences between actual and budgeted variable cost amounts.False–Management by exception directs executives’ attention to all significant differences between actual and budgeted amounts.(p. 1029)

F 9. Most companies consider company divisions as cost centers.False–Most companies consider company divisions as investmentcenters. (p. 1027)

F 10. The research and development department would be considered a profit center.False–The research and development department would be considered a cost center because the managers only control costs.(p. 1025)

1eSG_C19_0131792075.Qxd 10/24/06 2:08 PM Page 667

Page 700: polpoa_sg

668 Chapter 19 | Quick Practice Solutions

Multiple Choice

1. Which of the following statements regarding the budgetingprocess is (are) true? (p. 1010)a. The budget should be designed from the bottom up, with input from

employees at all levelsb. The budget should be approved by the company’s external auditorsc. The budget should be designed by top management and communi-

cated to lower-level personneld. All the preceding statements are true regarding the budgeting

process

2. Which of the following budgets is an operating budget? (p. 1011)a. Capital expenditures budgetb. Budgeted balance sheetc. Budgeted income statementd. Cash budget

3. Which of the following alternatives reflects the proper order ofpreparing components of the master budget? (p. 1011)1. Financial budget2. Operating budget3. Capital expenditures budget

a. 2, 3, 1b. 1, 3, 2c. 1, 2, 3d. 3, 1, 2

4. Desired ending inventory is 80% of beginning inventory. If costof goods sold is $300,000, purchases will always have whatrelationship with cost of goods sold? (p. 1015)a. Be more than cost of goods soldb. Be 80% of cost of goods soldc. Equal cost of goods soldd. Be less than cost of goods sold

5. During April, Cherry Company had actual sales of $180,000compared to budgeted sales of $195,000. Actual cost of goodssold was $135,000, compared to a budget of $136,500. Monthlyoperating expenses, budgeted at $28,000, totaled $25,000.Interest revenue of $2,500 was earned during April but had notbeen included in the budget. What is the net income variance onthe performance report for April? (p. 1027)a. $ (8,000) b. $(13,000)c. $ 8,000d. $ 13,000

6. Heath Company has beginning inventory of 21,000 units andexpected sales of 48,000 units. If the desired ending inventory is15,500 units, how many units should be produced? (p. 1014)a. 27,000b. 42,500c. 45,000d. 53,500

1eSG_C19_0131792075.Qxd 10/24/06 2:08 PM Page 668

Page 701: polpoa_sg

Quick Practice Solutions | Chapter 19 669

7. York Enterprises recorded sales of $160,000 during March.Management expects sales to increase 5% in April, 3% over Aprilin May, and 5% over May in June. Cost of goods sold is expectedto be 70% of sales. What is the budgeted gross profit for June? (pp. 1012–1016)a. $ 54,508b. $112,000c. $127,184d. $181,692

8. Which of the following is an example of a financial budget? (p. 1011)a. Sales budgetb. Budgeted balance sheet c. Budgeted income statementd. Operating expense budget

9. Jay Corporation desires a December 31 ending inventory of1,500 units. Budgeted sales for December are 2,300 units. TheNovember 30 inventory was 850 units. What are budgetedpurchases? (pp. 1012–1016)a. 2,350b. 2,950c. 3,150d. 3,800

10. Bolin’s, an elite clothier, expects its November sales to be 30%higher than its October sales of $200,000. Purchases were$100,000 in October and are expected to be $150,000 inNovember. All sales are on credit and are collected as follows:30% in the month of the sale and 70% in the following month.Purchases are paid 25% in the month of purchase and 75% in thefollowing month. The beginning cash balance on November 1 is$9,000. What is the ending cash balance on November 30? (p. 1018)a. $ 87,000b. $114,500c. $140,000d. $149,000

1eSG_C19_0131792075.Qxd 10/24/06 2:08 PM Page 669

Page 702: polpoa_sg

670 Chapter 19 | Quick Practice Solutions

Quick Exercise Solutions

19-1. Solve for the following independent situations: (p. 1015)

a. A sporting goods store budgeted September purchases of ski jack-ets at $17,900. The store had ski jackets costing $1,200 on hand atthe beginning of September, and to cover part of anticipatedOctober sales they expect to have $4,200 of ski jackets on hand atthe end of September. What was the budgeted cost of goods sold forSeptember?

Let X ! Cost of Goods Sold $1,200 # $17,900 – X ! $4,200

$19,100 – X ! $4,200 X ! $14,900

b. A department store has budgeted cost of goods sold for October of$29,300 for its women’s coats. Management also wants to have$8,000 of coats in inventory at the end of the month to prepare forthe winter season. Beginning inventory in October was $6,000.What dollar amount of coats should be purchased to meet theseobjectives?

Let X ! Purchases $6,000 # X – $29,300 ! $8,000

X – $23,300 ! $8,000 X ! $31,300

19-2. Redfield Company prepared the following forecasts ofmonthly sales:

January February March April

Sales (in units) 4,500 5,200 4,700 2,800

Redfield Company decided that the number of units in its inventoryat the end of each month should equal 75% of next month’s sales. Thebudgeted cost per unit is $20. (p. 1015)

January February

Next month’s budgeted sales (units) 5,200 4,700

Ratio of inventory to future sales # 75% # 75%Desired ending inventory 3,900 3,525

Budgeted sales for the month (units) 4,500 5,200Required units of available merchandise 8,400 8,725

Deduct beginning inventory 3,375 3,900Number of units to be purchased 5,025 4,825

a. How many units should be in January’s beginning inventory?75% " 4,500 units ! 3,375 units

1eSG_C19_0131792075.Qxd 10/24/06 2:08 PM Page 670

Page 703: polpoa_sg

Quick Practice Solutions | Chapter 19 671

b. What amount should be budgeted for the cost of merchandise pur-chases in January?

5,025 units " $20 ! $100,500

c. How many units should be purchased in February?

4,825

19-3. The sales budget of Mulls Company for the fourth quarter of2008 is as follows: (pp. 1017–1018)

October November December

Sales $91,000 $76,000 $108,000

Purchases 61,000 58,800 65,000

Sales are 20% cash, 80% credit.

Cost of goods sold is 70% of total sales.

Desired ending inventory for each month is equal to 25% of cost ofgoods sold for the following month.

Collections on credit sales are as follows:

50% in the month of sale30% in the month following sale15% in the second month following sale5% uncollectible

October 1 inventory is $16,000.

Expected sales for January 2009, are $84,000.

Payments for inventory are 70% in the month following purchase and30% two months following purchase.

a. Compute the cash collections for December.

b. Compute the cash disbursements for purchases during December.

October Purchases ($62,00 " 0.30) $18,300November Purchases ($58,800 " 0.70) 41,160December Cash Disbursements for Purchases $59,460

October Credit Sales $10,920 ! ($ 91,000 " 0.80 " 0.15)November Credit Sales 18,240 ! ($ 76,000 " 0.80 " 0.30)December Cash Sales 21,600 ! ($108,000 " 0.20)December Credit Sales 43,200 ! ($108,000 " 080 " 0.50)December Cash Collection $93,960 !

1eSG_C19_0131792075.Qxd 10/24/06 2:08 PM Page 671

Page 704: polpoa_sg

672 Chapter 19 | Quick Practice Solutions

19-4. Soccer Forever gathered the following information as of May 31,2007:

May 31 inventory balance $11,100

May payments for inventory 8,300

May payments of accounts payable and accrued liabilities 9,800

May 31 accounts payable balance 5,400

April 30 equipment balance 37,500

April 30 accumulated depreciation, equipment balance 20,900

Cash purchase of equipment in May 2,700

May operating expenses, excluding depreciation(75% paid in May, 25% accrued on May 31) 4,200

May depreciation expense 600

April 30 stockholders’ equity 42,485

April 30 cash balance 27,040

May budgeted sales 17,300

May cash receipts from sales on account 12,110

Cost of goods sold is 65% of sales.

May 31 accounts receivable balance is 30% of May sales.

Prepare the budgeted balance sheet on May 31, 2007. (pp. 1021–1022)

AssetsCurrent assets:CashAccounts receivable ($17,300 3 0.30)Inventory Total current assetsPlant assets:EquipmentLess: Accumulated depreciationTotal assets

LiabilitiesCurrent liabilitiesAccounts payableAccrued liabilities ($4,200 3 0.25) Total liabilitiesStockholders’ equityTotal liabilities and stockholders’ equity

31,490

18,700$50,190

$ 6,45043,740**

$50,190

$15,200*5,190

11,100

40,200(21,500)

$55,4001,050

*[$27,040 " $8,300 " $9,800 " $2,700 " (0.75 3 $4,200) ! $12,110] $ $15,200**Net income for May $ $17,300 " (0.65 3 $17,300) " $4,200 " $600 $ $1,255 $42,485 ! $1,255 $ $43,740

SOCCER FOREVERBudgeted Balance Sheet

May 31, 2007

1eSG_C19_0131792075.Qxd 10/24/06 2:08 PM Page 672

Page 705: polpoa_sg

Quick Practice Solutions | Chapter 19 673

19-5. Answer the following completely. (pp. 1008–1011)

a. Define the term budget.A budget is a quantitative expression of a plan of action that helpsmanagers coordinate and implement the plan.

b. Identify four benefits of budgeting.The four benefits of budgeting include:

1. Budgeting compels planning. Budgeting helps managers setrealistic goals by requiring them to plan specific actions tomeet their goals. Budgeting also helps managers prepare for arange of conditions and plan for contingencies.

2. Budgeting promotes coordination and communication. Themaster budget coordinates the activities of the organization.It forces managers to consider relationships among operationsacross the entire value chain.

3. Budgeting aids performance evaluation. To evaluate a depart-ment or activity, its actual results may be compared either toits budget or its past performance. In general, the budget is abetter benchmark because it considers current changes stem-ming from past conditions.

4. Budgeting can affect behavior and motivate employees. Thebudgeting process prompts managers to look further into thefuture than they would look otherwise and foresee and avoidproblems.

c. Should employees participate in the budgeting process or shouldmanagement prepare the budgets alone? Explain.Employees should participate in the budgeting process so that theycan bear ownership of the plan. If employees are excluded from thisprocess and unrealistic goals are set, employee morale can bedeflated, especially if the employees’ performance is judged againstthese standards. If employees help to create the budget, it can be amotivating factor to achieve the desired outcome.

1eSG_C19_0131792075.Qxd 10/24/06 2:08 PM Page 673

Page 706: polpoa_sg

674 Chapter 19 | Do It Yourself! Question 1 Solutions

b. Inventory, purchases, and cost of goods sold budget

c. Operating expense budget

AugustJuly TotalCash sales, 10%Credit sales, 90%Total

$ 8,40575,643

$84,048

$ 8,16073,440

$81,600

$ 16,565149,083

$165,648

BLAKE’S BRAKESSales Budget

July AugustCost of goods sold! Desired ending inventory Total inventory required" Beginning inventory Purchases

$63,03615,18778,22315,152

$63,071

$61,20015,15276,35213,000

$63,352

BLAKE’S BRAKESInventory, Purchases, and Cost of Goods Sold Budget

Sales commissionDepreciation expenseRent expenseUtility expenseInsurance expenseTotal operating expenses

$ 4,2026,0004,0001,5001,400

$17,102

$ 4,0806,0004,0001,5001,400

$16,980

$ 8,28212,0008,0003,0002,800

$34,082

AugustJuly Total

BLAKE’S BRAKESOperating Expense Budget

2 Prepare an operatingbudget

2 Prepare an operatingbudget

2 Prepare an operatingbudget

Do It Yourself! Question 1 Solutions

Requirement

Prepare the following budgets for July and August:

a. Sales budget

1eSG_C19_0131792075.Qxd 10/24/06 2:08 PM Page 674

Page 707: polpoa_sg

Do It Yourself! Question 1 Solutions | Chapter 19 675

d. Budgeted income statement

AugustJuly TotalSales revenueCost of goods soldGross profitOperating expensesNet income

$84,04863,03621,01217,102

$ 3,910

$81,60061,20020,20016,980

$ 3,420

$165,648124,23641,41234,082

$ 7,330

BLAKE’S BRAKESBudgeted Income Statement

2 Prepare an operatingbudget

1eSG_C19_0131792075.Qxd 10/24/06 2:08 PM Page 675

Page 708: polpoa_sg

676 Chapter 19 | Do It Yourself! Question 2 Solutions

July AugustCash salesCollections from current month credit salesCollections from previous month credit salesCollections from two-months previous credit salesTotal collections

$ 8,40515,12936,72021,600

$81,854

$ 8,16014,68836,00021,060

$79,908

BLAKE’S BRAKESBudgeted Cash Collections from Customers

b. Budgeted cash payments for purchases

AugustJuly TotalPayment of last month purchases $63,352$56,000 $119,352

BLAKE’S BRAKESBudgeted Cash Collections from Customers

c. Budgeted cash payments for operating expenses

Sales commission from this monthSales commission from last monthRent expenseUtility expenseInsurance expenseTotal

$ 2,1012,0406,0001,5001,400

$13,041

$ 2,0402,0006,0001,5001,400

$12,940

$ 4,1414,040

12,0003,0002,800

$25,981

AugustJuly Total

BLAKE’S BRAKESBudgeted Cash Payments for Operating Expenses

3 Prepare a financialbudget

3 Prepare a financialbudget

3 Prepare a financialbudget

Do it Yourself! Question 2 Solutions

Requirement

For the months of July and August, prepare the following:

a. Budgeted cash collections from customers

1eSG_C19_0131792075.Qxd 10/24/06 2:08 PM Page 676

Page 709: polpoa_sg

Do It Yourself! Question 1 Solutions | Chapter 19 677

d. The cash budget

July AugustBeginning cash balanceCash collectionsCash availableCash payments Purchases of inventory Operating expensesTotal cash paymentsEnding cash balance

$ 24,96881,854

106,822

63,35213,04176,393

$ 30,429

$14,00079,90893,908

56,00012,94068,940

$24,968

BLAKE’S BRAKESCash Budget

3 Prepare a financialbudget

1eSG_C19_0131792075.Qxd 10/24/06 2:08 PM Page 677

Page 710: polpoa_sg

The Power of PracticeFor more practice using the skills learned in this chapter, visit MyAccountingLab.There you will find algorithmically generated questions that are based on theseDemo Docs and your main textbook’s Review and Accounting Practice sections.

To go to MyAccountingLab and follow these steps:

1. Direct your URL to www.myaccountinglab.com.2. Log in using your name and password.3. Click the MyAccountingLab link.4. Click Study Plan in the left navigation bar.5. From the table of contents, select Chapter 19, The Master Budget and

Responsibility Accounting.6. Click a link to work on the tutorial exercises.

678 Chapter 19 | The Power of Practice

1eSG_C19_0131792075.Qxd 10/24/06 2:08 PM Page 678

Page 711: polpoa_sg

Flexible Budgets andStandard Costs20

WHAT YOU PROBABLY ALREADY KNOW

Various budgets including a cash budget were covered inChapter 19. Assume you had budgeted $400 as your monthlyautomobile expenditures and found that you actually spent $425.At first glance, it appears to be an unfavorable difference. Thebudget of $400 was created assuming that your travel would be1,000 miles and your costs consist of auto insurance of $100,gasoline of $125 (1,000 miles/20 miles per gallon ! $2.50 pergallon), and a $175 repayment of the car loan. If you find that youused the car 1,200 miles this month, you probably already knowthat you should not be held to the original, static budget. Thebudget should be “flexible” to allow for differences in the level ofactivity that impacts costs. The flexible budget for 1,200 milesshould be $100 + $175 + (1,200 miles/20 miles per gallon ! $2.50per gallon) = $425. When the actual results are compared to theflexible budget, the result is no variance. In this chapter, we willstudy the creation and evaluation of flexible budgets.

Learning Objectives

Prepare a flexible budget for the income statement.

Flexible budgets are budgets that summarize cost and revenueinformation for several different volume levels within a relevant range.To prepare a flexible budget, the variable cost per unit and total fixedcosts must be determined. You will prepare a flexible budget in DemoDoc 1. Review the flexible budget in Exhibit 20-2 (p. 1069).

Prepare an income statement performance report.

The income statement performance report shows the differencebetween the actual results and expected results. The differencebetween the actual results at actual volume and the budgeted results atexpected sales volume is the static budget variance. A component ofthe variance may be due to the actual sales volume differing from thebudgeted sales volume, called the sales volume variance. The othercomponent of the variance may be due to the actual amounts at actualvolume differing from the budgeted amounts at actual volume; thisdifference is called the flexible budget variance. The static budgetvariance is the total of the sales volume variance and the flexiblebudget variance. Review Exhibit 20-3 (p. 1070) for an illustration of therelationship between the static budget variance, the flexible budgetvariance, and the sales volume variance. Review the income statementperformance report in Exhibit 20-4 (p. 1071).

1

2

1eSG_C20_0131792075.Qxd 10/24/06 2:10 PM Page 679

Page 712: polpoa_sg

680 Chapter 20 | Flexible Budgets and Standard Costs

Identify the benefits of standard costs and learn how to setstandards.

Many managers use standard costing because it helps them better planand control levels of performance and provide a goal for employees towork toward. In addition, record-keeping costs are reduced because it’s easier to value all inventory at standard, and it provides cost informationto determine the sales prices. See Exhibit 20-5 (p. 1073) for standard settingissues relating to standard costing.

Compute standard cost variances for direct materials and directlabor.

The actual production costs are compared to the standard costsperiodically. A variance may result due to the price or the quantity used.A price variance measures the difference between the actual price andthe standard price multiplied by the actual quantity.

A quantity or efficiency variance measures how well the business usesits materials or human resources. The quantity variance measures thedifference in the actual and standard quantities multiplied by thestandard price per unit. Review Exhibit 20-7 (p. 1076) for an illustrationof the relationship between the variances, and Exhibit 20-9 (p. 1078) fora computation of price and quantity variances.

Analyze manufacturing overhead in a standard cost system.

The total manufacturing overhead variance is the difference betweenthe actual overhead incurred and the standard overhead. The standardoverhead amount is the product of the standard overhead rate and thestandard allocation base at the actual volume. Recall that the standardoverhead rate is computed before the period begins as:

The total overhead variance can be separated between the overheadflexible budget variance and the production volume variance. Theoverhead flexible budget variance shows how well management hascontrolled overhead costs. It is computed in exactly the same way as theflexible budget variances for direct materials and direct labor. Theoverhead flexible budget variance can be measured as the differencebetween the actual overhead cost and the flexible budget overhead forthe actual number of outputs. The production volume variance ariseswhen actual production differs from expected production. It is thedifference between the flexible budget for the actual outputs and thestandard overhead allocated to actual production.

Standard OverheadRateExpectedManufacturi

!nngOverhead Costs

Expected Quantity of AlloccationBase

5

4

3

1eSG_C20_0131792075.Qxd 10/24/06 2:10 PM Page 680

Page 713: polpoa_sg

Record transactions at standard cost and prepare a standard costincome statement.

The standard cost system requires that the cost of inventory berecorded at the standard amount. This procedure permits variances tobe recorded as soon as possible. If the variance is unfavorable, actualcosts exceed the standard costs. The unfavorable variance is recordedas a debit to the Variance account and is treated as an expense on theincome statement. If the variance is favorable, actual costs are less thanthe standard costs. The favorable variance is recorded as a credit to theVariance account and is treated as a contra-expense on the incomestatement. The recorded variances are shown on the standard costincome statement as an adjustment to the cost of goods sold. Reviewthe Standard Cost Accounting System section in the main text forsample journal entries and Exhibit 20-12 (p. 1086) for the flow of costs ina standard costing system. Review Exhibit 20-13 (p. 1086) for a samplestandard cost income statement.

6

Chapter 20 | Flexible Budgets and Standard Costs 681

1eSG_C20_0131792075.Qxd 10/24/06 2:10 PM Page 681

Page 714: polpoa_sg

682 Chapter 2 | Demo Doc 1 Solutions

Budgeted Actual

Units sold 5,400 5,200

Variable costs $620 $630

Fixed costs $2,100,000 $2,050,000

Flexible Budgets and Income Statement PerformanceReport Management at Virus Detection Sensor predicted 2010 sales of 5,400 sensors at aprice of $1,000 per sensor. Actual sales for the year were 5,200 sensors at $1,050per sensor. Following is a breakdown of its costs for 2010:

1 Prepare a flexible budget for the incomestatement

2 Prepare an incomestatement perfor-mance report

Demo Doc 1

Requirements

1. Why would Virus Detection prepare a flexible budget? Using VirusDetection’s estimated budget values for costs and sales price, developflexible budgets for 5,200, 5,400, 5,600, and 6,000 sensors. Would VirusDetection Sensor’s managers use this flexible budget for planning orcontrolling? What specific insights can Virus Detection Sensor’s managersgain from this flexible budget?

2. Using the format shown in Exhibit 20-4 (p. 1071) of the textbook, prepareVirus Detection’s income statement performance report for 2010.

3. What was the effect on Virus Detection’s operating income of selling 200units less than the static budget level of sales?

682 Chapter 20 | Demo Doc 1

1eSG_C20_0131792075.Qxd 10/24/06 2:10 PM Page 682

Page 715: polpoa_sg

Demo Doc 1 | Chapter 1 683

Sales revenueVariable expensesFixed expensesTotal expensesOperating income (loss)

$1,000620

$5,200,0003,224,0002,100,0005,324,000

$ (124,000)

$5,400,0003,348,0002,100,0005,448,000

$ (48,000)

$5,600,0003,472,0002,100,0005,572,000

$ 28,000

$6,000,0003,720,0002,100,0005,820,000

$ 180,000

5,200 5,400 5,600 6,000

Flexible Budgetper Output Unit Output Units (sensors)

VIRUS DETECTION SENSORFlexible BudgetYear Ending 2010

Part 1 Part 2 Part 3 Demo DocComplete

Requirement 1

Why would Virus Detection prepare a flexible budget? Using VirusProtection’s estimated budget values for costs and sales price, developflexible budgets for 5,200, 5,400, 5,600, and 6,000 sensors. Would VirusDetection Sensor’s managers use this flexible budget for planning orcontrolling? What specific insights can Virus Detection Sensor’s managersgain from this flexible budget?

Demo Doc 1 Solutions

It is difficult for management to analyze results when the actual volume differsfrom the static budget. Therefore, companies will produce flexible budgets, whichare budgets that summarize cost and revenue information for several differentvolume levels within a relevant range. Virus Detection would prepare a flexiblebudget if managers aren’t sure about their projected sales volume of 5,400 sen-sors. Virus Detection’s flexible budget would show how its revenues and expensesshould vary at different volume levels.

Depending upon the volume within the relevant range, a budget can bedetermined for each amount of activity using the following formula:

For Virus Detection, the following would be its flexible budget for 5,200,5,400, 5,600, and 6,000 sensors:

Flexible Budget Total Cost Number ofOut

! pput Units

Variable Costper Output Un

" iit

TotalFixed Cost!

"#$

%& #

Notice how the total fixed expenses stay constant at $2,100,000 across allfour scenarios, yet revenue and variable expenses change. Remember that vari-able expenses are constant per unit; they are called variable because total vari-able costs change with level of activity.

As you can see, with an output of 5,200 units, Virus Detection would gener-ate 5,200 ! $1,000 per unit = $5,200,000 in sales revenue. Meanwhile, itsexpenses would be 5,200 ! $620 = $3,224,000 variable expenses, plus $2,100,000fixed expenses = $5,324,000 in total expenses.

1 Prepare a flexible budget for the incomestatement

Demo Doc 1 Solutions | Chapter 20 683

1eSG_C20_0131792075.Qxd 10/24/06 2:10 PM Page 683

Page 716: polpoa_sg

So with an output of 5,200 units, Virus Detection would incur a loss of$5,200,000 – $324,000 = $124,000.

Because the output levels in the flexible budget are estimated levels of sales,these flexible budgets are being developed before the period to help managersplan. In this case, the budgets indicate a danger for Virus Detection Sensor ofoperating at a loss for 2010 at sales levels below 5,600 units. Managers shoulddevote considerable effort to ensure that Virus Detection Sensor sells more than5,600 sensors, or find ways to either decrease the expenses associated with man-ufacturing each sensor or increase the sales price per unit.

Requirement 2

Using the format shown in Exhibit 20-4 (p. 1071) of the textbook, prepareVirus Detection’s income statement performance report for 2010.

684 Chapter 20 | Demo Doc 1 Solution

2 Prepare an incomestatement perfor-mance report

Part 1 Part 2 Part 3 Demo DocComplete

Output UnitsSales revenueVariable expensesFixed expensesTotal expensesOperating income (loss)

$5,460,0003,276,0002,050,0005,326,000

$ 134,000

$5,400,0003,348,0002,100,0005,488,000

$ (48,000)

5,200

1

Actual Resultsat Actual Prices

2

Flexible BudgetVariance

3Flexible Budget

for Actual Numberof Output Units

4

Sales VolumeVariance

5,400

5

Static (Master)Budget

VIRUS DETECTION SENSORIncome Statement Performance Report

Year Ended 2010

Columns 1 and 5 of the income statement performance report represent VirusDetection’s actual 2010 results and its expected (budgeted) results, respectively.These figures come directly from the comparison of actual results with the staticbudget statement for 5,400 units.

Actual results at actual prices are what actually happened. This column (1)represents Virus Detection’s true income statement for 2010, calculated by multi-plying actual quantity by actual price.

Static budget (column 5) represents Virus Detection’s original budget withbudgeted volume and costs. This budget uses output of 5,400 sensors, when inreality Virus Detection only had a volume of 5,200 sensors in 2010.

We can begin to build our performance report using the actual results andthe static budget.

The next column that we can complete is the flexible budget for actual num-ber of output units (column 3), which represents budgeted amounts based onwhat Virus Detection actually sold (the other two variance columns depend onthe values calculated in this column). So for this column, we forget about thenumber of units that Virus Detection expected to sell in 2010, but we instead cal-culate what its operating income would have been had it sold the 5,200 sensors

1eSG_C20_0131792075.Qxd 11/22/06 11:41 AM Page 684

Page 717: polpoa_sg

Demo Doc 1 Solutions | Chapter 20 685

Output UnitsSales revenueVariable expensesFixed expensesTotal expensesOperating income (loss)

$5,460,0003,276,0002,050,0005,326,000

$ 134,000

$5,400,0003,348,0002,100,0005,488,000

$ (48,000)

$5,200,0003,224,0002,100,0005,324,000

$ (124,000)

5,200 5,200

1

Actual Resultsat Actual Prices

2

Flexible BudgetVariance

3Flexible Budget

for Actual Numberof Output Units

4

Sales VolumeVariance

5,400

5

Static (Master)Budget

VIRUS DETECTION SENSORIncome Statement Performance Report

Year Ended 2010

using its actual budget figures for revenues and expenses. These figures givemanagement the basis with which to compare the flexible budget variance withthe sales volume variance.

Remember that a flexible budget based upon actual outputs provides thegreatest ability to evaluate and control because you are comparing what youexpected it to be against what it actually is.

From the question, we know that Virus Detection budgeted to sell each sensorat $1,000. Regardless of how many sensors it expected to sell, if it had priced the5,200 sensors it did sell at its original estimate of $1,000 per sensor, VirusDetection’s sales revenue would have been 5,200 ! $1,000 = $5,200,000. This num-ber is called the flexible budget for sales revenue based on actual units produced.

Virus Detection budgeted its variable costs at $620 per sensor. If its bud-geted variable costs had stayed at $620, then its flexible budget for variablecosts would be 5,200 sensors ! $620 = $3,224,000.

Similarly, had Virus Detection’s budgeted fixed costs remained at$2,100,000, then its flexible budget for total costs would be $3,224,000 vari-able costs + $2,100,000 fixed costs = $5,324,000.

Given these data, then, Virus Detection’s flexible budgeted operating incomewould be calculated as:

$5,200,000 sales revenue – $5,324,000 total costs ! $(124,000) operating loss

So, how do we use these data? We use them to complete the two variancecolumns of the income statement performance report. The differences betweencolumns 1 and 3 are presented in column 2, which shows us the flexible budgetvariance.

Flexible budget variance is the difference between the actual results (atactual prices) and the flexible budget (for actual number of output units). Thisvariance represents more or less revenue earned than expected, and more or lessexpenses incurred than expected, for the actual level of output. For sales revenue,the variance is favorable if the actual is greater than the flexible budget. In thiscase, actual sales revenue of $5,460,000 exceeds the flexible budget of $5,200,000by $260,000, which is a favorable outcome.

1eSG_C20_0131792075.Qxd 11/22/06 11:41 AM Page 685

Page 718: polpoa_sg

686 Chapter 20 | Demo Doc 1 Solutions

Output UnitsSales revenueVariable expensesFixed expensesTotal expensesOperating income (loss)

$5,460,0003,276,0002,050,0005,326,000

$ 134,000

$5,400,0003,348,0002,100,0005,488,000

$ (48,000)

$5,200,0003,224,0002,100,0005,324,000

$ (124,000)

5,200$260,000 F

52,000 U50,000 F

2,000 U$258,000 F

0 5,200

1

Actual Resultsat Actual Prices

2

Flexible BudgetVariance

3Flexible Budget

for Actual Numberof Output Units

4

Sales VolumeVariance

5,400

5

Static (Master)Budget

VIRUS DETECTION SENSORIncome Statement Performance Report

Year Ended 2010

So what does this result mean? The flexible budget variance of $260,000for sales revenue indicates that Virus Detection actually received $260,000more for the 5,200 sensors than it would have had if it sold the 5,200 units atits budgeted sales price of $1,000. On average, it received $50 more per sensorthan it thought it would. At the same time, it spent $52,000 more in variablecosts than it expected to, and while fixed costs were trimmed by $50,000, itstill spent $2,000 more than it budgeted on total costs. Even though the over-all results are favorable, management will still want to know why costsexceeded expectations.

The last column to compute is the sales volume variance column (4). The dif-ferences between columns 3 and 5 are presented in column 4.

Sales volume variance is the difference between the flexible budget (foractual number of output units) and the static budget. The only reason for thisvariance is because the number of units actually sold differs from the static bud-get, but knowing whether these variances are favorable or unfavorable givesmanagers insight into how sales and marketing are performing in relation to theoverall report.

For sales revenue, this variance is favorable if the flexible budget isgreater than the static budget, which is not the case with Virus Detection,however. The flexible budget sales revenue of $5,200,000 is lower than the bud-geted sales of $5,400,000 because the sales force sold fewer units thanexpected.

For costs, this variance is favorable if the flexible budget is less than the sta-tic budget. In this case, the results are favorable, with the flexible budget show-ing $5,324,000 in total costs and the static budget showing $5,448,000, for afavorable variance of $124,000 in total costs. However, this variance results fromthe fact that the sales force sold fewer units than expected. In actuality, variablecosts increased by $10 per unit over what Virus Detection had budgeted. Thisvariance is favorable only because Virus Detection budgeted costs for more unitsthan it actually made.

For costs, the variance is unfavorable if the actual is greater than the flexiblebudget. In this case, actual total costs of $5,326,000 exceed the flexible budget of$5,324,000 by $2,000, which is unfavorable. However, total operating incomedemonstrates a favorable result of $258,000.

1eSG_C20_0131792075.Qxd 11/22/06 11:41 AM Page 686

Page 719: polpoa_sg

Demo Doc 1 Solutions | Chapter 20 687

Output UnitsSales revenueVariable expensesFixed expensesTotal expensesOperating income (loss)

$5,460,0003,276,0002,050,0005,326,000

$ 134,000

$5,400,0003,348,0002,100,0005,488,000

$ (48,000)

$5,200,0003,224,0002,100,0005,324,000

$ (124,000)

5,200$260,000 F

52,000 U50,000 F

2,000 U$258,000 F

0$200,000 U124,000 F

0 124,000 F

$ 76,000 U

200U5,200

1

Actual Resultsat Actual Prices

2

Flexible BudgetVariance

3Flexible Budget

for Actual Numberof Output Units

4

Sales VolumeVariance

5,400

5

Static (Master)Budget

VIRUS DETECTION SENSORIncome Statement Performance Report

Year Ended 2010

2 Prepare an incomestatement perfor-mance report

Part 1 Part 2 Part 3 Demo DocComplete

Part 1 Part 2 Part 3 Demo DocComplete

Requirement 3

What was the effect on Virus Detection’s operating income of selling 200units less than the static budget level of sales?

Virus Detection’s sales volume variance is $200,000 unfavorable. So, Virus selling200 fewer units than budgeted reduced its sales volume by $200,000. Virus wasable to more than overcome this reduction in sales volume by selling at a higherprice than anticipated, as indicated by the favorable flexible budget sales rev-enue variance of $260,000. Had Virus Detection sold what it budgeted for, itwould have incurred a loss of $48,000. But, by increasing the sales price andreducing the fixed expense costs, even by selling 200 fewer units, Virus incurredan operating income of $134,000 instead of incurring the loss.

1eSG_C20_0131792075.Qxd 11/22/06 11:41 AM Page 687

Page 720: polpoa_sg

Demo Doc 2

688 Chapter 20 | Demo Doc 2

Standard CostsLearning Objectives 3–6

Bumpy Road Maps manufactures road maps. Bumpy uses a standard cost systemto control manufacturing costs.The following standard unit cost information is based on the static budget vol-ume of 120,000 road maps per month:

In this example, total budgeted fixed manufacturing overhead:

$0.80 ! 120,000 road maps ! $96,000

Actual cost and production volume information:

• Actual production was 118,000 maps.

• Actual direct materials usage was 41 square yards per map, at anactual cost of $0.016 per square yard.

• Actual direct labor usage of 11,600 hours at $12.10 per hour, total costof $140,360.

• Total actual overhead was $126,000.

Requirements

1. Bumpy developed its standards. What are the five benefits of standardcosting?

2. Compute the price and efficiency variances for direct materials and directlabor. Are these variances favorable or unfavorable? Why?

3. Journalize the usage of direct materials, including the related variance.

4. For manufacturing overhead, compute the total variance, the flexiblevariance, and the production volume variance. Are these variancesfavorable or unfavorable? Why?

Direct materials (40 sq. yards @ $0.015 per sq. yard)Direct labor (0.10 hours @ $12.00 per hour)Manufacturing overhead: Variable (0.10 hours @ $3.00 per hour) Fixed (0.10 hours @ $8.00 per hour)Total cost per map

$0.601.20

1.10$2.90

0.300.80

1eSG_C20_0131792075.Qxd 11/22/06 11:41 AM Page 688

Page 721: polpoa_sg

Demo Doc 2 Solutions | Chapter 20 689

Requirement 1

Bumpy developed its standards. What are the five benefits of standardcosting?

Demo Doc 2 Solutions

3 Identify the benefits of standard costs andlearn how to set standards

4 Compute standardcost variances fordirect materials anddirect labor

Part 1 Part 2 Part 3 Part 4 Part 5 Part 6 Part 7Demo DocComplete

Part 1 Part 2 Part 3 Part 4 Part 5 Part 6 Part 7Demo DocComplete

A standard cost is a budget for a single unit. A standard is developed for thequantity and price of direct materials, direct labor, and manufacturing overhead.

Material price standards consider the cost of purchases plus freight in andreceiving costs less discounts. Direct labor cost standards include the labor rates,payroll taxes, and fringe benefits. Overhead cost standards are developed bydividing estimated variable and fixed overhead costs by an appropriate allocationbase. The quantity standards are usually developed using the input from knowl-edgeable employees taking into account an expected amount of spoilage, waste,and downtime.

Standards help managers in five areas:

1. Plan by providing unit amounts for budgeting.

2. Control by setting target levels of performance.

3. Motivate employees by serving as performance benchmarks.

4. Set sales prices of products or services by providing unit costs.

5. Simplify record keeping and reduce clerical costs.

Requirement 2

Compute the price and efficiency variances for direct materials and directlabor. Are these variances favorable or unfavorable? Why?

Direct Materials Price Variance

The actual quantity of direct materials used was the total number of maps pro-duced (118,000) multiplied by the amount of direct materials used per map.Actual direct materials usage was 41 square yards per map, for an actual quan-tity of 4,838,000 square yards of materials.

To compute the variance, multiply the difference between the actual andstandard costs per unit by the actual quantity. Bumpy estimated a cost of $0.015per square yard, but the actual cost was $0.016 per square yard, for a differenceof $0.001, which multiplied by 4,838,000 yields a variance of $4,838:

Direct Materials Price Variance (ActualPri! cce Standard Price) Actual Quantity of Ma" # tterial($0.016 $0.015) (41sq. Yards Usa! " # gge per Unit 118,000 Units)$0.0013 4,83

#! 88,000

$4,838 U!

1eSG_C20_0131792075.Qxd 11/22/06 11:41 AM Page 689

Page 722: polpoa_sg

690 Chapter 20 | Demo Doc 2 Solutions

This variance is unfavorable because the actual price is greater than thestandard price.

Direct Materials Quantity Variance

This variance measures whether the quantity of materials actually used to pro-duce the actual number of output units is within the standard allowed for thatnumber of outputs. It is calculated by multiplying the difference between quanti-ties of material (actual vs. standard) by the standard price per unit of material.

We know from computing the direct materials price variance that the actualquantity of direct materials used was 41 square yards per map ! 118,000 maps =4,838,000 square yards. From this amount, we subtract the standard quantity(40 square yards per map ! 118,000 maps = 4,720,000 square yards) for a differ-ence of 118,000 square yards.

We then multiply that difference in quantity by the standard price per unit(remember, efficiency variance is measured against the standard flexible budget,not actual costs), for a total variance of 118,000 ! $0.015 = $1,770:

This variance is unfavorable because the actual material used per map, 41square yards, was greater than the standard per map, 40 square yards.

Direct Materials Quantity Variance (Actual! QQuantity Standard Quantity) StandardPri$ " cce[(41 118,000) (40 118,000)] $0.01! " $ " " 55(4,838,000 4,720,000) $0.015118,000

! $ " ! " !

$0.015$1,770 U

Part 1 Part 2 Part 3 Part 4 Part 5 Part 6 Part 7Demo DocComplete

Direct Labor Rate Variance

This variance measures the difference between the actual price per unit (in thiscase, rate per hour for labor) and the standard price per unit, multiplied by theactual quantity of input (that is, hours worked).

In this case, Bumpy estimated a rate of $12.00 per hour for labor, and actualcosts were $12.10, so the difference is $0.10 per hour, which multiplied by theactual hours of 11,600 yields a variance of $1,160:

This variance is unfavorable because the actual hourly rate for labor, $12.10,was greater than the standard rate for labor, $12.00.

Direct Labor Efficiency Variance

This variance measures the difference between the actual quantity of input (inthis case, the number of hours Bumpy actually purchased) and the standardquantity of input, multiplied by the standard price per input unit (hourly cost ofthe labor).

In this case, we know that the actual hours purchased was 11,600. To com-pute standard hours, multiply the standard rate per map, 0.10, by the actualnumber of maps produced, 118,000 = 11,800. So the difference between actual

Direct Labor Rate Variance (ActualRate Sta! $ nndardRate) ActualHours($12.10 $12.00)

" ! $ 11,600

$0.10 11,600$1,160 U

" ! " !

1eSG_C20_0131792075.Qxd 10/24/06 2:10 PM Page 690

Page 723: polpoa_sg

Demo Doc 2 Solutions | Chapter 20 691

and standard hours is 200, multiplied by the standard price per hour of $12.00 =$2,400:

This variance is favorable because actual hours used were fewer than stan-dard hours.

Requirement 3

Journalize the usage of direct materials including the related variance.

Direct Labor Efficiency Variance (ActualHo! uurs StandardHours) StandardPrice[11,6

$ " ! 000 (0.10 118,000)] $12.00

(11,600 11,8$ " "

! $ 000) $12.00200 $12.00

$2,400 F

" !$ " !

6 Record transactions at standard cost andprepare a standardcost income statement

Part 1 Part 2 Part 3 Part 4 Part 5 Part 6 Part 7Demo DocComplete

Journal Entry:

Work in Process Inventory (118,000 3 40 3 $0.015) 70,800 ??? 1,770 Materials Inventory (118,000 3 41 3 $0.015) 72,570

PostDate Accounts Ref. Dr. Cr.

Journal Entry:

Work in Process Inventory (118,000 3 40 3 $0.015) 70,800 Direct Materials Quantity Variance (118,000 3 $0.015) 1,770 Materials Inventory (118,000 3 41 3 $0.015) 72,570

PostDate Accounts Ref. Dr. Cr.

Usage of Direct MaterialsBumpy debits Work in Process Inventory for the standard price multiplied by thestandard quantity of direct materials that should have been used for the actualoutput of 118,000 maps. This maintains inventory at standard cost. MaterialsInventory is credited for the actual quantity of materials put into productionmultiplied by the standard price:

So where does the rest of the debit side of this entry come from?We learned in Requirement 2 that because Bumpy used more materials than

the standard, its direct materials quantity variance was $1,770 unfavorable. Thisunfavorable variance increases the cost of production. Unfavorable variances willalways be debited.

1eSG_C20_0131792075.Qxd 10/24/06 2:10 PM Page 691

Page 724: polpoa_sg

692 Chapter 20 | Demo Doc 2 Solutions

Requirement 4

For manufacturing overhead, compute the total variance, the flexiblevariance, and the production volume variance. Are these variancesfavorable or unfavorable? Why?

5 Analyze manufacturingoverhead in a standardcost system

Part 1 Part 2 Part 3 Part 4 Part 5 Part 6 Part 7Demo DocComplete

Total Overhead Variance

Total overhead variance is the difference between actual overhead cost and stan-dard overhead allocated to production.

The standard overhead allocated to production is the standard cost of theoverhead per map times the number of maps actually produced. We know fromthe question that the standard overhead cost per map is $0.30 (variable cost) +$0.80 (fixed cost) = $1.10. So the standard overhead allocated to production =118,000 ! $1.10 = $129,800. Actual overhead cost as given in the question is$126,000, the difference being $3,800:

Part 1 Part 2 Part 3 Part 4 Part 5 Part 6 Part 7Demo DocComplete

Overhead Flexible Budget Variance

The flexible budget variance is equal to the difference between the actual overheadcost and the flexible budget overhead for the actual number of maps produced.

To compute the flexible budget overhead variance, the fixed portion of theoverhead must be separated from the variable part. The variable part of the over-head is flexible, therefore the variable cost of overhead per map ($0.30) is multi-plied by the actual number of maps produced (118,000); the variable part of over-head is thus $35,400.

The fixed portion of the overhead is not flexible within the relevant range, soto compute the full fixed part of the overhead, the fixed cost of overhead per map($0.80) must be multiplied by the static expected budget output of 120,000 maps.The fixed part of overhead is thus $96,000, the same as the original budgetedamount. Total flexible budget overhead is $35,400 + $96,000 = $131,400.

We know that the actual overhead cost was $126,000, the difference betweenactual and flexible thus being $5,400:

Overhead Flexible Budget Variance Actual Ov! eerhead Cost – Flexible BudgetOverhead for the ActualNumber of Outputs$126,000 [(1! " 118,000 $0.30)+ (120,000 $0.80)]$126,00

# # ! 00 ($35,400 + $96,000)

$126,000 $131,400$5,4

"! "! 000 F

Total Overhead Variance Actual Overhead Cos! tt Standard Overhead Allocated to Productio" nn$126,000 (118,000 $1.10)$126,000 $129

! " # ! " ,,800

$3,800 F!

The variance is favorable because the actual overhead is less than the stan-dard overhead. Overapplied overhead is favorable because enough cost was putinto production.

1eSG_C20_0131792075.Qxd 11/22/06 11:42 AM Page 692

Page 725: polpoa_sg

Demo Doc 1 | Chapter 1 693Demo Doc 2 Solutions | Chapter 20 693

The flexible budget variance is favorable because the actual overhead cost isless than the flexible budget.

Part 1 Part 2 Part 3 Part 4 Part 5 Part 6 Part 7Demo DocComplete

Part 1 Part 2 Part 3 Part 4 Part 5 Part 6 Part 7Demo DocComplete

Production Volume Variance

The production volume variance arises when actual production differs fromexpected production. It is calculated as the difference between the flexible budgetoverhead for the actual number of outputs and the standard overhead allocatedto actual production.

Standard overhead allocated to actual production is calculated by multiply-ing the number of maps actually produced by the standard overhead per unit:

118,000 ! $1.10 ! $129,800

We know the flexible budget overhead for the actual number of outputs fromcalculating the overhead flexible budget variance:

Production Volume Variance Flexible Budget! OOverhead for ActualNumber of Outputs$ SStandard Overhead Allocated to ActualProduuction

$131,400 ($1.10 118,000)$131,400

! $ " ! $$!

$129,800$1,600 U

This variance accounts for Bumpy producing fewer maps, 118,000, thanexpected output, 120,000. Bumpy didn’t use its production capacity as efficientlyas possible. Whenever a business produces less than expected, the production vol-ume variance will be unfavorable.

So the difference is calculated as follows:

Flexible Budget Overhead for ActualNumber oof Outputs (118,000 $0.30) (120,000 $0! " # " ..80)$35,400 $96,000$131,400

! #!

1eSG_C20_0131792075.Qxd 10/24/06 2:10 PM Page 693

Page 726: polpoa_sg

694 Chapter 20 | Quick Practice Questions

Quick Practice QuestionsTrue/False

_____ 1. Total variable costs change as production volume changes in a flexiblebudget.

_____ 2. At the end of the period, all variance accounts are closed to zero outtheir balances.

_____ 3. The sales volume variance arises because the number of units actuallysold differs from the number of units expected to be sold according tothe master budget.

_____ 4. A price variance is the difference between the actual unit price of aninput and the standard unit price of the input, multiplied by the stan-dard input quantity.

_____ 5. If the standard quantity allowed is less than the actual quantity used,the efficiency variance is favorable.

_____ 6. An efficiency variance is the difference between the actual quantity ofinput and the standard quantity of input, multiplied by the actual unitprice of input.

_____ 7. Manufacturing overhead allocated to production equals the standardpredetermined manufacturing overhead rate times the actual quantityof allocation base allowed for the standard number of outputs.

_____ 8. The overhead flexible budget variance is the difference between theactual overhead cost and the flexible budget overhead for budgetedproduction.

_____ 9. The production volume variance is favorable whenever actual output isless than expected output.

_____10. In standard costing, the journal entry to record the direct labor costsincurred includes a debit to Manufacturing Wages for the actual hoursworked at the standard price for direct labor.

Multiple Choice

1. Which of the following is true for a static budget?a. Adjusted for changes in the level of activityb. Prepared for only one level of activityc. A budget that stays the same from one period to the nextd. Also known as a fixed budget

1eSG_C20_0131792075.Qxd 10/24/06 2:10 PM Page 694

Page 727: polpoa_sg

Quick Practice Questions | Chapter 20 695

2. Sweet Baby Diaper Company sells disposable diapers for $0.20each. Variable costs are $0.05 per diaper while fixed costs are$75,000 per month for volumes up to 850,000 diapers and $112,500for volumes above 850,000 diapers. What is the monthly operatingincome for 800,000 diapers and 900,000 diapers of volume?a. $22,500 and $7,500, respectivelyb. $60,000 and $45,000, respectivelyc. $45,000 and $22,500, respectivelyd. $7,500 and $60,000, respectively

3. A graph of a flexible budget formula reflects fixed costs of$30,000 and total costs of $90,000 at a volume of 6,000 units.Assuming the relevant range is 1,000 to 12,000 units, what is thetotal cost of 10,000 units on the graph?a. $100,000b. $130,000c. $160,000d. $180,000

4. The sales volume variance is the difference between whichamounts?a. Number of units actually sold and number of units expected to be sold

according to the static budgetb. Amounts in the flexible budget and the static budgetc. Actual results and amounts in the flexible budgetd. Actual sales volume and normal sales volume

5. The flexible budget variance is the difference between whichamounts?a. Actual results and amounts in the static budgetb. Amounts in the flexible budget and the actual resultsc. Amounts in the flexible budget and the static budgetd. The budgeted amounts for each level of activity in the flexible budget

6. What does a flexible budget help to measure?a. The efficiency of operations at the actual activity levelsb. The amount by which standard and expected prices differc. Both a and bd. None of the above

7. Global Engineering’s actual operating income for the current yearis $50,000. The flexible budget operating income for actual volumeachieved is $40,000, while the static budget operating income is$53,000. What is the sales volume variance for operating income?a. $10,000 Ub. $10,000 Fc. $13,000 Ud. $13,000 F

8. Tiger’s Golf Center reported actual operating income for thecurrent year of $60,000. The flexible budget operating incomefor actual volume achieved is $55,000, while the static budgetoperating income is $58,000. What is the flexible budget variancefor operating income?a. $2,000 Fb. $3,000 Uc. $5,000 Fd. $5,000 U

1eSG_C20_0131792075.Qxd 10/24/06 2:10 PM Page 695

Page 728: polpoa_sg

696 Chapter 20 | Quick Practice Questions

9. What does a favorable direct materials efficiency varianceindicate?a. Actual cost of direct materials was less than the standard cost of

direct materialsb. Actual quantity of direct materials used was less than the standard

quantity for actual outputc. Standard quantity of direct materials for actual output was less than

the actual quantity of direct materials usedd. Actual quantity of direct materials used was greater than the stan-

dard quantity for budgeted output

10. Western Outfitters Mountain Sports projected 2008 sales of75,000 units at a unit sale price of $12.00. Actual 2008 sales were72,000 units at $14.00 per unit. Actual variable costs, budgeted at$4.00 per unit, totaled $4.75 per unit. Budgeted fixed coststotaled $375,000, while actual fixed costs amounted to $400,000.

What is the flexible budget variance for variable expenses?a. $12,000 Fb. $25,000 Fc. $54,000 Ud. $54,000 F

Quick Exercises

20-1. Smart Toys Manufacturing projected 2008 sales of 10,000 unitsat $12.00 per unit. Actual sales for the year were 15,000 unitsat $12.50 per unit. Actual variable expenses, budgeted at $5.00per unit, amounted to $4.80 per unit. Actual fixed expenses,budgeted at $60,000, totaled $62,500.

Prepare Smart Toys’ income statement performance report for 2008,including both flexible budget variances and sales volume variances.

Output UnitsSales revenueVariable expensesFixed expensesTotal expensesOperating income (loss)

1

Actual Resultsat Actual Prices

2

Flexible BudgetVariance

3Flexible Budget

for Actual Numberof Output Units

4

Sales VolumeVariance

5

Static (Master)Budget

SMART TOYS MANUFACTURINGIncome Statement Performance Report

Year Ended 2008

1eSG_C20_0131792075.Qxd 10/24/06 2:10 PM Page 696

Page 729: polpoa_sg

Quick Practice Questions | Chapter 20 697

20-2. Dawkins Company established a master budget volume of19,500 units for September. Actual overhead costs incurredamounted to $17,800. Actual production for the month was22,000 units. The standard variable overhead rate was $0.50 perdirect labor hour. The standard fixed overhead rate was $0.25per direct labor hour. One direct labor hour is the standardquantity per finished unit.

a. Compute the total manufacturing overhead cost variance.

b. Compute the overhead flexible budget variance.

c. Compute the production volume variance.

20-3. Standard Products Company recognizes variances from stan-dards at the earliest opportunity, and the quantity of directmaterials purchased is equal to the quantity used. The follow-ing information is available for the most recent month:

Direct Materials Direct Labor

Standard quantity/unit 6.00 lbs. 2.5 hrs.Standard price/lb. or hr. $8.10/lb. $8.00/hr.Actual quantity/unit 6.25 lbs. 2.8 hrs.Actual price/lb. or hr. $8.00/lb. $7.50/hr.Price variance $562.50 F $1,260.00 FEfficiency variance $1,822.50 U $2,160.00 UStatic budget volume 800 unitsActual volume 900 units

Actual overhead $11,000.00Standard variable overhead $5/unitStandard fixed overhead $5,600Overhead flexible budget variance $900 UProduction volume variance $700 F

1eSG_C20_0131792075.Qxd 10/24/06 2:10 PM Page 697

Page 730: polpoa_sg

a. Journalize the purchase and usage of direct materials includingthe related variances.

Journal Entry:

PostDate Accounts Ref. Dr. Cr.

Journal Entry:

PostDate Accounts Ref. Dr. Cr.

Journal Entry:

PostDate Accounts Ref. Dr. Cr.

Journal Entry:

PostDate Accounts Ref. Dr. Cr.

b. Journalize the direct labor costs incurred and the application ofdirect labor, including the related variances.

c. Journalize the application of overhead costs including the recogni-tion of the overhead variances.

Journal Entry:

PostDate Accounts Ref. Dr. Cr.

698 Chapter 20 | Quick Practice Questions

1eSG_C20_0131792075.Qxd 10/24/06 2:10 PM Page 698

Page 731: polpoa_sg

Quick Practice Questions | Chapter 20 699

Sales revenueVariable expensesFixed expensesTotal expensesOperating income (loss)

25,000 30,000 37,000

Flexible Budgetper Output Unit Output Units (games)

PARKLAND, INC.Flexible BudgetYear Ending 2010

20-4. Jeremy Industries has the following information regardingdirect materials:

Actual pounds purchased and used 42,000Standard quantity 3 pounds per finished goodActual production 15,000 finished goodsDirect materials efficiency variance $10,500 FDirect materials price variance $8,820 U

Compute Jeremy’s standard price per pound and actual price per pound.

20-5. Parkland, Inc., sells board games for $15.00, resulting in a con-tribution margin of $9.00 per game. Fixed costs are budgetedat $212,000 per quarter for volumes up to 30,000 games and$242,000 for volumes exceeding 30,000 games.

Prepare the flexible budget for the next quarter for volume levels of25,000, 30,000, and 37,000 games.

Journal Entry:

PostDate Accounts Ref. Dr. Cr.

1eSG_C20_0131792075.Qxd 10/24/06 2:10 PM Page 699

Page 732: polpoa_sg

700 Chapter 20 | Do It Yourself! Question 1

Sales revenueVariable expensesFixed expensesTotal expensesOperating income (loss)

32,000 34,000 36,000

Flexible Budgetper Output Unit Output Units (beds)

FLUFFY FOAM BEDSFlexible BudgetYear Ending 2010

2. Prepare Fluffy’s income statement performance report for 2010.

Output UnitsSales revenueVariable expensesFixed expensesTotal expensesOperating income (loss)

1

Actual Resultsat Actual Prices

2

Flexible BudgetVariance

3Flexible Budget

for Actual Numberof Output Units

4

Sales VolumeVariance

5

Static (Master)Budget

FLUFFY FOAM BEDSIncome Statement Performance Report

Year Ended 2010

Do It Yourself! Question 1

Flexible Budgets and Income StatementPerformance ReportManagement at Fluffy Foam Beds predicted 2010 sales of 32,000 beds at a priceof $180 per bed. Actual sales for the year were 34,200 beds at $185 per bed.Following is a breakdown of Fluffy’s costs for 2010:

Budgeted Actual

Units sold 32,000 34,200

Variable costs $ 70 $ 75

Fixed costs $3,000,000 $3,100,000

Requirements1. Using Fluffy’s estimated budget values for costs and sales price, develop

flexible budgets for 32,000, 34,000, and 36,000 beds.

1eSG_C20_0131792075.Qxd 10/24/06 2:10 PM Page 700

Page 733: polpoa_sg

Do It Yourself! Question 2

Standard CostsCircle CD manufactures blank CD cases. Circle uses a standard cost system tocontrol manufacturing costs.

The following standard unit cost information is based on the static budget vol-ume of 80,000 CD cases per month:

In this example,

Actual cost and production volume information:

• Actual production was 88,000 cases.

• Actual direct materials usage was 102 square feet per case, at anactual cost of $0.24 per square foot.

• Actual direct labor usage of 42,000 hours at $18.15, total cost of $762,300.

• Total actual overhead was $980,000.

Requirements

1. Compute the price and efficiency variances for direct materials and direct labor.

Direct materials price variance

Direct materials efficiency variance

Direct labor rate variance

Direct labor efficiency variance

2. For manufacturing overhead, compute the total variance, the flexiblevariance, and the production volume variance.

Total overhead variance

Overhead flexible budget variance

Production volume variance

TotalBudgetedFixedManufacturingOverhead!! " !

$10 80,000 CD Cases$800,000

Direct materials (100 sq. ft. @ $0.25 per sq. ft.)Direct labor (0.50 hours @ $18.00 per hour)Manufacturing overhead: Variable (1 hour @ $2.00 per hour) Fixed (2 hours @ $5.00 per hour)Total cost per CD case

$25.009.00

12.00$46.00

2.0010.00

1eSG_C20_0131792075.Qxd 10/24/06 2:10 PM Page 701

Page 734: polpoa_sg

702 Chapter 20 | Quick Practice Solutions

Quick Practice SolutionsTrue/False

T 1. Total variable costs change as production volume changes in a flexible budget. (p. 1069)

T 2. At the end of the period, all variance accounts are closed to zero out their balances. (p. 1087)

T 3. The sales volume variance arises because the number of units actually sold differs from the number of units expected to be sold according to the master budget. (p. 1070)

F 4. A price variance is the difference between the actual unit price of an input and the standard unit price of the input, multiplied by thestandard input quantity.False–A price variance is the difference between the actual unit price of an input and the standard unit price of the input,hmultiplied by the actual input quantity. (p. 1075)

F 5. If the standard quantity allowed is less than the actual quantity used, the efficiency variance is favorable.False–If the standard quantity allowed is less than the actual quantity used, the efficiency variance is unfavorable. (p. 1075)

F 6. An efficiency variance is the difference between the actual quantity of input and the standard quantity of input, multiplied by the actual unit price of input.False–An efficiency variance is the difference between the actual quantity of input and the standard quantity of input, multiplied by the standard unit price of input. (p. 1075)

F 7. Manufacturing overhead allocated to production equals the standard predetermined manufacturing overhead rate times the actual quantity of allocation base allowed for the standard number of outputs.False–Manufacturing overhead allocated to production equals the standard predetermined manufacturing overhead rate times the standard quantity of allocation base allowed for the actual number of outputs. (p. 1081)

F 8. The overhead flexible budget variance is the difference between the actual overhead cost and the flexible budget overhead for budgeted production.False–The overhead flexible budget variance is the difference between the actual overhead cost and the flexible budget overhead for the actual number of outputs. (p. 1082)

1eSG_C20_0131792075.Qxd 10/24/06 2:10 PM Page 702

Page 735: polpoa_sg

Quick Practice Solutions | Chapter 20 703

F 9. The production volume variance is favorable whenever actual output is less than expected output.False–The production volume variance is unfavorable whenever actual output is less than expected output. (p. 1083)

T 10. In standard costing, the journal entry to record the direct labor costs incurred includes a debit to Manufacturing Wages for the actual hours worked at the standard price for direct labor. (p. 1084)

Multiple Choice

1. Which of the following is true for a static budget? (p. 1068)a. Adjusted for changes in the level of activityb. Prepared for only one level of activityc. A budget that stays the same from one period to the nextd. Also known as a fixed budget

2. Sweet Baby Diaper Company sells disposable diapers for $0.20each. Variable costs are $0.05 per diaper while fixed costs are$75,000 per month for volumes up to 850,000 diapers and$112,500 for volumes above 850,000 diapers. What is the monthlyoperating income for 800,000 diapers and 900,000 diapers ofvolume? (p. 1068)a. $22,500 and $7,500, respectivelyb. $60,000 and $45,000, respectivelyc. $45,000 and $22,500, respectivelyd. $7,500 and $60,000, respectively

3. A graph of a flexible budget formula reflects fixed costs of$30,000 and total costs of $90,000 at a volume of 6,000 units.Assuming the relevant range is 1,000 to 12,000 units, what is thetotal cost of 10,000 units on the graph? (p. 1069)a. $100,000b. $130,000c. $160,000d. $180,000

4. The sales volume variance is the difference between whichamounts? (p. 1070)a. Number of units actually sold and number of units expected to be sold

according to the static budgetb. Amounts in the flexible budget and the static budgetc. Actual results and amounts in the flexible budgetd. Actual sales volume and normal sales volume

5. The flexible budget variance is the difference between whichamounts? (p. 1070)a. Actual results and amounts in the static budgetb. Amounts in the flexible budget and the actual resultsc. Amounts in the flexible budget and the static budgetd. The budgeted amounts for each level of activity in the flexible

budget

1eSG_C20_0131792075.Qxd 10/24/06 2:10 PM Page 703

Page 736: polpoa_sg

704 Chapter 20 | Quick Practice Solutions

6. What does a flexible budget help to measure? (p. 1069)a. The efficiency of operations at the actual activity levelsb. The amount by which standard and expected prices differc. Both a and bd. None of the above

7. Global Engineering’s actual operating income for the currentyear is $50,000. The flexible budget operating income for actualvolume achieved is $40,000 while the static budget operatingincome is $53,000. What is the sales volume variance foroperating income? (p. 1070)a. $10,000 Ub. $10,000 Fc. $13,000 Ud. $13,000 F

8. Tiger’s Golf Center reported actual operating income for thecurrent year of $60,000. The flexible budget operating incomefor actual volume achieved is $55,000, while the static budgetoperating income is $58,000. What is the flexible budget variancefor operating income? (p. 1070)a. $2,000 Fb. $3,000 Uc. $5,000 F

d. $5,000 U

9. What does a favorable direct materials efficiency varianceindicate? (p. 1078)a. Actual cost of direct materials was less than the standard cost of

direct materialsb. Actual quantity of direct materials used was less than the standard

quantity for actual outputc. Standard quantity of direct materials for actual output was less than

the actual quantity of direct materials usedd. Actual quantity of direct materials used was greater than the stan-

dard quantity for budgeted output

10. Western Outfitters Mountain Sports projected 2008 sales of75,000 units at a unit sale price of $12.00. Actual 2008 sales were72,000 units at $14.00 per unit. Actual variable costs, budgeted at$4.00 per unit, totaled $4.75 per unit. Budgeted fixed coststotaled $375,000 while actual fixed costs amounted to $400,000.

What is the flexible budget variance for variable expenses? (p. 1060)a. $12,000 Fb. $25,000 Fc. $54,000 Ud. $54,000 F

1eSG_C20_0131792075.Qxd 10/24/06 2:10 PM Page 704

Page 737: polpoa_sg

Quick Practice Solutions | Chapter 20 705

Quick Exercise Solutions

20-1. Smart Toys Manufacturing projected 2008 sales of 10,000 unitsat $12.00 per unit. Actual sales for the year were 15,000 unitsat $12.50 per unit. Actual variable expenses, budgeted at $5.00per unit, amounted to $4.80 per unit. Actual fixed expenses,budgeted at $60,000, totaled $62,500. (p. 1071)

Prepare Smart Toys’ income statement performance reportfor 2008, including both flexible budget variances and salesvolume variances.

Output UnitsSales revenueVariable expensesFixed expensesTotal expensesOperating income (loss)

$187,50072,00062,500

134,500$ 53,000

$120,00050,00060,000

110,000$ 10,000

15,000

1

Actual Resultsat Actual Prices

2

Flexible BudgetVariance

3Flexible Budget

for Actual Numberof Output Units

4

Sales VolumeVariance

10,000$60,000 F

25,000 U0

25,000 U$35,000 F

5,000F$180,000

75,00060,000

135,000$ 45,000

15,000$7,500 F3,000 F2,500 U

500 F$8,000 F

0

5

Static (Master)Budget

SMART TOYS MANUFACTURINGIncome Statement Performance Report

Year Ended 2008

20-2. Dawkins Company established a master budget volume of19,500 units for September. Actual overhead costs incurredamounted to $17,800. Actual production for the month was22,000 units. The standard variable overhead rate was $0.50 perdirect labor hour. The standard fixed overhead rate was $0.25per direct labor hour. One direct labor hour is the standardquantity per finished unit. (pp. 1080–1083)

Actual overhead ! $17,800Flexible budget overhead for actual production ! ($0.50 ! 22,000) # ($0.25 ! 19,500)

! $15,875Standard overhead allocated to production ! ($0.50 ! 22,000) # ($0.25 ! 22,000)

! $16,500a. Compute the total manufacturing overhead cost variance.

Total manufacturing overhead cost variance ! $17,800 – $16,500 ! $1,300 U

b. Compute the overhead flexible budget variance.Overhead flexible budget variance ! $17,800 – $15,875 ! $1,925 U

c. Compute the production volume variance.Production volume variance ! $15,875 – $16,500 ! $625 F

1eSG_C20_0131792075.Qxd 10/24/06 2:10 PM Page 705

Page 738: polpoa_sg

706 Chapter 20 | Quick Practice Solutions

20-3. Standard Products Company recognizes variances from stan-dards at the earliest opportunity, and the quantity of directmaterials purchased is equal to the quantity used. The follow-ing information is available for the most recent month:(pp. 1083–1085)

Direct Direct Materials Labor

Standard quantity/unit 6.00 lbs. 2.5 hrs.

Standard price/lb. or hr. $8.10/lb. $8.00/hr.

Actual quantity/unit 6.25 lbs. 2.8 hrs.

Actual price/lb. or hr. $8.00/lb. $7.50/hr.

Price variance $562.50 F $1,260 F

Efficiency variance $1,822.50 U $2,160 U

Static budget volume 800 units

Actual volume 900 units

Actual overhead $11,000

Standard variable overhead $5/unit

Standard fixed overhead $5,600

Overhead flexible budget variance $900 U

Production volume variance $700 F

a. Journalize the purchase and usage of direct materials includingthe related variances.

b. Journalize the direct labor costs incurred and the application ofdirect labor, including the related variances.

c. Journalize the application of overhead costs including the recogni-tion of the overhead variances.

General Journal

Date Accounts Dr. Cr.

a. Materials Inventory Accounts Payable Direct Materials Price Variance

45,562.5045,000.00

562.50

General Journal

Date Accounts Dr. Cr.

a. Work in Process InventoryDirect Materials Efficiency Variance Materials Inventory

43,740.001,822.50

45,562.50

1eSG_C20_0131792075.Qxd 10/24/06 2:10 PM Page 706

Page 739: polpoa_sg

Quick Practice Questions | Chapter 20 707Chapter 20 | Quick Practice Solutions 707

General Journal

Date Accounts Dr. Cr.

b. Manufacturing Wages Wages Payable Direct Labor Price Variance

20,160.0018,900.00

1,260.00

General Journal

Date Accounts Dr. Cr.

b. Work in Process InventoryDirect Labor Efficiency Variance Manufacturing Wages

18,000.002,160.00

20,160.00

General Journal

Date Accounts Dr. Cr.

c. Work in Process Inventory Manufacturing Overhead [($7 ! 900) " ($5 ! 900)]

10,800.0010,800.00

General Journal

Date Accounts Dr. Cr.

c. Overhead Flexible Budget Variance Production Volume Variance Manufacturing Overhead

900.00700.00200.00

1eSG_C20_0131792075.Qxd 10/24/06 2:10 PM Page 707

Page 740: polpoa_sg

708 Chapter 20 | Quick Practice Solutions

20-4. Jeremy Industries has the following information regardingdirect materials: (pp. 1076–1080)

Actual pounds purchased and used 42,000Standard quantity 3 pounds per finished goodActual production 15,000 finished goodsDirect materials efficiency variance $10,500FDirect materials price variance $8,820U

Compute Jeremy’s standard price per pound and actual priceper pound.

Let Y ! Standard price per poundDirect materials efficiency variance ! [(3 " 15,000) – 42,000]Y ! $ 10,500

3,000Y ! $10,500Y ! $3.50

Let Y ! actual price per poundDirect materials price variance ! (Y – $3.50) " 42,000 ! $8,820

Y – $3.50 ! $8,820/42,000Y – $3.50 ! $0.21

Y ! $3.71

20-5. Parkland, Inc., sells board games for $15.00, resulting in a con-tribution margin of $9.00 per game. Fixed costs are budgetedat $212,000 per quarter for volumes up to 30,000 games and$242,000 for volumes exceeding 30,000 games. (p. xx)

Prepare the flexible budget for the next quarter for volumelevels of 25,000, 30,000, and 37,000 games.

Sales revenueVariable expensesFixed expensesTotal expensesOperating income (loss)

$15.006.00

$375,000150,000212,000362,000

$113,000

$450,000180,000212,000392,000

$558,000

$555,000222,000242,000464,000

$991,000

25,000 30,000 37,000

Flexible Budgetper Output Unit Output Units (games)

PARKLAND, INC.Flexible BudgetYear Ending 2010

1eSG_C20_0131792075.Qxd 11/22/06 11:43 AM Page 708

Page 741: polpoa_sg

Do It Yourself! Question 1 Solutions | Chapter 20 709

Sales revenueVariable expensesFixed expensesTotal expensesOperating income (loss)

$18070

$5,760,0002,240,0003,000,0005,240,000

$ 520,000

$6,120,0002,380,0003,000,0005,380,000

$ 740,000

$6,480,0002,520,0003,000,0005,520,000

$ 960,000

32,000 34,000 36,000

Flexible Budgetper Output Unit Output Units (beds)

FLUFFY FOAM BEDSFlexible BudgetYear Ending 2010

Output UnitsSales revenueVariable expensesFixed expensesTotal expensesOperating income (loss)

$6,327,0002,565,0003,100,0005,665,000

$ 662,000

$5,760,0002,240,0003,000,0005,240,000

$ 520,000

34,200$171,000 F

171,000 U100,000 U271,000 U

$100,000 U

0$6,156,0002,394,0003,000,0005,394,000

$ 762,000

34,200$396,000 F

154,000 U0

154,000 U$242,000 F

2,200F

1

Actual Resultsat Actual Prices

2

Flexible BudgetVariance

3Flexible Budget

for Actual Numberof Output Units

4

Sales VolumeVariance

32,000

5

Static (Master)Budget

FLUFFY FOAM BEDSIncome Statement Performance Report

Year Ended 2010

1 Prepare a flexible bud-get for the incomestatement

2 Prepare an incomestatement perfor-mance report

Do It Yourself! Question 1 Solutions

Requirement 1

Using Fluffy’s estimated budget values for costs and sales price, developflexible budgets for 32,000, 34,000, and 36,000 beds.

Requirement 2Prepare Fluffy’s income statement performance report for 2010.

1eSG_C20_0131792075.Qxd 10/24/06 2:10 PM Page 709

Page 742: polpoa_sg

710 Chapter 20 | Do It Yourself! Question 2 Solutions

4 Compute standardcost variances fordirect materials anddirect labor

Do it Yourself! Question 2 Solutions

Requirement 1

Compute the price and efficiency variances for direct materials and directlabor.

Direct Materials Price Variance

Direct Materials Efficiency Variance

Direct Labor Rate Variance

Direct Labor Efficiency Variance

Direct Labor Efficiency Variance (ActualHo! uurs StandardHours) StandardPrice[42,0

$ " ! 000 (0.50 88,000)] $18.00

$36,000 F$ " "

!

Direct Labor Rate Variance (ActualRate S! $ ttandardRate) ActualHours($18.15 $18.0

" ! $ 00) 42,000 Hours

$6,300 U"

!

Direct Materials Quantity Variance (Actual! QQuantity Standard Quantity) StandardPri$ " cce[(102 88,000) (100 88,000)] $0.25$4

! " $ " " ! 44,000 U

Direct MaterialPrice Variance (ActualPric! ee StandardPrice) Actual Quantity($0.24

$ " ! $$ " " !

$0.25) (102 88,000 units)$89,760 F

1eSG_C20_0131792075.Qxd 10/24/06 2:10 PM Page 710

Page 743: polpoa_sg

Do It Yourself! Question 2 Solutions | Chapter 20 711

Production Volume Variance Flexible Budget! OOutput for ActualNumber of OutputsSta$ nndard Overhead Allocated to ActualProductiion

$976,000 ($12 88,000)$80,000 F

! $ " !

5 Analyze manufacturingoverhead in a standardcost system

Overhead Flexible Budget Variance Actual Ov! eerhead Cost Flexible Budget Overhead for$ActualNumber of Outputs$980,000 [(88! $ ,,000 $2.00) (80,000 $10.00)]$4,000 U

" # " !

Requirement 2

For manufacturing overhead, compute the total variance, the flexiblevariance, and the production volume variance.

Total Overhead Variance

Overhead Flexible Budget Variance

Production Volume Variance

Total Overhead Variance Actual Overhead Cos! tt Standard Overhead Allocated toProductio$ nn$980,000 (88,000 $12)$76,000 F

! $ " !

1eSG_C20_0131792075.Qxd 10/24/06 2:10 PM Page 711

Page 744: polpoa_sg

The Power of PracticeFor more practice using the skills learned in this chapter, visit MyAccountingLab.There you will find algorithmically generated questions that are based on theseDemo Docs and your main textbook’s Review and Accounting Practice sections.

To go to MyAccountingLab and follow these steps:

1. Direct your URL to www.myaccountinglab.com.2. Log in using your name and password.3. Click the MyAccountingLab link.4. Click Study Plan in the left navigation bar.5. From the table of contents, select Chapter 20, Flexible Budgets and Standard

Costs.6. Click a link to work on the tutorial exercises.

712 Chapter 20 | The Power of Practice

1eSG_C20_0131792075.Qxd 10/24/06 2:10 PM Page 712

Page 745: polpoa_sg

Special BusinessDecisions and CapitalBudgeting

21WHAT YOU PROBABLY ALREADY KNOW

People purchase lottery tickets in the hopes of winning the grandjackpot. On November 15, 2005, seven employees of a medicalcenter won the Mega Millions grand prize of $315,000,000.Winners can choose to receive the money in 26 equal paymentsover the upcoming 25 years or in a lump sum. The seven winnersdecided to take a lump-sum payment. You probably already knowthat opting for the lump-sum payment means that something lessthan $315,000,000 is received. The reason is the time value ofmoney. Receiving a dollar now is worth more than receiving it in ayear, 5 years, or 25 years in the future. The winnings are“discounted,” reduced to the present value. The seven winnersreceived $187,100,000. In this chapter, we will study how tocalculate the present value of future cash flows and how thatinformation is used to help management make decisions.

Learning Objectives

Identify the relevant information for a special business decision.

To make business decisions, management will compare relevantinformation between alternative courses of action. Relevant informationincludes items that will differ between alternatives and be incurred in thefuture. Relevant information, therefore affects the decisions that are made.

Make five types of short-term special business decisions.

Some short-term special business decisions that management makesinclude accepting a special sales order (see Exhibit 21-4, p. 1121),dropping a business segment (see Exhibits 21-5 through 21-7, pp. 1122–1123), which product to emphasize in a product mix (seeExhibit 21-8, p. 1124), when to outsource production (see Exhibits 21-9and 21-10, pp. 1126–1127, and selling as is or processing further (seeExhibit 21-12, p. 1128).You will explore these types of decisions in DemoDoc 1.

In terms of product mix, a constraint is something that restricts theproduction or sale of a product, such as labor hours, machine hours, oravailable materials. Given sufficient demand, the products that generatethe highest contribution margin per constrained resource will maximizeoperating income.

When deciding to sell a product as is or process it further, managementshould consider the split-off point, which is the point at which the productis complete and can be sold or continues to be processed further and then

1

2

1eSG_C21_0131792075.qxd 10/24/06 3:12 PM Page 713

Page 746: polpoa_sg

714 Chapter 21 | Special Business Decisions and Capital Budgeting

3

sold after incurring more costs. The analysis compares the revenue fromselling the product as it is to the higher revenue less additional costs fromselling the product after processing further. Whichever option results in ahigher net revenue is the more profitable alternative.

Use payback and accounting rate of return models to makelonger-term capital budgeting decisions.

The decision to purchase a long-term asset is referred to as capitalbudgeting. The focus of capital budgeting is on the net cash inflows thatare projected in the future. One tool, used to assess capital investments,is the payback period which is the amount of time it takes to recover theinitial investment. You will calculate the payback period for two differentproducts in Demo Doc 2. Review Exhibit 21-13 (p. 1130) for an illustrationof the payback period concept.

You can use the accounting rate of return to determine the rate of returnfrom an asset over its useful life. You will calculate the accounting rate ofreturn for two different products in Demo Doc 2.

The average annual operating income can be computed as the net cashinflow from the asset less depreciation expense. The average amountinvested in the asset is the average of the cost amount and the residualvalue. Review Exhibit 21-14 (p. 1132) for an accounting rate of returncalculation.

Use discounted cash flow models to make longer-term capitalbudgeting decisions.

Discounted cash flow models use the concept of present value or the timevalue of money. It simply means that it is more valuable to receive moneynow than in the future. To make the best capital budgeting decision, thevalue of future cash flows must be considered in terms of the current orpresent value. The discounted cash flow models widely used to evaluatepotential expenditures are the net present value and the internal rate ofreturn (IRR). You will explore these methods in Demo Doc 2. ReviewExhibit 21-16 (p. 1135) for a computation of the net present value methodassuming equal net cash inflows and Exhibit 21-18 (p. 1136) assumingunequal net cash inflows.

Compare and contrast the four capital budgeting methods.

• Payback period.This method is simple to employ, but it does not con-sider the project profitability or the time value of money.

• Accounting rate of return.This method measures profitability, but itdoes not consider the time value of money.

• Net present value (NPV). NPV considers the time value of money, butit does not indicate the investment’s projected rate of return.

• Internal rate of return (IRR). IRR considers the time value of moneyand determines the project’s rate, but does not indicate the dollar dif-ference between the project’s present value and its investment cost.

Review Exhibit 21-20 (p. 1138) for a summary of the strengths andweaknesses of the payback period, accounting rate of return, NPV, andIRR methods.

4

5

1eSG_C21_0131792075.qxd 10/24/06 3:12 PM Page 714

Page 747: polpoa_sg

Demo Doc 1 | Chapter 21 715

Special Business DecisionsLearning Objectives 1–2

Doctor Key produces key blanks. A Kenya company offered to purchase 2,000cases of keys at a per-case price of $30. Assume selling to the Kenya company willnot affect regular customers, will not change fixed costs, will not require anyadditional variable nonmanufacturing expenses, and will use manufacturingcapacity that would otherwise be idle.

Sales revenue (35,000 cases @ $38.00 per case)Less: Manufacturing cost of goods soldGross profitLess: Marketing and administrative expensesOperating income

$1,330,000800,000

$ 530,000410,000

$ 120,000

DOCTOR KEYIncome Statement (without considering the special sale)

Demo Doc 1

Requirement

1. Assuming $500,000 of the manufacturing costs is fixed and $200,000 of themarketing and administrative costs is fixed, should Doctor accept theorder from the Kenya company?

Pack-It offered to package the key blanks for Doctor Key. Currently, Doctor pack-ages the product in-house at the following per-case costs:

Direct MaterialsDirect LaborVariable OverheaadFixed OverheadTotal Manufacturing CostCosst per Case ($105,000/35,000)

$ 30,00055,00005,000

15,000$105,000

$3.00

1eSG_C21_0131792075.qxd 10/24/06 3:12 PM Page 715

Page 748: polpoa_sg

716 Chapter 21 | Demo Doc 1

Requirement

2. Pack-It’s price per case is $2.80. Doctor would avoid all variable costsassociated with the packaging and would reduce the fixed cost by $5,000if Pack-It packages Doctor’s key blanks. Should Doctor outsource thepackaging work to Pack-It?

Doctor sells 35,000 cases of keys. Of those, 3,000 cases are for a special blue-coloredkey made for one customer. The blue key sells for the same price, $38, as all theother keys, but seems to require more expenses to produce. Doctor is consideringdropping the blue key. An analysis determined that the variable cost of producingthe blue key is $20 per case. If the blue key was not produced, fixed costs would notchange.

Requirement

3. Should Doctor stop producing the blue key?

1eSG_C21_0131792075.qxd 10/24/06 3:12 PM Page 716

Page 749: polpoa_sg

Demo Doc 1 Solutions | Chapter 21 717

Demo Doc 1 Solutions

Part 1 Part 2 Part 3 Demo DocComplete

The key to this problem is to determine what effect the special sale would have onDoctor’s operating income. Fixed costs will not change as a result of the sale;therefore, we will compare only the change in revenue with the change in vari-able costs as a result of the special sale. The decision to accept a special order willbe made if the incremental revenues exceed the incremental expenses for thespecial order.

The increase in revenue is determined by multiplying the additional casessold by the offered selling price per case:

2,000 ! $30 " $60,000

To determine the variable cost of producing the additional 2,000 units, we mustfirst determine the variable cost per unit. The manufacturing cost of producing35,000 cases was $800,000. Of the $800,000, fixed cost makes up $500,000.Therefore, manufacturing variable cost is $300,000. The variable marketing andadministrative cost is determined by subtracting the fixed portion, $200,000,from the total marketing and administrative cost of $410,000, to get variablemarketing and administrative cost of $210,000. Thus, the total variable cost ofproducing the 35,000 cases is:

$300,000 # $210,000 " $510,000

The variable cost per unit is computed by dividing the total variable cost by thenumber of units.

$510,000/35,000 cases " $14.57

The increase in variable cost with the special sales order is:

2,000 cases ! $14.57 variable cost per unit " $29,140

Incremental Analysis of Special Sales Order

Increase in revenues:

Sale of 2,000 cases @ $30 per case $60,000

Increase in variable expenses:

2,000 cases ! $14.57 29,140

Increase in operating income $30,860

Creating a profit of $15.43 per case ($30,860/2,000 cases).

1 Identify the relevantinformation for a spe-cial business decision

2 Make five types ofshort-term specialbusiness decisions

Requirement 1

Assuming $500,000 of the manufacturing costs is fixed and $200,000 of themarketing and administrative costs is fixed, should Doctor accept the orderfrom the Kenya company?

1eSG_C21_0131792075.qxd 10/24/06 3:12 PM Page 717

Page 750: polpoa_sg

718 Chapter 21 | Demo Doc 1 Solutions

Part 1 Part 2 Part 3 Demo DocComplete

Outsourcing decisions consider whether management will make or buy (out-source) products or services. Incremental analysis can be performed comparing(1) the cost to make the product, which includes direct materials, direct labor, andoverhead, to (2) the cost to buy, which includes the purchase price plus possiblysome portion of the fixed overhead that will continue with outsourcing.

In addition, a relevant opportunity cost may need to be considered. An oppor-tunity cost represents the benefit forgone by not choosing an alternative course ofaction. If the production process is outsourced and the goods are purchased, otheropportunities may be available to use the idle resources to generate profits. Theopportunity cost reduces the cost of outsourcing.

In this case, the key to this decision is to compare the Doctor’s cost savings ofnot packaging their keys with the additional cost charged by Pack-It. On the sur-face, it might seem that Pack-It is making an attractive offer to package the keyblanks for $0.20 cheaper per case than Doctor spends to package the key blanksthemselves. An incremental analysis is necessary to consider the overall effects oftotal costs and savings on the make-or-buy decision.

The following shows the differences in costs between making and outsourc-ing (buying) the packaging:

Packaging Costs Make Buy Difference

Direct materials $ 30,000 $30,000

Direct labor 55,000 55,000

Variable factory overhead 5,000 5,000

Avoidable fixed costs 15,000 10,000 5,000

Purchase cost from Pack-It (35,000 units ! $2.80 per unit) (98,000) 98,000

Total cost of packaging $105,000 $108,000 $(3,000)

Cost per unit $ 3.00 $ 3.09* $ (0.09)(divide by 35,000 cases)

*Rounded.

This analysis makes it clear that the fixed costs have an impact on this decision.In this case, Doctor will incur that $10,000 in fixed costs whether it outsources the

1 Identify the relevantinformation for a spe-cial business decision

2 Make five types ofshort-term specialbusiness decisions

Doctor should accept the special order because it would increase operatingincome.

Requirement 2

Pack-It’s price per case is $2.80. Doctor would avoid all variable costsassociated with the packaging and would reduce the fixed cost by $5,000if Pack-It packages Doctor’s key blanks. Should Doctor outsource thepackaging work to Pack-It?

1eSG_C21_0131792075.qxd 10/24/06 3:12 PM Page 718

Page 751: polpoa_sg

Demo Doc 1 Solutions | Chapter 21 719

1 Identify the relevantinformation for a spe-cial business decision

2 Make five types ofshort-term specialbusiness decisions

Part 1 Part 2 Part 3 Demo DocComplete

Part 1 Part 2 Part 3 Demo DocComplete

As in this case, management may want to consider whether discontinuing a prod-uct line, department, or component of the business would increase the operatingincome. The decision to eliminate a business segment may be made if droppingthe segment results in a decrease in total expenses greater than the decrease inrevenues.

In this case, fixed costs remain the same whether the blue key product line iscontinued or dropped; therefore, fixed costs are not relevant to the decision. Onlythe revenues and variable expenses are relevant.

Segment margin is calculated as:

In the absence of any discretionary fixed costs, contribution margin is the criteriafor deciding whether a product or product line should be dropped.

If the blue key product line is dropped, revenues will decrease by:

3,000 ! $38 " ($114,000)

Variable costs will decrease:

3,000 ! $20 " ($60,000)

Contribution margin will decrease as well:

3,000 ! $18 " ($54,000)

A cost savings of $60,000 is realized, which is really a “negative” cost. Therefore,the “income” generated from dropping this line is $60,000.

The actual revenue generated from having this line is $114,000.Because $114,000 is greater than $60,000 (a $54,000 difference), the com-

pany is better off by NOT dropping this product.A decision rule might be: If the actual revenue of the product exceeds the cost

savings (in reduced costs, either variable or fixed or both), then the productshould not be dropped.

SalesVariable CostsCont

$rribution Margin

Discretionary Fixed CostsS

$eegment Margin

packaging to Pack-It or packages their own key blanks. The total cost for the35,000 cases is $3,000 higher if Doctor decides to outsource the packaging toPack-It.

So it would make sense for Doctor to continue packaging its own product.

Requirement 3

Should Doctor stop producing the blue key?

1eSG_C21_0131792075.qxd 10/24/06 3:12 PM Page 719

Page 752: polpoa_sg

720 Chapter 21 | Demo Doc 2

Capital BudgetingLearning Objectives 3–4

Yummy Eats produces packaged food products. The company desires to produceone of two possible new products: either yogurt or cottage cheese. Each productwould require an additional investment of $400,000. Yogurt would have a usefullife of six years, with no residual value. Cottage cheese would have a useful life offour years, with no residual value. The expected annual net cash inflows are asfollows:

Demo Doc 2

Net Cash Inflows

Useful Life Yogurt Cottage Cheese

Years Annual Accumulated Annual Accumulated

1 $110,000 $110,000 $140,000 $140,000

2 110,000 220,000 140,000 280,000

3 110,000 330,000 140,000 420,000

4 110,000 440,000 140,000 560,000

5 110,000 550,000

6 110,000 660,000

Requirements

1. Determine the payback period for each product. What is the majorweakness of payback analysis?

2. Calculate the accounting rate of return. What is the major weakness of theaccounting rate of return?

3. Assuming that Yummy requires a 14% return on each possible new product,what is the net present value of each project?

4. Compute the internal rate of return for each product.

1eSG_C21_0131792075.qxd 10/24/06 3:12 PM Page 720

Page 753: polpoa_sg

Demo Doc 2 Solutions | Chapter 21 721

The rate of return calculated is compared to the required return set by manage-ment. The company will invest in the asset if the return is more than that requiredand will not invest if the return is less than the required return. When evaluatingseveral alternatives, the higher the return, the better the alternative is.

The accounting rate of return is determined by dividing the average operat-ing income from the asset by the average amount invested in the asset.

The average amount invested in the asset is calculated by dividing the costof the asset less the residual value by two.

3 Use payback andaccounting rate ofreturn models to makelonger-term capitalbudgeting decisions

Part 1 Part 2 Part 3 Part 4 Demo DocComplete

Demo Doc 2 Solutions

Part 1 Part 2 Part 3 Part 4 Demo DocComplete

3 Use payback andaccounting rate ofreturn models to makelonger-term capitalbudgeting decisions

Requirement 1

Determine the payback period for each product. What is the majorweakness of payback analysis?

Payback period is a simple approach that is used sometimes to screen potentialprojects. A company would want to recoup the investment amount as quickly aspossible, assuming all other factors are constant.

In this case, payback is the length of time it takes Yummy to recover, in netcash inflows, the $400,000 initial outlay. Yogurt and cottage cheese each haveequal cash inflows each year, so the payback period is calculated as:

The payback method favors the cottage cheese because it recovers the initialinvestment more quickly. The major weakness of payback analysis is that it onlyfocuses on time, not on profit. The payback period must be shorter than the use-ful life to provide any profit.

Requirement 2

Calculate the accounting rate of return.What is the major weakness of theaccounting rate of return?

Payback PeriodAmount Invested

Expected Annu"

aal Net Cash InflowPayback Period for

Yogurt""

"

$400,000$110,000

Years

Payback Perio

3.636

dd forCottage Cheese

$400,000$140,000

"

" 2.857 Years

1eSG_C21_0131792075.qxd 10/24/06 3:12 PM Page 721

Page 754: polpoa_sg

722 Chapter 21 | Demo Doc 2 Solutions

Net present value brings future cash inflows and future cash outflows to a com-mon time period, which makes the feasibility of each product easier to compare.

The net present value (NPV) method compares the present value of the netcash inflows to the initial investment of the capital project. To obtain the presentvalue of the net cash inflows, the minimum desired rate of return is used to findthe appropriate discounting factor in the present value tables. If the cash flows

4 Use discounted cashflow models to makelonger-term capitalbudgeting decisions

Part 1 Part 2 Part 3 Part 4 Demo DocComplete

Average amount invested in yogurt and cottage cheese is:

To determine the average operating income, depreciation must first be calcu-lated. The depreciation associated with the yogurt is:

Depreciation associated with the cottage cheese is:

The depreciation expense is now subtracted from the expected annual net cashinflows to calculate the average operating income:

The accounting rate of return can now be calculated:

The major weakness of the accounting rate of return is that it doesn’t considerthe timing of the income. The accounting rate of return doesn’t consider whetherthe income is greater early in the life of the asset or near the end of the useful lifeof the asset. Management would prefer greater income early in the life of theasset so the funds could be used to generate additional income.

Requirement 3

Assuming that Yummy requires a 14% return on each possible newproduct, what is the net present value of each project?

Yogurt$43,333$200,000

21.667%

Cottage Cheese

"

$$40,000$200,000

20%"

Yogurt Average OperatingIncomme $110,000 $66,667$43,333

Cottage Cheese

" $

"

AAverage OperatingIncome $140,000 $100,000" $

"" $40,000

$400,0004-Year Life

$100,000"

$400,0006-Year Life

$66,667"

$400,000 $02

$200,000$

"

1eSG_C21_0131792075.qxd 10/24/06 3:12 PM Page 722

Page 755: polpoa_sg

Demo Doc 2 Solutions | Chapter 21 723

are equal each year, it is considered to be an annuity and Exhibit 21-15 (p. 1134of the textbook) is used; if the cash flows are unequal, Exhibit 21-17 (p. 1136 ofthe textbook) is used for each year’s present value calculation. If the presentvalue of the inflows to be received is equal to or greater than thatinvested, the project is considered to be desirable.Consider the following:

Both of these projects expect a stream of equal periodic cash flows, which is calledan annuity. The present value of an annuity is the periodic cash flow multiplied bythe present value of an annuity of $1. Exhibit 26-15 (reproduced here) shows thepresent value of annuity factors for various interest rates and numbers of periods:

Net Present Value Present Valueof AL

" LL CashFlows IN

Present Value ofALL C

$aashFlows OUT

PVCFI -PVCFO"

Present Value of Annuity of $1

Period 4% 6% 8% 10% 12% 14% 16%

1 0.962 0.943 0.926 0.909 0.893 0.877 0.862

2 1.886 1.833 1.783 1.736 1.690 1.647 1.605

3 2.775 2.673 2.577 2.487 2.402 2.322 2.246

4 3.630 3.465 3.312 3.170 3.037 2.914 2.798

5 4.452 4.212 3.993 3.791 3.605 3.433 3.274

6 5.242 4.917 4.623 4.355 4.111 3.889 3.685

7 6.002 5.582 5.206 4.868 4.564 4.288 4.039

8 6.733 6.210 5.747 5.335 4.968 4.639 4.344

9 7.435 6.802 6.247 5.759 5.328 4.946 4.607

10 8.111 7.360 6.710 6.145 5.650 5.216 4.833

Appendix C-2 provides a more comprehensive table for the present value of an annuity of $1.

As shown in the table, the present value factor for the yogurt investment, 14%and 6 periods, is 3.889, and the present value factor for the cottage cheese invest-ment, 14% and 4 periods, is 2.914.

The present value of a project is equal to the present value factor multipliedby one period’s cash flow.

The net present value is the present value less the cost of the investment,where the investment is made at time zero (today), giving an interest factor of1.0000, for a “present value” approach.

So to calculate net present value for each product:

Yogurt:PVCFI $110,000 3.889 $427,790PVCFO $

! "

4400,000 1.000 400,000Net Present Value $

! "

" 227,790

Cottage Cheese:PVCFI $140,000 2.914! " $$407,960PVCFO $400,000 1.000 400,000

Net! "

PPresent Value $ 7,960"

1eSG_C21_0131792075.qxd 10/24/06 3:12 PM Page 723

Page 756: polpoa_sg

724 Chapter 21 | Demo Doc 2 Solutions

The internal rate of return (IRR) method calculates the estimated rate of returnYummy can expect to earn by investing in the product. The IRR uses the sameconcepts as the net present value. The net present value uses management’s min-imum desired rate of return to determine whether the present value of the futurenet cash inflows equals or exceeds the cost of the initial investment. If they areequal, the product is earning exactly the minimum desired rate of return. If theamount is positive, the product is earning a rate of return in excess of the mini-mum, but it is unknown. The IRR method calculates the projected rate of returnthis way:

a. Identify the projected future cash flows (as is done with the net presentvalue method).

b. Estimate the present value of an annuity factor as:

c. Referring to the present value of an annuity table, find the number of cashflow periods in the left column and follow that row to the right until youlocate the factor that is closest to the present value of an annuity factor com-puted in part b. Follow that number up to the interest rate at the top of thecolumn to determine the rate of return. If the rate of return calculated isequal to or higher than management’s minimum desired rate of return, theproduct is acceptable.

To determine the internal rate of return, we must find the discount rate thatmakes the total present value of the cash inflows equal to the present value of thecash outflows. Work backward to find the discount rate that makes the presentvalue of the annuity of cash inflows equal to the amount of the investment bysolving the following equation for the annuity present value factor (which is, inreality, the figure obtained when calculating the payback period, as we did inRequirement 1):

Annuity PV Factorfor Yogurt

$400,000$110,00

"00

Annuity PV Factorfor Cottage C

" 3.636

hheese$400,000$140,000

"

" 2.857

Annuity PV FactorInvestment

Expected Annual"

Net Cash Flow

4 Use discounted cashflow models to makelonger-term capitalbudgeting decisions

Part 1 Part 2 Part 3 Part 4 Demo DocComplete

Because the net present value of both products are positive, accept both products,unless coming up with the needed funds of $800,000 is a problem. In that case,the company should choose the product with the highest net present value.

In our case, the product with the highest net present value is yogurt:$27,790.

Requirement 4

Compute the internal rate of return for each product.

1eSG_C21_0131792075.qxd 10/24/06 3:12 PM Page 724

Page 757: polpoa_sg

Demo Doc 2 Solutions | Chapter 21 725

Scan the row in the present value of an annuity table corresponding to the prod-uct’s expected life: period 6 for yogurt, period 4 for cottage cheese. Choose the col-umn with the number closest to the annuity PV factor previously calculated. The3.636 annuity factor for yogurt is closest to the 16% column and the 2.857 annu-ity factor for cottage cheese is closest to the 14% column. Therefore, the internalrate of return for yogurt is approximately 16% and the internal rate of return forcottage cheese is approximately 14%.

Part 1 Part 2 Part 3 Part 4 Demo DocComplete

1eSG_C21_0131792075.qxd 10/24/06 3:12 PM Page 725

Page 758: polpoa_sg

726 Chapter 21 | Quick Practice Questions

Quick Practice QuestionsTrue/False

_____ 1. Relevant information is future data that differ among alternatives.

_____ 2. If the lost revenues from dropping a product exceed the cost savingsfrom dropping the product, it should be retained.

_____ 3. Investments with longer payback periods are more desirable, all elsebeing equal.

_____ 4. A sunk cost is a past cost that cannot be changed regardless of whichfuture action is taken.

_____ 5. The payback method can only be used when net cash inflows are thesame for each period.

_____ 6. To maximize profits, produce the product with the highest contributionmargin per unit of the constraint.

_____ 7. The accounting rate of return is a measure of profitability computed bydividing the average annual cash flows from an asset by the averageamount invested in the asset.

_____ 8. Fixed costs are irrelevant to a special decision when those fixed costsdiffer between alternatives.

_____ 9. Net present value and the payback period are examples of discountedcash flow models used in capital budgeting decisions.

_____10. In calculating the net present value of an investment in equipment, therequired investment and its terminal residual value should be sub-tracted from the present value of all future cash inflows.

1eSG_C21_0131792075.qxd 10/24/06 3:12 PM Page 726

Page 759: polpoa_sg

Quick Practice Questions | Chapter 21 727

Multiple Choice

1. For incremental analysis, which of the following would beirrelevant?a. The cost of an asset that the company is considering replacingb. Fixed overhead costs that differ among alternativesc. The cost of further processing a product that could be sold as isd. The expected increase in sales of one product line as a result of a deci-

sion to drop a separate unprofitable product line

2. Which of the following is an irrelevant cash inflow or outflow?a. Future disposal value of an assetb. Future operating cash flowsc. Current cash outlay to acquire a new assetd. Cash outlay to acquire equipment 10 years ago

3. Lowwater Sailmakers manufactures sails for sailboats. Thecompany has the capacity to produce 25,000 sails per year, but iscurrently producing and selling 20,000 sails per year. Thefollowing information relates to current production:

Sale price per unit $ 150Variable costs per unit:Manufacturing 55Marketing and administrative 25Total fixed costs:Manufacturing 640,000Marketing and administrative $280,000

If a special sales order is accepted for 5,000 sails at a price of$125 per unit, and fixed costs remain unchanged, what wouldthe effect on operating income be?a. Decrease by $5,000b. Increase by $190,000c. Decrease by $125,000d. Increase by $225,000

4. Using the data in Question 3, compute the effect on operatingincome if a special sales order is accepted for 2,500 sails at aprice of $70 per unit, when fixed costs increase by $10,000, andvariable marketing and administrative costs for the orderdecrease by $5 per unit?a. Increase by $10,000b. Decrease by $22,500c. Increase by $22,500d. Decrease by $82,500

1eSG_C21_0131792075.qxd 10/24/06 3:12 PM Page 727

Page 760: polpoa_sg

728 Chapter 21 | Quick Practice Questions

5. DC Electronics uses a standard part in the manufacture ofseveral of its radios. The cost of producing 30,000 parts is $90,000,which includes fixed costs of $33,000 and variable costs of $57,000.The company can buy the part from an outside supplier for $2.50per unit, and avoid 30% of the fixed costs. If DC Electronics makesthe part, what is the effect on its operating income?a. $6,500 greater than if the company bought the partb. $8,100 greater than if the company bought the partc. $15,000 less than if the company bought the partd. $5,100 less than if the company bought the part

6. The following data are available for the Forte Co.:

Toaster Ovens Bread Machines

Sale price $60 $135

Variable costs 38 62

The company can manufacture five toaster ovens per machinehour and three bread machines per machine hour. The company’sproduction capacity is 1,500 machine hours per month. Tomaximize profits, what should the company produce?a. 4,500 bread machinesb. 2,250 toaster ovens and 3,750 bread machinesc. 3,750 toaster ovens and 2,250 bread machinesd. 7,500 toaster ovens

7. Shine Bright Company has three product lines: D, E, and F. Thefollowing information is available:

D E F

Sales $60,000 $38,000 $26,000

Variable costs 36,000 18,000 12,000

Contribution margin 24,000 20,000 14,000

Fixed expenses 12,000 15,000 16,000

Operating income (loss) $12,000 $ 5,000 $ (2,000)

Shine Bright Company is thinking of dropping product line Fbecause it is reporting an operating loss. Assuming Shine BrightCompany drops line F and does not replace it, what is the effecton the operating income?a. Increase $2,000b. Increase $14,000c. Decrease $14,000d. Increase $16,000

8. Using the data in Question 7, assume that Shine Bright Companydrops line F and rents the space formerly used to produce productF for $17,000 per year. What is the effect on the operating income?a. Decrease $3,000b. Increase $3,000c. Decrease $14,000d. Increase $15,000

1eSG_C21_0131792075.qxd 10/24/06 3:12 PM Page 728

Page 761: polpoa_sg

Quick Practice Questions | Chapter 21 729

9. Logan, Inc., is evaluating two possible investments indepreciable plant assets. The company uses the straight-linemethod of depreciation. The following information is available:

Investment A Investment B

Initial capital investment $60,000 $90,000

Estimated useful life 3 years 3 years

Estimated residual value 0 0

Estimated annual net cash inflow $25,000 $40,000

Required rate of return 10% 12%

The present value of $1 due 3 years from now:8% 0.7938

10% 0.751312% 0.711814% 0.675016% 0.6407

The present value of $1 per year due at the end of each of 3 years:8% 2.5771

10% 2.486912% 2.401814% 2.321616% 2.2459

What is the internal rate of return for Investment A?a. 8%b. 10%c. 12%d. 14%

10. Using the information in Question 9, what is the net presentvalue of Investment B?a. $ (164)b. $ 6,072c. $40,000d. $61,528

1eSG_C21_0131792075.qxd 10/24/06 3:12 PM Page 729

Page 762: polpoa_sg

730 Chapter 21 | Quick Practice Questions

Quick Exercises

21-1. Label each of the following items as relevant or irrelevant inmaking a decision.

a. Cost of insurance on a new vehicle

b. Cost of roof repair made on rental property last year

c. Original cost of old equipment that is being evaluated for replacement

d. Cost of new equipment under evaluation to replace used equipment

e. Accumulated depreciation on old equipment being evaluated forreplacement

f. Cost of previous year’s insurance policy on old equipment beingevaluated for replacement

21-2. Dynamic Enterprises produces and sells a part used in theproduction of automobiles. The unit costs associated with thispart are as follows:

Jupiter Company approached Dynamic Enterprises with an offer topurchase 20,000 units of this part at a price of $0.75 per unit. Acceptingthis special sales order will put idle manufacturing capacity to use andwill not affect regular sales. Total fixed costs will not change.

Determine whether the special order should be accepted. Justify yourconclusion.

Direct Materials $0.15Direct Labor $0.30Variabble Manufacturing Overhead $0.25Fixed Manufaacturing Overhead $0.10Total Cost $0.80

1eSG_C21_0131792075.qxd 10/24/06 3:12 PM Page 730

Page 763: polpoa_sg

Management is considering dropping the kids’ shoes product line.Accountants for the company estimate that dropping the kids’ shoesline will decrease fixed costs of goods sold by $50,000 and fixed mar-keting and administrative expenses by $6,000.

Prepare an analysis supporting your opinion about whether the kids’shoes product line should be dropped.

Currently If Dropped

Quick Practice Questions | Chapter 21 731

21-3. Lightning Shoes manufactures kids’ sneakers and kids’ shoes.The company’s product line income statement follows:

Kids’ Kids’

Total Sneakers Shoes

Sales revenue $841,000 $580,000 $261,000

Cost of goods sold

Variable 220,000 116,000 104,000

Fixed 290,000 184,000 106,000

Total cost of goods sold 510,000 300,000 210,000

Gross profit 331,000 280,000 51,000

Marketing and administrative expenses

Variable 140,000 58,000 82,000

Fixed 105,000 79,000 26,000

Total marketing and administrative expenses 245,000 137,000 108,000

Operating income (loss) $ 86,000 $143,000 $ (57,000)

1eSG_C21_0131792075.qxd 10/24/06 3:12 PM Page 731

Page 764: polpoa_sg

21-5. Identify four capital budgeting models described in the chap-ter and discuss the strengths and weaknesses of each model.

Costs Make Buy part and leave Buy part and usefacilities idle facilities to make

another product

21-4. Sharp Image Company makes a part used in the manufacture ofvideo cameras. Management is considering whether to continuemanufacturing the part or to buy the part from an outsidesource at a cost of $21.30 per part. Sharp Image needs 100,000parts per year. The cost of manufacturing 100,000 parts is com-puted as follows:

Sharp Image would pay $0.30 per unit to transport the parts to itsmanufacturing plant. Purchasing the part from an outside sourcewould enable the company to avoid 35% of fixed manufacturing over-head. Sharp Image’s factory space freed up by purchasing the partfrom an outside supplier could be used to manufacture another prod-uct with a contribution margin of $59,000.

Prepare an analysis to show which alternative makes the best use ofSharp Image’s factory space assuming Sharp Image:

a. Makes the partb. Buys the part and leaves facilities idlec. Buys the part and uses facilities to make another product

Direct Materials $ 765,000Direct Labor 612,000VVariable Manufacturing Overhead 510,000Fixedd Manufacturing Overhead 663,000Total MManufacturing Costs $2,550,000

1eSG_C21_0131792075.qxd 10/24/06 3:12 PM Page 732

Page 765: polpoa_sg

Do It Yourself! Question 1 | Chapter 21 733

Do It Yourself! Question 1

Special Business Decisions

Part 1

Easy Access produces keyless entry remotes for automobiles. A Yemen companyhas offered to purchase 500 cases of keyless entry remotes at a per-unit price of$80. Assume selling to the Yemen company will not affect regular customers,will not change fixed costs, will not require any additional variable nonmanu-facturing expenses, and will use manufacturing capacity that would otherwisebe idle.

Sales revenue (5,000 cases @ $140.00 per case)Less: Manufacturing cost of goods soldGross profitLess: Marketing and administrative expensesOperating income

$700,000400,000

$300,000220,000

$ 80,000

EASY ACCESSIncome Statement (without considering the special sale)

Requirement

1. Assuming $250,000 of the manufacturing cost is fixed and $100,000 of themarketing and administrative cost is fixed, should Easy Access accept theorder from the Yemen company?

Part 2

Great Graphics offered to print the graphics on the keyless remotes. Currently,Easy Access does the printing themselves at the following per-case costs:

Direct materialsDirect laborVariable overheadFixed overheadTotal manufacturing costCost per case ($80,000/5,000)

$20,00040,0005,000

15,000$80,000$ 16

1eSG_C21_0131792075.qxd 10/24/06 3:12 PM Page 733

Page 766: polpoa_sg

734 Chapter 21 | Do It Yourself! Question 1

Graphics Cost Make Buy Difference

Requirement

2. Great Graphics’ price per case is $15. Easy Access would avoid all variablecosts associated with the printing and would reduce the fixed cost by$3,000 if Great Graphics prints Easy Access’s graphics. Should Easy Accessoutsource the printing work to Great Graphics?

Part 3

Easy Access’s sales of 5,000 cases include 200 cases of a special shaped keylessremote made for one customer. The special keyless remote sells for the sameprice, $140, as all the other keyless remotes, but seems to require more expenses.Easy Access is considering dropping the special shaped keyless remote. An analy-sis determined that the variable cost of producing the special keyless remote is$135 per case. If the special keyless remote was not produced, fixed costs wouldnot change.

Requirement

3. Should Easy Access stop producing the special shaped keyless remote?

1eSG_C21_0131792075.qxd 10/24/06 3:12 PM Page 734

Page 767: polpoa_sg

Do It Yourself! Question 2 | Chapter 21 735

Do It Yourself! Question 2

Capital BudgetingOffice Home produces various office products. It wants to produce one of twopossible new products: either pens or pencils. Each product would require anadditional investment of $340,000. Useful life for the pencils would be sevenyears, while useful life for the pens would be four years. The expected annualnet cash inflows are as follows:

Useful Life

Pencils Pens

Years Annual Accumulated Annual Accumulated

1 $60,000 $ 60,000 $90,000 $90,000

2 60,000 120,000 90,000 180,000

3 60,000 180,000 90,000 270,000

4 60,000 240,000 90,000 360,000

5 60,000 300,000

6 60,000 360,000

7 60,000 420,000

Requirements

1. Determine the payback period for each product.

2. Calculate the accounting rate of return.

3. Assuming that Office Home requires a 12% return on each possible newproduct, what is the net present value of each project?

4. Compute the internal rate of return for each project.

1eSG_C21_0131792075.qxd 10/24/06 3:12 PM Page 735

Page 768: polpoa_sg

736 Chapter 21 | Quick Practice Solutions

Quick Practice SolutionsTrue/False

T 1. Relevant information is future data that differ among alternatives.(p. 1118)

T 2. If the lost revenues from dropping a product exceed the cost savingsfrom dropping the product, it should be retained. (p. 1123)

F 3. Investments with longer payback periods are more desirable, all else being equal.False–Investments with shorter payback periods are more desirable, all else being equal. (p. 1129)

T 4. A sunk cost is a past cost that cannot be changed regardless of which future action is taken. (p. 1127)

F 5. The payback method can only be used when net cash inflows are the same for each period.False–The payback method can be used when net cash inflows are equal or unequal for each period. (p. 1131)

T 6. To maximize profits, produce the product with the highest contribution margin per unit of the constraint. (p. 1124)

F 7. The accounting rate of return is a measure of profitability computed by dividing the average annual cash flows from an asset by the average amount invested in the asset.False–The accounting rate of return is a method of profitability computed by dividing the average annual operating income of the asset by the average amount invested in the asset. (p. 1121)

F 8. Fixed costs are irrelevant to a special decision when those fixed costs differ between alternatives.False–Fixed costs are relevant to a special decision when those fixedcosts differ between alternatives. (p. 1120)

F 9. Net present value and the payback period are examples of discounted cash flow models used in capital budgeting decisions.False–Net present value and the internal rate of return are examples of discounted cash flow models used in capital budgeting decisions. (p. 1133)

F 10. In calculating the net present value of an investment in equipment,the required investment and its terminal residual value should be subtracted from the present value of all future cash inflows.False–In calculating the net present value of an investment in equipment, only the required investment should be subtracted from the present value of all future cash flows. (p. 1134)

1eSG_C21_0131792075.qxd 10/24/06 3:12 PM Page 736

Page 769: polpoa_sg

Quick Practice Solutions | Chapter 21 737

Multiple Choice

1. For incremental analysis, which of the following would beirrelevant? (p. 1121)a. The cost of an asset that the company is considering replacingb. Fixed overhead costs that differ among alternativesc. The cost of further processing a product that could be sold as isd. The expected increase in sales of one product line as a result of a deci-

sion to drop a separate unprofitable product line

2. Which of the following is an irrelevant cash inflow or outflow? (p. 1129)a. Future disposal value of an assetb. Future operating cash flowsc. Current cash outlay to acquire a new assetd. Cash outlay to acquire equipment 10 years ago

3. Lowwater Sailmakers manufactures sails for sailboats. Thecompany has the capacity to produce 25,000 sails per year but iscurrently producing and selling 20,000 sails per year. Thefollowing information relates to current production:

Sale price per unit $ 150Variable costs per unit:Manufacturing 55Marketing and administrative 25Total fixed costs:Manufacturing 640,000Marketing and administrative $280,000

If a special sales order is accepted for 5,000 sails at a price of$125 per unit, and fixed costs remain unchanged, what wouldthe effect on operating income be? (pp. 1120–1121)a. Decrease by $5,000b. Increase by $190,000c. Decrease by $125,000d. Increase by $225,000

4. Using the data in Question 3, compute the effect on operatingincome if a special sales order is accepted for 2,500 sails at aprice of $70 per unit, when fixed costs increase by $10,000, andvariable marketing and administrative costs for the orderdecrease by $5 per unit? (pp. 1120–1121)a. Increase by $10,000b. Decrease by $22,500c. Increase by $22,500d. Decrease by $82,500

1eSG_C21_0131792075.qxd 10/24/06 3:12 PM Page 737

Page 770: polpoa_sg

738 Chapter 21 | Quick Practice Solutions

5. DC Electronics uses a standard part in the manufacture ofseveral of its radios. The cost of producing 30,000 parts is$90,000, which includes fixed costs of $33,000 and variable costsof $57,000. The company can buy the part from an outsidesupplier for $2.50 per unit, and avoid 30% of the fixed costs. IfDC Electronics makes the part, what is the effect on itsoperating income? (pp. 1125–1127)a. $6,500 greater than if the company bought the partb. $8,100 greater than if the company bought the partc. $15,000 less than if the company bought the partd. $5,100 less than if the company bought the part

6. The following data are available for the Forte Co.:

Toaster Ovens Bread Machines

Sale price $60 $135

Variable costs 38 62

The company can manufacture five toaster ovens per machinehour and three bread machines per machine hour. The company’sproduction capacity is 1,500 machine hours per month. Tomaximize profits, what should the company produce? (p. 1124)a. 4,500 bread machinesb. 2,250 toaster ovens and 3,750 bread machinesc. 3,750 toaster ovens and 2,250 bread machinesd. 7,500 toaster ovens

7. Shine Bright Company has three product lines: D, E, and F. Thefollowing information is available:

D E F

Sales $60,000 $38,000 $26,000

Variable costs 36,000 18,000 12,000

Contribution margin 24,000 20,000 14,000

Fixed expenses 12,000 15,000 16,000

Operating income (loss) $12,000 $ 5,000 $ (2,000)

Shine Bright Company is thinking of dropping product line Fbecause it is reporting an operating loss. Assuming Shine BrightCompany drops line F and does not replace it, what is the effecton the operating income? (pp. 1122–1123)a. Increase $2,000b. Increase $14,000c. Decrease $14,000d. Increase $16,000

1eSG_C21_0131792075.qxd 10/24/06 3:12 PM Page 738

Page 771: polpoa_sg

9. Logan, Inc., is evaluating two possible investments in depreciableplant assets. The company uses the straight-line method ofdepreciation. The following information is available:

The present value of $1 due 3 years from now:8% 0.7938

10% 0.751312% 0.711814% 0.675016% 0.6407

The present value of $1 per year due at the end of each of 3 years:8% 2.5771

10% 2.486912% 2.401814% 2.321616% 2.2459

What is the internal rate of return for Investment A? (p. 1137)a. 8%b. 10%c. 12%d. 14%

10. Using the information in Question 9, what is the net presentvalue of Investment B? (pp. 1133–1134)a. $(164)b. $6,072c. $40,000d. $61,528

Quick Practice Solutions | Chapter 21 739

8. Using the data in Question 7, assume that Shine BrightCompany drops line F and rents the space formerly used toproduce product F for $17,000 per year. What is the effect on theoperating income? (pp. 1122–1123)a. Decrease $3,000b. Increase $3,000c. Decrease $14,000d. Increase $15,000

Investment A Investment B

Initial capital investment $60,000 $90,000

Estimated useful life 3 years 3 years

Estimated residual value 0 0

Estimated annual net cash inflow $25,000 $40,000

Required rate of return 10% 12%

1eSG_C21_0131792075.qxd 10/24/06 3:12 PM Page 739

Page 772: polpoa_sg

740 Chapter 21 | Quick Practice Solutions

Quick Exercise Solutions

21-1. Label each of the following items as relevant or irrelevant inmaking a decision. (p. 1118)

a. Cost of insurance on a new vehicle relevantb. Cost of roof repair made on rental property last year irrelevantc. Original cost of old equipment that is being evaluated for replace-

ment irrelevantd. Cost of new equipment under evaluation to replace used equipment

relevante. Accumulated depreciation on old equipment being evaluated for

replacement irrelevantf. Cost of previous year’s insurance policy on old equipment being

evaluated for replacement irrelevant

21-2. Dynamic Enterprises produces and sells a part used in theproduction of automobiles. The unit costs associated with thispart are as follows:

Jupiter Company approached Dynamic Enterprises with an offer topurchase 20,000 units of this part at a price of $0.75 per unit. Acceptingthis special sales order will put idle manufacturing capacity to use andwill not affect regular sales. Total fixed costs will not change.

Determine whether the special order should be accepted. Justify yourconclusion. (pp. 1120–1121)

Variable manufacturing expenses per unit:

$0.15 # $0.30 # $0.25 " $0.70

$0.75 – $0.70 " $0.05/unit

$0.05/unit ! 20,000 units " $1,000 increase in operating income

Dynamic Enterprises should accept the offer as because it wouldincrease operating income by $1,000.

Points to consider:

Total fixed costs will not change

Idle capacity exists

No effect on regular sales

Direct Materials $0.15Direct Labor $0.30Variabble Manufacturing Overhead $0.25Fixed Manufaacturing Overhead $0.10Total Cost $0.80

1eSG_C21_0131792075.qxd 10/24/06 3:12 PM Page 740

Page 773: polpoa_sg

Quick Practice Solutions | Chapter 21 741

21-3. Lightning Shoes manufactures kids’ sneakers and kids’ shoes.The company’s product line income statement follows:

Kids’ Kids’

Total Sneakers Shoes

Sales revenue $841,000 $580,000 $261,000

Cost of goods sold

Variable 220,000 116,000 104,000

Fixed 290,000 184,000 106,000

Total cost of goods sold 510,000 300,000 210,000

Gross profit 331,000 280,000 51,000

Marketing and administrative expenses

Variable 140,000 58,000 82,000

Fixed 105,000 79,000 26,000

Total marketing and administrative expenses 245,000 137,000 108,000

Operating income (loss) $ 86,000 $143,000 $ (57,000)

Management is considering dropping the kids’ shoes product line.Accountants for the company estimate that dropping the kids’ shoesline will decrease fixed costs of goods sold by $50,000 and fixed mar-keting and administrative expenses by $6,000.

Prepare an analysis supporting your opinion about whetherthe kids’ shoes product line should be dropped. (p. 1124)

Contribution margin income statement for kids’ shoes:

Currently If Dropped

Sales revenue $261,000 $ 0

Variable expenses:

Manufacturing 104,000 0

Marketing and administrative 82,000 0

Contribution margin 75,000 0

Fixed expenses:

Manufacturing 106,000 56,000

Marketing and administrative 26,000 20,000

Operating income (loss) $ (57,000) $(76,000)

The company should keep producing and selling kids’ shoes becauseoperating income would decrease by $19,000 if the product line isdropped.

1eSG_C21_0131792075.qxd 10/24/06 3:12 PM Page 741

Page 774: polpoa_sg

Sharp Image should buy the part and use the facilities to makeanother product.

742 Chapter 21 | Quick Practice Solutions

Buy part and Buy part use facilitiesand leave to make

Make facilities idle another product

Costs

Direct materials $ 765,000

Direct labor 612,000

Variable manufacturing overhead 510,000

Variable transportation $ 30,000 $ 30,000

Fixed manufacturing overhead 663,000 430,950 430,950

Purchase price 2,130,000 2,130,000

Profit contribution from another

Product 0 0 (59,000)

Total cost $2,550,000 $2,590,950 $2,531,950

21-4. Sharp Image Company makes a part used in the manufacture ofvideo cameras. Management is considering whether to continuemanufacturing the part, or to buy the part from an outsidesource at a cost of $21.30 per part. Sharp Image needs 100,000parts per year. The cost of manufacturing 100,000 parts is com-puted as follows:

Sharp Image would pay $0.30 per unit to transport the parts to itsmanufacturing plant. Purchasing the part from an outside sourcewould enable the company to avoid 35% of fixed manufacturing over-head. Sharp Image’s factory space freed up by purchasing the partfrom an outside supplier could be used to manufacture another prod-uct with a contribution margin of $59,000.

Prepare an analysis to show which alternative makes the bestuse of Sharp Image’s factory space assuming Sharp Image:(pp. 1125–1127)

a. Makes the partb. Buys the part and leaves facilities idlec. Buys the part and uses facilities to make another product

Direct Materials $ 765,000Direct Labor 612,000VVariable Manufacturing Overhead 510,000Fixedd Manufacturing Overhead 663,000Total Manufacturing Costs $2,550,000

1eSG_C21_0131792075.qxd 10/24/06 3:12 PM Page 742

Page 775: polpoa_sg

Quick Practice Solutions | Chapter 21 743

21-5. Identify four capital budgeting models described in the chap-ter and discuss the strengths and weaknesses of each model.(pp. 1129–1138)

The four capital budgeting models discussed in the chapter were thepayback model, the accounting rate of return model, the net presentvalue model, and the internal rate of return model.

The strengths and weaknesses of the different models are as follows:

The payback model is easy to understand, highlights risks, and isbased on cash flows, which are of primary concern to many busi-nesses. However, it ignores profitability and the time value of money.

The accounting rate of return model measures profitability, but itignores the time value of money.

The net present value model and the internal rate of return modelare both based on cash flows, profitability, and the time value ofmoney. These models don’t have any of the weaknesses identifiedwith the payback method and the accounting rate of return.

1eSG_C21_0131792075.qxd 10/24/06 3:12 PM Page 743

Page 776: polpoa_sg

744 Chapter 21 | Do It Yourself! Question 1 Solutions

Graphics Costs Make Buy Difference

Direct materials $20,000 $20,000

Direct labor 40,000 40,000

Variable factory overhead 5,000 5,000

Fixed costs $15,000 $12,000 3,000

Purchase cost from Great Graphics(5,000 cases ! $15 per case) 75,000 (75,000)

Total cost of labels $80,000 $87,000 $ (7,000)

Cost per unit (divide by 5,000 cases) $ 16.00 $ 17.40 $1.40

2 Make five types ofshort-term specialbusiness decisions

2 Make five types ofshort-term specialbusiness decisions

Do It Yourself! Question 1 Solutions

Part 1

Requirement 1

Assuming $250,000 of the manufacturing cost is fixed and $100,000 of themarketing and administrative cost is fixed, should Easy Access accept theorder from the Yemen company?

Increase in revenues:

500 cases ! $80 " $40,000

Increase in expenses variable costs:

500 cases ! $54 " $27,000

Increase in operating income:

$40,000 – $27,000 " $13,000

Therefore, Easy Access should accept the special order because it increases oper-ating income.

Part 2

Requirement 2

Great Graphics’ price per case is $15. Easy Access would avoid all variablecosts associated with the printing and would reduce the fixed cost by$3,000 if Great Graphics prints Easy Access’s graphics. Should Easy Accessoutsource the printing work to Great Graphics?

Variable Cost per Unit($400,000 $250,000)

"$ #(($220,000 $100,000)

5,000 Cases

$150,000 $1

$

"# 220,000

5,000 Cases

$270,0005,000 Cases

$54

"

" pper Case

1eSG_C21_0131792075.qxd 10/24/06 3:12 PM Page 744

Page 777: polpoa_sg

Do It Yourself! Question 1 Solutions | Chapter 21 745

2 Make five types ofshort-term specialbusiness decisions

Therefore, Easy Access should not accept the printing offer from Great Graphics.

Part 3

Requirement 3

Should Easy Access stop producing the special shaped keyless remote?

Reduction in revenue by dropping special keyless remote:

200 ! $140 " $28,000

Reduction in expenses by dropping special keyless remote:

Because the difference between the absolute value of the loss of revenue less thecost savings from dropping the product is positive, Easy Access would not dropthe special keyless remote.

200 $135Decrease in Operating Incom

! " $27,000ee " $ 1,000

1eSG_C21_0131792075.qxd 10/24/06 3:12 PM Page 745

Page 778: polpoa_sg

746 Chapter 21 | Do It Yourself! Question 2 Solutions

3 Use payback andaccounting rate ofreturn models to makelonger-term capitalbudgeting decisions

3 Use payback andaccounting rate ofreturn models to makelonger-term capitalbudgeting decisions

Do It Yourself! Question 2 Solutions

Requirement 1

Determine the payback period for each product.

Requirement 2

Calculate the accounting rate of return.

Average Amount Invested$340,000

2$170,000

D

" "

eepreciation per Year,Pencils

$340,0007

$48" " ,,571

Depreciation per Year, Pens$340,000

4" " $$85,000

Average Annual OperatingIncome From Pencils $60,000 $48,571 $11,429

Average An

" $ "

nnual OperatingIncome From Pens

$90,000 $85" $ ,,000 $5,000

Accounting Rate of Return(Penci

"

lls)$11,429$170,000

6.722%

Accounting Rate

" "

oof Return(Pens)

$5,000$170,000

2.941%" "

Pencils Payback$340,000$60,000

P

" " 5.67 years

eens Payback$340,000$90,000

" " 3.78 years

1eSG_C21_0131792075.qxd 10/24/06 3:12 PM Page 746

Page 779: polpoa_sg

Do It Yourself! Question 2 Solutions | Chapter 21 747

4 Use discounted cashflow models to makelonger-term capitalbudgeting decisions

4 Use discounted cashflow models to makelonger-term capitalbudgeting decisions

Requirement 3

Assuming that Office Home requires a 12% return on each possible newproduct, what is the net present value of each product?

Pencils

Pens

Both products’ net present value is negative, therefore neither product should beaccepted.

Requirement 4

Compute the internal rate of return for each product.

Annuity PV FactorInvestment

Expected Annual"

Net Cash FlowAnnuity PV Factor

for Pencils""

"

$340,000$60,000

5.667Pencil Internal Ratee of Return 6%

Annuity PV Factorfor Pens

$3

"

"440,000

$90,0003.778

Pen Internal Fate of Re"

tturn 4%"

PVCFI $ 90,000 3.037 $273,330PVCFO $340,000 1.

! "

! 0000 $340,000Net Present Value ($66,670)

"

"

PVCFI $ 60,000 4.564 $273,840PVCFO $340,000 1.

! "

! 0000 $340,000Net Present Value ($66,160)

"

"

1eSG_C21_0131792075.qxd 10/24/06 3:12 PM Page 747

Page 780: polpoa_sg

The Power of PracticeFor more practice using the skills learned in this chapter, visit MyAccountingLab.There you will find algorithmically generated questions that are based on theseDemo Docs and your main textbook’s Review and Accounting Practice sections.

To go to MyAccountingLab and follow these steps:

1. Direct your URL to www.myaccountinglab.com.2. Log in using your name and password.3. Click the MyAccountingLab link.4. Click Study Plan in the left navigation bar.5. From the table of contents, select Chapter 21, Special Business Decisions

and Capital Budgeting.6. Click a link to work on the tutorial exercises.

748 Chapter 21 | The Power of Practice

1eSG_C21_0131792075.qxd 10/24/06 3:12 PM Page 748

Page 781: polpoa_sg

AAccounting. The information system

that measures business activity, pro-cesses the results of activities intoreports, and communicates the resultsto decision makers, 2

Accounting cycle. The process ofaccounting for the transactions of abusiness for a period of time so thatresults of these transactions can bereported in financial statements, 107

Accounting equation. The basic tool ofaccounting that measures the resourcesof the business and the claims to thoseresources: Assets = Liabilities + Owner’sEquity, 2

Accounting principles, 71Accounting rate of return. A measure

of profitability computed by dividingthe average annual operating incomefrom an asset by the average amountinvested in the asset, 714exercise, 721–722

Accounts. The basic summary device ofaccounting; the detailed record of all thechanges in a particular asset, liability,or owner’s equity as a result of transac-tions, 1role of, 33

Accounts receivable. An asset repre-senting amounts due from customers towhom the business has sold goods or forwhom the business has performed ser-vices, 207internal control, 207See also Uncollectible accounts

Accrued expense (liability). An ex-pense that the business has incurredbut not yet paid, 72

Accrued liability (expense). An ex-pense that the business has incurredbut not yet paid, 72

Accrued revenue, 72Accumulated depreciation. A contra-

asset account that holds the cumulativesum of all depreciation recorded for anasset, 286

Acid-test (quick) ratio. Ratio thatreveals how well the entity can pay itscurrent liabilities, 208

Additional Paid-In Capital (Paid-inCapital in Excess of Par). The amountreceived above par value from issuingstock at a premium, 370

Adjusted trial balance. A list of all theaccounts of a business with their adjustedbalances, 72

Adjusting entries. Entry made at theend of the accounting period to accu-rately measure the period’s income andbring the related asset and liabilityaccounts to correct balances before thefinancial statements are prepared,71–72exercise, 73–80process, 72types of, 72

Allowance for Uncollectible Accounts.A contraasset account that holds the

estimated amount of uncollectible ac-counts receivable, 208

Allowance method. The method of ac-counting for uncollectible accounts thatestimates these amounts and uses anallowance account so that the balancesheet shows the amount of accountsreceivable expected to be collected inthe future, 208

Amortization. The allocation of the costof an intangible asset to expense overits useful life, 286

Assets. Economic resources that are ex-pected to be of benefit in the future,9, 33double-entry accounting, 34See also Current assets; Intangible

assets; Long-term assetsAverage cost method. Inventory cost-

ing method where, after each purchaseof inventory, a new weighted averagecost per unit is computed and used tovalue ending inventory and cost ofgoods sold, 246exercise, 258–261

BBad debts. See Uncollectible accountsBalance sheet. An entity’s assets, lia-

bilities, and owner’s equity as of a spe-cific date, 2cash reported on, 172current liabilities reported on, 332inventory reported on, 246long-term assets on, 286preparation of, 11vertical analysis, 490–491

Bank reconciliation. A document thatidentifies and explains the differencesbetween a depositor’s record of a cashaccount and a bank’s record of the samecash account, 173–181

Bonds payable or bonds. Groups ofloans issued to multiple lenders calledbondholders, 412on balance sheet, 413

effective-interest amortization,419–425

interest, 412issuance of, 412vs. mortgages, 425par value, 412pricing of, 412repayment, 413straight-line amortization, 414–418

Breakeven point. The sales level atwhich operating income is zero: Totalrevenues equal total expenses, 610contribution margin approach, 612contribution margin ratio approach,

612–613income-statement approach, 613–614

Budgeted income statement, 648–649Budgeted (predetermined) manufac-

turing overhead rate. Estimatedmanufacturing overhead allocation ratecomputed at the beginning of the year,calculated as the total estimated quan-tity of the manufacturing overhead allo-cation base, 552

Budgets and budgeting, 641See also Capital budgeting; Financial

budget; Flexible budgets;Master budget

Businesses. An organization that sellsproducts or services to customers, 1forms of, 2, 369types of, 1

CCapital. See Owner’s equityCapital budgeting. Budgeting for the

acquisition of capital assets-assets usedfor a long period of time, 714exercises, 720–725

Capital lease. Lease that essentiallyrepresents a purchase agreement for anasset, 411

Cash. Coin, currency, checks, petty cash,checking accounts, payroll accounts,money orders, traveler’s checks, andanything the bank will accept as adeposit, 172internal control over, 172

Cash budget. Details how the businessexpects to go from the beginning cash bal-ance to the desired ending balance, 642exercise, 651–656

Cash dividend. Distribution of a corpo-ration’s earnings to stockholders in theform of cash, 370exercises, 383

Cash equivalents. Highly liquid, highlysafe investments that so closely resemble

Glindex A Combined Glossary/Subject Index

G-1

1eSG_GLDX_0131792075.QXD 10/23/06 10:40 AM Page G-1

Page 782: polpoa_sg

G-2 Glindex

cash they may be shown with cash on thebalance sheet, 172

Chart of accounts. A list of all the ac-counts of a business and the numbersassigned to those accounts, 36

Closing entries. Journal entries thatcomplete the accounting cycle at theend of the accounting period; to closethe revenue, expense, and withdrawalsaccounts to set their balances to zero sothat accounting can begin for the nextperiod, 108exercises, 109–118

Collection period. The number of daysit takes to collect the average level ofreceivables, 208

Common-size statement. Financialstatement in which all amounts areexpressed as percentages of a commonbase amount, 486

Common stock. The class of stock thatrepresents the basic ownership of thecorporation, 369exercises, 372–375issuance, 373–374par value, 370, 373

Constraint. To restrict production orsale of a product that varies from com-pany to company, 713

Contingent liability. A potential liabilitythat is dependent on a future event, 331exercises, 336

Contributed (paid-in) capital. Capi-tal invested by stockholders, 369

Contribution margin income state-ment. Income statement that groupscosts by behavior—variable costs orfixed costs—and highlights the contri-bution margin, 610

Contribution margin ratio. Ratio ofcontribution margin to sales revenue, 610

Corporate capital. See Stockholders’ equityCorporations. A business owned by

stockholders that is an entity legallyseparate from its owners, 2

Cost centers, 642Costing systems, 577

See also Job order costing; Processcosting

Cost of goods manufactured. Themanufacturing or plant-related costs ofthe goods that finished the productionprocess this period, 514, 522–523

Cost of goods sold. The cost of theinventory that the business has sold tocustomers, 514budget, 643–645calculation of, 248–261

Cost per unit, 519–520Cost-volume-profit (CVP) analysis.

Expresses the relationships amongcosts, volume, and profit or loss, 609–610

breakeven points, 610cost behavior, 609for profit planning, 610, 611–612for sensitivity analysis, 608, 614–620

Credit. The right side of any account; anentry made to the right side of anaccount, 33–34

Current assets. Assets that areexpected to be converted to cash, sold,or consumed within one year or thebusiness’s operating cycle if the cycle islonger than a year, 108exercise, 117

Current liabilities. Debts due to bepaid with cash or with goods and ser-vices within one year or the entity’soperating cycle if the cycle is longerthan a year, 108on balance sheet, 332contingent, 331current ratio, 331estimated, 331exercises, 118, 333–337, 339of known amount, 331working capital, 331See also Payroll

Current ratio. The ratio of currentassets to current liabilities; a key mea-sure of liquidity, 331exercise, 336–337

CVP. See Cost-volume-profit (CVP)analysis

DDays’ sales in receivables. The num-

ber of days it takes to collect the aver-age level of receivables, 208

DDB. See Double-declining balance(DDB) method

Debit. The left side of any account; anentry made to the left side of anaccount, 33–34

Deferred (unearned) revenue. A lia-bility created when a business collectscash from customers in advance of pro-viding goods or services, 72

Depletion expense. Portion of a nat-ural resource’s cost used in a particularperiod. Computed in the same way asunits-of-production depreciation, 286exercise, 299–300

Depreciation. Allocation of the cost of along-term asset to expense over its use-ful life, 285exercise, 287–297methods of, 285–286

Direct labor. The compensating ofemployees who physically convert mate-rials, not the company’s products’ laborcosts, that are directly traceable to fin-ished products, 548exercises, 549–550variances, 690–691

Direct materials. Materials that be-come a physical part of a finished prod-uct and whose costs are traceable to thefinished product, 548exercises, 549variances, 689–691

Direct method. Format of the operatingactivities section of the statement ofcash flows that lists the major cate-gories of operating cash receipts andcash payments, 448, 458–461

Direct write-off method. The methodof accounting for uncollectible accountsthat writes off a customer’s account asan uncollectible when the businessknows the customer will not pay, 207exercise, 210–211

Discount. Amount of a bond’s maturityvalue over its issue price, 412, 420

Disposal, of plant assets, 286Dividends. Distributions of earnings by

a corporation to its stockholders, 369See also Cash dividend; Stock

dividendDouble-declining-balance (DDB) method.

An accelerated depreciation methodthat computes annual depreciation bymultiplying the plant asset’s decreasingbook value by a constant percent that istwo times the straight-line rate, 286exercises, 291–293, 296–297

Double-entry accounting. The rule of accounting that specifies, becausetransactions are measurable exchanges,every transaction involves at least twoaccounts and will thus be recorded withequal amounts of debits and credits,33–34

EEconomic value added (EVA®). EVA

= Net income + Interest expense - Capitalcharge; used to evaluate a company’soperating performance, 486

Effective-interest amortization,419–425

Efficiency (quantity) variance. Measurewhether the quantity of materials orlabor use to make the actual number ofoutputs is within the standard allowedfor that number of outputs. This is com-puted as the difference in quantities(actual quantity of input used minusstandard quantity of input allowed forthe actual number of outputs) multi-plied by the standard price per unit ofthe input, 680direct labor, 690direct materials, 690

Employee compensation. See PayrollEmployee earnings record. Payroll

record that contains the pay informa-tion of an employee, 332

1eSG_GLDX_0131792075.QXD 10/23/06 10:40 AM Page G-2

Page 783: polpoa_sg

Glindex G-3

Equivalent units. The amount of workdone during a period in terms of fullycomplete units of output, 578exercise, 585

Estimated warranty payable, 331, 336Ethics. Principles of right behavior, 2

exercise, 11standards, 514, 524

FFactory overhead. See Manufacturing

overheadFIFO. See First-in, first-out (FIFO)

methodFinancial accounting. Accounting that

provides information for people outsidethe business, 513vs. management accounting, 513, 516

Financial budget. The cash budget(cash inflows and outflows), the bud-geted period-end balance sheet, and thebudgeted statement of cash flows, 642exercise, 650–656

Financial statement analysis. Theprocess of using financial statementsand other data to predict the future of abusiness and make decisions about it,485–486common-size statements, 486economic value added, 486horizontal analysis, 485, 488–489ratio analysis, 486, 491–492vertical analysis, 485, 490–491

Financial statements. Historical, objec-tive reports, prepared according to GAAP,that communicate financial informationabout a business to those outside thebusiness, 1preparation from trial balance, 34, 72–73preparation with worksheet, 108for retailers, 142types of, 2

Financing activities. The activities ofa business that involve transactionswith long-term creditors and stockhold-ers, 447, 455–457

Finished goods inventory. Completedgoods that have not yet been sold, 514job order costing, 546

First-in, first-out (FIFO) method.Inventory costing method in which thefirst inventory costs incurred are thefirst costs to be assigned to cost of goodssold; FIFO leaves in ending inventorythe last, most recent costs incurred dur-ing the period, 245cost of goods sold, 246exercise, 249–253

Fixed costs. Costs that tend to remainthe same in amount, regardless of vari-ations in level of activity, 609

Flexible budgets. A summarized budgetthat managers can easily compute for

several different volume levels. Flexiblebudgets separate variable costs fromfixed costs; it is the variable costs thatput the “flex” in the flexible budget, 679preparation of, 683–684

Flexible budget variance. The differ-ence arising because the company actu-ally earned more or less revenue, orincurred more or less cost, than ex-pected for the actual level of output.This equals the difference between theactual amount and a flexible budgetamount, 679

FOB destination. Shipping term speci-fying that title to goods passes to thebuyer when the goods are received atbuyer’s destination; thus, the sellerpays the cost of shipping the goods tothis destination, 142

FOB shipping point. Shipping termspecifying that title to goods passes tothe buyer when the goods are shippedat the seller’s place of business; thus,the buyer pays the cost of shipping thegoods to its location, 142

Form W-2, Wage and Tax Statement.Internal Revenue Service form pre-pared by employers used to reportemployee earnings and withholdings fora calendar year, 332

Fraud. Deceit or trickery involving in-tentional actions that cause harm to abusiness or its stakeholders; accordingto the accounting profession, fraudresults in misstatements of the finan-cial statements, 171

GGenerally Accepted Accounting Prin-

ciples (GAAP). Accounting rules,created by the Financial AccountingStandards Board, that govern howaccountants measure, process, and com-municate financial information, 2

Goodwill. Excess of the cost of an ac-quired company over the sum of themarket values of its net assets, 286exercise, 303–304

Google, 369Gross margin. Net sales revenue minus

cost of goods sold, 261Gross margin percentage. A measure

of profitability equal to gross margindivided by net sales revenue, 142

Gross pay. Total amount of salary,wages, commission, bonus, or any otheremployee compensation before taxesand other deductions, 332

Gross profit. Net sales revenue minuscost of goods sold, 261

Gross profit method. A way of estimat-ing inventory by estimating gross profit,using estimated gross profit to estimate

cost of goods sold, and using estimatedcost of goods sold to estimate endinginventory, 246exercise, 263–264

Gross profit percentage. A measure ofprofitability equal to gross profit divi-ded by net sales revenue, 142

HHigh-low method. A method used to

separate mixed costs into variable andfixed components, using the highest andlowest total cost, 609

Horizontal analysis. Analysis of a per-centage change in an item on compara-tive financial statements, 485exercise, 488–489

IIncome-statement approach, to

breakeven points, 608Income statements. Summary of a

business’s revenues, expenses, and netincome or net loss for a specific period, 2budgeted income statement, 648–649for manufacturing company, 523–524performance reports, 679, 684–687preparation of, 9–10vertical analysis, 490

Indirect labor. Labor costs that are dif-ficult to trace to specific products,549–550

Indirect manufacturing costs. SeeManufacturing overhead

Indirect materials. Materials whosecosts cannot conveniently be directlytraced to particular finished products,549

Indirect method. Format of the operat-ing activities section of the statement ofcash flows that starts with net incomeand reconciles to net cash provided byoperating activities, 447–448statement of cash flows using, 449–457

Initial public offering (IPO), 369Intangible assets. Long-lived resources

without physical form that representrights, 286exercise, 301–304

Internal control. The organizationalplan and all related measures to safe-guard assets, report financial informa-tion properly, operate efficiently andeffectively, and comply with applicablelaws and regulations, 171for cash, 172over inventory, 245over receivables, 207

Internal rate of return (IRR). Therate of return (based on discounted cashflows) that a company can expect toearn by investing in the project. Thediscount rate that makes the net

1eSG_GLDX_0131792075.QXD 10/23/06 10:40 AM Page G-3

Page 784: polpoa_sg

G-4 Glindex

present value of the project’s cash flowsequal to zero, 714exercise, 724–725

Inventoriable product costs. All costsof a product that GAAP requires compa-nies to treat as an asset for externalfinancial reporting. These costs are notexpensed until the product is sold, 514for manufacturing companies, 514for merchandise companies, 514

Inventory. All the goods purchased forresale to customers in the normalcourse of merchandising operations, 141on balance sheet, 246budgeting, 645–647errors, 246, 263–264estimation of ending, 246, 264internal control over, 245manufacturing companies, 514transaction analysis exercise, 143–150valuation methods, 245–246, 247–262

Inventory turnover. The ratio of costof goods sold to average inventory. Mea-sures the number of times a companysells its average level of inventory dur-ing a year, 142financial statement analysis,

491–492Investing activities. The activities of a

business that involve buying or dispos-ing of long-term assets, 447exercises, 453–455

Investment centers, 642IRR. See Internal rate of return (IRR)

JJob cost record. Document that accu-

mulates the direct materials, direct la-bor, and manufacturing overhead costsassigned to each individual job, 545, 548

Job order costing. A system that accu-mulates costs for each batch or job.Law firms, music studios, health-careproviders, mail-order catalog compa-nies, building contractors, and customfurniture manufacturers are examplesof companies that use job order costingsystems, 545–546manufacturing companies, 545–546,

547–554vs. process costing, 545

LLabor, 548–550Last-in, first-out (LIFO) method.

Inventory costing method in which thelast inventory costs incurred are thefirst costs to be assigned to cost of goodssold; LIFO leaves in ending inventorythe first, oldest costs incurred duringthe period, 245–246cost of goods sold, 246exercise, 253–258

LCM. See Lower-of-cost-or-market (LCM)rule

Lease Payable. The amount owed for acapital lease, 411

Leases. A rental agreement in which thelessee obtains the use of an asset bypaying rent to the property owner, thelessor, 411

Ledgers. The accounting record summa-rizing, in accounts, the transactions of abusiness and showing the resultingaccount balances, 41

Liabilities. Outsider claims to the assetsof a business; the debts owed to out-siders, 33double-entry accounting, 34See also Current liabilities; Long-term

liabilitiesLIFO. See Last-in, first-out (LIFO)

methodLiquidity ratios, 208Long-term assets. Long-lived, tangible

assets such as land, buildings, equip-ment, and furniture used in the opera-tion of a business lasting for more thana year, 108on balance sheet, 286exercise, 117life cycle of, 285

Long-term business decisions, 714Long-term debt, 338–339Long-term liabilities. Liabilities other

than those that are current, 108on balance sheet, 413exercise, 118See also Bonds payable or bonds;

Leases; Mortgages (mortgagenotes payable)

Lower-of-cost-or-market (LCM) rule.Rule that a business must report inven-tory in the financial statements atwhichever is lower, the historical cost orthe market value, of each inventoryitem, 246exercise, 262

MManagement accountability. The

manager’s fiduciary responsibility tomanage the resources of an organiza-tion, 513, 516

Management accounting. The branchof accounting that focuses on informa-tion for internal decision makers of abusiness, 513–515ethical standards, 514, 524vs. financial accounting, 513, 516for manufacturing companies, 514for merchandise companies, 514role of, 513, 516for service companies, 513–514in today’s business environment, 513

Manufacturing companies. 1. Busi-nesses that make their own productsthat are sold to the final customer or toother companies 2. A company that useslabor, plant, and equipment to convert

raw materials into new finished prod-ucts, 1–2accounting characteristics, 514cost of goods manufactured, 514,

522–523income statement, 523–524inventory, 514job order costing, 545–546, 547–554overhead (See Manufacturing

overhead)unit product cost, 519–520

Manufacturing overhead. All manu-facturing costs other than direct mate-rials and direct labor. Also called factoryoverhead or indirect manufacturingcosts, 545–546allocation of, 551–554job cost record, 548variances, 680, 692–693

Master budget. The budget preparedfor only one level of sales volume,641–642exercise, 643–649

Matching principle. Recording ex-penses in the time period they wereincurred to produce revenues, thusmatching them against the revenuesearned during that same period, 71

Merchandise companies. 1. Busi-nesses that sell products made byanother company 2. A company thatresells products previously bought fromsuppliers, 1accounting characteristics, 514financial statements, 142gross profit percentage, 142income statement, 518–519inventory transaction analysis,

143–150inventory turnover, 142shipping and delivery, 142transactions with customers, 141transactions with suppliers, 141

Merchandise inventory. See InventoryMixed costs. Costs that have both vari-

able and fixed components, 609Mortgages (mortgage notes payable).

Long-term note or loan taken out forthe purchase of land, buildings, or bothand secured by lender’s right to takethe specified assets, called collateral, ifthe borrower is unable to repay theloan, 411vs. bonds, 425

Multistep income statement. Incomestatement format that calculates netincome or net loss by listing importantsubtotals such as gross profit and oper-ating income, 142

NNatural resources. Plant assets exist-

ing in or on land, 286depletion expenses, 286exercise, 298–300

1eSG_GLDX_0131792075.QXD 10/23/06 10:40 AM Page G-4

Page 785: polpoa_sg

Glindex G-5

Net income. See ProfitNet pay. Gross pay minus all deduc-

tions, 332Net present value (NPV). The decision

model that brings cash inflows and out-flows back to a common time period bydiscounting these expected future cashflows to their present value using aminimum desired rate of return, 714exercise, 722–724

Note receivable. A written promise offuture payment received by the busi-ness, 208exercise, 217–220

OOperating activities. The activities of

a business that determine net income ornet loss, 447exercises, 451–453, 459–461

Operating budget. Projects sales rev-enue, cost of goods sold, and operatingexpenses, leading to the budgeted in-come statement that projects operatingincome for the period, 641preparation of, 644–649

Overallocated (manufacturing) over-head. When the manufacturing over-head allocated to Work in ProgressInventory is more than the amount ofmanufacturing overhead costs actuallyincurred, 546, 554

Overhead flexible budget variance.Shows how well management has con-trolled overhead costs. It is the differ-ence between the actual overhead costand the flexible budget overhead for theactual number of outputs, 680

Owner’s equity. The insider claims tothe assets of a business by the owner;the owner’s interest in the business, 33double-entry accounting, 34

PPaid-in (contributed) capital. Capital

invested by stockholders, 369Paid-in Capital in Excess of Par

(Additional Paid-In Capital). Theamount received above par value fromissuing stock at a premium, 370

Partnership. A business with two ormore owners, 2

Par value (bonds). The amount theborrower must pay back to the lender;also called the maturity value whenamounts are borrowed by issuingbonds, or principal, 412

Par value (stocks). Arbitrary amountassigned to a share of stock and used toestablish legal capital, 370exercise, 373

Patent. A federal government grant givingthe holder the exclusive right to produceand sell an invention for 20 years, 286exercise, 301–304

Payback. The length of time it takes torecover, in net cash inflows, the dollarsof a capital outlay, 714exercise, 721

Payroll. A major expense of businesseswith employees, 332basic transactions, 332deductions, 332employee earnings record, 332exercise, 340–344gross vs. net pay, 332process, 332

Payroll register. Payroll record that con-tains information about a pay period, 332

Payroll tax form, 332Period costs. Operating costs that are

expensed in the period in which theyare incurred, 513–514

Periodic inventory system. An inven-tory system in which the business doesnot keep a continuous record of inven-tory on hand. At the end of the period, aphysical count of inventory is taken anddetermines the inventory owned as wellas the cost of the goods sold, 141

Perpetual inventory system. An in-ventory system in which the businesskeeps a continuous record of inventoryowned and the cost of the goods sold, 141exercise, 143–150

Petty cash. Fund containing a smallamount of cash that is used to pay forminor expenditures, 172exercise, 182–184

Plant assets. The tangible, long-livedassets of a business including land,buildings, furniture, fixtures, and equip-ment. Also called fixed assets and arecommonly shown on the balance sheetas property, plant, and equipment, 285costs of, 285depreciation of, 285–286disposal of, 286

Posting. Copying amounts from thejournal to accounts in the ledger, 34

Predetermined (budgeted) manufac-turing overhead rate. Estimatedmanufacturing overhead allocation ratecomputed at the beginning of the year,calculated as the total estimated quan-tity of the manufacturing overhead allo-cation base, 552

Preferred stock. Stock that gives itsowners certain advantages over com-mon stockholders, 369–370dividends, 377–379exercises, 376–379issuance, 377

Premium (bonds). Amount of a bond’sissue price over its maturity value,412, 415

Premium (stock). Amount receivedabove par value when issuing stock, 370

Prepaid expenses. Amounts that areassets of a business because they repre-

sent items to be used later but arealready paid for, 72

Present value. Amount a person wouldinvest or pay now, 712

Price (rate) variance. Measures howwell the business keeps unit prices ofmaterial and labor inputs within stan-dards. This is computed as the differ-ence in prices (actual price per unitminus standard price per unit) of aninput multiplied by the actual quantityof the input, 680direct labor, 690direct materials, 689–690

Process costing. System for assigningcosts to large numbers of identical unitsthat usually proceed in a continuousfashion through a series of uniform pro-ductions steps or processes, 545cost flows, 578exercises, 579–583vs. job order costing, 577weighted-average method, 578, 584–588

Production volume variance. Ariseswhen actual production differs fromexpected production. It is the differencebetween (1) the manufacturing over-head cost in the flexible budget or actualoutputs and (2) the standard overheadallocated to production, 680, 693

Profit. The difference between the rev-enues, the sales price of the goods orservices sold by the business, and ex-penses, the cost of the resources used toprovide these goods and services, 9CVP for planning, 610, 611–615

Profit centers, 642Proprietorship. A business with a sin-

gle owner, 2Purchase discount. Discount received

on purchases by paying early with cashwithin a discount period, 141

Purchase returns and allowances. Areduction in the amount owed for a pur-chase from returning merchandise oraccepting damaged goods, 141

QQuantity variance. See Efficiency (quan-

tity) varianceQuick ratio. Ratio that reveals how

well the entity can pay its current lia-bilities, 208

RRate of return on common stock-

holders’ equity. A ratio of net incomeminus preferred dividends, divided byaverage common stockholders’ equitythat measures profitability, 491–492

Ratio analysis, 486, 491–492Raw materials. Inventory items used

in the production of goods, 514Receivables

internal control over, 207types of, 207

1eSG_GLDX_0131792075.QXD 10/23/06 10:40 AM Page G-5

Page 786: polpoa_sg

G-6 Glindex

Receivables (cont.)See also Accounts receivable; Note

receivableRelevant information. Expected fu-

ture data that differs among alterna-tives, 713

Relevant range. The band of volumewhere total fixed costs remain constantand the variable cost per unit remainsconstant, 609

Responsibility center. A part or sub-unit of an organization whose manageris accountable for specific activities, 642

Retail companies. See Merchandisecompanies

Retained earnings. Capital earnedthrough profitable operation of thebusiness, 369

Return on equity. A ratio of net incomeminus preferred dividends, divided byaverage common stockholders’ equitythat measures profitability, 491–492

Revenue centers, 642Revenue recognition principle. Re-

cording revenues when they are earnedby providing goods or services to cus-tomers, 71

SSales

internal control over, 207types of, 207

Sales budget. A detailed plan thatshows the estimated sales revenue for afuture period, 641preparation of, 644–645

Sales discount. Discount granted onsales for the customer’s early paymentwithin a discount period; a contraaccount to Sales Revenue, 141

Sales mix. Combination of productsthat make up total sales, 610

Sales returns and allowances. A re-duction in the amount owed by a cus-tomer from returning merchandise oraccepting damaged goods; a contraaccount to Sales Revenue, 141

Sales tax, 336Sales volume variance. The difference

arising only because the number ofunits actually sold differs from the sta-tic budget units. This equals the differ-ence between a static budget amountand a flexible budget amount, 679

Sensitivity analysis. A “what if” tech-nique that asks what results will be ifactual prices or costs change, or if anunderlying assumption changes, 610exercise, 616–622

Service companies. 1. Businesses thatprovide services to customers 2. A com-pany that sells intangible services, ra-ther than tangible products, 1

accounting characteristics, 513–514income statement, 516–517unit costs, 546

Shipping and delivery, 142Short-term business decisions, 713Single-step income statement. Income

statement format that groups all reve-nues together and lists all expensestogether, subtracting total expenses fromtotal revenues and calculating net in-come or net loss without computing anysubtotals, 142

Standard cost. A budget for a singleunit, 680exercises, 688–693income statement, 681journal entries, 681, 691variance analysis, 680, 689–691

Stated value. An arbitrary amountassigned to a share of stock that ac-countants treat as though it were parvalue, 370

Statement of budgeted cash receipts andpayments. See Cash budget

Statement of cash flows. Summary ofthe changes in a business’s cash balancefor a specific period, 2direct method of, 448, 458–461elements in, 447vs. income statement, 461indirect method of, 447–448, 449–457operating, investing, and financing

activities, 447Statement of financial position. See

Balance sheetStatement of owner’s equity. Summary

of the changes in a business’s owner’sequity during a specific period, 2preparation of, 10

Static budget. See Master budgetStatic budget variance, 679Stock. Ownership interest in a corpora-

tion, 369–370classes of, 369–370issuance of, 370See also Common stock; Preferred stock

Stock dividend. Distribution of corpo-rate earnings to stockholders in theform of additional shares of stock, 370exercises, 380, 381–382

Stockholders’ equity. Owners’ equityof a corporation, 369reporting of, 371sources of, 369

Stock split. Increase in the number ofshares authorized, issued, and out-standing and the corresponding de-crease in its par value, 370exercises, 380, 381

Straight-line amortization, 414–418Straight-line depreciation. 1. A method

of estimating depreciation: (Cost of theAsset - Salvage Value)/ Useful Life of the

Asset 2. Depreciation method that allo-cates an equal amount of depreciation toeach year of plant asset’s useful life, 268exercises, 288–289, 293–295

Supply chain. The chain of transac-tions between businesses that supplygoods and the customers who ulti-mately use them, 141

TT-account. An informal account form

used to summarize transactions, wherethe top of the T holds the account titleand the base divides the debit andcredit sides of the account, 41exercises, 36–37

Take-home pay. Gross pay minus alldeductions, 332

Time period concept. Accounting peri-ods must be of equal length so thatincome measurement is comparablefrom one period to another; ensuresthat information is reported at regularintervals, 71

Time value of money. The fact thatmoney can be invested to earn incomeover time, 712

Total variable costs. Costs that changein total in direct proportion to changesin volume, 609

Transactions. An event that affects thefinancial position of a particular entityand can be measured reliably, 2exercises, 3–11, 35–43recording and summarizing, 34

Treasury stock. Shares of stock repur-chased by a corporation, 371exercises, 384–387

Trial balance. A list of all the accountsof a business and their balances in bal-ance sheet order; its purpose is to verifythat total debits equal total credits, 34

Turnover, of inventory. See Inventoryturnover

UUncollectible accounts. Receivable

amounts due that are never collected,207–208exercise, 209–216

Underallocated (manufacturing) over-head. When the manufacturing over-head allocated to Work in ProgressInventory is less than the amount ofmanufacturing overhead costs actuallyincurred, 546

Unearned (deferred) revenue. A lia-bility created when a business collectscash from customers in advance of pro-viding goods or services, 72

Units-of-production (UOP) method.Depreciation method that allocates a

1eSG_GLDX_0131792075.QXD 10/23/06 10:40 AM Page G-6

Page 787: polpoa_sg

Glindex G-7

fixed amount of depreciation assignedto each unit of output produced by aplant asset, 286exercises, 289–291, 295–296

VVariable costs. Costs that change in

total in direct proportion to changes involume, 609

Vertical analysis. Analysis of a financialstatement that shows the relationship ofeach statement item to a specified base,

expressing every item on the statementas a percentage of that base, 485exercise, 490–491

WWeighted-average process costing

method. A process costing methodthat costs all equivalent units of workwith a weighted average of the previousperiod’s and the current period’s costper equivalent unit, 578exercise, 584–588

Working capital. The excess of currentassets over current liabilities; a keymeasure of liquidity, 331

Work in process inventory. Goodsthat are partway through the manufac-turing process but not yet complete, 514

Worksheet. A document holding thetrial balance, adjustments, and adjustedtrial balance of a business and issued toprepare financial statements, 107–108

1eSG_GLDX_0131792075.QXD 10/23/06 10:40 AM Page G-7